Full Chemistry

Download as pdf or txt
Download as pdf or txt
You are on page 1of 1477

QUALITATIVE

ANALYSIS
QUIZRR 3

QUALITATIVE ANALYSIS

Classification of Anions

Methods available for the the detection of anions are not as systematic as those used for the
detection of cations. Furthermore anions are classified essentially on the basis of process employed.
Class A : Includes anions that are identified by volatile products obtained on treatment with
acids. It is further divided into two sub groups.
(i) Gases evolved with dil. HCl/dil H2SO4
(ii) Gases or acid vapours evolved with conc. H2SO4
Class B : Includes anions that are identified by their reactions in solution.

Class A : (i) Anions which evolve gases on reaction with dil. HCl/dil. H2SO4.

1. Carbonate CO32

(i) Dilute HCl : It gives effervescence, due to the evolution of carbon dioxide

CO23  2H  CO2   H2O

The gas gives white turbidity with lime water and baryta water.

CO2 + Ca2+ + 2OHă  CaCO3  + H2O

CO2 + Ba2+ + 2OHă  BaCO3  + H2O

On prolonged passage of carbon dioxide in lime water, the turbidity slowly disappears due
to the formation of soluble hydrogen carbonate of calcium.

CaCO3  + CO2 + H2O  Ca(HCO3)2

The following tests performed with then aqueous salts solution.

(ii) Barium chloride or Calcium chloride solution : White ppt. of barium or calcium
carbonate is obtained, which is soluble in mineral acid.

CO23  Ba 2   BaCO3 

CO23   Ca 2  CaCO3 

(iii) Silver nitrate solution : White ppt. of silver carbonate is obtained.

CO23  2Ag   Ag 2CO3 

QUALITATIVE ANALYSIS
4 QUIZRR
The ppt. so obtained is soluble in nitric acid and in ammonia, the ppt. becomes yellow or
brown on addition of excess reagent and same may also happened if the mixture is boiled,
due to the formation of silver oxide.

Ag2 CO3   Ag2O  + CO2 

2. Sulphides (S2ă)

(i) Dilute HCl or dilute H2SO4


Sulphide on treatment with dilute HCl or dilute H2SO4 gives a pungent smelling gas, H2S
(smell of rotten eggs).

S2ă + 2H+  H2S 

The blackening of filter paper moistened with lead acetate solution may identify the gas.

(CH3COO)2Pb + H2S   PbS  + 2CH3COOH


Shining black

Alternatively, a filter paper moistened with cadmium acetate solution turns yellow.

H2S  + Cd2+  CdS  + 2H+


Yellow ppt

(ii) Silver nitrate solution


With AgNO3 solution, sulphides gives a black precipitate of silver sulphide insoluble in cold
but soluble in hot dilute nitric acid.

S2ă + 2Ag+  Ag2S


Black ppt

(iii) Sodium nitroprusside solution


With sodium nitroprusside solution, ionic sulphide gives a purple colour in alkaline medium.

Na2S + Na2[Fe(CN)5NO]  Na4[Fe(CN)5NOS]


Purple colour

No reaction occurs with solutions of hydrogen sulphide or free gas. In however, filter paper
moistened with a solution of the reagent is made alkaline with sodium hydroxide or ammonia
solution, a purple colouration is produced with free hydrogen sulphide also.
Class A (ii) : Gases or acid vapours evolved with conc. sulphuric acid. It includes Clă,

Bră, Iă, NO3

QUALITATIVE ANALYSIS
QUIZRR 5

3. Chloride (Clă)

Most chloride are soluble in water except Hg2Cl2, CuCl, AgCl, PbCl2, BiOCl and SbOCl. PbCl2 is
soluble in hot water.

(i) Concentrated H2SO4


With concentrated H2SO4, chlorides gives vapours of HCl.

Clă + H2SO4  HCl  + HSO4


Vapour evolved
(a) turns blue litmus paper red.
(b) gives white fumes of NH4Cl when a glass rod moistened with aqueous ammonia
solution is brought close to the mouth of test tube.

(ii) Manganese dioxide and concentrated sulphuric acid


When a solid chloride is treated with MnO2 and concentrated H2SO4, yellowish-green coloured
gas (Cl2) is evolved.

MnO2 + 2H2SO4 + 2Clă  Mn2+ + Cl2  + 2SO24  + 2H2O


Yellowish green

(iii) Silver nitrate solution


Chlorides on treatment with AgNO3 solution gives a white curdy precipitate of AgCl insoluble
in water and in dilute nitric acid but soluble in dilute ammonia solution due to the formation
of soluble complex.

Clă + Ag+  AgCl 

White curdy ppt

AgCl + 2NH3  [Ag(NH3)2Cl


Soluble

(iv) Lead acetate solution


With (CH3CO2)2Pb solution, chlorides gives a white precipitate of lead chloride.

2Clă + Pb2+  PbCl2 


White ppt

QUALITATIVE ANALYSIS
6 QUIZRR
(v) Chromyl chloride test
When a mixture containing chloride ion is heated with K2Cr2O7 and concentrated H2SO4,
deep orange-red fumes of chromyl chloride (CrO2Cl2) are formed.

K2Cr2O7 + 4NaCl + 6H2SO4  2KHSO4 + 4NaHSO4 + 2CrO2Cl2  + 3H2O


Orangeăred

fumes

When chromyl chloride vapours are passed into sodium hydroxide solution, a yellow solution
of sodium chromate is formed, which when treated with lead acetate gives yellow precipitate
of lead chromate.

CrO2Cl2  + 2NaOH  Na2CrO4 + 2HCl


Yellow solution

Na2CrO4 + (CH3COO)2Pb  2CH3COONa + PbCrO4 

Yellow ppt

4. Bromide (Bră)

(i) Conc. H2SO4 : It gives reddish brown vapours of bromine.

2KBr + H2SO4  K2SO4 + 2HBr

2HBr + H2SO4  2H2O + SO2  + Br2 


(reddish brown)

(ii) Manganese dioxide and conc. sulphuric acid : When a mix of solid bromide, MnO2 and
conc. H2SO4 is heated reddish brown vapours of bromine are evolved.

2KBr + MnO2 + 2H2SO4  Br2  + K2SO4 + MnSO4 + 2H2O

(iii) The following tests are performed with the salt solution.

(a) Silver nitrate solution : A pale yellow ppt. of silver bromide is obtained. This ppt
is sparingly soluble in dil but readily soluble in conc. ammonia solution and insoluble
in dil. HNO3.

Bră + Ag+  AgBr

QUALITATIVE ANALYSIS
QUIZRR 7

(b) Lead acetate solution : White crystalline ppt. of lead bromide which is soluble in
boiling water.

2Bră + Pb+2  PbBr2 

(c) Chlorine water : When this solution is added to a solution of bromide and chloroform
free bromine is liberated, which colours the organic layer orange-red.

2KBr + Cl2 (water)  2KCl + Br2

Br2 + Chloroform  Orange red colour

(d) Potassium dichromate & conc. H2SO4 : When a mixture of solid bromide, K2Cr2O7,
and conc. H2SO4 is heated and passing the evolved vapours into water, a yellowish
brown solution is obtained.
2KBr + K2Cr2O7 + 7H2SO4  3Br2  + Cr2(SO4)3 + 4K2SO4 + 7H2O

5. Iodide (Iă)

Iodides are generally soluble in water except AgI, Hg2I2, HgI2, CuI and PbI2.
(i) Concentrated H2SO4
With concentrated H2SO4, iodides gives violet vapours of iodine on warming.

2Iă + 2H2SO4  I2 + SO24 + 2H2O + SO2 


Violet vapours

(ii) Silver nitrate solution


With AgNO3 solution, iodides form a yellow curdy precipitate of silver iodide, which is very
slightly soluble in concentrated ammonia solution, but is completely insoluble in dilute
HNO3.

Iă + Ag+  AgI


Yellow curdy ppt

(iii) Lead acetate solution


With (CH3CO2)2Pb solution, a yellow precipitate of lead iodide (PbI2) is formed, which is
soluble in hot water forming a colourless solution and yielding goldenăyellow plates on
cooling.

2Iă + Pb2+  PbI2


Yellow ppt

QUALITATIVE ANALYSIS
8 QUIZRR
(iv) Potassium dichromate and concentrated sulphuric acid
When iodides are warmed with K2Cr2O7 and concentrated H2SO4, iodine is liberated.

6Iă + Cr2 O72 + 2H2SO4  3I2  + 2Cr3+ + 7SO24  + 7H2O

Violet vapours

(v) Chlorine water


When this reagent is added dropwise to a solution of an iodide, free iodine is liberated, which
colours the solution brown and on shaking with CS2, CHCl3 or CCl4, it dissolves in organic
layer forming a violet solution, which settles below the aqueous layer.

2KI + Cl2  I2 + 2KCl

I2 + Chloroform  Violet solution

(vi) Copper sulphate solution


With CuSO4 solution, iodide produces a brown precipitate consisting of a mixture of copper
(I) iodide and iodine. On addition of hypo (Na2S2O3) to the solution, brown precipitate
changes to white (colourless).

4Iă + 2Cu2+  Cu2I2  + I2


Brown ppt

I2 + 2S2 O32  2Iă + S4 O62

(vii) Mercury(II) chloride solution


With HgCl2 solution, iodide gives a scarlet precipitate of HgI2.

2Iă + HgCl2  HgI2 + 2Clă


Scarlet ppt

The precipitate dissolves in excess of KI, forming tetraiodomercurate(II) complex.

HgI2 + 2Iă  [HgI4]2ă


Soluble

QUALITATIVE ANALYSIS
QUIZRR 9

5.

Nitrate NO3  
(i) Conc. H2SO4 : It gives reddish-brown vapours of nitrogen dioxide

4NO3 + 2H2SO4  4NO2  + 2SO24 + 2H2O + O2 

The following tests are performed with the salt solution

(ii) Brown ring test : When a freshly prepared solution of iron (II) sulphate is added to nitrate
solution & conc. H2SO4 is poured slowly down the side of the test tube, a brown ring is
obtained.

2NO3 + 4H2SO4 + 6Fe 


2+ 3+
 6Fe + 2NO + 4SO4 + 4H2O

Fe2+ + NO  [Fe(NO)]2+

On shaking and warming the mix, the brown colour disappears, nitric oxide is evolved and
a yellow solution of Iron(III) ions remains.

Action of heat : The result varies with the metal


(1) Nitrates of sodium and potassium evolve oxygen (test with glowing splint) & leave
solid nitrites (brown fumes with dilute acid).

2NaNO3  2NaNO2 + O2

(2) Ammonium nitrate yields dinitrogen oxide & steam.

NH4NO3  N2O  + 2H2O

(3) Nitrates of the noble metals leave a residue of the metal and a mix of nitrogen dioxide
and oxygen is evolved.

2AgNO3  2Ag + 2NO2 + O2 

(4) Nitrates of other metals, such as those of lead and copper, evolve oxygen and nitrogen
dioxide and leave a residue of the oxide.

2Pb(NO3)2  2PbO + 4NO2 + O2

QUALITATIVE ANALYSIS
10 QUIZRR
(B) Group ‘B’ Radicals

7.
2
Sulphate SO4  
Sulphate of barium, strontium and lead are practically insoluble in water, those of calcium and
mercury(II) are slightly soluble and most of the remaining metallic sulphates are soluble.

(i) Barium chloride solution


On adding BaCl2 solution to a sulphate salt solution, a white precipitate of barium sulphate
(BaSO4) insoluble in warm dilute hydrochloric acid and in dilute nitric acid but moderately
soluble in boiling concentrated hydrochloric acid is formed.

SO24  Ba 2  BaSO4 


White ppt

(ii) Silver nitrate solution

With silver nitrate solution, SO24 gives a white crystalline precipitate of silver sulphate.

SO24 + 2Ag  Ag2SO4


+

White ppt

(iii) Mercury(II) nitrate solution


With Hg(NO3)2 solution, sulphates form a yellow precipitate of basic mercury(II) sulphate.

SO24  3Hg 2  2H2O  HgSO4 . 2HgO   4H 


Yellow ppt

Physical appearance of inorganic salts

Sr No. Salt Colour


1 MnO, MnO2, FeO, CuO, Co3O4, Ni2O3,
sulfides of Ag+, Cu+, Cu2+, Fe2+, CO2+, Pb2+, Hg2+ Black
2 Hydrated Cu2+ salts Blue
3 HgO, HgI2, Pb3O4 Red
3+ 6+ 2+ 2+
4 Cr , Cr , Ni , Hydrated Fe salts Green
2+
5 Hydrated Mn salts Light Pink
6 KO2, K2Cr2O7, Sb2S3, ferricyanides Orange
7 Hydrated Co2+ salts Reddish Pink
8 Chromates AgBr, AgI, PbI2, CdS Yellow
9 CdO, Fe2O3, PbO2, CuCrO4 Dark brown

QUALITATIVE ANALYSIS
QUIZRR 11

Effect of Heating

1. When a gas is evolved


(a) Coloured gas :
(i) NO2, (Brown) turns starch Iodine paper blue.
Substance : Nitrites and Nitrates of heavy metals


e.g. 2Cu(NO3)2   CuO + 4NO2  + O2

(ii) Br2(Reddish Brown) turns starch paper yellow


Substance bromide : 2CdBr2 + O2  2CdO + 2Br2 

(iii) I2 (Violet) turns starch paper blue


Iodides 2CdI2 + O2  2CdO + 2I2 

(iv) Cl2 (Greenish yellow) bleaches moist litmus paper


Chlorides CuCl2 + H2O  CuO + 2HCl
(moisture)  Cu + H2O + 1/2Cl2

(b) Colourless gas (odourless)


(i) O2ăsupports glowing
Alkali nitrates : 2KNO3  2KNO2 + O2 

(ii) CO2 ă Turns lime water milky


Carbonates/Oxalates CaCO3  CaO + CO2 

(c) Colourless gas (with odour)


(i) NH3 ă turns red litmus blue
Ammonium salts : (NH4)2SO4  NH4HSO4 + NH3 

(ii) SO2 ă Smell of burning sulphur, turns acidified K2Cr2O7 paper green
Sulphites, thiosulphates
CaSO3  CaO + SO2

(iii) HCl ă pungent smell, white fumes with ammonia


hydrates chlorides
CaCl2.6H2O  Ca(OH)2 + 4H2O + 2HCl 

QUALITATIVE ANALYSIS
12 QUIZRR
(iv) H2S ă Smell of rotten eggs, turns lead acetate paper black
sulphides.
Na2S + 2H2O  2NaOH + H2S 

2. A residue (oxide) is left and colour


(i) Yellow (on hot) & white (on cold) ă ZnO
(ii) Reddish brown (hot) yellow (cold) ă PbO
(iii) Black (hot) Red (cold) ă HgO, Pb3O4
(iv) Black (hot) Red Brown (cold) ă Fe2O3

3. Substance melts : Salts of alkali metals & salts having water of crystallisation.

4. Substance makes crackling noise : NaCl, KI, Pb(NO3)2, Ba(NO3)2

5. Substance Swells : Alums, borates & phosphates

6. Substance sublimes & colour of sublimate is :


(a) White : HgCl2, Hg2Cl2, AlCl3, As2O3, Sb2O3
(b) Yellow : As2S3, HgI2 turns red on burning with glass rod
(c) Blue Black or Violet : Iodides

BASIC RADICALS
Separation of basic radicals into groups
For systematic analysis, the basic radicals have been classified into groups. Each group has a
reagent, which shows the presence of the basic radical belonging to that particular group. It is
necessary that the radical is completely removed in the respective group by using excess of group
reagent, otherwise its presence creates difficulty in higher group.

Group Group Basic Composition and colour


reagent radical of the precipitate

1. Dilute HCl Ag + AgCl : white  Chloride


2+ 
Pb PbCl2 : white  insoluble in

Hg 22 Hg2Cl2 : white  cold dilute HCl

2. H2S in Hg 2+ HgS : black 


presence of Pb 2+
PbS : black  Sulphides

dilute HCl Bi3+ Bi2S3 : black  insoluble in

Cu2+ Cus : black  dilute HCl
2+ 
Cd CdS : yellow

QUALITATIVE ANALYSIS
QUIZRR 13

Group Group Basic Composition and colour


reagent radical of the precipitate

3. NH4OH in Fe3+ Fe(OH)3 :



presence of reddish brown  Hydroxides

NH4Cl Cr 3+
Cr(OH)3 : green  are insoluble

Al3+ Al(OH)3 : white  in NH4OH

4. H2S in Zn 2+ ZnS  Sulphides are




presence of greenish white  insoluble in

NH4OH Mn 2+ MnS : buff  NH4OH

5. (NH4)2 CO3 in Ba2+ BaCO3 : white 




presence of  Carbonates

NH4OH Ca2+ CaCO3 : white  are insoluble

6. Na2HPO4 Mg 2+ Mg(NH4)PO4 :

7. NaOH NH4 Ammonia gas is

evolved.

It should be noted that :

1. Group I radicals (Ag+, Pb2+, Hg 22 ) are precipitated as chlorides because the solubility
product of these chlorides (AgCl, PbCl2, Hg2Cl2) is less than the solubility products of
chlorides of all other metal ions, which remain in solution.

2. Group II radicals are precipitated as sulphides because of their low solubility, whereas
sulphide of other metals remain in solution because of their high solubility products. HCl
acts as a source of H+, which decreases the concentration of S2ă due to common ion effect.
Hence, the decreased concentration of S2ă is only sufficient to precipitate the Group II metals
ions as sulphides.

3. Group III A radicals are precipitated as hydroxides and the NH4Cl suppresses the ionisation
of NH4OH so that only the group III A cations are precipitated as hydroxide because of their
low solubility product.

QUALITATIVE ANALYSIS
14 QUIZRR
(i) Excess of NH4Cl should not be added, else manganese will precipitate as MnO2.H2O

(ii) (NH4)2SO4 cannot be used in placed of NH4Cl because the SO24 will give precipitate

of BaSO4.

(iii) NH4NO3 cannot be used in place of NH4Cl as NO3 ions will oxidise Mn2+ to Mn3+ and
thus, Mn(OH)3 will be precipitated in III A group.

4. Ammonium hydroxide increases the ionisation of H2S by removing H+ from H2S as


unionised water.


H2S  2H+ + S2ă , H+ + OHă  H2O

Now the excess of S2ă ions is available and hence the ionic product of group III B exceeds
their solubility product and will be precipitated. In case H2S is passed through a neutral
solution, incomplete precipitation will take place due to the formation of HCl, which decreases
the ionisation of H2S. For example,

MnCl2 + H2S  MnS + 2HCl

QUALITATIVE ANALYSIS
QUIZRR 15

Group I
Mixture + dil HCl + filter

AgCl, PbCl2 (Hg2Cl2) Test for other


(All in white colour) than Ist group

Hot water + filter

filterate (PbCl2)

KI
PbI2
(yellow ppt.)
Residue (AgCl, Hg2Cl2)
H2SO4
PbSO4
(white ppt.) excess NH4 OH
K2Cr2O7 +
PbCr2O7 filter
(yellow ppt.)
H2S
PbS()
Black ppt. Residue filterate

Black ppt Cl Soluble complex


Ag(NH3)2Cl
Hg + Hg
NH2

Aqua KI
AgI + KCl + 2NH3
Regia
(pale yellow)
SnCl2 KI
HNO3
AgCl + 2NH4NO3
White ppt. Blood Red colour (white ppt.)
(colour disappears
excess SnCl2 on adding excess KI)

grey (Hg) K2Cr2O7 H2SO4

4NH3 + 9KCl + Ag2Cr2O7 Ag2SO4() + 2NH3 + NH4Cl


(Brick red) (white)

QUALITATIVE ANALYSIS
16 QUIZRR
Some Salient Reactions :

Cl
NH4OH
(1) Hg2Cl2 Hg() + Hg
NH2

Black (shing black)

(2) Aqua Regia is a very strong oxidizing agent and it converts the black precipitate into soluble
HgCl2.

Aqua Regia is a mixture of (3HCl + HNO3  NOCl + 2H2O + Cl2)

HgCl2 + SnCl2 Hg2Cl2 (white)


(white)

SnCl2

2Hg() + SnCl4
(grey)

HgCl2 + KI  HgI2 + 2KCl


(Blood red)

HgI2 + 2KI  K2[HgI4]


(nesslerÊs Reagent)

QUALITATIVE ANALYSIS
QUIZRR 17

Group II
Mix + HCl

White ppt. Soluble Compound


(Test for Ist group)
Pass H2S gas in
Presence of dil HCl

Residue (solid) shows Presence of Group II

yellow ammonium sulphide

Soluble thio metallite (IIB) Residue (IIA)


{not in syllabus} conc. NHNO3 + filter

filterate

forms soluble Nitrates of Insoluble black ppt.


Pb2+, Cu2+, Bi3+ as
Pb (NO3)2, Bi(NO3)3 , Cu(NO3)2
Aqua Regia

dil H 2SO4 + filter SnCl2 KI

Hg2Cl2() HgI2
(white ppt.) (Blood red)
Residue (PbSO4) Bi2(SO4)3 Or
white ppt. CuSO4
excess SnCl2 excess KI

CH3COONH4 Excess
(conc.) NH4OH Hg() Colour disappears
(grey) soon
(CH3COO)2Pb + NH4SO4 filterate

K2Cr2O7 Soluble blue colour Residue


complex Bi(OH)3
(white)
PbCrO4 + 2CH3COOK
(yellow) CH3COOH +
K4[Fe(CN)6]

Chocolate or
Reddish Brown

QUALITATIVE ANALYSIS
18 QUIZRR
Some Salient Reactions

(1) 3PbS + 8HNO3  3Pb(NO3)2 + 2NO + S + H2O

(soluble)

Bi  OH 3 
dil. HCl
(2)  BiCl3

(3) BiCl3 + 3Na2SnO2 + 6NaOH  3Na2SnO3 + 2Bi() + 6NaCl + 3H2O


(Black)

(4) CuSO4 + NH4OH  [Cu(NH3)4] SO4 + H2O

(Blue colour)

(5) [Cu(NH3)4]SO4 + 4CH3COOH  CuSO4 + 4CH3COONH4

2CuSO4 + K4Fe (CN)6  Cu2 [Fe(CN)6] + 2K2SO4


Chocolate or
reddish brown

QUALITATIVE ANALYSIS
QUIZRR 19

Group III
Mix + dil. HCl

Residue shows presence Soluble Solution


of Group II

Boil off H2S gas

NH4OH + NH4Cl
+
conc. HNO3 + filtre

Residue (ppt.) filterate


(used for subsequent groups)

Cr(OH)3 Al(OH)3 Fe(HO)3


(Dirty green) (white ppt.) (Brown)

Br2 + NaOH excess


+ water NaOH K4[Fe(CN)6]
Prussian Blue

Yellow ppt. dissolves


solution forming NaAlO2 KCNS or
Red colouration
NH4CNS
CH3COOH + NH4Cl
(CH3COO)2Pb

Yellow white ppt.


precipitate of Al(OH)3

QUALITATIVE ANALYSIS
20 QUIZRR

Note :
Initially HNO3 was added to convert Fe2+ (light green) into Fe3+ (deep yellow)

Some salient reactions :

(1) Al(OH)3 + NaOH  NaAlO2 + 2H2O


NaAlO2 + NH4Cl + H2O   NaCl + Al(OH)3() + NH3
white ppt

(2) Cr(OH)3 + 3Br2 + 10NaOH  2Na2CrO4 + 6NaBr + 8H2O

Na2CrO4 + (CH3COO)2Pb  PbCrO4 () + 2CH3COONa


(yellow ppt)

aqua Regia
(3) Fe(OH)3 
 HNO3  HCl 
 FeCl + 3H O
3 2

(4) FeCl3 + KCNS  Fe(CNS)3 + NH4Cl/KCl


or
NH4CNS

(5) 4 Fe(OH)3 + 3K4[Fe(CN)6]  Fe4[Fe(CN)6] + 12KOH


(Prussian blue)

4Fe(OH)2 + 4K4[Fe(CN)6]  4KFe[Fe(CN)6] + KOH


(soluble)

* These reactions can be used to distinguish between Fe2+ and Fe3+

QUALITATIVE ANALYSIS
QUIZRR 21

Group IV

filterate from IIIrd Group

NH4OH + NH4Cl

Heat

H2S

Residue filterate for


subsequent Groups

Buff coloured White ppt.


(MnS) (ZnS)

dil HCl + NaOH + Br2 + H2O dil HCl + NaOH

Black ppt. of MnO2 Soluble compound Na2ZnO2

conc. HNO3 + Pb3O4 (red lead) H2S

Purple coloured White ppt.


solution of ZnS

QUALITATIVE ANALYSIS
22 QUIZRR
Some salient reactions

(1) ZnCl2 + NaOH  Zn(OH)2 + 2NaCl

NaOH

Na2ZnO2 + 2H2O
Sodium zincate

Na2ZnO2 + H2S  2NaOH + ZnS ()


(White ppt)

(2) MnCl2 + NaOH  Mn(OH)2 + 2NaCl

air [O]

MnO2 + H2O
(Black ppt)

2MnO2 + 3Pb3O4 + 10HNO3  2 HMnO4 + 4Pb(NO3)2 + 8H2O


(PbO2 can (Purple
also be colour)
used)

(3) ZnCl2 + K4[Fe(CN)6]  Zn2[Fe(CN)6] () + 2KCl


Bluish white
ppt

QUALITATIVE ANALYSIS
QUIZRR 23

Group V
th
Filterate from IV group

Boil off H2S gas and add solid


NH4Cl and NH4OH followed by
(NH4)2CO3 solution

White ppt. shows presence Filterate use for


of Group V subsequent groups

CH3COOH

K2CrO4 (NH4)2C2O4

Yellow ppt. of White ppt. of


BaCrO4 CaC2O4

Some Salient Reactions

(1) BaCO3 + 2CHCOOH  (CH3COO)2Ba + CO2 + H2O

CaCO3 + 2CH3COOH  (CH3COO)2Ca + CO2 + H2O

(2) Ba(CH3COO)2 + K2CrO4  BaCrO4 + 2CH3COOK

(3) Ca(CH3COO)2 + (NH4)2 C2O4  CaC2O4 + 2CH3COONH4


(White ppt)

QUALITATIVE ANALYSIS
24 QUIZRR
Group VI (Mg2+)
Filterate of group V + NH4OH + Na2HPO4  a fine crystalline ppt on scratching the side of the
test tube.

MgCl2 + NH4OH + Na2HPO4  Mg(NH4)PO4 () + 2NaCl + 2H2O


(white ppt)

Mg2P2O7 + 2NH3 + H2O


(white)

Group zero :  NH 

4

Ć Salt + NaOH 
 gas giving white fumes with HCl

NH4Cl + NaOH 
 NH3 (g) + NaCl + H2O

NH3(g) + HCl  NH4Cl


white fumes

add Nesler's
 gas  
Ć 
Salt + NaOH  Brown ppt or brown or yellow colouration
Reagent
(oxydimercuri ammonium iodide) called iodide
of MillonÊs base.

NH4Cl + NaOH  NH3

+
Hg

NH3 + 3NaOH + 2K2HgI4 O NH2 Iă + 4KI + 3NaI + 2H2O


Nessler's
Reagent Hg

QUALITATIVE ANALYSIS
QUIZRR 25

S olubility Trends

dil. HNO3 NH4OH Hypo soln CNă soln.

1. AgCl ï 3 3 3

AgBr ï 3 3 3

AgI ï ï 3 3

Ag2S 3 ï 3 3

2. All water insoluble black metal sulphide are soluble in hot and dil. HNO3 except HgS.
(Soluble in aqua regia)

3. FeS is the only black metal sulphide soluble in non- oxidising acid.

4. In general Pb salts are soluble in dil. HNO3 & Ex. NaOH Soln.

5. All water insoluble C2O24 salts are insoluble in acetic acid except BaC2O4 (white).

Dry Tests

Dry tests are of great importance as these tests give clear indications of the presence of certain
radicals. The following tests are performed in dry state :
(i) Flame test
(ii) Borax bead test
(iii) Microcosmic salt bead test

(i) Flame test


Some volatile salts impart characteristic colour to the non-luminous flame. The chlorides of
the metals are more volatile in comparison to other salts. The metal chloride volatilises and
its thermal ionisation takes place.

NaCl  Na+ + Clă

CaCl2  Ca2+ + 2Clă

The cations impart a characteristic colour to the flame as these absorb energy from the flame
and transmit the same as light of characteristic colour.

QUALITATIVE ANALYSIS
26 QUIZRR
Procedure : The platinum wire fused in a glass rod is heated in the flame till it imparts
colourless flame. This is achieved by dipping the wire in conc. HCl and heating it. The
process is repeated till it gives a colourless flame. The tip of the wire is now dipped in conc.
HCl and then into the substance. The tip of the wire is strongly heated in the non-luminous
flame and the colour of the flame is observed by the naked eye.

Colour of flame Inference


1. Golden yellow Sodium
2. Violet Potassium
3. Brick red Calcium
4. Crimson red Strontium
5. Apple green Barium
6. Green with a blue centre Copper
Note : Flame test should not be performed in the presence of As, Sb, Bi, Sn and Pb as these
radicals form alloy with platinum and hence, the wire is spoiled.

(ii) Borax bead test


On heating borax the colourless glassy bead formed consists of sodium metaborate and boric
anhydride.

Heat Heat
Na 2 B4 O7 .10H2O 
 Na 2 B4 O7 
 2NaBO2  B2 O3
 
Glassy bead

On heating with a coloured salt, the glassy bead forms a coloured metaborate in oxidising
flame.

CusO4  CuO + SO3

CuO + B2O3  Cu(BO2)2


Copper metaborate
(Blue)

The metaborates posses different characteristic colours. The shade of the colour gives a clue
regarding the presence of the radical.
However, in reducing flame the colours may be different due to different reactions. For
example, copper metaborate may be reduced to colourless cuprous metaborate or to metallic
copper, which appears red and opaque.

2Cu(BO2)2 + C  2CuBO2 + B2O3 + CO

2Cu(BO2)2 + 2C  2Cu + 2B2O3 + 2CO

QUALITATIVE ANALYSIS
QUIZRR 27

Procedure : The free end of a platinum wire is coiled into a small loop and heated in the
Bunsen flame until red hot. It is dipped in borax and again heated, when borax swells up
and then fused into a glassy bead.
The bead is moistened with water and dipped in the coloured salt. It is now heated first in
the oxidising flame and then in the reducing flame and colours are noted in both the flames
in hot and cold conditions.
Colour of the bead in
Metal Oxidising flame Reducing flame
Hot Cold Hot Cold
Copper Green Blue Colourless Brown-red
Iron Brown-yellow Pale-yellow Bottle green Bottle-green
Chromium Green Green Green Green
Cobalt Blue Blue Blue Blue
Manganese Violet Amethyst red Grey Grey
Nickel Violet Brown Grey Grey

(iii) Microcosmic salt bead test


This test is similar to borax bead test. When microcosmic salt is heated, a colourless transparent
bead of sodium metaphosphate is formed.

Na(NH4)HPO4  NaPO3 + NH3 + H2O


Sodium
metaphosphate

Sodium metaphosphate combines with metallic oxides to form orthophosphates which are
usually coloured. The shade of the colour gives a clue regarding the presence of metal. Like
borax bead test, colours are noted both in oxidising and reducing flames in hot and cold
conditions.
Colour of the bead in
Metal Oxidising flame Reducing flame
Hot Cold Hot Cold
Copper Green Blue Colourless Red
Iron Yellow or Yellow Yellow Colourless
reddish-brown
Chromium Green Green Green Green
Manganese Violet Violet Colourless Colourless
Nickel Brown Brown · Grey

QUALITATIVE ANALYSIS
28 QUIZRR
S OLVED EXAMPLES

Example 1

Complete the following reactions :

(i) CaCO3 
 [A] + [B]
gas

(ii) [A] + H2O  [C]

(iii) [C] + [B]  CaCO3 + H2O


(iv) [D] + [C]  E
gas

(v) [E] + H2O + [B]  [F]

(vi) NaCl + [F]  [G] + [D]


(vii) [G]   Na2CO3 + H2O + [B]
Solution :


(i) CaCO  CaO + CO2
(A) (B)

(ii) CaO + H2O  Ca(OH)2


(A) (C)

(iii) Ca(OH)2 + CO2  CaCO3 + H2O


(iv) 2NH4Cl + Ca(OH)2  2NH3  + CaCl2 + 2H2O
(D) (C) (E)

(v) NH3 + H2O + CO2  NH4HCO3


(E) (B) (F)

(vi) NaCl + NH4HCO3  NaHCO3 + NH4Cl


(F) (G) (D)

(vii) 2NaHCO3   Na2CO3 + H2O + CO2
(B)

QUALITATIVE ANALYSIS
QUIZRR 29

Example 2

True or False :
(a) The carbonates of barium and calcium are soluble in acid.
(b) Copper sulphate forms a violet colour with potassium ferrocyanide solution.
(c) Ammonium sulphate can be used in place of ammonium chloride in third group.
Solution :
(a) True
(b) False. Chocolate precipitate.
(c) False. Sulphates of V group radicals will be precipitated.

Example 3

When a crystalline compound (X) is heated with K2Cr2O7 and concentrated H2SO4, a reddish-
brown gas (A) is evolved. On passing (A) into caustic soda, a yellow coloured solution of
(B) is obtained. Neutralizing the solution of (B) with acetic acid and on subsequent addition
of lead acetate, a yellow precipitate of (C) is obtained. When (X) is heated with NaOH, a
colourless gas is evolved and on passing this gas into K2HgI4 solution, a reddish-brown
precipitate (D) is formed. Identify (A), (B), (C), (D) and (X). Write the equation of reactions
involved.
Solution :
The given information in the question infers that compound (X) should be NH4Cl.

K2Cr2O7 + 2H2SO4  2KHSO4 + 2CrO3 + H2O

NH4Cl + H2SO4  NH4HSO4 + HCl

CrO3 + 2HCl  CrO2Cl2  + H2O


(A)

Reddish-brown gas

CrO2Cl2 + 4NaOH  Na2CrO4 + 2NaCl + 2H2O


(B)
Yellow coloured solution

Na2CrO4 + (CH3COO)2Pb 


CH3COOH
 PbCrO  + 2CH COONa
4 3

(C)
Yellow ppt

NH4Cl + NaOH  NaCl + NH3 + H2O


Colourless gas

QUALITATIVE ANALYSIS
30 QUIZRR

+
O

NH3 + 2K2[HgI4) + H2O Hg Hg I + 4KI + 3HI

NH2

Example 4

An aqueous solution of gas (X) shows the following reaction


(i) It turns red litmus blue
(ii) When added in excess to a copper sulphate solution, a deep blue colour is obtained
(iii) On addition of FeCl3 solution a brown ppt soluble in dilute nitric acid is obtained.
Identify (X) and give equations for the reactions at step (ii) & (iii)
Solution :
X-NH3

Reactions : (i) CuSO4 + 4NH4OH  Cu(NH3)4 SO4 + H2O


deep blue

(ii) FeCl3 + 3NH4OH  Fe(OH)3  + 3NH4Cl


brown ppt

Fe(OH)3 + 3HNO3  Fe(NO3)3 + 3H2O


Soluble

Example 5

An aqueous solution of a gas (X) gives the following reactions :


(i) It decolourizes an acidified K2Cr2O7 solution
(ii) On boiling with H2O2, cooling it and then adding an aqueous solution of BaCl2,
a white ppt insoluble in dilute HCl is obtained.
(iii) On passing H2S into the solution, turbidity is obtained.
Identify (X) and given equations for the steps (i), (ii), (iii).
Solution :
X - SO2
Reactions : (i) K2Cr2O7 + H2SO4 + 3SO2  K2SO4 + Cr2 (SO3)3 + H2O

(ii) SO2 + H2O2  H2SO4

QUALITATIVE ANALYSIS
QUIZRR 31

H2SO4 + BaCl2  BaSO4  + 2HCl


White ppt

(iii) SO2 + 2H2S  3S  + 2H2O


white turbidity

Example 6

A white amorphous powder (A) on strongly heating gives a colourless non-combustible gas
(B) and solid (C). The gas (B) turns lime water milky and turbidity disappears with the
passage of excess of gas. The solution of (C) in dilute HCl gives a white ppt. with an
aqueous solution of K4[Fe(CN)6]. The solution of (A) in dilute HCl gives a white ppt. (D) on
passing H2S in presence of excess of NH4OH. Identify (A) to (D) by giving chemical equations.
Solution :
(A) ă ZnCO3 (B) CO2 (C) ZnO (D) ZnS

Reactions : (i) ZnCO3   ZnO + CO2


(A) (C) (B)

(ii) CO2 + Ca(OH)2  CaCO3 + H2O


(B) White
CaCO3 + H2O + CO2   Ca(HCO3)2
Excess Soluble

(iii) ZnO + 2HCl  ZnCl2 + H2O


2ZnCl2 + K4Fe(CN)6  Zn2[Fe(CN)6] + 4KCl
White ppt.

(iv) ZnCl2 + H2S  ZnS + 2HCl


(D)

Example 7
A certain compound (X) is used in laboratory for analysis. Its aq. solution gave the following
reactions.
(i) On addition to copper sulphate solution, a brown ppt. is obtained which turns
white on addition of excess of Na2S2O3 solution.
(ii) On addition to Ag+ ion solution, a yellow ppt. is obtained which is insoluble in
NH4OH.
Identify (X), giving reactions.

QUALITATIVE ANALYSIS
32 QUIZRR
Solution :
X - KI

Reactions : (i) 2CuSO4 + 2KI  2CuI2 + K2SO4

2Cul2  Cu2I2 + I2


white

I2 + 2Na2S2O3  Na2S4O6 + 2NaI

(ii) Ag+ + KI  AgI + K+


Yellow ppt.

The white ppt. of Cu2I2 is coloured brown due to the presence of I2. On adding sodium thiosulphate,
I2 is consumed. Therefore, the ppt appears white.

Example 8

An aqueous solution of inorganic compound (X) gives the following reactions :


(i) With an aq. solution of barium chloride a ppt. insoluble in dil. HCl is obtained.
(ii) Addition of excess of Kl gives a brown ppt. which turns white on addition of excess of
hypo.
(iii) With an aqueous solution of K4[Fe(CN)6] a chocolate coloured ppt. is obtained.
Identify (X) and give equations for the reactions for (i), (ii) and (iii) observations.
Solution :
X - CuSO4

Reactions : (i) CuSO4 + BaCl2  BaSO4 + CuCl2


White ppt
(insoluble in HCl)

(ii) 2CuSO4 + 4KI  2CuI2 + 2K2SO4



Cu2I2 + I2

I2 + Na2S2O3  Na2S4O6 + 2NaI

(iii) CuSO4 + K4[Fe(CN)6]  Cu2[Fe(CN)6] + 2K2SO4


(Chocolate, coloured ppt.)

QUALITATIVE ANALYSIS
QUIZRR 33

Example 9

Two ores of the same metal (M) are (A1) and (A2).

 A1   Black ppt  C   CO 2  H 2 O;
calcination
  A1   I 2  ppt  D 
KI
HCl

 A2   gas  G   metal  M  ;


Roasting
G  acidified
K 2 Cr2 O7  Green solution

Identify (M), (A1), (A2), (C), (D) and (G).


Solution :

calcination
CuCO3 .Cu(OH)2 
 2CuO   CO2   H2 O
Malachite  A1  Black (C)

CuCO3 .Cu  OH 2  4HCl  2CuCl2  CO2   3H2O

2CuCl2  4KI  C u 2 I2   4KCl  I2


D

Roasting
2Cu2 S  3O2 
 2Cu 2O  2S O2
copper glance (A 2 ) G 

Cu 2 S  2Cu 2 O   K 2 SO4  Cr2  SO4 3  4H2 O


 6 Cu  SO2 ; 3SO2  K 2 Cr2 O7  H2 SO4 
 M
 green 

Example 10

Na 2 CO3 
2
 A  
SO 2
  sulphur   C 
3  B Na CO
 
2 D elements I

Find (A), (B), (C) and (D). Give oxidation state of sulphur in each compound.
Solution :
(A) : NaHSO3 (B) : Na2SO3
(C) : Na2S2O3 (D) : Na2S4O6
Oxidation state of sulphur in (A) is + 4, in (B) is also + 4, in (C) is + 2 and in (D) is + 5/2.

Example 11

A mixture consisting of A (yellow solid) and B (colourless solid) gives lilac colour in flame.
(a) Mixture gives black precipitate C on passing H2S gas in acidic medium.
(b) C is soluble in aqua-regia and on adding SnCl2, it gives white precipitate, which
finally turns to greyish black precipitate D.

QUALITATIVE ANALYSIS
34 QUIZRR
The white precipitate on treatment with NH4OH gives a brown precipitate.
(i) The sodium carbonate extract of the salt A with CCl4/FeCl3 gives a violet layer.
(ii) The sodium carbonate extract of the salt A gives yellow precipitate in AgNO3 solution,
which is insoluble in NH3. Identify A and B and the precipitate C and D.
Solution :
(A) : Kl3 (B) : HgCl2 (C) : HgS (D) : Hg

(a) HgCl2 + H2S  HgS + 2HCl


(B) Black ppt

(C)

(b) 3HgS + 6HCl + 2HNO3  3HgCl2 + 2NO + 3S + 4H2O

2HgCl2 + SnCl2  Hg2Cl2  + SnCl4


White ppt

Hg2Cl2 + SnCl2  2Hg  + SnCl4


Grey ppt

Hg2Cl2 + 2NH4OH  Hg + Hg(NH2)Cl + NH4Cl


Brown

(i) I  FeCl3  I2


CCl

4  CCl  I
4 2
extract
From soda
 Violet layer

(ii) I  AgNO3  AgI 


 From soda extract  Yellow ppt
 Insoluble in NH3 

KI3 + HgCl 2  Lilac


yellow white colour in
[A] [B] flame

Example 12

An aqueous solution containing one mole of HgI2 and two mole of NaI is orange in colour.
On addition of excess NaI the solution becomes colourless. The orange colour reappears on
subsequent addition of NaOCl. Explain with equations.

QUALITATIVE ANALYSIS
QUIZRR 35

Solution :
A solution containing one mole of HgI2 and two mole of NaI is orange in colour due to the partial
solubility of HgI2. On addition of excess of NaI, the colourless complex Na2HgI4 is formed.

2NaI + HgI2  Na2HgI4


Excess

The Na2HgI4 on addition of NaOCl, oxidises as :

3Na2HgI4 + 2NaOCl + 2H2O  3HgI2 + 2NaCl + 4NaOH + NaI3

Thus, colour of partially soluble HgI2 is restored.

Example 13
An inorganic Lewis acid (X) shows the following reactions :
(i) It fumes in moist air.
(ii) The intensity of fumes increases when a rod dipped in NH4OH is brought near it.
(iii) An acidic solution of (X) on addition of NH4Cl and NH4OH gives a precipitate which
dissolves in NaOH solution.
(iv) An acidic solution of (X) does not give a precipitate with H2S. Identify (X) and give
chemical equation for steps (i) to (iii).
Solution :
X ă AlCl3
Reactions : (i) AlCl3 + 3H2O  Al(OH)3 + 3HCl 
fumes

(ii) HCl + NH4OH  NH4Cl  + H2O


White fumes

(iii) AlCl3 + 3NH4OH  Al(OH)3 + 3NH4Cl


White ppt

Al(OH)3 + NaOH  NaAlO2 + 2H2O


Soluble

Example 14

(i) A black mineral (A) on treatment with dilute sodium cyanide solution in presence of
air gives a clear solution of (B) and (C).
(ii) The solution of (B) on reaction with zinc gives a precipitate of metal (D).

QUALITATIVE ANALYSIS
36 QUIZRR
(iii) (D) is dissolved in dil. HNO3 and the resulting solution gives a white precipitate (E)
with dil. HCl.
(iv) (E) on fusion with sodium carbonate gives (D).
(v) (E) dissolves in aqueous solution of ammonia giving a colourless solution of (F). Identify
(A) to (F) and give chemical equations for reactions involved in steps (i) to (v).
Solution :
(A) ă Ag2S (B) ă NaAg(CN)2 (C) ă Na2SO4 (D) Ag
(E) ă AgCl (F) ă Ag(NH3)2Cl

Reactions : (i) Ag2S + 4NaCN + 2O2  2NaAg(CN)2 + Na2SO4


(A) (B) (C)

(ii) 2NaAg(CN)2 + Zn  Na2Zn(CN)4 + 2Ag


(D)

(iii) 3Ag + 4HNO3  3AgNO3 + NO + 2H2O

(iv) HCl
AgNO3   AgCl + HNO3

(v) AgCl + 2NH3  Ag(NH3)2Cl


(E) (F)

(vi) 4AgCl + 2Na2CO3  4Ag + 4NaCl + 2CO2 + O2

Example 15
A solid laboratory reagent (A) gives the following reactions :
(i) it imparts green colour to flame
(ii) Its solution does not give ppt. on passing H2S
(iii) When it is heated with K2Cr2O7 and conc. H2SO4 a red gas is evolved. The gas when
passed in aq. NaOH solution turns it yellow. Identify (A) giving chemical reactions.
Solution :
A ă BaCl2

Reactions : (i) 2BaCl2 + K2Cr2O7 + 3HSO4  K2SO4 + 2CrO2Cl2 + 2BaSO4 + 3H2O


red gas

(ii) CrO2Cl2 + 4NaOH  Na2CrO4 + 2NaCl + 2H2O

yellow solution

QUALITATIVE ANALYSIS
QUIZRR 37

Example 16

(A) is yellow coloured solid partially soluble in aqueous NH3. (A) is soluble in Na2S2O3
(hypo) solution forming a complex (B) which on heating is converted into (C) (black). (C)
is converted into white ppt (D) on reaction with HCl and HNO3 . (D) is soluble in aqueous
NH3 forming (E). Identify (A), (B), (C), (D) and (E) and explain reactions.
Solution :
(A) is AgBr (present in photographic plate)

AgBr + 2Na2S2O3 Na3[Ag(S2O3)2] + NaBr


hypo soluble
(B)

NH3 HCl + HNO3


[Ag(NH3)2Cl] AgCl Ag2S
(E) (D) (C)

Example 17
Identify (A), (B), (C) and (D) based on following observations :

(i) A  glassy transparent bead (B) on platinum wire (B) + CuSO4  coloured

bead (C)

(ii) ignite
(A) + conc. H2SO4 + CH3CH2OH   green flame (D)
(iii) Aqueous solution of (A) is alkaline.
Solution :
(i) (A) forms glassy transparent bead which is characteristic property of borax.
 (A) is borax (Na2B4O7.10H2O)


(ii) Na 2 B4 O7 .10H2O  NaBO2 + B2 O3  10H2O
  
 B  glassy bead


B2O3  CuSO4  Cu  BO2 2  SO3 
coloured bead C

H3 BO3  3C2 H5OH   


C2 H5 3 BO3  Na 2SO4  4H3 BO3
green flame (on ignition)
D

QUALITATIVE ANALYSIS
38 QUIZRR

(iii) Na 2 B4 O7  5H2O  2H3 BO3  2Na[B  OH 4 ]


weak acid

Na[B(OH)4] reacts with acid (HCI) hence aqueous solution (A) is alkaline.

Example 18

Name one common reagent that can precipitate or react and differentiate following pairs :
(a) Ag+ and Ba2+ (b) Cu2+ and Pb2+
(c) Iă and Clă (d) Iă and Bră

(v) SO 23  and SO 24  (f) Fe3+ and Cu2+

Solution :
(a) K2CrO4 :

2Ag   CrO24  Ag 2CrO4 


red

Ba 2  CrO42  BaCrO4 


yellow

(b) K2CrO4 :

Cu 2  CrO42   CuCrO4 


chocolate

Pb2  CrO42  PbCrO4 


yellow

(c) AgNO3 :

Ag   I  AgI 
yellow

Ag   Cl   AgCl 
white

(d) Cl2 water + CHCl3 :

2I   Cl2  I2  2Cl 


(violet in
CHCl3 layer)

2Br   Cl2  Br2  2Cl


(orange in
CHCl3 layer)

QUALITATIVE ANALYSIS
QUIZRR 39

(e) BaCl2 :

BaCl2  SO23  BaSO3 


white ppt soluble in conc. HCl

BaCl2  SO24  BaSO4 


white ppt insoluble in conc. HCl

(f) NH4SCN :

Fe 3  SCN   [Fe  SCN ]2


red colour

Cu2  2SCN  Cu  SCN 2 


black

Example 19

The gas liberated on heating a mixture of two salts with NaOH, gives a reddish brown
precipitate with an alkaline solution of K2HgI4. The aqueous solution of mixture on treatment
with BaCl2 gives a white precipitate which is sparingly soluble in conc. HCl. On heating the
mixture with K2Cr2O7 and conc. H2SO4, red vapours (A) are produced. The aqueous solution
of mixture gives a deep blue colouration (B) with potassium ferricyanide. Identify the
radicals in given mixture and write the balanced equation for formation of (A) and (B).
Solution :
(i) Gas given by heating mixture with NaOH gives brown ppt with NesslerÊs reagent K2HgI4.
It shows that the salt is a ammonium salt.
(ii) BaCl2 solution gives white precipitate sparingly soluble in HCl. It shows the presence of

SO24 ion in the mixture

(iii) Blue colour with K3Fe(CN)6 indicates the presence of Fe2+ ion.

(iv) Mixture + K2Cr2O7 


H SO
2 4  Red vapours

 CrO2Cl2 
A

It shows that the mixture contains Clă ion. Reactions involved may be given as :


(i) NH4 Cl  NaOH  NaCl  NH3   H2O

Hg
K 2 HgI4 + NaOH + NH3  O NH 2 I + KI + H2 O + NaI
Hg
 Brown ppt 

QUALITATIVE ANALYSIS
40 QUIZRR

(ii) FeSO4  BaCl2  BaSO4  FeCl2


 White ppt 

(iii) 3FeSO4  2K 3 Fe  CN 6  3K 2SO4  Fe3 [Fe  CN 6 ]2


Ferrous ferricyanide
 Blue 

(iv) 4NH4 Cl + K 2 Cr2 O7 + 3H2 SO4  2CrO2 Cl2 + K 2 SO4 + 3H2 O + 2  NH4 2 SO4
Chromyl chloride

Example 20

A white substance (A) reacts with dilute H2SO4 to produce a colourless gas (B) and a
colourless solution (C). The reaction between (B) and acidified K2CrO7 solution produces a
green solution and a slightly coloured precipitate (D). The substance (D) burns in air to
produce a gas (E) which reacts with (B) to yield (D) and a colourless liquid. Anhydrous
copper sulphate is turned blue on addition of this colourless liquid. Addition of aqueous
NH3 or NaOH to (C) produces first a precipitate, which dissolves in the excess of respective
reagent to produce a clear solution in each case. Identify (A), (B), (C), (D) and (E) write the
equations of the reactions involved.
Solution :
The white substance (A) is ZnS.

ZnS + H2 SO4  ZnSO4  H 2S


A Colourless Gas  colourless 
 C  B

K 2 Cr2O7  4H2SO4 + 3H2 S  Cr2  SO4 3  K 2SO4  7H 2O + 3S


White grey
Green solution
D

S + O2  SO2
D  air  E

SO2 + 2H2 S  3S  2H2 O


E B  D Colourless liquid

CuSO4  5H2 O  CuSO4 5H2 O


White Blue
 Anhydrous 

QUALITATIVE ANALYSIS
QUIZRR 41

ZnSO4  2NaOH  Zn  OH 2  Na 2SO4


 C White ppt

Zn  OH 2  2NaOH  Na 2 ZnO2  2H2O


Soluble in water
 Clear solution 

ZnSO4  2NH4 OH  Zn  OH 2   NH4 2 SO4

Example 21

Identify (A) to (G) in the following scheme and name the process.


CaCO3  A   B gas

 A  H 2 O   C

 C   B  CaCO3  H 2 O


 D   C   E gas

 E  H2 O   B   F

NaCl +  F  G   D



 G  Na 2 CO3  H 2 O +  B
Solution :
Scheme represents the Solvay process of manufacture of Na2CO3.

CaCO3  CaO  CO2  g 



A  B

CaO + H2O  Ca  OH 2


A  C

Ca  OH 2  CO2  CaCO3  H2O


2NH4 Cl + Ca  OH 2  2NH3   CaCl2  2H2O
 D  C  E

QUALITATIVE ANALYSIS
42 QUIZRR

NH3 + H 2O  CO2  NH4 HCO3


E  B  F

NaCl + NH4 HCO3  NaHCO3  NH4 Cl


F G (D)


2NaHCO3  Na 2CO3  H2O + CO2
B

Example 22

Identify (A), (B), (C)..... (H) in the following and explain reactions :

 A black + dil H 2SO4  B  aq   C  aq   D  g 

 B  KI  brown coloured solution

hypo

white ppt (E)

 C  MnO4  H+  colourless solution

 C  K 3 [Fe  CN 6 ]  blue  F 

 D   CH3 COO 2 Pb  black  G 

(B) or (C) + BaCl2  white ppt (H)

(B) + (D)  black ppt (I)

Solution :

(D) + (CH3COO)2Pb  black (G)

 (D) is H2S gas and (G) is PbS


 (A) also has S2ă

(A) + dil. H2SO4  (B), (C)


(B) and (C) both are sulphates as confirmed by white ppt (H) with BaCl2

(B) + KI  brown coloured solution changing to white ppt (E)

QUALITATIVE ANALYSIS
QUIZRR 43

Thus, (B) is CuSO4

2CuSO4  4KI  Cu 2I2  2K 2 SO4  I2


E

I 2  KI  KI3

Due to KI3, solution appears reddish brown. On adding hypo, I2 disappears and white ppt of
Cu2I2 appears.

I  I

3 2  2Na 2S2O3  2NaI + Na 2S4 O6
hypo

(C) decolorises MnO4 / H+ . (C) also gives blue colour with K3[Fe(CN)6 thus (C) is FeSO4.

FeSO4  K 3 [Fe  CN 6 ]  KFeII [FeIII  CN 6 ]  K 2 SO4


Turnbull's blue (F)

(A) : CuS + FeS (CuFeS2) (B) : CuSO4


(C) : FeSO4 (D) : H2S
(E) : Cu2I2 (F) : KFeII [FeIII(CN)6]
(G) : PbS (H) : BaSO4
(I) : CuS

Example 23

Black coloured (insoluble in H2O) solid (A) does not dissolve in dil. HNO3. Aqua regia can
dissolve (A) forming (B). (B) gives yellow ppt. (C) with NaOH. (B) also gives orange ppt (D)

with KI; (D) dissolves in excess of KI forming (E). (E) gives brown ppt with NH 4 salt in
presence of NaOH. (A) is precipitated if H2S gas is passed into solution of (B) in dil. HCl.
Identify (A) to (E) and explain reactions.
Solution :
(A) is insoluble in dil. HNO3.
 (A) is of group II (copper sub-group) and is HgS [only HgS (black) is insoluble in dil. HNO3]

A  3HCl + HNO3  HgCl2


  (B)
aqua regia

HgCl2  2NaOH  HgO  + 2NaCl + H2O


yellow ppt (C)

QUALITATIVE ANALYSIS
44 QUIZRR

HgCl2  2KI  HgI2  + 2KCl + H2 O


(D)  orange 

HgI2  2KI  K 2 HgI4


 E   soluble 


NH+4  NaOH  NH3  H 2O + Na +

Hg
3NaOH + NH3  2K 2 HgI 4  O NH 2 I   4KI + 3NaI + 2H2 O
Hg
brown ppt
 iodide of MIllion's base

HgCl2  H2 S  HgS  + 2HCl


B black

Example 24
Colourless salt, (A) (insoluble in water) + dil. H2SO4 (B) + C(g)

+ NaOH NaOH
(B) (D) (white ppt) (E) (soluble)
(excess)

NH4Cl + NH4OH (C)

(C)
no ppt (A) (A) (white ppt)

(C) + (F) (yellow coloured solution) light green (G)

K4[Fe(CN)6]
blue (H)

AgNO3
+O2 white ppt (I) (soluble in NH3)

+C
H2O + (J) colloidal white turbidity

Cr2O72ă/H+
green

Ca(OH)2
milky

Identify (A) to (J) and explain reactions.

QUALITATIVE ANALYSIS
QUIZRR 45

Solution :
Gas (C) is vital compound in this scheme.

Gas  C   J


O
2 H O +
2

Ca  OH  2 / H 
Cr2O7
milky  2  Gas (J)   green

 (J) is SO2
 (C) is H2S (giving H2O and SO2 by combustion in O2).
(B) gives white ppt only if (C) is passed into its ammoniacal solution containing NH4Cl
(NH4OH + NH4Cl)
 (B) has Zn2+ and (B) is ZnSO4
 (A) is ZnS
(F) gives blue colour with K4[Fe(CN)6]
 (F) has Fe3+ ion
(F) gives white ppt with AgNO3
 (F) has Clă ion. Thus, F is FeCl3.

E xplanation :

ZnS + H2SO4  ZnSO4  H2S


(A) (B)  C

ZnSO 4 + 2NaOH  Zn(OH) 2  Na 2SO4


(B) (D)

Zn(OH) 2 + 2NaOH  Na 2 ZnO2  2H2 O


(D) soluble (E)

Na 2 ZnO2 + H2S  ZnS  2NaOH


(E)  C (A)

H2S + 2 FeCl3  2FeCl 2  2HCl + S


 C F (G) green
yellow

III II
FeCl3  K 4 [Fe  CN 6 ]  K Fe [ Fe  CN 6 ]  3KCl
F   H , Prussian blue

QUALITATIVE ANALYSIS
46 QUIZRR

FeCl3  3AgNO3  Fe  NO3 3  3AgCl 


white ppt (I)

AgCl + 2NH3  [Ag  NH3 2 ]Cl


I  soluble


2H2S + 3O2  2H2O + 2SO2
 J

SO2  2H 2S  2H2O + 3S 


J  C colloidal
white turbidity

3SO2  Cr2 O72  2H+  2Cr3+  3SO42   H2 O


green

SO2  Ca  OH 2  CaSO3  H2O


milky

QUALITATIVE ANALYSIS
BIOMOLECULES
QUIZRR 3

BIOMOLECULES

1.1 Carbohydrate

Carbohydrates have general formula, Cx(H2O)y, and were considered as hydrates of carbon from
where the name carbohydrate was derived. But this is not completely true.
For example, the molecular formula of glucose (C6H12O6) fits into this general formula,C6(H2O)6.
But all the compounds which fit into this formula may not be classified as carbohydrates. Acetic
acid (CH3COOH) fits into this general formula, C2(H2O)2 but is not a carbohydrate.
Similarly, rhamnose, C6H12O5 is a carbohydrate but does not fit in this definition. A largenumber
of their reactions have shown that they contain specific functional groups.
Chemically, the carbohydrates may be defined as optically active polyhydroxy aldehydes or ketones
or the compounds which produce such units on hydrolysis.

1.2 Classification

Carbohydrates are classified into three groups based on the number of sugar units and upon their
behaviour towards hydrolysis.
They are
 Monosaccharides
 Oligosaccharides
 Polysaccharides.
1. Monosaccharides : These are simplest group of carbohydrates and are referred as simple
sugars as they are sweet in taste. The cannot be further hydrolyzed to simpler compounds.
They have the general formula Cn(H2O)n. Examples: Glucose and fructose.
Monosaccharides can be further classified into :
(a) Aldose is polyhydroxy aldehyde (glucose, galactose)
(b) Ketose is polyhydroxy ketone (fructose)
CHO CH2OH CHO

H OH O H OH
HO H HO H HO H
H OH H OH HO H
H OH H OH H OH

CH2OH CH2OH CH2OH


D-glucose D-fructose D-galactose
aldehxose ketohexose aldehexose

 Glucose is also called grape sugar or blood sugar; fructose is also called fruit sugar.

BIOMOLECULES
4 QUIZRR
2. Oligosaccharides : These carbohydrates liberate two to ten monosaccharide molecules on
hydrolysis. They are further classified as disaccharides, trisaccharides, tetrasaccharides, etc.
based on the number of monosaccharide units. For e.g., disaccharides like sucrose produce
two molecules of monosaccharides on hydrolysis.

Hydrolysis
Sucrose 
 glucose + fructose

A trisaccharide like raffinose on hydrolysis gives glucose, fructose and galactose.


3. Polysaccharides : This carbohydrates liberate a large number of monosaccharide molecules
on hydrolysis. They are usually amorphous, insoluble in water and tasteless and are called
non-sugars.
Examples : Starch, cellulose and glycogen

2. Carbohydrates may also be classified as either reducing or non-reducing sugars.

(a) Reducing Sugar


All those carbohydrates which contain free aldehyde or ketonic group and reduce FehlingÂs
solution and TollenÂs reagent referred to as reducing sugars. All monosaccharides whether
aldose or ketose are reducing sugars. Their functional groups are free. Examples: Maltose
and lactose.
(b) Non Reducing Sugar
In disaccharides if the reducing group of monosaccharides i.e., aldehydic or ketonic groups
are bonded, these are non reducing sugars e.g., sucrose, while others in which these functional
groups are free are reducing sugars. They donÂt reduce Felling or TollentÂs reagent.
Examples : Sucrose

CHO CHO CH2OH

(CHOH)4 (CHOH)4 CHO C=O

CH2 O CH2 (CHOH)4 (CHOH)3


Maltose
CH2 O CH2
Sucrose

Trioses

D and L Terminology : The simplest of all carbohydrates that CHO CH2OH


fit the definition we have given for carbohydrates are the trioses,
*CHOH C=O
glyceraldehydes and dihydroxyacetone. Glyceraldehyde is
aldotriose, and dihydroxyacetone is a ketotriose. CH2OH CH2OH
Glyceraldehyde Dihydroxyacetone
(an aldotriose) (a ketotriose)

BIOMOLECULES
QUIZRR 5

Glyceraldehyde contains one asymmetric carbon atom (marked CHO CHO


OH H
by an asterisk) and can thus exist in two optically active
forms, called the D-form and the L-form. Clearly, the two H HO
forms are mirror images that cannot be superimposed, that
is they are ena ntiomers. HO HO
DăGlyceraldehyde LăGlyceraldehyde

O O
1 1
CHO D Means on CH
the right
H · 2C · OH HO · 2C · H L Means on
3 3 the Left
CH2OH CH2OH
DăGlycerldehyde LăGlycerldehyde

The letters ÂDÊ or ÂLÊ before the name of any compound indicate the relative configuration of
a particular stereoisomer. This refers to their relation with a particular isomer of
glyceraldehydes. Glyceraldehyde contains one asymmetric carbon atom and exists in two
enantiomeric forms
For assigning the configuration of monosaccharides, it is the lowest asymmetric carbon atom (as
shown below) which is compared. As in glucose,·OH on the lowest asymmetric carbon is on the
right side which is comparable to glyceraldehydes, so it is assigned D-configuration. For this
comparison, the structure is written in a way that most oxidized carbon is at the top
CHO

H OH

CHO OH H
H OH
H OH
H OH
CH2OH
CH2OH
Dă(+) ă Glyceraldehyde Dă(+) Glucose

D and L have nothing to take with direction of rotation of light. The maximum number of optical
isomers of a sugar is related to the number of asymmetric carbon atoms in the molecule and may
be calculated by the following simple equation.
Maximum Number of Optical Isomors = 2n, where n = the number of asymmetric carbon
atoms. Since glyceraldehydes contains only one asymmetric carbon atom, the number of optical
isomer is 21. We know that 21 is = 2, and we have seen that there are indeed two different
glyceraldehydes.

BIOMOLECULES
6 QUIZRR
A ldotetroses
1. CHO
If we examine the general formula of an aldotetrose, we see
2. *CHOH
that they contain two asymmetric carbon atoms (marked by
asterisks). 3. *CHOH
This means that 22 or 4 optical isomers are possible. They may
4. CH2OH an aldotetrose
be represented as the following two pairs :

CHO CHO CHO CHO

H C OH HO C H HO C H H C OH

H C OH HO C H H C OH HO C H

CH2OH CH2OH CH2OH CH2OH


Mirror images (racemic pair) Mirror images (racemic pair)

D (ă)-erythrose L (+)-erythrose D (ă)-threose L (+)-threose

All four isomers have been prepared synthetically. The D- and L-erythrose are mirror images, that
is, they are enantiomers. They have exactly the same degree of rotation but in opposite directions.
Equal amounts of the two would constitute a racemic mixture, that is, a mixture that would allow
a plane-polarised light to pass through the solution unchanged but could be separated into
dextrorotatory and laevorotatory isomers. The same comments hold for D- and L-threose. However,
D-erythrose and L-threose are not images, that is, they are diastereomers (optical isomers that are
not mirror images are called diastereomers), and the degree of rotation of each would probably
differ.
1. CHO
Aldopentoses
2. *CHOH
If we examine the general formula of an aldopentose, we see that they contain
three asymmetric carbon atoms. 3. *CHOH
This means that 23 or 8 optical isomers are possible. These are : D(ă) lyxose, 4. *CHOH
L(+)-lyxose, D(ă) xylose, L(+) xylose, D(ă) arabinose, L(+)-arabinose, D(ă)-ribose,
L(+)-ribose 5. CH2OH
an aldopentose
Aldohexoses
1. CHO
If we examine the general formula of aldohexose, we see that it contains four
2. *CHOH
asymmetric carbon atoms. This means that 24 or 16 optical isomers are possible.
D and L forms of altrose, allose glucose, mannose, galactose, talose, arabinose 3. *CHOH
and idose.
4. *CHOH
Only three of the sixteen possible aldohexoses are found in nature (all sixteen
isomers have been prepared synthetically). They are D-glucose, D- mannose, 5. *CHOH
and D-galactose. No one of these three optical isomers is a mirror image of any
of the others, so all three are diastereomers of each other. 6. CH2OH
an aldohexose
BIOMOLECULES
QUIZRR 7

Epimers
A pair of diastereomers that differ only in the configuration about of a single carbon atom are said
to be epimers. e.g D(+)- glucose is empimeric with D(+)-mannose and D(+) ăgalactose as shown
below :

Epimers Epimers
O H O H O H
C C C

H · C · OH H · C · OH HO · C · H

HO · C · H HO · C · H HO · C · H

HO · C · H H · C · OH H · C · OH

H · C · OH H · C · OH H · C · OH

CH2OH CH2OH CH2OH


D (+)ăGalactose D (+)ăGlucose D (+)ăMannose

Example 1

Why are optically active tartaric acid and 1,2,3,4-butanetetrol called D?


Solution :
Since the configuration of asymmetric (chiral) carbons do not change during the oxidation or
reduction, the D configuration is retained.

COOH CH2OH

HO · C · H LiAlH4 HO · C · H

H · C · OH H · C · OH

COOH CH2OH
Dătartaric acid Dă1,2,3,4ăButanetetrol

PREPARATION OF GLUCOSE AND FRUCTOSE

H+
C12 H22O11  H2 O 
 C6 H12O6  C6H12 O6
Ć
sucrose glucose fructose

Glucose and fructose are separated by fractional crystallisation.

BIOMOLECULES
8 QUIZRR

H +
(C6 H12 O5 )n  nH2 O 
 nC6 H12 O6
Ć 120C, 2-3 atm
starch glucose

After neutralisation with CaCO3, and filtration, filtrate is decolourised by boiling with animal
charcoal and then concentrated under reduced pressure and crystallised.

H SO
(C6 H10 O5 ) n  nH2 O 
2 4 nC6 H12 O6
Ć 
insulin fructose

Physical Properties

Ć Glucose and fructose both are soluble in H2O, sparingly soluble in alcohol and insoluble in
ether.
Ć Melting points : glucose : 160ĈC, fructose : 102.4ĈC
Ć Glucose is dextrorotatory hence, called dextrose.
Ć Fructose is laevorotatory hence, called laevulose.

CHEMICAL PROPERTIES

1. On prolonged heating with HI, it forms n-hexane, suggesting that all the six carbon atoms
are linked in a straight chain.

CHO
HI, 
(CHOH)4 CH3 · CH2 · CH2 · CH2 · CH2 · CH3
(năHexane)
CH2OH

2. Glucose reacts with hydroxylamine to form an oxime and adds a molecule of hydrogen
cyanide to give cyanohydrin. These reactions confirm the presence of a carbonyl group
(>C = 0) in glucose.

CN
CHO CH N · OH CHO CH
OH
NH2OH HCN
(CHOH)4 (CHOH)4 (CHOH)4 (CHOH)4

CH2OH CH2OH CH2OH CH2OH

3. Glucose gets oxidized to six carbon carboxylic acid (gluconic acid) on reaction with a mild
oxidising agent like bromine water. This indicates that the carbonyl group is present as an
aldehydic group.

BIOMOLECULES
QUIZRR 9

CHO COOH
Br2 water
(CHOH)4 (CHOH)4

CH2OH CH2OH
Gluconic acid

4. Acetylation of glucose with acetic anhydride gives glucose pentaacetate which confirms the
presence of five ăOH groups. Since it exists as a stable compound, five ăOH groups should
be attached to different carbon atoms.

CHO CHO O
Acetic anhydride
(CHOH)4 (CH·O·C·CH3)4
O
CH2OH CH2·O·C·CH3

5. On oxidation with nitric acid, glucose as well as gluconic acid both yield a dicarboxylic
acid, saccharic acid. This indicates the presence of a primary alcoholic (-OH) group in
glucose.

CHO COOH COOH


Oxidation Oxidation
(CHOH)4 (CHOH)4 (CHOH)4

CH2OH COOH CH2OH


Saccharic Gluconic
acid acid

6. D-glucose reacts with phenyl hydrazine to give glucose phenyl hydrazine which is soluble.
If excess of phenyl hydrazine is used, a dihydrazone, known as osazone is obtained.

CHO CH NNHC6H5
C6H5NHNH2
H C OH HCl
H C OH

(CHOH)4 (CHOH)4

CH2OH CH2OH
Dăglucose Dăglucose phenyl hydrazone

2 C6H5NHNH2

BIOMOLECULES
10 QUIZRR
CH NNHC6H5
NH3 + C6H5NH2 + C NNHC6H5

(CHOH)4

CH2OH
DăClucosazone

7. On oxidation with nitric acid, glucose as well as gluconic acid both yield a dicarboxylic acid
saccharic acid. This again indicates presence of a primary alcoholic group in glucose.

HNO3
HOCH2 ă (CHOH)4 ă CHO HOOC ă (CHOH)4 ă COOH
Glucose Saccharic acid

8. Glucose reduces ammoniacal silver nitrate solution (TollentÊs reagent) to metallic silver and
also FehlingsÊ solution to reddish brown cuprous oxide and itself gets oxidized to gluconic
acid. This confirms the presence of an aldehydic group in Glucose.

HOCH2 ă (CHOH)4 ă CHO + Ag2O HOCH2 ă (CHOH)4 ă COOH + 2Ag


Glucose Gluconic acid

CONVERSION OF A KETOSE INTO AN ALDOSE AND VICE-VERSA

The ketoses are reduced to corresponding polyhydric alcohol, which is then oxidized to a
monocarboxylic acid. On warming the acid it is converted into -lactone, which on reduction with
Na/Hg in faintly acidic medium gives aldose.

CO CHO

CHOH CHOH
O
CH2OH CH2OH CHOH CHOH

C O CHOH CH CHOH
H2/Ni dil. HNO3 AgOH

(CHăOH)3 (CHăOH)3 heat CHOH CHOH

CH2OH CH2OH CH2OH CH2OH


(Ketohexose) ălactone Aldohexose

Theoretically, two polyhydric alcohol can be formed by reduction of ketose due to the formation
of the new asymmetric carbon atom. In practice, however, one isomer is obtained in great yield.
The aldose is converted into its osazone, which is then treated with PhCHO to form osone. On
reduction with zinc and acetic acid, the osone is converted into the ketose.

BIOMOLECULES
QUIZRR 11

CH NNHC6H5 CHO + CH2OH


2PhCHO e:H
ă2PhCH=NNHPh reduction
C NNHC6H5 CO CO

Glucosazone Osone Fructose

Cyclic Structure of D-Glucose


The open chain structure of glucose proposed by Baeyer explained most of its properties. However,
it could not explain the following:
1. Despite having an aldehydic group, glucose does not gives SchiffÊs test and it does not react
with sodium bi-sulphite and ammonia.
2. The penta acetate of glucose does not react with hydroxylamine indicating absence of -CHO
group.
3. Mutarotation. When glucose was crystallized from a concentrated solution at 30Ĉ C it gave
a form of glucose (Melting point 146ĈC) whose optical rotation is 111Ĉ. The  form (Melting
point 150Ĉ) obtained on crystallization of glucose from a hot saturated aqueous solution at
a temperature above 98ĈC has an optical rotation of 19.2Ĉ. These two forms of glucose are
called anomers.
This behaviour could not be explained by the open chain structure (I) for glucose. It was proposed
that one of the ·OH groups may add to the ·CHO group and form a cyclic hemiacetal structure.
It was found that glucose forms a six-membered ring in which ·OH at C-5 is involved in ring
formation. This explains the absence of ·CHO group and also existence of glucose in two forms
as shown below. These two cyclic forms exist in equilibrium with open chain structure.

Anomeric Carbon

A pair of stereoisomers that differ in configurations around C-1 are called anomers and the C-1
carbon is called anomeric carbon. The  and ă cyclic forms of D-glucose are known as anomers.
In this case, -anomers of glucose contains the -OH group towards right at C-1 position and -
anomer of glucose contains the -OH group towards left at C-1 position. So, D-glucose exists in two
stereo isomeric forms with different specific rotations and melting points. The structures of  and
 anomers of carbon are shown below :
O
1 1 1
H · C · OH H·C HO · C · H

2 2 2
H OH H OH H OH
O O
3 3 3
HO H HO H HO H
4 4 4
H OH H OH H OH
5 5 5
H H OH H
6 6 6
CH2OH CH2OH CH2OH
 · D · (+) · Glucose (open structure)  ă D ă (+) ă Glucose

BIOMOLECULES
12 QUIZRR
The six membered cyclic structure of glucose is called pyranose structure, in analogy with
pyran. Pyran is a cyclic organic compound with one oxygen atom and five carbon atoms in the
ring. The cyclic structure of glucose is more correctly represented by Haworth structure as given
below.

6 6
CH2OH CH2OH
O
H 5 O H H 5 O OH
4H 1 4H 1
OH H OH H
HO 3 2 OH HO 3 2 H
Pyran
H OH H OH
ăD ă (+) ă Glucopyranose ăD ă (+) ă Glucopyranose

D isaccharides

A disaccharide upon hydrolysis liberates two monosaccharide units. These two molecules are held
together by a glycosidic bond. The monosaccharides liberated due to hydrolysis may be of similar
or different molecules. The disaccharides are of two types. They are ă Reducing sugars and non-
reducing sugars.
The most common disaccharides are :
Ć Sucrose (cane sugar) made up of glucose + fructose
Ć Maltose (Malt sugar) made up of glucose + glucose

Sucrose

Sucrose is made up of -D-Glucose and -D-fructose held together by a glycosidic bond, between
C1 of -glucose and C2 of -fructose. The reducing groups of glucose and fructose are involved
in glycosidic bond, so it is a non-reducing sugar.

anomeric carbon
CH2OH
5 1
H O H HOH2C O H
4 H 1 2 5
OH H O H OH CH OH
OH  ă Link
2
3 2 3 4
H OH OH H
ă D ă Glucopyranose ăD ă Fructose

Sucrose is a colourless, crystalline and sweet substance soluble in water. Sucrase is the enzyme
that can hydrolyze sucrose in the body.

BIOMOLECULES
QUIZRR 13

Sucrose is a non-reducing sugar as carbony1 group is not free


(i) Inversion of cane sugar On hydrolysis with dilute acids or enzyme invertose, canesugar
gives equimolar mixture of D(+) glucose and D(-) fructose.

HCl
C12 H22 O11 + H2O C6 H12 O6 + H12 O6
Sucrose D ă Glucose D fructose

[]D = + 66.5Ĉ []D = + 52.5Ĉ []D = ă 92.4Ĉ

Sucrose is dextrorotatory but after hydrolysis gives dextrorotary glucose and laevorotatory
fructose. Since the laevorotation of fructose (-92.4Ĉ ) is more than dextrotation of glucose
(+52.5) the mixture is laevorotatory. Thus hydrolysis of sucrose brings a change in the sign
of rotation from dextro(+) is laevo(-) and is known as inversion and the mixture is known
as invert sugar.
Sucrose solution is fermented by yeast when the enzyme invertase hydrolyses sucrose to
glucose and fructose.

Invertase
C12 H22 O11 + H2O C6 H12 O6 + C6 H12 O6
Sucrose glucose fructose

Enzyme zymase converts these monosaccarides to ethyl alcohol.


zymese
C6 H12 O6 2C2H5OH + 2 CO2
monosaccaride

(ii) Maltose : Another disaccharide, maltose is composed of two -D-glucose units in which C1
of one glucose (I) is linked to C4 of another glucose unit (II). The free aldehyde group can
be produced at C1 of second glucose in solution and it shows reducing properties so it is a
reducing sugar.

6 6
CH2OH CH2OH
H 5 O H H 5 O H
4H 1 4H 1
OH H O OH H
HO 3 2 3 2 OH

H OH H OH
(I) (II)
ăD ă Glucose ăD ă Glucose
Maltose

T ESTS FOR CARBOHYDRATE

(i) When heated in a dry test tube, it melts, turns brown and finally black, giving a characteristic
smell of burning sugar.
(ii) When warmed with a little concentrated H2SO4, it leaves a charred residue of carbon.

BIOMOLECULES
14 QUIZRR
(iii) MolischÊs test : A drop or two of alcoholic solution of -naphthol is added to 2 ml of glucose
solution and 1 ml of concentrated H2SO4 is added carefully along the sides of the test tube.
The formation of a violet ring, at the junction of two liquids confirms the presence of a
carbohydrate.

T est Of Starch

Ć When few drops of I2 are added into starch solution, it turns into blue color which disappears
on heating and reappears again on cooling.

BIOMOLECULES
QUIZRR 15

AMINO ACID

Amino Acid

Amino acids are molecules which contain amino (ăNH2) and carboxyl (ăCOOH) functional groups.
Depending upon the relative position of amino group with respect to carboxyl group, the amino
acids can be classified as    and so on. Only -amino acids are obtained on hydrolysis of
proteins. They
An amino acid cab be representd by :

R · CH · COOH

NH2
ăamino acid
(R = side chain)

Classification of Amino

Based on Polarity Amino acids are classified into different ways based on polarity, structure,
nutritional requirement, metabolic rate, etc. Generally used classification is based on polarity.
Based on polarity amino acids are classified into four groups.
1. Non-polar amino acids : They have equal number of amino and carboxyl groups and are
neutral. These amino acids are hydrophobic and have no charge on the ÂRÊ group. The amino
acids in this group are alanine, valine, leucine, isoleucine, etc.

CH3
CH2
ă ă
CH · CH · COO H · CH · COO
H3C + +
NH3 NH3
Isoleucine Glycine

2. Polar amino acids with no charge : These amino acids do not have any charge on the
ÂRÊ group. These amino acids participate in hydrogen bonding of protein structure. The
amino acids in this group are - serine, threonine etc.

ă
CH2 · CH · COO ă H3C · CH · CH · COO
+ +
OH NH3 OH NH3
Serine Threonine

BIOMOLECULES
16 QUIZRR
3. Polar amino acids with positive charge : Polar amino acids with positive charge have
more amino groups as compared to carboxyl groups making it basic. The amino acids, which
have positive charge on the ÂRÊ group are placed in this category. They are lysine, arginine
etc.

ă
CH2 · CH2 · CH2 · CH2 · CH · COO
+ +
NH3 NH3
Lysine

4. Polar amino acids with negative charge : Polar amino acids with negative charge have
more carboxyl groups than amino groups making them acidic. The amino acids, which have
negative charge on the ÂRÊ group are placed in this category. They are called as dicarboxylic
mono-amino acids. They are aspartic acid and glutamic acid.

ă ă
OOC · CH2 · CH · COO
+
NH3
Aspartic acid

Physical Properties Of Amino acids

Ć Amino acids are usually colourless, crystalline solids.


Ć These are water-soluble, high melting solids and behave like salts rather than simple amines
or carboxylic acids.
Ć This behaviour is due to the presence of both acidic (carboxyl group) and basic (amino
group) groups in the same molecule, the carboxyl group can lose a proton and amino group
can accept a proton, giving rise to a dipolar ion known as zwitter ion.
• This is neutral but contains both positive and negative charges.

O O
ă
R · CH · C · O · H R · CH · C · O
+
:NH2 NH3
(Zwitter ion)

In zwitter ionic form, amino acids slow amphoteric behavior as they react both with acids
and bases.
In acidic solution, the carboxylate function (ăCOOă) accepts a proton and gets converted to
carboxyl substituent (ăCOOH) while in basic solution the

BIOMOLECULES
QUIZRR 17

O O
ă
R · CH · C · OH R · CH · C · O
+
:NH2 NH3

+
ammonium substituent ( N H3 ) changes to amino group(-NH2) by losing a proton.

O O O
ă +
ă OH ă H
R · CH · C · O R · CH · C · O R · CH · C · OH
+ +
NH2 NH3 NH3
Negative ion Zwitter ion Positive ion

In acidic solution, an amino acid exists as a positive ion and migrates towards the cathode
in an electric field, while in alkaline solution it exists as a negative ion and migrates towards
anode. At a certain hydrogen ion concentration (pH),the dipolar ion exists as a neutral ion
and does not migrate to either electrode. This pH is known as the isoelectric point of the
amino acid.
If the amino acid has amino group and one carboxyl group, it has two pK values. The
isoelectric point of this amino acid has the average value of the both pK values.
We take example of glycine.

H3 N·CH2 · COOH 



 + ă
 H3 N · CH2 · COO + H
+
...(1)
Conjugated acid (CA) Dipolar Ion(DI)

[DI] [H+ ]
At equilibrium K1 =
[CA]

H3 N · CH2 · COO 



 
 H2 N · CH2 · COO  H

...(2)
DI Conjugated Base (CB)

[CB] [H+ ]
At equilibrium K2 =
[DI]

[DI][H+ ]
[CA] =
K1

BIOMOLECULES
18 QUIZRR

K 2 [DI]
[CB] =
[H+ ]

At isoelectric point [CA] = [CB]

[D I] [H+i ] K 2 [DI]
 Where [H+i ]  conc. of [H+ ] at isoelectric point
K1 [Hi ]

or, [H+i ]2  K1 K 2

or, 2 log [H+i ]  log K1  log K 2

or 2 log (H+i ]   log K1  log K 2

or 2pHi = pK1 + pK2

pK1  pK2
or pHi =
2

The acid base properties also become understandable when it is realized that the measured Ka

actually refers to the acidity of an ammonium ion, RNH+3

+ 
H3 NCHRCOO ă  H2O 

 +
 H3O  H2 NCHRCOO

[H3O+ ][H 2NCHRCOO ]


Ka 
[+ H3 N  CH  RCOO ]

and Kb actually refers to the basicity of a carboxylate ion, RCOOă


+  
H3 N·CHR·COO ă  H2O 

 H3 N · CHR·COOH  OH

[3 H3NăCHRăCOOH][OH ]
Kb 
[+ H3 NCHRCOOă ]

When the solution of an amino acid is made alkaline, the dipolar ion(I) is converted to the anion
(II); the stronger base, hydroxide ion, removes a proton from the ammonium ion and displaces the
weaker base, the amine.
+
H3 N·CHRCOO ă  OHă 

 ă
 H2 NCHRCOO  H2O
(I) (II)
Stronger Stronger Weaker Weaker
acid base acid base

BIOMOLECULES
QUIZRR 19

When the solution of an amino acid is made acidic; the dipolar ion I is converted into the cation
(III); the stronger acid H3O+, gives up a proton to the carboxylate ion, and displaces the weaker
carboxylic acid.
+ 
H3 N CHRCOO ă  H3O+ 

 H3 NCHRCOOH  H2O
(I) (II)
Stronger Stronger Weaker Weaker
base acid base acid

In summary, the acidic group of a simple amino acid like glycine is NH3 not ăCOOH, and basic
group is ăCOOă not ăNH2.

Example 2

What are essential and nonessential amino acids? Give the two examples of each.
Solution :
-Amino acids which are needed for health and growth of human beings but are not synthesized
by the human body are called essential amino acids. For example, valine, leucine, phenylalanine
etc. On the other hand, ăamino acids which are needed for health and growth of human beings
and are synthesized by the human body are called nonessential amino acids. For example,
glycine, alanine, aspartic acid etc.

ELECTROPHORESIS
If a filter paper-strip moistened with a solution of a mixture of AAÊs is placed between two
electrodes, the charged molecule will migrate to one electrode or the other at a rate that depends
on its net charge and the applied voltage. The net charge depends on the pH. The strip is then
stained with a reagent that reacts with the amino acid, thereby forming a color whose position
on the strip is compared for identification with that of a known sample. This process known as
electrophoresis is used for identification of amino acids.

SYNTHESIS OF AMINO ACIDS

1. AMINATION OF -HALO ACIDS

Amination of ăhalo acids of the many methods that have been developed for synthesizing amino
acids, we shall take up only one: amination of ăhalo acids. Considered in its various modifications,
this method is probably the most generally useful, although, like any of the methods, it cannot
be applied to the synthesis of all the amino acids.
Sometimes an ăchloro or ăbromo acid is subjected to direct ammonolysis with a large excess of
concentrated aqueous ammonia. For example,

BIOMOLECULES
20 QUIZRR
Br2 , P NH3 (excess) ă
CH3CH2COOH CH3CHCOOH CH3CHCOO

Br NH3
ăBromopropionic acid Alanine

The necessary ăhalo acids or esters can be prepared by the HellăVolhardăZelinsky halogenation
of the unsubstituted acids, or by a modification of the malonic ester synthesis, the usual route to
the unsubstituted acids. For example,

2. FROM DIETHYL MALONATE

COOC2H5 COOC2H5 COOH


C6 H5CH2Cl KOH HCl
CH2 HC · CH2C6H5 H · C · CH2C6H5
heat
COOC2H5 COOC2H5 COOH
Sodiomalonic ester Ethyl benzylmalonate Benzylmalonic acid

Br2, ether/base
reflux

COOH
NH3 (excess) heat
C6H5CH2CHCOOH C6H5CH2CHCOOH Br · C · CH2C6H5
ăCO2
+
NH3 Br COOH
Phenylalanine
35% overall yield

3. GABRIEL PHTHALIMIDE SYNTHESIS

O O

C C
ă + ăKCl
N K + Cl · CH2COOC2H5 N · CH2COOC2H5
Ethyl chloroacetate
C C

O O
Potassium
phthalimide HCl, H2O

ă+
Cl H3N · CH2COOH + phthalic acid
Glycine hydrochloride

BIOMOLECULES
QUIZRR 21

4. REDUCTIVE AMINATION

Another method of preparing ăamino acids is reductive amination of ăketo acids.


For example

O OH NH3
H2 /Pt NH3 ă
Me2CH · C · C · OH Me2CH · CH · C · OH 
Me2CH · CH · CO2
Valine
O O

REACTIONS OF AMINO ACID

(1) REACTIONS OF THE CARBOXYL GROUP


Ć They form salts with base

RCHCOO + NaOH RCHCONa + H2O

NH3 NH2

Ć They form ester with alcohol

C2H5OH/HCl AgOH
RCHCOO RCHCOOC2H5 RCHCOOC2H5

NH3 NH3Cl NH2

AgOH is used to obtain free ester.

Ć It can be decarboxylated to get amine

Ba(OH)2
CH3CHCOOH CH3CH2NH2 + BaCO3 + H2O

NH2

O
LiAlH4
Ć RCHCOH RCHCH2OH
reduction

NH2 NH2

BIOMOLECULES
22 QUIZRR
(2) REACTIONS OF THE AMINO GROUP

Acylation can also be done using acetic anhydride, CH3 CO 2 O.

Ć HNO2 (NaNO2 +dil.HCI) converts ăNH2 group into ăOH with liberation of N2.

O O

CH2CO + HNO2 CH2COH + N2 + H2O

NH3 OH
glycollic acid

O O

CH3CHCO + HNO2 CH3CHCOH + N2 + H2O

NH3 OH
alanine lactic acid

This reactions forms the basis of the Van Slyke method for the estimation of amino acids
in which volume of N2 gas collected is measured quantitatively.

(3) REACTIONS INVOLVING BOTH THE CARBOXYL AND AMINO GROUPS


Ć All -amino acids (with primary ăNH2 group) react with ninhydrin to form an intense
purple coloured complex.

BIOMOLECULES
QUIZRR 23

Ć Cu2+ salts form blue coloured complex with amino acids which is a bidentate legand.

O O
2+ 
C·O NH2 ă CH2
2 CH2COH + Cu Cu +
+ 2H
CH2ăNH2 O·C
NH2 cupric glycinate
O
(deep blue)

Ć Effect of Heat :
Ć -amino acids undergo intermolecular dehydration on heating at about 200ĈC to give
diketopiperazines.

NH · H HO NH
CH2 C O CH2 C O

+2H2O
O C CH2 O C CH2
OH H · NH NH
glycine 2,5 ă diketopiperazine

Ć -amino acids undergo intramolecular deamination on heating to form -unsaturated


acids.
O O
 

CH2 CHCOH CH2 CHCOH + NH3
acrylic acid
NH2H

Ć -amino acids and -amino acids undergo intramolecular dehydration to form cyclic
amides called Lactams.

CH2 · CH2 CH2 · CH2


O 
+ H2O
CH2 C CH2 C
NHăH OH NH O
ăamino butyric acid ăbutyrolactam

O
CH2 CH2 O
CH2 C · OH  CH2 C
H
CH2 N CH2 NH
H
CH2 CH2
ăamino acid ălactam

Ć In case of -amino acid, intramolecular cyclisation would give a seven-membered ring,


which is formed with difficulty. Hence, there is intermolecular polymerisation forming
nylon-6.
BIOMOLECULES
24 QUIZRR

nH2N(CH2)5COOH · N(CH2)5 C · NH(CH2)5 C · NH(CH2)5C ·

H O O O
monomer unit
Nylonă6

P EPTIDES

As the amino acid molecules contain both basic as well as acidic group it might be expected that
an intermolecular reaction may take place between the carboxyl group of one amino acid and the
amino group of another amino acid, with the elimination of a molecule of water.

R R´ R

NH COOH
H2N COOH + H2N COOH H2N CO

Since the resulting milecule still has a free amino and a carboxyl group, it may react with other
amino acids at either of the ends to give a higher molecular weight linear or condensation product.
Every two amino acids are linked by means of a ăCO-NH group, which is commonly referred as
peptide bond. So now we can define peptides a s the a mides formed by intera ction between
a mino groups a nd ca rboxyl groups of a mino a cids.
Depending upon the number of amino acid residues per molecule, they are known as depeptides,
tripeptides and so on and finally polypeptides.
Ć Proteins are polypepties in which no. of repeating unit of peptide is very large. The peptide
units are arrange in a uneque fashion in a partiular protein.

PROTEINS

Proteins are categorized according to (a) shape and (b) their biological function. Proteins according
to shape are further classified as globular, somewhat spherical and fibrous, long fibres or planar
sheets.
According to their biological action, they are classified as enzymes, hormones, antibodies, etc.
A protein has, secondary, tertiary and quaternary structures.
The primary structure is simply the amino acid sequence of the peptide chain. The secondary
structure is a result of the different conformations that the chain can take. The tertiary structure
is determined by any folding of the chain in on itself. A quaternary structure results when two
or more peptide chains in some proteins are linked together by weak forces of attraction of their
surface groups. Such proteins are called oligomers (dimers, trimers etc.).

BIOMOLECULES
QUIZRR 25

The Hăbonding in secondary structure exists between an NăH of one amino acid residue and the
O=C of another properly situated amino acid residue. There are 3 types of secondary structures
for protein. (1) The peptide sequence is coiled into a rightăhanded spiral in the -helix, with the
R groups positioned on the outside of the spiral. Each amide HăN bonds to the O=C on the next
turn of the coil, four residues away by Hăbonds, stabilizing this arrangement. (2) In the pleated
sheet of -structure, the peptide chains lie sideăbyăside in an open structure, with interăchain
amide Hăbonding holding the chains together. Parallel pleated sheets have chains running in the
same direction, all with their Năterminal residues starting at the same end. Antiparallel pleated
sheets have their chains running in opposite directions. The CÊs rotate slightly out of the plane
of the sheet to minimize repulsions between their bulky R groups, giving rise to the crimps or
pleats. In both cases, the R groups alternate positions above and below the sheet. (3) The random
coil structure has no repeating geometric pattern; encompassed within it are sequences in a helical
conformation, a pleated sheet conformation and regions that appear to have no discernible repeating
structure, but are actually not random conformations.
The unique threeădimensional shape of a protein in tertiary structure is the result of the
intramolecular forces of attraction that cause bending and coiling in the helical coil. These forces
are a function of the nature of the amino acid side chains within the molecule. Globular proteins
have their nonăpolar R groups pointing to the interior (the hydrophobic or nonăaqueous region)
and their polar side chains projecting toward the aqueous environment, somewhat like a micelle.
They are slightly water soluble. Fibrous proteins are insoluble in water. Their polypeptide chains
are held together by interăchain Hăbonds. The following are the attractive forces responsible for
the tertiary structure :

(1) Ionic: bonding between COOă and NH +3 at different sites.

(2) Hăbonding: mainly between sideăchain NH2 and COOH, also involving OHÊs (of serine, for
example) and the NăH of tryptophan.

(3) Weakly hydrophobic van der WaalÊs attractive forces engendered by sideăchain R groups
and

(4) Disulfide cross linkages between loops of the polypeptide chain.

The same kind of attractive and repulsive forces responsible for the tertiary structure operate to
hold together and stabilize the subunits of the quaternary structure.
Protein found in living system with definite configuration and biological activity is termed as
native protein. If a native protein is subjected to physical or chemical treatment, which may
disrupt its higher structures (conformations) without affecting its primary structure, the protein
is said to be denatured. During denaturation, the protein molecule uncoils from an ordered and
specific conformation into a more random conformation leading to precipitation. Thus denaturation
leads to increase in entropy and loss of biological activity of the protein. The denaturation may
be reversible or reversible. Thus, the coagulation of egg white on boiling of egg protein is an
example of irreversible protein denaturation. However, in certain cases it is found that if the

BIOMOLECULES
26 QUIZRR
disruptive agent is removed the protein recovers its original physical and chemical properties and
biological activity the reverse of denaturation is known as renaturation.

T est for Proteins

(i) Biuret test : On adding a dilute solution of CuSO4 to alkaline solution of protein, a violet
colour is developed.

O H

This test is due to the presence of peptide · C · N · linkage.

(ii) MillonÊs test : MillonÊs reagent consist of mercury dissolved in nitric acid (forming a
mixture or mercuric and mercurous nitrates).
When MillonÊs reagent is added to a protein, a white precipitate forms, which turn brick red
on heating.
This test is given by protein which yield tyrosine on hydrolysis (due to HCI of phenolic
group).

(iii) Ninhydrin test: Given by all proteins. When protein is boiled with a dilute solution of
ninhydrin, a violet colour is produced.
ă
O O O

C OH H O C C
C + R · C · C · OH N N·C
C OH NH2 C C
O O O
(Ninhydrin) Violet Complex

Example 3

Treatment of (R)ăMeCH(OH)CCl3, first with alkaline NaN3 and then reducing the product

with H2/Pd yields (S)ăalanine, CH 3 CH( N H 3 )CO2 . Explain.

Solution :
In alkaline solution, the OH will be converted to Oă which displaces Cl of adacent carbon to form
an oxirane ring. In the oxirane ring, the configuration of chiral carbon is still (R). The oxirane
ring is then opened by nucleophilic attack of azide ion, which occurs with inversion of configuration
i.e. configuration becomes (S). The acid chloride will be hydrolysed in alkaline conditions to give
acid. This is followed by reduction of N3 group to give (S)ăalanine.

BIOMOLECULES
QUIZRR 27
ă
OH O Me O Cl
ă
OHă ăCl
Me · C · CCl3 Me · C · CCl2 C·C
(R) Cl
Cl ă
N3

Me Me Me Oă
ă ă
ă OH ăCl
N3 · C · C · O N3 · C · C · Cl N3 · C · C · Cl

O O Cl

H2/Pd, ă N2

Me

H3N · C · CO2ă

(S) ă alanine

Example 4

Pick out incorrect statement :


(A) In an electrolysis experiment, -amino acids migrate at the isoelectric point point
towards electrodes.
(B) p-aminobenzenesulphonic acid as a dipolar ion; while p-aminobenzoic acid does not
(C) Sulphanillic acid is soluble in base, but not in acid

(D) H3 NCH2COOH (pKa = 2.4) is more acidic than RCH2COOH (pKa = 4 ă 5)
Solution :
(A) The pH at which [Anion] = [Cation] is called isoelectric point. At isoelectric point, -amino
acids donot migrate when electric field is applied.
(B) ăSO3H is strongly acidic and donates H+ to weakly basic arylamino group. ArCOOH is not
acidic enough to transfer H+ to the arylamino group.
 
(C) in p ă H3 NC6H4SO3, H3N is acidic anough to transfer H+ to bases to give the soluble
anion, p-H2NăC6H4SO3, ăSO3 is too feebly basic and cannot accept H+ from acids.

(D)  ăH3 N group increases acidity, because of its electron ă withdrawing inductive effect.

Example 5

Show the fundamental unit of structure common to all polypeptides and proteins and show
how cross linking occurs between two chains by H ăbonding.

BIOMOLECULES
28 QUIZRR
O H R

N
N
O
R H

Solution : R H O

N
N N

H O R

Example 6

Glycine exists as (H3N+CH2COOă) while anthranilic acid (PăNH2ăC6H4 ă COOH) does not
exist as dipolar ion.
Solution :
ăCOOH is too weakly acidic to transfer H+ to the weakly basic ăNH2 attached to the electron
withdrawing benzene ring. When attached to an aliphatic carbon, the ăNH2 is sufficiently basic
to accept H+ form ăCOOH group.

Example 7

(i) Sulphanilic acid although has acidic as well as basic group, it is soluble in alkali but
insoluble in mineral acid
(i) Sulphanilic acid is not soluble in organic solvents.
Solution :
(i) Sulphanilic acid exist as Zwitterion

+
HO3S NH2 O3S NH3

The weakly acidic ă+NH3 transfers H+ to OHă to form a soluble salt, PăNH2ăC6H4ăSO 3ă Na +

on the other hand SO3 is too weakly basic to accept H+ from strong acids.
(ii) Due to its ionic character it is insoluble in organic solvents.

Example 8

The pKa of the carboxyl group in an amino acid valine, (CH3)2CHCH(NH2)(COOH) is 2.31
and the pKa for the amino group of the same amino acid is 9.69. Compute the isoelectric
point (pI) for valine and draw the structure of this amino acid when the pH of the solution
equals to pI. Also draw the structures of valine that predominate at pH = 2 AND pOH = 2.

BIOMOLECULES
QUIZRR 29

Solution:
The isoelectric point (pI) is the pH at which the amino acid exists only as a dipolar ion with net
charge zero.

( pKa1  pKa2
At isoelectric point, for a neutral amino acid, pI 
2

The dissociation of cationic form of valine can be represented as


ă ă
CO2H CO2 CO2
+ +
CH N H3 (pKa1) CH N H3 (pKa2) CHNH2
ă ă
OH OH
CH(CH3)2 H
+ CH(CH3)2 H
+ CH(CH3)2

Net charge (+1) (0) (ă1)

The species with zero net charge exists between species with (+1) and (ă1) net charges.

(pK a1  pKa2 ) 9.69  2.31


pI   6
2 2

ă
CO2
+
CH N H3
When the pH of the solution equals to pI, the structure of valine is
CH(CH3)2

CO2H
+
When the pH of the solution is two, the structure of valine is CH N H3

CH(CH3)2

ă
CO2

When the pH of the solution is 12, the structure of valine is CHNH2

CH(CH3)2

BIOMOLECULES
30 QUIZRR

POLYMERS

1 .1 Polymers

The word ÂpolymerÊ is coined from two Greek words : poly means many and mer means unit or
part. The term polymer is defined as very large molecules having high molecular mass. These are
also referred to as macromolecules

1.2 Monomers

The repeating structural units are derived from some simple and reactive molecules known as
monomers. They are linked to each other by covalent bonds.

1.3 Polymerisation

This process of formation of polymers from respective monomers is called polymerisation.


Examples :

Polymerisation
nCH2 = CH2 n·CH2 · CH2 · ·CH2 · CH2 ·n
Ethene Repeating unit Polythene polymer

H H O O
Polymerisation
nNH2 (CH2)6 NH2 + nHOOC (CH2)4 COOH · N · (CH2)6 · N · C · (CH2)4 · C ·
n
Hexamethylene Adipic acid Nylon 6.6
diamine

Classification Of Polymers

There are several ways of classification of polymers based on some special considerations. The
following are some of the common classifications of polymers:
(I) Classification based upon source
Under this type of classification, there are three sub categories.
1. Natura l polymers
These polymers are found in plants and animals. Examples are proteins, cellulose, starch,
resins and rubber.
2. Semi-synthetic polymers
Cellulose derivatives as cellulose acetate (rayon) and cellulose nitrate, etc. are the usual
examples of this sub category.
3. Synthetic polymers
A variety of synthetic polymers as plastic (polythene), synthetic fibres (nylon 6,6) and
synthetic rubbers (Buna - S) are examples of manmade polymers extensively used in daily
life as well as in industry.

POLYMERS
QUIZRR 31

(II) Classification based upon structure


There are three different types based on the structure of the polymers.
(i) Linea r polymers
These polymers consist of long and straight chains. The examples are high density polythene,
polyvinyl chloride, etc. These are represented as :

(ii) Branched cha in polymers


These polymers contain linear chains having some branches, e.g., low density polythene.
These are depicted as follows :

Examples are glycogen, low density polyethylene, etc.

(iii) Cross linked or Network polymers


These are usually formed from bi-functional and tri-functional monomers and contain strong
covalent bonds between various linear polymer chains, e.g. bakelite, melamine, etc. These
polymers are depicted as follows :

(III) Classification based upon synthesis


(i) Addition polymers :
monomer molecules possessing double or triple bonds, e.g., the formation of polythene from
ethene and polypropene from propene. However, the addition polymers formed by the
polymerisation of a single monomeric species are known as homopolymers, e.g., polythene.

POLYMERS
32 QUIZRR

nCH2  CH2   (CH2  CH2 ) n ă Homopolymer


Ethene Polythene

The polymers made by addition polymerisation from two different monomers are termed as
copolymers, e.g., Buna-S, Buna-N, etc.

C6 H5
|
nCH2  CH  CH  CH2  n C6 H5 CH  CH2   (CH 2  CH  CH  CH2 ăCH2 ă C H)ăn
1, 3-Butadiene Styrene Butadiene-styrene copolymer
(Buna-S)

(ii) Condensation polymers


The condensation polymers are formed by repeated condensation reaction between two
different bi-functional or tri-functional monomeric units. In these polymerisation reactions,
the elimination of small molecules such as water, alcohol, hydrogen chloride, etc. take place.
The examples are terylene (dacron), nylon 6,6, nylon 6, etc. For example, nylon 6,6 is
formed by the condensation of hexamethylene diamine with adipic acid.

C6 H5
|
nCH2  CH  CH  CH2  n C6 H5 CH  CH2   (CH 2  CH  CH  CH2 ăCH2 ă C H) ăn
1, 3-Butadiene Styrene Butadiene-styrene copolymer
(Buna-S)

(IV) Classification based on inter-molecular forces (secondary forces)


1. Elastomers
Elastomers are polymers in which the polymer chains are held by weakest intermolecular
forces. These forces permit the polymers to be stretched. A few cross links are introduced
between the chains to help the polymer retract to its original position after the force is
released.
Example : Vulcanized rubber
2. Fibers
These polymers possess high tensile strength and high modulus, because of strong
intermolecular forces like hydrogen bonding which operate in polyamides. These strong
forces also lead to close packing of chains and thus impart crystalline nature. As a result
these polymers show sharp melting points. These polymers are used for making fibers.
Examples : Nylon and terylene

POLYMERS
QUIZRR 33

3. Thermoplastics
In thermoplastics, the intermolecular forces are intermediate between elastomers and fibers
and the polymer chain has no cross-links.
Thus thermoplastics can be moulded on heating. These polymers have no cross-linking
between chains.
Examples : Polyethylene, polystyrene etc.
4. Thermosetting polymers
Polymers which become hard on heating are called thermo setting polymers. Thermo setting
polymers can be heated only once when it permanently sets into a solid, which cannot be
remelted by heating.
Thermosetting polymers made from relatively low molecular mass semi-fluid polymers which
when heated in a mould forms an insoluble hard mass which is infusible. This is due to
extensive crosslinks between the different polymer chains forming three dimensional network
of bonds.
Example : Bakelite and melamine

Polymerization, the joining together of many small molecules to form very large molecules. The
simple compounds from which polymers are made are called monomers.
Polymers are formed in two general ways.
(a) In chain-reaction polymerization, there is a series of reactions each of which consumes
a reactive particle and produces another, similar particle; each individual reaction thus
depends upon the previous one. The reactive particles can be free radicals, cations, or
anions. A typical example is the polymerization of ethylene. Here the chain-carrying particles
are free radicals, each of which adds to a monomer molecule to form a new, bigger free
radical.

CH CH
Rad.+CH2  CH2  Rad CH2CH2 . 
2 2
 RadCH2CH 2CH2 CH2 .  etc.

(b) In step reaction polymerization, there is a series of reactions each of which is essentially
independent of the preceding one; a plymer is formed simply because the monomer happens
to undergo reaction at more than one functional group. A diol, for example, reacts with a
dicarboxylic acid to form an ester; but each moiety of the simple ester still contains a group
that can react to generate another ester linkage and hence a larger molecule, which itself
can react further, and so on.

O O

HO +
OH
HO OH
ethylene glycol terephthalic acid

POLYMERS
34 QUIZRR

OH

O O

O OH

HOCH2CH2OH

OH

OH
O O

O O
OH
p- C6 H4(COOH)2
O O

O O
HO O

O O

FREE RADICAL VINYL POLYMERIZATION

CH2 CH2 CH2 CH2


nCH2 initiator
CH CH CH CH
vinyl monomer G
G G G G

or ·CH2ăCH·
n
G
polymer

at the doubly bonded carbons · the vinyl groups · and is called vinyl polymerization. A wide
variety of unsaturated monomers may be used, to yield polymers with different pendant groups
(G) attached to the polymer backbone. For example :

POLYMERS
QUIZRR 35

CH2 CH2 CH2


H2C
CH CH CH
Cl
vinyl chloride
Cl Cl Cl
Poly (vinyl chloride) (PVC)

CH2 CH2 CH2


H2C
CH CH CH
CN
acrylonitrile
CN CN CN
Polyacrylonitrile (Orion)

CH2 CH2 CH2


H2C CH
Styrene
CH CH CH

C6H5 C6H5 C6H5 C6H5


Polystyrene

CH3 CH3 CH3 CH3


H2C CH2 CH2 CH2
C C C
O
O COOCH3 COOCH3 COOCH3
Poly(methyl methacrylate)
CH3 (Plexiglas, Lucite)
Methyl methacrylate

Polymerzation involves addition of free radicals to the double bond of the monomer: addition, first,
of the free radicals generated from the initiator, and then of the growing polymer molecule. This
is, of course, an example of chain-reaction polymerization.

1. Peroxide Rad
2. Rad. + H2C CH
Rad CH2 CHĆ
Chain · initiating steps
G
G

CH2 · CH Ć

3. Rad CH2 CHĆ+ H2C CH Rad CH2 · HC Chain propagating steps

G G G

then steps like (3) repeated, until finally :

POLYMERS
36 QUIZRR
4. 2Rad(CH2CH)n CH2CHĆ

G G
combination

Rad(CH2CH)n CH2CH · CHCH2(CHCH2)n Rad

G G G G

or Chain terminating steps

5. 2Rad(CH2CH)n CH2CHĆ

G G
disproportionation

Rad(CH2CH)n CH2CH2 + Rad(CH2CH)n CH = CH

G G G G

In each step the consumption of a free radical is accompanied by the formation of a new, bigger
free radical. Eventually, the reaction chain is terminated by steps that consume but do not form
free radicals; combination or disproportionation of two free radicals.
Copolymerization: Polymerization of a single monomeric compound to form a homopolymer is
known as homopolymerization. Evidently such a polymer is made up of identical units. In contrast,
a copolymer results when two different kinds of monomers are polymerized together to give a
product containing both the monomers. Such as process, known as copolymerization, is of great
industrial importance as it enables us to get a polymer possessing desired properties. To a certain
extent we can control the composition of a copolymer by varying the proportions of the monomer
in the polymerizing mixture. However, the composition of the polymer depends not only on the
relative concentrations of the two monomers, but also on the relative reactivities of the monomers
toward free radical addition. In other words, a more reactive monomer has greater chance of being
incorporated into a copolymer. As the reactivity of a carbon-carbon double bond toward free
radical addition is affected by the stability of the new free radical being formed, the more stable
free radicals will preferentially be produced.
Copolymerization of styrene with methyl methacrylate to produce polystyrene (co-methyl
methacrylate) (A) is an industrially important process.

CH3
CH3 CH2 CH2
CH C
H2C CH + H2C C
C6H5
C O O O
C6H5
H3C · O
CH3
n
(A)

POLYMERS
QUIZRR 37

Saran Wrap (B) is another copolymer formed by the copolymerization of vinylidene and chlorides.

Cl Cl
H2C C + H2C CH
CH2 ă C · CH2
Cl Cl
Cl CH ·
n
Cl
(B)

I ONIC POLYMERIZATION: LIVING POLYMERS

Chain-reaction polymerization can proceed with ions instead of free radicals as the chain-carrying
particles : either cations or anions, depending on the kind of initiator that is used.
Cationic Polymerization

+
Y H2C CH Y : H2C · CH
An acid
G A carbocation G
G

CH2 · CH
+ +
Y : H2C · CH H2C CH Y : H2C · HC etc.
A carbocation G
G G

Anionic polymerization

ă
Z: H2C CH Z : H3C · CH:
A base
G G
A carbanion

CH2 · CH:
ă ă
Z : H2C · CH: H2C CH Z : H2C · HC etc.
G
G G
A carbanion

Cationic Polymerization is initiated by acids. Isobutylene, for example, undergoes cationic


polymerization to a tacky material used in adhesives. Copolymerization with a little isoprene gives

POLYMERS
38 QUIZRR
butyl rubber, used to make automobile inner tubes and tire liners. A variety of acids can be used:
sulfuric acid; AlCl3 or BF3 plus a trace of water.

CH3 CH3
CH3 CH3 CH3
H+ H2C H2C H2C H3C · C · CH2 · C · CH2
CH3 CH3 CH3
CH3 CH3

Anionic polymerization, as we might expect, is initiated by bases: Li+, NH 2 , for example, or


organometallic compounds like n-butyllithium. For example :

CH2 CH2
ă +
NH2ă K+ + H2C CH H2N CH2 = CHPh H2N CH CH K etc.

Ph Ph Ph
Styrene + ă
K HC
Ph

H3C
CH3 ă +
O C Li
n ă BuLi + H2C H3C etc.
O O năBu
O
CH3
methyl methacrylate

Active metals like Na or Li can be used: here the initiation becomes a little more complicated, as
in the polymerization of styrene by the action of sodium metal and naphthalene. A sodium atom
transfers an electron (1) to naphthalene to form a radical-anion:
+ ă
1. Na + naphthalene Na naphthalene .
Naphthalene radicalăanion

ă
2. naphthalene. + HC CH2 naphthalene + CH CH2 .

Ph Ph
styrene radicalăanion Formed by
oneăelectron transfer

ă
CH CH2 Ph
3. 2 HC CH2ă. Ph CH2 CH
ă

Ph A dianion

POLYMERS
QUIZRR 39

The radical-anion then donates the electron to styrene (2) to form the styrene radical-anion. Like
many other free radicals, these dimerize (3). The resulting dianion is the true initiator, and begins
to grow at both ends :

ă
CH CH2 Ph
Ph CH2 CH
ă
CH2=CHPh

Ph Ph
ă
CH CH CH CH2 CH2 CH2 Ph
Ph CH2 CH2 CH2 CH CH CH
ă

Anionic polymerization is not limited to the vinyl kind, involving addition to carbon-carbon double
bonds. Ethylene oxide, for example, is converted by a small amount of base into a high molecular
weight polyether.

O
ă ă
CH3O + H3C O
O

ă
O O O
ă + H3C O
H3C O
O

Some important polymers

(a) Polythene :
There are two types of polythene as given below:
(i) Low density polythene :
Ć It is obtained by the polymerisation of ethene under high pressure of 1000 to 2000
atmoshpheres at a temperature of 350 K to 570 K in the presence of traces of dioxygen
or a peroxide initiator (catalyst).
Ć The low density polythene (LDP) obtained through the free radical addition and H-atom
obstraction has highly branched structure.
Ć Low density polythene is chemically inert and tough but flexible and a poor conductor
of electricity.
High density polythene : It is formed when addition polymerisation of ethene takes place
in a hydrocarbon solvent in the presence of a catalyst such as triethylaluminium and
titanium tetrachloride (Ziegler-Natta catalyst) at a temperature of 333 K to 343 K and under
a pressure of 6-7 atmospheres.
POLYMERS
40 QUIZRR
Ć High density polythene (HDP) thus produced, consists of linear of Ziegler-Natta catalyst.
molecules and has a high density due to close packing.
Ć It is also chemically inert and more tougher and harder.
(b) Polyacrylonitrile :
Ć The addition polymerisation of acrylonitrile in presence of a Acrylic fibres leads to the
formation of polyacrylonitrile.
Ć It has resistance to stains, chemicals, insects and fungi.
Ć Polyacrylonitrile is used as a substitute for wool in making commercial fibres as orlon or
acrilan

CN
Polymerisation
n CH2 CHCN Peroxide catalyst · CH2 · CH ·
n
Acrylonitrile Polyacrylonitrile

(c) Natural Rubber : Natural rubber is an addition polymer of isoprene (2-methyl-1,3-


butadiene). Rubber has an average chain length of 5000 monomer units of isoprene.
The rubber in which the arrangement of carbon chain is trans with respect to the double
bond is known as Gutta Percha and this is the natural rubber obtained from bark of
various trees. Natural rubber is sticky material. This disadvantage is removed by
ÂVULCANISATIONÊ which involves addition of sulphur to rubber and heating the mixture,
sulphur forms short chains of sulphur atoms that link hydrocarbon (isoprene) units together.

CH3 CH3

CH · C CH · CH2 · CH · C CH · CH2

S S

CH · C CH · CH2 · CH2 · C CH · CH · CH

CH3 CH3

When tension is applied the chains can strengthen out but they cannot slip over each other
because of sulphur bridges. Thus rubber can be stretched only to a certain extent and
hydrocarbon chains have the tendency to regain their shape when tension is removed.
Vulcanised rubber is thus stronger and less sticky than the natural rubber.
(d) Synthetic rubber : (Polychloroprene or Neoprene) it is obtained by free radical polymerisation
of chloroprene
Cl Cl

nCH2 C · CH CH2 (· CH2 · C CH · CH2 ·) n


Neoprene

POLYMERS
QUIZRR 41

it is a thermoplastic and need not to be vulcanised. It is a good general purpose rubber and
superior to natural rubber as it is resistant to the reaction of air, heat, light chemicals,
alkalis and acids below 50% strength. It is used for making transmission belts, printing rolls
and flexible tubing employed for conveyence of oil and petrol.

(e) Buna rubbers : Butadiene polymerises in the presence of sodium to give a rubber substitute
viz. Buna. It is of two types :
(i) Buna - N or GRA: it is synthetic rubber obtained by copolymerisation of one part of acryl
nitrile and two parts of butadiene.

nCH2 CH · CH CH2 + nCH2 CH · CH2 · CH CH · CH2 · CH2 · CH

CN BUNAăN CN n

It is more rigid, responds less to heat and very resistant to swelling action of petrol, oils
and other organic solvents.
(ii) Buna -S or GRS (General purpose Styrene rubber): It is a copolymer of three moles of
butadiene and one mole of styrene and is an elastomer. It is obtained as a result of free
radical copolymerisation of its monomers.

nC6H5CH CH2 + nCH2 CH · CH CH2 [ăCH2·CH CH·CH2·CH·CH2·]n

C6H5
BUNAăS

It is generally compounded with carbon black and vulcanised with sulphur. It is extremely
resistant to wear and tear and finds use in manufacture of tyres and other mechanical
rubber goods.
(e) Teflon : It is polymer of tetrafluroethylene (F2C=CF2) which on polymerization gives telfon.
(NH4) 2S2O8 870 · 1020K
nCF2 CF2 (· CF 2 · CF 2 ·) n
It is thermoplastic polymer with a high softening point (600K). It is very tough and difficult
to work. It is ipert to most chemicals except fluorine and molten alkali metals. It withstands
high temperatures. Its electrical properties make it an ideal insulating material for high
frequency installation.

Example 1

Which of the following is an elastomer?


(1) Vulcanized rubber (2) Dacron (3) Polystyrene
Solution :
Vulcanized rubber is an elastomer.

POLYMERS
42 QUIZRR
N YLON

Nylon is used as a general name for all synthetic fibres forming polyamides, i.e., having a protein
like structure. A number is usually suffixed with the name ÂnylonÊ which refers to the number of
carbon atoms present in the monomers.

NYLON–66

(read as nylon six, six)


It is obtained by the condensation polymerisation of hexamethylenediamine (a diamine with six
carbon atoms) and adipic acid (a dibasic acid having 6 carbon atoms).

nHOOC(CH2)4COOH + nH2N(CH2)6NH2 280Ĉ C · OC(CH2)4CONH(CH2)6NH ·n


High pressure Nylonă66
Adipic acid Hexamethylene
ă(n ă1)H2O
diamine

NYON–610

(read as nylon six, ten)


It is obtained by condensation polymerisation of hexamethylenediamine (six carbon atoms) and
sebacic acid (a dibasic acid with 10 carbon atoms).
Nylon fibres are stronger than natural fibres and so are used in making cords and ropes. The
fibres are elastic, light, very strong and flexible. They have drip dry property and retain creases.
It is inert towards chemicals and biological agents. It can be blended with wool. Nylon fibres are
used in making garments, carpets, fabrics, tyre cords, ropes, etc.

NYLON–6 (Perlon L)

A polyamide closely related to nylon is known as perlon L (Germany) or Nylonă6 (USA). It is


prepared by prolonged heating of caprolactum at 260ă270ĈC. It is formed by self condensation of
a large number of molecules of amino caproic acid. Since, caprolactum is more easily available,
it is used for polymerization, which is carried out in the presence of H2O that first hydrolyses the
lactam to amino acid. Subsequently, the amino group of the amino acid can react with the lactam
and the process goes on and onto form the polyamide polymer.
Carpolactam is obtained by Beckmann rearrangement of cyclohexanone oxime.

OH O NOH

[O] [O] NH2OH

Cyclohexane Cyclohexanol Cyclohexanone Cyclohexanone


oxime

POLYMERS
QUIZRR 43

NOH
O
H2SO4
Beckmann
O 260 ă 270Ĉ C
· C ·(CH2)5 · NH ·
rearrangement H2O
NH Nylonă6

Caprolactam (Six carbon atoms in the


repeating units)

N YLON–2–NYLON–6

It is an alternating polyamide copolymer of glycine and amino caproic acid and is biodegradable.

Example 2

Nylonă66 is obtained by condensation polymerization of


(a) Adipic acid and hexamethylene diamine
(b) Phenol and formaldehyde
(c) Terephthalic acid and ethylene glycol
(d) Sebacic acid and hexamethylene diamine
Solution :
(a) Adipic acid and hexamethylene diamine on condensation will generate Nylonă66

Example 3

Which of the following type of forces are present in Nylonă66?


(a) Van der WaalÊs forces of attraction
(b) Hydrogen bonding
(c) Threeădimensional network of bonds
(d) None of these
Solution :
(b) Hydrogen bonds are present in Nylonă66.

POLYETHYLENE

Polyethylene is of two types:


(a) Low Density Poly Ethylene (LDPE) : It is manufactured by heating ethylene at 200ĈC
under a pressure of 1500 atmoshperes and in the presence of traces of oxygen. This
polymerisation is a free radical polymerisation.

200Ĉ C
nCH2 CH2 · CH2 · CH2 ·n
1500 atmosphere

The polyethylene produced has a molecular mass of about 20,000 and has a branched
structure. Due to this, polyethylene has a low density (0.92) and low melting point (110ĈC).

POLYMERS
44 QUIZRR
That is why polyethylene prepared by free radical polymerisation is called low density
polyethylene. It is a transparent polymer of moderate and as insulation for electrical wires
and cables.

(b) High Density Poly Ethylene (HDPE) : It is prepared by the use of ZieglarăNatta catalyst
at 160Ĉ under pressure of 6 to 7 atmoshphere.
The polymer is linear chain, hence it has high density (0.97) and has high melting point
(130ĈC). That is why it is called high density polyethylene. It is a translucent polymer. It
has greater tough ness, hardness and tensile strength than low density polyethylene. It is
used in the manufacture of containers (buckets, tubes), house wares, bottles and toys.

M ELAMINE–FORMALDEHYDE RESIN

This resin is formed by condensation polymerisation of melamine and formaldehyde.

H2N N NH2 H2N N NHCH2OH ·H2N N NH·CH2·


+ HCHO
N N N N Polymerisation N N

NH2 NH2 NH2


n
(Intermediate) Melamineăformaldehyde polymer

It is a quite hard polymer and is used widely for making plastic crockery under the name
melamine. The articles made from this polymer do not break even when dropped from considerable
height.

BAKELITE

Phenolăformaldehyde resins are obtained by the reaction of phenol and formaldehyde in the
presence of either an acid or a basic catalyst. The reaction starts with the initial formation of ortho
and paraăhydroxymethyl phenol derivatives, which further react with phenol to form compounds
where rings are joined to each other with ăCH2 groups. The reaction involves the formation of
methylene bridges in ortho, para or both ortho and para positions. Linear or crossălinked materials
are obtained depending on the conditions of the reaction.

OH OH OH
CH2OH
+ ă
H or OH
+ HCHO +

(Intermediate)
CH2OH
Oăand păhydroxymethyl phenol

POLYMERS
QUIZRR 45

OH OH OH
CH2OH Polymerisation
CH2
n

Linear polymer n
Novolac

OH OH OH OH OH
CH2OH
Polymerisation
CH2 CH2 CH2 CH2·
n +n

CH2OH CH2 CH2

·CH2 CH2 CH2 CH2·

OH OH OH
Crossălinked polymer (Bakelite)

P OLYESTERS

Dacron is a common polyester, prepared using ethylene glycol and terephthalic acid. The reaction
is carried out at 140Ĉ to 180ĈC in the presence of zinc acetate and Sb2O3 as catalyst.

nHOCH2CH2OH + nHO2C CO2H · OCH2CH2 · C C·

Dacron
O O n

TABLE OF POLYMERS
S.N. Name of Polymer Structure Monomer Uses
1. Polythene An insulator,
· CH2 · CH2 · CH 2 CH2 anticorrosive,
n packing material,
household and
laboratory wares.
2. Polysyrene · CH · CH2 · HC CH2 An insulator,
wrapping material,
C6H5
C6H5 n manufacture of
toys and household
articles.
POLYMERS
46 QUIZRR
3. Polyvinylchloride (PVC) Cl In manufacture of
CH2 CHCl raincoats, hand
· CH2 · CH2 ·
n bags, vinyl
flooring and
leather clothes.
4. Polytetrafluoro ethylene CF 2 CF2 As lubricant,
· CF2 · CF2 ·
(PTFE) or Telflon n insulator and
making cooking
wares.
5. Polymethyl metha CH3 CH3 As subsitute of
acrylate (PMMA) or glass and making
· CH2 · C · H 2C CCOOCH3
Plexi glass decorative
COOCH3 n materials.

6. Polyacrylonitrile CN In making
(Orlon)(PAN) CH2 CHCN synthetic fibres
· CH2 · CH ·
n and synthetic wool.

7. Styrene butadiene (a) H2C CH · CH CH2 In making


rubber (SBR or (b) HC CH2 automobile tyres
BuNa-S) CH and footwear.
6 5

8. Nitrile rubber (Buna-N) (a) H2C CH · CH CH2 In making oil


(b) HC CH2 seals, manufacture
of hoses and tank
CN
linings.
9. Neoprene As insulator,
· CH2 · C CH · CH2 · H2C C · CH CH2
making conveyor
Cl Cl belts and printing
rollers.
10. Polyethyl acrylate In making films,
· CH2 · CH · CH2 CH · COOC2H5 house pipes and
COOC2H5 n finishing fabrics.

11. Terylene (Dacron) For making fibres,


(a) HOOC COOH
safety belts, tyre
cords, tents etc.
(b) HO · CH2 · CH2 · OH

POLYMERS
QUIZRR 47

12. Glyptal (a) HOOC COOH As binding


· OCH2.CH2OOC COO·
material in
n preparation of
mixed plastics and
(b) HO · CH2 · CH2 · OH
paints.

O H
13. Nylon 6 N O In making fibres,
C plastics, tyre cords
· NH · (CH2)5 · C · n
and ropes.

14. Nylon 66 (·NH(CH2)6NHCO(CH 2)4CO·) n (a) HOOC · (CH2)4 · COOH In making


(b) H2N · (CH2)6 · NH2 brushes, synthetic
fibres, parachutes,
ropes and carpets.

15. Bakelite OH OH (a) HCHO For making gears,


(b) C6H5OH protective coating
CH2 CH2·
and electrical
fittings.

CH3 CH3 n

16. Urea formaldehyde (a) HCHO For making un-


(· NH · CO · NH · CH3 ·) n (b) NH2CONH2
resin breakable cups
and laminated
sheets.

17. Melamine (a) H2N N NH2 In making plastic


· NH · NH N NH · CH2 ·
formaldehyde crockery,
resin N N unbreakable cups
N N
and plates.
NH2
NH2 n
(b) HCHO

18. Poly--hydroxy OH As packaging


butyrate-co-- · O · CH · CH2 · CO · orthopaedic
(a) H3C · CH · CH2 · COOH
hydroxy valerate devices and in
R O OH
(PHBV) n controlled drug
R CH3, C2H5
(b) H3C · CH2 · CH · CH2 · COOH release.

POLYMERS
48 QUIZRR
Example 4

A polymeric substance, tetrafluoroethylene, can be represented by the formula (C2F4) where


x is a large number. The material was prepared by polymerizing C2F4 in the presence of a
sulphur bearing catalyst that served as a nucleus upon which the polymer grew. The final
product was found to contain 0.012% S. What is the value of x if each polymeric molecule
contains 2 sulphur atoms? Assume that the catalyst contributes a negligible amount to the
total mass of polymer.
Solution :
Final product contain 0.012% S
If contains two S atoms

Molecular weight ï 0.012


  2  32
100

Weight of monomer, i.e. C2F4 is 100


 100 ï x = 5.33 ï 105
 x = 5.33 ï 103

Example 5

A
nCF2 = CF2 
heat, high pressure
 (ăCF ă CF ă) Teflon
2 2 n

Solution :
A = (NH4)2S2O8

Example 6

Oxidation NH2OH H2 SO4  


A B C H2 O
D E

Identify A to E.

O NOH H
Solution :
O
N
Oxidation NH2OH H2SO4

Caprolactam

H2O4
O

· NH · (CH2)5 · C · H2N · (CH2)5 · COOH


Nylon ă 6 Amino caproic acid

POLYMERS
QUIZRR 49

Example 7

nCH2 CH
K2 S2 O8
C CH2 B

Cl

Solution :

· CH2 · C CH · CH2 ·
B
Cl n
neoprene

Example 8

(a) Show how an aldohexose can be used to syntheisze 2-ketohexose. (b) Since glucose is
converted to fructose by this method, what can you say about the configurations of C3, C4
and C5 in the sugars.
Solution :
(a) CHO CH N.NHC6H5
2C6H5CHO
(CHOH)4 + 3C6H5NH.NH2 C N.NHC6H5
ăC6H5NH2 ă2C6H5CH = NNHC6H5
ăH2O
CH2OH ăNH3 (CHOH)3
Aldohexase
CH2OH
Osazone

H·C O CH2OH

C O C O
Zn/CH3COOH
(CHOH)3 (CHOH)3

CH2OH CH2OH
Osazone 2ăketohexose

Here aldohexose reacts with one molecule of phenylhyrazine which condenses with the
aldehyde group to give phenylhydrazone. When warmed with excess of phenyl hydrazine,
the secondary alcoholic group adjacent to the aldehyde group is oxidized by another molecule
of phenylhydrazine, to a ketonic group. With this ketonic group, the third molecule of
phenylhydrazine condenses to give osazone. The phenylhydrazinyl group is transferred
from osazone to C6H5CHO giving C6H5CH = N.NHC6H5 and a dicarbonyl compound called
an osone. The more reactive aldehyde group of the osone is reduced, not the less reactive
keto group and it gives the 2-ketohexose.

POLYMERS
50 QUIZRR
(b) The configurations of these carbons which are unchanged in the reactions, must be identical
in order to get the same osazone.

Example 9

Compound (A) C5H10O5, give a tetra-acetate with Ac2O and oxidation of (A) with Br2ăH2O
gives an acid, C5H10O6. Reduction of (A) with HI and red phosphorous gives 2-methylbutane.
What is structure of (A)?
Solution :
The formation of tetraacetate indicates presence of 4 OH groups and oxidation with bromine
water indicates presence of CHO group. Reduction with red phosphorous and HI indicates presence
of one carbon in the side chain. Thus, the structure of (A) would be
CHO

CHOH

HO · CH2 · C · OH

CH2OH

Example 10

Compound (A) C5H10O4, is oxidised by Br2ăH2O to the acid, C5H10O5, which readily forms a
lactone. (A) forms a triacetate with Ac2O and an osazone with PhNHNH2. (A) is oxidised by
HIO4, only one molecule of which is consumed. What is the structure of (A) ?
Solution :
(A) contains three hydroxyl groups and an aldehyde group. Formation of a lactone shows that one
hydroxyl group is in the  ă or -position with respect to the carboxyl group. Since (A) contains
four oxygen atoms and these have been accounted for as three hydroxyl groups and an aldehyde
group, the formation of an osazone shows the presence of the group ăCH(OH)CHO. Since only
one molecule of periodic acid is consumed, (A) contains only one set of adjacent oxidisable groups.
This must therefore be the CH(OH)CHO group and so the other CHOH groups must be ÂseparatedÊ
from each other and from the CHOH of CH(OH)CHO. Also, the absence of four hydroxyl groups
in a five-carbon chain suggests (A) is a deoxy-compound. this Âdeoxy-carbon atomÊ is therefore the
one that ÂseparatesÊ all the three CHOH groups. (A) structure which fits all the facts is

Example 12

(a) Supply structures of H through K. Given :

NH2OH/base Ac2O/NaOAc ăHOAC NaOMe/MeOH


An aldohexose H I J K

POLYMERS
QUIZRR 51

(b) Explain the last step (c). What is net structural change (d) Name this overall method.
(e) Discuss the possibility of epimer formation.
Solution :
(a) H is an oxime HOCH2(CHOH)4CH = NOH; I is the completely acetylated oxime,
AcOCH2 (CHOAc)4CH = NOAc that loses 1 mole of HOAc to form J, AcOCH2(CHOAc)4CN; K
is an aldopentose, HOCH2(CHOH)2CHO.
(b) The acetates undergo transesterification to give methyl acetate freeing all the sugar OHÊs. This
is followed by reversal of HCN addition.
(c) There is loss of one C from the carbon chain.
(d) Wohl degradation
(e) The -CHOH becomes the ăCH = O without any configurational changes of the other chiral
carbons. Thus no epimers are formed.

POLYMERS
AMINES
QUIZRR 75

AMINES

1. Amines

Amines are organic compounds and functional groups that contain a basic nitrogen atom with
a lone pair. Amines are derivatives of ammonia, wherein one or more hydrogen atoms have
replaced a substituent such as an alkyl or aryl group. Important amines include amino acids,
biogenic amines, trimethylamine and aniline.

1.1 Aliphatic Amines

As displayed in the images below, primary amines arise when one of three hydrogen atoms in
ammonia is replaced by an organic substituent. Secondary amines have two organic substituent
bound to N together with one H. In tertiary amines all three hydrogen atoms are replaced by
organic substituents.

(a) Primary amines

CH3 CH2 NH2 CH3 · CH · NH2


1-ethanamine |
CH3
1-methyl-1-ethanamine

(b) Secondary amines


CH3 CH2 NHCH3 NHCH3

N-methyl ethanamine
N-methyl cyclohexanamine
(C) Tertiary amines

CH2 CH2 CH2 · N · CH2CH3 (CH3)3N


|
CH3 N-N-dimethyl methanamine
N-ethyl-N-methyl-1-1propanamine

1.2 Aromatic amine


An aromatic amine is an amine with an aromatic substituent ă that is ăNH2, ăNHă or nitrogen
group(s) attached to an aromatic hydrocarbon, whose structure usually contains one or more
benzene rings. Aniline is an example.

NH2 NHCH(CH3)2

Benzenamine (Aniline) N-isopropyl benzenamine

AMINES
76 QUIZRR
1 .3 Physical properties

1.3.1 General properties

1. Hydrogen bonding significantly influences the properties of primary and secondary amines
as well as the protonated derivatives of all amines. Thus the boiling point of amines is higher
than those of the corresponding phosphines, but generally lower than those of the
corresponding alcohols. Alcohols, or alkanols, resemble amines but feature an OH group in
place of NR2. Since oxygen is more electronegative than nitrogen, RO-H is typically more
acidic than the related R2N-H compound.
2. Methyl, dimethyl, trimethyl and ethylamine are gases under standard conditions, whereas
diethylamine and triethylamine are liquids. Most other common alkyl amines are liquids;
high-molecular-weight amine are solids.
3. Most aliphatic amines display some solubility in water, reflecting their ability to form hydrogen
bonds. Solubility decreases with the increase in the number of carbon atoms, especially
when the carbon atom number is greater than 6.
4. The aromatic amines, such as aniline, have their lone pair electrons conjugated into the
benzene ring, thus their tendency to engage in hydrogen bonding is diminished. Otherwise
they display the following properties :
Ć Their boiling points are usually still high due to their larger size.
Ć Diminished solubility in water, although they retain their solubility in suitable organic
solvents only.
Ć They are toxic and are easily absorbed through the skin : thus hazardous.

R1 R1

R3 N N R3

R2 R2

1.3.2 Chirality

Tertiary amines of the type NHRR´ and NRR´R´´ are chiral : the nitrogen atom bears four distinct
substituents counting the lone pair. The interconversion of the stereoisomers has been compared
to the inversion of an open umbrella into a strong wind. Quaternary ammonium salts with four
distinct groups on the nitrogen are capable of exhibiting optical activity.

1.3.3 Properties as bases

Like ammonia, amines act as bases but are reasonably weak. The basicity of amines depends
on :

AMINES
QUIZRR 77

1. The electronic properties of the substituents (alkyl groups enhance the basicity, aryl
groups diminish it).
2. Steric hindrance offered by the groups on nitrogen.
3. The degree of solvation of the protonated amine.
The nitrogen atom features a lone electron pair that can bind H+ to form an ammonium ion
R3NH+. The water solubility of simple amines is largely due to hydrogen bonding between
protons on the water molecules and these lone electrons pairs.
1. Inductive effects of alkyl group : + I effect of alkyl groups raises the energy of the lone pair
of electrons, thus elevating the basicity. Thus the basicity of an amine may be expected to
increase with the number of alkyl groups on the amine. However, there is no strict trend
in this regard, as basicity is also governed by other factors. The increase in Kb from
methylamine to dimethylamine may be attributed to + I effect; however, there is a decrease
from dimethylamine to trimethyl amine due to the predominance of steric hindrance offered
by the three methyl groups to the approaching Lewis acid.
2. Mesomeric effect of aromatic systems : ă M effect of aromatic ring delocalises the lone pair
of electrons on nitrogen into the ring, resulting in decreased basicity. Substituents on the
aromatic ring, and their positions relative to the amine group may also considerably alter
basicity.
3. The degree of solvation of protonated amines : The degree of solvation of the protonated
amine depends on the approachability of solvent molecules. If the molecule is sterically
hindered (as in the case of trimethylamine), the protonated form is not well-solvated, thereby
reducing basicity. This also explains the oroder of basicity of the methyl amines.
The major determinant of basic strength in alkyl-substituted anilines remain mesomeric stabilisation
of the aniline molecule with respect to the cation; born out by the irregular effect of introducing
Me groups into the o-, m- and p-positions in aniline.
A group with a more powerful (electron-withdrawing) inductive effect, e.g. NO2 is found to have
rather more influence. Electron withdrawal is intensified when the nitro group is in the o- or p-
position, for the interaction of the unshared pair of the amino nitrogen with the delocalised 
orbital system of the benzene nucleus is then enhanced. The neutral molecule is thus stabilised
even further respect to the cation, resulting in further weakening as a base. Thus the nitro-
amines are found to have related pKa values.

+
NH2 NH2

PhNh2 NO2C6H4NH2
4.62 o ă 0.28
m ă 2.45
p ă 0.95
N N
ă + ă +
O O O Oă

AMINES
78 QUIZRR
The extra base-weakening effect, when the substituent is in the o-position, is due in part to the
short distance over which its inductive effect is operating, and also to direct interaction, both steric
and by hydrogen bonding with the NH2 group. o-Nitroaniline is such a weak base that its salts
are largely hydrolysed in aqueous solution, while 2, 4-dinitroaniline is insoluble in aqueous acids,
and 2, 4, 6-trinitroaniline resembles an amide; it readily undergoes hydrolysis to picric acid (2, 4,
6-trinitrophenol).
With substituents such as OH and OMe that have unshared electron pairs, an electron-donating,
i.e. base-strengthening, mesomeric effect can be exerted from the o- and p-, but not from the m-
position, with the result that the p-substituted aniline is a stronger base than the corresponding
m-compound. The m-compound is a weaker base than aniline itself, due to the electron-withdrawing
inductive effect exerted by the oxygen atom in each case. As so often, the effect of the o-
substituent remains somewhat anomalous, due to the interaction with the NH2 group by both
steric and polar effects.

NH2 NH2

OMe OMe
+

Example 1

Compare the basicities of


PhNH2 Ph2NH
NH2

I II III
Solution :
III > I > II
In structure III, lone pair electron of Nitrogen does not participate in resonance.

Example 2

Arrange the following compound according to their basic strength (with suitable
explanation) : C6H5NH2, C6H5NHMe, C6H5NMe2, o-MeC6H4NH2
Solution : C6H5NH 2 C6H5NHMe C6H5NMe2 o-MeC6H4NH2
pKa = 4.62 pKa = 4.84 pKa = 5.15 pKa = 4.38

AMINES
QUIZRR 79

The major determinant of basic strength in alkyl substituted anilines remains mesomeric stabilization
of aniline molecule (A) w.r.t. to the cation (B). Where (A) and (B) are
+ +
NH2 H NH2

(A) (B)

1 .4. Preparation of Amines

1.4.1 Alkylation of Ammonia

+
R ă X + 2NH3  R ă NH2 + N H4 X 

Ć Ammonia reacts as a nucleophile with alkyl halides to give primary amines in a nucleophilic
substitution reaction.
Ć Yields are often poor as the product, a primary amine, RNH2, is itself a nucleophile and can
react with more alkyl halide.
Ć The result are mixtures containing primary amines, secondary amines, tertiary amines and
quaternary ammonium salts.

R´CH2 X R´CH2X R´CH2 X + ă


RNH2 RNH(CH2R´) RN(CH2R´)2 RN(CH2R´)3X
primary secondary tertiary quaternary

Ć As aryl halides do not undergo simple nucleophilic substitution, they cannot be prepared
using this method.

1.4.2 Alkylation of Azide ion and Reduction

A much better method for preparing a primary amine from alkyl halide first to convert the alkyl
halide to an alkyl azide (RN3) by a nucleophilic substitution reaction with sodium azide (NaN3).

+ + ă
N N N N N N
Azide ion

ă + ă ă + ă ă
N N N + R·X N N N·R + X
Alkyl azide

The alkyl azide is then reduced to a primary amine with Na/C2H5OH or LiAlH4.
+ ă Na/C2H5OH
R·N N N R · NH2 + N2
or LiAlH4

AMINES
80 QUIZRR
1 .4.3 Gabriel Phthalimide Synthesis

Another method used for the preparation of primary aliphatic amines only is the Gabriel
Phthalimide synthesis. Phthalimide (pKa = 9) is quite acidic in nature. It can be converted to
potassium phthalimide by its reaction with KOH. The phthalimide anion is a strong nucleophile.
It reacts with alkyl halide, preferably methyl halide and primary alkyl halide only, by an SN2
mechanism to give Năalkylphthalimide. The secondary and tertiary alkyl halides are not employed
because they undergo elimination reactions also. The Năalkyl phthalimide is hydrolysed with
dilute HCl or KOH solution to give primary aliphatic amine.

O O O
COOK
KOH ă + RăX 2KOH
N·H N K N·R + RNH2
(1Ĉ only)
ăKX COOK

O O O
Năalkyl phthalimide

Năalkylphthalimide can also be converted to primary amine and phthalazineă1, 4ădione by


treating it with hydrazine.

O
O O

C · NH · NH2
NH
N · R + NH2 · NH2 + R · NH2
NH
C · NHR
O O
O
Phthalazineă1, 4 ă dione

This method is not suitable for preparing aromatic primary amines as aryl halides are not good
substrates for nucleophilic substitution.

1.4.4 Reduction of Nitro Compounds

Aliphatic primary amines can be synthesised by reducing nitroalkanes with metal and acid or
with H2 in presence of nickel as a catalyst.
Ni
R · NO2  3H 2 
 R · NH 2  2H2 O

Sn/HCl
+ Base
R · NO2   R · N H3  R · NH2

For example :

H / Ni
O2 N · CH2 CH2 · OH 
2  H2 N · CH2CH2 · OH
2-nitro ethanol 2-amino ethanol

AMINES
QUIZRR 81

Primary aromatic amines are also prepared by the reduction of corresponding nitro compounds.

+
NO2 NH3 NH2

Sn/HCl Base

Other reducing agents such as LiAlH4 and H2/Pt also reduce nitrobenzene to aniline.

R eduction in neutral medium

1. With Fe and steam, a nitro group gets reduced to nitroso group.

Fe/steam
C6 H5 · NO2 
 C6 H5 · NO

2. Zn/NH4Cl H2O or Zn/CaCl2/H2O reduces nitro group to hydroxylamine.

Zn/NH Cl/H O
C6 H5 · NO 2 
4 2  C H · NHOH
6 5
50-50C

Reduction in alkaline medium

Nitro benzene when reduced with Zn/NaOH/C2H5OH gives hydroazobenzene.

Zn/NaOH/C H OH
2C6 H5 · NO2 
2 5 
 C6 H 5 · NH · NH · C6 H5

By means of aqueous ethanolic NH4HS, aqueous Na2S or SnCl2 in HCl, nitro groups in a
polynitro compound can be reduced one at a time. For example, m-dinitrobenzene can be reduced
to m-nitroaniline.
NO2 NH2

+ 3NH4HS + 3NH3 + 3S + 2H2O


NO2 NO2
It is not always possible to predict that which nitro group will be reduced first. For instance, 2,
4ădinitrotoluene when treated with NH4HS, the 4ănitro group is reduced whereas treatment with
SnCl2/HCl results in the reduction of 2ănitro group.
CH3 CH3 CH3

NO2 NO2 NH2


NH4HS SnCl2/HCl

NH2 NO2 NO2


2,4ădinitrotoluene

AMINES
82 QUIZRR
1 .4.5 Reduction of nitriles, isonitriles & oximes

H2(g)
R·C N R · CH2
Pt
NH2

Nitriles are reduced to amines using hydrogen in the presence of a nickel catalyst, although acidic
or alkaline conditions should be avoided to avoid hydrolysis of -CN group. LiAlH4 is more commonly
employed for the reduction of nitriles on the laboratory scale. Both aliphatic & aromatic amines
can be synthesised by reduction.


C6 H6 · N  C 
H2 / Ni
C6 H5 · NH · CH3
LiAlH4

(2Ĉ amine)

H3C H3C
Na/C3H5OH
C N · OH or LiAlH4
CH · NH2 + H2O

C2H3 H5C2
(oxime) (1Ĉ amine)

1.4.6 Reductive Amination of Carbonyl Compounds

Aldehydes and ketones are converted to amines through catalytic or chemical reduction in the
presence of ammonia or amine. Primary, secondary and tertiary amines can be prepared this
method.

R
NH3
R´ · CH · NH2 (1Ĉ amine)
[H]

R
R
R´´NH2
C=O R´ · CH · NH · R´´ (2Ĉ amine)
[H]

R
R´´
R´NHR´´´
R´ · CH · N (3Ĉ amine)
[H]
R´´´

(R´ may be hydrogen or an alkyl group)

AMINES
QUIZRR 83

The above process appears to proceed through the following mechanism.

R
R R
ăH2O
C=O + H2N · R´´ R´ · C · NH · R´´ C = N · R´´
(1Ĉ amine)
R R (imine)
OH
unstable 2[H]

R´ · CH · NH · R´´
1 .4.7 Reduction of Amides (2Ĉ amine)

O R

LialH4 + C H C NH2

R NH2
H
Reactions usually in Et2O or THF followed by H3O+ work-ups.

Ć Amides RCONR´2, can be reduced to the amine, RCH2NR´2 by conversion of the C = O to


-CH2ă
Ć Amides can be reduced by LiAlH4 but not by the less reactive NaBH4.
Ć Typical reagents : LiAlH4/ether solvent, followed by aqueous work-up.
Ć Note that this reaction is different to that of other C = O compounds which reduce to
alcohols.
Ć The nature of the amine obtained depends on the substituents present on the original
amide.

LiAlH4
CH3 CH2 · N · C · CH3 CH3 CH2 · N · CH2 · CH3

CH3 CH3 (3Ĉ amine)

LiAlH4
NH · C · CH3 NH · CH2 · CH3

(2Ĉ amine)

AMINES
84 QUIZRR
1 .4.8 Hofmann rearrangement

Hofmann rearrangement is the organic reaction of a primary amide to a primary amine with one
fewer carbon atom.

O O
Br2 H2O
R C R·NH2
NaOH ăCO2
R NH2
N

The reaction of bromine with sodium hydroxide forms sodium hypobromite in situ, which transforms
the primary amide into an intermediate isocyanate. The intermediate isocyanate is hydrolyzed to
a primary amine giving off carbon dioxide.

The reaction is also sometimes called the Hofmann degradation, and should not be confused with
the Hofmann elimination.

Mechanism :

O O O
Br2 NaOH
NaOH
R NH2 R NHBr R N·Br

H
ă
N N OH
ă Br H2O ăCO2
R C R R·NH2
O O

Example 3 O

NH2
Br2
Identify the product : H2C Product
NaOH
C6H5
Solution :

O
NH2
NH2
Br2
H3C H3C
NaOH
C6H5
C6H5

This is a example of Hofmann bromide reaction.

AMINES
QUIZRR 85

Example 4

A compound X (C7H7Br) reacts with KCN to give Y (C8H7N). Reduction of Y with LiAlH4
yields Z (C8H11N). Z gives carbylamine reaction, reacts with HinsbergÊs reagent in the presence
of aq. KOH to give a clear solution. With NaNO2 and HCl at 0ĈC (Z) gives a neutral compound
which gives red colour. What are X, Y and Z ?
Solution :

KCN LiAlH 4
C7 H7 Br  
 C8 H7 N  C8 H11 N
-KBr
 X Y  Z
CH2Br CH2CN CH2CH2NH2

KCN LiAlH4
ăKBr

(X) (Y) (Z)

Example 5

Convert Benzene to benzylamine.


Solution :
Friedel-crafts alkylation, can easily obtain Toluene from benzene. The sequence of reaction is thus

CH3
CH3Cl Cl2 NH3
Cl NH2
AlCl3 (Anhyd.) hv

Example 6

One mole of each of bromo derivative (A) and NH3 react to give one mole of an organic
compound (B). (B) reacts with CH3I to give (C). Both (B) and (C) react with HNO2 to give
compounds (D) and (E), respectively. (D) on oxidation and subsequent decarboxylation
gives 2-methoxy-2-methyl propane. Give structures of (A) to (E) with proper reasoning.
Solution : H3CO H3CO H3CO
Br NH2 NH
A B C
CH3
CH3 CH3 CH3
CH3 CH3 CH3

NO
H3CO
OH H3CO
D N
E
CH3 CH3
CH3 CH3
CH3
AMINES
86 QUIZRR
REACTIONS OF AMINES

1.5.1 Hofmann Elimination

1. excess CH3I
H · C · C · NH2 C C + N(CH3)3 + H2O
2. Ag2O / H2O

Ć Like alcohols, amines can undergo elimination reactions.


Ć Quaternary ammonium salts undergo an E2 elimination when heated with silver oxide
Ag2O, in water.
Ć Amines can readily be converted into quaternary ammonium iodides by treating them with
excess methyl iodide.
Ć Ag2O/H2O reacts giving the quaternary ammonium hydroxide, silver iodide precipitates.

excess Ag2O
+ AgI
CH3I H2O ă
NH2 (CH3)3NIă (CH3)3NOH

Ć When heated the hydroxide induces a base promoted 1, 2- or -elimination giving an amine
and alkene.
Ć The addition is opposite to that predicted by SaytzeffÊs rule in that it leads to the less highly
substituted alkene.
Ć The less highly substituted alkene is sometimes referred to as the Hofmann product.

+ + N(CH3)3

major minor
heat
ă
(CH3)3NOH
+
Ć The outcome is dictated by steric effects of the large leaving group and the alkyl chain.

Ć NH2 and NR 2 are very poor leaving groups (both anionic), but NR3 is much better (neutral).

Mechanism of the Hofmann Elimination

The initial steps are an example of the alkylation of H


an amine by methyl iodide. The mechanism of the
elimination step is shown. CH3 CH2 · C · C · H
H2
When heated, the hydroxide removes the more (CH3)3 N
+
accessible proton, the  bond of the alkene C = C
forms and the leaving group, a neutral amine departs. HO

CH3 CH2 CH = CH2 + H2O + N(CH3)3

AMINES
QUIZRR 87

1 .5.2 Nucleophilic acyl substitution

Acyl chlorides and acid anhydrides react with primary and secondary amines in cold to form
amides in the Schotten-Baumann reaction. Tertiary amines cannot be acylated due to the absence
of a replaceable hydrogen atom. With the much less active benzoyl chloride, acylation can still be
performed by the use of excess aqueous alkali to facilitate the reaction.

O O

R1 · CH2 + R2 · C R2 · C + H·X

NH2 X NH · R1

O O O

R1 · CH2 + R2 · C R2 · C + R2 · C

NH2 O NH · R1 X

R2 · C
O

Because amines are basic, they neutralize carboxylic acid to form the corresponding ammonium
carboxylate salts. Upon heating to 200ĈC, the primary and secondary amine salts dehydrate to
form the corresponding amides.

R1 O R1 R1 O
+ heat
H · N: + R3 · C · OH H · N · H + R3 · COOă N · C · R3 + H2O
dehydration

R2 R2 R2
amine carboxylic substituted-ammonium amide water
acid carboxylate salt

1.5.3 Nitrosation of Amines


+

HNO2 + R · NH2 R·N N + 2 H2O


1Ĉ amine diazoniumion

R R
HNO2 + NH2 N·N O + H2O

R R
2Ĉ amine Nănitrosoamine

AMINES
88 QUIZRR
Ć Typical reagents : sodium nitrite and aq. HCl or H2SO4 (this mixture yields nitrous acid,
HNO2)
Ć The most useful reactions are probably those of primary aryl amines, Ar-NH2, which give

aryl diazonium salts, Ar- N+2 which can then be used to prepare substituted benzenes.

Ć The actual nitrosation reagent is the nitrosyl cation, NO+ which is formed in situ :

H
+ + ăH2O
H H
O·N O H·O·N O H·O·N O N O
+
+
Nitrite anion Nitrous acid Nitrosylcation

Ć The nature of the product depends on the nature of the initial amine
Ć Primary alkyl or aryl amines yield diazonium salts (hence the diazotisation reaction).

H H
+ ăH
+
+H
+ +
R · N: + :N O: R·N·N O: ăH2O
R·N N:

H
Primary Nitrosyl Diazonium
alkyl amine cation cation

Ć Alkyl diazonium salts are very unstable and yield carbocation-derived products by loss of the
very good leaving group, N2.

+
R·N N: R+ + :N N:

Alkyl Carbocation Nitrogen


diazoniumion

Ć Secondary alkyl or aryl amines yield N-nitrosoamines :

+
+ + ăH
R2N + N O R2N · N O R2N · N O

H H
Secondary Nitrosyl NăNitrosoamine
alkylamine cation

Ć Tertiary alkyl amines do not react in a useful fashion.

AMINES
QUIZRR 89

Ć Tertiary aryl amines undergo nitrosation of the ring (an electrophilic aromatic substitution
reaction).

CH3CH2 CH2CH3 CH3CH2 CH2CH3


N N

+
N=O

N
O

D iazonium Compound

N N

{phenyldiazonium compound}

Aromatic diazonium chlorides, sulphates, nitrates etc. are reasonably stable in aqueous solution
at room temperature or below but cannot be readily isolated without decomposition. The -orbital
system of benzene ring stabilizes the diazonium cation by resonance.

N N N N N N

N N N N

The diazonium salts are very important synthetic reagents, being the starting point in the preparation
of various aromatic compounds. Their reactions may be divided into two groups, those which

involve the liberation of N2 gas and the displacement of the diazo group N+2 by another univalent
group and those in which the two N-atoms are retained (coupling reactions).

AMINES
90 QUIZRR
1 .5.4 Replacement Reactions :

H2O, 
  C6 H 5 · OH + N2  H+

H3PO2O/ H2O 
  C6 H6  N2 + H3 PO3  H+

CuCl/HCl, 
 C6 H5Cl + N2  H+ 
 Sandmeyer
CuBr/HCl,    reaction
  C6 H5 Br + N2  H 

KI, 
 C6 H5 · I + N 2  K +


 C6 H5 N +2 BF4  C6 H5 F  N 2  BF3  Baltz Scheimann reaction


+ HBF ,  

4
C6 H5 · N  N 

Cu/KCN
  C6H5 · CN + N2  K +

NaNO2 /Cu
  C6 H5 · NO2  N2  Na+

C6H6 /NaOH
  C6H5 · C6 H5 + N2  H2O

1.5.5 Azo coupling

An azo coupling is an organic reaction between a diazonium compound and an aniline, phenol
or other aromatic compound which produces an azo compound. In this reaction the diazonium
salts is an electrophile and the activated arene is a nucleophile in an electrophilic aromatic
substitution. In most cases, including the example below, the diazodium compound is also aromatic.

OH
N N OH N
N
H
ăHCl

N N OH

(coupling)

AMINES
QUIZRR 91

Azo couplings are important in the production of dyes and pH indicators such as methyl red.
Coupling with benzene substrates occurs preferentially in the para position to the hydroxyl group.
But if this position is blocked, then the coupling occurs at the ortho position.

OH

OH
C6H5 · N N + OH
ă
C6H5 · N N

CH3

CH3

Example : 1 and 2 naphthols in alkaline soln. couple with diazonium salt in the 4 and 1-position
respectively.

N N · C6H5 OH

OH
2 ă naphthol +
1 ă naphthol
ă C6H5 N2Clă ă
+OH +OH

N N · C6H5

Note : With aromatic amines, there is the possibility of attack on either nitrogen or carbon. In
the case of primary amines, the attack of diazonium ion mainly takes place at the nitrogen
forming diazo-amino compound (A).
H
+ +
C6H5 · N N + H2N C6H5 · N N·N

H
+
ăH

C6H5 · N N · NH

In secondary amines two products will be formed.

NH · CH3 PhN N · N · CH3 NH · CH3

+
C6H6N2 + +

N N · C6H5
AMINES
92 QUIZRR
1 .5.6 Oxidation Reactions

Both the primary and secondary amines undergo oxidation. The oxidation products obtained
depend on the oxidising agent used and on the nature of alkyl group.

Primary amines :
(i) With KMnO4 :

O 
  H2O
R · CH2 · NH2   R · CH = NH   R · CHO + NH3
Aldimine

O 
  H2O
R 2CH · NH2   R 2C = NH   R 2C = O + NH3
Ketimine

(ii) With CaroÊs acid (H2SO5)/H2O2/Peroxy carboxylic acid :

OH
O O O  |
     
R · CH2 · NH2  R · CH2 · NHOH   R · CH = N · OH  R · C = N · OH
N-alkyl
H2O Aldoxime Hydroxamic
hydroxylamine acid

O  O
   
R 2CH · NH2   R 2 CH · NHOH  R 2 C  N · OH
 H2O
Ketoxime

Secondary amines :
(i) With KMnO4 :

R R
[O]
2R2NH N·N
ăH2O
R R
Tetraalkyl hydrazine

(ii) With CaroÊs acid (H2SO5)/H2O2/Peroxy carboxylic acid :

R
[O]
R2NH N · OH
R
N,Nădialkyl hydroxylamine

AMINES
QUIZRR 93

Tertiary amines

 O  
 
(i) With H2O2/Peroxy carboxylic acid  R3 N  R 3 N · O 
 

 
Tertiary amines are oxidised to amine oxide, R N · O (a dipolar ion or Zwitterion).
3

1 .5.7 Reactions of Quaternary Ammonium Salts

(1) Formation of 4Ĉ Ammonium Hydroxides

2R4N+Xă + Ag2O + H2O  2R4N+ OHă + 2AgX


very strong
bases like NaOH

(2) Hofmann Elimination of Quaternary Ammonium Hydroxides


When a quaternary ammonium hydroxide is heated strongly it decomposes to yield water,
a tertiary amine and alkene.

[CH3)3 NCH(CH3)CH2CH3]+ OHă  (CH3)3N + H2C = CHCH2CH3 + H2O


s-Butyltrimethylammonium hydroxide 1-Butene

This E2 elimination gives the less substituted alkene (Hofmann product) rather than the
more substituted alkene (Saytzeff product).

Example 7

Identify the end product (C).


CH3

CH3I moist 
  A  B  C
excess Ag 2 O
N

H
Solution :
CH3 CH3 CH3 CH3 CH3
H
2CH3I Ag2O/H2O  CH3I
+ + +
N N Iă N ă N N ă
OH I

H3C CH3 H3C CH3 H3C CH3 H3C CH3


H H3C

AMINES
94 QUIZRR
CH3 CH3
H
Ag2O/H2O 
+
N
H3C CH3
H3C

1 .5.8 Ring Reactions of Aromatic Amines

NH2, ă NHR and ă NR2 strongly activate the benzene ring towards electrophilic substitution.
(1) Halogenation
For monohalogenation, ă NH2 is first acetylated, because

CH3 · C · N ·
|| |
O H

is only moderately activating

NH2 NH2 NHCOCH3 NHCOCH3 NH2


Br Br
Br2 Ac2O Br2 H2O
ă
H2O OH

Br Acetanilide Br Br

(2) Sulfonation
+ ă +
NH2 NH3 HSO4 NHSO3H NH3

H2SO4 180Ĉ C 180Ĉ C


H2O 3 hours
ă
Anilinium Sulfamic SO3
Sulfate acid Sulfanilic acid
a dipolar ion
(3) Nitration
O O

NH2 NH · C · CH3 NH · C · CH3 NH2


ă
(CH3CO)2O HNO3.H2SO4 OH
heat
Aniline Acetanilide NO2 NO2
p-Nitroacetanilide p-Nitroaniline

To prevent oxidation by HNO3 and meta substitution of C6H5NH3+, amines are first
acetylated.

AMINES
QUIZRR 95

1 .5.9. Rearrangement Reactions

A remarkable property of monoă, diă, and trialkyl anilinium chlorides (or bromides) is their ability
to undergo rearrangement on strong heating, an alkyl group migrating from the Năatom and
entering preferentially the păposition. If this position is occupied, then the alkyl group migrates
to the oăposition. For example, when trimethyl anilinium chloride is heated under pressure, the
following rearrangement takes place.

+
ă
(1) N(CH3)3Cl N(CH3)2.HCl NH(CH3).HCl NH2.HCl

CH3 H3C CH3


300Ĉ C  

CH3 CH3 CH3

This reaction is known as the HofmannăMartius rearrangement.


(2) Rearrangements of this kind have been observed to take place with aniline derivatives of
the type C6H5 ă NH ă Z where Z is R, X, NH2, OH, NO or NO2. For example,

R · N · NO NHR NHR

+ HCl + NOCl + HCl

NO

This reaction is called FischerăHepp rearrangement.

NH2 · NH2.HCl NH2.HCl

250ĈC

NH2

(3) Benzidine Rearrangement


Hydroazobenzene, C6H5NH ă NHC6H5 undergoes rearrangement when heated in the presence
of acid to yield benzidine (4, 4´ădiamino diphenyl).

AMINES
96 QUIZRR
NH · NH
NH2
+
H
NH2 NH2 + NH2

4,4'ădiaminodiphenyl 2,4'ădiaminodiphenyl

This is known as benzidine rearrangement. The p, p´ăisomer is the major product (70%) and the
rest is o, p´ and o, o´ăisomer. The reaction probably follows the given mechanism.

+ +
NH · NH NH2 NH2 NH2 NH2

HH
+
ă2H

+ + (o,o'ăisomer)
2H
+ +
+ +
NH2 NH2
NH2 · NH2

+
ă2H
NH2 NH2

H H (p, p'ăisomer)

+
+
NH2 NH2
+ +
H ă2H
NH2 NH2
H

(o, p'ăisomer)

1 .5.10 Reaction with Diethyl Oxalate

1Ĉ, 2Ĉ and 3Ĉ amines can be distinguished by their reaction with diethyl oxalate. Primary (1Ĉ)
amines react with diethyl oxalate forming N, Năoxamide, which is a solid.

COOC2H5 HăNHăR CONHR


+ + 2C2H5OH
COOC2H5 HăNHăR CONHR
N, N´ădialkyl
oxamide (solid)

AMINES
QUIZRR 97

Secondary (2Ĉ) amines react with diethyl oxalate forming oxamic ester, which is a liquid.

COOC2H5 CONHR2
+ HNR2 + C2H5OH
COOC2H5 COOC2H5
oxamic ester
(liquid)

Tertiary (3Ĉ) amines do not react with diethyl oxalate.

T est for Amines

(1) Hinsberg Test : This reaction is used for the separation of amines from a mixture. The
mixture containing primary, secondary and tertiary amines is treated with an aromatic
sulphonyl chloride.

O
1Ĉ amine (RNH2)
p ă CH3 ă C6 H4 ă S ă NHR

O
O NaOH O
+
P ă CH3 ă C6 H6 ă S ă Cl p ă CH3 ă C6 H4 ă S ă N ă R K
2Ĉ a
min
O e (R O
2 NH
)
O

3Ĉ amine (R3N) p ă CH3 ă C6 H4 ă S ă NR2

O
KOH

No reaction No reaction

(2) Carbylamine Reaction : This is a chemical test for the detection of primary amines. Both
aliphatic and aromatic primary amines when heated with chloroform and ethanolic KOH
form isocyanide, also called carbylamine, a foul smelling compound.

R ă NH2 + CHCl3 + 3KOH RăN C + 3KCL + H2O


or or
Ar ă NH2 Ar ă N C

AMINES
98 QUIZRR
Mechanism :
OH
CHCl3 ă : CCl2
ăCl
ăH2O
H Cl
.. + ă
R · NH2 + : CCl2 R·N·C

H Cl

ă HCl

+ ă
ăHCl H Cl
R·N C
+
ă
R·N C

Example 8

How will you bring about the following conversion ? Aniline  Benzylamine
Solution :
ă
NH2 N2Cl

CH3
CH3Cl Cl2 Cl NH NH 2
NaNO2 + HCl H3PO4 2
0ă5Ĉ C  AlCl3 (Anhyd) h

Example 9

Convert :
Cl Cl

Br
NH2

Cl Cl

Solution :
Cl Cl Cl Cl Cl
COOH Br
NH2 OH
HNO2 [O]  Br2/Fe
ăCO2

Cl Cl Cl Cl Cl

AMINES
QUIZRR 99

Example 10

CH3 OH
NH

H
+
H3C CH3
O + CH3NH2
O

O
Predict the mechanism of above conversion.
Solution :
CH3 CH3

O Oă ă
+ CH3NH ă H + + CH3NH
H +

O O
+
H
CH3
NH

H3C OH

Example 11

Bring out the following conversion.


O

NH2

Solution :
O
CH3
H
O ă
NH2 1)NaNO2/HCl O3/zn/H2O OH
O

CH3

AMINES
100 QUIZRR
Example 12
Amphetamine is a liquid that is nearly insoluble in water, but quite soluble in aqueous
acid. It can be resolved into enantiomers and liberates nitrogen spontaneously when reacted
with nitrous acid. When amphetamine is quarternised with CH3I, heated with Ag2O and
heated, 3ăphenylă1ăpropene is the major product. Deduce the structure of amphetamine.
Solution : +
H2N (H3C)3N
CH3 CH3

H H

CH3I

Amphetamine Quaternary salt


( chiral carbon)

Ag2O
+
HNO2/H
ă N2 H

HO CH2
CH3

H + (CH3)3N

3 - Phenyl propene

1 - Phenyl 2 - propanol

Example 13

NaNO2 /HCl
C5 H13 N 
 N2
 Y  other products . (X) is optically active. Identify the
X 3 alcohol

structure of (X) and (Y). Explain the formation of (Y) from (X). Will (Y) also be optically
active ? Justify your answer. Draw the structure of important intermediates, if any.
Solution :

CH3 CH3 CH3 CH3 CH3 CH3 CH3


+ ă
CH3·CH·CH·NH2 .
(Optically active)
NaNO2/HCl
CH3·CH·CH·N N
·N2
CH3·CH·CH
+
·H
CH3·C·CH2CH3
+
3Ĉ carbocation
(X) 2Ĉ carbocation
+
ă H2O/·H
ăH
CH3 CH3 CH3 CH3
+
NaNO2 ·N2
CH3CH2·CH·CH2NH2 CH3CH2·CH·CH2·N N CH3·CH2·CH·CH2 CH3·C·CH2CH3
HCl +
1Ĉ carbocation
OH (Y)
Optically inactive

AMINES
Carboxylic Acid
& Derivative
QUIZRR 3

CARBOXYLIC ACIDS (–COOH)


When a carbonyl carbon also bears a hydroxyl group (ă COOH), then these compounds are
appreciably acidic, and are called carboxylic acid.

NOMENCLATURE

The aliphatic carboxylic acids are commonly known by their initial names.
Example {Common names}
Ć HCOOH Formic acid
Ć CH3COOH Acetic acid
Ć CH3 CH2 COOH Propionic acid
IUPAC naming : These are normally suffixed as „oic acid‰, when COOH is the principal functional
group and its carbon is counted while numbering longest chain. The carboxyl carbon is assigned
number 1 in the IUPAC nomenclature. For example,

5 4 3 2 1
1. CH3 · CH · CH · CH2 · COOH 3, 4,-dimethyl pentanoic acid
| |
CH3 CH3

2. HCOOH Methanoic acid

COOH CO2H H

3. 4.

Cyclohexane
Benzene Carboxylic acid Carboxylic acid
(or Benzoic acid)

COOH
Dicarboxylic acids are named as „alkanedioic acids‰. For example, | is named as
COOH

ethanedioic acid.

Naming Acid Chlorides & Anhydrides

Acids chlorides are named systematically as acyl chlorides.

O
H3C ·
Cl
Acetyl Chloride

CARBOXYLIC ACID & DERIVATIVE


4 QUIZRR
An acid anhydride is named by substituting anhydride for acid in the name of acid from which
it is derived.

O O

CH3 · C · O · C · CH3
Ethanoic Anhydride

If two different alkyl groups are present, then alkanoic are named in alphabetical order

O O

CH3CH2 · C · O · C · CH3
Ethanoic propionic anhydride

Naming Amides and Imides : The names of amides are formed by replacing ă oic acid by amide
or ă carboxylic acid by carboxamide.

O O

H3C · C · NH2 C · NH2


Acetamide

Cyclohexane Carboxamide

If the nitrogen atom of amide has any alkyl groups as substituents, the name of the amide is
prefixed by the capital letter N i to indicate substitution on nitrogen, followed by name(s) of alkyl
group(s).

CH3 · CH2 · C · NH

Năcyclohexyl propanamide

O CH2 CH3

CH3 CH2 CH2 · C · N · CH2 CH3

N, Nădiethyl butanamide

CARBOXYLIC ACID & DERIVATIVE


QUIZRR 5

Physical Properties

(a) Physical state and smell


The first three members are colourless liquids and have pungent smell. The next six members are
oily liquids with a faint unpleasant odour.
Still higher acids are colourless waxy solids.
Benzoic acids and its homologues are colourless solids.

(b) Boiling points : They have higher boiling points than the corresponding alcohols of comparable
molecular masses.
Carboxylic acids have higher boiling points due to the presence of intramolecular hydrogen
bonding. Due to the hydrogen bonding, carboxylic acids exists as dimers.

OăH----O
R R
O----HăO

(c) Melting point : In the case of first ten carboxylic acids, the melting points of acids containing
even number of carbon atoms is higher than the next lower and higher member containing odd
number of carbon atoms.
The melting and boiling points of aromatic acids are usually higher than those of aliphatic acids
of comparable molecular masses.

(d) Solubility in water : The first four members of aliphatic carboxylic acids are very soluble in
water. The solubility in water decreases gradually with rise in molecular mass. All are soluble in
alcohol or ether.
Benzoic acid is sparingly soluble in cold water but is soluble in hot water, alcohol and ether.

Example 1

Give the IUPAC names of the following compounds :

COOH COOH
(i) | (ii)
COOH HOOC

OH HOOC OH

(iii) H3C (iv)

COOH HO COOH

CARBOXYLIC ACID & DERIVATIVE


6 QUIZRR
Solution :
IUPAC name General name
(i) Ethanedioic acid Oxalic acid
(ii) Butanedioic acid Succinic acid
(iii) 2-Hydroxy propanoic acid Lactic acid
(iv) 2, 3 Dihydroxybutanedioic acid Tartaric acid
Aromatic acids Ar ă COOH are usually named as derivatives of the parent acid i.e. benzoic acid,
C6H5COOH.
COOH COOH

NO2

2ănitrobenzoic acid
Br
4ăbromobenzoic acid

Example 2

Acid and acid derivatives although contain > C = O group, do not undergo the usual
properties of carbonyl group explain.
Solution :
It is because of the possibility of resonance which compensates the electron deficiency of carbonyl
carbon to some extent for example.
ă
O O
+
R · C · NH2 R·C NH2
ă
O O
+
R · C · OH R·C OH

Example 3
Carbon-oxygen bond length in formic acid are 1.24 Å and 1.36 Å but in sodium formate both
the carbon-oxygen bonds have same value i.e. 1.27 Å.
Solution :
In formate ion resonance gives rise to identical bond lengths.

O
ă
H·C·O H·C O H·C ă

O
O Oă

CARBOXYLIC ACID & DERIVATIVE


QUIZRR 7

Whereas no such resonance is noticed in formic acid (H · COOH) and thus C · O bonds are
different in HCOOH.

Example 4
Why the bond length of C = O in carboxylic acids is bit longer than in aldehydes ?
Solution :

·
O
Due to resonance, there is considerable contribution of the resonance from R to the
+
O·H
hybrid from, i.e., there is some single bond character in C = O bond which makes it some what
longer than in aldehydes.

Acidity of Carboxylic Acids

The acidity of a carboxylic acid is due to the resonance stabilization of its anion.
ă
O O
ă
R·C·O R·C O

The acidity of carboxylic acid depends very much on the substituent attached to ă COOH group.
Since acidity is due to the resonance stabilization of anion, substituents causing stabilization of
anion increases acidity whereas substituent causing destabilization of anion decreases acidity. For
example, electron withdrawing group disperses the negative charge of the anion and hence makes
it more stable causing the increase in the acidity of the corresponding acid. On the other hand,
electron releasing group increases the negative charge on the anion and hence makes it less stable
causing the decrease in acidity.
Ć Increase in the number of halogen atoms on -position increases the acidity, e.g.
CCl3 COOH > CHCl2COOH > ClCH2 COOH > CH3COOH.
Ć Increase in the distance of halogen from COOH decrease the acidity, e.g.

CH3 · CH2 · CH · COOH > CH3 · CH · CH2 · COOH > CH2 · CH2 · CH2 · COOH
| | |
Cl Cl Cl

Example 5
Arrange the following in increasing order of acidic strength :
Benzoic acid, 4-nitrobenzoic acid, 3, 4-dinitrobenzoic acid, 4-methoxybenzoic acid
Solution :
Since electro-donating groups decrease the acid strength, therefore, 4-methoxybenzoic acid is a
weaker acid than benzoic acid.

CARBOXYLIC ACID & DERIVATIVE


8 QUIZRR
Further since electron-withdrawing groups increase the acid strength, therefore, both 4-nitrobenzoic
acid and 3, 4-dinitrobenzoic acids are stronger acids than benzoic acid. Further due to presence
of an additional NO2 at m-position w.r.t. COOH group, 3, 4-dinitrobenzoic acid is a little stronger
acid than 4-nitrobenzoic acid. Thus the overall acid strength increases in the order :
4-methoxybenzoic acid < benzoic acid < 4-nitrobenzoic acid < 3, 4-dinitrobenzoic acid

Example 6

(a) Why are both păMe and măMe substituted acids weaker than PhCO2H with the păacid
being the weakest one ?
(b) Why is the păOH acid weaker than PhCO2H while the măOH acid is stronger ?
Solution :
(a) Both păMe and măMe are electronădonating and acid weakening. măMe is electron donating
by induction while păMe is electronădonating by induction as well as hyperconjugation. pă
Me being more electron releasing is more acid weakening and păMe and măMe substituted
acids both are weaker than PhCO2H.
(b) OH group shows ă I effect from măposition while from păposition it shows ăI and + R effects.
Electronădonation by resonance (+ R) is more prominent than electron withdrawal by induction
(ă I) for OH group. So măOH is electronăwithdrawing and acid strengthening, thus măOH
acid is stronger than PhCO2H. păOH is electronădonating and acid weakening, thus păOH
acid is weaker than PhCO2H.

Example 7

Though phenoxide ions (C6H5Oă) has more resonating structure than carboxylate ion, but
carboxylic acid is stronger acid than phenol, explain.
Solution : ă
O O O O

ă ă

O ă
O
ă
R·C·O R·C
O

Acquisition of negative charge on more electropositive carbon atom, drives the phenoxide towards
lesser stability.

CARBOXYLIC ACID & DERIVATIVE


QUIZRR 9

Example 8

On the basis of H-bonding explain that the second ionization constant K2 for fumaric acid
is greater than for maleic acid.
Solution :
We know that H-bonding involving acidic H has an acid weakening effect and H-bonding in
conjugate base has an acid strengthening effect.
Both dicarboxylic acids have two ionisable hydrogen atoms. Considering second ionization step.

Since the second ionisable H of the Maleate ion participates in H-bonding more energy is needed
to remove this H because the H-bond must be broken. The maleate mono anion is, therefore, the
weaker acid.

General Methods of Preparation

1. By oxidation of primary alcohols or aldehydes oxidation of primary alcohols can be achieved


by acidified K2Cr2O7 or acidified/alkaline KMnO4.

(i) KMnO4 /OH 


RCH 2OH   RCOOH
(ii ) H3O

Aldehydes can be oxidized to carboxylic acids with mild oxidizing agents such as


Ag  NH3  OH
2

(i) Ag 2O or Ag (NH3 )+
R · CHO  2 R · COOH
(ii) H3O

2. By oxidation of alkenes when alkenes are treated with an oxidizing agent such as KMnO4
or on oxidative ozonlysis, it forms carboxylic acids.

CARBOXYLIC ACID & DERIVATIVE


10 QUIZRR


(i) KMnO4 , OH
RCH = CHR´   RCOOH + R´COOH
(ii) Heat, H2O

(i) O3
RCH = CHR´   RCOOH + R´COOH
(ii) H2O, distill

3. By haloform reaction
Ć This method involves oxidation of methyl ketones with hypohalite ion, XO ă (NaOH + X2)

O O
NaOH/I2
R · C · CH3 + 3NaOI R · C · ONa + CH3I3 + NaOH
HCl

R · C · OH
Note : In this method, the acid produced has one C atom less than parent ketone.

4. By oxidation of alkyl benzene


When the arenes containing an -hydrogen atom are reacted with a strong oxidizing
agent like alkaline KMnO4, it forms benzoic acid.

CH3 COOH

ă
(i) KMnO4/OH
+
(ii) H3O

5. Formation and Hydrolysis of Nitriles


Nitriles can be hydrolyzed by dilute acid to generate carboxylic acids.

H3O+
E.g. R · C  N   R · CO2 H
or  OH

Nitriles are easily made by the action of cyanide ion as a nucleophile on alkyl halides (or
tosylates).

NaCN H3O+
R · CH2 · Br   R · CH2 · C  N   R · CH2 · CO2 H

Again the overall transformation is from alkyl halide to a carboxylic acid with an extra
carbon atom.

CARBOXYLIC ACID & DERIVATIVE


QUIZRR 11

6. Using Grignard Reagent


Grignard Reagents react with carbon dioxide to yield magnesium carboxylate acidification
produces carboxylic acids.

dry ether
R · X + Mg R · MgX
CO2

+
H3O
R COOH R-COOMgX

Note : The acid thus formed has one carbon more than the alkyl group of the Grignard
reagent.
7. By hydrolysis of 1, 1, 1-Trihalogen Derivatives

KOH
R CX3 [ R · C(OH)3]
unstable

·H2O

R · COOH + H2O
8. By alkenes (Koch Reaction)
A recent method for manufacturing fatty acids is to heat an olefin with carbon monoxide
and steam under pressure at 300-400ĈC in the pressure of catalyst. e.g. phosphoric acid.

H3PO4
CH2  CH2  CO + H2O   CH3 · CH2 · COOH
steam 300-400C

9. Decarboxylation of gemdicarboxylic acid


Monocarboxylic acids may also result from the decarboxylation of dicarboxylic acids (when
the two CO2H groups are present on same carbon). Acetic acid is thus obtained by heating/
warming malonic acid.

COOH

H2C CH3COOH + CO2
COOH

10. Hydrolysis of acid derivatives


Another useful method constitutes the hydrolysis of carboxylic acid derivatives such as acyl
halides, acid amides, acid anhydrides or esters.

O
||
H2O/H+
CH3 CH2 · C · Cl   CH3 CH2 COOH + HCl

CARBOXYLIC ACID & DERIVATIVE


12 QUIZRR
O +
H2O/H
CH3CH2 · C +
CH3CH2COOH + NH4
NH2
+
H2O/H
CH3CH2 · C · O · C · CH2CH3 2CH3CH2CO2H

O O

+
H2O/H
CH3CH2 · C · OCH3 CH3CH2CO2H + CH3OH

Example 9

How will you synthesize ?


(i) Acetic acid from methyl iodide
(ii) Acetic acid from propionic acid
(iii) Acetic acid from acetamide
Solution :
+
(i) H2O / H
CH3I + KCN CH 3 CN CH3COOH

NH3  Br2/KOH
(ii) CH3CH2COOH CH3CH2COONH4 CH3CH2CONH2 CH3CH2NH2

HNO2
[O] Na2Cr2O7
CH3COOH CH3CHO CH3CH2OH
H2SO4

(iii) CH3 CONH2  HNO2  CH3COOH + N2  H2 O

Example 10

(CH3)2C = CHCOCH3 can be oxidised (CH3)2C = CHCOOH by


(a) Chromic acid (b) NaOH and I2
(c) Cu at 300ĈC (d) KMnO 4
Solution :
O O
NaOH and I2 ă Acidification
(CH3)2C CH · C · CH3 (CH3)2C CH · C · O (CH3)2C CHCOOH
(Iodoform test)

 (b)

CARBOXYLIC ACID & DERIVATIVE


QUIZRR 13

Example 11

Identify A to E
OH

ă +
Conc. H2SO4 Cl2/H2O CN H3O
[A] [B] [C] [D]
2ă +
Cr2O7 / H

[E]
Solution :
Cl CN

OH OH

A: B: C:

COOH COOH

OH O

D: E:

CHEMICAL PROPERTIES :

Reactions of carboxylic acids can be discussed under four heads :


(1) Reactions due to H atom of ă COOH group
(2) Reactions due to ă OH part
(3) Reactions due to ă COOH group
(4) Reactions due to R-part
I. Reactions due to H atom of ă COOH part
O
NaOH ă +
R · C · O Na + H2O

O
K2CO3 ă +
O 2R · C · O Na + H2O + CO2

R·C·O·H O
NH3 ă +
R · C · O NH4

O
N2
R · C · OCH3 + N2 ( )
CARBOXYLIC ACID & DERIVATIVE
14 QUIZRR
II. Reaction due to ă OH group
O
PCl5
R · C · Cl
or SOCl2 or
PCl3
O
NH3 ·H2O
O R COONH4 R · C · NH2
Heat
amide
R · C · OH O
R´OH
R · C · OR´ + H2O
Conc. H2SO4 Ester
(This is called esterification)

O O
P2O5
R·C·O·C·R
·H2O
Acid anhydride

M echanism of esterification by acid catalysis

A direct reaction between a carboxylic acid and alcohol under the catalytic effect of sulphuric acid
yields an ester. This is a reversible reaction.

+
O O·H OH
+ Fast +
R · C · OH + H R · C · OH R·C

OH

OH

+ R·C
OH2 OH
+ H
IMPE +O · H
R · C · OR´ R·C·O
R´ O·H
OH O·H
R

·H2O

+ +
·H
R · C · OR´ R · C · OR´ R · C · OR´
fast
OH +O ·H O

CARBOXYLIC ACID & DERIVATIVE


QUIZRR 15

Ć The reactivity of esterification decreases with increase in crowding of alkyl groups around
ă OH among alcohols and around ă COOH among acids. For example,
CH3 OH > C2H5OH > (CH3)2 CHOH > (CH3) COH
Also, HCOOH > CH3COOH > (CH3) CHCOOH
Ć The reactivity of aromatic acids towards esterification decreases with increase in the number
of alkyl substituents at ortho position.

III. Reaction involving complete ă COOH group


(1) Soda lime decarboxylation : Sodium or potassium salts of carboxylic acids on heating
with soda lime (NaOH and CaO) give alkanes with one carbon less than the parent acids.

CaO
RCOONa + NaOH   R ă H + Na2CO3
Sodium salt of acid alkane

(2) Reduction

LiAlH4
R COOH   R CH2OH

(3) Reaction with hydrazoic acid (HN3)

O
||
H2SO4
R · C · OH + HN3   R · NH2  CO2  N2

(4) Brominative decarboxylation (Hunsdicker reaction)

 R · Br + AgBr     CO2   
Ag 2O Br2
R · COOH   R CO2 Ag + 
in CCl4

(5) Heating of dicarboxylic acids :

COOH Conc. H2SO4


(a) CO( ) + CO2 ( ) + H2O

COOH

COOH

(b) CH2 CH3COOH + CO2 ( )
COOH

CH2 · CH2
(c) O C C  , ·H2O

O O O O
O
H H
Succinic acid Succinic anhydride

CARBOXYLIC ACID & DERIVATIVE


16 QUIZRR

(CH2)3 
O
(d) HO2C CO2H ăH2O

Cyclopentanone

(6) Heating of ,  and -hydroxy Carboxylic acids

O O
CH3 CH3 CH3 H
O O
 
(i) 2 CH3 · CH · COOH + 
·2H2O
O O
OH H H H CH3
O O
Cis · Lactide (racemic) trans ă lactide (Meso)


CH2


(ii) CH3 · CH C O CH3 · CH CH · CO2H
(, ă unsaturated Carboxylic acid)
OH OH
(ăhydroxy acid)

 
CH2 · CH2  

(iii) CH3 · CH C O  H
 O
·H2O
OH OH CH3 O
 ă hydroxy acid 5 ă Methyl ă  ă butyrolactone
or pentano ă 4 ă Lactone

 
 H
(iv) CH3 · CH 

C=O CH3 O
·H2O
OH H O O
ă hyrdroxy acid
6 ă methyl ă  ă valero lactone
or Hexano ă 5 ă Lactone

7. Decarboxylation : -keto acids on slightest warming alone or in presence of a base undergoes


ready loss of CO2. The process of loss of CO2 is called „decarboxylation‰. For example,

  
CH3 · C · CH2 · C · OH  CH3 · C · CH3  CO2 
|| || ||
O O O

CARBOXYLIC ACID & DERIVATIVE


QUIZRR 17

Mechanism :
CH2 CH2
 Y·C CH2 + CO2
Y·C C O Y·C C O
OH
O O O O
Enol form
H H
Transition state
(6ămembered) Tautomerizes

Y · C · CH2

O
Keto form

Here Y can be substituents like CH3, OH etc.


O
CO2H COOH Further
 heating
CO2H + CO2 O
·CO2
CO2H COOH
(gemădicarboxylic acid) O
cis and trans anhydride

I V. Reactions involving R-Part

(1) Hell-Volhard Zelinsky (HVZ) reaction


It is also the -Halogenation of aliphatic acids. When carboxylic acids are treated with Cl2
or Br2 in presence of red phosphorus, the -hydrogen atoms of carboxylic acids are replaced
by chlorine or bromine.

O Br
Red P/Br2
RCH2 · C · OH R · CH · C · OH

O
ăbromoacid

Mechanism :
P + Br2 Tautamerize
CH3 CH2 COOH CH3 CH2 C · Br CH3 · CH C · Br
(Or PBr3)
O OH


Br · Br

+
Br Br Br O·H
CH3CH2COOH ·H+
CH3 · CH · C · OH CH3 · CH · C · Br CH3 · CH C · Br

O O
CARBOXYLIC ACID & DERIVATIVE
18 QUIZRR
Example 13

Acetic acid can be halogenated in presence of phosphorus and chlorine but formic acid
cannot be halogenated in the same way.
Solution :
This is HVZ reaction. It occurs only in those carboxylic acids which have -hydrogen atoms. Acetic
acid possesses three -hydrogen atoms but formic acid does not have even a single -hydrogen
atom. Thus, formic acid does not undergo this reaction.

Example 14

Predict the stereochemistry at ăC for ăbromo acid (optically active) when treated with
NaOH.
Solution :
The configuration (if chiral carbon) at ăcarbon remains same i.e. reaction proceeds with retenition
of configuration.

Br H OH
O
ăBr OH
R · C· C R·C·C O Inversion
R·C·C O
Oă O (SN2)
H H Oă
(R) (S) (R)

Example 15

Write possible mechanism for the given reaction.

H+
Ph · COCOOH  PhCOOH + CO

Solution :
O O O O O
+ +
H +
Ph · C · C · OH Ph · C · C · OH2 Ph · CO · C

O
+
+ Ph · C · C O
H2O ăCO
Ph · C O Ph · C O
+
OH2
+
ăH
O
Ph · C
OH

CARBOXYLIC ACID & DERIVATIVE


QUIZRR 19

Example 16
Identify A, B, C, D, E and F in the following reactions :
HgSO4 + H2SO4
CH CH (A) Oxidation (B)
(D) PCl5

(C)
(CH3)2Cd

(E)
+
CH3MgBr, H3O

(F)
Solution :
O O O
HgSO4 + H2SO4 PCl5
oxidation
CH CH CH3 · C · H CH3 · C · OH CH3 · C · Cl
(A) (B) (C)
Pd, BaSO4, H2 (CH3)2Cd

(D)

OH O
CH3MgBr
CH3 · C · CH3 +
CH3 · C · CH3
H3O (E)
CH3

Example 17

An organic acid (A), C5H10O2 reacts with Br2 in the presence of phosphorus to give (B).
Compound (B) contains an asymmetric carbon atom and yields (C) on dehydrobromination.
Compound (C) does not show geometric isomerism and on decarboxylation gives as alkene
(D) which on ozonolysis gives (E) and (F). Compound (E) gives a positive Schiffs test but (F)
does not. Give structures of (A) to (F) with reasons.
Solution :

CH3 CH3 CH3


P + Br2
CH · CH2COOH CH · CHBr · COOH CH3 · C CH · COOH
(A) (B) (C)
CH3 CH3
decarboxylation

CH3 CH3
Ozonolysis
CH3 · C O + HCHO CH3 · C CH2
(F) (E) (D)
Does not give Give Schiff's
Schiff´s Test Test

CARBOXYLIC ACID & DERIVATIVE


20 QUIZRR
Example 18

An organic compound (A) C6H12O3 on treatment with concentrated H2SO4 gives CO, H2O and
(B). Compound (B) can be prepared by passing vapours of 1ăpentanol over heated copper
at 570 K. Compound (A) on heating gives (C) C12H20O4. Give structures of (A) to (C) with
proper reasoning.
Solution :

Cu
CH3(CH2)3 CH2OH   CH (CH ) ă CHO + H O
570 K 3 2 3 2

OH
| Conc. H2SO4
CH3  CH2 3 · CH · COOH   CH3  CH2 3 CHO + CO2  H2O

CH3(CH2)3CHOH HOOC
+
COOH HO · CH · (CH2)3CH3

, · 2H2O

O · CO
CH3(CH2)3CH CH(CH2)3CH3
CO ·O
Lactide

Example 19

An optically active organic compound (A) (C8H13Cl) does not decolourises bromine water
solution. (A) on treatment with alcoholic KOH can produce 2 products in principle but
infact only one product (B) (C8H12) is obtained. (B) on treatment with ozone followed by
work up with H2O2 yields (C) (C8H12O4) which cannot be resolved into enantiomers. (C) on
heating with NaOH/CaO yields (D) (C6H12) which on monochlorination yields C6H11Cl single
isomer. Deduce structures of (A) to (D).
Solution :

COOH

Cl
alc. KOH Ozonolysis NaOH/CaO
in absence of Zn
(A) (B)
(D)
COOH
(C)

CARBOXYLIC ACID & DERIVATIVE


QUIZRR 21

CARBOXYLIC ACID DERIVATIVES


Carboxylic derivatives are described as compounds that can be converted to carboxylic acids via
simple acidic or basic hydrolysis.
The most important acid derivatives are esters, amides and nitriles, although acid halides and
anhydrides are also derivatives (really activated forms of a carboxylic acid).

O O O O O

C C C C C
R X R O R R OăR´ R NH2 R·C N
acid halide anhydride ester amide nitrile
RCOX (RCO)2O RCO2R" RCONH2 RCN

Reactivity of Carboxylic Acid Derivatives

Carboxylic acid derivatives react tend to react via Nucleophilic Acyl substitution where the group
on the acyl unit, R-C=O undergoes substitution :


O O
ă
Nu 
ă
LG Nu LG

Study Tip : Note that unlike aldehydes and ketones, this reactivity of carboxylic acids retains the
carbonyl group, C=O.
The observed reactivity order is shown below :

O O O O O O O
> > > >
C C C C C C C
R Cl R O R R OR R OH R NHR R Oă

This reactivity order is important. You should be able to understand, rationalise and
use it.

It is useful to view the carboxylic acid derivatives as an acyl group, R-C=O, with a different
substituent attached.
The important features of the carboxylic acid derivatives that influences their reactivity are
governed by this substituent in the following ways :
Ć the effect the substituent has on the electrophilicity of the carbonyl C
O

C
R Z

CARBOXYLIC ACID & DERIVATIVE


22 QUIZRR
* if the substituent is electron donating, then the electrophilicity is reduced,  less reactive.
* if the substituent is electron withdrawing, the electrophilicity is increased,  more reactive.
Ć the ability of the substituent to function as a leaving group.

There are 3 resonance structures to consider for carboxylic acid derivatives.


I and II are similar to those of aldehydes and ketones, but there is also a third possibility III
where a lone pair on the heteroatom Z is able to donate electrons to the adjacent positive center.
The stronger this electron donation from Z the less positive the carbonyl C and the less electrophilic
the carbonyl group. The ability of Z to donate electrons is linked to its electronegativity... the more
electronegative Z is, the less the stabilising effect.

ă ă
:O: :O: :O:
+
C C C
R Z R + Z R Z

I II III

It is also useful to appreciate where aldehydes and ketones fit into the reactivity scale
towards nucleophiles.
acyl halides > anhydrides > aldehydes > ketones > esters = carboxylic acids > amides

Acid Halides

Nomenclature

Acid halides are named by taking the -ic acid suffix of the related carboxylic acid, replacing it with
-yl, and adding the halide name.
E.g.

O Br O

CH3CH2 · C · F H3C · CHCH2 · C · Cl


propanoyl fluoride 3ăbromobutanoyl chloride

 O 
 || 
(a) Preparation of acid halides   C · X 
 
 
 

(i) R · COOH + SOCl2  RCOCl + SO2   HCl 

CARBOXYLIC ACID & DERIVATIVE


QUIZRR 23

(ii) 3R · COOH + PBr3  3RCOBr + H3 PO3

O O
NaF + Anhydrous HF
(iii) R·C R·C
ă
O F

(iv) RCOOH + PCl5  RCOCl + POCl3 + HCl

The lower acyl chlorides are colourless liquids with irritating odour. Higher members
are colourless solids.

(b) Reactions of acid halides

(i) Interconversion Reactions of Acyl Chlorides

O O
RCO2H
C C acid anhydrides
R O R
O

C N
R Cl
O
R´OH
C esters
R OR´

O
H2O
C acids
R OH
O

C O
R Cl
R2NH
base C Amides
R NR2

All are nucleophilic acyl substitution reaction


Ć Acyl chlolrides are the most reactive of the carboxylic acid derivatives and therefore
can be readily converted into other carboxylic acid derivatives (see above).
Ć They are sufficiently reactive that they react quite readily with cold water and hydrolyse
to the carboxylic acid.
Ć The HCl by-product is usually removed by adding a base such as pyridine or triethyl
amine.

CARBOXYLIC ACID & DERIVATIVE


24 QUIZRR
(ii) Formation of ketones : Alkyl Grignard reagent react with anhydrous cadmium chloride,
CdCl2 to form a dialkylcadmium compound, R2Cd. This intermediate, on reaction with an
acyl chloride, yields a ketone. Organo cadmium compounds are less reactive than Grignard
reagents and are thus more selective in their attack as nucleophiles.

2R ă MgX + CdCl2  R2Cd + 2MgXCl

O O

2CH3 · C · Cl + R2Cd 2CH3 · C · R + CdCl2

Both aliphatic and aromatic ketones can be prepared in this matter.

2O2N COCl + (CH3)2Cd 2O2N C·CH3 + CdCl2

păNitrobenzoyl chloride păNitroacetophenone

(iii) Rosenmund Reduction : This reaction requires hydrogen and Pd, which is a catalyst,
adsorbed on BaSO4 and a catalytic poison like quinoline to slow down the reaction. The
result is the formation of an aldehyde. If the given catalytic poison is not employed the
product would have been a primary alcohol.

O
H2/Pd ă BaSO4
C2H5 · C · Cl C2H5CHO + HCl
Sulfur ă quinoline

Acid Hydrazides

ROCl on treatment with hydrazine yields acid hydrazides.

RCOCl + 2NH2 · NH 2  RCONH NH2  NH2 NH3+ Cl 

The acid halide, on treating with the salt of corresponding acid, yields an anhydride.

O O
|| ||
RCOCl + RCOO Na   R · C · O · C · R + NaCl

Esters

These are derivatives of carboxylic acids where the hydroxyl group is replaced by an alkoxy
group.

CARBOXYLIC ACID & DERIVATIVE


QUIZRR 25

O O
+
H
R · C · OăH + R´ · OăH R · C · OăR´ + H2O

Nomenclature
The names of esters are derived from the names of the compounds that are used to create them.
The first word of the name comes from the alkyl group of the alcohol, and the second part comes
from the carboxylate group of the acid used.
O O
+
H
E.g. H3C · OăH + HăO · C · CH3 H3CO · C · CH3 + H2O
methanol ethnoic methyl ethanoate
acid

A cyclic ester is called a lactone, and IUPAC names of lactones are derived by adding the term
lactone at the end of the name of the parent carboxylic acid it came from.
E.g. O
O
C
C
H2C OH H2C
O + H2O
H2C OH H2C
C
C
H2
H2
4ăhydroxybutanoic 4ăhydroxybutanoic
acid acid lactone

Preparation :

O O
|| ||
(i) conc. H2SO4
R · C · OH + R´OH   R · C · OR´ + H2O

(ii)  CH3 2 C = CH2  CH3COOH 


conc. H SO
 3 3
2 4  CH · C · OC CH
3
||
O

O O
ă +
  ă +
(iii) R · C · OH + CH2 N2 R · C · O + CH3 · N N

2
SN

R · C · OCH3 + N2( )

CARBOXYLIC ACID & DERIVATIVE


26 QUIZRR
(iv) Transesterification : This is basically a process called as „alcoholysis‰. This is carried out
by refluxing the ester with large excess of alcohol, preferably in the presence of small
amount of acid or sodium alkoxide as catalyst.

C2H5O NO  +
CH3 COO C4H9 + C2H5OH 
  CH3CO2C2H5 + C4H9OH

This is usually effective in replacing a higher alcohol by a lower one.

REACTIONS :

(1) Interconversion Reactions of Esters

O O
H2O
C C acids

R OR´ R OH
O
R2NH
C amides
base
R NR2

(2) Hydrolysis of Ester

O O
H2O
C C + R´OH
heat
R OR´ R OH

Ć Carboxylic esters hydrolyse to the parent carboxylic acid and an alcohol.


Ć Reagents : aqueous acid (e.g. H 2SO4) / heat, or aqueous NaOH/heat (known as
„saponification‰).
Ć Both are based on the formation of a tetrahedral intermediate which then dissociates.
Ć In both cases it is the C-O bond between the acyl group and the oxygen that is cleaved.

R eaction under Basic conditions :

Ć The mechanism shown below leads to acyl-oxygen cleavage


Ć This reaction is known as „saponification‰ because it is the basis of making soap from
glycerol triestsers in fats.

CARBOXYLIC ACID & DERIVATIVE


QUIZRR 27

M echanism of the base hydrolysis of Esters

:O:
Step 1 : The hydroxide nucleophiles attacks at
the electrophilic C of the ester C = O,
breaking the  bond and creating the HO: CH3 OCH3
tetrahedral intermediate.

:
:O:
Step 2 : The intermediate collapses, reforming CH3 OCH3
the C = O results in the loss of the OH
leaving group the alkoxide, leading to
the carboxylic acid.

:O:

CH3 OăH OCH3


Step 3 : An acid/base reaction. A very rapid
equilibrium where the alkoxide fast
functions as a base deprotonating the
carboxylic acid (an acidic work up would :O:
allow the carboxylic acid to be obtained
from the reaction). + HOCH3
CH3 O

Reaction under Acidic conditions :


Ć Note that the acid catalysed mechanism is the reverse of the Fischer esterification.
Ć The mechanism shown below also leads to acyl-oxygen cleavage

CARBOXYLIC ACID & DERIVATIVE


28 QUIZRR
M echanism of the Acid Catalysed Hydrolysis of Esters
:O:

Step 1 : An acid/base reaction. Since we only have a CH3 OCH3


weak nucleophile and a poor electrophile we
+
need to activate the ester. Protonation of the H

ester carbonyl makes it more electrophilic. + H


:O

CH3 OCH3
Step 2 : The water O functions as the nucleophile
attacking the electrophilic C in the C = O, with H2O
the electrons moving towards the oxonium ion,
H
creating the tetrahedral intermediate.

:
:O
CH3 OCH3
+
O:
H H

Step 3 : An acid/base reaction. Deprotonate the oxygen H2O


that came from the water molecule.
H

:
:O
CH3 OCH3
OH
Step 4 : An acid/base reaction. Need to make the -OCH3 +
H
leave, but need to convert it into a good leaving
group first by protonation. H
:

:O
+
CH3 OCH3
OH H
Step 5 : Use the electrons of an adjacent oxygen to help
„push out‰ the leaving group, a neutral
methanol molecule. + H
:O

CH3 OH + HO · CH3

Step 6 : An acid/base reaction. Deprotonation of the H2O

oxonium ion reveals the carbonyl in the


:O:
carboxylic acid product and regenerates the acid
catalyst. +
CH3 OH + H3O

CARBOXYLIC ACID & DERIVATIVE


QUIZRR 29

(3) Acyloin condensation

O R·C O
Na / Ether
2R · C
OR R · CHOH
( ă hydroxy ketone)

Na Na+ + eă
ă
O O
ă
R · C · OR + e R · C.

OR
(Radical anion)

ă ă ă
O O O O O . ă ă
ă R·C·O R·C·O
ă20R 2Na
2R · C. R·C·C·R R·C·C·R ă ă
R·C
. ·O R·C·O
OR OR OR +
2H

R·C·O R · C · OH
Tautomerises

R · CH · OH R · C · OH
ăhydroxy ketone Enediol

(4) Claisen ester condensation

O O O
 ă
EtO Na
+
2CH2 · C · OC2H5 CH3 · C · CH2 · C · OC2H5
ăketo ester
H

(5) Reaction of NH3 with ester and keto ester

O O
R·C + NH3 R·C + EtOH
OC2H5 NH2

RCOCH2CO2C2H5 + NH3 R·C CHCO2C2H5 + H2O


(1 equivalent)
NH2

CARBOXYLIC ACID & DERIVATIVE


30 QUIZRR
(6) Reactions of RLi and RMgX with Esters
2R Li O OH
or + C R · C · R´
2R Mg X R OR´´
R
Reaction usually in Et2O followed by H3O+ work-up
Ć Carboxylic esters, R´CO2R´´, react with 2 equivalents of organolithium or Grignard reagents
to give tertiary alcohols.
Ć The tertiary alcohol that results contains 2 identical alkyl groups (from R in the scheme)
Ć The reaction proceeds via a ketone intermediate which then reacts with the second equivalent
of the organometallic
Ć Since the ketone is more reactive than the ester, the reaction cannot be used as a preparation
of ketones
Ć The mechanism is an example of the reactive system type.

R eaction of RMgX with Ester O


Step 1 : The nucleophilic C in the organometallic C
reagent adds to the electrophilic C in the CH3 OEt
polar carbonyl group of the ester. Electrons CH3ă MgBr
from the C = O move to the electronegative ă + MgBr
O
O creating an intermediate metal alkoxide
complex. C
CH3 OEt
Step 2 : The tetrahedral intermediate collapses and CH3
displaces the alcohol portion of the ester as
a leaving group, this produces a ketone as +ă
O BrMg OEt
an intermediate.
C
Step 3 : The nucleophilic C in the organometallic CH3 CH3
reagent adds to the electrophilic C in the CH3ă MgBr
polar carbonyl group of the ketone. Electrons ă+
from the C = O move to the electronegative O MgBr

O creating an intermediate metal alkoxide C


complex. CH3 CH3
CH3
Step 4 : This is the work-up step, a simple acid/base H
+

reaction. Protonation of the alkoxide oxygen


H
creates the alcohol product from the O
intermediate complex. C
H3C CH3
CH3
CARBOXYLIC ACID & DERIVATIVE
QUIZRR 31

(7) Reduction of Esters

O OH

LiAlH4 + C H · C · R´ + R´´O · H
R´ OR´´
H

Ć Carboxylic esters are reduced give 2 alcohols, one from the alcohol portion of the ester and
a 1Ĉ alcohol from the reduction of the carboxylate portion.
Ć Esters are less reactive towards Nu than aldehydes or ketones.
Ć They can only be reduced by LiAlH4 and Not by the less reactive NaBH4.

For example :

O O OH
|| ||
LiAlH4 |
CH3 · C · CH2 · C · OC2 H5   CH3 CH · CH2 CH 2 · OH + C2 H5 OH

Example 20
Propose mechanism of the following esterfication reaction :
O

+
ăH O
HO · CH2CH2CH2CH2COOH

Solution :
O
O O
+
ăH
+ O
H3C · CH2 CH2 · CH2 · CH2 · C · OH HO · CH2CH2 · CH2 · CH2 · C · OH +
·H3O
H

Example 21

(a) Write a mechanism of the following reaction.

Br
NaOH
OH  O

(b) Give the mechanism for the reaction of acetyl chloride with ethyl alcohol.

CARBOXYLIC ACID & DERIVATIVE


32 QUIZRR
Solution :

Br Br
ă
OH
(a) ă
ăH2O ă ăBr
OH O O

O O O

ă O
ă O O
O
H3C Cl + H +
ăH
+ C2H5 OH H3C O H3C O
(b) +
H3C Cl H5C2 O H
+
Acetyl chloride C2H5 C2H5
Ethyl acelate

Example 22

The order of reactivity of the following esters towards hydrolysis is


O O O O
Cl Cl
C
Ph OMe Me OMe OMe OMe
Cl
(I) (II) (III) (IV)

(a) (I) > (II) > (III) > (IV) (b) (III) > (I) > (III) > (IV)
(c) (IV) > (III) > (II) > (I) (d) (IV) > (III) > (I) > (II)
Solution :
More are the electron withdrawing groups, more is the positive charge density on ÂCÊ and more
readily the attack can occur.
 (c)

Acid Amides

An amide is a composite of a carboxylic acid and an amine (or ammonia).


Heating the salt formed when an amine and carboxylic acid react drives off the water produced,
and an amide is formed.

O O
|| +
H  ||

R · C · OH + R´ · NH2   R · C · NH · R´ · H2O

Amides are much less basic than their parent amines since the lone pair of electrons on Nitrogen
are delocalized onto the carbonyl oxygen.

CARBOXYLIC ACID & DERIVATIVE


QUIZRR 33

+ H H

·
O O O O
+
H
C C + concentrated acid C C +
·
·

·
·

·
·

·
·
:

:
R N · R´ R N · R´ R N · R´ R N · R´

R´ R´ R´ R´
very weakly basic protonation on oxygen

In fact in strong acid, it is the oxygen that gets protonated first!

The C-N bond has partial double bond character, and the rotational barrier is 18 kcal/mol.

Amides of the form R-CO-NH2 are called primary amides since the nitrogen is only bound to one
carbon atom (like R-NH2 is primary).

Amides that are of the form R-CO-NHR´ are called secondary amides, or N-substituted amides.

O O O
|| || ||
R · C · NH 2 R · C · NH · R´ R · C · N · R´
primary secondary |
R´´
tertiary

Amides of the form R-CO-NR2 are tertiary amides, or N, N-disubstituted amides.

Nomenclature of Amides

To name a primary amide, identify the acid part and remove the -oic acid suffix and add amide.
E.g.

O O O
|| || ||
H3C · C · NH2 H3C · C · NH · CH2CH3 H · C · N(CH3 ) 2
ethanamide N-ethylethanamide N, N-dimethylmethanamide

To name a secondary or tertiary amide, the alkyl groups on nitrogen are treated as substituents,
and are given the prefix N (since they are the nitrogen).

Reactions :

(1) Interconversion of Amides


O

C
OH heat
N·H
H2N C +H2O

O
4-aminobutanoic acid 4-aminobutanoic acid lactam

CARBOXYLIC ACID & DERIVATIVE


34 QUIZRR
They are named by adding the word lactam to the correct IUPAC name of the parent acid.

O O

C H2O C + R´2NH

·
·

·
·
R NR´2 heat R OH

Ć Amides are the least reactive of the neutral carboxylic acid derivatives.
Ć The only interconversion reaction that amides undergo is hydrolysis back to the parent
carboxylic acid and the amine.
Ć Reagents : Strong acid (e.g. H2SO4) or strong base (e.g. NaOH) / heat

(2) Hydrolysis of Amides

O O

C H2O C + R´2NH
·
·

·
·
R NR´2 heat R OH

Ć Amides hydrolyse to the parent carboxylic acid and the appropriate amine.
Ć The mechanisms are similar to those of esters.
Ć Reagents : Strong acid (e.g. H2SO4)/heat (preferred) or strong base (e.g. NaOH)/heat.

O
||
2OH+ / H
R CONH2   R · C · OH + NH+4

OH / H2O
R CONH2   R · COO + NH3

(3) Dehydration

O
||
P2O5
R · C · NH2   R · C  N + H2 O

(4) Hoffmann Degradation Reaction


A primary amide (aliphatic or aromatic) on treatment with bromine in te presence of an
aqueous base forms an amine. The reaction is referred to as „Hoffmann reaction‰.

C6 H5 CH2 · C · NH2 + Br2 + NaOH 2H2O + Na2 CO3 + 2NaBr + C6 H5 CH2 NH2

CARBOXYLIC ACID & DERIVATIVE


QUIZRR 35

Mechanism :

O O
||  ||
(5) R · C · NH2  HNO2  R · C · OH + N2     H2 O

A cid Anhydrides

The word anhydride literally means without water, and an acid anhydride is the combination of
two molecules of carboxylic acid with the elimination of one molecule of water.

Anhydrides are also considered as activated forms of carboxylic acids, although anhydrides are
not as reactive as acid halides.

The anhydride group also inductively withdraws electron density from the carbonyl carbon, and
the carboxylate anion serves as a good leaving group.

CARBOXYLIC ACID & DERIVATIVE


36 QUIZRR
Half of the anhydride is ÂlostÊ as the leaving group, and if the carboxylic acid is very precious
(expensive or limited quantity) then this is an undesirable way of making an activated carboxylic
acid, and the acid chloride route would be more desirable.

N omenclature

(Simple) anhydrides are simply named by replacing the acid suffix of the parent carboxylic acids
with the word anhydride. E.g.

Mixed anhydrides that consist of two different acid derived parts are named using the names of
the two individual acids, with the highest priority acid being the first name.

Preparation

O
||
(1) Heating of 2 moles of R · C · OH with dehydrating agents also produces (RCO)2O

CH · COOH CH · C
150Ĉ c
O
ăH2O
CH · COOH CH · C
Maleic acid
O
Maleic anhydride

CARBOXYLIC ACID & DERIVATIVE


QUIZRR 37

Reactions :
(1) Friedel Crafts acylation

(2) Reaction with hydroxylamine

(3) Interconversion Reactions of Acid Anhydrides

Ć Acid anhydrides are the second most reactive of the carboxylic acid derivatives and
can therefore, be fairly readily converted into the other less reactive carboxylic acid
derivatives.
Ć A base in often added to neutralise the carboxylic acid by product that is formed.

CARBOXYLIC ACID & DERIVATIVE


38 QUIZRR
SOLVED EXAMPLES

Example 1

O O O
+
NaBH4 H3O
(A)
OH
The compound (A) would be
O

O O
(a) (b)
O

O O

O O
(c) (d)

O
O
Solution :

(1) NaBH4  O
CH3CO(CH2)3COOH 
(2) H3O
 CH CH(OH)(CH ) COOH 
3 2 3


(A)
-lactone
-hydroxy acid

Example 2

Which of the following compound would be expected to decarboxylate when heated ?

O O O O
(a) (b)
OH

(c) (d)

Solution : O O O


OH
In case of -keto acid, the ready decarboxylation occurs due to 6ămembered low energy transition
state formation.
 (a)

CARBOXYLIC ACID & DERIVATIVE


QUIZRR 39

Example 3
O
COOC2H5
+
C 2H5Oă/C2H5OH I) H3O
+ (A) (B)
II) 
COOC2H5
The compound B is

O O

(a) (b)
COOH

COOH COOH

O O

(c) (d)
COOEt

COOEt COOEt

Solution :
O O O
COOC2H5
+
+ C2H5Oă/C2H5OH I) H3O
COOEt
II) 
COOC2H5

COOEt COOH
Hence, (A) is correct.

Example 4
The case of alkaline hydrolysis is more for

COOCH3 COOCH3 COOCH3


COOCH3

(a) (b) (c) (d)

NO2 Cl OCH3

CARBOXYLIC ACID & DERIVATIVE


40 QUIZRR
Solution :

COOCH3

There is more electron deficiency on carbonyl carbon.

NO2

Hence, (A) is correct.

Example 5

Identify the missing reagents or products.

O
ă +
HCN/OH H2O/H conc H2SO4 I) BH3/THF
H3C (A) (B) (C) (E)
II) CH3COOH
ă
CH3 i) BH3/THF ii) H2O2/OH

(D)
Solution :

OH OH O
A= H3C CN B= H3C
CH3 OH
CH3

O O

OH HO
OH
C= D=
H3C

CH2 CH3

H3C O

E=

H3C OH

Example 6

R · CH2 · CH2OH can be converted to R·CH2CH2COOH. The correct sequence of reagents is


(A) PBr3, KCN, H+ (B) PBr3, KCN, H2
+
(C) KCN, H (D) HCN, PBr3, H+

CARBOXYLIC ACID & DERIVATIVE


QUIZRR 41

Solution :

PBr3 KCN H2O/H+


RCH2CH2OH  
 RCH2CH2 Br   RCH2 CH2CN   RCH2CH2COOH

Hence, (A) is correct.

Example 7

On subjecting mesityl oxide to the iodoform reaction, one of the products is the sodium salt
of an organic acid. Which acid is obtained ?
(A) (CH3)2C = CH · CH2COOH (B) (CH3)2CH · COOH
(C) (CH3)2C = CH · COOH (D) (CH3)2C = CH · CO · COOH
Solution :
H3C CH3 H3C OH
NaOH/I2 + CHI3

CH3 O CH3 O
Mesityl oxide

Hence, (C) is correct.

Example 8

What is the most likely product form the following reaction ?

EtONa H+ , H 2 O
CH2(COOC2H5)2 + Br(CH2)3Br   
EtOH

COOC2H5 (COOC2H5)2

(a) (b)

COOH COOH

(c) COOH (d)

Solution :

EtONa 
CH2(COOC2H5)2   CH(COOC2 H5 ) 2
EtOH

CARBOXYLIC ACID & DERIVATIVE


42 QUIZRR
ă
Br(CH2)3Br + CH(COOC2H5)2 Br(CH2)3CH(COOC2H5)2

EtONa

COOH CH2 · CH2

CH2 · COOC2H5

COOC2H5
 (d)

Example 9

What happens when :


(i) Dry chlorine is passed through acetic acid in presence of sunlight.
(ii) Formic acid is reacted with ammonical silver nitrate solution.
Solution :

Cl
(i) CH3COOH   CH2ClCOOH
sunlight

(ii) HCOOH + Ag(NH3)2NO3  Ag + 2NH4NO3 + CO2

Example 10

(a) Convert 2-chlorobutanoic acid into 3-chlorobutanoic acid


(b) Prepare malonic acid from acetic acid
(c) Prepare 4-p tolylbutanoic acid from toluene and succinic anhydride.
Solution :

H H
+ +
(a) CH3CH2CHClCOOH alc. KOH H Cl
CH3CH CHCOOH CH3CHCHCOOH CH3CHCHCOOH

Br2 / PBr3
(b) CH3COOH  CN H2O
 BrCH2COOH  NCCH2COOH   CH2(COOH)2

H2C · CH2

(c) MeC6H4Br + O AlCl3


C C O păMeC6H4CCH2CH2COOH
O
O
Zn/Hg, HCl
păMeC6H4CH2CH2CH2COOH

CARBOXYLIC ACID & DERIVATIVE


QUIZRR 43

Example 11

Reductive ozonolysis of (A) having molecular formula C11H12O gives CH2O and (B) with
molecular formula C10H10O2. Both (A) and (B) gives a precipitate from reaction with NaOI.
(B) on reaction with NaOI gives a precipitate of CHI3 along with the formation of compound
(C) as sodium salt. Compound (C) on acidification gives a compound which on mononitration
gives only one product. Identify compound (A), (B) and (C).
Solution :
Compounds (A) and (B) responding positively to iodoform reaction reveals that they have either
CH3COă or CH3CH(OH)ă structural unit. Compound (A) has 6Ĉ of unsaturation while (B) has 5Ĉ
of unsaturation. Compound (A) has a benzene ring (according for 4 degree of unsaturation), a
CH3COă unit and a double bond in the side chain (accounting for rest two degree of unsaturation).
CH3COă unit and side chain with a double bond (= CH2 unit) must be at para position since (C)
on mononitration gives only one product.

O CH3
CH3 · C CH2 C

O3/H2O + CH2 O
Zn

C C
O CH3 O CH3
(A) (B)

NaOI

CO2Na CO2H CO2H

NO2
+
2H HNO3/H2SO4
2CHI3 +

CO2Na CO2H CO2H


(C)

CARBOXYLIC ACID & DERIVATIVE


ALDEHYDES
AND KETONES
QUIZRR 3

ALDEHYDES AND KETONES


Ć Aldehydes and ketones are first oxidation products of alcohols and carry carbonyl group (
> C = O) in their molecules.

Ć In aldehyes >CO group must also carry one H atom linked to C while in ketones no H atom
should be linked to C atom of CO group. Thus functional group of aldehydes is · CHO

|
while that for ketones is · CO .

Ć In aliphatic aldehydes, · CHO group is linked to alkyl (Group R · CHO) while in aromatic
aldehyde, · CHO group is linked to aryl group (Ar · CHO).
Ć In aliphatic ketones, the CO group is linked to alkyl groups (RCOR) while in aromatic
ketones, the CO group is linked to aryl groups (ArCOAr).
Ć In simple ketones, the alkyl or aryl groups linked to CO group are same but in mixed
ketones, the alkyl or aryl groups linked to CO group are different.
Ć Aldehydes and ketones are functional isomers of each other and can be represented by the
formula CnH2nO.

Nomenclature

Ć Common names of aldehyde is derived from the corresponding carboxylic acids by replacing
the suffix ic acid with aldehyde.
Ć Common name of ketones is written by using the names of the alkyl groups followed by the
word ketone.
Ć IUPAC name of aldehydes is alkanals while that for ketones is alkanones.
Ć Aldehydes are isomeric with ketones, unsaturated ethers and unsaturated alcohols, cyclic
ethers as well as cyclic alcohols.
O
||
HCHO CH3CH2CHO CH3 · C ·CHO

formaldehyde propionaldehyde pyruvaldehyde

COCH3

CH3 ă CH = CH ă CHO CH2 = CH ă CHO


Crotonaldehyde acrolein Acetophenone

ALDEHYDES AND KETONES


4 QUIZRR

CHO O

C CH2 · CO · CH3

Benzaldehyde Benzophenone Benzyl Methyl ketone

COCH2CH3 O O O

C C C C

OH
Propiophenone Benzil Benzion

Preparation of Aldehydes & Ketones

1. From Alcohols

(a) By Controlled oxidation using


K2Cr2O7/H2SO4 or KMnO4 / H2SO4

(i) aldehydes from 1Ĉ alcohols

[O] [O]
RCH2OH 
 RCHO 
 RCOOH

(ii) ketones from 2Ĉ alcohols

[O]
R 2CHOH 
 R 2CO + H2O

Ć Primary alcohols can be oxidised to aldehydes (or further to carboxylic acids).


Ć In aqueous media, the carboxylic acid is usually the major product.
Ć PCC or PDC, which are used in dichloromethane, allow the oxidation to be stopped at
the aldehyde.

OH O O
[O] [O]
R C H C C
[R] [R]
R H R OH
H
1Ĉ alcohol aldehyde

ALDEHYDES AND KETONES


QUIZRR 5

Ć Secondary alcohols can be oxidised to ketones, no further;

OH O
[O]
R C R´ C No reaction
[R]
R H
H
2Ĉ alcohol ketone

Ć Tertiary alcohols cannot be oxidised

OH
[O]
R C R´ No reaction

R´´
3Ĉ alcohol

Oppenauer Oxidation :

Using a specific oxidising agent like Aluminium tertiary Butoxide {[(CH3)3C · O]3Al} in
presence of Acetone, secondary alcohols can be oxidised to ketones.

R R
[C(H3)3ăCăO]3 AL
CHOH C=O
acetone
R R

(b) Cr oxidation of alcohols.

OH O
CrO3 in glacial acetic acid

Pyridinium dichromate (PDC) or Pyridinium Chlorochromate (PCC) in anhydrous media


such as CH2Cl2 oxidises primary alcohols to aldehydes and secondary alcohols to ketones.

O
PDC or PCC
RCH2OH R C H
CH2Cl2
O
PCC
RCHR´ R C R´
CH2Cl2
OH
ALDEHYDES AND KETONES
6 QUIZRR
Example 1

Identity A and B in the following

(1) alk. KMnO4 Cr2O72ă


A B
warm

Solution :
OH OH
A , B
OH O

oppenauer
(2) CH2 = CH · CH · CH3 A
oxidation
OH

Solution :

A CH2 = CH · C · CH3

(3) Suggest suitable reagents for the following conversions :

(a) CH 3 CH 2 CH 2 CH 2 CH 2 OH  CH 3 CH 2 CH 2 CH 2 CHO

OH O

(b) H3C CH3 H3C CH3

Solution :

Cu or CuO
(a) CH3CH2CH2CH2CHO

(b) K2Cr2O7,H´
H3C CH3

ALDEHYDES AND KETONES


QUIZRR 7

(4) In acylium ion, the structure R · C  O+ is more stable than R · C+ = O. Explain.


Solution :
Both C and O are non metals and try to complete their octet. In R · C  O+ each has complete
octet whereas in R · C+ = O, carbon atom has incomplete octet.

2. From Acid Chlorides (RCOCl) :

(a) For aldehydes


By reduction with H2 in the presence of Pd/BaSO4 in Xylene.

Pd/BaSO4
RCOCl + H 2  RCHO + HCl
Xylene

Ć This reaction is called Rosenmund’s Reaction.


Ć BaSO4 poisons the catalyst and reduce its efficiency. Thus, it checks the further
reduction of aldehydes to alcohols.
Ć If LiAlH4 is used as a reducing agent, the produced is an alcohol.

LiAlH
R · COCl or Ar · COCl 
4
RCH2OH or ArCH2OH

Note : Formaldehyde cannot be prepared by this method because formyl chloride is


highly unstable and non-existing.
(b) For ketones : Acid chlorides on reaction with lithium organocuprates, R2 CuLi or Ar2CuLi
yield ketones. Here, the R part of organocopper compound acts as nucleophile and displaces
Cl of acid chloride to undergo nucleophilic substitution.

2 Li CuX
R´X or ArX 
 R´Li or ArLi 
 R´ 2 CuLi or Ar2 CuLi
 LiX  LiX

R´2 CuLi + 2RCOCl  2RCOR  CuCl  Li

Ć Grignard reagent can also react with acid Chlorides, but the product is tertiary alcohols
because the Ketone produced reacts with additional RMgX. This shows that organo
copper reagents are less reactive than grignard reagents towards the carbonyl group
of ketones and the reaction stops at the ketone formation stage.
This low reactivity shows that the organo copper compounds donot react with the
functional groups with which organomagnesium and organo lithium reagents react.

Thus, the presence of some functional groups (like ă NO2 , ă CN, ă CO ă , ă CO2 R etc.)
does not interfere with the synthesis of ketone.

ALDEHYDES AND KETONES


8 QUIZRR
Example :

CH3 · C · CH2 · C · Cl + (CH3)2 CuLi CH3 · C · CH2 · C · CH3

O O O O

3. FROM CALCIUM SALT OF CARBOXYLIC ACID (PYROLYSIS)

Formal dehyde is formed when calcium salt of formic acid is dry distilled.

dry distillation
(HCO2 )2 Ca 
 HCHO + CaCO3

RCOO HCOO
Ca + Ca 2RCHO + HCHO + 2CaCO3 + RCOR
RCOO HCOO

Ketones are formed when calcium salt of monobasic acid (other than formic acid) is heated.

R COO R
Ca C = O + CaCO3
RCO O R

Illustration

CH2 C Oă CH2
Ca2+ C=O
ă
CH2 C O CH2 Cyclopropanone

Ć Instead of using calcium salt of an acid vapours of an acid or mixture of acids can be passed
over heated MnO at 300ĈC.

MnO
CH3 COOH + HCOOH 
 CH3CHO + CO2 + H2O
300Ĉ C

MnO
2CH3COOH 
 CH3 COCH3 + CO2 + H2O
300Ĉ C

MnO
CH3 COOH  CH3 CH2 COOH 
 CH3COCH2CH3 + CO2 + H2 O
300Ĉ C

ALDEHYDES AND KETONES


QUIZRR 9

Ć From a cetoa cetic ester

O O

CH3 · C · CH2 · C · OC2H5

· C2H5OH C2H5ONa

O O
ă
CH3 · C · CH · C · OC2H5
Rx
ăNax

O O

CH3 · C · CH · C · OC2H5

R
Hydrolysis
(ăEtOH)

O O

CH3 · C · CH2 · R CH3 · C · CH · COOH


· CO2
R

4. From alkenes

(i) By reductive ozonolysis


Aldehydes can be prepared by the ozonolysis of alkenes of the type R · CH = CH · R in
presence of Zn (Reductive Ozonolysis)

O
O3
R · CH = CH · R R · CH CH · R

O·O

H2O/Zn

2RCHO + ZnO + H2O

Ketones can also be prepared by the ozonolysis of alkenes of the type R2 C = CR2

ALDEHYDES AND KETONES


10 QUIZRR
O
O3 H2O
R2C = CR2 R2C CR2 2RCOR + H2O2

O·O

(ii) Oxo process : It involves the conversion of alkenes to aldehydes with one C atom more
than parent alkene.

[CoH(CO) ]
R · CH = CH2  CO + H2 
4  R · CH · CH · CHO
2 2
{Watergas}

(iii) WackerÊs Process

CuCl
CH2 = CH2 + PdCl2 + H2O 
2  CH CHO + Pd + 2HCl
3

CuCl
CH3 · CH = CH2 + PdCl2 + H2O 
2  CH COCH + Pd + 2HCl
3 3

5. From geminal dihalides :

By alkaline hydrolysis, the geminal halides gets converted into aldehydes and ketones.

OH
CH3CH2CHCl2 + 2NaOH CH3CH2CH
OH
· H2O

CH3 CH2 CHO

Cl

CH3 · C · CH3 + 2NaOH CH3 · C · CH3


HO OH
Cl

· H2O

CH3 · C · CH3

O
Illustration :

Cl2/h H2O
Br CH3 Br CHCl2 Br CHO
2eq

P-bromo
benzaldehyde

ALDEHYDES AND KETONES


QUIZRR 11

6. From Alkynes

(1) Hydration :
Hydration of alkynes gives ketones (except CH  CH that gives CH3CHO)

H O/H+
CH  CH 
2
 CH3CHO
HgSO4

H O/H+
CH3 · C  CH 
2
 CH3COCH3
HgSO4

(2) Hydroboration ă oxidation


Hydroboration of a non-terminal alkyne followed by oxidation of the intermediate yields a
ketone but terminal alkyne yields aldehyde.

BH3.THF
CH3 C C CH3 CH3 CH = C · CH3 B
3

ă
H2O2/OH

tautomerises
CH3 CH2 C CH3 CH3 CH = C · CH3

O OH

BH3 THF
CH3 C C CH3 CH3 CH = C · B
3
H
ă
H2O2 / OH

tautomerises
CH3 CH2 CHO CH3 CH = CHOH

Example 2

HgSO / H SO
4 4 BD . THF
B 
2
 CH 3 CH 2 C  CH 
3
 A identity A & B.
H 2O 2 , OH

Solution :

A  CH3 · CH2 · CH · CHO B  CH3CH2COCH3


|
D

ALDEHYDES AND KETONES


12 QUIZRR
Example 3

4 HgSO / H SO
4 BH . THF
B 
2
 CH 3 CH 2 CH 2 C  C · CH 3 
3
 A
H 2O 2 , OH

Solution :

A  CH3CH2 CH2 CH2 COCH3 , B  CH3CH 2CH 2CH2 COCH2CH3

7. From Nitriles

(i) StephenÊs Reduction (only for aldehydes)

SnCl2/HCl
R·C N R · CH = NH.HCl
Aldiminie hydrochloride

H2O

RCHO + NH4Cl

The formation of aldimine hydrochloride is unstable and hydrolyse to give aldehydes.

(ii) Treatment with Grignard Reagent (for ketones)

R´ O
RMgX 2H2O
R´ · C N R´ · C · R + NH3 + Mg(OH)Br
R · C = N·MgBr

(iii) Using di-isobutyl aluminium hydride (DIBAL ă H) to imines and then to by aldehydes

1. AlH (iso ă Bu)


RCN 
2
 R · CHO
2. H2O

Example 4

(i) MgBr
(1) H·C N CHO
(ii) H2O

O
(i) MgBr
(2) CH3CN C · CH3
(ii) H2O

ALDEHYDES AND KETONES


QUIZRR 13

Mg (i) CH3CN
(3) + NBS A B C
ether (ii) H2O/H+

Ans. A Br , B MgBr , C C · CH3

O
||
(4) Convert CH 3 CH 2 CH 3 to (CH 3 )2 CH · C · CH (CH 3 )2

Ans.
Br2, Mg/ether
CH3CH2CH3 CH3 · CH · CH3 CH3 · CH · CH3

Br MgBr

O
+
H3O
(CH3)2 CH · C · CH · (CH3)2
CH3 · CH · CH3

CN

8. From Dialkyl cadmium :

(Only for Ketones)

O O
|| ||
R´ · Cd · R´  2R · C · Cl  2R · C · R´ + CdCl2

9. From Lithium Dialkyl cuprates

(Only for ketones)

O O
|| ether
||
R2 · CuLi  2R · C · Cl   2R · C · R´

10. Cleavage of Diols

By oxidative clearage of 1, 2 diols (glycols) by periodic acid (HIO4) or by lead tetra acetate, both
aldehyde and ketones may be produced.

ALDEHYDES AND KETONES


14 QUIZRR

H H
| | HIO4
C6 H5 · C · C · C6 H5   2C6 H5 CHO
| |
OH OH

Preparation of aromatic aldehydes and ketones

1. Oxidation of toluene
(a) Using chromyl chloride CrO2Cl2

Cr O2Cl2
C6H5 · CH3 
 C6H5CHO
H2O
Benzaldehyde

This is called EtardÊs Reaction


Note : if side chain is bigger than · CH3 group, then terminal C is changed to · CHO
group. For example,

Cr O Cl
2 2
C6H5 CH2 CH3   C6H5CH2CHO
Phenyl ethanal

(b) Oxidation using CrO3 in acetic anhydride

CrO3[O]
C6H5 · CH3 [C6H5CH(OCOCH3)2]
(CH3CO)2O

H2O

C6H5CHO + 2CH3COOH

2. Gattermann Koch Reaction :


This involves formylation of aromatic ring with CO.
CHO
AlCl3
+ CO
HCl

3. Gattermann Aldehyde Synthesis

HCl CH NH H2O CHO + NH3


+ HCN
AlCl3

ALDEHYDES AND KETONES


QUIZRR 15

OH
OH
(i) AlCl3/HCl
+ HCN + NH4Cl
(ii) H2O
CHO

OH OH
This process involves the treatment of benzene or other aromatic compounds having activating
groups like, · OH, · OR etc.

4. Friedel-Crafts Acylation of Benzene


O
O
CR
+ RCCl + HCl
AlCl3

Ć Electrophilic species : the acyl cation or acylium ion (i.e. RCO+) formed by the ÂÂremovalÊÊ of
the halide by the Lewis acid catalyst.
Ć The acylium ion is stabilised by resonance as shown below. This extra stability prevents the
problems associated with the rearrangement of simple carbocations :

+ +
CH3C O CH3C O

Ć The reduction of acylation products can be used to give the equivalent of alkylation but
avoids the problems of rearrangement
Ć Friendel-Crafts reactions are limited to arenes as or more reactive than mono-halobenzenes.

Mechanism
The acyl halide reacts with the Lewis acid to form a more electrophilic C, an acylium ion

O O

CH3C · Cl AlCl3 CH3C


H
ă +
Cl · AlCl3

O
fast
ă
CH3C+ Cl · AlCl3 O

AlCl3 + CH3C
+
slow
HCl

ALDEHYDES AND KETONES


16 QUIZRR
For example :

C · Cl
An hydrous
C
+
O AlCl3
O

5. Reimer ă Tiemann Reaction

When phenol is treated with chloroform and aqueous NaOH/KOH, an aldehyde group is introduced
in the aromatic ring, generally at the ortho position.

OH ONa ONa

CHCl2 CHO
CHCl3, aq. NaOH 1. 2OHă/ă2Clă
2. ăH2O

ăNaCl HCl

OH

CHO

Salicyldehyde

PROPERTIES OF ALDEHYDES AND KETONES


Physical Properties
Ć Formaldehyde is gas but other aldehydes are liquids at room temperature.
Ć Lower ketones are colourless liquids while higher ketones are colourless solids.
Ć Boiling points of ald/ketones are relatively higher than hydrocarbons but lower than alcohols
of comparable molecular mass.
Ć Boiling points of ketones are more than the corresponding isomeric aldehydes.
Ć Lower aldehydes / Ketones (up to 4 carbons) are soluble in water but higher aldehydes /
ketones are almost immiscible with water.

Chemical Properties

Ć Aldehydes and ketones owe their highly reactive nature to the polar carbonyl group
+ ă
C = O in their molecules.

ALDEHYDES AND KETONES


QUIZRR 17

Ć The electron deficient carbon atom of CO group readily undergo the nucleophilic attack for
initiation of reaction.
Ć The general mode of reaction being :

ă
O OH
ă +
H
C O + Z C C
(Nuclephile)
Z Z
(Intermediate)

Ć Crowding at the carbonyl carbon and presence of electron releasing group hinderÊs the
nucleophilic attack on the carbonyl carbon and decreases the reactivity of the compound. For
example, crowding as well as electron releasing effect increases from formaldehyde to other
aldehydes to ketones. Hence, reactivity decreases as follows.

H R R
C O > C O > C O

H H R

Ć The presence of electron withdrawing groups at the carbonyl carbon increases the reactivity.
Ć The attachment of benzene ring to CO group decreases the reactivity because C = O bond
comes in conjugation with aromatic ring and resonance stabilisation of molecule occurs.
Ć CO group withdraws electron density from the ring. Hence it becomes less susceptible to the
electrophilic substituion.
Ć Ring substitution in benzaldehyde occurs at m-position.

REACTIONS

I. ADDITION REACTIONS

1. Addition of NaHSO3

C O + NaHSO3 C·O

SO3Na
(Sodium bisulphite)

Mechanism :

O O
ă
H2O + H · O · S · O ă S · Oă
H3O+ + O · ..

ALDEHYDES AND KETONES


18 QUIZRR

2 2
Ć Here SO3 is a stronger base than SO3 because of more electron density.

2
Hence, SO3 will be the attacking species


ă
C O + S O ·C·O · C · OH
+
Oă S SO3
ă

Oă Oă
O

Note : Hindered ketone does not give the reaction.

H O H
NaHSO3
(a) H3C · C · C · C · CH3

O CH3

Hindered

C NaHSO3
(b)

Hindered

2. Addition of HCN

C O + HCN · C · OH

CN
(cyano hydrin)
Mechanism :


  
 H  CN
HCN 

Note :
(1) As done in the case of NaHSO3, here also hindered ketone does not give the reaction.

HCN
Ph · CO · Ph

ALDEHYDES AND KETONES


QUIZRR 19

Reduce
CH2N2
H2/Li or LiAlH4
(2) ·C N·
· COOH
Hydrolysis

Illustration : R R

C O + HCN C · OH

H H CN

+
H3O (Partial hydrolysis)

OH OH
+
H 3O
R · CH · COOH R · CH · CONH2
complete
hydrolysis

3. Addition of GriguardÊs reagent (RMgX)

OMgX OH
RMgX H2O C
C O C

R R

Ć Aldehydes form 2Ĉ alcohol (except HCHO which gives RCH2OH) (1Ĉ alcohol) while
ketones form 3Ĉ alcohol.

4. Addition of Alcohols
R R OH
dryHCl
C O + R´OH C (Hemiacetal)

R R OR´

ăH2O R´OH

R OR´
+
H3 O
RCHO + 2R´OH C
fast

R OR´
(acetal)

ALDEHYDES AND KETONES


20 QUIZRR
Ć Hemiacetal is very reactive and easily forms acetal
Ć The above reaction is not so favourable with ketones but can be favourable if 1, 2 or 1, 3-
diols are used. With diols, cyclic ketals are formed.

R HO · CH2 R O · CH2
+
H
C O + CH2 C + H2O

R HO · CH2 R O · CH2

Note : ketals are formed only by unhindered ketones.


Ć Acetals or ketals are stable in neutral or basic conditions but in acidic medium, they undergo
acid catalysed cleavage similar to that of ethers to regenerate carbonyl compounds. Thus
this reaction can be used to protect carbonyl groups.

For example

O
O O
CO2Et CH2 · OH
CO2Et
CH2 · OH

·EtOH LiAlH4

O
O O
CH2OH CO2OH
Aqueous acid

5. Addition of water
+
OH2 OH
IMPE
C O + H2O C C
ă
O OH
(gemdiol)

The rate of hydration decreases due to the presence of electron donating substituents while
electron withdrawing substituents increase the rate.

ALDEHYDES AND KETONES


QUIZRR 21

Some examples of stable hydrated


H
Cl Cl O
O
(a) Cl · C · C + H2O Cl · C C·H
H Cl O
Cl

This is stable due to the intramolecular H-bonding.

H H
O O O

(b) Ph C Ph O C O
+ H2O
C C C C

O O Ph Ph

OH
(c)
O + H2O
OH

The diving force of this reaction is the relief in angle strain on going from carbonyl compound
(C – C – C bond angle = 60°, compar ed wit h nor mal sp2 bond angle of 120Ĉ) to hydrate
(C ă C ă C bond angle = 60Ĉ, compared with normal sp3 bond angle of 109.5Ĉ.)

6. Addition of ammonia derivatives

Slightly
C O + H2N · Z acidic
C N · Z + H2O

(pH controlled reaction)

ALDEHYDES AND KETONES


22 QUIZRR
Mechanism :
+ +
C O + H C O ·H

H·N·H

H
Intramolecular
proton exchange · C · OH
·C·O·H
+ +
H·N·Z
H·N·Z
H
ăH2O
H
Z
+
·C N·Z C N

Ć pH should be controlled carefully. Medium should be neither too acidic nor too basic.
Ć HCHO reacts with NH3 differently forming UROTROPINE (hexamethylene tetraamine)
N

6HCHO + 4NH3 (CH2)6N4 + 4H2O N N


N
Ć The reaction of ammonia with aldehydes and ketones forms imines, but the products are
unstable and cannot usually be isolated. If a secondary amine is used, an enamine is formed.
An enamine has an amino group bonded to one of the carbons of a carbon-carbon double
bond and can be said to form when enolic form of a carbonyl compound reacts.
The table give below uses the following abbreviations (A, B, C)
H+ OH
C = O + NH2G C C = NG
(A) NHG (C)
(B)

Table : Reactions of NH2G with carbonyl compounds


G A B C
·H NH3 OH C == NH
C
CH3CH == NH
NH2
aldimine

CH3
C NH
CH3
ketimine
ALDEHYDES AND KETONES
QUIZRR 23

OH
· OH NH2OH C == NOH
C
hydroxylamine oxime
NHOH

OH
· R RNH2 C == NR
C
(aliphatic) (Schiff base) azomethine
NHR

OH
· NH2 NH2NH2 C C == N NH2
hydrazine hydrazone
NHNH2

OH
· NHC6H5 NH2NHC6H5 C == NNHC6H5
C
phenyl hydrazine phenylhydrazone
NHNHC6H5

· NHC6H3(CO2)2 NH2NHC6H3(NO2) 2 OH C==NNHC6H3 (NO2)2


C
2, 4-dinitro phenyl 2, 4-dinitro phenylhydrazone
hydrazine (2 : 4 DNP) NHNHC6H3(NO2)2

OH
· NHCONH2 NH2NHCONH2 C == NNHCONH2
C
semicarbazide semicarbazone
NHNHCONH2

In aqueous acidic solution, an enamine is hydrolysed back to the carbonyl compound and secondary
amine.

O OH N(CH3)2
(CH3)2NH
CH3 · C · CH3 CH3 · C · CH2 CH3 · C · CH2

N(CH3)2 O
+
H3 O
CH3 · C CH2 CH3 · C · CH3 + (CH3)2NH

ALDEHYDES AND KETONES


24 QUIZRR
Example 5

O
+
H3 O
CH3MgBr + CH2 CH · C · H X. X is

OH

(a) CH2 CH · C · H (b) CH2 · CH CH · CH3

CH3 CH3

(c) CH 3 · CH 2 · CH 2 · CHO (d) None is correct

Solution :
O

CH2 CH · C · H is conjugate system


(I)
OH
+
H 3O
hence (I) + CH3 · Mg · Br CH3 · CH2 · CH C·H

tautomerises

CH3 · CH2 · CH2 · C · H

Hence, (C) is correct.

Example 6
O
HgSO4/H2SO4
C CH A

A is

O O

(a) C · CH3 (b) CH2CHO

O O

(c) CH2COOH (d) CH2CHO

ALDEHYDES AND KETONES


QUIZRR 25

Solution :
C = O is (EWG) hence  electron transfer is towards ring.
O

C CH

Hence, (B) is correct.

Example 7
End product of the following sequence of reactions is

CH MgBr
2 2 4 2 4 CO /H O+ HgSO / H SO Ag 2 O
CH  CH 
3     

(a) CH3 · C · COOH (b) CH2(COOH)2

O O

(c) CH3 · C · CHO (d) H · C · CH3COOH

Solution :

CH CH + CH3MgBr CH C · MgBr

O2 /H3 O+

· COOH is (EWG) CH C COOH

HgSO4/H2 SO4

OHC · CH2 · COOH


Ag/O

HOOC · CH2 · COOH

CH3COOH

Hence, (B) is correct.

ALDEHYDES AND KETONES


26 QUIZRR
II. Reaction with PCl5 or SOCl2 / pyridine

Cl

C O + PCl5 C + POCl3

Cl
Dichloroalkane

Ć The products are geminal dihalides.

III. Reaction with SeO2

Acetic
CH3CHO + SeO2 
 OHC · Se + H2O
acid

O
Acetic
||
CH3 COCH2CH3 + SeO2   CH3CO C CH3 + Se  H2O
acid

The reaction is only given by methylene group adjacent to the carbonyl group.
IV. Witting Reaction
It is a reaction used for converting carbonyl compounds to alkenes

R1 R3 R1 R3

C O + Ph3P C C C + Ph3P O
R2 R4 R2 R4
phosphorus ylide

V. Oxidation
Ć Aldehydes are easily oxidised to carboxylic acids with same number of C atoms.
[O]
RCHO RCOOH
Ć Not only strong oxidizing agents (KMnO4, K2Cr2O7) but even mild oxidants like Ag+, Cu2+
etc., can oxidise aldehydes. Thus aldehydes are very powerful reducing agents.
(a) TollenÊs reagent : This is also called silver mirror test. It is reduced by aldehyde to
Ag.

RCHO + 2[Ag(NH3)2]OH RCOONH4 + 2Ag() + 3NH3 + H2O


Tollen's reagent (silver mirror)
{colour less solution}

Note : ketones do not react with them, hence it is used in differentiating aldehydes from
ketones.

ALDEHYDES AND KETONES


QUIZRR 27

(b) FehlingÊs solution : Aldehydes (except benzaldehyde) reduce fehlingÊs solution (Cu2+
reduced to Cu+) ion complexed with tartarate ion.

 
RCHO + 2Cu2+  3OH  RCOO   2Cu ()  2H2O
Red ppt.

(c) Adehydes also reduce Benedict’s solution (Cu2+ complexed with citrate ion) to Cu+
(d) Oxidation of Ketones
In general, oxidation of ketones give a mixture of carboxylic acid.
Ć During oxidative cleavage of mixed ketones, the lower alkyl group is retained by
carbonyl group. This is known as Popoff’s Rule.

O
K2 Cr2 O7
CH3CH2CH2 · C · CH3 CH3CH2COOH + CH3COOH
H2SO4
propionic acid acetic acid
2 - pentanone

(e) Oxidation with sodium hypohalite

Ć The reaction is given by aldehydes or ketones having CH3 · C · group


||
O

Ć Hypohalite is a selective oxidizing agent as it does not attack carbon carbon


double bonds.
Ć This is also called haloform reaction

O O
|| ||
R · C · CH3  3NaOI  R · C · O Na   CHI3 ()  2 NaOH
ă

(yellow ppt.)

(f) Baeyer ă Villiger Oxidation


ketones are oxidised by caroÊs acid (H2SO5) or peroxy benzoic acid (C6H5CO3H) to
esters.

H2 SO5
RCOR´ + O RCOOR´
O O

C6H5CO3H O
+ O

ALDEHYDES AND KETONES


28 QUIZRR
C6H5CO3H
Ph · C · CH3 PhO · C · CH3

O O

Ć The insertion of oxygen atom takes place between the carbonyl group and the
group having greater migrating tendency.

Example 8

Halogen acids HX easily add to > C = C < bond but they do not add on > C = O bond.
Why ?
Solution :
The high degree of polarity in HX as well as in > C = O bond shows the easy addition of HX on
> C = O bond but as soon as addition products are formed, the product loses HX to show the
backward reaction.

OH
O + HX O
Decomposes ă HX
X

Example 9

Presence of acids and bases activates carbonyl compounds for reaction. Explain.
Solution :
Presence of acid intensifies the partial positive charge on carbonyl carbon and hence activates the
group.

+ ă
+ +
O + H O H

Presence of base activates a-methylene component of the carbonyl compounds by converting them
in carbanions.
ă
R · CH2 · CHO + : B  R · CH · CHO + BH

Formation of DDT

Chloral (trichloro acetaldehyde) on reaction with chlorobenzene in presence of H2SO4 gives dichloro
diphenyl trichloroethane (DDT). The product is used as an insecticide for killing insects and
mosquito.

ALDEHYDES AND KETONES


QUIZRR 29

It is used in household under the name DDT

Cl

O Cl Cl + H2O
H2SO4
Cl3C · C + 2 Cl · C · C · H
H
Cl

Cl
DDT

VI. REDUCTION
(a) Mild Reduction
Ć This is carried out by reducing agents like H2/Pt, H2/Pd, LiAlH4, NaBH4 etc. and converts
CO to CHOH

C O C
OH
Alcohol

Ć Aldehydes give 1Ĉ alcohol and ketones give 2Ĉ alcohols.

Ć NaBH4 and LiAlH4 reduce C = O bond without reducing C = C bonds in the compound.

Ć However, LiAlH4 reduced the carbon-carbon double bond, which is in conjugation with
carbonyl group only when the -carbon bears an aryl group.

O
LiAlH4
CH CH · C · H Ph · CH2CH2 · CH2OH

NaBH4
Ph · CH CH · CH2OH

O
H2/wilkinsons catalyst
Ph · CH2 · CH2 · C · H
H2/Ni
Ph · CH2CH2CH2OH

ALDEHYDES AND KETONES


30 QUIZRR
(b) MPV reduction : This is Meervein ă Ponndroff ă Verley reduction and it involves reduction of
C = O group to CHOH without affecting the other reducible group in the molecule. The reagent
used are, aluminium isopropoxide and isopropyl alcohol.
CH3
Al[OCH(CH3)2]3
C O + (CH3)2CHOH CHOH + C O

isoproply CH3
alcohol Alcohol Acetone

(c) Strong reduction :

C O CH2

Ć Various reducing agents employed are Zn ă Hg/HCl (Clemmensen’s reduction); NH2 · NH2
/ KOH (wolf ă Kishner reduction); Na/alcohol; HI/red P.

NH2 · NH 2 Zn(Hg) + conc. HCl


C N · NH2 C O CH2
Wolf-Kishner Clemensen

Glycol, KOH

CH2

CHO CH3
O
Wolf - Kishner
or
Clemensen

COOC2H5 COOC2H5

H2/Ni

CH2OH
HO

COOC2H5
ALDEHYDES AND KETONES
QUIZRR 31

Note : NaBH4 also does not reduce esters or acid chlorides but LiAlH4 reduces them to
alcohols.

(d) Reduction to pinacol


CH3
CH3
CH3 · C O
Mg,C6H6
+ H3C · C · O
CH3 · C O Mg
H3C · C · O
CH3
CH3 CH3

H3C · C · OH H2O

H3C · C · OH

CH3
(pinacol)

VII. CANNIZARO’S REACTION


2 moles of aldehydes Conc. basic Undergo disproportionation to produce one
 
containing no -hydrogen medium mole of alcohol and one mole of carboxylic acid
(OH )

(1) 2HCHO + NaOH  CH3OH + HCOONa

(2) 2PhCHO + NaOH  C6 H5 CH2OH + C6 H5COONa

ă
Mechanism : O O
ă
H·C·H + OH H·C·H

OH
RDS

O O

ă ă
O H C · OH + H H·C·H

H·C·H

H
{This is a strong base, hence it abstracts proton from acid}

ALDEHYDES AND KETONES


32 QUIZRR
ă
O O O

H · C · H + H · C · OH C · Oă + CH3OH

H H

R ocking Illustration :

ă
OH
1. CCl3 CHO 
 ?

ă
O O
ă
OH
Cl3C · C · H Cl3C · C · H

OH

{Here CCl 3 is more stable than Hă, hence Hă is not removed, thus cannizaro reaction does

not occur}

O O
ă ă
H C · OH + CCl3 CHCl3 + H · C · O

ă
OH
2. CH C · CHO ?

ă
O O
ă
OH
CH C·C·H CH C·C·H

OH
ă
{in this case also, CH C is more stable than H ă }
ă
Hence, products are HC  CH and H COO

Cross Cannizaro’s Reaction

HCHO + PhCHO + NaOH  PhCH2OH + HCOONa


(conc.)

This reaction yields benzyl alcohol and not benzoic acid because carbonyl carbon of
formaldehyde is more electrophilic than that of benzaldehyde.

ALDEHYDES AND KETONES


QUIZRR 33

So, ăOH finally attacks at formaldehyde (steric affects can also be said to have a major role in
this attack).

Note : the presence of electron withdrawing substituent increases the rate of cannizaro reaction
while electron releasing substituent decreases the rate.

I ntramolecular Cannizzaro

Glyoxal on reaction with conc. NaOH gives 2-hydroxy ethanoate by intramolecular cannizaro
reaction. The product is a -hydroxy acid.

O O H O
ă
Conc. OH ă
Ph · C · C · H Ph · C · C · O

more
OH
reactive

Rocking Illustration

conc.
CH2 = CH · CH = O 
ă
 ?
OH

Solution :
ă
Conc. OH ă
CH2 CH · CH O CH2 · CH CH · O
O
H
HOH

ă ă
OH
O · CH2 · CH2 · CHO H O · CH2 · CH2 · CH O

ă
{Here CHO · CH2 is a good leaving group
as negative charge is in conjugation}

O
ă
HCHO + CH2 · C · H

HOH

CH3CHO

ALDEHYDES AND KETONES


34 QUIZRR
Example 10
Cannizzaro
2CD2 = C + OH   X and Y (Y is alcohol, D is deuterium)
X and Y will have structure :

O D O D
ă ă
(A) D · C · O , D · C · OH (B) D · C · O , D · C · OH

D H

O D
ă
(C) D · C · O , D · C · OH (D) None is correct

Solution :
D D D
D D
ă ă ă
D·C O + OH O·C·D + C O O C + D·C·O

OH D D D
Proton
transfer

D
D
O C + D · C · OH
ă
O D
Hence, (A) is correct.

VIII. CONDENSATION REACTIONS :


(1) Aldol Condensation : This is shown by aldehyde / ketones having atleast one  ă H atom. Here,
 ă C of one of the molecules joins with the carbonyl carbon of the other.
(a) O OH
Dil NaOH  
CH3 · C · H + H · CH2CHO CH3 · C · CH2CHO

H
Acetal

ăH2O Heat

CH3 · CH CH · CHO
(Croton aldehyde)

ALDEHYDES AND KETONES


QUIZRR 35

(b) O OH O
ă
Dil OH  
CH3 · C · CH3 + H · CH2COCH3 CH3 · C · CH2 · C · CH3

CH3

Heat
O

CH3 · C CH · C · CH3

CH3

Ć The aldol reaction is more favourable for aldehydes than for ketones because of more
acidic -Hydrogen atoms and more electrophilic carbon.
Ć The dehydration product is ,  ă unsaturated carbonyl compound due to the conjugated
structure formed.

O OH O
ă
OH
(c) Ph · C · CH3 Ph · C · CH2 · C · Ph

CH3

ăH2O
O

Ph · C CH · C · Ph

CH3

Mechanism :
ă
O O O
ă ă
OH
CH2 · C · H CH2 · C · H CH2 · C · H
ăH2O

H
CH3 · C · H (slowstep)

OH O H O
H2 O
CH3 · CH · CH2 · C · H CH3 · C · CH2 · C · H
ă
O

ALDEHYDES AND KETONES


36 QUIZRR
CROSS ALDOL

When two different carbonyl compounds are used, it is called cross-aldol condensation.
ă
OH
CH3CHO + CH3CH2CHO (4 different products)
(A) (B)

ă
O
OH ă A/H2O
CH3CHC · H CH2 · CH · CH · C · H
O
ă OH CH3
CH2 · CH (I)
B/H2O
O

A/H2O CH3CH2 · CH · CH · C · H
B/H2O
OH CH3
O (II)
O
CH3 · CH · CH2 · C · H
CH3CH2 · CH · CH2 · C · H
OH
OH (III)
(IV)

All the four products are -hydroxy carbonyl compounds.


When one of the carbonyl compound does not have any -hydrogen, it cannot form carbanion
and number of possible products reduces to two.

 ă
Conc. OH dil. OH
Note : Cannizzaro products   HCHO or C6 H5CHO   no reaction

ă ă
In the case of CH3CHO + CH3CH2CHO, since CH2 CHO is more stable than CH3 CH CHO , hence
ă
major product will be of CH2 CHO

similarly,
O

O C·H OH

CH3 · C + NaOH
CH3 · C · CH2 · CHO

H H
(major)
O
||
But if Ph · CH is in excess then,

ALDEHYDES AND KETONES


QUIZRR 37

O O O
ă
OH
Ph · C · H + CH3 · C · H Ph · CH · CH2 · C · H
(excess)
OH
(major)

ăH2O

Ph · CH CH · C · H
another way to obtain a single aldol product is to convert one carbonyl compound completely into
carbanion using a strong base like LDA (Lithium diisopropyl amide). Thus, this carbanion attacks
the other carbonyl compound.
O
||
LDA
CH3 · C · H + Ph · CHO  Ph · CH = CH · CHO
(major)

because here all the substrate is converted into carbanions.


O O
ă
CH3 · C · H CH2 · C · H
(becomes non-
reversible)

I llustration
O
(1)
+
H /
ă
? ?
OH

O
O
Solution :

ă
OH
OH
{since -hydrogen is present,
hence aldol condensation occurs}

+
H / ăH2O

ALDEHYDES AND KETONES


38 QUIZRR
(2) Convert
O O O
CH2 O

Solution :
O O

ă
LDA
+ H·C·H

O O O H
H
O CH2
O3  C·H

OH

Intramolecular aldol condensation : When a compound has two carbonyl groups, it can
undergo intramolecular aldol condensation in presence of dilute base (if ăH atoms are present
in the compound). An intramolecular reaction is readily favoured if the reaction leads to the
formation of a 5 or 6-membered ring.

O O
ă ă
OH
ăH2O
O O

ă
OH

ă
O
O CH3
ă

O
H2O O

ALDEHYDES AND KETONES


QUIZRR 39

CH3 OH OH CH3

C · CH3 O
O ăH2O 
ăH2O
CH3
CH3

{major} O
C · CH3
{minor}
O

{Hence, in intra-molecular aldolization, the major product is decided according to the stability of
ring formed}

(2) Claisen Condensation

C H ONa
2 moles of ester containing atleast 2 -hydrogen 
2 5
   keto ester

O O O O
C2H5ONa
CH3 · C · OC2H5 + H · CH2 · C · OC2H5 CH3 · C · CH2 · C · OC2H5

Mechanism :

O O
ă
C2H5O ă
H · CH2 · C · OEt CH2 · C · OEt
ăC2H5O H

ă
O

CH2 C · OEt

ALDEHYDES AND KETONES


40 QUIZRR
ă
O O O O
ă
CH3 · C + CH2 · C · OEt CH3 · C · CH2 · C · OEt

OEt OEt
ă
ăOEt

O O

CH3 · C · CH2 · C · OEt

Although it is expected to be -ketoester as formed above, but the final product is the sodium salt
of -ketoester. This is because acetoacetic ester (having -hydrogen between two carbonyl groups)
is much stronger acid than ethyl alcohol. So, the ester reacts with ethoxide ion to form ethyl
alcohol and the anion of sodium ester. The salt is stabilized by resonance.

OEt OEt OEt


ă
C O C O C·O

HC HC ă HC
ă
C·O C O C O

CH3 CH3 CH3

M ixed Claisen Condensation :

Just like mixed aldol, a mixed claisen condensation also takes place when two different ester are
present. Here also, only one product will be formed when one of the ester has no -hydrogen and
is taken in excess while the other ester is added slowly to the reaction mixture.

O O

Ph · C · OEt + CH3 · C · OEt


+
NaOEt / H

Ph · C · CH2 · C · OEt

O
O
ă Na+
Ph · C · CH · C · OEt

O
ALDEHYDES AND KETONES
QUIZRR 41

(3) Dieckmann Condensation :


Intramolecular Claisen condensation of esters with -hydrogen atoms in the presence of sodium
ethoxide leading to cyclization is called Dieckmann Candensation.
O

C · OEt
O O
CH
C2 H5ONa
EtO · C · (CH2)4 · C · OEt
C O

+
H3O

COOH


O O
(decarboxylation)

(4) Decarboxylation of  -keto Carboxylic acid


-keto acids on slightest warming alone or in presence of base undergoes ready removal of CO2.

O O O
|| || ||

CH3 · C · CH2 · C · OH  CH3 · C · CH3  CO2

Mechanism :

CH2 CH2

CH3 · C C O CH3 · C C O

O
O O O
H H

CH3 · C CH2 + CO2

CH3 · C · CH3
OH
Enol form
O

ALDEHYDES AND KETONES


42 QUIZRR
(5) Perkin Condensation :

aromatic alkanoic alkonoate ,  unsaturated


 
aldehydes anhydride Carboxylic acid

This is also similar to aldol condensation.

CHO CH CH · COOH
O O
CH3COONa
+ CH3 · C · O · C · CH3

(cinnamic acid)

Mechanism :
Step I :
O O O O
CH3COONa ă
H · CH2 · C · O · C · CH3 CH3COOH + CH2 · C · O · C · CH3

Step II : O H3C

C C
O ă ă O
CH2 O O O
+
slow
Ph H C O
Ph O
CH3

H3C O
H3C ă
O
C
ă
O O O O

O O
Ph Ph

Step III :
H3C
O
C
ă ă
O O MeCO 2
ă O + OH
H3 O

O O Ph O
Ph
H

ALDEHYDES AND KETONES


QUIZRR 43

(6) Benzoin Condensation


aqueous
2 moles of Benzoin
aromatic aldehyde ethanoic KCN
OH O

C·C

Mechanism :
ă
O O
ă
Ph · C · H + CN Ph · C · H

C N
O OH

Ph · C · C · Ph IMPE

OH O
ă
O OH
Ph · C ă + C · Ph

Ph · C · C · Ph H
C N
C N H

IMPE OH Oă

Ph · C · C · Ph

C N H

O O
OH O
C·C KOH
Ph · C · C · OH

Ph
Benzillic acid Rearrangement

ALDEHYDES AND KETONES


44 QUIZRR
I X. PINACOL – PINACOLONE REARRANGEMENT

CH3 CH3 CH3 O


dil. H2 SO4
CH3 · C · C · CH3 
CH3 · C · C · CH3

OH OH CH3
(pinacol) (pinacolone)

Mechanism : CH3 CH3 CH3 CH3


+
H
CH3 · C · C · CH3 ăH2O
CH3 · C · C · OH
+

OH OH CH3

+
CH3 O CH3 OH CH3 CH3

CH3 · C · C · CH3 + CH3 · C · C · CH3 CH3 · C · C · OH


·H +

CH3 CH3 CH3

Here, migration of group will take place according to the migration aptitude.

Illustration
Ph
dil. H2 SO4
CH3 ?
OH OH

Solution :
Ph +
Ph
1 2 H
CH3 +
CH3
ăH2O
OH OH OH
+
{Here H will abstract the OH, which
gives more stable carbocation}

O
HO
Ph Ph
+
CH3 H
+ CH3

{5-mem ring}
ALDEHYDES AND KETONES
QUIZRR 45

X. Tischenko Reaction :

This reaction is only given by aldehydes, where 2 molecules of aldehydes undergo condensation
type reaction to give an ester when treated with aluminium ethoxide or iso propoxide.

O
||
(C2H5O)3Al
RCHO 
 R · C · O · CH2R

O
(C2H5O)3 Al
||
2C6 H5 CHO 
 C6 H5 · C · O · CH2 C6 H5
benzyl benzoate

XI. Halogenation

(a) CH3CHO + 3Cl2  CCl3CHO + 3HCl


Chloral

O O
|| ||
H3C · C · CH3  3Cl2  Cl3C · C · CH3 + 3HCl

(b) ketones can be halogenated in presence of dilute solution of bases or an acid catalyst.

base or
CH3 COCH3  Br  CH3 COCH 2 Br + HBr
acid

Mechanism :
(1) In presence of base
O
ă RDS ă
CH3 · C · CH3 + B CH2 · C · CH3 + B · H

O
fast Br2

O
ă
CH2 · C · CH2Br + Br

ALDEHYDES AND KETONES


46 QUIZRR
(2) In presence of acid

+
O O·H

CH3 · C · CH3 + HX CH3 · C · CH3 + Xă

ă
X

CH3 · C CH2

OH

O Br2
ă
X
CH3 · C · CH2Br CH3 · C · CH2Br + Bră

+O ·H

Note : The above substitution can also be carried out using SO2Cl2

O O
|| SO2Cl2
||
H3C · C · CH3  H3C · C · CH2Cl

XII. Reformatsky Reaction

It is an addition reaction in which an organozinc reagent is used, instead of Grignard reagent,


to attack the carbonyl group of an aldehyde or ketone. Because the organozinc reagent is less
reactive than a grignard reagent, a second nucleophilic addition to the ester group does not occur.
The organozinc reagent is prepared by treating an -bromoester with zinc.

O CH3 O BrZnO CH3 O


Zn
(1) CH3 · C · H + Br · C · C · OEt CH3 · CH · C · C · OEt
Benzene

CH3 CH3
+
H3O

OH CH3 O

CH3 · CH · C · C · OEt

CH3

ALDEHYDES AND KETONES


QUIZRR 47

O CH3 O OH CH3 O
(1) Zn/Benzene
(2) C6H5 · C · H + Br · CH · C · OEt + C6H5 · CH · CH · C · OEt
(2) H3O

XIII. Beckmann Rearrangement

OH R
N H2SO4 (conc) NH

R R´ O R´

An acid-induced rearrangement of oximes to give amides. When oximes are treated with acidic
catalyst like H+, PCl5, SOCl2, SO3, P2O5 etc., they are transformed into substituted amides. This
reaction is related to the Hofmann and Schmidt Reactions and the Curtius Rearrangement, in
that an electropositive nitrogen is formed that initiates an alkyl migration.
Example

Ph OH
PCl5in
(1) C N ether, 
Ph · C · NHCH3
H3C
Syn (Ph) O

CH3 OH
PCl5in
C N ether, 
H3C · C · NHPh
(2)
Ph
anti (Ph) O

Mechanism :

Oximes generally have a high barrier to inversion, and accordingly this reaction is envisioned to
proceed by protonation of the oxime hydroxyl, followed by migration of the alkyl substituent

ALDEHYDES AND KETONES


48 QUIZRR
ÂÂtrans’’ to nitrogen. The NăO bond is simultaneously cleaved with the expulsion of water, so that
formation of a free nitrene is avoided.

H
+
O
ă
N H
+
N +
N
ăH2O

H
+ +
N N O N O
+ + +
H H H ăH / + H

H H H
+ +
N O·H N O N O
H

The migrating group is always anti to · OH.

ALDEHYDES AND KETONES


QUIZRR 49

SOLVED EXAMPLES

Example 1

Solution :

Example 2

Cl2 1. DMSO 1. CH3 MgI H2SO4


A ă B C D
h 2. HCO3 2. H2O+ 

HF ă
.T OH
3
BH O 2, O3/H2O/Zn
(1) H 2
)
(2
CrO3/Pyridine
G F
E
Solution :

ALDEHYDES AND KETONES


50 QUIZRR
Example 3

Suggest reagent required at each step :

Solution :

CH2OH
(A) | to protect keto grp from being reduced in step (B).
CH2OH

(b) LiAlH4/ether, H3O+, (C) PCC, (D) NH2NH2/glycol ă KOH, (wolf ă Kisher), (E) H3O+

Example 4
What are the final products of the following reactions :
O

H3O+ + CH3CH2SH
(a) (b)
O O
O
O
+
H3O
(c) N (d) + CH2 N2

O O
|| || (1) CH3MgBr
(e) CH3 · C · CH 2 · CH2 · C · OCH2 CH3  
+ (2) H3O

(f)

ALDEHYDES AND KETONES


QUIZRR 51

Solution :

H3 O+

(a) cleavage of ·H2O


O O epoxy linkage
HO HO OH OH HO O H OH

O O OH
ă +
CH3CH2SH
(b)
SCH2 CH3
O O O
enolisation
OH

SCH2 CH3
OH

H3O+
(c) N OH
+ NH

+
tautomerises H3O

NH2
O

O O

(d) +CH2N2 +N2

cycloheptanone

ALDEHYDES AND KETONES


52 QUIZRR
O O OH O
1 CH3MgBr
(e) CH3·C·CH2·CH2·C·O·CH2·CH3 +
CH3·C·CH2·CH2·C·O·CH2
2 H3O

reaction at CH3 CH3


keto group
Cyclization
·CH3 CH2 OH

H3C O
O

H3C

(f)

Example 5

A (C8H14) by ozonolysis forms B (C8H14O2). B on reaction with HCN followed by acidic


hydrolysis forms a dibasic acid (C) with two chiral carbon atoms. HI reduces [C] to another
optically active dibasic acid (D). Identity A, B, C and D and explain reactions if [A] has one
six-membered rings.
Solution :

Degree of unsaturation in A (C8H14) = 16 ă 14  2 = 2


2
Ozonolysis product B(C8H14O2) has also eight carbon atoms that indicates A is cyclic compound
(one saturation) with a double bond in it (another saturation). Thus, A is symmetrical cyclohexene
(given six membered ring).

O
CN CH3
CH3 CH3
O3/H2O HCN OH
OH
CH3 CH3
CN CH3
O
(A) (B)
H 3O +
(* indicates chiral carbon atom)

ALDEHYDES AND KETONES


QUIZRR 53

COOH COOH
CH3 CH3
H HI H
H H
CH3 CH3
COOH COOH
(C)

Example 6

Two moles of an ester A are condensed in the presence of sodium ethoxide to give a -keto
ester, B and ethanol. On heating in an acidic solution, B gives ethanol and a -keto acid, C.
On decarboxylation C gives 3-pentanone. Identify A, B and C with proper reasoning.
Solution :
The reaction of 2 mol of an ester giving -keto ester and alcohol in the presence of sodium
ethoxide is known as Claisen condensation.
Let the given reactions may be depicted as shown in the following.

ă
OC2H5
RCH2 · C · OR´ + RCH2 · C · OR´ R · CH2 · C · CH · C · OR´ + R´OH
ethanol
O O O R O
(A) (A) -keto ester
(B)

+
H
R · CH2 · C · CH · C · OR´ R · CH2 · C · CH · C · OH + R´OH
heat
ethanol
O R O O R O
-keto ester
(C)

heat
R · CH2 · C · CH · C · OH R · CH2 · C · CH2

O R O O R

From these reactions, it is obvious that

R´  · CH2CH3
R  · CH3

ALDEHYDES AND KETONES


54 QUIZRR
Hence, the compounds A, B and C are

A: CH3CH2C · OCH2CH3 B: CH3CH2C · CH · C · OC2H5 C: CH3CH2C · CH · COOH

O O CH3 O O CH3

Example 7

H
| LDA
CH 3 · CH = CH · C = O (excess) 

O
O
(a) H (b)
H
OH

(c) (d)

Solution :
In the absence of ăH on saturated carbon, ăH becomes acidic. This acidic ăH would be
abstracted by LDA to give carbanion, which condense with another molecule to give
cyclooctatetraene. The high temperature reaction condition helps in the dehydration of aldol
product.
O
O

LDA H
H
 H

Example 8

1 [(CH3)2CH]2N Li
2
+ CH3CHO

ALDEHYDES AND KETONES


QUIZRR 55

O OH O

(a) (b)

OH

O H
O
OH
H
(c) (d)
O

Solution :
LDA is a sterically hindered and strong base. It will abstract H+ from position (1) rather than
position (2). Carbanion formed will attack on CH3CHO to give intermolecular aldol reaction.
 (b)

Example 9

Which of the following ketones is more acidic ? Give a reason.


O
O

Solution :
O O

Base
H

This ketone is more acidic because the resulting enolate ion obeys HuckelÊs rule of aromaticity and
is thus more stable.

Example 10

A ketone A which undergoes a haloform reaction gives compound B on reduction. B on


heating with sulphuric acid gives compound C, which forms mono-ozonide D. The compound
D on hydrolysis in presence of zinc dust gives only acetaldehyde. Identify A, B and C. Write
down the reactions involved.

ALDEHYDES AND KETONES


56 QUIZRR
Solution :
We are given that

reduction
A 
 B

H 2SO
B 
4
 C

C forms mono-ozonide, D

H O/Zn
D 
2

 CH3CHO

The compound A gives a haloform reaction; it must contain CH3CO group. The compound C
contains a double bond as it forms mono-ozonide D. Since, the compound D on hydrolysis gives
only CH3CHO, the structure of C would be

CH3CH = CHCH3 2-butene


(C)

The compound C is obtained by dehydration of B, thus the latter should be

CH3 · CH · CH2 · CH3


|
OH 2-butanol
(B)

Finally, B is obtained by the reduction of A. Hence, the compound A should be

CH3 · C · CH2 · CH3


||
O 2-butanone
(A)

The equations involved are as follows :

ALDEHYDES AND KETONES


QUIZRR 57

Example 11

In which of the following substrate, rate of Benzoin condensation will be maximum ?

(a) O2N CHO (b) H3C CHO

(c) HO CHO (d) H2N CHO

Solution :

ă
O O
O O
·
N C + CN N C H
O H O
(I) (II) C N

O·H
O
ă
N C
O
(III) C N

Benzoin condensation is due to stability of intermediate (III) when negative charge on C is


extensively delocalised in benzene ring, Nitro and C  N group. In all other cases, such dispersal
is not extensively possible. On the other hand, NO2ă is also creating positive charge centre on
carbonyl carbon, making it more susceptible to nucleophilic attack of CNă.
 (a)

Example 12

An ester (A) is condensed in the presence of sodium methoxide to give a -keto ester (B) and
Methanol. On mild hydrolysis with cold conc. HCl, (B) gives methanol and 3-oxo-acid (C).
(C) underwent readily decarboxylation to give cyclopentanone.
(a) Identify (A), (B) and (C)
(b) Name the reaction involved in conversion of (A) to (B)
(c) Give the mechanism of decarboxylation

ALDEHYDES AND KETONES


58 QUIZRR
Solution : OMe

NaOMe
O O + MeOH
(a) O

CO2Me
OMe
(A) -ketoester
(B)

O O
1
3 CO2H
mild 2 
(B) + MeOH (C) + CO2
hydrolysis ăCO2

3-oxo-acid or -keto acid

(b) Diekmann condensation (intramolecular Claisen ester condensation)


(c) Mechanism of decarboxylation

C O

O ăCO2
H
O H enol OH keto
O

Example 13

A organic compound (A), C10H16O, reacts with hydroxylamine to yield a compound of formula
C10H17ON (B), and with TollenÊs reagent to give a silver mirror and a compound of formula
C10H16O2(C). Upon vigorous oxidation it gives acetone, oxalic acid and -ketovaleric acid
(CH3COCH2CH2COOH). What are (A), (B) and (C) ?
Explain these reactions.
Solution :
(i) (A) reacts with hydroxylamine to form (B), C10H17ON; hence it contains a carbonyl group.
(ii) Since (A) reacts with TollenÊs reagent to give (C), C10H16O2, hence the > C == O function
is an aldehydic group i.e., (A), C9H15CHO

ăH2O
C9H15CHO + H2NOH C9H15CH NOH
(A) (B)
Ag2 O
C9H15COOH
ă2Ag
(C)

ALDEHYDES AND KETONES


QUIZRR 59

(iii) Since (A) on oxidation gives three products hence it contains double bonds.

H3C
O O
O CH3
+
+
H3C acetone
O
O
a-axopentanoic acid oxalic acid

CH3 CH3

H3C
H O
(A)

OH
H3C
O
CH3
(C)

Example 14

Write the products of the following reaction sequence :


OHă HA H2
2RCH2CHO H 2O (A) ·H2O (B) Pd,C (C)

LiAlH 4

Solution : D

R R O
O
A= B=
HO R R

R R OH
O
C= D=
R R

ALDEHYDES AND KETONES


60 QUIZRR
Example 15

Pure HCN fails to react with aldehydes and ketones. Explain.


Solution :
HCN is weak acid and have low degree of dissociation but in presence of a base (even H2O), the
dissociation increases appreciably to provide appreciable CN to attack > C = O bond.

Example 16

Acetone when treated with hydroxylamine forms only one oxime whereas ethyl methyl
ketones (or acetophenone) forms two isomeric oximes under the same conditions. Give
their structures.
Solution :

H3C H3C
NH2OH
O N
H3C H3C OH

H3C H3C H5C6 H5C6

O + NH2OH N ; O + NH2OH N
H5C2 H5C2 OH H3C H3C OH
(acetophenone)

Oximes of aldehydes and unsymmetrical ketones show geometrical isomerism i.e.

H3C C2H5 H3C C2H5

N N
HO OH
anti syn

Whereas oximes of symmetrical ketones do not show geometrical isomerism.

ALDEHYDES AND KETONES


ElectropHilic
Aromatic
Sustitution
QUIZRR 3

ElectropHilic Aromatic Sustitution


1. AROM ATI CI TY

H H
C C
HC CH HC CH

HC CH HC CH
C C
H H

In organic chemistry, the structures of some rings of atoms are unexpectedly stable. Aromaticity
is a chemical property in which a conjugated ring of unsaturated bonds, lone pairs, or empty
orbitals exhibit a stabilization stronger than would be expected by the stabilization of conjugation
alone. It can also be considered a manifestation of cyclic delocalization and of resonance.
This is usually considered to be because electrons are free to cycle around circular arrangements
of atoms, which are alternately single and double-bonded to one another. These bonds may be seen
as a hybrid of a single bond and a double bond, each bond in the ring identical to every other. The
benzene consists of two resonance forms, which corresponds to the double and single bondsÊ switching
positions. Benzene is a more stable molecule than would be expected without accounting for charge
delocalization.

1 .1 T h e o r y
H
H H

H H
H

A C=C bond is shorter than a CăC bond, but benzene is perfectly hexagonalăall six carbon-carbon
bonds have the same length, intermediate between that of a single and that of a double bond.
A better representation is that of the circular  bond in which the electron density is evenly
distributed through a -bond above and below the ring. This model more correctly represents the
location of electron density within the aromatic ring.

ELECTROPHILIC AROMATIC SUBSTITUTION


4 QUIZRR

The single bonds are formed with electrons in line between the carbon nucleiăthese are called
-bonds. Double bonds consists of a -bond and a -bond. The -bonds are formed from overlap of
atomic p-orbitals above and below the plane of the ring. The following diagram shows the positions
of these p-orbitals :

Side view Projection


Since they are out of the plane of the atoms, these orbitals can interact with each other freely,
and become delocalised. This means that instead of being tied to one atom of carbon, each electron is
shared by all six in the ring. Thus, there are not enough electrons to form double bonds on all the
carbon atoms, but the „extra‰ electrons strengthen all of the bonds on the ring equally. The resulting
molecular orbital has  symmetry.

H H

H H H H
H H
H H

6-p-orbitals delocalised orbital clouds

1 .2 Ch a r a c t e r i s t i c s o f a r o m a t i c (A r yl ) c o m p o u n d s
An aromatic compound contains a set of covalently-bound atoms with specific characteristics :
1. A delocalized conjugated  system, most commonly an arrangement of alternating single
and double bonds.
2. Coplanar structure, with all the contributing atom in the same plane.
3. Cyclic in nature
4. A number of  delocalized electrons that is even, but not a multiple of 4. That is, 4n +
2 of  electrons, where n = 0, 1, 2, 3 and so on. This is known as HuckelÊs Rule.
l Aromatic molecules typically display enhanced chemical stability, compared to similar non-
aromatic molecules. A molecule that can be aromatic will tend to alter its electronic or
conformational structure to be in this situation. This extra stability changes the chemistry
of the molecule.
l Many of the earliest-known examples of aromatic compounds, such as benzene and toluene,
have distinctive pleasant smells. This property led to the term „aromatic‰ for this class of
compounds, and hence the term „aromaticity‰ for the eventually-discovered electronic property.

ELECTROPHILIC AROMATIC SUBSTITUTION


QUIZRR 5

1 .3 A n t i a r o m a t i c i t y
+ ·
C C

A B C Cyclooctatetraene

Antiaromatic molecules are cyclic systems containing alternating single and double bonds,
where the pi electron energy of antiaromatic compounds is higher than that of its open-chain
counterpart. Therefor antiaromatic compounds are unstable and highly reactive; often antiaromatic
compounds distort themselves out of planarity to resolve this instability. Antiaromatic compounds usually
fail HuckelÊs rule of aromaticity.
Examples of antiaromatic systems are cyclobutadiene (A), the cyclopentadienyl cation (B) and
the cyclopropenyl anion (C). Cyclooctatetraene is a 4n system but neither aromatic or antiaromatic
because the molecule escapes a planar geometry.
By adding or removing an electron pair via a redox reaction, a  system can become aromatic
and therefore more stable than the original non- or anti-aromatic compound, for instance the
cyclooctatetraenide dianion. The IUPAC criteria for antiaromaticity are as follows :
1. The molecule must have 4n  electrons where n is any integer.
2. The molecule must be cyclic.
3. The molecule must have a conjugated pi electron system.
4. The molecule must be planar.

Ex a m p l e 1
Is this compound aromatic or antiaromatic ?

N
Solution :
The given compound, pyridine is aromatic as total no. of conjugated electrons
= 6 = 4n + 2 where n = 1.
Here lone pair of Nitrogen is not involved in delocalisation.

EX CEPT I ON S
(1 )

H H

Here no. of  eă = 10

ELECTROPHILIC AROMATIC SUBSTITUTION


6 QUIZRR

Molecule will be aromatic if planar. But due to the repulsion between hydrogens, molecule lose
its planarity.
(2 )

Here no. of  eă = 8
This molecules should be antiaromatic.
But its not planar. Its real structure is non planar.

Nonplanar

Therefore this molecule is nonaromatic.

1 .4 St a b i li t y o f Co m p o u n d s
Order of stability is :
aromatic > nonaromatic > antiaromatic
Stability order is explained on basis of constructive and destructive resonance in aromatic and
antiaromatic compound respectively; which comes under molecular orbital theory.

1 .5 Cy c l o p r o p e n i u m s a l t s
H
+

H H
This is the cyclopropenyl cation and may be represented as a resonance hybrid.

H H H H
+ +

+ +
H H H H H H H H

Hence the cyclopropenyl cation should be stable. Many cyclopropenium salts have actually been
prepared.

ELECTROPHILIC AROMATIC SUBSTITUTION


QUIZRR 7

e.g. (i) hydroxydiphenyl propenyl bromide


(ii) cyclopropenyl hexachloroantimonate.

Ph Ph
O H·Br OH Br·
(i)
Ph Ph

·
(ii) Cl SbCl5 SbCl6

1 .6 Cy c l o b u t e n i u m s a l t s

Br
Ph Ph
Ph1 Ph
2AgBF4
++ 2BF4
Ph Ph
Ph Ph
Br

1 .7 A n n u l e n e s
Conjugated monocyclic polyenes, CnH n, in which n  10 are usually called onnulenes.
The annulenes prepared have n = 12, 14, 16, 18, 20, 24 and 30, of these only [14], [18] and [30]
annulenes are (4n + 2) electron molecules and rest ore 4n molecules.

H
H H
H H
HH H HHH
H

dehydro
[14] annulene [18] annulene [14] annulene

ELECTROPHILIC AROMATIC SUBSTITUTION


8 QUIZRR

1 .8 A zu l e n e

Azulene

+ ă

dipolar structure

There are two KeKule resonating structures containing 10  electrons (n = 2; and 10 peripheral
-electrons). The five membered ring has five and the seven membered ring has seven electrons (two
electrons are common to both rings). If one electron is transferred from seven to the five ring each
ring will now have a closed shell of six electrons. In this condition, the molecule will have a dipolar
structure.

2. EL ECT ROPH I LI C A ROM A T IC SU B ST I T U T ION


Overall an electrophilic aromatic susbtituton can be represented as follows :

E
E+ + + H+

There are three fundamental compounds to an electrophilic substitution reaction :


1. Formation of the new  bond from a C=C in the arene nucleophile
2. removal of the proton by breaking the C-H  bond
3. reform the C=C and restore aromaticity

E
H E
E+
+ H+
+

The mechanism is represented by the following series of events :


l Formation of the reactive electrophile, E+
l Slow reaction of the arene C=C with the E+ to give a resonance stabilised carbocation (see
below)

ELECTROPHILIC AROMATIC SUBSTITUTION


QUIZRR 9

l Loss of H+ from the carbocation to restore the C=C and the aromatic system

+
E E E

+ +

The reaction of the electrophile E+ with the arene is the slow step since it results in the loss of
aromaticity even though the resulting cation is still resonance stablised.

Wh y Su b s t i t u t i o n n o t A d d i t io n ?
Overall an electrophilic aromatic substitution can be represented as follows :

E
E+ + + H+

But we have previously seen the C=C generally react via an electrophilic addition pathway :

E
Nu·E + =
Nu

So why donÊt arenes react in a similar fashion to alkenes and give overall addition ?
The first step is common to both, E
E + adds a C=C to give a
E+
carbocation intermediate. +

For an addition pathway, the


E E
nucleophile then adds to the
electrophilic carbocation giving Nuă
+ addition
a cyclohexadien e derivative. Nu

For a substitution pathway


„nucleophile‰ functions as a base E
H E
and removes a proton from the
sp3 C to recreate the C=C and Nuă + NuH
+ substitution
restore the aromaticity.
The resonance energy of benzene is about 152 kJ / mol (36 kcal / mol) and a conjugated diene
is 16 kJ / mol (4 kcal / mol).This extra stability of the aromatic system is responsible for favouring the
substitution reaction.

ELECTROPHILIC AROMATIC SUBSTITUTION


10 QUIZRR

l An electrophilic substitution reaction on benzene does not always result in monosubstitution.


While electrophilic substituents usually withdraw electrons from the aromatic ring and thus
deactivate it against further reaction, a sufficiently strong electrophile can perform a second
or even a third substitution. This is especially the case with the use of catalysts.

3 . SOM E B A SI C REA CT I ON S
3 .1 N i t r a t i o n

Arom at ic nit rat ion


In a aromatic nitration, aromatic organic compounds are nitrated via an electrophilic
aromatic substitution mechanism involving the attack of the electron-rich benzene ring by the
nitronium ion.

H2SO4
+ HNO3 NO2 + H2O

Benzene is nitrated by refluxing with concentrated sulfuric acid and concentrated nitric acid at
50ĈC. The sulfuric acid is regenerated and hence acts as a catalyst. It also absorbs water.
The formation of a nitronium ion (the electrophile) from nitric acid and sulfuric acid and subsequent
reaction of the ion with benzene is shown below :

NO2 H2SO4 + NO2 H2SO4 + ă


H·O H·O NO2 + H3O + HSO4

H
+
HSO4


   
i.e. HNO3  2H2SO4  NO2  H3O  2HSO4

Now the NO2+ ion attacks the benzene nucleus and forms an intermediate cation, a benzenonium
ion; which loses a proton to yield the nitro derivative.

H
electrophilic
+ NO2 NO2 NO2
attack
Nitronium
ion

NO2 H H
+ NO2 NO2
H +

Nitro benzene

ELECTROPHILIC AROMATIC SUBSTITUTION


QUIZRR 11

The nitronium ion is well known existing in salts such as NO2 ClO4 and NO2 BF4 which
smoothly nitrate benzene at room temperature. It supports the mechanism in which the electrophile
species attacking the aromatic compound is nitronium ion NO2 .
Highly reactive aromatic compounds such as phenol, are found to undergo ready nitration even
in dilute nitric acid and at a far more rapid pace than can be explained on the basis of the
concentration of NO
2 that is present in the mixture. This has been shown due to the presence of 
nitrous acid in the system which nitrosates the reactive nucleus via the nitrosonium ion N O .



  
HNO2  2HNO3 
 H3O  2NO3  NO

OH OH OH OH

NO NO3 HNO3 + HNO2

H NO NO NO2

3 .2 .1 So m e I m p o r t a n t Po i n t s

NO2 can be generated by :

(i) Mixture of HNO3 & any other acid stronger than HNO3 to generated NO2 .

HNO3 / conc. H2SO4

It is behaves base It is behaves as an acid

H2 SO4  OH  NO 2  NO 2  HSO4  H2O

(ii) Conc. HNO3 + HO-NO2  NO2 + H2O + HNO3ă

(iii) N2O5  NO2 + NO3ă

 
(iv) NO2 BF4ă  NO2 + BF4ă

Ist step is slow step mechanism because compound loses its aromaticity.
In IInd step compound gains aromaticity, it is highly fast.
Potential energy curve with the progression of reaction is given by :

ELECTROPHILIC AROMATIC SUBSTITUTION


12 QUIZRR

Slowstep (step1)

Fast step (step2)

P.E.

reaction coordinate
progression of reacter

If H in Benzene is replaced by D, then

D D
D D D E+
+ D
+E
D D D D
D D

ăD+

Here E+ = NO2

Co m p a r is o n o f r a t e s
Let for the first reaction the value of rate constant is kH and that for IInd is kD then which can
be true.
(i) kH > kD
(ii) kH < kD
(iii) kH = kD
(iv) canÊt say Ans. : KH > KD

Ex p l a n a t i o n
C ă D bond is stronger than C-H.

ELECTROPHILIC AROMATIC SUBSTITUTION


QUIZRR 13

3 .2 A r o m a t i c Su l f o n a t i o n
Aromatic sulfonation is an organic reaction in which a hydrogen atom on an arene is
replaced by a sulfonic acid functional group in an electrophilic aromatic substitution.

Heat
+ H2SO4 SO3H + H2O

Sulfur trioxide is the electrophile in this reaction generated from concentrated sulfuric acid
(or fuming sulfuric acid) when heated.
O O O O

HO S OH + HO S OH HO S OH2+ + ăO S OH

O O O O

HO S + H2O SO3 + H3O+

O
ă
ă 
O O O O O

S S+ S+ S+ S ă
ă ă ă 
O O O O O O O O  O O
l In contrast to aromatic nitration and other electrophilic aromatic substitution this reaction
is reversible. Sulfonation takes place in concentrated acidic conditions and desulfonation is
the mode of action in a dilute hot aqueous acid.

Mec hanism of sulphonat ion


ă H
O
SO3
+
+ S
O O

H
SO3H H SO3
SO3

S is bonded to three more electronegative oxygen atoms. Hence, S is electron deficient, acting as
electrophile.

ELECTROPHILIC AROMATIC SUBSTITUTION


14 QUIZRR

M o r e I m p o r t a n t Po i n t s
Increase of sulphonation the electrophile is SO3.
l SO3 has vacant orbital, therefore it is electrophile.
l Both the steps are in equilibrium in the mechanism.

Co m p a r is o n w it h De u t ro b e n ze n e :

D SO3D
D D D
D + H2SO4
D D D D
D D

KH > KD
This reaction is reversible.

En e r g y Pro f i l e

P.E.

Reaction coordinate

Reaction is reversible. Carbocation can proceed in both forward and backward direction. IInd step
is not the fast step. Both the step will contribute to stability of product.
C ă D bond is stronger than C-H bond.

3 .3 El e c t ro p h i li c h a l o g e n a t i o n
In organic chemistry, an electrophilic aromatic halogenation is also a type of electrophilic
aromatic substitution. This organic reaction is typical of aromatic compounds and a very useful
method for adding substituents to an aromatic system.

(catalyst)
+ X X + HX

ELECTROPHILIC AROMATIC SUBSTITUTION


QUIZRR 15

A few types of aromatic compounds, such as phenol, will react without a catalyst, but for typical
benzene derivatives with less reactive substrates, a Lewis acid catalyst is required. Typical Lewis
acid catalysts include AlCl3, FeBr3, and ZnCl2. These work by forming a highly electrophilic complex
which attacks the benzene ring.

Re a c t i o n m e c h a n i s m
+ ă ă
Br · Br + FeBr3 Br · Br ...... Fe Br3 Br+ + FeBr4
(catalyst)
ă
B
H
+ Br + Br + HB
Br

l The reaction mechanism for chlorination of benzene is the same as bromination of benzene.
Ferric bromide and ferric chloride become inactivated if they react with water, including
moisture in the air. Therefore, they are generated in situ by adding iron filings to bromine
or chlorine.
l Halogenation of aromatic compounds differs from the halogenation of alkenes, which do not
require a Lewis Acid catalyst. The formation of the arenium ion results in the temporary
loss of aromaticity, which has a higher activation energy compared to carbocation
formation in alkenes. In other words, alkenes are more reactive and do not need to have the
Br-Br or Cl-Cl bond weakened.

Im po rt an t po int
A similar dual mechanism can also operate when halogenation is carried out with hypochlorous
and hypobromous acid. The reaction is acid catalysed.
HăOăCl+H+  H2O+ăCl
H
Cl
+ H2O Cl +
+ H2O ă Cl +H
+

Ex a m p l e 4
Among pyrrole and pyridine, Which is more basic and why?

and
N
N
·

H
(pyrrole) (pyridine)

ELECTROPHILIC AROMATIC SUBSTITUTION


16 QUIZRR

Solution :
Pyridine is more basic than pyrrole because in case of pyridine nitrogen lone pair does not
participate in aromaticity.

Ex a m p l e 5
Write the product of mono bromination of C6H5CF3.
Solution :
F F
F F F F

FeBr2 + HBr
+ Br2
Br
ăCF3 shows (ă I) effect. Hence it is behave as M-directing group.

Ex a m p l e 6
Find out products in following conversion.
(i) C6H5CH (CH3)CH2CH2 (monosulfonation)  (A)

(ii)
(mononitration) (B)

Solution : H3 C
CH3

A

SO3 H
O O
CH3

B

NO2

ELECTROPHILIC AROMATIC SUBSTITUTION


QUIZRR 17

3 .4 Fr i e d e l -Cra f t s a l k y l a t i o n
Friedel-Crafts alkylation involves the alkylation of an aromatic ring and an alkyl halide
using a strong Lewis acid catalyst. With anhydrous Aluminium chloride as a catalyst, the alkyl
group attaches at the former site of the chloride ion. The general mechanism is shown below.

R  Cl  AlCl3  R   AlCl 4


Cl

·
ă
Cl · Al · Cl

·
+ H R
R R Cl

ă HCl
+ AlCl3 + HCl
+ catalyst regenerated
 ăcomplex
Im po rt an t po int s
1. This reaction has one big disadvantage, namely that the product is more nucleophilic than
the reactant due to the election donating alkyl-chain. Therefore, another hydrogen is
substituted with an alkyl-chain, which leads to overalkylation of the molecule.
2. Also, if the chlorine is not on a tertiary carbon, carbocation rearrangement reaction
will occur. This is due to the relative stability of the tertiary carbocation over the secondary
and primary carbocations.
Alkylations are not limited to alkyl halides : Friedel-Crafts reactions are possible with any
carbocationic intermediate such as those derived from alkenes.
3. Not only nature of alkyl group, but also temperature determines the nature of electrophile.
e.g. n-alkyl group can be introduced to a fair extent without rearrangement at low
temperature, because ionisation of oduct is retarted. But at higher temperature, carbonium
ion is formed which rearranges and the product is rearranged alkyl benzene. Thus n-
propylchloride gives isopropyl benzene.

ELECTROPHILIC AROMATIC SUBSTITUTION


18 QUIZRR

CH3
CH2 · CH2 · CH3 + AlCl3 + HCl
CH
CH3
+ AlCl3 + HCl

4. The order of effectiveness of Lewis acid catalysts has been shown to be


AlCl3 > FeCl3 > BF3 > TiCl3 > ZnCl2 > SnCl4
The action of Me3CCH2/AlCl3 on benzene is found to yield almost completely the rearranged
product PhCMe2 CH2Me, while Me3CCH2Cl/FeCl3 on benzene is found to yield almost
completely the unrearranged product.

3 .4 .2 Fr i e d e l -Cra f t s a c y l a t i o n
l Friedel-Crafts acylation is the acylation of aromatic rings with an acyl chloride using a
strong Lewis acid catalyst.
l Friedel-Crafts acylation is also possible with acid anhydrides.
l Reaction conditions are similar to the Friedel-Crafts alkylation mentioned above. This reaction
has several advantages over the alkylation reaction. Due to the electron-withdrawing effect
of the carbonyl group, the ketone product is always less reactive than the original molecule,
so multiple acylations do not occur. Also, there are no carbocation rearrangements, as the
carbonium ion is stabilized by a resonance structure in which the positive charge is on the
oxygen.
O

RCOCl or (RCO)2O
R
AlCl3 catalyst reflux
anhydrous conditions

Re a c t i o n m e c h a n i s m
In a simple mechanism view, the first step consists of dissociation of a chlorine atom to form an
acyl cation

O
+ ă
R ă C ă Cl + AlCl3 R ă C = O + AlCl4

ELECTROPHILIC AROMATIC SUBSTITUTION


QUIZRR 19

This is followed by electrophilic acylcation toward the arene.

+
+RăC=O

Finally, a chlorine atom reacts to form HCl, and the AlCl3 catalyst is regnerated.
O
O
H CăR
C
ă
+ AlCl4 R
+ HCl + AlCl3

Product

L i m i t a t io n s o f Fr ie d e l -Cr a f t s r e a c t i o n s
Several restrictions limit the usefulness of Friedel-Crafts reactions :
(1) When the carbocation formed from an alkyl halide, alkene, or alcohol can rearrange to a
more stable carbocation, it usually does so and the major product obtained from the reaction
is usually the one from the more stable carbocation. When benzene is alkylated with butyl
bromide, for example, some of the developing butyl cations rearrange by a hydride shift ă
some developing 1Ĉ carbocations (see following reactions) become more stable 2Ĉ carbocations.
Then benzene reacts with both kinds of carbocations to form both butylbenzene and sec-
butyl benzene.
+
CH2 CH3
AlCl3
Br +
H3 C H3 C H3 C

CH3
H3C

butylbenzene sec-butylbenzene

ELECTROPHILIC AROMATIC SUBSTITUTION


20 QUIZRR

(2) Friedel-Crafts reactions do not occur when powerful electron-withdrawing groups are present
on the aromatic ring or when the ring bears an ăNH2, ă NHR, or ăNR2 group. This applies
to alkylations and acylations.
O OH O R
+ +
NO2 N(CH3 )3 C C CF3 SO3H NH3

These do not undergo Friedel-Crefts reactions

We shall learn that the groups present on an aromatic ring can have large effect on the
reactivity of the ring towards electrophilic aromatic substitution. Electron-withdrawing groups
make the ring less reactive by making it electron deficient. Any substituent more electron
withdrawing (or deactivating) than a halogen, that is, any meta-directing group, makes an
aromatic ring too electron deficient to undergo a Friedel-Crafts reaction. The amino groups,
ăNH2, ă NHR, and ăNR2 are changed into powerful electron-withdrawing groups by the
Lewis acids used to catalyze Friedel-Crafts reactions. For example :
H
H H
·

+
·

N H · N · AlCl3

+ AlCl 3

Does not undergo a


Friedel-Crafts reaction

(3) Aryl and vinyl halides cannot be used as the halide component because they do not form
carbocations readily. Cl

No Friedel-Crafts reaction
AlCl3

No Friedel-Crafts reaction
CH3

Cl
H3 C

CH3

ELECTROPHILIC AROMATIC SUBSTITUTION


QUIZRR 21

(4) Polyalkylations often occur. Alkyl groups are electron -releasing groups, and once one is
introduced into the benzene ring it activates the ring toward further substitution.

H3C CH3 H3C CH3

H3C
BF3
+ OH +
60Ĉ C
H3C Isopropyl benzene (24%)

H3C CH3
p-disopropylbenzene (14%)

Polyalkylations are not a problem in Friedel-Crafts acylations, however the acyl group
(RCOă) by itself is an electron-withdrawing group, and when it forms a complex with AlCl3
in the last step of the reaction, it is made even more electron withdrawing. This strongly
inhibits further substitution and makes monoacylation easy.

Ex a m p l e 7
What happens when benzene is treated with methyl chloride in presence of anhydrous
AlCl3 and the product is treated with excess of chlorine in presence of UV light ?
Solution :

anhy Cl2 Cl2 Cl2  C H CCl


C6 H6  CH3Cl  C6 H5CH3   C6 H5CH2Cl   C6 H5 CHCl2  6 5 3
Benzene
AlCl3 UV light Benzotrichloride

Co m p a r is o n b e t w e e n a l k y l a t i o n a n d a c y l a t i o n
(i) A comparison of the electrophilic nature of both alkyl and acyl group indicates that acyl
group is a better electrophile on account of two electron withdrawing atoms attached to C.

H
ă
·

+ ă O
R · C · Cl
·
·
·

R · C · Cl
H +

Effective +ve charge density


on carbonyl compound is greater.

(ii) Acylation requires more catalyst than alkylation because much of the catalyst is removed by
the formation of a complex with the product (ketone) and is removed from further participation
in the reaction.

ELECTROPHILIC AROMATIC SUBSTITUTION


22 QUIZRR
ă
H3C · C O AlCl3
COCH3 +
+ AlCl3

(iii) Unlike polyalkylation, polyacylation does not take place as the product ketone is much less
reactive than the original hydrocarbon.
(iv) Rearrangement of R does not take place in acylation, but decarbonylation can take place,
especially where R would form a stable carbonium ion, so that the end result is then
alkylation rather that the expected anylation.


e.g. Me3 C  C  O  CO  Me3 C

CHe3
+
+ Me3 C

3 .5 So m e I m p o r t a n t Re a c t io n s

(1) Reaction of alcohols + Acid +

Solution :

(i) CH3 CH2OH + H+ ?

CH2CH3

H
+ +
CH3CH2 · O CH3CH2
H

(ii) CH2 OH

+
+H

ELECTROPHILIC AROMATIC SUBSTITUTION


QUIZRR 23

+
CH2OH CH2

+
Solution : + Rearrangement
H

(2) Reaction of Alkene + Acid +

CH3
| +
(i) H
CH3  CH  CH  CH2

Solution :

CH3 CH3
| |
H
CH3  CH  CH  CH2   CH3  CH ă CH  CH3
+

1, 2 hydride shift

CH3 CH3
·

CH3 · C · CH2 CH3 CH3 · C · CH2CH3


+
·

ELECTROPHILIC AROMATIC SUBSTITUTION


24 QUIZRR

(3) Reaction of Alkyl Halide + Lewis Acid +

CH3
·
AlCl
3
(i) CH3 · C · CH2Cl
·

CH3

CH3 CH3
| |
CH3  C  CH 2Cl
AlCl3
  CH3  C  CH ă
2  AlCl 4
| |
CH3 CH3

Rearrangement

CH3 CH3
·

·
CH3 · C · CH2 CH3 CH3 · C · CH2 · CH3
+
·

Ex a m p l e 8

AlCl3
+ O

+
Solution : ă
+ AlCl3 OAlCl3
O

OH O ă AlCl3
H
+
AlCl3 +

ELECTROPHILIC AROMATIC SUBSTITUTION


QUIZRR 25

O
(3)
AlCl3
+ O O

O OH
O O
O
+
O + AlCl3 ă
O AlCl3

O O
H3 C O
O C
·
·

(4) + CH3 · C · AlCl3


O
·
H3 C · C
·
·

O
Table for reactions
Reaction Reagents Electrophile Product Comments
NO2

Nitration HNO3 / H2SO4 NO2 E+ formed by loss of water


from nitric acid

SO3H

Sulfonation H2SO4 or SO3/H2SO4 SO3 Reversible

Cl

Halogenation Cl2/Fe or FeCl3 Cl+ E+ formed by Lewis acid


removing Clă

Br

Br2/Fe or FeBr3 Br+ E+ formed by Lewis acid

removing Bră
R

Alkylation R-Cl/AlCl3 R+ E+ formed by Lewis acid


removing Clă

ELECTROPHILIC AROMATIC SUBSTITUTION


26 QUIZRR

R-OH/H+ R+ E+ formed by loss of water


from alcohol

C=C/H+ R+ E+ formed by protonation


of alkene
O

CR
Acylation RCOCl/AlCl3 RCO+ E+ formed by Lewis acid

removing Clă
O

CR
RCO2COR/AlCl3 RCO+ E+ formed by Lewis acid

removing RCO2

Ex a m p l e 9
Deduce the structure of compound A, C9H8 from the following experimental data :
A decolorizes Br2 in CCl4 and adds one eq. of H2 under mild conditions, forming B, C9H10.
At high temperature and pressure A adds four eq. of H2.Vigorous oxidation of A yields
phthalic acid, 1, 2ăC6H4(COOH)2.
Solution :
A has 6Ĉ of unsaturation, four of which often signal the presence of benzene ring, as confirmed
by isolation of phthalic acid on vigorous oxidation. Reaction with Br2 and one eq. of H2 indicate there
is a C=C. Addition of three more eq. of H2 further indicates the presence of a benzene ring. So far 5Ĉ
of unsaturation have been accounted for ăthe sixth degree resists reduction and must be a ring with
C=C. Oxidation to the ortho-dicarboxylic acid indicates the ring is fused to the benzene ring. The
structure is indene.

COOH
(O) H2 3H2

COOH
Phthalic acid Indene, (A) Indane, (B) Bicyclo (4.3.0) nonane

ELECTROPHILIC AROMATIC SUBSTITUTION


QUIZRR 27

Ex a m p l e 1 0
(a) Give the structures for all possible chiral compounds, C10H12, that do not decolorize
Br2 and that can be oxidized to phthalic acid.
(b) Identify E, also chiral, with the same formula, but which is oxidized to PhCOOH.
Solution :
(a) The formula reveals a fifth degree of unsaturation in addition to the four of the benzene
ring. This fifth degree of unsaturation must be a ring, not C=C, because the Br2 test is
negative. Production of phthalic acid means the ring is fused to the benzene ring. This fused
ring has the chiral carbon and must be a monoăRăsubstituted fiveămembered or diăRă
substituted fourămembered ring. Only in this way can we account for the additional four
CÊs of the formula.
H Me H
Me
H Et
H
Me
1-Methylindane trans-1,2-Dimethylbenzcyclobutane 1-Ethylbenzcyclobutane

(b) The extra unsaturation is in the single side chain. E is 3ăphenylă1-butene,

CH3  CHCH  CH2


|
Ph

3 .6 Su b s t i t u t e d a r o m a t i c r in g s
1. Electrophiles may attack aromatic rings with functional groups. Performing an electrophilic
substitution on an already substituted benzene compound raises the problem of addition. In
case of a monosubstituted benzene, there are 4 different reactive positions. For a
monosubstituted benzene, the ring carbon atom bearing the substituent is position 1 or
ipso, the next ring atom is position 2 or ortho, position 3 is meta and position 4 is para.
Positions 5 and 6 are respectively equal to 3 and 2.
2. Substituents can generally be divided into two classes regarding electrophilic substitution :
activating and deactivating towards the aromatic ring. Activating substituents or
activating groups stablize the cationic intermediate formed during the substitution by
donating electrons into the ring system, by either inductive effect or resonance effects.
Examples of activated aromatic rings are toluene, aniline and phenol.
3. The extra electron density delivered into the ring by the substituent is not equally divided
over the entire ring, but is concentrated on atoms 2, 4 and 6 (the ortho and para positions).
These positions are thus the most reactive towards an electron-poor electrophile. The highest
electron density is located on both ortho positions, though this increased reactivity might be

ELECTROPHILIC AROMATIC SUBSTITUTION


28 QUIZRR

offset by steric hindrance between substituent and electrophile. The final result of
the electrophile aromatic substitution might thus be hard to predict, and it is usually
only established by doing the reaction and determining the ratio of ortho versus para
substitution.
4. On the other hand, deactivating substituents destablize the intermediate cation and
thus decrease the reaction rate. They do so by withdrawing electron density from the
aromatic ring, though the positions most affected are again the ortho and para ones. This
means that the most reactive positions (or, least unreactive) are the meta ones (atoms 3
and 5). Examples of deactivated aromatic rings are nitrobenzene, benzaldehyde and
trifluoromethylbenzene.
5. Functional groups thus usually tend to favour one or two of these positions above the others;
that is, they direct the electrophile to specific positions. A functional group that tends to
direct attacking electrophiles to the meta positions, for example is said to be meta-directing.

3 .6 .1 Or t h o /p a r a d i r e c t o r s
Groups with unshared pairs of electrons, such as the amino group of aniline, are strongly
activating and ortho/ para-directing. Such activating groups donate those unshared electrons to the
pi system.
NH2

+
A

H + H
NH2 NH2 N NH2
H + H H H
C
A A A A
+ +
C C
H H
+
ăH

NH2

ELECTROPHILIC AROMATIC SUBSTITUTION


QUIZRR 29

When the electrophile attacks the ortho and para positions of aniline, the nitrogen atom can donate
electron density to the pi system (forming an iminium ion), giving four resonance structures (as
opposed to three in the basic reaction). This substantially enhances the stability of the cationic intermediate.
NH2

+
A

+
ăH
H + H
NH2 NH2 N NH2
+
C
+
C C+
H H
A H A H A H A H

NH2

A
Compare this with the case when NH2

the electrophile attacks the meta position.


In that case, the nitrogen atom cannot
donate electron density to the pi system,
A
giving only three resonance contributors.
For this reason, the meta-substituted +
ăH H
product is produced in much smaller +
NH2 NH2 C NH2
proportion to the ortho and para products.
+
Other substituents, such as the C C+
alkyl and aryl substituents, may also H H
donate electron density to the pi system; A H A H A H
however, since they lack an available
unshared pair of electrons, their ability NH2
to do this is rather limited. Thus they only +
A
weakly activate the ring and do not
strongly disfavor the meta position.

ELECTROPHILIC AROMATIC SUBSTITUTION


30 QUIZRR

3 .6 .2 Sp e c i a l Ef f e c t o f H a l o g e n s
l Halogens are ortho/ para directors, since they possess an unshared pair of electrons just as
nitrogen does. However, the stability this provides is offset by the fact that halogens are
substantially more electronegative than carbon, and thus draw electron density away
from the pi system. This destablizes the cationic intermediate, and EAS occurs less readily.
Halogens are therefore deactivating groups.
Hence we can say that halogens are de-activating yet ortho, para directing.
A halogen substituted benzene (C6H5-X) by virtue of the presence of unshared electron pair on
the halogen, exhibits resonance. Thus it can be represented by the resonance hybrid of following :
+ +
X X X

+ +
X X X

ă ă ă

ă
Halogens withdraw electrons through its inductive effect (ă I) and releases electrons through its
resonance or mesomeric effect (+ M). Thus for halogen, the two effects are more evenly balanced and
we observe the operation of both.

3 .6 .3 M e t a d i r e c t o r s
Non-halogen groups with atoms that are more electronegative than carbon, such as the nitro
group (NO2) draw substantial electron density from the pi system. These groups are strongly
deactivating groups. Additionally, since the substituted carbon is already electron-poor, the resonance
contributor with a positive charge on this carbon (produced by ortho/ para attack) is less stable than
the others. Therefore, these electron-withdrawing groups are meta directors. ăCF3, ăCCl3, ăCBr3, ăCl3
are meta directors.
Electron withdrawing groups (EWG) with  bonds to electronegative atoms (e.g. ăC=O, ă
NO2) adjacent to the  system deactivate the aromatic ring by decreasing the electron density on the
ring through a resonance withdrawing effect. The resonance only decreases the electron density at
the ortho- and para- positions. Hence these sites are less nucleophilic, and so the system tends to react
with electrophiles at the meta sites.

ELECTROPHILIC AROMATIC SUBSTITUTION


QUIZRR 31


W ăW ăW ăW W

+ + +

+ +

l The deactivation of the aromatic system also means that generally harsher conditions are
required to drive the reaction to completion. An example of this is the nitration of toluene
during the production of trinitrotoluene (TNT). While the first nitration, on the activated
toluene ring, can be done at room temperature and with dilute acid, the second one, on the
deactivated nitrotoluene ring, already needs prolonged heating and more concentrated acid,
and the third one, on very strongly deactivated dinitrotoluene, has to be done in boiling
concentrated sulfuric acid.

Ex a m p l e s :
When the substituent has at least one strongly electronegative atom and a multiple bond
in conjugation with benzene ring :
Let ăA = B represent the group in which B is more electronegative than A. The highly electronegative
atom pulls the electron pair of the multiple bond which is form withdraws electron from benzene ring
(ăM effect) giving rise to the following five cononical forms :

ă ă
A= B A· B A· B

+
I II

ă ă
A B A= B Aă B

+ + +

+

Evidently electrons are withdrawn by the substituent group from the ring and more so from
ăO ăP positions where electron density declines. Also, the meta positions have relatively more electron
density and therefore electrophilic substitution takes place at the meta positions.

ELECTROPHILIC AROMATIC SUBSTITUTION


32 QUIZRR

The nitro and sulphuric acid groups are examples of the type of electron withdrawing substituents
which one characterized by the presence of a strong electronegative atom attached to another more
electronegative atom by a multiple bond.

Ex a m p l e 1 1
Draw the resonance structure for electrophilic substitution reactions of nitro benzene and
explain why it is meta-directing group ?
Solution : NO2 NO2 NO2
H H H
+ +
E E E E
NO2
ortho
+ +
(A)
NO2 NO2 NO2

+
Para
+ +
(B)
H E H E H E

NO2 NO2 NO2

+ +

Meta H H H
(B) +
E E E
In ortho-para attack of electrophile on nitrobenzene, we are getting two structures (A) and (B)
in which positive charge is appearing on the carbon atom directly attached to the nitro group. As nitro
group is electron withdrawing by nature, it decreases the stability of such product and hence meta
attack is more feasible when electron withdrawing substituents are attached.

Ex a m p l e 1 2
Convert benzene  p-bromonitrobenzene
Solution :

Br Br Br

NO2
Br2 conc. HNO3
+ (Separate)
Fe conc. H2 SO4

NO2

ELECTROPHILIC AROMATIC SUBSTITUTION


QUIZRR 33

Here is a table that shows the effect of substituents on a benzene ring have on both the rate and
orientation of electrophilic aromatic substitution reactions.
These effects are a combination of Resonance and Inductive effects. The effects are also
important in other reactions and properties (e.g. acidity of the substituted benzoic acids).

Here are some general pointers


·O
for recognising the substituent
· NR2 effects :
· NH2 Strongly Activating
· OH
l The H atom is the standard
· OR
and is regarded as having no
O
Activating effect.
·
·

· NHCR ortho
O Moderately Activating direct
·
·

EDG l Activating groups increase


· OCR
the rate
·R

l Deactivating groups
Weakly Activating
decrease the rate
· C = CR 2
H

Reference ·H
l EDG = electron donating
·X Weakly Activating group
O
·
·

· CH l EDG can be recognised by


O lone pairs on the atom
·
·

adjacent to the  system, eg :


· CR
OMe
·S·R
·
·

O Moderately deactivating
exc e p t -R, -Ar or -vinyl
·C·
l
·
·N
EWG Meta directed (hyperconjugation, electrons)
· C · OR
·
·

O
l EWG = electron
· C · Cl
·
·

withdrawing
O
·N·O
·

O
Strongly deactivating
· CF3
+
· NR3

ELECTROPHILIC AROMATIC SUBSTITUTION


34 QUIZRR

(i) The directing power of electron donating group is generally in the following order.
Oă > NH2 > NR2 > OH > OMe, NHAC > Me > X
(ii) The directive power of electron withdrawing group is generally in the following order :
Me3N+ > NO2 > CN > SO3H > CHO >COMe > CO2H

Ex a m p l e 1 3
Arrange the following substances in order to activating a benzene ring.
Benzamide, Aniline, Acetanilide
Solution : O O

NH2 NH ă C ă CH3 C ă NH2

> >

In aniline, benzene ring is directly attached to ăNH2 which releases electrons by resonance effect,
where as the activation of ăNH2 group can be decreased by converting it to anilide and in amide, ring
is attached to electron withdrawing group which deactivates the ring.

3 .6 .4 U s e o f p r o t e c t i n g g r o u p
Groups like ăNH2 and ăOH activate the benzene ring to a great extent and cause undesirable
reactions e.g. nitration of aniline with nitric acid is not possible directly as nitric acid oxidizes and
destroys the highly activated ring. But its nitration can be carried out by use of protecting group as
follows : NH · COCH3 NHCOCH3 NHCOCH3
NH2

NO2
CH2COCl or HNO2
+
(CH3 CO)2 O

NO2

(i) H2 O/H2SO4/
(ii) OH ă

NH2 NH2

NO2
+

NO2

ELECTROPHILIC AROMATIC SUBSTITUTION


QUIZRR 35

In the above reaction, a mixture of ortho and para substituted products is formed. If we desire
to form only ortho product then the para position is blocked with help of blocking reagents e.g. conc.
H2SO4 and tert butyl halides.

NH2 NHAC NHAC NHAC NH2


NO2 NO2
CH3COCl Conc. H4SO4 HNO3 (i) H2 O/H2 SO4/
(ii) OH ă

SO H SO H
3 3
P - position blocked

Please note that the blocking reagents are bulky, hence they do not form ortho substituted
product.
Thus they can be used to block p positions.

3 .6 .5 Ori e n t a t io n i n d i s u b s t i t u t e d B e n ze n e
Both the group present on ring collectively decide the position of incoming third group.
Case I : If both are electron donating group then position of incoming group is decided by the
group with more electron donating power.
Case II : If one group is eă donating and other is eă withdrawing, then position of incoming
group is governed by eă donating group.
Case III : When both are eă withdrawing groups, then position of incoming group is decided by
group which is more deactivating power. But it is difficult to introduce the incoming
group.

Ex a m p l e 1 6

CH3

Conc. OHNO3
?
Conc. H2SO4
NO2

Solution :
ăCH3 is eă donating and NO2 is eă withdrawing. Therefore incoming group position will be decided
by ă CH3.

ELECTROPHILIC AROMATIC SUBSTITUTION


36 QUIZRR

Hence products are


CH3 CH3
O2N

NO2 NO2
NO2

Ex a m p l e 1 7
CN

Conc. H2SO4
?
Conc. HNO3

NO2
Solution :
Both are eă withdrawing group; CN being less withdrawing. Therefore ăNO2 will decide the
position of incoming group.
Hence product is CN
NO2

NO2

Case IV : All other things being equal, a third group is least likely to enter between two groups
in the meta position. This is the result of steric hindrance and increases in importance with the size
of the groups on the ring and with the size of attacking electrophile.
Example :
OCH3
least
most favoured
favoured attack
attack

SO3H

ELECTROPHILIC AROMATIC SUBSTITUTION


QUIZRR 37

3 .6 .5 .1 Se l e c t i v e A d d i t io n
The selective addition for further substitution of disubstituted benzenes can usually be predicted
by looking at the cumulative effects of the substituents.
As a suggested method, look at each of the substituents, label their directing effects, then indicate
the sites where they would promote reactivity with small arrows. Some issues that can arises are shown
by the following worked examples :

CH o,p CH O
3 3
O O CCH3
|| ||
CH3COCCH3
all positions equal
AlCl3

CH3 o,p CH3

CH3 o,p CH3


Br
Br2
substituents reinforce
each other Fe

NO2 m NO2

OH o,p OH
Br
stronger activator Br2
controls Fe

CH3 o,p CH3

CH3 o,p CH3

NO2
activating effects similar, but HNO2
steric effects favour ortho to
the smaller methyl group H2 SO4

C(CH3 )3 o,p C(CH3 )3

ELECTROPHILIC AROMATIC SUBSTITUTION


38 QUIZRR

Ex a m p l e 1 8
The alkylation of phenol and aniline with alkyl halide in presence of AlCl3 gives poor
yields. Explain.
Solution :
In presence of Lewis acid (e.g. AlCl3), the group (ăNH2 or ăOH) becomes electron withdrawing.

Ex a m p l e 1 9
What happens when p-xylene is treated with concentrated sulphuric acid and the resultant
product is fused with KOH ?
Solution :
CH3 CH3 CH3

SO3 H OH
conc. fusion
H2 SO4 with KOH

CH3 CH3 CH3

p-Xylene 2-5. Dimethyphenol

Ex a m p l e 2 0
Explain the product formation in following reaction.

O
Br2
?
Fe

Solution :
Br

O O
and

Br

The oxygen-containing ring can be viewed as an OR group and an R group attached to the
benzene ring. Although both groups are activating. The OR group is more strongly activating. So the
positions ortho and para so it will be faster than the bromination of benzene which requires a
catalyst.

ELECTROPHILIC AROMATIC SUBSTITUTION


QUIZRR 39

Ex a m p l e 2 1
What happens when toluene reacts with Br2 in presence of light ?
Solution :

Br
CH3

+ Br2 hv + HBr, due benzyl free radical stability

3 .6 .6 M a k i n g Po l y s u b s t it u t e d B e n ze n e s
Since the position of electrophilic attack on a substituted benzene ring is controlled by the substituent
already present rather than the approaching electrophile, the order of events in the synthesis of
polysubstituted benzenes need careful planning to ensure success.
The two factors that need to be monitored are :
l regiochemistry (selective addition)
l reactivity (for example Friedel-Crafts reactions are limited to halobenzenes and activated
benzenes)
Tip : Students often get this concept of what component is in control confused. Try thinking about
it in terms of an aircraft (the E+) coming into land at an airport (the Ar-X)... it is the control tower
at the airport on the ground. (-X) that does the „directing‰ of which runway and which ramp the
aircraft should go to.
l Draw the target product and label the substituents with their directing effects
l Look at these directing effects to see which align to allow the introduction of the appropriate
group
l If there is a choice, introduce the more activating group first.... it will make the subsequent
reactions easier
l Remember that some functional groups can undergo reactions that changes their directing
effects
l Check the order of reactions for compatibility
Here are a couple of examples :
1. Here the nitro group could direct to bromine to the correct position
Therefore in planning the synthesis we should use the nitro group to introduce the bromine

Br o,p

NO2 m NO2

ELECTROPHILIC AROMATIC SUBSTITUTION


40 QUIZRR

NO2 Br
Actual Synthesis :
HNO3 Br2
H2 SO4 Fe
NO2
2. Here neither group directs to the correct location of the other.
But since the carboxylate could be introduced by the oxidation of a methyl group, which
directs o, p- we can use the methyl to direct the nitro group to the correct position.

CO2 H m CH3 o,p CH3

NO2 m NO2 m

CH3 CH3 CH3


Actual Synthesis :

CH3Cl HNO3 [O]


AlCl3 H2SO4 KMnO4

NO2 NO2
Note that the oxidation has to be after the nitration to get the correct orientation and that the
Friedel-Crafts alkylation has to before the nitration since nitrobenzene is too deactivated to undergo
alkylation.

3 .6 .7 Ele c t ro p h i li c A r o m a t i c Su b s t it u t i o n o f Po l y c y c l i c A r o m a t i c s
l Polycyclic aromatics such as naphthalene, anthracene etc. react with similar reagents to
those used for benzene
l Typically they are more reactive than benzene
l Reactions are usually less selective than those of benzene and mixtures of products are often
obtained
E E
1
2
+
E

1 - or  ă substitution 2 - or  ă substitution

ELECTROPHILIC AROMATIC SUBSTITUTION


QUIZRR 41

For example, the nitration of naphthalene proceeds to give mainly 1-nitronaphthalene.

NO2

HNO3
H2 SO4

napthalene

This selectivity can be rationalised by drawing the resonance structures for intermediates produced
by attack of the electrophile at the 1- and 2- positions. Attack at the 1-position gives an intermediate
that is represented by 7 resonance contributors of which 4 leaves the aromaticity of the other ring
intact. In contrast attack at the 2-positions gives an intermediate with 6 resonance contributors in
which only 2 have the aromaticity of the other ring intact.

Attack at position-1

E E E
1
2 +
+ +
E

+
(1) (2) (3)

E E E

+ +
(6) (5) (4)

E
structure 1, 2, 6 & 7 have
+ aromaticity of the other ring infact.

(7)

ELECTROPHILIC AROMATIC SUBSTITUTION


42 QUIZRR

Attack at position-2
1 + E E E
2
E
+
+

(1) (2) (3)

E + E E

(6) (5) (4)


There are 6 resonating structure out of them only 2 have aromaticity of the other ring intact.
In anthracene, the electrophile attacks preferentially at the 9 or 10 positions since the arenium
ion formed by the electrophilic attack at any of these positions can have two intact benzene ring in its
canonical forms, while attack of electrophile at any other position (1 or 2) would give arenium ion
having a naphthalene ring in its canonical forms. The resonance energy of 2 benzene rings is more
than the resonance energy of a naphthalene ring.

1 9 8
7
2
(here position 1, 4, 5 and 8 are
3 6 identical, positions 2, 3, 6 and 7
4 10 5 are identical and positions 9 and
10 are same).

3 .6 .8 Ele c t r o p h i li c A r o m a t ic Su b s t i t u t i o n o f H e t e r o a r o m a t i c s
1. Aromatic compounds which contain heteroatoms (e.g. O, N, S) are called heteroa roma tics.
2. The presence of the heteroatom influences the reactivity compared to benzene.
3. The 5-membered ring heterocycles (furan, pyrrole, thiophene) are  -electron rich
aromatics (6  electrons over 5 atoms).
4. This makes them more reactive than benzene (since the aromatics the nucleophilic component
in these electrophilic substitution reactions)
5. For example furan is similar to an activated benzene like methoxybenzene
6. Pyridine is less reactive than benzene (more like nitrobenzene) due to the electronegativity
of N, it is described as a -electron deficient aromatic

ELECTROPHILIC AROMATIC SUBSTITUTION


QUIZRR 43

7. The basic nature of the N atom of pyridine often interferes and interacts with the electrophile
E+ which further deactivates the system.
Compound Name Line Drawing 3D Model

Furan
O

Thiophene
S

Pyrrole
N

Pyridine
N
3 .6 .8 .1 Fi v e M e m b e r e d H e t e ro c y c l i c Ri n g
The 5 membered heterocycles react in a similar way. As an example here is the general pattern
for the electrophilic aromatic substitution reactivity of furan, complete with the 3 important resonance
structures that justify the reactivity of the 2-position.
+
E +

O O E
Mechanism :
+
+ E E E
E
+
O O H O H O H

H H

E E
3ăsubstitution:
+
+
N N

H H
(Stable contributorăoctet of
every atom is complete)

ELECTROPHILIC AROMATIC SUBSTITUTION


44 QUIZRR

Furan and thiophen are also activated towards electrophiles and react predominantly at the 2-
position. Theory is similar to that for pyrrole, namely, that the heteroatom is able to delocalize the
positive charge on the transition state. Since oxygen accommodates a positive charge less readily than
nitrogen, furan is less reactive than pyrrole. Just as phenol is less reactive than aniline. The +M effect
of sulphur is smaller than that of oxygen because the overlap of the differently sized păorbitals of
carbon and sulphur is less than furan. Thus, the reactivity order of 5ămembered heterocyclics towards
electrophilic substitution would be
pyrrole > furan > thiophene

3 .6 .8 .2 Six -M e m b e r e d H e t e r o c y c l i c Ri n g s
The principles governing the reactivity of these compounds are illustrated by reference to pyridine.
The transition states for substitution at the 3ă and 4ăpositions can be represented as the hybrids.
H H H
+

E E E
3ăsubstitution:
+ +
N N N

H E H E H E

+ +
4ăsubstitution:
+
N N N

In each case, the positive charge is less well accommodated than in reactions on benzene because
nitrogen is more electronegative than carbon. Hence, both the 3ă and 4ăpositions are deactivated, the
latter more strongly because of the high energy of the contributing structure which contains divalent
positive nitrogen. The 2ăposition resembles the 4ăposition, as reference to the appropriate resonance
structures will show.

3 .6 .9 . Re a c t io n s o f B e n ze n e a n d I t s H o m o l o g u e s
(i) Benzene on vigorous oxidation (combustion) gives CO2 and H2O.

15
C6 H6 (l)  O2 (g)  6CO2 (g)  3H2O(l )
2
Benzene on oxidation by air at 723 K in presence of V2O5 gives maleic anhydride.
O
C
V2 O5 CH
+ 9 O2 O + 2H2O + 2CO2
2 at 723 K CH C
O
ELECTROPHILIC AROMATIC SUBSTITUTION
QUIZRR 45

(ii) Side chain oxidation of alkyl benzenes


An alkyl benzenes (irrespective of the chain length) on oxidation by acidified or alkaline KMnO4
(followed by acidification) are degraded to benzoic acid provided they have a benzylic hydrogen.
CH3 CO2H

KMnO4, H +

CH2CH3 CO2 H

KMnO4, H +

H3 C CH3
CH CO2H

KMnO4, H +

CH2CH2CH2R CO2H

KMnO4, H +

Even if the side chain contains some functional groups, whether they are electron pushing or
electron withdrawing, are also degraded to benzoic acid on oxidation.

CH2CH2ăCHăCH3 CO2 H
·

NO2
KMnO4, H +

CH=CH2 CH=CH

and
are also oxidised by KMnO4, H+ or KMnO4, OHă to give benzoic

acid due to oxidative cleavage of C=C and ăC=Că

ELECTROPHILIC AROMATIC SUBSTITUTION


46 QUIZRR

Compounds that do not contain a benzylic hydrogen will not get oxidised to benzoic acid.
R

R·C·R

KMnO4, H +
No reaction

Compounds which contain, two alkyl carbons bearing benzylic hydrogen are oxidised to give two
ăCO2H groups on the benzene ring.

CH2 R CO2H
KMnO4, H +

CH2 R CO2H
Phthalic acid

CH2 R CO2H
KMnO4, H +

CH2 R CO2H
Isophthalic acid

CH2 R CO2H

KMnO4, H +

CH2 R CO2H
Terephthalic acid

CO2H
KMnO4, H +

CO2H

CO2H
KMnO4, H +

CO2H

ELECTROPHILIC AROMATIC SUBSTITUTION


QUIZRR 47

(iii) Addition reactions :


Benzene is so stable that it gives addition reactions much less readily. It can be reduced by
catalytic hydrogenation, when three molecules of hydrogen add to it in presence of Pt as a
catalyst.

Pt
+ 3H2
423 K, 100 atm

Addition of 3 molecules of chlorine also takes place to one molecule of benzene in presence of
sunlight. The product formed is 1, 2, 3, 4, 5, 6ăhexachlorocyclohexane (also called as benzene
hexachloride). This compound is used as an important insecticide and is sold commercially in the
name of BHC or gammaxene.

Cl
Cl Cl
Sunlight
+ 3Cl2

Cl Cl
Cl

Ex a m p l e 2 2
Find the product

CH = CHCHO

KMnO4, H +
Product ?

Solution : COOH

ELECTROPHILIC AROMATIC SUBSTITUTION


48 QUIZRR

M ISCEL LA N EOU S EX A M PLES


Ex a m p l e 1

CH3 · CH = CH2
A
HF or BF3

Identify A and also the electrophile involved in the reaction.


Solution :
+ +
H
CH3 · CH = CH2 CH3 · CH · CH3
CH3 · CH · CH3

+
CH3 · CH · CH3 +

(A)

is iso propyl benzene or cumene and electrophile involved is CH3 ă CH ă CH3 (isopropyl
carbocation). Rearrangements are also possible in Friedel Crafts reactions when benzene is treated with
n-butyl chloride and Lewis acid the product obtained is isobutyl benzene.

Ex a m p l e 2
Synthesize mănitroacetophenone from benzene.
Solution :
COCH3 COCH3

(CH2CO)2 O, anty AlCl2 conc. HNO3


F.C. alkylation conc. H2 SO4
NO2
Acetophenone

Ex a m p l e 3
In the following reaction,

O
conc. HNO3
X
conc. H2SO4
N

t h e st r u ct u r e of t h e m aj or pr odu ct ‘X’ i s

ELECTROPHILIC AROMATIC SUBSTITUTION


QUIZRR 49

(a) (b) O2 N
O
NO2 O

N
N
H
H

(c) O (d) O

N N
O2 N
H H
NO2

Solution :
The ring to which ăNH group is attached is activated due to the lone-pair on N; while the ring
O
·

to which C is attached is deactivated. Hence, the electrophile would go to the para-position of the
·
·

activated ring.
 (b)

Ex a m p l e 4
Irradiation of an equimolar mixture of cyclohexane and PhCH3 gives mostly cyclohexyl
chloride with Cl2 and PhCH2Br with Br2. Explain.
Solution :
In these competitive reactions the reactivities of cyclohexane and toluene are compared ClĆ, being
more reactive and less reactive than BrĆ, reacts with the kind of H present in greatest number, which
in this case is one of the twelve equivalent HÊs of cyclohexane. The less reactive and more selective BrĆ
reacts with the most reactive H, in this case one of the three alkyl HÊs of PhCH3.

Ex a m p l e 5 O

Starting with benzene and succinic O anhydride synthesise -tetralone.

O O

ELECTROPHILIC AROMATIC SUBSTITUTION


50 QUIZRR

Solution : O
O
C
AlCl3 CH2
+ O
FCR
CH2
O Zn ă Hg
C
conc. HCl
HO O SOCl2

CH2
AlCl3 CH2
FCR
CH2
O C
 ă tetralone Cl O

Ex a m p l e 6
Compound (A), B and (C) are three isomeric derivatives of benzene. Identify which is Oă,
mă or pă from the products of nitration ?
conc. HNO
A  3
two mononitro product
H 2SO 4

conc. HNO
B  3
three mononitro product
H 2SO4

conc. HNO
C  3
one mononitro product
H 2SO 4

Solution :
CH3 CH3 CH3

CH3 CH3 CH3


HNO3
(A) +
H2 SO4
NO2

NO2

CH3 CH3 CH3 CH3


NO2
HNO3
(B) + +
H2 SO4
CH3 CH3 CH3 O H CH3
2
NO2

ELECTROPHILIC AROMATIC SUBSTITUTION


QUIZRR 51

CH3 CH3
NO2
HNO3
(C)
H2 SO4
CH3

CH3

Ex a m p l e 7
Write the structure of the major product (only mono-substitution is involved in each case).

F
CH2 ă Cl
+ AlCl3
?

Solution :
F

CH2 ă Ph

Ex a m p l e 8
Complete the following :

H3C
BF3
+ CH2 ? + HF
(a) (b) ?
H3C

C(CH3 )3

Br2 HNO3
(c) ? (d)
acetic acid
CH(CH3 )2

ELECTROPHILIC AROMATIC SUBSTITUTION


52 QUIZRR

OCH3
CF3

NH2 H2 SO4
Br2 + (CH3)2C = CH2 ?
(e) ? (f)

O2 N
CH3

Br2
(g) OH ?
CHCl3

Solution :

CH3 CH3

(a) CH3 (b)

C(CH3 )3
H Br

(c) (d)
CH(CH3 )2
H Br NO2

OCH3
C
CF
3
C(CH3)3
NH2
(e) (f)

O2 N Br
CH3

OH
(g)
Br

ELECTROPHILIC AROMATIC SUBSTITUTION


ALCOHOL
PHENOL
ETHER
QUIZRR 3

ALCOHOLS

1. Physical Properties :

Ć The polar nature of the O-H bond (due to the electronegativity different of the atoms) results
in the formation of hydrogen bonds with other alcohols molecules or other H-bonding systems
(e.g. water). The implications of this are :
Ć High melting and boiling points compared to analogous alkanes
Ć High solubility in aqueous media

2. Structure :

Ć The alcohol functional group consists of an O atom bonded to a C atom and a H atom via
 bonds.
Ć Both the C-O and the O-H bonds are polar due to the high electronegativity of the O atom.
Ć Alcohols are bent shaped molecules. The central ÂOÊ atom is also sp3 hybridised and the
C-O-H bond angle is 105Ĉ.

105Ĉ
R H

3. Reactivity :

Ć The alcohol O atom are a region of high electron, density due to the lone pairs.
Ć Alcohol oxygen atoms are Lewis bases.
Ć So alcohols can react as either bases or nucleophiles at the oxygen.
Ć There is low electron density on H atom of the O-H group alcohol, i.e. H+ character.
Ć So alcohols are acidic (pKa ~ 16).
Ć The -OH group is a poor leaving group and needs to be converted to a better leaving group
before substitution can occur.

4. Solubility of alcohols :

The first three members are completely miscible with water. The solubility rapidly decreases with
increase in molecular mass. The higher members are almost insoluble in water but are soluble in
organic solvents like benzene, ether etc. The solubility of lower alcohols is due to the existence of
hydrogen bonds between water and polar -OH group of alcohol molecules.Phenols too are sparingly
soluble in water. The -OH group in alcohols contains a hydrogen bonded to an electronegative
oxygen atom. Thus they form hydrogen bonds with water molecules.

ALCOHOLS, PHENOLS & ETHERS


4 QUIZRR

H H
| |
R · O - - -H · O - - -H · O - - -H · O - - -
| |
H R

The solubility of alcohols in water decreases with increase in molecular mass because greater
carbon content makes alcohol less hydrophilic. In addition, among the isomeric alcohols, the
solubility increases with branching of chain. As the surface area of the non-polar part in the
molecule decreases, the solubility increases.

5. Acidity :

Ć Due to the electronegativity of the O atoms, alcohols are slightly acidic


Ć The anion derived by the deprotonation of an alcohol is the alkoxide.
Ć Alkoxides are important bases in organic chemistry.
Ć Alcohols react with Na (or K) like water to give the alkoxide :

2 RăOăH + 2 Na  2 NaOR + H2

2 HăOăH + 2 Na  2 NaOH + H2

NOTE : Alcohols containing more than one ăOH group linked to the same carbon atom are
highly unstable and lose water to form more stable aldehydes or ketones.

OH H
ăH2 O
CH3 CH OH CH3 CH = O
Unstable Ethanal

EXCEPTION : Few exception to the above point can be noted.

H
O Cl O
H2 O
(1) Cl3C C Cl C CH

H Cl O
H

ALCOHOLS, PHENOLS & ETHERS


QUIZRR 5

In this compound, owing to the presence of hydrogen bonding, alcoholic content is found to
be considerably higher.

H
Oă O
O

(2) Oă HOH

O
Oă O H
minhydrin hydrate

Classification of alcohols :
(1) Monohydric alcohols  only one ăOH group is present.
For example
CH3 CH3 OH OH
Ethanol

Cyclohexanol

(2) Dihydric alcohol : two ăOH groups are present in same molecule.
OH
CH2 · CH2 OH
| |
OH OH
1, 2-Ethanediol or
Ethylene glycol
1,2-Dihydroxy benzene
or catechol

(3) Trihydric alcohols : three ăOH groups present.

OH CH · C H · CH2
| | |
OH OH OH
Glycerol

OH OH
1,3,5-Trihydroxy benzene
or phloroglucinol

ALCOHOLS, PHENOLS & ETHERS


6 QUIZRR
Example 1

Explain the following :


(a) When equal volumes of ethanol and water are mixed, the total volume is less than
the sum of the two individual volumes.
(b) Propanol (MW = 60) has a higher boiling point than butane (MW = 58)
Solution :
(a) The water solubility of alcohols is attributed to intermolecular H-bonding with H2O. As the
molecular weights of the alcohols increase, their solubility in water decreases, because greater
carbon content makes the alcohols less hydrophilic. Conversely, their solubility in hydrocarbon
solvents increases.
(b) Hăbonding between ethanol and water molecules.
(c) Alcohol molecules attract each other by relatively strong Hăbonds and somewhat weaker
dipole ă dipole interactions, resulting in a higher bp. In alkanes only weaker van der WaalsÊ
attractive force must be overcome to vaporize the hydrocation.

PREPARATION OF R-OH

A summary of all the methods of preparation is given below.

ă
H2O or HO
Hydrolysis of Alkyl Halides CăX C OH

+
H
Hydration of Alkenes C=C H2O H C C OH

1.BH3
Hydroboration-Oxidation of Alkenes C=C H C C OH
2. H2O2/NaOH

Oxymercuration Demercuration
of Alkenes
1.Hg(OAc)2/THE/H2O
C=C H C C OH
2. NaBH4

ALCOHOLS, PHENOLS & ETHERS


QUIZRR 7

Organometallics with Aldehydes


and Ketones
O OH
R Li
or + C R C R´
work-up
R Mg X with H+
R R´ R

O OH
LiAlH4
or + C H C R´
Reduction of Aldehydes work-up
NaBH4 with H+
and Ketones R R´ R

O OH
R Li
or + C R C R´
Organometallics with Esters work-up
R Mg X with H+
R R´ R

O OH

LiAlH4 + C H C R´
Reduction of Esters work-up
with H+
R´ OR´´ H

O OH

Reduction of Carboxylic LiAlH4 + C H C R´


work-up
with H+
Acids R´ OH R

RMgX
+ O R C C OH
Opening Epoxides or work-up
RLi with H+

ALCOHOLS, PHENOLS & ETHERS


8 QUIZRR
1. From alkyl Halides

Alkyl halides when boiled with aqueous solution of KOH or moist silver oxide furnish alcohols.

C2H5Br + KOH (aq)  C2H5OH + KBr

Primary alkyl halides give good yield of alcohols. However, tertiary alkyl halides mainly undergo
dehydrohalogenation resulting in formation of alkenes.

CH3 CH3

CH3 C Br + KOH CH3 C CH2 + KBr + H2O

CH3

2. FROM ALKENES

2.1 Hydration of Alkenes

CH3 CH CH2 + H2O CH3 CH CH3

OH

MECHANISM :

CH3 CH3
+
H
CH3 C CH = CH2 + H2O CH3 C CH · CH3
+
H H

CH3 CH3

H3C C CH2 + CH3 H3C C CH2 CH3


+

OH

Some points :
(1) Addition is predicted by MarkonikovÊs rule
(2) Reaction proceeds via protonation to give more stable carbocation intermediate.

ALCOHOLS, PHENOLS & ETHERS


QUIZRR 9

2 .2 Hydroboration / oxidation of Alkenes

1.BH3
C C + H2O H C C OH
2.BH3/NaOH

Alkenes react with diborane to form trialkyl boranes, which upon treatment with alkaline H2O2
give alcohols via anti-Markowni koffÊs addition of water. Also, it is a ÂsynÊ reaction.

Mechanism :

CH3 CH CH2 CH3 CH2 CH2

BH2
H B H

H2O2
3(CH3 CH2 CH2 OH ) + B(OH)3 ă CH3 (CH2 ) B (CH2 )2 CH3
OH

(CH2 )2 CH3

Now, let us also see the mechanism of the following reaction.

B  OH 3
NaOH
R3 B 
 3ROH 
H2O2

ă
H O O H + OH H O Oă + H2O

ă
R O OH R
ă
R B R B O O H

R R

Here lone pair - lone pair


repulsion is present, hence
the bond breaks

ă
OH

ALCOHOLS, PHENOLS & ETHERS


10 QUIZRR
OR R
Repeat
B OR R B OR
it again
O 2 times

R
ă
OH

2ROH + B(OH)3

Some Important Points :

B2H6/THF
(1) R C CH (R HC CH2 )3B

protolysis
CH3COOH Oxidation
H2O2/NaOH

R CH CH2
R CH CH OH

tautomerism

RCH2CHO

(1) B2H6/THF
R C C R R C C R
(2) CH3CHOOH

H H

(2) Double bond can also be selectively reduced by B2H6/THF at the least hindered
position.
For example :
unhindered

CH = CH2 CH2 CH2 BH2 CH2 CH3


(1)

B2H6/THF CH3COOH

hindered
alkene
CH · CH2 CH(CH3)2 CH(CH3)2

CH3

ALCOHOLS, PHENOLS & ETHERS


QUIZRR 11

(i) (BH3)2
(2) (ii) H2O2,OH
CH3 H

H3C H OH
1-methyl cyclopentene 2-methyl cyclopentene
syn-addition

2 .3 By oxymercuration-dimercuration

Alkenes can be converted into alcohols by oxymercuration-demercuration reaction. In this


reaction, addition of water takes place according to MarkownikoffÊs rule.

CH3COO 2 Ag
CH2  CH  CH2  H2 O   CH3  CH  CH3
NaBH4 |
OH

Mechanism :

H
H H H
O O+

CH3 CH CH2 CH3 CH CH2


+
Hg HgOCOCH3
ăH +

OH OH
NaBH4
CH3 C CH2 CH3 C CH2

H HgOCOCH3

ALCOHOLS, PHENOLS & ETHERS


12 QUIZRR
Example 2

D 2O,  CH3COO  Hg
CH 3  CH 2  CH 2  2  ?
Na BD4

Solution :
OD
D2O
CH3 CH CH2 CH3 CH CH2
(CH3COO)2Hg

HgOCOCH3

NaBD4

OD

CH3 CH CH2 D

Example 3

CH3 CH3 HgOCOCH3


Hg(OAC)2
H3C C CH CH2 H3C C CH CH2
+H2O

CH3 CH3 OH

NaBH4

CH3

H3C C CH CH3

CH3 OH

3. FROM ALDEHYDES AND KETONES

3.1 By using Grignard Reagents


When a solution of an alkyl halide in dry ethyl ether, (C2H5)2O, is allowed to stand over turnings
of metallic magnesium, a vigorous reaction takes place. The resulting solution is known as Grignard
reagent. It is one of the most useful and versatile reagents known.

Dry ether
RX  Mg  R MgX
Alkyl halide Alkyl magnesium halide

ALCOHOLS, PHENOLS & ETHERS


QUIZRR 13

The carbon-magnesium bond is covalent but highly polar, with carbon pulling away electrons
 
from electropositive magnesium but the magnesium-halogen bond is essentially ionic, R M gX

The Grignard reagent belongs to a class of compounds called organometallic compounds, in which
carbon is bonded to a metal like lithium, potassium, sodium, zinc, mercury etc. Each kind of
organometallic compound has its own set of properties
But whatever the metal, it is electro negative than carbon and the carbon metal bond is always
highly polar. Although the organic group is not a full fledged carbanion but has considerable
carbanionic character. The, organometallic compounds can serve as a source of carbon bearing
negative charge.
So, when Grignard reagent is added to carbonyl compounds, the organic group attaches to carbon
and magnesium to oxygen.

+ ă ă +
C = O + R M gX C OMgX

H2O

C OH + Mg(OH)X

HX
Mg (OH) X 
 MgX 2  H2 O

Products formed by Grignard synthesis :


(a) Formaldehyde yields primary alcohol
(b) Any other aldehyde yields secondary alcohol.
(c) Ketones yield tertiary alcohols.

NOTE :
(1) Sterically hindered t-alcohols cannot be prepared by grignard reagent, but can be
prepared using lithium alkyls. This is because lithium compounds are usually more
reactive and yield a better product.

ALCOHOLS, PHENOLS & ETHERS


14 QUIZRR
O
tBuMgBr
tBu C tBu X
tBu
tBuLi
tBu C OH

tBu

(2) For compounds like CH2 CH2 , the reaction is :

OH Cl

Mg
CH2 CH2 CH2 CH2 MgCl
ether

OH Cl OH
CH2 CH2

OH Cl

CH2 CH3 CH2 CH2 OMgCl


+
OH Cl

(3) Reaction of RMgX with epoxide ring

O OMgX
+ ă
R (MgX)
H3C C CH2 CH3 C CH2R
SN2
CH3 CH3

H3O+

CH3

CH3 C CH2R

OH

ALCOHOLS, PHENOLS & ETHERS


QUIZRR 15

O
OMgX
(4) RMgX
CH2 CH CH CH2 CH2 CH CH CH2

{1Ĉ alcohol will be formed because of the allylic nature of 2Ĉ carbon}

3 .2 By reduction

The chemical reducing agents like Lithium aluminium hydride (LiAlH4), H2/Ni, B2H6/THF etc. are
used to produce alcohols.

(i) LiAlH4

LiAlH
(a) R · CHO 
4 · R · CH2 OH

LiAlH
(b) C  O 
4 CH · OH

LiAlH
(c) R · COOH 
4 R · CH2OH

LiAlH
(d) R´ · COOR 
4 R´ · CH2 · OH ·  ROH

LiAlH
(e) R · COCl 
4 R · CH2 · OH

(f)  RCO 2 O 4LiAlH


 R CH2 · OH

O LiAlH4
(g)
OH

CH2 CH2
LiAlH4
(h) R CH C=O R CH CH2 CH2 CH2 OH
O
OH
 Lactone

ALCOHOLS, PHENOLS & ETHERS


16 QUIZRR
Note : LiAlH4 does not reduce double or triple bonds however if double or triple bonds is in
conjugation with (C = O) group, then double or triple bond is also reduced.

LiAlH
CH2  CH · CH2 · CHO 
4 CH2  CH · CH2 · CH2 OH

LiAlH
Ph · CH = CH · CHO 
4 Ph · CH2 · CH2 · CH 2 OH

O OH

LiAlH4

(ii) Using NaBH4


It can reduce all the groups which are reduced by LiAlH4 but cannot reduce carboxylic acid
and ester. It can also reduce only the double bond which are in conjugation with C = O
group.

(iii) By H2/Ni
It can reduce all the groups reduced by LiAlH4 and it can also reduce double and triple
bonds in the carbonyl compounds irrespective of the position.

H / Ni
CH3 · CH  CH · CHO 
2  CH3 · CH 2 CH2 CH2 · OH

H / Ni
CH2  CH · CH2 · CHO 
2  CH3 CH2 CH2 CH2 · OH

Note : Hydrogenolysis (cleavage by hydrogen) of an ester requires more severe conditions


than simple hydrogenation of (addition of hydrogen to) a carbon-carbon double bond. High
pressure and elevated temperatures are required : the catalyst used most often is a mixture
of oxides known as copper chromite, of approximately the composition CuO. CuCr2O4. For
example

CH3  CH2 10 COOCH3   CH3  CH2 10 CH2 OH  CH3 OH
H , CuO CuCr O
2 2 4
150, 5000 lb/in2
 Methyl dodecanoate  1 Dodecanol

ALCOHOLS, PHENOLS & ETHERS


QUIZRR 17

Chemical reduction is carried out by use of sodium metal and alcohol, or more usually by
use of lithium aluminium hydride. For example

CH3  CH2 14 COOC2H5  CH3  CH2 14 CH2 OH


4 LiAlH

 Ethyl hexadecanoate  1Hexadecanol

We have learnt that one molecule of an ester on reaction with excess of Grignard reagent
followed by acidification produces a tertiary alcohol.

O · R´ R CH3
(i) CH3MgX(excess)
R C + R´OH
(ii) H3O+
O OH CH3

This simple reaction has a beautiful application in synthesis of cyclic tertiary alcohols, from
which alicyclic hydrocarbons can be synthesized.

For Example :

CH2 CH2 R
MgBr
+
H3O CH2 C
RCO2Et + (CH2)5
CH2 CH2 OH
MgBr
H3O+

H2/Ni

Example 4

(a) Why are ethers such as Et2O used as solvents for (i) BF3 and (ii) RMgBr ?
(b) Compare the Lewis basicities of tetrahydrofuran, diethyl ether and diăisopropyl
ether and give your reasoning.

ALCOHOLS, PHENOLS & ETHERS


18 QUIZRR
Solution :
(a) Because of their unshared eă´s, there are also Lewis bases capable of reacting with Lewis
acids such as BF3 and RMgBr to form coordinate covalent bonds.

+ ă
(i) Et2O + BF3 Et2O BF3

R
(ii) 2Et2O + RMgBr Et2O Mg OEt2
Br

Note that two molecules of ether coordinate tetrahedrally with one Mg2+.

(b) In addition to the amount of electron density on the nucleophilic site, steric affects also
influence basicities.The greater the steric hindrance encountered in the formation of the
coordinate bond, the weaker is the Lewis basicity. In tetrahydrofuan, the R groups (the sides
of the ring) are „tied back‰ leaving a very exposed O atom free to serve as a basic site. The
2Ĉ R groups in diăisopropyl ether furnish more steric hindrance than do the 1Ĉ ethyl groups
in ethyl ether. The order of decreasing Lewis basicity in thus tetrahydrofuran > ethyl ether
> iăpropyl ether.

(iv) By B 2H6/THF

2H2 BH3

O O O+
(THF)
ă
BH3

This reduces all the groups reduced by LiAlH4 except acid chloride (ăCOCl). It does reduce
the double / Triple bond but Selectively.

4. CHEMICAL PROPERTIES OF ALCOHOLS

In alcohols, ăOH group is the functional group. The oxygen atom of ăOH group polarize both the
C ă O bond and the OăH bond of any alcohol. Hence, alcohols can be classified into three types :

1. Reactions involving the cleavage of oxygen hydrogen bond.


2. Reactions involving the cleavage of carbon-oxygen bond.
3. Reactions involving both the alkyl and the hydroxyl groups.

ALCOHOLS, PHENOLS & ETHERS


QUIZRR 19

I. Reactions involving cleavage of oxygen-hydrogen bond.

Na
C2H5ONa + 1 H2
2

NaOH
RONa + 1 H2O
2

RMgX X
R · H + Mg
OC2H5

C2H5OH

CH3COOH/H+
CH3COOC2H5 + H2O
Esterification

CH3COCl
CH3COOC2H5 + HCl
Acylation

(CH3CO)2O
CH3COOC2H5 + CH3COOH
Acylation

(i) The first three reactions illustrate the reactions of alcohols as acids.

ă the order of acidic strength of different types of alcohol is

CH3OH > 1Ĉ > 2Ĉ > 3Ĉ

ă the order of acidity for some compounds

H2O > ROH > HC = CH > NH > RH

ALCOHOLS, PHENOLS & ETHERS


20 QUIZRR
(ii) Mechanism for esterification
+
O O H OH
+ + H
H (fast) slow
R C OH R C OH R C O
R´OH R´
OH

+
H (fast)

+
OH2
fast R C OR´ +
R C OR´ R C OR´ R C OR´
ăH+ + O H
O OH OH

(iii) Acylation : In this reaction, R ă OH should be 1Ĉ or 2Ĉ.


ă
Mechanism : O O
+
CH3 C O R CH3 C O R CH3COOR

Cl H Cl H
I llustration :

CH3 O CH3 O

CH3 C OH + R C CH3 C O C R + HCl

CH3 Cl CH3

So, HCl formed above again attacks the oxygen atom.


O CH3 O CH3
HCl +
R C O C CH3 R C O C CH3 R · COOH

CH3 H CH3

(iv) O O

ROH + CH3 C O C CH3

ă
O O

CH3COOR + CH3COOH CH3 C O C CH3

+O R

H
ALCOHOLS, PHENOLS & ETHERS
QUIZRR 21

 | 
II. Reactions involving cleavage of carbon-oxygen  R · C ·· OH  Bond
 | 
 

1. Reaction with hydrogen halide

HCl/ZnCl2
C2H5Cl + H2O

HBr
C3H5OH C2H5Br + H2O
(NaBr + Conc. H2SO4)
HI
C2H5I + H2O
(KI + H3PO4)

The order of reactivity of various halogen acids is HI > HBr > HCl

I llustration :

H
+ +
CH3 CH CH CH2 ăH2O
CH3 CH CH CH2

OH

+
CH3 CH CH CH2

Hence, product formed is a mixutre of

CH3 · C H · CH = CH2 + CH3 · CH = CH · C H2


| |
Br Br

2. Reaction with phosphorus halides

PCl5
C2H5Cl + POCl3 + HCl
PCl3
C2H5Cl + H3PO3

C3H5OH
PBr3 C2H5Br + H3PO3
(P + Br2)
PI3
C2H5I + H3PO3
(P + I2)

ALCOHOLS, PHENOLS & ETHERS


22 QUIZRR
3. Reactions with Thionyl Chloride

Pyridine


ROH + SOCl2 RCl + SO2 +HCl
Thionyl
chloride

The order of reactivity is 3Ĉ > 2Ĉ > 1Ĉ. The reaction in presence of pyridine shows inverted
configuration, while in absence of pyridine, it proceeds via SNi mehcanism showing
retention of configuration.

III. Reactions involving alkyl group as well as hydroxyl group

1. Acidic Dehydration

Conc. H SO
CH3 CH2 OH 
2 4
 CH2  CH2  H 2O

Al O
CH3 CH2 OH 
2 3  CH  CH  H O
2 2 2

The above reaction with conc. H2SO4 proceeds by E1 mechanism. Hence, for

+
H
+
CH3 OH CH2

Stability of Rings 6 > 7, 5 > 8, 9 > > 4 > 3


The ease of dehydration is 3Ĉ > 2Ĉ > 1Ĉ

2. Oxidation
Alcohols undergo oxidation with oxidizing agents (in neutral or acidic or alkaline medium)
such as CrO3, K2Cr2O7, KMO4 or HNO3.

ALCOHOLS, PHENOLS & ETHERS


QUIZRR 23

(i) PCC {Mixture of pyridine, HCl and CrO3}

ClCrOă3
Pyridinium Chloro Chromate
N+
H

This reagent is used for Âcontrolled oxidationÊ. It oxidizes 1Ĉ alcohols to aldehydes, but
cannot convert them to carboxylic acids.

P.C.C.
· CH2OH  · CHO

P.C.C.
R 2CHOH  · R 2CO

It is a mild oxidizing agent

P.C.C.
3 alcohol  No reaction

(ii) MnO2 : It can selectively oxidise 1Ĉ allylic or 1Ĉ benzylic alcohols into aldehydes.

CH2OH CHO
MnO2

MnO
CH2 = CH · CH2 OH 
2  CH
2 = CH · CHO

It does not oxidize or rearrange the double bonds.

Example :

OH
| MnO2
CH3 · CH · CH2 · CH = CH · CH2 OH  

OH
|
CH3 = CH · CH2 OH
| |
CH  CH · CH · H

ALCOHOLS, PHENOLS & ETHERS


24 QUIZRR
(iii) H2CrO4 (Jones Reagent)
This reagent in presence of sulphuric acid is used for oxidation of alcohols. This is more
oxidizing as compared to the above reagents, hence 2Ĉ alcohols react to form ketones.
However, it does not oxidize double or triple bonds.

OH O
| ||
· C · CH · CH3  CH3 · C ·
CH3 · C · · C · C · CH3

(iv) K2Cr2O7 or KMnO4 or HNO3 are strong oxidizing agents and directly oxidize 1Ĉ alcohols
to carboxylic acids.

O
[O]
(a) CH3 CH2 OH CH3 C H

[O]

CH3COOH

CH3
[O]
(b) CH3 CH OH CH3 C O + H2O

CH3

{under prolonged treatment with oxidizing agent, they get oxidized to acids with lesser
number of carbon atoms than the alcohol}

[O]
CH3 · C  O 
 CH3COOH + CO2 + H2O
|
CH3

(c) 3Ĉ alcohols are not oxidised with mild oxidizing agents in aqueous, alkaline or under
neutral conditions. However, when oxidation is carried out under acidic conditions,
tertiary alcohols first undergo dehydration to alkene. The alcohol formed is then
oxidized to ketone which is finally oxidized to carboxylic acid with lesser number of
carbon atoms that the starting alcohol.

ALCOHOLS, PHENOLS & ETHERS


QUIZRR 25

CH3 CH3
Hot conc HNO3
CH3 C OH CH3 C CH2

CH3 [O]

CH3
[O]
CH3COOH + CO2 + H2O C = O + CO2 + H2O
CH3

3. Dehydrogenation (Reduced with hot reduced copper)

(i) 1Ĉ alcohol.

H H
| |
Cu / 575 K
R · C · OH   R · C  O + H2
|
H

(ii) 2Ĉ alcohol

R R
| |
Cu / 575
R · C · OH   R · C  O + H2
|
H

(iii) 3Ĉ alcohol

CH3
|
Cu / 575 K
(CH3 )3 C · OH   CH3 · C · CH2 + H2 O

Note: The reagent can also be given a name of Âred hot copper tubeÊ in the question.
However, it is the same as we have used above.

ALCOHOLS, PHENOLS & ETHERS


26 QUIZRR
Example 5
Give the product of each reaction
(a) CH3CH2CH = CHCH2OH + MnO2
(b) m·O2NC6H4CH2COOH + LiAIH4
(c) Ph2C = CHCH3 + BH3, THF followed by H2O2/OHă
(d) PhCOCH2CH2Br + LiAID4, followed by H2O
(e) PhCH = CHCH(OH) CH3 + PBr3/
Solution :
(a) CH3CH2CH = CHCHO
(b) măH2NC6H 4CH2CH2OH
(c) Ph2CHCH(OH)CH3
(d) PhCD(OH)CH2CH2D
(e) PhCH = CHCHBrCH3

Example 6

OH

+7 (i) C6H5CH2 ă MgBr Zn/Hg ă conc. HCl NBs/h


Mn
(A) (B) (C) (D)
(ii) HOH 
alc.KOH CF3 ăCOOH
(E) (F) (G)
 CH3OH/HCl

Solution :

OH O O

+7 (i) C6H5CH2 ă MgBr Zn/Hg ă conc. HCl


Mn
(ii) HOH 

C6H5 C6H5

NBs/h alc.KOH CF3 ăCOOH CH3OH/HCl


C6H5 O CH3
Br 

O OH C6H5
C6H5 C6H5

ALCOHOLS, PHENOLS & ETHERS


QUIZRR 27

O xidative Cleavage of Diols

OH OH

C C + HIO4 C=O + O=C + HIO3 + H2O

Reaction type : Oxidation-reduction

Ć 1, 2- or vicinal diols are cleaved by periodic acid, HIO4,


OH
into two carbonyl compounds. O O
I
Ć The reaction is selective for 1, 2-diols.
O O
Ć The reaction occurs via the formation of a cyclic periodate ester
CăC
Ć This can be used as a functional group test for 1, 2-diols.

Ć The products are determined by the substituents on the diol.

Example 7

Identify, each of the following glucose derivative

A + 5HIO4  4HCOOH + 2HCHO

B + 3HIO4  2HCOOH + 2OHC ă COOH

Solution :

A  HOCH2CH(OH)CH(OH)CH(OH)CH(OH).CH2OH

B  HOOC.CH(OH) . CH(OH) . CH(OH) . CH(OH) . COOH

ALCOHOLS, PHENOLS & ETHERS


28 QUIZRR
PINACOL REARRANGEMENT

The pinacol rearrangement or pinacol-pinacolone rearrangement is a method for converting a


1, 2-diol to a carbonyl compound. This rearrangement takes place under acidic conditions. The
name of the reaction comes from the rearrangement of pinacol to pinacolone.

H+ H
+
H H H H H
O O O O H3C O

+
H3C CH3 H3C CH3 CH3
H3C CH3 H3C CH3 H3C CH3

pinacol

O O
H3C H3C
+
CH3 CH3
H3C H3C
CH3 CH3

pinacolone

An overview of mechanism

Protonation of one of the ăOH groups occurs and a carbocation is formed. If both the ăOH groups
are not alike, then the one which yields a more stable carbocation participates in the reaction.
Subsequently, an alkyl group from the adjacent carbon migrates to the carbocation center. The
driving force for this rearrangement step is believed to be the relative stability of the resultant
oxonium ion, which has complete octet configuration at all centers (as opposed to the preceding
carbocation). The migration of alkyl groups in this reaction occurs in accordance with their usual
migratory aptitude, i.e. Ph- > 3-alkyl > 2-alkyl > 1-alkyl > H.

Stereochemistry of the rearrangement

In cyclic systems, the reaction presents more features of interest. In these reactions, the
stereochemistry of the diol plays a crucial role in deciding the major product.
Ć An alkyl group which is situated trans-to the leaving ăOH group alone may migrate. If
otherwise, ring expansion occurs, i.e. the ring carbon itself migrates to the carbocation
centre.
Ć If the migrating alkyl group has a chiral center as its key atom, the configuration at this
center is retained even after migration takes place.

ALCOHOLS, PHENOLS & ETHERS


QUIZRR 29

Exampl 8

Ph Ph
O ăH
+
Ph HO Ph
Ph
Ph
Ph

Explain the mechanism.

Solution :

HO
OH
Ph Ph Ph
Ph
O H
+
+ +
Ph O Ph + Ph3C
Ph Ph
Ph H Ph
Ph

NOTE : A number of experiments have been carried out to determine the relative
migratory aptitude of -groups in Pinacol/Pinacolone type rearrangement.
H > Ph > Me3C > MeCH2 > Me

DISTINCTION BETWEEN 1°, 2°, 3° ALCOHOLS

1. Haloform Reaction
The general reaction is represented as

ă
R · CH · CH3  X 2 + NaOH  R · C · O Na   CHX3 + NaH + H2 O
| |
CH C

The CH3 carbon is lost as CHX3 and the remaining part exists as acid salt, which can be acidified
to liberate free acid.
The structural feature essential in the compound to show haloform reaction is that any of the
following moieties should be present in the molecule attached to some electron-withdrawing group
or electron donating group by +I only.

CH3 · CH · or XCH2 · CH · or X2CH · CH · or X3C · CH ·


| | | |
CH OH OH OH

ALCOHOLS, PHENOLS & ETHERS


30 QUIZRR

CH3 · C · or XCH2 · C · or X 2CH · C · or X 3C · C ·


|| || | ||
O O O O

or any other grouping that can be converted to any of the above moieties.
The mechanism of the reaction can be outlined as :
The reaction has 3 important steps. Step I is the oxidation, caused by mild oxidizing agent (hypo
halite ion). The second step is base-promoted halogenation and the third step is cleavage of CăC bond.

I2 + 2OHă Iă + OIă + H2O R CH CH2 ăI

OH

OIă

ă + ă
ă  ă
OI OH I I
R CH CH3 R C CH3 R C CH2 ă R C CH2 ăI
(Oxidising ăH2O ăI
agent)
OH O O O
Carbanion I
(Resonance stabilized)

OH
ă ă + ă ă
OH I2/OH I I
R C CI3 R C CI3 R C CHI2 ă R C CH ă I
ăI
Oă O O O
Carbanion I
(more resonance stabilized
OIă OIă due to-I effect of iodine)

+
R C OH CI3 R CH CI3 R CH CHI2

O OH OH

Proton
exchange

ă

R C O + CI3

O Yellow crystalline ppt.

ALCOHOLS, PHENOLS & ETHERS


QUIZRR 31

Some of the compounds which responds positively to iodoform test are

CH3CH 2OH (only primary alcohol)

CH3 · CH · CH3 , CH3 · CH · CH2 CH3 , Ph · CH · CH3 (secondary alcohols)


| | |
OH OH OH

CH3 · C · H (only aldehyde)


||
O

CH3 · C · CH3 , CH3 · C · CH2CH3 , Ph· C· CH3


|| || || (ketones)
O O O

X
|
CH3 · CH · CH3 , CH3 · C · CH3 ,
| |
X X

CH3 · C · CH2 · C · CH3 , CH3 · C · OEt (after heating with OHă)


|| || ||
O O O

O
||
CH3 · C · CH2 CO2H (after heating)
 

The compounds that respond negatively to iodoform test are

CH3 · C · OH, CH3 · C · Cl, CH3 · C · NH2 , CH3 · C · OR , CH3 · C · O · C · CH3


|| || || || || ||
O O O O O O

CH3 · C · CH2 · C · OH, CH3 · C · CH2 · C · OEt


|| || || ||
O O O O

ALCOHOLS, PHENOLS & ETHERS


32 QUIZRR
Example 9

Explain the following reactions with a proper mechanism.

OH
(a) Base 18
X O +
O O O
O
18 18

CH3
CH3

CH2
(b) H2C

CH2
H3C

OH O
+
H CH3
CH3
(c) H3C H3C
OH

Solution :

(a) HO + ă
H O
X
X
O
18 O
18

O
X
O O ă
O O
18 O
18

ALCOHOLS, PHENOLS & ETHERS


QUIZRR 33

(b) CH3 CH3

H2C 

H2C CH2

H3C CH2 H3C CH2

H
+
(c) H3C OH H3C O H H3C
+
H +

HO CH3 HO CH3 HO CH3

H3C H3C CH3 H3C


+

+
O CH3 H O CH3 HO CH3

2. Lucas Test

Alcohols react with concentrated hydrochloric acid in the presence of anhydrous zinc chloride to
form alkyl chlorides. The alkyl chlorides appear as cloudiness because of its insolubility in Lucas
reagent.

2 ZnCl
R · OH + HCl   R · Cl + H2O
Alkyl halide

The three types of alcohols undergo this reaction at different rates. The rates of reaction with
Luca s rea gent [conc. HCl + ZnCl2 (anhydrous)] follow the given order :
Tertiary alcohol > Secondary alcohol> Primary alcohol
An unknown alcohol (monohydric) is mixed with conc. HCl and anhydrous ZnCl2 at room
temperature. The alkyl chloride formed is insoluble in the medium, thus the solution becomes
cloudy before it separates out as a distinct layer. The following observations are made,

(a) If cloudiness (white turbidity) appears immediately, the alcohol is tertiary.


(b) If cloudiness appears within 5 minutes, the alcohol is secondary.
(c) If the solution remains clear, i.e., no cloudiness is formed the alcohol is primary.

Remember that the benzyl and allyl alcohol react as rapidly as tertiary alcohol with Lucas reagent
because their cations are resonance stabilised and as stable as 3Ĉ carbocations.

ALCOHOLS, PHENOLS & ETHERS


34 QUIZRR
3. Victor Mayer Test :
It follows in four stages :

(i) P + I2, (ii) AgNO2, (iii) HNO2, (iv) Base

Primary alcohol Secondary Alcohol Tertiary alcohol

H3C CH3 CH3


H3C OH
H3C CH3
OH
OH

P + I2
CH3 CH2 I P + I2

H3C CH3
AgNO2 CH3

I H3C CH3

CH3 CH2 NO2 I


AgNO2

HNO2 AgNO2
H3C CH3

H3C NO2
NO2 CH3

NO H3C CH3
HNO2
Nitrolic acid NO2

NaOH NO HNO2

H3C CH3
Red colour X (Noreaction)
NO2

Pseudo Nitrolic acid


NaOH

NaOH
Colourless

Blue colour

ALCOHOLS, PHENOLS & ETHERS


QUIZRR 35

4. Oxidation Test

(i) Primary alcohol on oxidation in presence of acidic chromate gives aldehyde

O
ă2
R OH + Cr2O7 R C H + Cr+3
Orange Green

Colour will charge from orange to green.

(ii) Secondary alcohols gives Ketones by chromic acid

R´ O
K2Cr2O7
R OH R C R´ + Cr+3
Or CrO3 Green

Here also colour will not change.

(iii) 3Ĉ alcohol donÊt oxidise in alkaline condition but in acidic condition, it rapidly dehydrates to
alkene. Colour doesnÊt change.

Exercise 10

Effect the following conversions.

CH2 OH CH2 OH

(a)

(b)

CH2 CH2OH

(c) CH3 OH

ALCOHOLS, PHENOLS & ETHERS


36 QUIZRR
Solution :
+
CH2 OH CH2
+
+
(a) H

H2O

CH2 OH O OH
HBO PPh2 = CH2 K2Cr 2O 7

(b)
HCO3H MeMgl
O OH +
H3O+
OH

H+ H+

CH3 CH2Cl
PCl3 Cl2
(c) CH3OH CH3Cl
AlCl3 h

Mg/Ether

CH2 CH2 OH CH2MgCl


HCHO
H3O+

ALCOHOLS, PHENOLS & ETHERS


QUIZRR 37

Exercise 11

Compound (X) (C5H8O) does no react appreciably with Lucas reagent at room temperature
but gives a precipitate with ammonical silver nitrate. With excess of MeMgBr, 0.42 g of (X)
gives 224 ml CH4 at STP. Treatment of (X) with H2 in presence of Pt. catalyst followed by
boiling with excess HI gives n-pentane. Suggest structure for (X) and write the equations
involved.
Solution :
(1) (X), (C5H8O) does not react with Lucas reagent appreciably at room temperature but gives
precipitate with ammonical AgNO3 and thus, (X) has terminal alkyne linkage as well as
primary alcoholic group.

(2) (X) on hydrogenation and then reacting with HI gives pentane and thus, (X) is straight
chain compound.

(3) Keeping in view of the above facts (X) may be

CH2 . CH2 . CH2 . CH2 . C  CH*


(x)
OH* Pent ă 4 ă yn ă 1 ă ol

(4) Its reaction with MeMgBr gives CH4. (It has two acidic or active H* atoms) and thus, 1 mole
of (X) will give two mole of CH4.

CH2 . CH2 . CH2 . C  CH + 2CH3 MgBr  2CH4


(X)
OH

 84 g (X) gives 2 ï 22.4 litre CH4

2  22.4  0.42
 0.42 g (X) will give = 224 ml CH4
84

Given fact is confirmed.

Thiols
Nomenclature :
Thiols are the sulfur analogues of alcohols.
Functional group suffix = ă thiol
Functional group prefix = mercapto

ALCOHOLS, PHENOLS & ETHERS


38 QUIZRR
Physical Properties :
Ć Hydrogen bonding is much weaker than that in alcohols.
Ć Lower boiling points than similar alcohols.

Structure :
Ć Generally similar to alcohols, but bonds to S are longer and weaker than those to O.
Ć The thiol functional group consists of an S atom bonded to a C atom and a H atom via 
bonds.
Ć The SăH bonds is less polar than that in alcohols since S is less electronegativity than O.

Reactivity :
Ć Thiols are much more acidic than similar alcohols, e.g., RSH (pKa = 10) versus ROH (pKa
= 16 to 19)

ALCOHOLS, PHENOLS & ETHERS


QUIZRR 39

PHENOLS
When hydroxyl group is directly attached to the benzene ring, the compounds are called phenols.

Some examples :

OH OH OH OH
CH3

CH3

CH3
phenol P-cresol m-cresol o-cresol

OH

OH

1-naphthol 2-naphthol
(-naphthol) (-naphthol)

Physical Properties :

Ć The polar nature of the OăH bond (due to the electronegativity difference of the atoms)
results in the formation of hydrogen bonds with other phenol molecules or other H-bonding
systems (e.g. water). The implications of this are :

* high melting and boiling points compared to analogous arenes


* high solubility in aqueous media

Ć The presence of intramolecular hydrogen bonding is believed responsible for the significantly
lower boiling points of certain ortho-substituted phenols vs the meta-and para-analogs.

Structure :

Ć The alcohol functional group consists of an O atom bonded to an sp2ăhybridised aromatic


C atom and a H atom via  bonds.

Ć Both ther CăO and the OăH bonds are polar due to the high electronegativity of the O atom.

Ć Conjugation exists between an unshared electron pair on the oxygen and the aromatic ring.

ALCOHOLS, PHENOLS & ETHERS


40 QUIZRR
Ć This results in, compared to simple alcohols :
Ć a shorter carbon-oxygen bond distance
Ć a more basic hydroxyl oxygen
Ć a more acidic hydroxyl proton (ăOH)

OH + OH + OH + OH

H H H H H H H H
ă
ă

ă
H H H H H H H H
H H H H

A cidity

Ć Phenols are more acidic (pKa  10) than alcohols (pKa  16 ă 20), but less acidic than
carboxylic acids (pKa  5)
Ć The negative charge of the phenolate ion is stabilised by resonance due to electron
delocalisation onto the ring as shown below :

ă
O O O O

H H H H H H H H
ă
ă

ă
H H H H H H H H
H H H H

Ć The acidity difference means that it is possible to separate phenols from alcohols and/or
carboxylic acids.

* Mixing an ether solution, of either phenol and alcohol or phenol and carboxylic acid,
with dilute base (sodium hydroxide and sodium bicarbonate, respectively), results in
the stronger acid being converted to its alkali salt, which is then extracted to the
aqueous phase and can be separated from the organic phase.

Ć Nucleophilic substitution reactions of phenols are generally carried out under basic conditions
as the phenolate ion is a better nucleophile.

ALCOHOLS, PHENOLS & ETHERS


QUIZRR 41

Substituent Effects on Acidity

Substituents, particularly those located ortho or para to the ăOH group, can dramatically influence
the acidity of the phenol due to resonance and / or inductive effects. Electron withdrawing groups
enhance the acidity, electron donating substituents decrease the acidity. The resonance stabilisation
of o-nitrophenol is shown below :

ă ă
O O O O

N N
+ +
O O
ă

Acidity of Phenol can be explained by :

(a) electron ă attracting and


(b) electron ă releasing substituents on the acid strength of phenols are as follows.

Electron ă attracting substituents disperse negative charges and therefore stabilize ArOă and
increase acidity of ArOH. Electron ă releasing substituents concentrate the negative charge on O
destabilizes ArOă and decreases acidity of ArOH.

Electron ă Attracting Electron ă Releasing


(· NO2, · CN, · CHO, · COOH, · NR3, · X)

OH Oă OH Oă

+ B BH + + B BH +

A A R R

More acidic Charge dispersed lon Less acidic Charge concentrated


stabilized lon destabilized

I. In terms of resonance and inductive effects we can account for the following relative acidities.

(a) păO2NC6H4OH > măO2C6H4OH > C6H5OH

(b) măClC6H4OH > p-ClC6OH > C6H5OH

ALCOHOLS, PHENOLS & ETHERS


42 QUIZRR
(a) The ă NO2 is electron ă withdrawing and acid ă strengthening. Its resonance effect,
which occurs only from para and ortho positions, predominates over its inductive
effect, which occurs also from the meta position. Other substituents in this category
are

C = 0 ă CN ăCOOR ăSO2R

(b) Cl is electron ă withdrawing by induction. This effect diminishes with increasing


distance between Cl and OH. The meta is closer than the para positions and m-Cl is
more acid ă strengthening than the p-Cl. Other substituents in this category are F,
Br, I, +NR3.

II. We can compare the relative acid strengths in the following groups :
(a) phenol, m-chlorophenol, m-nitrophenol, m-cresol;
(b) phenol, benzoic acid, p-nitrophenol, carbonic acid
(c) phenol, p-chlorophenol, p-nitrophenol, p-cresol
(d) phenol, o-nitrophenol, m-nitrophenol, p-cresol
(e) phenol, p-chlorophenol, 2, 4, 6 ă trichlorophenol, 2, 4ădichlorophenol
(f) phenol, benzyl alcohol, benzenesulfonic acid, benzoic acid

O
+
(a) N
O

Because NO2 has + on N, it has a greater electron ă withdrawing inductive effect than Cl.
The decreasing order of relative acid strengths
m-nitrophenol > m-chlorophenol > phenol > m-cresol

(b) The decreasing order of relative acid strengths


Benzoic acid > carbonic acid > p-nitrophenol > phenol

(c) The resonance effect of păNO2 exceeds the inductive effect of păCl p-CH3 is electron releasing.
The decreasing order of relative acid strengths
p-nitrophenol > p-chlorophenol > phenol > p-cresol

(d) Intramolecular H-bonding makes the o-isomer weaker than the p-isomer.
The increasing order of relative acids strengths
p-nitrophenol > o-nitrophenol > m-nitrophenol > phenol

ALCOHOLS, PHENOLS & ETHERS


QUIZRR 43

(e) The decreasing order of relative acids strengths


2, 4, 6ătrichlorophenol > 2, 4,ădichlorophenol > p-chlorophenol > phenol

(f) The decreasing order of relative acid strengths


benzenesulfonicacid > benzoic acid > phenol > benzyl alcohol

Example 12

Compare acidic order of


OH OH OH OH
CH3

CH3
CH3
ortho meta para

Solution :
· CH3 is electron donating group by inductive effect as well as resonance. Therefore ortho and
para will be least acidic and phenol most. Among ortho & para, inductive effect is greater at ortho
position, therefore it will be less acidic than para.
Therefore
phenol > meta > para > ortho

Example 13

Compare acidity of

OH OH OH OH
NO2

NO2
NO2

Solution :
NO2 is eă withdrawing both by resonance.
Therefore order should be

ortho > para > meta > phenol

ALCOHOLS, PHENOLS & ETHERS


44 QUIZRR
but its not the case due to H bonding at ortho position

O·H O

This correct order is


para > ortho > meta > phenol

Example 14

Compare acidity of

OH OH OH OH
Cl

Cl

Cl

Solution :
ă Cl is eă withdrawing by inductive effect
and eă donating by resonance & former dominates over later.

 o > m > p > 

where o · ortho, m · meta, p · para,  · phenol

Example 15

OH OH
H3C CH3

H3C CH3

NO2 NO2

Which is more acidic ?

ALCOHOLS, PHENOLS & ETHERS


QUIZRR 45

Solution :
(1) is more acidic than (2)
OH

In (2) repulsion between CH3 groups changes the plane

N of and it doesnÊt undergo resonance as porbitals are


H3C CH3
O O not in same plane.
N
O O

Therefore (1) is more acidic than (2)

Also remember

SO3H COOH OH

> > H2CO3 > (acidity order)

Example 16

OH ONa

+ NaHCO3 + H2CO3

COOH COONa

+ NaHCO3 + H2CO3

Which of the two reactions is feasible ?

ALCOHOLS, PHENOLS & ETHERS


46 QUIZRR
Solution :

COOH OH

> H2CO3 > (acidic strength)

Therefore only (2) is feasible.

PREPARATION OF PHENOLS

1. Reaction of Benzene Sulfonic Acid with Hydroxide

SO3H OH
350Ĉ C
NaOH +

Reaction type : Nucleophilic Aromatic Substitution

Mechanism :

SOă3Na+ OăNa+ OH
+
NaOH/350Ĉ C H
ăNa 2SO3

SOă3Na+ OăNa+
NaOH,
fusion

+
H

OH

ALCOHOLS, PHENOLS & ETHERS


QUIZRR 47

Note : There an limitations to this method. Firstly, halophenols can not be prepared in this ways
because at high temperature OHă displaces the halogens more readily than the replacement of

SO32 ion. Secondly, a substituent of ă R type which is meta to the sulphonic acid group activates
the ortho and para position for hydride displacement.

2. Base Hydrolysis of Chlorobenzene (Dow’s Process)

Cl OH
350Ĉ C H2O
NaOH +

Reaction type : Nucleophilic Aromatic Substitution

Note :
Ć The reaction occurs via the elimination-addition mechanism via a benzyne intermediate.
Ć Elimination of HCl creates the benzyne that then undergoes addition of H2O to produce the
phenol.

3. Preparation of Phenols from Aryl Diazonium Salts

Benzene or arene diazonium salts on distillation with steam forms phenol or substituted phenols
respectively, with liberation of nitrogen.

+
N2 HO
H2O, H2SO 4

+ N2 + H2SO4
100Ĉ C

Note :

Ć Aryl diazonium salts can be converted into phenols using H2O /H2SO4 / heat

Ć Aryl diazonium salts are prepared by reaction of aryl amines with nitrous acid HNO2.

4. By Distillation of Phenolic Acids with Soda Lime

Phenolic carboxylic acids (ortho, meta or para) on heating with sodalime undergo carboxylation
to give sodium phenoxide, which on acidification gives phenol.

ALCOHOLS, PHENOLS & ETHERS


48 QUIZRR
OH OăNa+ OH
COOH
+
NaOH(CaO), H
ăNa2 CO3

5. From Grignard Reagents

Phenyl magnesium halide or reaction with electrophilic O2 gives a peroxide salt, which on reaction
with another mole of phenyl magnesium halide gives 2 moles of magnesium salt of phenol. This
on acidification gives 2 moles of phenol.

MgBr O ă OMgBr OMgBr OH

+
O2 PhMgBr H
2 2

6. From Cumene Hydroperoxide

This is carried out by oxidising cumene to its hydroperoxide and subsequently decomposing it with
acid into phenol and acetone. The mechanism involves 1, 2-shift of phenyl group.

CH(CH3)2

250Ĉ C
+ CH2 = CHCH3
H3PO 4

Isopropyl benzene
(Cumene)

CH3 CH3
95 ă 135Ĉ C
C6H5 C H + O2 C6H5 C O OH

CH3 CH3
Cumene hydroperoxide

ALCOHOLS, PHENOLS & ETHERS


QUIZRR 49

CH3
+
H , H 2O
C6H5 C O OH C6H5 + CH3COCH3
Phenol Acetone
CH3
Cumene hydroperoxide

In the rearrangement, Ph migrates from carbon to oxygen. Most acid-catalyzed rearrangements


require the intermediacy of a positively charged atom to which the group migrates, most of these
positively charged sites were carbocations but in this case it is an O+.

Ph Ph H Ph
H3C + H3C H3C ă
H ăH2O ăPh
C O O H C O O H C O +
+
H3C H3C H3C
An oxonium ion Electronădeficient
intermediate

H3C + H3C
+ ăH2O
C O Ph C O Ph
H3C H3C
Carbocation

+
OH2
H3C H3C OH H3C
+
ăH
C O Ph C C O + PhOH
H3C H3C OPh H3C
Hemiacetal Acetone Phenol

The rearrangement of Ph may be synchronous with the loss of H2O.

ALCOHOLS, PHENOLS & ETHERS


50 QUIZRR
R EACTIONS OF PHENOL

+Na
C6H5ONa + H2

as acid
+NaOH C6H5ONa + H2O

+CH3COCl/Pyridine/AlCl3
C6H5COOCH3
ăHCl
Acylation
+(CH3CO2)O/H2SO4/AlCl3
C6H5COOCH3
ăCH3COOH

+ C6H5COCl Benzoylation
C6H5COO6H5 (schotten Baumann
aq. NaOH
reaction)
+ NH3/ZnCl2
C6H5 ă NH2 + H2O
Heat

Zn/
C6H6 + ZnO

C6H5 ă OH
NaOH (CH3)2SO4
C6H5ONa C6H5OCH3

Br2 /(aq)
2, 4, 6 - Tribromophenol

Br2/CS2
Ortho and para bromophenol

Electrophilic
Fuming HNO3 substitution
2, 4, 6 - Trinitrophenol (picric acid)
reactions

dil. HNO3
Ortho and para nitrophenol
293K

CH3Cl
Ortho and para cresol
AlCl3

+
C6H5N2 Clă
HO N=N Coupling
reaction

ALCOHOLS, PHENOLS & ETHERS


QUIZRR 51

OH

(i) CHCl3, aq. NaOH CHO Reimer Tiemann


Reaction
(ii) H3O+, 340K

(i) CO3, 400K


C6H5OH ONa + OH Kolbe
(ii) H3 O
Reaction
COOH

OH

HCl + HCN
Gattermann's
AlCl3 Reaction

CHO

(i) Phenol as acid :

Order of acidity for following :

OH

H2O < < HCO3ă < R COOH

Hence,
OH ONa

+ Na

But
OH ONa

+ NaHCO3 + CO2

This reaction does not occur as HCO3 is a stronger acid than phenol.

ALCOHOLS, PHENOLS & ETHERS


52 QUIZRR
Now, introducing electronă withdrawing groups increase the acidic strength of phenol in general.

OH
NO2
(a) + NaHCO3

OH ONa

O2N NO2 O2N NO2


(b)
+ NaHCO3 + CO2 + HO2

Now, 2 nitro groups make phenol more acidic than HCO3 .

OH ONa

O2N NO2 O2N NO2

+ NaHCO3 + CO2 + H2O


(c)

NO2 NO2

( ii) Acylation :

O O
AlCl3 ă
R C Cl R C+ AlCl4

O
O O
+ ă
O + ă O C AlCl4 O C R
C ă AlCl4

H R H R

Important Point : It is also known the aryl esters in presence of AlCl3 readily rearrange to form
aryl ketones. This is known as the FRIES REARRANGEMENTS.

ALCOHOLS, PHENOLS & ETHERS


QUIZRR 53

The reaction is :

O C CH3 OH OH

COCH3
AlCl3
+
(CS2)

COCH3

Generally low temperature (60Ĉ or less) favours the formation of the p-isomer whereas high
temperature (above 160ĈC) favours the o-isomer.

Mechanism :

ă
Cl2Al + COCH3
O CO CH3 O
Cl

AlCl3

ăClă
ă[ + COCH3]

O AlCl2 O AlCl2
COCH3
+
COCH3
H

Clă

Cl2Al O OH

COCH3 COCH3
H2O

ALCOHOLS, PHENOLS & ETHERS


54 QUIZRR
( iii) Bromination :

OH OH
Br Br
Br2
H2O

Br

Mechanism :



Br2 + H2O  HBr + HOBr

H+ (from HBr) 
HOBr 
 H2 O ·· Br

also, OH Oă

+ H+

{phenol in aqueous medium is partially ionized and phenoxide ion is more reactive than phenol
itself towards electrophilic attack}.

Oă Oă Oă

+ ă H+

+
Br OH2 H Br
Br

+
Br OH2

OH Oă Oă
Br Br Br Br Br
+
+
H BrOH2

Br Br Br

ALCOHOLS, PHENOLS & ETHERS


QUIZRR 55

But in presence of non-aqueous medium like CS2 or CCl4, only mono-substitution takes place. This
is because phenol is not ionized and hence benzene ring is less activated than that of phenoxide

ion. Also, bromine in Br2 is not as electrophiloic as in Br ·· OH2

OH OH OH
Br
CS2
+ Br2 +
ăHBr

Br

Now, if we have to prepare only the orthobromo phenol, then we wake use of protecting groups.

For example.

OH OH OH
SO3H Br SO3H
H2SO 4 Br2
NaOH

SO3H SO3H

boil H+

Br

OH

The activation of benzene ring in aqueous medium is so strong that derivatives of phenol containing
· SO3H group or · COOH group either at ortho or at para position are displaced by Br in
bromination reaction.

ALCOHOLS, PHENOLS & ETHERS


56 QUIZRR
OH
COOH
Br2
OH
H2O
Br Br

OH

Br2
Br
H2O

COOH

Trisubstitution of benzene is also observed in case of aniline, since aniline has same reactivity
as that of phenoxide ion towards electrophilic attack.

(iv) Nitration

When treated with dilute HNO3, phenol gives o and p-nitrophenol. When treated with nitrating
mixture, phenol gives 2, 4, 6ătrinitrophenol (picric acid) but the yield is poor.

OH OH OH OH
NO2 O2N NO2

dil HNO3 HNO3


+
20Ĉ C H2SO4

NO2 NO2

oăNitrophenol păNitrophenol 2, 4, 6ătrinitrophenol


(Picric acid)

Picric acid can be alternatively prepared as follows :

Cl Cl OH OH
NO2 NO2 O2N NO2
HNO3 aqueous HNO3
H2SO4 Na2CO3 H2SO4

NO2 NO2 NO2


2, 4, 6ătrinitrophenol
(Picric acid)
ALCOHOLS, PHENOLS & ETHERS
QUIZRR 57

(v) Reaction with HNO 2 (Nitrosation)


OH OH OH
NO

+ HNO2 +

NO
păNitrosophenol

dil. HNO3
Ć Here NO+ is electrophile.
Ć In 2nd step oxidation of Nitrogen in OH
p-nitrosophenol takes place.

NO2

(vi) Reaction with H2SO 4 (SULPHONATION)

OH OH OH
SO3H

+ H2SO4 +

SO3
(ortho) (para)

At 25ĈC ortho will be major as it is kinetically controlled product.


At 100ĈC (high temperature) para will be major as it is thermodynamically controlled product.

(vii) Oxidation by KMnO 4

Phenol can easily be oxidised by KMnO4 and ring is broken

OH

CH(OH) · COOH
KMnO4

CH(OH) · COOH
Meso tartaric acid

ALCOHOLS, PHENOLS & ETHERS


58 QUIZRR
Oxidation with Air

Ph · OH + O2

O
Benzoquinone

( viii) Action with bisulphate ion

ă
OH
Ph OH + K2S2O8 HO OH

păquinol

Mechanism :

O O O O

Oă S O O S Oă Oă S O+ +Oă S Oă

O O O O

OH O

OH OH O
H
+ ă
OH H OSO3 OSO3
+
H

ortho

ALCOHOLS, PHENOLS & ETHERS


QUIZRR 59

Similarly a para product can also be formed.

OH

OH
(major)

Example 17

OH
O
LiAlH4
+ H3C S Cl ?

Solution :

OH
O O

LiAlH4
+ H3C S Cl H3C Ph S O

O O

LiAlH4 is source of hydride Hă.

O
ă ||
O · S · Ph · CH3 is very good leaving group. Therefore it will leave very easily on attack
||
O

of Hă.

ALCOHOLS, PHENOLS & ETHERS


60 QUIZRR
Exmaple 18

Which has more B.P.

OH OH

C CH3 Or

C CH3

O
(a) (b)

Solution :
(a) has less b.p. than (b) due to intramolecular hydrogen bonding in (a) & intermolecular
hydrogen bonding in (b)

O·H O O H O H

C CH3

Intermolecular
Hydrogen bonding
O C O C

CH3 CH3
Intermolecular
H bonding

(ix) Reimer–Tiemann Reaction

The Reimer–Tiemann reaction is a chemical reaction used for the ortho-formylation of phenols.
In the simplest case, the product is salicylaldehyde :

OH OH O

CHCl3 H
+ 3NaCl + 2H2O
3 KOH

ALCOHOLS, PHENOLS & ETHERS


QUIZRR 61

R eaction mechanism

Chloroform (1) reacts with strong base to form the chloroform carbanion (2), which will quickly
alpha-eliminate to give dichlorocarbene (3). Dichlorocarbene will react in the ortho-and para-
position of the phenate (5) to give the dichloromethyl substituted phenol (7). After basic hydrolysis,
the desired product (9) is formed.

Cl Cl Cl
Cl KOH Cl
H ă
Cl Cl Cl
1 2 3

Cl

OH Oă O
Cl Cl
Cl
ă
KOH 3
H

4 5 6

H
OH O Oă Oă
Cl
O
Cl Cl
KOH
H H H

9 8 7

Ć oăhydroxy and p-hydroxy benzaldehyde are formed, which are separated by steam distillation.
Ć If oăpositions are blocked, p-hydroxy benzaldehyde is the main product.

When phenol is refluxed with CCl4 in alkaline medium, salicylic acid is formed.

OH OH

CO2H

+ CCl4 + NaOH + 4NaCl + 2H2O

ALCOHOLS, PHENOLS & ETHERS


62 QUIZRR
(x) Carboxylation of Phenols (Kolbe-Schmitt reaction)

ONa OH
1. CO2/125Ĉ C, 100 atm

2. H+
CO2H

Ć Heating the nucleophilic phenolate salt with carbon dioxide under high pressure / temperature
results in ortho-substitution.

Ć This process is also known as the Kolbe-Schmitt synthesis.

Ć o-hydroxybenzoic acid is more commonly known as salicyclic acid.

O
H The geometry allows for
ă an intramolecular
O
C hydrogen bond

Mechanism :

OH OăNa+

NaOH

Now, ă
O O
O
H Oă
C
C
O
O

tautomerize

OH OH
COOH CO2
+
H

(salicylic acid)

ALCOHOLS, PHENOLS & ETHERS


QUIZRR 63

Note : Salicylic acid is used for preparation of Asprin, oil of winter green (methyl salicylate) and
Salol (phenyl salicylate)

OH OH OCOCH3
CO2Me CO2H CO2H
MeOH (CH3CO)2O
Conc. H2SO4 NaOH

Aspirin
PhOH
Conc. H2SO4

OH
CO2Ph

(Phenyl salicylate)

Example 19
Find the final product

OH

CO2, OH
ă +
H /CH3 · C · Cl
[A] ?

Solution :
We know
OH Oă

CO2, OH
ă COOă

Kolbe Reaction

O
ă
O OH O C CH3
O
O

C Oă H
+ COOH CH3 · C · Cl C OH

O
Aspirin

ALCOHOLS, PHENOLS & ETHERS


64 QUIZRR
( xi) Oxidation of Phenols

Ć Oxidation can achieved by reaction with silver oxide (Ag2O) or chromic acid (Na2Cr2O7), or
other oxidising agents.

Ć Particularly important are the oxidation of 1,2-and 1,4-benzenediol (pyrocatechol and


hydroquinone, respectively) and their derivatives :

OH O OH O
OH O
Ag2O Na2Cr2O7
ether H2SO4, H2O
,

CH3 CH3 OH O

* WITH HNO3

OH O

HNO3

O
Benzoquinone

(xii) Claisen Rearrangment of Aryl Allyl Ethers :

O OH

heat

Summary
Ć Aryl allyl ethers undergo a thermal rearrangement to give ortho-allylphenols.
Ć This reaction is an intramolecular process.

ALCOHOLS, PHENOLS & ETHERS


QUIZRR 65

Ć Note how closely the mechanism of this reaction resembles that of the Diels-Alder reaction :

Claisen Diels-Alder

Caution : Do no confuse this reaction with the Claisen condensation of esters.

Mechanism :

O O tautomerize to form OH
more stable anomatic enol

( xiii) Dakin Reaction

C CH3 OH
O

+ CH3 C OH
H2O2/OH ă

OH OH

Mechanism :

Step 1 : Formation of hypoperoxide ion


H2O2 + OH   O · O · H + H2O

ALCOHOLS, PHENOLS & ETHERS


66 QUIZRR
Step 2 : Attack on carbonyl group
OH

O O
ă
C CH3 O C CH3

OH OH

Step 3 : Shift of phenyl group.

O
+
O OH2 C CH3

H3C C Oă O

OH OH

Setp 4 :

OH
O
O
C O OH H2O
+ CH3 C OH
CH3

OH

ALCOHOLS, PHENOLS & ETHERS


QUIZRR 67

Example 20
OH

O
NaOH
CH3 + CHCl3 S, (Product), Product S, is

OH

O
CH3
Solution :

COH

(xiv) Reaction with Formaldehyde

OH OH OH
O CH2OH
+
H
H C H +

CH2OH

Mechanism :

Step 1 : +
O H+ OH OH

H C H H C H H C H
+

Step 2 : OH OH OH
OH

CH2
+

OH
OH C H
H C H
+ H

ALCOHOLS, PHENOLS & ETHERS


68 QUIZRR
(xv) Gattermann Aldehyde Synthesis

When phenol is treated with liquid HCN and gaseous HCl in presence of anhydrous AlCl3,
product formed is hydrolysed Ph ă hydroxy benzaldehyde

OH CHO

AlCl3
+ HCN + HCl

Phenol OH

Mechanism :

Step 1 : Formation of electrophile

+
H
H C N H C NH
+

Step 2 : Electrophulic substitution Reaction an Phenol

OH
OH

H C NH
+

C NH
H

Step 3 :

OH OH

+
H
Hydrolysis

C NH C O
H H

ALCOHOLS, PHENOLS & ETHERS


QUIZRR 69

T est of Phenol

1. Phenol reacts with FeCl3 solution to form red and violet coloured solution.

OH

6 + FeCl3 [Fe · (O · C6H5)]3ă+ 3HCl + 3H+


Red or Violet

Ć This is also test for enol form.

2. Liebermann Test :
When phenol is dissolved in concentrated sulphuric acid and a few drops of aqueous sodium
nitrite is added, a red colour is obtained on dilution and turns blue when aqueous sodium
hydroxide is added.

Example 21

CH2
MgBr

NBS NaCN
X Y Z1 + Z2

OH

Solution :

Br CN CN

X= , Y= , Z1 = , Z1 =

OH OH OMgBr

ALCOHOLS, PHENOLS & ETHERS


70 QUIZRR
Example 22
Prepare (a) Picric acid (2, 4, 6-trinitrophenol), and (b) 2, 4-dichlorophenol from benzene.
Solution :
Even through the NO2Ês are in the o, o, p positions where OH directs electrophilic substitution,
phenol cannot be nitrated because the ring is susceptible to oxidative ring cleavage by
nitric acid. Instead, we take advantage of a nucleophilic addition ă elimination.

Cl Cl
NO2
Cl2 HNO3 1. NaOH
Fe H2SO4 2. H3O+

NO2
2,4-Dintrochlorobenzene

OH OH
NO2 O2N NO2
HNO3
H2SO4
NO2 NO2
2,4-Dintrophenol Picric acid

PhCl cannot be trinitrated because the Cl and two NO2Ês deactive the ring toward further
electrophilic substitution. 2, 4-Dintrophenol can be nitrated because the two deactivating NO2Ês
prevent ring oxidation. (b) Phenol cannot be chlorinated because the ring is susceptible to oxidation
by Cl2. Again nucleophilic addition ăelimination is used.

Cl OH
Cl Cl
3Cl2 / Fe OHă
PhH

Cl Cl
1,2,4-Trichlorobenzene 2,4-Dichlorophenol

The displaced Cl is ortho to one other Cl and para to the other and is activated by both. Each
of the other ClÊs is meta to at least one deactivating Cl.

ALCOHOLS, PHENOLS & ETHERS


QUIZRR 71

Example 23

Compound (A), C9H13O3N shows optical isomerism when 1, 2-dihydroxybenzene is treated


with chloroethanoyl chloride in the presence of POCl3, compound (B), C8H7O3Cl, is obtained
which upon treatment with methyl amine gives compound (C), C9H11O3N. Reduction of (C)
with Pd/H2 yields (A). Compound (B) when heated with NaOI and subsequent treatment of
the product formed with dilute acid solution yields 3, 4-dihydroxybenzoic acid. Deduce the
structure of (A).
Solution :

OH OH OH OH O

OH OH OH OC CH2Cl
ClCH2COCl
Or Or
POCl3

C · CH2Cl CH2COCl

O (I) (II) (III)

1, 2ădihydroxybenzene may undergo ring acylation or ring alkylation or esterification to form


compound (B).
Since (B) gives haloform test, the only possibility is (I).

OH OH OH
OH OH OH
NaOI +
H
CHCl2 +

C · CH2Cl COONa COOH

O (B) 3,4-dihydroxybenzoic acid

OH OH OH
OH OH OH
CH3NH2 H2/Pd

C · CH2Cl C · CH2NHCH3 *CH(OH)CH2NHCH3

O O (C) (A)

ALCOHOLS, PHENOLS & ETHERS


72 QUIZRR
Example 24

Write mechanism for the oxidation of a 2Ĉ alcohol with Cr (VI) as HCrO 4ă .


Solution :
A chromate ester is formed in the first step :

O O
fast +
H
R2C OH + Cr OH R2C O Cr OH
H2O
ă
H O O H O
Chromate ester

H O
H2O
R2C O Cr OH R2C = O + H3O+ + HOCr · Oă [Cr(IV)]
slow

O O
E2 elimination

The Cr(IV) and Cr(VI) species react to form 2Cr(V), which in turn also oxidizes alcohols giving
Cr(III) with its characteristics colour.

Example 25

(a)
CHCl3 H2O2/OH
A B+C D+E
KOH solution 
followed by
acidification

H2O2 /OH
catechol + E

followed by
acidification

Identify A to E.

(b) Carryout the following transformations.

Br
OH
(i)

ALCOHOLS, PHENOLS & ETHERS


QUIZRR 73

(ii)

NH2 CH2COOH

(iii)

Solution :

OH OH OH OH
CHO

(a) A= ; B= ; C= ; D= ; E = HCO2H

CHO OH

Br
OH
(b) (i) alcoholic (i) B2H6
KOH (ii) H2O2/OH

O O
O OH OH
(ii) CH2MgBr CH3MgBr OH
+
H3O+ H3O

ALCOHOLS, PHENOLS & ETHERS


74 QUIZRR
(iii) NH2 N2Cl

NaNO2 + HCl H3PO2


OĈ ă5Ĉ C

HCl + CO
anhydrous AlCl3

CH2Cl CH3 CHO

Cl2 HCl
ZV Zn ă Hg

KCN

CH2CN CH2COOH

+
H3O

Example 26

Give a simple test tube reaction that distinguishes between the compounds in each of the
following pairs. What would you do, see and conclude ?
(a) t-butyl and n-butyl alcohol,
(b) ethyl and n-propyl alcohol,
(c) allyl and n-propyl alcohol,
(d) benzyl methyl ether and benzyl alcohol
(e) cyclopentanol and cyclopentyl chloride.
Solution :

(a) Add Acid Cr2O7 (orange). The 1Ĉ n-butyl alcohol is oxidized; its solution changes colour to
green Cr(III). The 3Ĉ t-butyl alcohol is unchanged. Alternatively, when Lucas reagent
(HCl + ZnCl2) is added, the 3Ĉ ROH quickly reacts to form the insoluble t-butyl chloride that
appears as a second (lower) layer or a cloudiness. The 1Ĉ ROH does not react and remains
dissolved in the reagent.

ALCOHOLS, PHENOLS & ETHERS


QUIZRR 75

(b) Add I2 in OHă until the I2 colour persists. A pale yellow precipitate of CHI3 appears,
indicating that ethyl alcohol is oxidized. n-Propyl alcohol does not have the · CH(OH) CH3
group and is not oxidized.
(c) Add Br2 in CCl4; as the Br2 adds to the C = C of the colorless allyl alcohol, its orange colour
disappears. The orange colour persists in the unreactive n-propyl alcohol.

(d) Add acid Cr2O7 . It oxidizes the alcohol, and the colour changes to green. The ther is
unreactive. Alternately, if the two compounds are absolutely dry, add a small piece of Na
to each. H2 is released from the alcohol; the ether does not react.
(e) The simplest test is to add conc. H2SO4 to each dry compound. There will be only one layer
as the alcohol dissolves, evolving some heat. The layers will be discernable for the chloride,
which is not soluble in H2SO4.

Example 27

A compound (A), C10H12O2, is soluble in aqueous NaOH, but does not dissolve in aqueous
NaHCO3. When (A) is treated with dimethyl sulphate and aqueous NaOH, compound (B),
C11H14O2, is formed with hot HI (A) gives methyl iodide and with hot and concentrated base,
compound (C) C10H12O2 is obtained. Compound (B) is insoluble in aqueous NaOH but readily
decolourises KMnO4 solution. When treated with hot and concentrated base, compound (B)
produces (D), C11H14O2. Ozonolysis of C produces a compound, which is isomeric with 4-
hydroxy-3-methoxybenzaldehyde. Ozonolysis of (D) gives a compound, which is also obtained
by treating 4-hydroxy-3-methoxybenzaldehyde with di-methyl sulphate. Deduce the structure
of A with proper reasoning.
Solution :
Since (A) is soluble in aqueous NaOH, it is phenolic compound. Methylation with (CH3)2SO4/
NaOH introduces ă CH3 group.

 (B) is C11H14O2
(A) is C10H12O2

A difference of ăCH2 also indicates phenolic group in (A). (A) is cleaved with hot HI to produce
CH3I, indicating the presence of second oxygen.
That ozonolysis yields 4ăhydroxy-3-methoxybenzaldehyde, indicates that (A) is trisubstituted
benzene ring.

C10H12O2(A) C10H 11O C9H 8


ăOH ăOCH 3 ăC6H 3 (Trisubstituted aromatic ring)
C10H 11O C9H 8 C3H 5

ALCOHOLS, PHENOLS & ETHERS


76 QUIZRR
The residual group, C3H5 corresponds to an unsaturated side chain.
Treatment of (A) with strong and hot base results into isomerisation to C.

HO ă HO
HO
C6H3 · CH2 · CH = CH2  C6H3CH = CH · CH3
CH3O (A) CH3O (C)
C10H12O2 C10H12O2

The same isomerisation occurs when (B) is treated with hot and concentrated base.

CH3O ă CH3O
HO
C6H3 · CH2 · CH = CH2  C6H3CH = CH · CH3
(B) (D)
CH3O CH3O
C11H14O2 C11H14O2

Orientation in the compounds can be worked out as follows :

CHO CHO

O3 (CH3)2SO4
(D) ă
HO
OCH3 OCH3

OCH3 OH

CH2CH = CH2

OCH3
CH2CH = CH2 CH = CH ă CH3 CHO
OH
(A) (CH3)2SO4 HO
+
O3
(or) HO
ă

OCH3 OCH3 OCH3
CH2CH = CH2

OCH3 OCH3 OCH3


(B) (D)
OH

OCH3
(A)

ALCOHOLS, PHENOLS & ETHERS


QUIZRR 77

(C) has the same orientation of groups as (D). Hence the only possible isomer of 4-hydroxyă3ă
methoxybenzaldehyde that can be formed by cleavage of (C) is the one in which the ăOH and
ăOCH3 groups are reversed.

CHO

O3
(C) (isomer of 4ăhydroxyă3ămethoxybenzaldehyde)
OH

OCH3

Thus, A is

CH2CH = CH2 CH = CH · CH3 CHO

ă
OH O3

OH  OH OH

OCH3 OCH3 OCH3


(A) (C) (D)

ALCOHOLS, PHENOLS & ETHERS


78 QUIZRR
ETHERS
Ethers have a general formula R·O·R´. The groups R and R´ may be same or different and
accordingly ethers are known as simple or mixed respectively. The general IUPAC name of ethers
is alkoxy alkane

CH3 · O · CH3 CH3 · O · C2H5 CH3CH2CH2 · OC2H5


Methoxy methane Methoxy ethane Ethoxy propane
(Dimethyl ether)

CH2 = CH · OC2H5 OCH3


ethoxyethene

Methoxy benzene
(anisole)

Physical Properties

Ć The polar nature of the CăO bond (due to the electronegativity difference of the atoms)
results in intermolecular dipole-dipole interactions.
Ć An ether cannot form hydrogen bonds with other ether molecules since there is no H to be
donated (no ă OH group)
Ć Ethers can be involved in H-bonding with systems able to donate H (e.g. water).
Ć The implications of these effects are :
lower melting and boiling points compared to analogous alcohols.
solubility in aqueous media similar to analogous alcohols.
Ć Ethers have a definite dipole moment. This is due to the bent structure of their molecule.
Ć They are lighter than water. Their density incolases with increase in molecular mass.

Reactivity :

Ć The ethereal O atom is a region of high electron density due to the lone pairs.
Ć Ether oxygen atoms are Lewis bases.
Ć Like an alcohol ă OH group, the ă OR group is a poor leaving group and needs to be
converted to a better leaving group before substitution can occur.
Ć The most important reaction of ethers is their cleavage by strong acids such as HI or HBr.

Methods of Preparation :

1. By Williamson Synthesis :

R · X  R´ONa  R · O · R´ + NaX

ALCOHOLS, PHENOLS & ETHERS


QUIZRR 79

Here ăX can be halides, sulphates (ăOSO2OR) or sulphonates (ăOSO2R). Also, the temperature
should be low for this reaction.

The alkyl halide used in this reaction should preferably be primary because in case of secondary
and tertiary alkyl halides, the elimination reaction predominates resulting in the formation of
alkene as the major product.

CH3 CH3
| |
C2 H5 Br + NaO · C · CH3  C2 H5 · O · C · CH3  NaBr
| |
CH3 CH3

Whereas,

CH3 CH3
| |
CH3 · C · Br  C2 H5 ONa  CH3 · C  CH2  NaBr + C2 H5 OH
| (major product)
CH3

Note : (1) For aromatic ethers generally alkyl sulphates are used.
(2) The reaction is SN2.

2. Dehydration of alcohols

2 4 conc. H SO
2C2H5OH   C2H5 · O · C2 H5 + H2O
413 K
(excess)

2 3 Al O
2C2 H5OH   C2H5 · O · C2H5 + H2O

Dehydration of alcohols to ether is an example of nucleophillic substitution. However, it can be


used only to prepare symmetrical ethers only, otherwise we get a mixture of ethers. For example,

2 4conc. H SO
R · OH + HOR    R · OR + R´OR + R´OR´
140Ĉ C, 

ALCOHOLS, PHENOLS & ETHERS


80 QUIZRR
3. Alkoxy Mercuration - Demercuration of Alkenes

1. Hg(OAC)2 / ROH
C C H C C OR
2. NaBH4

Note : 1. Product is formed following MarkownikovÊs rule.


2. Reaction proceeds via the formation of a cyclic mercurinium ion

+ +
C C
Hg +
AcO

4. Preparation of epoxides

RCO3H
(a) C C C C + R´CO2H
O

RCO3H is peroxy acid.

(b) epoxides can also be prepared by internal SN2 reaction of chlorohydrin

Cl Cl
ă ă
OH ă Cl
CH2 CH2 CH2 CH2 CH2 CH2
+
O
OH Oă

Example 28

(a) Upon treatment with sulphuric acid, a mixture of ethyl and n-propyl alcohols yield a
mixture of three ethers. What are they ?
(b) On the other hand, a mixture of tert-butyl alcohol and ethyl alcohol gives a good yield
of a single ether. What ether is this likely to be ? How do you account for this good
yield ?

ALCOHOLS, PHENOLS & ETHERS


QUIZRR 81

Solution :
(a) CH3CH2OH (ethyl alcohol) and CH3CH2CH2OH (n-propyl alcohol) by dehydration make a
mixture of three ethers since each alcohol is 1Ĉ and follow SN2 reaction

H2SO4
CH3CH2OH + CH3CH2OH 
 CH3CH2OCH2CH3 + H2O

H 2SO4
CH3 CH2 CH2 OH + CH3 CH2 CH2 OH 
 CH3 CH2 CH2 OCH2 CH2 CH3 + H2 O

H 2SO 4
CH3 CH2OH + CH3 CH2CH2OH 
 CH3CH2CH2OCH2 CH3 + H2O

CH3 CH3 CH3


+ +
(b) CH3 C OH + H+ CH3 C OH2 CH3 C

CH3 CH3 CH3


(3) most stable carbonium ion

CH3 CH3

CH3 C + + CH3CH2OH CH3 C OCH2CH3 + H+

CH3 CH3

Since pr ot onat ed 3° alcohol (t er t -but yl alcohol) can make most st able car bonium ion, hence
t his leads t o t he single et her in good quant it y.

R EACTIONS OF ETHERS

Ethers are relatively inert compounds and do not react with alkalies, reducing agent, oxidizing
agents and reactive metals. Because of their inertness ethers are frequently used as solvents. They,
however, undergo cleavage under acidic conditions.

1. Cleavage by acids
On reaction with conc. HBr or conc. HI, ethers undergo cleavage to form an alcohol or phenol and
an alkyl halide.

C2H5 · O · C2H5 + HX  C2H5 X + C2H5OH

Ć The order of reactivity of halogen acids is

HI > HBr > HCl

ALCOHOLS, PHENOLS & ETHERS


82 QUIZRR
Ć In case of excess acid, product formed is only alkyl halide

C2H5 · O · C2H5 + 2HX  C2H5 X + H2O


excess

Mechanism :

Step I : R · O · R´ + HX R · O · R´ + Xă
+

H H
SN2
Setp II : R · O · R´ + Xă X · R · O´ · R´ RX + R´OH
+ +

SN1

ă
+X
R·X R + + R´OH

So, reaction can go by SN1 or SN2 depending upon the conditions employed and the structure of
ether. When both the alkyl groups are methyl or 1Ĉ, it will follow SN2 pathway and when atleast
one of the alkyl group is 3Ĉ, it follows SN1 pathway.

For example,

H
HI
(a) CH3 · OC2H5
+

SN2

Here, CH3CH2 · O  is a good leaaving group and CH3 is also not hindered. Therefore,
|
H

CH3I is formed.

ALCOHOLS, PHENOLS & ETHERS


QUIZRR 83

But
ă CH3
I
+ This is also a good leaving
C´ O CH3 group but not as good as
+
H H H C2H5O ă H and C´ is also hindered

Hence, this product is not formed.

CH3
HI +
(b) CH3 C O CH2CH3 (CH3)3C O CH2CH3

CH3 H

1
SN

CH3 CH3
I
C Iă C
+
+ CH3CH2OH
H3C CH3 H3C CH3

{stable carbocation}

Note : When the reaction follows SN2 mechanism (R > R´) then Iă attacks R´ and forms R´I and
ROH.
But, if SN1 mechanism is followed than RI and R´OH is formed.
In acidic medium, in case of cleavage of ether, if strong nucleophile is used, then reaction takes
place by SN2 mechanism.

Example 29
HI
CH3 CH CH2 ?
O
strong
+ ă
nucleophute
Solution : H I
CH3 CH CH2 CH3 CH CH2 CH3 CH CH2I
O OH
+ OH
ă
I
I

CH3 CH CH2

I
ALCOHOLS, PHENOLS & ETHERS
84 QUIZRR
Ć Strong nucleophile will attack at less hundered carbon.
But if we use CH3OH instead of HI

OCH3
+
H CH3 · O ·H
CH3 CH CH2 CH3 CH CH2 CH3 C CH2
O OH OH
+

Ć In case of weak nucleophile, we break up the ring according to carbocation stability but
actual carbocation is not formed.

Example 30

O +
H
CH2 C CH3 ?
CH 3OH
CH3

Solution :

H a good leaving
group
+
O O
+
H
CH2 C CH3 H2C C CH3

CH3 CH3
O
H3C H

OH CH3 OH CH3
+
H
H2C C CH3 H2C C CH3

OCH3 +O H

CH3

ALCOHOLS, PHENOLS & ETHERS


QUIZRR 85

Example 31

O ă
CH3O
CH2 C CH3 ?
CH3OH

CH3

Solution :
this is a stronger base than
ă
CH3O , hence it will obstract the proton

ă
O +

+

1CH2 2
C CH3 2 H2C C CH3
SN
CH3 Cleavage would be OCH3 CH3
ă at 1 not at 2 because
CH3O
of hindrance

CH3OH

OH

H2C C CH3

H3CO CH3

Example 32

*
O +
(1) H CH3OH
ClH2C* CH C CH3 ?
(2) Base
CH3

ALCOHOLS, PHENOLS & ETHERS


86 QUIZRR
Solution :
H
*
O +
O*
H
ClH2C* CH C CH3 ClH2C* CH C CH3

CH3 CH3

O
* CH3

CH2 CH C CH3 CH3 ă O

OCH3 H

ă *
O CH3 CH3
ă
* OH * *
Cl CH2 CH C CH3 Cl CH2 CH C CH3
ă H2O
OCH3 *O H OCH3

Example 33

(a) Give SN2 and SN1 mechanisms for the cleavage of ethers with HI.
(b) Why does SN2 cleavage occur at a faster rate with HI than with HCl.
Solution :
+
(a) Step 1 R O R´ + HI R O R´ + Iă

H
base1 acid2 acid1 base2

H
slow
Step 2 for SN2 Iă+ R O R´ RI + HOR´(R´is 1Ĉ )
+

H
slow
Step 2 for SN1 R O R´ R+ (R is 3Ĉ ) + R´OH
+

ALCOHOLS, PHENOLS & ETHERS


QUIZRR 87

Setp 3 for SN1 R   I   RI

(b) The transfer of H+ to ROR´ in step 1 is greater with HI, which is a stronger acid, than with
HCl. Furthermore, in step 2, Iă, being a better nucleophile than Clă, reacts at a faster rate.

2. Reaction with PCl5 /SOCl2

Ethers on heating with PCl5 or SOCl2 undergoes cleavage to form alkyl chlorides.


C2 H5 + O + C2 H5 + Cl + PCl3 + Cl  2C2 H5 Cl + POCl3

O
|| 
C2 H5 + O + C2 H5 + Cl + S  Cl  2C2 H5 Cl + SO2

The cleavage by PCl5 or SOCl2 is and to distinguish metameric ethers as their cleavage.

3. Oxidation of Ether by Air

When ethers are exposed to oxygen for a long time they are oxidised to ether hydroperoxides
(solid) which explodes on slightest heating or on rubbing.

R · CH2 · CH2 · O · CH2 · CH2 · R + O2  RCH2 · C H · OCH2 CH2 R


|
O · OH

Ethers are purified by extraction with excess of FeSO4 solution, which reduces the ăOOH gap to
the non-explosive, innocuous · OH group.

4. Test of Peroxide ion


CH3 · O · CH · CH3 + Fe2  Fe 3  Fe (SCN)3
SCN
| Red Colour
O
|
OH

When ether hydroperoxide is reduced with Fe+2, it oxidises Fe+2 to Fe+3 which can easily be
detected by thiocyanide ion producing green colour.

ALCOHOLS, PHENOLS & ETHERS


88 QUIZRR
5. Halogenation of ether

Cl2
CH3CH2 O CH2 CH3 CH3CH O CH2CH3

Cl

Cl2

CH3 ă CH O CH CH3

Cl Cl

6. Epoxidation

Ag2O
CH2 CH2 + O2 300Ĉ C
CH2 CH2

7. Formation of aryl ether

OH ONa

NaOH
+ CH3 · OSO2OCH3

OCH3

+ CH3 · OSO2ONa

(Anisole)

ALCOHOLS, PHENOLS & ETHERS


QUIZRR 89

As discussed earlier, methyl sulphonates are better leaving group than halogens, hence are better
for this kind of substitution.

Aryl halides cannot be used in the william sonÊs synthesis because of their low reactivity towards
nucleophilic substitution. There can be two combination of reactants for the preparation of alkyl
aryl ether, but one can usually be ruled out.

+ ă
CH3CH2Br + Na O

CH3CH2CH2 · O

CH3CH2Oă Na+ + Br

ALCOHOLS, PHENOLS & ETHERS


ALKYL HALIDES
QUIZRR 3

A L K Y L H A L I DES
Alkyl halides are a class of compound where a halogen atom or atoms are bound to an sp3 orbital
of an alklyl group.

CHCl3 (Chloroform : organic solvent)


CF2Cl2 (Freon-12 : refrigerant CFC)
CF3CHClBr (Halothane : anesthetic)

Halogen atoms are more electronegative than carbon atoms, and so the CăX bond is polarized.
Here X stands for halogens.

C+ Cl
 ă
H

The CăX bond is polarized in such a way that there is partial positive charge on the carbon and
partial negative charge on the halogen.
Electronegativity decreases in the order of :

F > Cl > Br > I

Carbon-halogen bond lengths increase in the order of :

C-F < C-Cl < C-Br < C-I

Bond Dipole Moments decrease in the order of :

C-Cl > C-F > C-Br > C-I


ø = 1.56D 1.51D 1.48D 1.29D

Typically the chemistry of alkyl halides is dominated by this effect, and usually results in the
C-X bond being broken (either in a substitution or elimination process).
This reactivity makes alkyl halides useful chemical reagents.

1. N OM ENCLA T URE
According to IUPAC, alkyl halides are treated as alkanes with a halogen substituent.
The halogen prefixes are Fluoro-, Chloro-, Bromo- and Iodo-.

ALKYL HALIDES
4 QUIZRR
Examples :

H CH3
FăCH2CH3
H
Cl
fluoroethane trans-1-chloro-3-methylcyclopentane

IUPAC nomenclature of alkyl halides with one halogen in haloakanes and with two similar
halogens is halo alkanes. If the halogens present are different, they are prefixed in alphabetical order.

eg. Cl CH2 CH2 CH2 CH2 CH3 , CH3 CH CH2 CH2 CH3
1ăChloropentane
Cl
2ăChloropentane

ClCH2 CH2 CHClCH3


1, 3-dichlorobutane

Often compounds of CH2X2 type are called methylene halides (CH2Cl2 is methylene chloride).

CHX3 type compounds are called haloforms. (CHI3 is iodoform).

CX4 type compounds are called carbon tetrahalides. (CF4 is carbon tetrafluoride).

Alkyl halides can be primary (1Ĉ), secondary (2Ĉ) or tertiary (3Ĉ).

H R R

R X R X R X

H H R
1Ĉ 2Ĉ 3Ĉ

Other types :
A geminal (gem) dihalide has two halogens on the same carbon.
A vicinal dihalide has halogen on adjacent carbon atoms.

Cl Cl
Br Br

R R

R R
R R
gem-dibromide vicinal dichloride

ALKYL HALIDES
QUIZRR 5

PREPA RA T I ON OF A L K Y L H A L I DES
Ra d i c a l H a l o g e n a t io n o f A l k a n e s
The one big limitation of this method (especially chlorination) is that it almost inevitably leads to
a (often complex) mixture of products. Even chlorination of methane gives a mixture :

light
CH4  Cl 2  CH3 Cl, CH2 Cl 2 , CHCl3 and CCl4

The situation is even worse for the chlorination of alkanes that have more than one kinds of
hydrogen:


CH3 CH2 CH2 CH2 Cl
 1-chlorobutane
30%

light  hv  
CH3 CH2 CH3  Cl2   CH CH CHCH 2-chlorobutane
 3 2 3
 |
 Cl
 70%


CH3

CH3 CH3 C CH3 35% 2-chloro-2methylpropane

light (hv)
CH3 CHCH3 + Cl2 Cl
CH3 65% 1-chloro-2methylpropane

CH3 CHCH2 Cl
+ other (polychlorinated) products

In the first example, 30% of primary halide means that each one of the six primary hydrogens
(CH3) is responsible for 30%/6 = 5% of product, whereas 70% of secondary alkyl halides means that
each one of the four secondary hydrogens (CH2) is responsible for 70%/4 = 17.5 of the product.
Hence, chlorination at secondary carbon occurs about 3.5 times the rate of chlorination at primary
carbon.
Similarly, in the second example, chlorination occurs at the single tertiary carbon (CH) at about
5 times the rate of chlorination at the primary carbons.

ALKYL HALIDES
6 QUIZRR
T h e o r d e r o f r e a c t iv i t y t o w a r d r a d i c a l c h l o r in a t io n i s

R-CH3 < R2CH2 < R3CH


1Ĉ 2Ĉ 3Ĉ
relative rate 1 3.5 5

This order reflects the relative order of the C-H bond strengths : 3Ĉ < 2Ĉ < 1Ĉ. Since less energy
is needed to break a tertiary C-H bond than either a 2Ĉ or a 1Ĉ C-H bond, the resulting tertiary radical
is more stable than either a secondary or primary radical.

However, although the rates of chlorination at different carbon atoms are different, the difference
is not great, so that in practice, some chlorination occurs at all sites.

Ra d i c a l c h l o r in a t io n h a s p o o r s e l e c t iv i t y
Bromination, on the other hand, is much more selective, mainly because of the lesser reactivity
of the halogen species (in particular Br) :

CH3 CH3 CH3


Br2
CH3 CH CH3 CH3 C CH3 and CH3CHCH2 Br
h
< 1%
Br
> 99%

A l l y l i c b r o m i n a t i o n a n d c h l o r i n a t io n
(Allylic means adjacent to a C=C double bond)
The bromination of cyclohexene produces a high yield of 3-bromocyclohexene.

H H H Br

h
+ Br2 + H ă Br

An allylic hydrogen has been substituted for a bromine.

ALKYL HALIDES
QUIZRR 7

Ć Compounds with allylic and benzylic hydrogen atoms can be brominated readily using the mild
brominating agent N-bromosuccinimide (NBS) :

NBr (NBS) Br O

(a) allylic O + NH

O
(initiator)

(b) benzylic CH3 NBr CH2Br O

O + NH

(initiator) O
The reaction proceeds via radical substitution method.
One important synthetic consequence of the delocalized allyl radicals is that more than one
bromination product is often obtained :

NBS
Ć
initiator Ć
  CH  CH  C H CH  CH  CH

C5 H11 CH2 CH  CH2 
 C5 H11 CH 2 5 11 2

Br2 thermodynamic
control

Br
|
C5 H11  CH  CH = CH 2 C5 H11CH = CH ă CH2 ă Br
+ 83%
17%
1 bromo 2 octene
3 bromo 1 octene

Note : F2 is most reactive while I2 is least reactive. Infact reaction with I2 is reversible and is
carried out in the presence of some oxidising agents like HIO3, HNO3 etc. to oxidise HI.

ALKYL HALIDES
8 QUIZRR
So m e m o r e r e a c t io n s :


CH3 · CH = CH2   X · CH2 · CH = CH2

High temperature,
CH3 · CH = CH2 + Cl2 
400 to 500 C
 CH2 · CH  CH2
|
Cl

NBS in
CH3 · CH  CH2  Br2 
CCl4
 CH2 · CH  CH2
|
Br

SO Cl / hv
CH3 · CH  CH2 
2 2  CH2 · CH  CH2
|
Cl

Pre p a r a t i o n o f V i n y l H a l id e s

HCl
CH CH 
 CH2 CH
CuCl2 |
Cl

The above reaction proceeds via electrophilic addition as follows

Cu2+

..ă +
Cu2+ Clă H3O
CH CH CH CH CH = CH ăH2O
CH2 = CH

Cl Cl

B y A d d i t io n o f H –X t o A l k e n e s
Alkyl chlorides, bromides and iodides can be prepared by treating an alkene with corresponding
hydrogen halide (HCl, HBr or HI). The addition of these compounds to alkene takes place according
to MarkownikovÊs rule. The reaction proceeds by electrophilic addition of H+ to give more stable carbocation
followed by attack of Xă. Anti-Markownikov addition of HBr can be achieved, if the reaction is carried
out in presence of peroxides (H2O2 or benzoyl peroxide or diătertăbutyl peroxide). Addition of HBr to
alkenes in the presence of peroxide follows free radical mechanism.


HX Xă
CH3 · CH CH2 
 CH3 · CH · CH 3  CH3 · C H · CH3
 2 carbocation  |
X

ALKYL HALIDES
QUIZRR 9

 
HBr HBr
CH3 · CH CH2 
Peroxide
 CH3 · CH · CH2 · Br  CH3 · CH2 · CH2 Br  B r
 2 radical 

Ex a m p l e 1
The heterolytic bond dissociation energy of CăCl bond in vinyl chloride is 207 kcal/mol as
compared to 191 kcal/mol in the case of CăCl bond in ethyl chloride. Give explanation for
this observation.
Solution :

H 2C CH Cl (Vinyl chloride)

ă +
H2C CH Cl H2C HC Cl

As we can see that C=Cl bond is formed in vinyl chloride; and hence its bond dissociation energy
becomes 207 kcal/mol.

Ex a m p l e 2
Haloarenes are insoluble in water and soluble in benzene.
Solution :
Haloarenes are insoluble in water because neither can they form H-bonds with water nor can
they break the H-bond already existing in water. However in accordance with general principle of
solubility i.e. like dissolves like haloarenes due to presence of large hydrocarbon part (benzene ring) are
soluble in hydrocarbon solvents such as benzene, petroleum, ether.

Fr o m a l k e n e s a n d a l k y n e s :

X X

XăX
C=C CăC
in CCl4

H X

2HX
C C CCl4
CăC

H X

ALKYL HALIDES
10 QUIZRR
X X

XăX X2
C C C=C C C
CCl4

X X X X

Fr o m a l c o h o l s
(1) Using hydrogen halides

H X
R  OH 
RX

So m e Po i n t s :

Ć When treated with HBr or HCl alcohols typically undergo a nucleophilic substitution reaction
to generate an alkyl halide and water.

Ć Alcohol relative reactivity order : 3Ĉ > 2Ĉ > 1Ĉ > methyl

Ć Hydrogen halide reactivity order : HI > HBr > HCl > HF (paralleling acidity order).

Ć Reaction usually proceeds via an SN1 mechanism which proceeds via a carbocation
intermediate, that can also undergo rearrangement.

Ć Methanol and primary alcohols will proceed via an SN2 mechanism since these have highly
unfavourable carbocations.

Ć The reaction of alcohols with HCl in the presence of ZnCl2 (catalyst) forms the basis of the
Lucas test for alcohols.

ALKYL HALIDES
QUIZRR 11

SN1 m e c h a n i s m f o r r e a c t io n o f A l c o h o l s w it h H B r

H3C
Step 1 : An acid/base reaction, Protonation of H3C C OH
the alcoholic oxygen to make a better
leaving group. This step is very fast H3C
and reversible. The lone pairs on the
oxygen make it a Lewis base.

fast H+

H3C H

Step 2 : Cleavage of the C-O bond allows the H3C C O


loss of the good leaving group, H
H3C
intermediate. This is the rate
determining step (bond breaking is
endothermic) slow

CH3

H3C C+

CH3
Step 3 : Attack of the nucleophilic bromide ion
on the electrophilic carbocation
creates the alkyl bromide.
fast Br

H3C
H3C C Br
H3C

ALKYL HALIDES
12 QUIZRR
Re a c t io n o f a l c o h o l s w it h s u l f u r a n d p h o s p h o r o u s h a l id e s
Alcohols can be converted to alkyl halides by reaction with thionyl chloride, SOCl2; phosphorous
trichloride, PCl3, phosphorous pentachloride, PCl5; or phosphorous tribromide, PBr3. For example, ethyl
chloride or ethyl bromide can be prepared from ethyl alcohol via reactions with sulfur and phosphorous
halides.
Reaction of Alcohol with PCl3

3ROH + PCl3  RCl + H3PO3

Mec hanism :

+
R O H + Cl P Cl R O P Cl

Cl H Cl

ă
Cl

R Cl + HO PCl2

HOPCl2 reacts further to give H3PO3 and 2 RCl molecules.

Ć This method is the method because the product formed is gaseous in nature.


CH3CH2OH + SOCl2   CH3CH2Cl + SO2 + HCl

CH3CH2OH + PCl3 
 CH3CH2Cl + P(OH)3 + HCl

CH3CH2OH + PCl5 
 CH3CH2Cl + POCl3 + HCl


CH3CH2OH + PBr3  +
 CH3CH2Br + P(OH)3 HBr

ALKYL HALIDES
QUIZRR 13

H U N SDI EC K ER REA C T I ON

O
Br2
CCl4
R · Br + AgBr + CO2
ă +
R O Ag

The silver(I) salts of carboxylic acids reacts with halogens to give unstable intermediates which
readily decarboxylate thermally to yield alkyl halides. The reaction is believed to involve homolysis of
the C-C bond and a radical chain mechanism.

Mec hanism :

O O
Br2
R C Oă Ag+ ăAgBr Br
O Homolitic
R
fision

O
ăCO2
RBr R + Br R C O + Br

free radical form

FI N K EL ST EI N REA CT I ON

MX´
R CH2 X R CH2 X´
MX

Treatment of a primary alkyl halide or pseudohalide with an alkali metal halide (e.g. KF, KI)
leads to replacement of the halogen via an SN2 Reaction.

The classic case of finkelstein reaction includes the conversion of alkyl chloride or bromide to alkyl
iodide in presence of sodium iodide in acetone.

Nal
CH3 CH2 Br 
 CH 3CH 2I  NaBr
Acetone

The equilibrium position of the reaction depends on the nucleophilicity of the anion, whether a
good leaving group is present, and whether one anion is better stabilized than the other in a given

ALKYL HALIDES
14 QUIZRR
solvent. For example, reactions with KF will thus lead cleanly to fluoroalkanes, because fluoride is such
a poor leaving group due to the stability of the C-F bond.

OMs 5eq. KX, 5 eq. H2O X


CH3CN / [bmim][BF4]
X:F, Cl, Br, l

In general, the reaction is run with an excess of the metal halide. The use of metal salts that have
a high lattice energy require the addition of a crown ether.

The equilibrium position of the reaction also depends on the solubility of the metal salt in the
solvent used. Thus, the substitution of bromo-and chloroalkanes with KI in acetone leads cleanly to the
desired iodoalkane products, since KCl and KBr are insoluble in acetone and are consequently removed
from the equilibrium :

KI
R CH2 Cl Acetone
R CH2 I

Exception :

RăF + NaI 

The reaction does not occur because the bond energy of R-F is too high.

Ex a m p l e 3
Draw the structures of the major monohalo product in each of the following reactions :

CH3 Br2/Fe CH3 Br2,heat


(a) (b) or UV light
O2N O2N

CH2
peroxide
(c) CH3CH 2C  CH + HCl (1 equiv)  (d) + HBr
CH3

CH3
heat
(e) + HI (f) + Br2
or UV light

ALKYL HALIDES
QUIZRR 15

Solution :
(a) In presence of halogen carrier and absence of light, electrophilic substitution occurs at
position ortho (since the p-position is already occupied) to the electron-donating CH2CH3
group.

CH3 Br2/Fe CH3

O2N O2N Br

(b) In presence of light, side chain (free radical) halogenation occurs at the benzylic hydrogen.

Br

CH3 Br2,heat CH3


or UV light
O2N O2N

CH2
Markonikov
(c) CH3CH2C = CH + HCl H3C
Addition
Cl

CH2
Peroxide
(d) H3C + HBr H3C Br
Anti-Mark. Add.
CH3 CH3

CH3
CH3
(e) Markkonikov l
+ HI
Addition

Br

(f) heat or UV light


+ Br2
(Allylic bromination)
or NBS

ALKYL HALIDES
16 QUIZRR
Re a c t io n s o f A l k y l H a l id e s

alkene

ROH + KCl ROR + AgCl

alc.
aq KOH
KOH

2O
Ag
NH3 moist
R ă NH2 RCl 2ROH + 2AgCl
Ag2O

H
KS

Ag
N

NO
C
Ag

2
KN
O2

KCN
KCl + RSH O
(thiol)
R N + AgCl

RNC + AgCl O

R ă O ă N = O + KCl
RCN + KCl
(Alkyl nitrite)

 R 4 N+ Cl
RCl excess RCl excess
RNH2 
 R 2 NH  R 3 N 
 HCl RCl

So m e B a s i c T e r m s
(1) The halogen atom may leave with its bonding pair of electrons to give a halide ion which
is stable ă a halide is called a good leaving group.
(2) If an atom replaces the halide the overall reaction is substitution.
If the halide loss is accompanied by the loss of another atom, the overall reaction is called
an elimination.
(3) Very often the other atom lost is a hydrogen (as H+). The elimination of H-X is common, and
is called a dehydrohalogenation.
Note : Often substitution and elimination reactions will occur in competition with each other.

ALKYL HALIDES
QUIZRR 17

Mec hanism 1
Alcohol first reacts with SOCl2 to form an intermediate chlorosulphate ester, which gives alkyl
chlorosulphite and HCl. In presence of pyridine, HCl reacts with it to give pyridinium (PyH+) ion and
chloride (Clă) ion. The Clă displaces the leaving group ClSO2ă and chloro sulphite ester decomposes to
SO2, Clă and R-Cl with inversion of configuration.

O Cl
+
ă
R O +S Cl R O S O

H Cl H Cl

Cl

HCl + R O S= O R O S OH

(Alkyl Chlorosulphite)
Cl

+ ă
Py + HCl PyH + Cl

Cl

Clă + R O S=O R Cl + SO2 + Clă

I n t he absence of a base and polar solvent , t he chlor osulphit e est er dissociat es int o an int imat e
ion-pair. The Cl ă of the anion of ion-pair attacks from the front side of R+ to give retention of configuration.
The retention is observed because Cl cannot reach the rear of the R+ group but is close to its front side.

R O ă
O
+
S=O R S=O RCl + SO2

Cl
Cl

intimate ion-pair

ALKYL HALIDES
18 QUIZRR
This is referred as SNi (substitution nucleophilic internal) mechanism because a part of the
leaving group detaches itself from the rest of the leaving group during the process and attacks the
substrate.

H SA B T h e o r y (H a r d So f t A c i d B a s e t h e o r y)
Hard acid : Which has less tendency to accept the lone pair of electrons. For example H+,
Li+, Mg2+, 3Ĉ carbocation.
Soft acid : More tendency to accept pair of electrons. For example Ag+, Au+, Hg22+, Cu2+
Hard base : Which has less tendency to donate lone pair of electrons.

O 2 ,  O  , N  ,F 
  
N is a soft base w.r.t. O 
{Tip : More electronegative the element, higher is hard base character}.


Soft base : Which has more tendency to donate lone pair of electrons. ă Să, S2ă, ă P
1

Generally, hard acids react with hard base and soft acids reacts with soft base.

N u c l e o p h i l ic i t y a n d B a s i c i t y

Nucleophilicity : It is the tendency to attack on electron deficient atom.

ă
Nu + C + depends on rate constant K

Higher the K, higher the nucleophilicity.


Basicity : It is the tendency to abstract a proton

B   H2O 

 BH  OH

In comparing molecules with the same attacking atom, there is generally a direct relationship
between basicity and Nucleophilicity. Stronger bases are better nucleophiles.

T re n d s i n N u c l e o p h i l ic i t y
(1) Species with a negative charge are stronger nucleophiles than analogous species without a
negative charge. (Bases are always stronger nucleophiles than their conjugate acids).

ă ă ă
OH > H2O SH > H2S NH2 > NH3

ALKYL HALIDES
QUIZRR 19

(2) Nucleophilicity decreases from left to right across the periodic table. (The more electronegative
elements hold on more tightly to their non-bonding electrons).

ă
OH > Fă 
:NH3  H2O: (CH3CH2)3P > (CH3CH2)2S

(3) Nucleophilicity increases down the periodic table. (Increase in polarizability and size).

Iă > Bră > Clă > Fă HSeă > HSă > HOă
(CH3CH2)3P > (CH3CH2)3N

whereas basicity decreases down the group.


 Iă, Bră, Clă, Fă
increasing mucleophiticity

decreasing basicity
 

St e r i c Ef f e c t s :

In general, the steric bulk has a detrimental effect on nucleophilicity.

ă
CH3 CH3CH2 ă O

H3C Oă

CH3 t-butoxide
weak nucleophile
t-butoxide strong base
weak nucleophile
strong base

Since nucleophilicity involves the attack of the nucleophile at a carbon center, large groups tend
to hinder this process.

Ex a m p l e 5
In non-polar solvents, which of the following represents correct order of decreasing
nucleophilicity ?

(a) OHă > MeOă > CH3CO2ă (b) OHă > HSă

(c) Fă > Clă > Bră (d) CH3ă > NH2ă > OHă > Fă

ALKYL HALIDES
20 QUIZRR
Solution :
In non-polar solvents, the salts of nucleophile are present as ion-pairs in which nearby cations
diminish the reactivity of the anion. The order in which largest nucleophile appears first, will be the
right order. Ion-pairing in NaOH is stronger than NaOMe or NaHS. In NaF, ion-pairing is stronger
than NaCl and NaCl has stronger pairing than NaBr.
 (d)

Ex a m p l e 6
Explain why Iă is better nucleophile than Fă ?
Solution :
As the size of an atom increases, its outer electrons get further from the attractive force of nucleus.
The electrons are held less tightly and are said to be more polarizable ă they are more able to move
toward a positive charge.
More polarizable atoms can form bonds at greater distances, which gives rise to stronger bonding
in the T.S.

sp3 orbital
little bonding
back lobe
ă
+
+
H H
ă
C·X C X
H
Fă H H H

"hard," small valence shell transition state

more bonding

ă
+
+
H H
ă
C·X C X
H H H
H

"soft," large valence shell transition state

Fluoride is a „hard‰ or low polarizability nucleophile, with its electrons held close to the nucleus,
and it must approach the carbon nucleus closely before orbital overlap can occur.
The outer shell of the „soft‰ iodide has losely held electrons, and these can easily shift and overlap
with the carbon atom at a relatively far distance.

ALKYL HALIDES
QUIZRR 21

So l u b i l it y a n d Ro l e o f So l v e n t
Rule of thumb : Like Dissolves Like
This means that reactants dissolve in solvents of similar polarity i.e., non polar or weakly polar
dissolve in weakly polar solvents only.
That is the reason why hydrocarbons and alklyl halides do not dissolve in water, whose molecules
are high polar and held together by strong hydrogen bonding.
Some terms :

Ć Hydrophilic (water loving) i.e. soluble in water

Ć Hydrophobic/lilophilic (water hating) i.e. very less soluble in water as compared to solubility
in non polar solvents

Note : As the number of carbon increases, the solubility decreases.

CH3OH > CH3CH2OH > (CH3) CH2CH2OH > .... > CH3(CH2)n · CH2OH

N a t u r e o f So l v e n t s

Solvents

Polar protic Polar Aprotic

Ć Though there can be two more categories, i.e., weakly polar aprotic and non polar solvents
but we will constraint our explanation to the above 2 categories only.

1. Po l o r Pro t ic So l v e n t s
These are solvents containing hydrogen that is attached to oxygen or nitrogen and hence acidic
enough to form hydrogen bonds. In short, protic solvents are ones which have acidic hydrogen like
H2O, methanol etc.

H o w t h e y w o rk :
Substrate contains both cations and anions. Here in protic solvents, Cations are attracted to
negative pole of polar solvent. In H2O, negative part is the oxygen atom, as it is most electronegative
and also has unshared lone pair.
To add to this, O is attached with tiny hydrogen atoms which makes oxygen well exposed without
crowding it.

ALKYL HALIDES
22 QUIZRR
Whereas,
Anions are attracted to positive pole of polar solvent which is hydrogen in case of H2O, thus
forming hydrogen bonds.

Case of H2O
+
H
ă + ă
+ O ă H O
H H

2. Po l a r A p r o t i c So l v e n t s
These are solvents which do not have acidic hydrogen i.e. hydrogen is not attached to electronegative
O or N.
Common example of such solvents :

O O

S S CH3
H3C CH3 H3C N

DMSO DMF

H o w t h e y w o rk :
They also dissolve ionic compounds but their action is completely different from protic solvent.
Since they do not have acidic hydrogen they are not able to Solvate anions. Though they do solute
cations by their electro-negative part.
So only this effect takes place.

ALKYL HALIDES
QUIZRR 23

K ey t o suc c ess
Ć For ionic/polar substrate, the more solvated the cations and anions, faster is the reaction.

 Polar protic solvents will favour ionic reactants

i.e., SN1 mechanism (as carbocations are formed in the process)

Ć Solvation differs from ion to ion, for ex in gaseous state, reactivity/basicity of halides is Fă
> Clă > Bră > Iă

but when we talk of protic solvents, the order reverses i.e. Iă > Bră > Clă > Fă

Why ? :
Since F is a small atom, hence the Charge is concentrated in a small space & hence it is able to
form hydrogen bonding at a very large scale as compared to that of Iă which has charge over a large
area & hence weak hydrogen bonding.
Same reasoning can be given for, Li+ & Na+. Again, in Aprotic solvents, since here is no formation
of hydrogen bonding, hence order remains the same i.e Fă > Clă > Bră > Iă
So

Fă H2O
a weak nucleophile
a strong nucleophile
DMSO

Tip : Common polar protic solvents

O O
|| ||
CF3  C  OH H2O CH3OH CH3  C OH CH3CH2OH

Common polar Aprotioc solvents

 O 
 || 
DMSO DMF Acetone  CH3  C CH3 
 
 

Summing up the halogen series

Fă Basicity nucleophilicity nucleophilicity


Clă size increase increase in polar
Bră increases in polar protic solvents
Iă aprotic
solvents

ALKYL HALIDES
24 QUIZRR
N u c l e o p h i l ic Su b s t i t u t i o n (S N )

RX  Nu   R  Nu  X
(Substrate)  Reagent   Product   leaving 
 
 group 
 

SN1 SN2
(Unimolecular nucleophilic substitution) (bimolecular nucleophilic substitution)
 occurs only for those species   Only by those which do 
 capable of forming   not form stable 
   
 a stable carbocation   carbocation 
   
Se c o n d Or d e r / B i m o l e c u l a r n u c l e o p h i l ic Su b s t i t u t i o n (SN 2 )

CH3CH2Cl + ăOH  CH3CH2OH + Clă

Mec hanism :
ă 2
OH CH3

C OH attacks from backside.


3 1
H Cl
H
ă
'S' config OH CH3

H Cl
H

CH3
slow
ă ă
HO C Cl

H H
{trigonal bipyramidal}

ă fast CH3
OH shifts H to backside
i.e. the site left by Clă.
C 'R' config
1 3
HO H H
ALKYL HALIDES
QUIZRR 25

Hence, SN2 leads to inversion of configuration, but only when the same priority group is replaced.
Note : It does not lead to change in d-l configuration.

En e r g y p r o f i l e o f t h e Re a c t io n
The transition state is a point of highest energy (not an intermediate).
ă
CH3
ă ă
HO C Cl

H H
transition
state

CH3
energy

ă
HO C Cl
CH3
H H
ă
HO C Cl

H H
reaction coordinate

Kinetic information tells us that the rate is doubled when the CH3CH2Cl also doubled when the
[HOă] is doubled.
The rate is first order w.r.t. both reactants and is therefore 2nd order overall.

Rate = Kr [CH3CH2Cl] [HOă]

The rate and mechanism are consistent since the mechanism requires a collision between the
hydroxide ion and methyl iodide. Both species are present in the transition state, and the frequency
of collisions is proportional to the concentrations of the reactants.
SN2 = substitution, nucleophilic, bimolecular
Bimolecular means that the transition state of the RDS involves the collision of two molecules
(Bimolecular reactions generally have 2nd order overall rate equations).

Ex a m p l e 7
If the given compound is an ÂRÊ isomer, what configuration will the product have, if it
follows SN2 mechanism.

CH 3 C H  OCH 3  O H  ÂRÊ config
|
Cl
 'R' config. 

ALKYL HALIDES
26 QUIZRR
Solution :

ă 3
HO CH3 CH3

C C
2 1
H3CO Cl HO OCH3
H H
'R' config 'R' config

Here, no inversion takes place because priority of attacking group is not same as the leaving
group.

Ex a m p l e 8
The following reaction proceeds by an SN2 pathway.

C6H5CH2Br + NaN3  C6H5N3 + NaBr

(a) Write an equation for the expected rate constant.


(b) Draw a potential energy diagram for the reaction.
(c) How would the rate of the reaction change if concentration of sodium azide is
doubled?
Solution :
(a) Rate = k [C6H5CH2Br] [NaN3]

(b) ă
C6H3

N3 C Br

H H
Potential energy

C6H5CH2Br + N3ă
C6H5N3 + Bră

Reaction coordinate

(c) The rate will be doubled.

ALKYL HALIDES
QUIZRR 27

V e r sa l i t y o f t h e S N 2 m e c h a n i s m
The SN2 mechanism is a common reaction mechanism and can cover a variety of functional group
transformation of alkyl halides.

All of the type Nu : + R-X  Nu-R + Xă

Nucleophile Product Class

R-X + Iă  R-I alkyl halide

R-X + HOă  R-OH alcohol

R-X + RÊOă  R-O-RÊ ether

R-X + HS ă  R-SH thiol

RăX + : NH3  RăNH2 amine

ă
RăX + N3  RăN3 azide

ă
RăX + CCăRÊ  RăCCăRÊ alkyne

RăX + ăCN  RăCN nitrile

RăX + R´CO2ă  RăCO2R´ ester

RăX + :P(Ph)3  (RăPPh3)+ Xă phosphonium salt

R-X + RÊSă  R-S-RÊ thioether

Note : Halogen exchange reactions are normally used to prepare either iodo-or fluoro-compounds
from other alkyl halides since direct iodination is too slow and direct fluorination is too violent.

H2C=CH-CH2Cl + NaF  H2C=CH-CH2F + NaCl

18 crown 6
H3CăCH2Cl + KF   H3CăCH2F + KCl

N u c l e o p h i l e St r e n g t h
The rate of the SN2 reaction strongly depends on the nature of the nucleophile ă a good
nucleophile gives faster rates than a worse nucleophile.
Consider methanol (CH3OH) and methoxide (CH3Oă) reacting with CH3I. It is found that methoxide
reacts about a million times faster in SN2 reactions than methanol.

ALKYL HALIDES
28 QUIZRR
Generally negatively charged species are much better nucleophiles than analogous neutral species.
The two transition states are different energetically.
The T.S. with methoxide has the negative charge shared over the oxygen atom and the leaving
halide.

H H
methoxide ă ă methoxide + ă
lower Ea H3CO l higher Ea H3CO l
H H H H

In the methanol case, there is no negative charge. The halide has a partial negative charge and
the oxygen has a partial positive charge. This is of higher energy.

Su b s t r a t e Ef f e c t s

ăLeaving group effects


ăsteric effects

(1) Leaving group effects


A good leaving group has the following features :
(1) Electron withdrawing (to polarize the C-X bond, making the C electrophilic).
(2) Stable once it has left (not a strong base).
(3) Polarizable (to stabilize the T.S. ă like Iă previously)

Common leaving groups :


(ions) Clă; Bră, Iă, RSO3ă (sulfonate), RSO4ă (sulfate), RPO4ă (phosphate)
(neutral) H2O, R-OH, R3N, R3P
Hydroxide ions are not good leaving groups (strong bases), but in acidic media, the

ă ă
Br H3C OH Br CH3 OH

+ +
ă
H3C OH H Br H3C OH2 Br CH3
+
H O H

O gets protonated, and now H2O can serve as a good leaving group.

ALKYL HALIDES
QUIZRR 29

Neutral molecules can be good leaving groups from positively charged electrophiles. But the need
to protonate the electrophile first limits the choice of nucleophiles to those that are not strong bases,
since the nucleophile would simply get protonated.

(2) Steric Effects of the Substrate


Large groups on the electrophile hinder the approach of the nucleophile.

Rel. rates for SN2 : CH3X > 1Ĉ > 2Ĉ > 3Ĉ alkyl halides

For an SN2 reaction, the nucleophile must approach the small backside lobe of the C-X sp3 orbital.
Generally, one alkyl group slows the reaction, two alkyl groups make it difficult three alkyl close
to impossible.
Thus decreasing reactivity by SN2

CH3X > CH3CH2X > (CH3)2CHCH2X > (CH3)3CX

decreasing reactivity by SN2

CH3CH2X > CH3CH2CH2X > (CH3)2CHCH2X > (CH3)3CCH2X


decreasing reactivity by SN2

ă CH3 ă CH3 ă CH3


HO HO HO

C Br C Br C Br

H H3C H3C
H H H
ethyl bromide (1Ĉ ) isopropyl bromide(2Ĉ ) t-butyl bromide (3Ĉ )
attack is easy attack is possible attack is impossible

Ef f e c t o f So l v e n t o n S N 2

slow ă + fast
Nuă C Nu C L C
L Nu
(Transition state)

ALKYL HALIDES
30 QUIZRR
En e r g y Pro f i l e

{Here Ts is the transition Ts


state, and GS is the initial
state}

Potential energy
(exothermic
reaction)

path of reaction
Rate of reaction increases only when

(1) If HTS decreases (this is because less energy would be required to reach the transition state)

(2) If HGS increases

(3) If both of the above occur simultaneously.

So m e a p p l i c a t i o n s

(a) MeI + ăOH  MeOH + Iă

H
ă
 ă
HO C I
H H

(b) Meă S+ Me2 + ăOH  MeOH + S Me2

H
ă +
HO C SMe2
H H

(c) MeI + NH3  Me+ NH2 + Iă

H
+
 ă
H3N C I
H H

ALKYL HALIDES
QUIZRR 31

In case of (a) and (b), reacting mixture has full charges whereas transition state has partial
negative charges.

 Reacting species will be more solvated. Hence HGs will decrease more than HTs with the
introduction of protic solvent.

Whereas in (c) reacting mixture is charge free and in transition state some partial charge has
developed. Therefore, Ts will be more solvated. Thus HTs decreases more than HGs in this case.

Ex a m p l e
An optically active 3ăbromoă3ămethyl hexane on hydrolysis gives
(a) 3ămethylă3ăhexanol with retention of configuration.
(b) 3ămethylă3ăhexanol with inversion of configuration.
(c) a partially racemic mixture of 3ămethylă3ăhexanol.
(d) optically inactive 3ămethylă3ăhexanol.
Solution :

3ăbromoă3ămethyl hexane, on ionization gives a 3Ĉ carbocation, which can be attacked by


nucleophile (H2O) to give both dextro and levo in unequal amounts of 3ămethylăhexanol.

 (c)

Ex a m p l e
Identify A, B, C, D, E and F in the following series of reaction.

Br2 aq.KOH Na
A B C
h
alc. KOH
NBS +C
D E F

Solution :
Br OH ONa

Br2 aq.KOH Na
h
A B C
alc. KOH

NBS

Br

ALKYL HALIDES
32 QUIZRR
ONa Br

Williamson
+ synthesis
O
C E F

Ex a m p l e
Arrange following compounds in the decreasing order of their reactivity towards SN2
reaction.
PhCH2(Cl) PhCH(Cl)Me PhC(Cl)Me2
(A) (B) (C)
(a) (C) > (B) > (A) (b) (A) > (B) > (C)
(c) (B) > (A) > (C) (d) (B) > (C) > (A)
Solution :

The rate of SN2 reaction depends on steric crowding in the transition state. More is the steric
crowding, less stable the transition state is, less will be the rate of SN2 reaction. Thus, the reactivity
order of SN2 reaction would be (A) > (B) > (C).

 (b)

Fi r s t o r d e r N u c l e o p h i l ic Su b s t i t u t io n (S N 1 )
There is also an SN1 reaction. (Substitution, nucleophilic, unimolecular)
Consider the reaction of t-butylbromide and methanol :

(CH3)3C-Br + CH3-OH  (CH3)3C-O-CH3 + H-Br

The rate was found to depend only on the concentration of t-butylbromide.

Rate = kr [(CH3)3C-Br]

The rate is first order overall ă unimolecular


It appears that the nucleophile is not present in the R.D.S. (rate determining step), but must react
somewhere after the R.D.S. has occured.

Mec hanism :

Step 1 : Formation of carbocation (R.D.S.)

(H3 C)3 C  Br   CH3 3 C  Br   SLOW 

ALKYL HALIDES
QUIZRR 33

Step 2a : Nucleophilic Attack

+
(CH3 )3 C+ O  CH3   CH3 3 C  O  CH3  FAST 
| |
H H

Step 2 (b) : Loss of a proton

+ +
(CH3)3 C O CH3 + O CH3 (CH3)3 C O CH3 (FAST) + H O CH3

H H

The SN1 reaction is a two step process, with the first being a slow ionization reaction generating
a carbocation.
The second is the quick nucleophilic attack by the nucleophile on the carbocation. (In some case,
like when water or alcohol is the nucleophile, a quick loss of a proton gives the final product).
 Solvation assists the ionization process, since the energy of activation required for breaking
of the bond is largely recovered for solvation of ions produced.

In general :

R-X  R + + Xă (R.D.S., slow)


+ ă
R + Nuc  R-Nuc (fast)

rate limiting
transition state ă
+
+
intermediate Nuc C X
+
+1 +
+2
+
+ single
R +X + ă transition
state
+ Nuc ă
energy

ă ă
R · X + Nuc R · X + Nuc
ă
ă R · Nuc + X
R · Nuc + X

SN1 SN2

The SN1 reaction has two transition states, whereas the SN2 only has one transition state.
Consequently, SN1 rates depend on carbocation stability.

ALKYL HALIDES
34 QUIZRR
Since alkyl groups are known to stabilize carbocations (inductive effects and hyperconjugation),
SN1 reactivities decrease in the order of :

3Ĉ > 2Ĉ > 1Ĉ > methyl cation

(This is the opposite for SN2 reactivity).

H H
H
H H H
H + H +
H H2C Br
H H H H

ă
Nuc

H
H

H H2C Nuc

Resonance stabilized cation are also important for SN1 reactivity, for example, allyl bromide is
much more reactive than other primary halides in SN1 reactions.

L e a v i n g Gro u p Ef f e c t s

In the R.D.S. for an SN1 reaction, the bond to the leaving group is breaking therefore a highly
polarizable leaving group helps stabilize the T.S. through partial bonding as it leaves (like for SN2 case).

The leaving group should be stable after it has left with the bonding electrons, and also be a weak base.

The leaving group starts to take on partial negative charge as the cation starts to form. Good
leaving groups are essential for both SN1 and SN2 reactions.

So l v e n t Ef f e c t s

The R.D.S. of an SN1 reaction involves the formation of 2 ions, therefore polar solvents (which
stabilize ions) enhance SN1 reactivities.

ALKYL HALIDES
QUIZRR 35

Protic solvents are especially useful since the hydrogen bonding stabilizes the anionic leaving
group after ionization.

+ ă
H R O
ă
+ H R
O
H R
+ ă + ă
R·X R X O R O H X H
R H
ionization O
O H
R R
H R O R
solvated ions

St e r e o c h e m i s t r y o f t h e S N 1 Re a c t i o n
(Recall that the SN2 reaction is stereospecific, always proceeding with inversion).
The SN1 reaction is not stereospecific.
Consider the reaction below : CH2CH3
The carbocation produced
C ă
is planar and sp2 hybridized. Cl H
H CH3
O
'S' isomer
O
H H

slow

ă
OH
CH2CH3
(a) (b)
+
C
CH3

)
(a st

(b t
fa
)

s
fa
CH3 CH3

C C
H3C OH
CH3
OH
CH3 CH3
'R' isomer 'S' isomer

ALKYL HALIDES
36 QUIZRR
If the nucleophile attacks from the side that the leaving group is attached, the product displays
retention of configuration. So, S N1 lea ds to rea cemiza tion.

Re a r ra n g e m e n t s i n S N 1 Re a c t io n s

Carbocations will often undergo rearrangements, producing more stable ions.

For example the products of the SN1 reaction of 2-bromo-3-methylbutane and ethanol are a
mixture of structural isomers ă the expected product and a rearranged product.

Br H OC2H5 OC2H5
| | C2H5OH
| |
H3C  CH  C CH3   H3C  CH  CH  CH3  CH3 CH2  C  CH3
| | |
CH3 CH3 CH 3

unrearranged rearranged

The two products arise from the same carbocation. In one case the cation is trapped by the
nucleophile before it can rearrange, whereas the second product arises by quenching of the rearranged
cation by the nucleophile.

Mec hanism :
Step 1 : Formation of carbocation :

Br H H
+
H3C CH C CH3 H3C CH C CH3 Bră

CH3 CH3

Step 2 : Attack of solvent before and after rearrangement.

H OC2H5 H
+ C H OH
| |
H3C CH C CH3 
2 5  H3C  CH  C  CH3
without |
CH3 rearrangement CH3
(20)
rearrange

ALKYL HALIDES
QUIZRR 37

H
OC2H5
+ |
H3C C C CH3 C2H5OH
  CH3 CH2  C  CH3
|
H CH3 CH3
(30)

(Rearrangements do not occur in SN2 reactions since carbocations are not intermediates).

Ex a m p l e
Given the major product (with proper explanation) when following halogen compounds are
treated with sodium exthoxide.

H3C CH3
(a)
H3C Br

Br
(b)
CH3

(c)

Cl
CH3

Solution :

Br
slow + 1,2 hydride shift
(a) H3C H3 C H3C
SN1
CH3 CH3 + CH3
H3 C H3C H3C

2Ĉ carbonium ion (A) 3Ĉ carbonium ion (A)

H3C
H3C O
CH3 +
C2H5
H3C CH3
H3C
major minor

ALKYL HALIDES
38 QUIZRR

CH3 CH3
+ +
CH2 Br CH2 ring
slow
(b)
expansion
CH3 CH3 step

slow
(c) CH3 CH3
+

Cl +
2 carbonium ion (A)

12 hydride shift
(A) + + CH3

+
3 carbonium ion (B)

CH3
CH3
OC2H4
(B) +
OC2H5
Minor Major

Ex a m p l e
Rank the following halides in order of increasing rates of SN1 solvolysis. Explain your
answer.

(a) (b)
Br Cl

(c) (CH3CH2CH2)3CBr (d)


Br

ALKYL HALIDES
QUIZRR 39

Solution :
(c > d > a > b), because
(c) Forms a stable carbocation.
(d) Flexible structure, thus can support a carbocation.
(a) (b) being alkyl bromide are more reactive than an alkyl chloride.
(a) and (b) are less reactive because the three carbon atoms attached to the electron deficient
carbon are tied back units the bicyclic ring structure. They cannot become coplanar without introducing
a great deal of strain into the molecule.

Ex a m p l e
Write the structure of the products that would be obtained in an SN1 reaction of cis-1-
bromo-4-methylcyclohexane with dilute aqueous NaOH.

H3C Br
Aq. NaOH

H H

Solution :

H3C Br H3C H3C H H3C OH


ă
Aq. NaOH OH
+ +
H H H H OH H H

This is an example of syn-elimination where the two leaving groups cannot attain an anti-
periplanar conformation. The products are held in eclipsed conformation.

Ex a m p l e
Give the structure of major substitution product from reaction 1-bromobutane with
(a) (CH3)3N (b) (CH3)2S

(c) CH3COOAg

Solution :

+
(a) (CH3)3N + CH3CH2CH2CH2 ă Br (CH3)3N ă CH2CH2CH2CH3 Br

1-bromobutane Butytrimethylammonium bromide

ALKYL HALIDES
40 QUIZRR

+
(b) (CH3)2S + CH3CH2CH2CH2 ă Br (CH3)2S ă CH2CH2CH2CH3 Br

Butytrimethylammonium bromide

(c) CH3COOAg + CH3CH2CH2CH2 ă Br CH3COOCH2CH2CH2CH3 + Br

butyl ethanoate

Ex a m p l e
Give the structure of the major organic product obtained from the reaction of 2-butyl
bromide with the following reagents :
(a) NaCN
(b) NaSCH3
(c) CH3NH2
Solution :

H3C
CH3
CN CN
Na

H3C H3C
NaSCH3
CH3 CH3
Br S
CH
3N
CH3
H
2

H3C
CH3
NH2

SN 1 v s SN 2
Which mechanism the reaction will follow is decided upon the following 4 factors i.e.
1. Nature of substrate
2. Strength of Nucleophile
3. Nature of leaving group
4. Role of solvent
We will now see the affect of each of these factors.
ALKYL HALIDES
QUIZRR 41

1. Nature of Substrate
RX

1Ĉ 3Ĉ
 will never go for SN1   will always show SN1 
   
  will show SN2 only   because of 3Ĉ stable carbocation 

For 2Ĉ RX we will have to see for more factors

2. Strength of Nucleophile
“for SN2, key to success is the attack of nucleophile”.
So, generally,

weak Nuă : SN1


strong Nuă : SN2

Here in SN2, the effect of steric hindrance also has to be taken in count.

Ć In case, there is not much hindrance, nucleophile can attack easily.

C
X
H H

Now it has 2 alkyl groups present which shows some hindrance. Hence nucleophile has to
be a strong one so as to attack the carbon & go for SN2 attack.

C
X
R
H

Nucleophilicity vs Basicity
In general Nuă & basic character run in parallel but their order is reversed in the same group
of periodic table.

ALKYL HALIDES
42 QUIZRR
I , Bră, Clă, Fă
ă

nuelceophilicity

basicity


Nucleophilicity is a function of polarizability. And bigger the size, greater is polarizability.


And a better nucleophile favours substitution, whereas base favours elimination, which we will
deal later.

Note : Nu+ does not play an important role in SN1 because it does not play any role in rate
determining step.

3. Nature of Leaving group


For substitution, either SN1 or SN2 the presence of a good leaving facilitates the reaction.

A good leaving group a stable ion

a weak base

eg. Iă is a good leaving group because its a weak base. It is a weak base because its a conjugate
base of a strong acid HI.

H H
| |
 CH3 · C · I vs CH3 · C · Cl
| |
CH3 CH3

(a) (b)

(a) will more readily go for substitution as compared to (b) as Iă is a better leaving group than
Clă.

 RI > RBr > RCl > RF


decreasing


order of substitution

4. Role of Solvent
Remember the rule of thumb „like dissolves like‰ as done earlier.

 Ionising solvents solvate ionic compounds.

ALKYL HALIDES
QUIZRR 43

Solvent

Protic solvent Aprotic solvent

Ć (ionising solvents)   Ć 
   
   
 SN1 is preferred   SN2 is preferred 
   
   
   
Ć examples of protic solvents   Ć examples 
   
 H2O (aq)   DMSO 
   
   O 
   || 
 EtOH   
CH3  C  CH3
(acetone)

For eg.

CH3
aq. KOH aq medium
C Br SN1 present, giving
H3C acidic Hydrogen
for SN1.
aq. NaI
SN1

NaI
SN2
acetone
non polarising solvent
so no posibility of SN1

also I is a strong nucleophile


favouring SN2.

ALKYL HALIDES
44 QUIZRR
EL I M I N A T I ON /D EH Y DR OH A L OGEN A T I ON
Also known as 1-2 elimination. This mechanism involves the elimination of the halogen atom and
hydrogen atom from carbon adjacent to the one losing the halogen to form alkene.

ă
(Substrate) C C C=C + H B + X

H Alkene Protonated Leaving


base group

B
(base)
 - Carbon
 - Carbon
 - hydrogen

Elimination  loss of -hydrogen

 ă carbon  where halogen is attached


 ă carbon  carbon adjacent to -carbon
 ă hydrogen  hydrogen attached to -carbon

Note : Reaction is brought about by action of base only.


Before we go into details of 2 mechanisms governing eliminations i.e. E1, & E2, first do a rule
known as ÂSAYTZEFF RULEÊ for alkenes.

Sa y t ze f f Ru l e
It says that in alkenes, the alkene with greater number of alkyl groups attached to the doubly
bonded carbon atom is more stable and hence will be major product for example.

CH3 · CH = CH · CH3 CH3 · CH2 · CH = CH2


(major), more stable (minor)

CH3 · CH C CH3 CH3 · CH 2 · C CH2


| |
CH3 CH3
major, more stable (minor)

ALKYL HALIDES
QUIZRR 45

Ease of formation of alkenes

R2 C = CR2 > R2C = CHR > RCH = CHR > R2CH = CH2 > RCH = CH2 > CH2 = CH2

In dehydrohalogenation, the more stable the alkene the faster it is formed.


Elimination Mechanism

Eli m in a t io n c a n f o l l o w t w o m e c h a n i s m s

Ć E1 mechanism
Ć E2 mechanism

Let us do them in detail separately.

E2 M e c h a n i s m
As the name suggest, E2 follows second order kinetics. This means that the rate is dependent
upon the concentration of 2 substances :

1. Alkyl Hallide

2. Base
X

C C C=C + H B + Xă

rate = k [RX] [: B]

Hence is it also known as Bimolecular elimination.

How it occurs ?
E2 like SN2 is a single step reaction.

X
X
ă
OH ă
C C base
C C C=C +X + H · OH

ă
H H OH

T.S. (Transition State)

ALKYL HALIDES
46 QUIZRR
Bonds to hydrogen and the leaving groups are partly broken bond. Double bond is partly formed
in transition state. All these things are taking place simultaneously.
Factors that stabilize the alkene, also stabalizes the transition state as it is already partly alkene.
There are 2 factors that we look while talking about the stability of alkenes.

1. Saytzeff rule
2. Hoffman rule

We have already discussed Saytzeff rule, & we will discuss Hoffman rule a bit later (Hoffman rule
is opposite of Saytzeff Rule)
Since the transition state depends upon the stability of alkene, hence substituted alkenes are more
stable and hence the order of dehydrohalogenation becomes

3Ĉ > 2Ĉ > 1Ĉ

E1 M e c h a n i s m
As the name suggest it is first order kinetics and analogous to SN1.
here bond breaking & bond formation are same as in E2, however it happens in two steps rather
than just one step as in E1.
E1
(Unimolecular elimination)
Step 1 :

X
+
ă
C C X C [SLOW]

H H

Step 2 :

+
C C C=C + H B [FAST)

Here step 1 is identical to as in SN1, where the formation of carbocation takes place
& in step 2, carbocation reacts with base to yield elimination product, whereas in SN1 carbocation
reacted with nucleophile to yield substituted product.

ALKYL HALIDES
QUIZRR 47

Here again the first step is the rate determining step.

 r = rate = k [RX]

i.e. rate is dependent only on the concentration of substrate and independent of base
concentration.

You can see that there are so many similarities between E1 & SN1 mechanisms

Carbocations formed

 may combine with   rearrange   eliminate a proton 


     
 nucleophile, to   to a more   to form an alkene 
     
 show SN1 mechanism   stable carbocation   Thus showing E1 mechanism 

 Wherever E1 can occur there will be a possibility of SN1 also. We will discuss how to
eliminate this confusion later.

Reactivity in E1 : 3Ĉ > 2Ĉ > 1Ĉ

because you have to check for stabality of carbocation in E1.

Orientation in E1 : Again here saytzeff rule comes into play. Later on we will also see how
Hoffman rule affects the orientation.

ALKYL HALIDES
48 QUIZRR
Let us take an example to understand the mechanism.

CH3 CH3
ă
ăX
CH3 C C CH3 CH3 C CH CH3
+

H X H

now rearrangement
will take place

CH3
now this is the most
stable carbocation CH3 C CH2 CH3
+
that can be formed
from this product (hydride shift)

now -elimination
two options

CH3 CH3

CH2 C CH2 CH3 CH2 C CH CH3


+ +

H H

CH2 = C · CH2 · CH3 CH3 C CH CH3

CH3 CH3
(minor product) (major product)
more substituted alkene

ALKYL HALIDES
QUIZRR 49

E2 v s E1
How to figure out which elimination mechanism is favourable when the order of both mechanisms
is
3Ĉ > 2Ĉ > 1Ĉ
Ć The stronger the base, the more E2 is favoured over E1.
Why
 Because E1 mechanism is independent of base and E2 mechanism depends on the
concentration of base.
Ć Polar medium favours E1, whereas Non polar medium favours E2 This is analogous to SN1
and SN2. Though this effect is not that affective
Ć Bulky base favours E2.
{TIP : alc. KOH is classical reagent for elimination}
Ć E1 mechanism is encountered only with secondary and tertiary substractes or where base
is of low concentration or weak or where base is the solvent i.e. SOLVOLYSIS

EL I M I N A T I ON V S. SU B ST I T U T I ON
This is the most common confusion point among the students. We will try and help you to the best
possible extent
X

C C

SN
H
Z
E

if
Z is

Nucleophile Base
 
Substitution Elimination
 
Favourable conditions favourable conditions
 better nucleophiles, weak base  good base
 lower temperature  Higher temperature
 More polar solvent  Less polar solvent

ALKYL HALIDES
50 QUIZRR
Illust rat io n
Dehydrohalogenation of isopropyl bromide, which takes hours with alc. KOH, is brought
about in less than a minute at room temperature by t ă BuOă K+ in DMSO. Suggest a
possible explanation.

Solution :

CH3
alc. KOH
CH · Br weakly polar reagent suggesting elimination
CH3

CH3

CH3 C Oă A very strong base and presence of polar Aprotic


solvent i.e. ideal condition for elimination hence
CH3 a very fast reaction.
(DMSO)

ALKYL HALIDES
QUIZRR 51

The following chart says everything about elimination vs substitution.

Does substrate has a


good leaving group ?

NO YES
No Reaction

Is Nucleophile / Base strong ?

YES
NO

Nature of substrate

Nucleophile Base is
is strong strong
1Ĉ 2Ĉ 3Ĉ

No Rxn. SN1/E1 SN1/E1

Substrate Substrate

1Ĉ 2Ĉ 3Ĉ 1Ĉ 2Ĉ 3Ĉ
SN2 SN1 SN2 SN1

bulky base
c Po
roti Ap
la
r
P
S

ro
YE

t ic
E2 SN2/E2

SN1 SN2

TIP : In 3Ĉ Alkyl, generally elimination is the major product over substitution.

ALKYL HALIDES
52 QUIZRR
Taking some examples to understand the concept

1. EtO 90% SN2 + 10% E2


CH3CH2Br 

(major) (minor)
CH 3CH2OEt CH2 = CH2

{as EtOă is a good base and nucleophile}

EtO 79% E2  21% SN 2


2.  CH3 2 CHBr 
 (major) (minor)

CH2  CH (CH3 )2 CHOEt


|
CH3

{ a little hindered ÂCÊ atom, elimination preferrable}

EtOă
3.  CH3 3 · C · Br 90% E2 + 10% SN1
25Ĉ
{good base + 3Ĉ RX   favour E2}

55Ĉ ă 66Ĉ C
100%. (E1 + E2)
{high temperature favours elimination}

Here no point of SN2 as -Carbon is sterically hindered.

4. R
CH3COOă
CHBr SN2
R a weak base, aprotic polar solvent, hence SN2

CH3COOH SN1
Protic polar solvent.

5. CH3 CH2CH2 · Cl + CH3O 


 CH3Oă is a strong base and nucleophile
CH3OH

So possibilities of SN2 & E2, but SN2 is major


product, CH3CH2CH2OCH3

ALKYL HALIDES
QUIZRR 53

CH3
| CH3CH3OH
6. CH3 · C · CH2 CH3  ă OH   ăOH is a strong base
|
Br
Ć a 3Ĉ RX, hence possibilities of SN2 / E2.
Ć Hindered carbon for SN2
 E2 favoured

 CH3 · C  CH · CH3
|
CH3

OTs O
| || CH3CH2COOH
7. CH3 · CH2 · CH CH3 + CH3CH2 · C · O  

O
||
here CH3CH2 C ăO ă is a weak base

What happens when the base is bulky or the substrate is sterically hindered ?
Let us take the case when substrate is sterically hindered
In these cases ÂÂHOFFMAN RULEÊÊ is followed, which is anti-saytzeff rule. i.e. less substituted
alkene is the main product.
Take for example

CH3 CH3 CH3


| | | since the base is bulky there is scope for
CH3 · C · C · CH3 CH3 · C · Oă
| | | a SN attack and also the substrate is
H Cl H little bulky so complete NO to SN.

Only elimination, but since base is strong


E2 will be major product.

Now there are 2 positions to attack, i.e.


CH3 CH3
2 -hydrogens at position (1) & (2).
2
CH3 C C CH2 According to Saytzeff Rule -hydrogen
1 +
at position (1) should be eliminated.

H H

But is is not so, elimination of -hydrogen at position 2 takes place i.e. HoffmanÊs Elimination But
WHY ?

ALKYL HALIDES
54 QUIZRR
Position (1) is sterically hindered for such a big base, where -hydrogen at position 2 is free. Alkyl
groups at position (1) makes the attack difficult. Hence elimination occurs at position (2).

CH3 CH3
| |
CH3 · C = CHCH3 + CH3 · CH · CH = CH2
(minor) (major)
(Saytzeff rule) (Hoffman's Rule 99%)

INTERMOLECULAR AND INTRAMOLECULAR REACTIONS :

When a molecule has 2 functional group, also known as bifunctional molecule, two kind of
reactions are possible.
bifunctional molecule

Intermolecular Intramolecular
reaction reaction
 as the name suggests   here also, as the name 
   
 it is the reaction between   suggests, it is the reaction 
 2 molecules   of single molecule within itself. 
   

Let us take an example to understand these 2 mechanisms.


Bifunctional molecule ă
CH2 (CH2)n CH2 O

X
2 functional groups

Intermolecular reaction :

CH2 (CH2 )n CH2 Oă + Br CH2 (CH2)n CH2

X CH2 (CH2)n CH2 O CH2 (CH2)n CH2Oă + Br

Here molecules of the same compound combine together to give inter molecular reaction.

ALKYL HALIDES
QUIZRR 55

Intramolecular Reaction :

CH2 (CH2 )n CH2 L (CH2 )n

Oă CH2 CH2
O
(formation of ring)

I n t e r v s I n t r a -m o l e c u l a r r e a c t i o n s :
Ć Intra molecular reactions are prefered when Rings formed are stable
Ć 5-6 member rings are stable and hence favour intra mechanism.
Ć EXCEPTION : 3 membered rings are also stable and hence show intra mechanism.
For intermolecular
 If ring formation occurs in the form of 4 membered ring or greater than 6 membered rings,
then ring formation is discarded & inter molecular reaction takes place.
Note :
6 membered ring is more stable than 5 or 3 membered ring. So given the option 6 membered ring
will be the major product in reaction.
Some examples of intramolecular reactions

CH2 CH2
formation
CH2 CH2 Br + Bră
of 5 membered +
ring
N
N
CH3 CH3

CH2

CH2 CH2 6 membered


ă
+ Cl
ring
O
CH2 CH2 Cl

O
ă

Cl
3 membered ă
CH2 CH2 CH2 CH2 + Cl
ring
O

ALKYL HALIDES
56 QUIZRR
A M B I DEN T N U CL EOPH I L ES
Some nucleophiles have lone pair of electrons on more than one atom and can attack through
more than one site. Such nucleophiles are called ambident nucleophiles. In such cases, different products
are possible due to attack through different sites. Attack by a specific site can be promoted under special
conditions. Two well-known examples are discussed in detail.

A t t a c k b y CN – n u c l e o p h i le (: –
C  N :)

CN 
R ă N 
 R ăCN  R ăNC + Xă
nitriles isonitriles

In CNă, carbon (negatively charged) will be a soft base as compared to nitrogen. So, if the reaction
proceeds via SN1 mechanism, which produces a free carbocation (a hard acid), then attack through
nitrogen (hard base) will take place. But if the reaction proceeds via SN2 mechanism (small positively
charged carbon is soft acid) then attack through carbon (soft base) will take place. So, if we want to
increase relative yield of nitriles, we can use NaCN or KCN etc. in a less polar solvent, which facilitates
SN2 substitution. Similarly, if we want to increase the yield of isonitriles, we can use AgCN. Ag+ has
very strong affinity for Xă, so it favours the formation of R+ and the reaction proceeds via SN1
mechanism. This will result in attack by hard base given RăNC. Further if we compare primary,
secondary and tertiary alkyl halides, formation of RăNC should be favoured due to more favourable
SN1 substituion in tertiary alkyl halide. But the exception is that tertiary alkyl halides undergo
elimination and the yield decreases. This is because CNă is a strong base, which can also cause
elimination reaction.

A t t a c k b y NO2ă n u c l e o p h i l e ( –O–N = O)

NOă
R ă X 
2  R ă O ă N =O  R ă NO2  X
alkane nitrite nitro alkane

In NO2ă , oxygen (negatively charged) will be a hard base as compared to nitrogen. So, if the

reaction proceeds via SN1 mechanism, then attack through oxygen (hard base) will take place to
produce alkane nitrite. But if the reaction takes place via SN2 mechanism then attack through nitrogen
(soft base) takes place to give nitro alkane.
If we want to increase the yield of nitro alkane, the reaction should proceed via SN2 mechanism,
i.e. we can use NaNO2, KNO2 etc. Moreover, the yield will be best if we use primary alkyl halide and
less polar solvent. Formation of nitrite will dominate, if we use tertiary alkyl halide, more polar solvent
and AgNO2 because Ag+ has strong affinity for Xă and can form a carbocation to force the reaction to
proceed via SN1 mechanism. Primary alkyl halide with AgNO2 chiefly gives nitro alkane but if secondary
and tertiary alkyl halides are used then AgNO2 will yield nitrite as the major product.

ALKYL HALIDES
QUIZRR 57

Ex a m p l e 1 5
Provide the products of the reactions of the following substrates with NaNO2 in EtOH :
(i) năBuCl and (ii) ClCH2OCH2CH3.
Solution :

(i) n-Bu-NO2 and (ii) ONOăCH2OCH2CH3 + EtOăCH2OCH2CH3.

The less the positive charge on the attacked carbon, the more likely it will bond to the less
electronegative nucleophilic site of the ambident ion (N). This happens in the SN2 reaction in (i), where
a CăN bond forms. The greater the positive charge on the attacked carbon, the more likely it will bond
to the more electronegative nucleophilic site of the ambident ion (O). This happens in the SN1 reaction
in (ii), where a CăO bond forms. Since the R+ in (ii) is so stable, it has a long enough half-life to react
with any added nucleophile as well as nucleophillic solvent.

Ex a m p l e 1 6

For the given compound , the


Cl
nucleophilic substitution does not occur regardless of the conditions under which the
reaction is run explain.
Solution :
In the given compound, SN2 reaction does not occur due to impossibility of attack of nucleophile
from rear side while SN1 reaction is also not possible because the carbocation formed after ionization
is not stable as the bridge head carbon having one carbon bridge cannot be sp2 hybridized.

Ex a m p l e 1 7
What kind of substitution does vinyl halides undergo ?
Solution :
In general, vinyl halides are inert towards displacement reactions. This is because of resonance
in vinyl halides,

ă +
CH2 = CH X CH2 CH = X

Now, during SN1 reaction, they ionize to give vinyl cations, which are very unstable because the
carbon bearing positive charge is sp hybridized. More the S-character in carbocation, the less stable it
would be.
During SN2 attack, backside attack of nucleophile is difficult because it would approach in the
plane of molecule where it would be repelled by p electron density. Thus, vinyl haldies have little or
no tendency to undergo SN1 and SN2.

ALKYL HALIDES
58 QUIZRR
I m p o r t a n t Co n c e p t
Behaviour of Silver Ion
Ć Silver ion has got high affinity for halides
Ć Due to this affinity, Ag+ reacts with alkyl halides through SN1 mechanism {They have an
eagerness, carbocation is formed}
1Ĉ alkyl halide
 |
halide 
AgCN
2Ĉ alkyl  · C · NC
|
3Ĉ alkyl halide  (major)

But with KCN

R · X + KCN  R · CN + KX

Similarly

S 1
RX + AgNO2 
N RONO + AgX

S 2
O
RX + KNO2 
N 
 R·N + KX
O

Re a c t io n s o f A l k y l H a l id e
Wu r t z r e a c t i o n
It is a coupling reaction whereby two alkyl halides are reacted with sodium to form a new
carbon-carbon bond :

2R ă X + 2Na  R ă R + 2Na+Xă


The related reaction dealing with aryl halides is called the Wurtz-Fitting reaction.

Lim it at ions
The Wurtz reaction is limited to the synthesis of symmetric alkanes. If two dissimilar alkyl halides
are taken as reactants, then the product is a mixture of alkanes that is, often, difficult to separate.
Since the reaction involves free radical species, a side reaction also occurs to produce an alkene. This
side-reaction becomes more significant when the alkyl halides are bulky at the halogen-attached carbon.
This is because the activation energy required for the SN2 reaction in the second step becomes
significantly high, so the alternate elimination mechanism is favored.
H

ă ăBră
R´CH2 CH2 + H C CH2 RCH2CH3 + R´CH = CH2

R´ Br

ALKYL HALIDES
QUIZRR 59

Mec h an ism
This reaction involves the radical species RĆ.
One electron from sodium is transferred to the halogen to produce a sodium halide and an alkyl
radical.

R ă X + Na  RĆ + Na+Xă

The alkyl radical then accepts an electron from another sodium atom to form an alkyl anion and
the sodium becomes cationic.

RĆ + Na  RăNa+

The alkyl anion then displaces the halide in an SN2 reaction, forming a new carbon-carbon
covalent bond.

R ă Na+ + R ă X  R ă R + Na+Xă

Ex a m p l e 1 6

anhydrous
2CH 3 I  2Na  CH3CH3  2NaI
Sodium ether
Methyl Iodide ethane

anhydrous
2CH 3 CH 2 Cl  2Na  CH 3 CH 2 CH 2 CH 3  2NaCl
Sodium ether Sodium Chloride
n -butane

Solution :
Here methyl iodide and ethyl chloride are reacted with powdered sodium metal in anhydrous
ether. Alkanes containing double the number of carbon atoms are formed, i.e. methyl iodide give
ethane, and ethyl chloride gives n-butane.
The Wurtz Coupling is one of the oldest organic reactions, and produces the simple dimer derived
from two equivalents of alkyl halide. The intramolecular version of the reaction has also found application
in the preparation of strained ring compounds :

(1) Br Cl + 2 Na + 2 NaX

Cl Cl

(2) + 2 Na + 2 NaCl

ALKYL HALIDES
60 QUIZRR
CH EM I ST RY OF GRI GN A RD REA GEN T

Victor Grignard discovered that a dry alkyl halide will react with dry magnesium metal in a dry
ether solvent to produce an organometallic compound which behaves as if it has the structure R-
Mg-X It is now called an alkylmagnesium halide or Grignard reagent :

dry ether
R · X + Mg  R · Mg · X R = alkyl (all types) or vinyl or

alkynyl or aryl X = I, Br, Cl

Grignard Reagent can act both as a base and as a nucleophile. It happens due to the polar nature
of grignard reagent

R  · Mg  X

Also in the preparation of grignard reagent, the reactivity of halides is as follows

I > Br > Cl > F (though F is very less reactive)

Gri g n a r d Re a g e n t a s B a s e
As a base it will try to abstract hydrogen from other compounds like H2O, alcohols, amines
terminal alkynes etc. to form alkanes

HOH
ROH
H
RMgX R N H
RH
RC CH

HX

Gri d n a r d Re a g e n t a s N u c l e o p h i l e
As a nucleophile it will attack electrophiles. Let us take its reaction with compounds like aldehydes
& ketones, esters, acid halides etc to understand it. Also we take into consideration that which
reaction is step up (i.e increase in no. of carbon) or step down (i.e. decrease in no. of carbon)

ALKYL HALIDES
QUIZRR 61

1. With aldehydes :
With aldehydes. RMgX gives alcohols and its a Step up reaction

O O(MgX) O(MgX)
+
H3O
R´ C H + RMgX R´ C H R´ C H

R R
(2Ĉ alcohol)
+ Mg(OH)X

me chanism is simple

ă
O OMgX
ă +
R´ C
+
H + R MgX R´ C H

1Ĉ alcohol are only given by formaldehyde

O OMgX
+
H3O
H C H + RMgX H C H RCH2OH
1Ĉ alcohol
R

2. With Ketones
We have 1Ĉ & 2Ĉ alcohol as product from aldehydes, so for 3Ĉ alcohols Ketones are used. Ketones
on reaction with grignard reagent gives 3Ĉ alcohol.

O OMgX OH
+
H3O
R C R´ + R´´MgX R C R´ R C R´

R´´ R´´

ALKYL HALIDES
62 QUIZRR
+
OMgX H3O OH
O + RMgX
R R
(3Ĉ alcohol)

This is also a STEP UP reaction

3. With CO2
This is also a STEP UP reaction & used in formation of acids.

O O O
+
ă H3O
R MgX + C O C OMgX C OH
+ ă
(acid)

So for RX RCOOH

Mg
+
H3O
RMgX
CO2

4. With esters
It leads to formation of ketones & further to 3Ĉ alcohol

O OMgX

RMgX + R´ C OR´´ R´ C OR´´

O
X
R´ C R + Mg
OR´´
if more RMgX is present
You have to be careful of R´ & R´´ as R´´
is replaced by R. So step up & step down R
will depend upon R & R´´
R´ C OH

ALKYL HALIDES
QUIZRR 63

For aldehydes HCOOR´ is used.

O OMgX
+ X
H3O
RMgX + H C OR´´ H C OR´´ RCHO + Mg
OR´´
R

Note : OR´´ of ester does not appear in aldehyde or ketone

Example :

C2H5X phCOC2H5

Mg

1. phMgBr
C2H5MgX +
2. H3O

CO2

C2H5COOH C2H5COOCH3

5. With Acid Halides

O OMgCl

ă
R MgCl + R´ C Cl R´ C Cl
+

R´ C R + MgCl2
(ketone)

ALKYL HALIDES
64 QUIZRR
O

Here C Cl does not react as it is not a stable compound

 acid halide 


RMgX RMgX
 Ketone   3 alcohol

Note :

RMgX + RCONH2 
  No RXN
+ RCOOH 

6. With Cynides

RMgX + R´C N C N MgX

R
+
H3O

C R

O
(ketone)

with HC N H C N MgX

RCHO
(aldehyde)

7. With Oxiranes

R MgX + CH2 CH2 R CH2 CH2 O MgX

O H3O
+

RCH2CH2OH
(alcohol)

ALKYL HALIDES
QUIZRR 65

Special Case

CH3 CH CH C CH3

ă ă
O O
+
CH3 CH CH C CH3 CH3 CH CH C CH3
+

OMgX OMgX

CH3 CH CH C CH3 CH3 CH C C CH3

R R

OH O H

CH3 CH CH C CH3 CH3 CH C C CH3

R R
(3Ĉ alcohol) tautomerism
major product takes places

CH3 CH C C CH3

R
(minor)

ALKYL HALIDES
66 QUIZRR
A RY L H A L I DE
Preparation
(1) Halogenation of Arenes

NO2 NO2

Cl2, FeCl3

Cl

(2) SandmeyerÊs Reaction

(3) Balz Schiemann Reaction

NH2 N2+ BF4ă F

NaNO2 
ă + BF3 + N2
BF4

N u c l e o p h i l i c A r o m a t ic Su b s t i t u t i o n

(1) SNAr mechanism

When strong electron withdrawing groups are attached at ortho and/or para position to the
halogen atom, The reaction involves formation of carbanion as intermediate.

NH2 OH

275Ĉ C
+ NaOH + Clă
200 atm

(Dows process)

ALKYL HALIDES
QUIZRR 67

Note : Drastic conditions are required for the reaction to take place.

Cl OH
O2N NO2 O2N NO2
75Ĉ C
+ NaOH + Clă
2 atm

NO2 NO2
{easily carried out} {picric acid}

Nu
Cl Cl Nu Cl Nu
O2N NO2 O2N O2N NO2
ă
75Ĉ C NO2
2 atm
ă

NO2 NO2 NO2

ă
Nu Cl Nu
O2N NO2 O2N NO2
ă

NO2 NO2

Here in the above case, the negative charge is stabilized by electron withdrawing group on the
ring.
Note : If NO2 is attached at meta position, then the carbanion again is not stabilized. Hence,
again we will require drastic conditions.

(2) Benzyne Mechanism


The rings which are not activated towards bimolecular displacement, undergo substitution by
benzyne mechanism

Cl NH3
NaNH2
Liq. NH3
H

ALKYL HALIDES
68 QUIZRR
Mec hanism :

Cl ă NH3 ă
NH2
ăNH3 +
ă NH2 · H
H ăCl
(benzyne)
{aromatic}

NH2

OCH3 OCH3
Cl
NaNH2
Liq. NH3

Ex a m p l e :
Which of the following aryl halide undergo nucleophilic substitution by benzyne mechanism ?

Br Br

(a) (b)
CF3 NO2

Br Br
CF3 NO2
(c) (d)

Solution :
Unsubstituted aryl halide and those containing electron releasing groups or electron withdrawing
groups at meta position undergo nucelophilic substitution reactions by benzyne mechanism.
Hence only (a) contain electron withdrawing groups at meta position.

Ex a m p l e :
Heating many alkyl chlorides or bromides in water affects their conversion into alcohol
through SN1 reaction. Order each of the following sets of compounds with respect to
solvolytic reactivity.

ALKYL HALIDES
QUIZRR 69

Br Br

Br
(a)

I II III

Br

Br
(b) Br
I II III

Cl Cl
Cl

(c)

I II III

Solution :
Since they follow SN1 mechanism, hence their reactivity will be governed by the stability of
carbocation formed.
(a) I < II < III
since, III will form 3Ĉ carbocation
(b) II < I < III
III is benzylic carbocation
(c) III < I < II
II is allylic carbocation

Ex a m p l e :
What are the products of the following reactions ?

CH 3
| ă
(a) CH 3 · C · Cl + O CH 3 
|
CH 3

CH 3
| ă
(b) CH 3 · C · O  CH 3 · X 
|
CH3

ALKYL HALIDES
70 QUIZRR
Solution :

CH3 CH3
|  E2
|
(a) CH3 · C · Cl  CH3O   CH3 · C = CH2
|
CH3


OCH3 (nucleophile) cannot attack 3Ĉ carbon, as this carbon has high-electron density.
Hence nucleophilic substitution is ruled out.
ă
Now this CH3O attacks to give elimination product.

CH3 CH3
|  SN 2
|
(b) CH3 · C · O + CH3 X   CH3 · C · OCH3
| |
CH3 CH3

Methyl halides can only undergo SN2 mechanism. Here, (CH3)3 CO ă acts as a nucleophile.

 Doubt : Why would the new nucleophile replace X ă from methyl ? 


 
 Answer : Because X ă is more stable than (CH3)3 CO ă 

Ex a m p l e :
When alkyl halides are treated with aqueous AgNO3, silver halide precipitates and an
alcohol is formed. From what you know about the SN1 reaction, propose a mechanism for
the following conversion.

Ag
+ OH
l and
H2O OH

Solution :
Step I :

CH2 +
+ CH2
Ag
l
+
1 carbonium ion

ALKYL HALIDES
QUIZRR 71

Step II :

CH2+ CH3
1,2 hydride shift +
H
I II
H2O

CH3

OH

OR
Step II :
+
CH2

alkyl shift H2O


(ring expansion) +

Ex a m p l e :
2-Bromopentane, when treated with alcoholic KOH yields a mixture of three alkenes A, B
and C. Identify A, B and C. Which is predominant ?
(Assume reaction proceeds through E2 mechanism)
Solution :

2 5 C H OK +
CH3 ·CH2 ·CH2 · CH · CH3   CH3 · CH2 · CH2 ·CH=CH2
| (alcoholic KOH)
1 ăpentene (A)
Br

and CH3 · CH2 · CH=CH · CH3


2 ăpentene (A)
cis (B) and trans (C)

By Saytzeff rule, substituted alkenes are more stable, hence B or C is predominant than A. Out
of cis and trans, trans is more stable. Hence C is predominant.

Ex a m p l e :
When CH3·CH = CH ă CH2Cl reacts with a lcoholic KCN, a mixture of isomeric products is
obtained. Explain.
Solution :
It can undergo SN1 and SN2 reaction. By SN2 reaction only one product is formed. But by SN1
reaction, intermediate is carbonium ion.

ALKYL HALIDES
72 QUIZRR

Cl
CH3 · CH =CH · CH2 Cl + CN 
S 2

N 
 CH3 · CH = CH · CH2
slow
CN

slow SN 1 fast
CH3 · CH=CH·CH2 CN
(A)
+
CH3 · CH = CH · CH2  CH3 · CH · CH = CH2
+
CN ă CN ă

CH3 · CH = CH · CH2 CN; CH3 · CH · CH = CH2


|
CN
(A) (B)

Thus we get two isomeric products by SN1 reaction.

Ex a m p l e :
Identify A to I in the given sequence of reactions and also give the reactions involved.
[Ignore geometrical isomers].

Br2
A + B + C
(major) (minor)

alc KOH
Now, A  E  D
(major) (minor)

alc KOH
B  E

alc KOH
C  D

NBS alc KOH


D 
 F 
G

NBS
E 
 H  H
(major) (minor)

alc KOH
H  G

ALKYL HALIDES
QUIZRR 73

Solution :
Br
Br
+ + Br

Br
major (A) (B) (C)

Now Br
Alc. KOH
+
(A) minor (D) major (E)

Alc. KOH

Br
(B) (E)

Alc. KOH
Br
(C) (D)

NBS Alc. KOH

Br
(D) (F) (G)

Br
NBS
+

(E) (H) Br (I)


major minor

Br
Alc. KOH

(H) (G)

ALKYL HALIDES
74 QUIZRR
Ex a m p l e :
Give the products of the following displacement reactions.
(a) (R) · CH3CHBrCH2CH3 + MeOă (b) (S) · CH3CHBrCH2CH3 + EtOă

CH 3
(c) cis·4·iodoethylcyclohexane + OHă (d) (S) · Br CO 2 Et  CN 
H

Solution :
These are SN2 reactions involving inversion of configuration. Change in configuration from (R)
to (S) or (S) to (R) is observed only when the nucleophile and leaving group have same priorities.
Thus, reaction (a), (b) and (c) show inversion of configuration whereas reaction (d) shows retention
of configuration.
(a) (S) ă CH3CH (OMe) CH2CH3
(b) (R) ă CH3CH (OEt) CH2CH3
(c) trans ă 4 ă ethylcyclohexanol

CH3
(d) (S) · EtCO2 CN
H

Ex a m p l e :
Give plausible mechanism of the following reaction.
SOCl
MeCH = CHCH2OH 
2 MeCH(Cl)CH = CH 2 (100%)

Solution :
SOCl
(a) MeCH = CHCH2OH 
2  MeCH = CHCH OSOCl 
2


CH2 O

CH S=O Me ă CH(Cl)CH = CH2 + SO2

CH Cl

Me
Transition state

ă
The intermediate MeCH = CHCH2OSOCl dissociates to an intimate ion pair whose ClSO 2 loses
SO2 and Clă attacks at C3 with movement of piăeÊs to give a double bond between C1 and C2.

ALKYL HALIDES
QUIZRR 75

Ex a m p l e 1 1 :
Explain the fact that a small amount of NaI catalyzes the general reaction :

R ă Cl + R´O:ă Na+  R ă OR´ + NaCl


Solution :
With Iă ion, the overall reaction occurs in two steps, each of which is faster than the uncatalyzed
reaction.

Step 1. R ă Cl + Iă  RăI + Clă. This step is faster because Iă, a soft base has more
nucleophilicity than ORă, a hard base.

Step 2. R ă I + R´O:  RăOR´ + Iă. This step is faster because Iă, a soft base has more
nucleophilicity than ORă, a hard base.

Ex a m p l e 1 2 :
Account for the following observations : (a) tăBuF is solvolyzed only in very acidic solutions.
(b) tăBuCl is solvolyzed more slowly than 2-chloroă2, 3, 3ătrimethylbutane (A). (c) tăBuCl is
solvolyzed much faster than 2ăchloroă1, 1, 1ătrifluoroă2ămethylpropane (B). (d) t-BuCl is
solvolyzed more slowly in 90% D2Oă10% dioxane than in 90% H2Oă 10% dioxane solution.
Solution :
(a) Fă is a poor leaving group but H-bonding with a strong acid encourages its departure. This
is an example of electrophilic catalysis.

Me
|
(b) Formation of Me3 C · C+ alleviates some of the steric crowding in (A) induced by the two
|
Me

MeÊs and the tăBu on the -carbon. This is an example of steric acceleration.

Me
|
(c) F3 C · C+ is destabilized by the strongly electron-withdrawing ăCF3 group, making the

solvolysis of (B) slower.


(d) D-bonds are not as stabilizing as H-bonds.

Ex a m p l e 1 3 :
Compare the rates of SN1 and SN2 reactions of (a) cyclopropyl and cyclopentyl chloride and
(b) 1ăchlorobicyclo[2.2.2] octane and 9ăchlorodecalin (A).

ALKYL HALIDES
76 QUIZRR
Solution :

(a) Cyclopropyl chloride is much less reactive than cyclopenytyl chloride in each type of reaction
because the sp2 hybridised carbon (120Ĉ bond angle) created in each transition state augments
the ring strain.

(b) The bridgehead halide is inert by both reaction types. A flat R+ cannot be formed at the
bridgehead carbon, making SN1 reaction impossible and the three bridges prevent the
backside attack necessary for SN2 reaction. Furthermore, inversion is impossible. (A) is a
typical 3Ĉ halide and reacts rapidly via SN1 reaction, but poorly via SN2 reaction.

Ex a m p l e 1 4
Optically active 2-iodobutane on treatment with NaI in acetone gives a product, which
does not show optical activity. Explain why ?

Solution :

In this reaction CăI bond is broken and reformed to give a recemic mixture of 2-iodobutane and
thus remains optically inactive. The presence of Iă provides opportunity for the remaking of the CăI
bond.

CH3 CH3
H I I H
C2H5 C2H5

Ex a m p l e 1 5
Dehydrobromination of compounds (A) and (B) yield the same alkene (C). Alkene (C) can
regenerate (A) and (B) by the addition of HBr in the presence and absence of peroxide
respectively. Hydrolysis of (A) and (B) give isomeric products, (D) and (E) respectively. 1, 1-
diphenyl ethane is obtained on reaction of (C) with benzene in the presence of H+. Give
structures of (A) to (E) with reasons.
Solution :
The given reactions suggest that

alc. KOH HBr


(i) (B) is C6H5CH (Br)CH3  C6 H5CH =CH2 
 C6H5CH(Br)CH3
ă HBr
(C) (B)

HOH

 C6H5CH(OH)CH3

ALKYL HALIDES
QUIZRR 77

alc. KOH HBr


and (A) is C6 H5CH2CH2 Br  C6 H5 CH =CH2 
ă HBr Peroxide
(E)

HOH
C6H5CH2CH2Br 
 C6 H5CH2CH2OH
(A) (D)

(ii) (D) and (E) are isomeric alcohols.

H
(iii) C6H5CH = CH2 + C6H6 
 C6 H5 · CH · CH3
|
C6 H5
1, 1-diphenyl ethane

Ex a m p l e 1 6
The alkyl halide C4H9Br (A) reacts with alcoholic KOH and gives an alkene (B), which
reacts with bromine to give dibromide (C). (C) is transformed with sodamide to a gas (D)
which forms a precipitate when passed through an ammonical silver nitrate solution. Give
the structural formulae of the compounds (A), (B), (C) and (D) and explain the reactions
involved.
Solution :

(1) (D) is a terminal alkyne since it forms precipitate with ammonica

alc. KOH Br NaNH


(2) C4 H9Br  C4 H8 
2  C H Br 
4 8 2
2C H
4 6
ăHBR 2HBr
(A) (B) ( C) (D)
Alkene Dibromide

(3) Thus, if (D) is RăC  CH, then (C) should be R · CH · CH2 because it is formed by the
| |
Br Br

action of Br2 on (B). Thus, (B) should be RăCH=CH2.

(4) Since (B) is obtained by dehydrobromination of (A) and thus, (A) should be RăCH2CH2Br.
(A) is of 4 carbon atoms and therefore, (A) is CH3CH2CH2CH2Br.

ALKYL HALIDES
78 QUIZRR
Re a c t io n s a n d a n s w e r s :

alc. KOH
CH3CH2CH2CH2Br  CH3CH2CH=CH2
ă HBr
1ăbromobutane
(A) (B)

Br
CH3 CH2 CH =CH2 
2  CH CH CH(Br)CH Br
3 2 2
Bută1ăene
(B) (C)

NaNH
CH3 CH2 CH(Br)CH2 Br 2  CH CH C  CH
3 2
2HBr
1ă2ăbromobutane
(C) (D)

CH3 CH2 C  CH  Ag 2 O  CH3 CH2 C  Ag


Bută1ăyne Tollen's While ppt.
reagent

ALKYL HALIDES
Tr ansition Element
& Coor dination
Compound
QUIZRR 3

TRANSITION ELEMENTS & COORDINATION COMPOUND

Introduction

Definition and Electronic Configuration of Atoms

The elements lying between s and p-block elements of the periodic table are collectively known
as tra nsitional elements (T.E.Ês) : These elements either in their atomic state or in any of their
common oxidation state have partly filled (n-1) orbitals of (n-1)th main shell. In these elements
the differentiating electron enters (n-1)d orbitals of (n-1)th main shell and such these are called
d-block elements.
The valence shell configurations of these elements can be represented by (n-1)d 1ă10.ns0, 1, 2. The
configurations clearly show that strictly, according to the definition of d-block elements, Cu, Ag
and Au should be excluded from d-block elements, since these elements, both in their atomic state
[with configuration (n-1)d 10ns1] and in their + 1 oxidation state [with configuration (n-1)d 10], do
not have partly filled (n-1)d-orbitals. Similarly Zn, Cd and Hg which, both in their atomic state
[(n-1)d 10ns2] and in + 2 oxidation [(n-1)d 10] do not contain partly filled (n-1)d orbitals, should
also be excluded from d-block elements. Similar is the case with Pd atom with configuration
4d 105s0. Yet, in order to maintain a rational classification of elements, these elements (viz Cu, Ag,
Zn, Cd, Hg and Pd) are also generally studied with d-block elements.
All the d-block elements are classified into four series viz 3d, 4d, 5d and 6d series corresponding
to the filling of 3d, 4d, 5d and 6d orbitals of (n-1)th main shell. Each of 3d, 4d and 5d series
has ten elements while 6d series has at present only one element viz AC89 whose valence shell
configuration is 6d 1 7s2.

Important Points To be remembered

(i) Electron configuration of Cr and Cu


Cr (24) [Ar]183ds5s1
Cu (29) [Ar]183d 104s1
The 3d 5s1 and 3d 104s1 configuration are preferred due to greater stability of half and fully
filled orbitals (HUNDÊs Rule).
(ii) Electron configuration of second and third transition series elements is not similar to first
transition series. The full explanation is beyond the scope of IIT-JEE. It can only be said
this point that small energy gap with increasing atomic number is one of the reasons.
(iii) Zinc, Cd and Hg have fully filled (n-1)d 10 orbitals in their configuration.
Zn 3d 104s2
Cd 4d 105s2
Hg 5d 106s2

TRANSITION ELEMENTS & COORDINATION COMPOUND


4 QUIZRR
These elements exhibit oxidation state of + 2 in their compounds. In + 2 oxidation state
d-orbitals are fully filled,
Zn+2 ă 3d 10, Cd+2 ă 4d 10, Hg+2 ă 5d 10
Therefore, Zn, Cd and Hg do not exhibit general properties shown by transition elements
but form a group (12) by themselves with similar properties.
(iv) Elements Sc, Y and La exhibit the common oxidation states + 3, in which (n ă 1)d orbitals
are vacant (d 0). Therefore, these elements also do not show general characteristic properties
of transition elements.
(v) True transition elements, thus, are
3d (i) Ti V Cr Mn Fe Co Ni Cu
4d (ii) Zr Nb Mo Tc Ru Rh Pd Ag
5d (iii) Hf Ta W Re Os Ir Pt Au
6d (iv) Rf Db Sg Bh Hs Mt
It is because all the above elements have partially filled (n -1)d orbitals in their atoms or
in one or more of the of the chemically important ions. Therefore, these are true transition
elements in the strict sense.

General Properties

(a) They all are metals.


(b) They are hard, strong, high melting, high boiling metals that conduct heat and electricity.
(c) Many are electropositive to dissolve in mineral acids, a few are noble (Pt, Au etc.)
(d) They exhibit variable oxidation states.
(e) Form paramagnetic compounds.
(f) Form variety of complex compounds.
(g) Many of the compounds are coloured.
(h) Compounds undergo hydrolysis.
(i) Many of the transition metals and their compounds act as catalyst.
(j) Form very important alloys.

Atomic and Ionic radii

The atomic and ionic radii decrease from Sc to Cu, but the variation is small. The outer most
electron configuration is (n ă 1)d Xns2. The screening power of d-orbitals is small and electrons are
being added in inner orbitals [(n ă 1)d orbitals. Therefore, change in radii, atomic and ionic both,
are small.

TRANSITION ELEMENTS & COORDINATION COMPOUND


QUIZRR 5

Ionic radii

Ti2+ V+2 C+2 Mn +2 Fe2+ Co+2 Ni2+ Cu+2 Zn +2

r (Å) 0.80 0.73 0.80 0.90 0.85 0.80 0.76 0.69 0.74

In a group, the atomic and the ionic radii increase with increase in atomic number (because
new orbitals are added in the configuration). It is followed in the first and the second
transition series. But the radii of the third transition series elements are almost equal to the
second transition series elements. The reason for this equality, in radii is the lanthanide
contraction.

Radii in Å

Sc Ti V Cr Mn Fe Co Ni Cu Zn

r (Å) 1.44 1.32 1.22 1.17 1.17 1.17 1.16 1.15 1.17 1.25

Y Zr Nb Mo Tc Ru Rh Pd Ag Cd

r (Å) 1.62 1.45 1.34 1.29 ă 1.24 1.25 1.28 1.34 1.41

La Hf Ta W Re Os Ir Pt Au Hg

r (Å) 1.69 1.44 1.34 1.30 1.28 1.26 1.26 1.29 1.34 1.44

Bonding in Transition metals and its effect on properties

All transition elements are metals. They are high melting, high boiling and have high heat of
atomization and high densities. All these properties show that the atoms in transition elements
are held together by strong metallic bonds. The strength of metallic bond very much depends
upon number of valence electrons. Larger the number of valence electrons stronger the metallic
bond. In the transition elements a minimum of three electrons are present, (n ă 1)d 1ns2. The
presence of d-electrons contribute much towards metallic bonding (d ă d overlap) and the
strength increases with in the number of unpaired electrons in the orbitals. Also the difference
between the mp and the bp is large. This indicates that metallic bond is present in molten state
also.

Ti V Cr Mn Fe Co Ni Cu

mp 1660 1710 1600 1260 1535 1490 1450 1083

bp 3260 3450 2660 2150 3000 2900 2730 2600

TRANSITION ELEMENTS & COORDINATION COMPOUND


6 QUIZRR
M elting points of Transition metals

W
Ta Re
Nb
Hf Os
Ru Ir
Mo Te
Zr
Rh Pt
V
mp(Ĉ C)
La Ti Fe Co Ni
Y Cr Pd Au
Sc. Al Cu
Ag
Mn
Zn
Cd
Hg
Transition metals

The above graph shows thată


(i) 5d metals have highest mp.
(ii) mp increases with increase in the number of unpaired electron in d orbitals and then
decreases, when number of unpaired electrons in dăorbitals decreases.
(iii) Mn and Tc show anomalous values.
(iv) W has highest mp.

Ionization Energy

In a period, the first ionization energy gradually increases from left to right. This is mainly due
to increase in nuclear charge. Generally, the ionization energies of transition elements are
intermediate between those of s and p-block elements. The first ionization potential of the 5d-
elements are higher than those of 3d and 4d-elements due to poor shielding of 4f-electrons.
From 3d  4d series, general trend is observed but not from 4d  5d series because of incorporation
of the 14 lanthanides elements between La and Hf. Third period of transition elements have the
highest ionization energy. This reflects the fact that increase in radius due to addition of extra
shell is compensated by the decrease in radius due to lanthanide contraction.
As the radius of 4d and 5d-elements more or less remains the same, due to which Zeff of elements
of 5d series is higher, which results in high ionization energy of the 5d-elements of transition
series.

Oxidation states

All transition metals exhibit variable oxidation states. It arises due to removal of electrons from
inner d-orbital.
TRANSITION ELEMENTS & COORDINATION COMPOUND
QUIZRR 7

Fe(26) [Ar]183d 64s2


Fe2+ [Ar] 183d 6
Fe3+ [Ar] 183d 5

Element and Electron Arrangement of Unpaired


oxidation states configuration outermost electrons electrons

3d 4s
Ti(22) [Ar]183d 24s2 2

3d
Ti2+ [Ar] 183d 2 2

3d
Ti+3 [Ar] 183d 1 1

V(23) [Ar]183d 34s2 3

V+2 [Ar] 183d 3 3

V+3 [Ar] 183d 2 2

V+5 [Ar]18 _________ 0

Cr(24) [Ar]183d 54s1 6

Cr+2 [Ar] 183d 4 4

Cr+3 [Ar] 183d 3 3

Cr+6 [Ar]18 _________ 0

Mn(25) [Ar]183d 54s2 5

Mn +2 [Ar] 183d 5 5

Mn +4 [Ar] 183d 3 1

Mn +6 [Ar] 183d 1 3

Mn +7 [Ar]18 _________ 0

TRANSITION ELEMENTS & COORDINATION COMPOUND


8 QUIZRR

Element and Electron Arrangement of Unpaired


oxidation states configuration outermost electrons electrons

Fe(26) [Ar]183d 64s2 4

Fe2+ [Ar] 183d 6 4

Fe3+ [Ar] 183d 5 5

Co(27) [Ar]183d 74s2 3

Co+2 [Ar] 183d 7 3

Co+3 [Ar] 183d 6 4

Ni(28) [Ar]183d 84s2 2

Ni2+ [Ar] 183d 8 2

Ni+4 [Ar] 183d 6 4

Cu(29) [Ar]183d 104s1 1

Cu+ [Ar]183d 10 0

Cu+2 [Ar] 183d 9 1

Besides, the above given oxidation states, lower oxidation state (0 or + 1) are also exhibited by
transition metals. But these are stable only in complexes especially with ăacid ligands. Compounds
in different oxidation states differ in properties. Common observations are :
(i) Most common oxidation state is + 2 for first series. (It shows greater stability of 3d orbitals
over 4s)
(ii) The + 1 state is stable (dry state) only for Cu in simple compounds.
(iii) Bonds in + 2 or + 3 states are mostly ionic.
(iv) + 2 state forms basic oxide and hydroxides.

(v) Higher oxidation states + 4, + 5, + 6, + 7 etc. form covalent compounds ( MnO4 has covalent
Mn ă O bond).

TRANSITION ELEMENTS & COORDINATION COMPOUND


QUIZRR 9

(vi) Oxides (or hydroxides) in higher oxidation states are acidic

VO Basic
V2O5 Acidic
CrO Basic
CrO3 Acidic

(vii) Higher oxidation state is more stable down a group

Fe + 2, + 3 Mn + 7, MnO4 S O agent

Ru + 4, + 6 Te

Os + 8 (OsO4) Re + 7, ReO4 not an O agent

(viii) Higher oxidation states are found in oxygen and fluorine compounds only

Mn2O7, MnF7, CrO4 , MnO4 etc.

(ix) Oxides in intermediate oxidation states are amphoteric.

TiO2 

VO2  Amphoteric
Mn2 O3 

M agnetic properties

The substances can be either paramagnetic or diamagnetic. A paramagnetic substance is one


which is weakly attracted into a magnetic field and a diagmagnetic substance is one which is
repelled by a magnetic field. The paramagnetic behaviour arises due to the presence of one or
more singly occupied atomic orbitals, while diamagnetic behaviour is due to presence of unpaired
electrons in the atomic orbitals.

It is represent as ø in Bohr magnetons,

  n  n  2  BM,

where n = number of unpaired electrons.


Transition metals and most of their compounds contain unpaired electrons in their different
oxidation states. Therefore, they are paramagnetic.

TRANSITION ELEMENTS & COORDINATION COMPOUND


10 QUIZRR
Example 1

Compare the magnetic moments of Fe2+ and Fe3+.


Solution :
Following the same procedure as above try to calculate the magnetic moments of the two given
species

n for Fe2+ = 4ø = n(n  2) B.M. = 46 = 24 B.M.

n for Fe3+= 5ø = n(n  2) B.M. = 57 = 35 B.M.

Fe2+ 24
3+

Fe 35

Example 2

Find the oxidation state of Cr in the given complex K2[Cr(NO)(NH3)(CN)4], ø = 1.73 BM.
Solution :
ø = 1.73

ø= n(n  2)  1.73

n (n + 2) = 3
 n = 1
 Number of unpaired electron = 1
 Cr is in + 1 oxidation state.

Colour of Complex Compounds

Many ionic and covalent compounds of transition elements are coloured. In contrast compounds
of the să and păblock elements are almost always white. When light passes through a material,
it is deprived of those wavelengths that are absorbed. If wavelength of the absorption occurs in
the visible region of the spectrum, the transmitted light is coloured with the complementary colour
to the colour of the light absorbed. Absorption in the visible and UV regions of the spectrum is
caused by changes in electronic energy. Thus, the spectra are sometimes called electronic spectra.
Colour may arise from an entirely different cause in ions with incomplete d or f-subshells. This
source of colour is very important in most of the transition metal ions.
In a free isolated gaseous ion, the five dăorbitals are degenerate that is they are identical in
energy. In actual practice, the ion will be surrounded by solvent molecules if it is in solution, by
other ligands if it is in a complex, or by other ions if it is in a crystal lattice. The surrounding

TRANSITION ELEMENTS & COORDINATION COMPOUND


QUIZRR 11

groups affect the energy of some dăorbitals more than others. Thus, the dăorbitals are no longer
degenerate and at their simplest they form two groups of orbitals of different energy. Thus, in
transition element ions with a partly filled dăsubshell it is possible to promote electrons from one
dălevel to another dălevel of higher energy. This corresponds to a fairly small energy difference
and so light it absorbed in the visible region. The colour of a transition metal complex is dependent
on how big the energy difference is between the two dălevels. This in turn depends on the nature
of the ligand and on the type of complex formed. Thus, the octahedral complex [Ni(NH3)6]2+ is
blue, [Ni(H2O)6]2+ is green and [Ni(NO2)6]4ă is brown-red. The colour changes with the ligand
used. The colour also depends on the number of ligands and the shape of the complex formed.

The source of colour in the lanthanides and the actinides is very similar, arising from f  f
transitions. With the lanthanides, the 4făorbitals are deeply embedded inside the atom and are
well-shielded by the 5s and 5p-electrons. The făelectrons are practically unaffected by complex
formation. Hence, the colour remains almost constant for the particular ion regardless of the
ligand.
Some compounds of the transition metal are white, for example Cu2Cl2, ZnSO4 and TiO2. In this
compounds, it is not possible to promote electrons within the dălevel. Cu+ and Zn2+ has a d 10
configuration and the dălevel is completely filled. Ti4+ has a d 0 configuration and dălevel is
empty. In the series Sc(III), Ti(IV), V(V), Cr(VI) and Mn(VII), these ions may all be considered
to have an empty dăsubshell; hence dăd spectra are impossible and they should be colourless.
However, as the oxidation state increases, these states become increasingly covalent. Rather than

forming highly charged simple ions, they form oxoions like TiO2+, VO+2 , VO34 , CrO24 and MnO4 .

VO+2 is pale yellow, but CrO24 is strongly yellow coloured and MnO4 has an intense purple
colour in solution, though the solid is almost black. The colour arises by charge transfer mechanism.
In MnO4 , an electron is momentarily transferred from O to the metal, thus momentarily changing
O2ă to Oă and reducing the oxidation state of the metal from Mn(VII) to Mn(VI). Charge transfer
requires the energy levels on the two different atoms to be fairly close. Charge transfer always
produces more intense colours than the colours generated due to dăd transitions. Charge transfer
is also possible between metalăion and metalăion as seen in prussian blue, Fe4[Fe(CN6)]3.
The s and păblock elements do not have a partially filled dăsubshell, so there cannot be any dă
d transitions. The energy required to promote an s or păelectron to a higher energy level is much
greater and corresponds to ultraviolet light being absorbed. Thus, compounds of s and păblock
elements are typically not coloured.

C omplex Formation

The transition elements have a huge tendency to form coordination compounds with Lewis Base,
i.e., with groups which are able to donate an electron pair. These groups are called ligands. A
ligand may be a neutral molecule such as NH3 or ion such as Clă or CNă etc.

TRANSITION ELEMENTS & COORDINATION COMPOUND


12 QUIZRR
Co3+ + 6NH3  [Co(NH3)6]3+

Fe2+ + 6CNă  [Fe(CN)6]4ă


The reason for transition elements are good in forming complexes are :
(i) Small size and high effective nuclear charge
(ii) Availability of low lying vacant dăorbitals which can accept lone pair of electrons donated
by a ligand.

Catalytic Properties

Transition metals and their compounds are known to act as good catalyst due to the following
reasons :
(i) Due to the variable oxidation states, they form unstable intermediate compounds and provide
a new path with lower activation energy for the reaction (Intermediate compound formation
theory)
(ii) In some cases the finely divided metals or their compounds provide a large surface area for
adsorption and the adsorbed reactants react faster due to the closer contact (Adsorption
theory)
1. TiCl3 Used as Natta catalyst for coordination polymerization of ethene to polyethene
2. V2O5 Converts SO2 to SO3 in the contact process for making H2SO4
3. MnO2 Used as a catalyst to decompose KClO3 to give O2
4. Fe Used in HaberăBosch process for making NH3 (as promoter)
5. Pd Catalyst for hydrogenation
6. Pt/PtO Adam catalyst used for reduction of triple bond to double bond
7. CuCl2 Deacon process or making Cl2 from HCl
8. Ni As Raney nickel for hydrogenation reaction

Formation of Alloys

As the transition elements have similar atomic sizes hence in the crystal lattice, one metal can be
readily replaced by another metal giving solid solution and smooth alloys. The alloys so formed
are hard and often have high melting point.

Interstitial Compound

Transition metals form a number of interstitial compounds. In which they take up atoms of small
size e.g. H, C, B and N in the vacant spaces in their lattices. The presence of these atoms results
in decrease in malleability and ductility of the metals but increases their tensile strength.

TRANSITION ELEMENTS & COORDINATION COMPOUND


QUIZRR 13

Z inc, Cadmium and Mercury

(a) These are the elements of group 12 having electronic configuration (n ă 1) d 10 ns2 and +
2 oxidation state. In these elements the d-subshell is full, hence these are regarded as non-
transition elements which is evident from the following characteristics.
(i) They do not show variable valency except mercury.
(ii) Many of their compounds are white
(iii) Their melting and boiling points are very low.
(b) Unique structure of mercurous ion ă Unlike Zn and Cd, Hg exhibits + 1 well as + 2 oxidation
state. Thus mercurous ion exists are Hg22+ and not as Hg+.
(c) Structure of mercurous ion ă It consists of two atoms linked by a covalent and (ă Hg ă
Hg ă)2+ and explains the diamagnetic character of mercurous ions if it was Hg+ (presence
of an unpaired electron in 6s orbital), mercurous salt should have been paramagnetic.
(d) Anomalous behaviour of mercury,
(i) It is liquid at ordinary temperature while Zn and Cd are solids.
(ii) It is less electropositive than hydrogen and therefore does not displace hydrogen from
acids while Zn and Cd does.
(iii) It does not form hydroxide or peroxide, while Zn and Cd do so
(iv) Mercuric oxide, on heating, gives metallic mercury and oxygen while oxides of Zn and
Cd are stable towards heat.
(v) HgCl2 is covalent while zinc and Cd chlorides are ionic. With NH3, HgCl2 gives a white
ppt. of Hg(NH2)Cl, while Zn and Cd salts from complex ions, [M(NH3)4]2+.

COMPOUNDS OF IRON

Ferrous Sulphate or Green vitriol (FeSO4.7H2O)


The name green vitriol is given because of its green colour.
(i) On heating, green FeSO4.7H2O gives a white anhydrous salt. On strong heating it forms
Fe2O3, SO2 and SO3.

heating
FeSO4 .7H2O  FeSO4  7H2 O
White

heating
2FeSO4  Fe2O3  SO2   SO3 
Brown

(ii) With nitric oxide, ferrous sulphate turns brown due to the formation of nitroso ferrous
sulphate FeSO4.NO.

TRANSITION ELEMENTS & COORDINATION COMPOUND


14 QUIZRR
Ferric Oxide ă Fe2O3 (Reddish Brown)
Preparation :
(i) In lab, it can be prepared by heating ferrous sulphate or ferric hydroxide.
2FeSO4  Fe2O3 + SO2  + SO3 
2Fe(OH)3  Fe2O3 + 3H2O
(ii) It is also obtained as a by product in the manufacture of sulphuric acid from iron pyrites.
4FeS2 + 11O2  2Fe2O3 + 8SO2
Properties :
It is a reddish brown powder, insoluble in water but soluble in acid. Amphoteric in nature and
reacts with acids and alkalies.

NaFeO2 + H2O  Fe2O3 + NaOH

Fe2O3 + 6HCl  2FeCl3 + 3H2O

Fe2O3 + Na2CO3  2NaFeO2 + CO2


Sodium ferrate

Ferric Chloride (Red Black)

Anhydrous ferric chloride, Fe2Cl6 is obtained by heating iron filing in a stream of dry chlorine.
The compound sublimes and collects in the form of deep red-black flaky crystals.
2Fe + 3Cl2  Fe2Cl6
It dissociates on heating above 973 K first into FeCl3 and then into FeCl2 and Cl2.

973K 
Fe2 Cl6  2FeCl3  2FeCl2  Cl2 

Its aq. solution is acidic in nature due to hydrolysis.

FeCl3 + 3H2O  Fe(OH)3 + HCl

It is soluble in water, alcohol and ether and forms hydrates. The most common hydrate of iron
FeCl3.6H2O is yellow in colour.

Ferrous Ammonium Sulphate (MohrÊs Salt), FeSO4.(NH4)2SO4.6H2O


It is prepared by crystallising a solution containing equivalent amounts of iron (II) sulphate and
ammonium sulphate. It is a pale green crystaline compound which does not effloresce like ferrous
sulphate.

COMPOUNDS OF COPPER

Cupric chloride, CuCl2.2H2O (Green Crystal)

TRANSITION ELEMENTS & COORDINATION COMPOUND


QUIZRR 15

Preparation :
2Cu + 4HCl + O2  2 CuCl2 + 2H2O

CuO + 2HCl  CuCl2 + H2O

Properties :
(i) It is deliquescent compound and is readily soluble in water. The dilute solution is blue but
concentrated solution is, however, green. The blue colour is due to complex cation
[Cu(H 2O) 4] 2+
(ii) On heating forms Cu2Cl2
2CuCl2  Cu2Cl2 + Cl2
(iii) CuCl2 + Cu  Cu2Cl2
2CuCl2 + 2H  Cu2Cl2 + 2HCl
2CuCl2 + SnCl2  Cu2Cl2 + SnCl4

Copper Sulphate (Blue Vitriol), CuSO 4.5H2O

Preparation :
(i) It is prepared by dissolving copper (II) oxide or copper (II) carbonate in dil. sulphuric acid.
CuO + H2SO4  CuSO4 + H2O
CuCO3 + H2SO4  CuSO4 + CO2 + H2O
On evaporation, solution is concentrated, blue crystals of CuSO4.5H2O separate out on
cooling.
(ii) On a large scale, copper (II) sulphate is obtained by passing air through a hot mixture of
copper and dil. H2SO4.

2Cu + 4H+ + O2  2Cu2+ + 2H2O

Properties :
(i) On heating CuSO4.5H2O loses water molecules as follows.

373K 423K
CuSO4 .5H2O   CuSO4 .H2O  CuSO4  CuO  SO3
Strong 100C 150C

(ii) CuSO4 + 6NH4OH  [Cu(NH3)4] (OH)2 + (NH4)2SO4 + 4H2O


SchweitzerÊs reagent
(iii) 2CuSO4 + 4KI  Cu2I2  + I2  + 2K2SO4.
A mixture of copper sulphate and lime, under the name of Bordeaux mixture is used as a
fungicide in agriculture.

TRANSITION ELEMENTS & COORDINATION COMPOUND


16 QUIZRR
Cupric Oxide, CuO (Black)

It is called black oxide of copper.


Preparation :
(i) It is prepared by heating following copper compounds :

1
Cu2O + O  2CuO
2 2

2Cu + O2  2CuO


Cu(OH)2  CuO + H2O
2Cu(NO3)2  2CuO + 4NO2 + O2
(ii) On commercial scale, it is obtained by heating malachite, which is found in nature.
CuCO3.Cu(OH)2  2CuO + CO2 + H2O
Properties :
It is black powder, insoluble in water but dissolves in acids. It is reduced to metallic copper by
reducing agents like H, C and Co.
CuO + H2  Cu + H2O

Compounds of Silver

Silver Nitrate, AgNO3 (Lunar Caustic)


Preparation :

Heat
3Ag + 4HNO3   3AgNO3  2H2 O + NO 
(Dilute)

Properties :

(i) Heat
2AgNO3   2AgNO 2 + O 2

red Heat
2AgNO3 +   2Ag + 2NO2 + O 2

(ii) It reacts with iodine in two ways.


(a) When iodine is in excess
5AgNO3 + 3I2 + 3H2O  HIO3 + 5AgI + 5HNO3
(b) When AgNO3 is in excess
6AgNO3 + 3I2 + 3H2O  AgIO3 + 5AgI + 6HNO3
(iii) When treated with alkali, it gives precipitate of silver oxide, which dissolves in excess of
NH4OH

TRANSITION ELEMENTS & COORDINATION COMPOUND


QUIZRR 17

2AgNO3 + 2NaOH  Ag2O  + 2NaNO3 + H2O


brown ppt

2AgNO3 + 2NH4OH  Ag2O  + 2NH4NO3 + H2O


Ag2O + 4NH4OH  2[Ag(NH3)2]OH + 3H2O

(iv) 2AgNO3 + Na2S2O3  Ag2S2O3 + 2NaNO3


Ag2S2O3 + 3Na2S2O3  2Na3[Ag(S2O3)2

Silver Bromide, AgBr

Preparation :
By adding AgNO3 solution to soluble bromide solution.
Properties :
(i) It is insoluble in water and conc. acid but soluble in excess of strong solution of ammonia
(e.g. AgCl is soluble in dilute solution of NH4OH, AgI is insoluble in NH4OH solution.
(ii) Silver halides, are also soluble in KCN and hypo solutions
(iii) On heating, it melts to red liquid.
(iv) It is used as the light sensitive material in photographic films. It is the most sensitive silver
halide to photo-reduction.

COMPOUNDS OF ZINC

Zinc Oxide ă ZnO


Preparation :
It is prepared by burning zinc metal in air or by heating the zinc carbonate or zinc nitrate.
2Zn + O2  2ZnO
ZnCO3  ZnO + CO2
2Zn(NO3)2  2ZnO + 4NO2 + O2

Properties :
Zinc oxide is a white power and becomes yellow on heating and the colour disappears on colling.
(i) It dissolves readily in mineral acids and forms the corresponding salts with alkalies, it forms
zincates [Zn(OH)4]2ă
ZnO + 2HCl  ZnCl2 + H2O
ZnO + 2NaOH + H2O  Na2[Zn(OH)4]
(ii) It reduced to metallic zinc when heated with carbon or hydrogen
ZnO + C  Zn + CO
ZnO + H2  Zn + H2O

TRANSITION ELEMENTS & COORDINATION COMPOUND


18 QUIZRR
Chlorides of Lead

Lead chloride, PbCl2


Preparation :
Prepared by slow direct combination or by the action of boiling concentrated HCl on lead (its oxide
or carbonate).

Pb + Cl2  PbCl2

Pb + 2HCl  PbCl2 + H2 


PbO + 2HCl  PbCl2 + H2O
The usual method of preparation is to precipitate PbCl2 as a white crystalline precipitate by
adding a soluble chloride to a lead salt solution.
Pb (NO3)2 + 2NaCl  PbCl2 + 2NaNO3
Properties :
It is sparingly in cold water but more soluble in hot water. In concentrated solutions of Clă ions,
it dissolves forming complex ions, [PbCl3]ă and [PbCl4]2ă.

Lead tetrachloride, PbCl4

Properties :
PbCl4 is a yellow only liquid. it is heavy and dissolves in organic solvents. It is a covalent and
unstable compound, readily decomposes on heating.


PbCl4  PbCl2 + Cl2 

It is easily hydrolysed by water and forms a double salt with NH4Cl.


PbCl4 + 2H2O  PbO2 + 4HCl
Lead tetrabromide and tetraiodide are unknown, so the dihalides dominate the halogen chemistry
of lead.

OXIDES OF LEAD

Lead monoxide, PbO


Preparation :


2Pb(NO3)2  2PbO + 4NO2  + O2 


PbCO3  PbO + CO2 
Dry air has no action on lead, but in moist air it tarnishes, forming a film of oxide first and finally
basic carbonate, which protects it from further action. On heating in air or oxygen, it forms
litharge, PbO. But prolonged heating gives red lead, Pb3O4.

TRANSITION ELEMENTS & COORDINATION COMPOUND


QUIZRR 19

2Pb + O2  2PbO


6PbO + O2  2Pb3O4
Properties :
(i) At room temperature, it is a yellow amorphous powder that is insoluble in water but dissolves
in acids as well as alkalies.
PbO + 2HCl  PbCl2 + H2O
PbO + 2NaOH  Na2PbO2 + H2O
Thus, it behaves as an amphoteric oxide. The acetic properties being rather feeble.
(ii) It is easily reduced to the metallic state by hydrogen, carbon or carbon monoxide.
(iii) In the red form of PbO, the Pb(II) ions are four co-ordinate but the O2ă ions around the
Pb(II) lie in a square.

Lead dioxide, PbO 2

Preparation :
Action of cold concentrated nitric acid on red lead gives lead nitrate in solution while lead dioxide
is thrown as a chocolate powder.
Pb3O4 + 4HNO3  PbO2  + 2Pb(NO3)2 + 2H2O
Properties :
(i) It is a chocolate coloured powder insoluble in water and dilute acids.
(ii) It liberates oxygen on gentle heating.
2PbO2  2PbO + O2 
(iii) At 440ĈC, it is converted into red lead, Pb3O4.

440C
3PbO2   Pb3O4  O2 

(iv) PbO2 is an amphoteric oxide.


PbO2 + 4HCl  PbCl2  + Cl2  + 2H2O
2PbO2 + 2H2SO4  2PbSO4  + 2H2O + O2 
(v) It is a good oxidizing agent. It oxidizes manganous salts to pink permanganic acid when
boiled in nitric acid solution.
2MnSO4 + 5PbO2 + 6HNO3  2HMnO4 + 2PbSO4 + 3Pb(NO3)2 + 2H2O
(vi) In alkaline medium, chromium hydroxide is oxdized to yellow chromate by PbO2.
2Cr(OH)3 + 10KOH + 3PbO2  2K2CrO4 + 3K2PbO2 + 8H2O
The maroon form of lead(IV) oxide, PbO2, crystallizes in the rutile structure. This oxide is
a component of the cathode of a lead-acid battery.

TRANSITION ELEMENTS & COORDINATION COMPOUND


20 QUIZRR
R ed lead, Pb 3O 4

Preparation :
Roasting of litharge in air at 450ĈC gives a bright red powder.
6PbO + O2  2Pb3O4
It is also known as sindur.

Properties :
Sparingly soluble in water but dissolves in dilute nitric acid.
Pb3O4 + 4HNO3  2Pb(NO3)2 + PbO2  + 2H2O
The above reaction indicates that red lead may be considered as plumbous ortho plumbate,
2PbO.PbO2.

POTASSIUM DICHROMATE (K2Cr 2O 7)

When chromite [Fe(CrO2)2] is fused with Na2CO3 in air (or NaNO3) sodium chromate Na2CrO4
is formed.


2 Fe (CrO2)2 + 4 Na2CO3 + O2  4Na2CrO4 + 4CO2 + Fe2O3

The fused mass is extracted with water and filtered. The filterate is concentrated and acidified
with H2SO4. It produces Na2Cr2O7.
2Na2CrO4 + H2SO4  Na2Cr2O7 + Na2SO4 + H2O
The sodium is concentrated and then saturated with KCl orange crystal of K2Cr2O7 separates out.
Na2Cr2O7 + 2 KCl  K2Cr2O7 + 2NaCl
Sodium dichromate is hygroscopic but not the K2Cr2O7. Therefore, K2Cr2O7 is preferred over
Na2Cr2O7.

Properties :
(i) It is orange crystalline solid. Soluble in water, colour is due to charge transfer.

(ii) With alkali (i.e. pH > 7) it turns yellow due to the formation of chromate.
K2Cr2O7 + 2KOH  2K2CrO4 + H2O
(iii) When K2Cr2O7 is heated it gives O2

 4K CrO + 2Cr O + 3O
4K2Cr2O7  2 4 2 3 2

TRANSITION ELEMENTS & COORDINATION COMPOUND


QUIZRR 21

But (NH4)2Cr2O7 on heating gives N2


(NH4 )2 CrO7  Cr2O3  4H2O + N2
(Orange) (Green)

(iv) Solid K2Cr2O7 with conc. H2SO4 forms a red solution. The red is dichromic acid.
K2Cr2O7 + H2SO4  H2Cr2O7 + K2SO4
When the solution is concentrated bright orange (or red) crystals of CrO3 are formed.

Crytallisation
Red solution 
 CrO3

That is K2Cr2O7 is dehydrated by conc. H2SO4 to bright orange solid CrO3. The (K2Cr2O7
+ conc. H2SO4 ) mixture is used for cleaning glassware as it is highly oxidizing.
(v) K2Cr2O7 as oxidizing agent
K2Cr2O7 is a good oxidizing agent in acid medium and reacts as

Cr2O72  6 e  14H+  2Cr(aq)


3+
 7H2O EĈ = 1.36 V
(green)

It oxidizes :
Fe2+  Fe3+
(H 2S) Să2  S
Iă  I2

SO2  SO42

Reactions :

Cr2O72  6 e  14H+  2Cr 3+  7H2O

Fe 2+  Fe3+  e]  6

Cr2 O72  6Fe2+  14H+  2Cr +3  6Fe2+  7H 2 O

Thus SO2 and H2S turn orange solution of K2Cr2O7 (acidic) to green due to the reduction
+3
of Cr2 O72 ion to Cr(aq) .

(vi) K2Cr2O7 as qualitative reagent


To test H2O2 : K2Cr2O7 (or K2CrO4) in acid solution + amyl alcohol, when treated with H2O2,
CrO5 is formed. It dissolves in amyl alcohol making deep blue solution.

CrO42  2H+  2H2 O2  CrO5  3H2 O

CrO72  2H+  4H2 O2  2CrO5  5H2 O

TRANSITION ELEMENTS & COORDINATION COMPOUND


22 QUIZRR

CrO5 + amyl alcohol  deep blue


CrO5 is not stable in aq. medium and decomposes to Cr+3 and O2.
4CrO5 + 12H+  4Cr+3 + 7O2 + 6H2O
(vii) K2Cr2O7 as quantitative reagent
The acidic solution of K2Cr2O is an oxidizing agent. The solution is a primary standard as :
(a) K2Cr2O7 prepared very pure
(b) Solution of K2Cr2O stable
(c) The solution is much less readily reduced by organic substances
(d) Not affected by light.

Structure of Cr2 O72

The structure involves two tetrahedra joined through Oăatom. Cr is sp3 in Cr2 O72 . The structure

is
Cr ă O (bridging) = 1.78 Å
Cr Cr Cr ă O (terminal) = 1.61 Å
O
Cr Cr = 131Ĉ
(bridging)

The Cr ă O bonds have multiple bond nature due to (d ă p) interaction.

POTASSIUM PERMANGNATE (KMnO 4 )

When MnO2 (Pyrolusite) is fused with KOH and KNO3 (O.A.) K2MnO4 (green) is formed. It
(K2MnO4) is oxidized by Cl2 to KMnO4.

(i) fuse
MnO2 + 2KOH + KNO3 
(ii) water
 K MnO + KNO + H O
2 4 2 2

2K2MnO4 + Cl2  2KMnO4 + 2KCl


Oxidation of K2MnO4 may be caused electrolytically (better).

2MnO 42  2MnO4  2e


ă
Anode reaction

Cathode reaction 2H2O + 2eă  2HOă + H2


Oxidized solution on concentration gives violet (deep purple) crystals of KMnO4 (The crystals
however, appear almost black).

TRANSITION ELEMENTS & COORDINATION COMPOUND


QUIZRR 23

Properties :
(i) Violet solid colour is due to charge transfer.

O · Mn · O

(ii) KMnO4 is soluble in water and the solution is red violet.


(iii) KMnO4 is widely used as a laboratory oxidizing agent. It is oxidant in neutral, acid and
basic solutions. The reaction are :
(a) In acid medium :

MnO4  5e + 8H+  Mn+2  4H2 O, EĈ = 1.54 V


(pink) (colourless)

(b) Neutral or dil. alkali :

MnO4  3e + 2H 2O  MnO 2  4HO 


(black solid)

(c) conc. alkali :

MnO4  e  MnO42

Hydrochloric acid (HCl) is not used to create acid medium as MnO4 oxidizes Clă to Cl2

(Clă ă Cl2, EĈ = 1.34 V).

MnO4 + 5Cl + 8H  Mn + /2Cl2


ă + +2 5

In acid medium MnO4 oxidizes,


Să2  S
Fe2+  Fe3+
Iă  I2

NO2  NO3

SO2  SO42

H2O2  O2

C2O42  CO2

TRANSITION ELEMENTS & COORDINATION COMPOUND


24 QUIZRR
and itself is reduced to Mn+2. Therefore, pink solution of KMnO4 is decolourised in all of the
above reactions. In the case of Să2, white S precipitates.

MnO4 + 5e + 8H  Mn + 4H2O] ï 2


+ +2

C2 O 42  2CO2  2e ] ï 5

2 MnO4  16H+  2C2 O42  2Mn +2  10CO2  8H2 O


Other reactions can be written in the same way.

FeSO4 solution (light green) decolourises MnO4 solution in acid medium. But if excess

MnO4 is added a black ppt. is formed. It is due to oxidation of Mn to MnO2.


+2

MnO4  5Fe2+ + 8H+  Mn+2


acid
 5Fe3+ + 4H2O
(pink) (colourless)

Mn+2 + 2H2O  MnO2 + 2e + 4H+] ï 3

MnO4  3e + 4H+  MnO 2  2H2 O ] ï 2

3 Mn +2  2 MnO 4  2H2 O  5MnO2  4H+

In alkaline solution (Iă) is oxidized by MnO4 to iodate (IO3 )

Iă + 6HOă  IO3 + 6e + 3H2O

MnO4  3e + 2H2 O  MnO 2  4HO  ] ï 2

I   2 MnO4  H2 O  IO3  2MnO2  2OH 

(iv) KMnO4 as volumetric reagent


Standard acidic solution of KMnO4 is oxidizing agent and is used as a volumetric reagent.
But it is a secondary standard (not a primary standard like K2Cr2O7), because KMnO4
converts to MnO2 in water and oxidizes H2O to O2. The reaction is, however, very slow.

MnO4 + 3e + 4H  MnO2 + 2H2O] ï 2


+

H2 O  2H+  2e + 1 / 2 O2 ]  3

2 MnO4  2H+  2MnO2  H2 O + 3 / 2 O 2

It is for the above reason; KMnO4 solution in volumetric analysis is standardized before
every use.

TRANSITION ELEMENTS & COORDINATION COMPOUND


QUIZRR 25

Titration with KMnO4 solution does not require indicator. It acts as a self-indicator because
even a drop of dil. KMnO4 imparts pale pink colour to the solution.

It is used for the estimation of iron, H2O2, NO2 , MnO2 (in pyrolusite) etc.

Example 3

50 mL aqueous solution of FeSO4 required 12 mL of 0.02 M KMnO4 in acid medium for


complete oxidation. Calculate the molarity of ferrous sulphate solution.
Solution :
Balanced redox reaction for oxidation of ferrous ion to ferric ion is

MnO4 + 5Fe + 8H  Mn + 5Fe + 4H2O


2+ + 2+ 3+

1 mole of KMnO4 = 5 mole of FeSO4

MV
Number of moles of KMnO4 used =
1000

0.02 ï 12
=  2.4  10 4
1000

 Number of moles of FeSO4 in 50 mL solution = 5 ï 2.4 ï 10ă4


= 12.0 ï 10ă4

n
Thus, molarity of FeSO4 =  1000
V

12  10 4
=  1000  0.024
50

f-Block Elements

These are also referred as Âinner transition elementsÊ. These are two series of elements, formed by
filling of 4f and 5f-subshells. The elements in which 4f-subshell is filled are called lanthanides
and the elements in which 5f-subshell is filled are called actinides.

Electronic Configuration

(i) Lanthanides
Ce (Z = 58) to Lu (Z = 71) ă (6th period)
General configuration : [Xe] 4f1ă14 5d 0ă1 6s2

TRANSITION ELEMENTS & COORDINATION COMPOUND


26 QUIZRR
(ii) Actinides
Th (Z = 90) to Lr (Z = 103) ă (7th period)
General configuration [Rn] 5f1ă14 6d 0ă1 7s2

General Characteristics

Lanthanides exhibits (III) oxidation state (some elements show (II) and (IV) also).
Many of the compounds are coloured. In the Lanthanide elements, there is a regular decrease in
the radius as the period is traversed. This is known as „Lanthanide contraction‰.
The reason for this contraction is that as we proceed from one element to the next element in the
Lanthanide series, the nuclear charge, i.e., atomic number increases by one unit and one electron
is added to the 4f- energy shell. On account of the very diffused shapes of f-orbitals, the 4f
electrons shield each other quite poorly from the nuclear charge. Thus, the effect of nuclear
charge increase is more than the changed shielding effect. This brings valence shell nearer to the
nucleus and hence the size of atom or ions goes on decreasing as we move in the series.

CO-ORDINATION COMPOUND
Some Basic terms
To be read from NCERT book

Classification of Ligands

There are two ways ligands can be classified :


(I) Classification based on donor and acceptor properties of the ligands.
(i) Ligands having one or more lone pair(s) of electrons are further classified as
(a) Ligands containing vacant -type orbitals can receive back donated -electrons
from the metal ion in low oxidation state. Such ligands have filled donor orbitals
in addition to unsaturated organic molecules. Such ligands have filled donor orbitals
in addition to vacant -acceptor orbitals. Thus, in the complexes formed by such
  
ligands, both metal and the ligand act as donor and acceptors  M   L.
  
(b) Ligands, which have no vacant orbitals to get back donated electrons from the
metal. e.g. H2O, NH3, Fă etc.
(ii) Ligands having no lone pair of electrons but -bonding electrons. e.g. C2H4, C6H6
C5H9ă etc.
(II) Classification based on te number of donor atoms present in the figuers :
Such ligands are of following types :

TRANSITION ELEMENTS & COORDINATION COMPOUND


QUIZRR 27

(i) Monodentate or unidentate ligands


The ligands that can co-ordinate to the central metal ion at one site only are called
monodentate ligands. Such ligands may be neutral molecules, negatively or positively
charged ions. For example,

Fă, Clă, CNă, OCNă, H2O, NH3, NO2, NO+, NH2 NH+3 etc.

A monodentate ligand having more than one lone pair of electrons may simultaneously
co-ordinate with two or more atoms and thus acts as a bridge between the metal ions.
In such a case, it is called a bridging ligand and the complex thus formed is known

as bridged complex. For example, OHă, Fă, NH2 , CO, O2ă, SO24 etc.

(ii) Bidentate ligand : Ligands, which have two donor atoms and have the ability to co-
ordinate with the central atom/ion at two different sites are called bidentate ligands.
For example,

 
H2 N · (CH2 )2 · NH2 (ethylenediamine)

(iii) Tridentate ligands : The ligands having three coordination sites are called tridentate
ligands. For example,

  
e.g. H2 N · CH2 · CH2 · NH · CH2 · NH2

Diethylenetriamine

(iv) Polydentate ligands : The ligands having four or more co-ordination sites are called
polydentate ligands. For example,

H H
N · CH2 · CH2 · N
N · (CH2)2 · N
CH2 CH2
(CH2)2 (CH2)2 H2C O=CăO O·C=O CH2

N N CăOă ăO ·C
H H O O
H H
Triethylenetetramine Ethylenediamine
(trine) tetraacetate ion (EDTA)

(v) Ambidentate ligands : They have two or more donor atoms but, while forming
complexes only one donor atom is attached to the metal ion. The examples of such

ligands are CNă, NO2 NCSă, NCOă etc.

TRANSITION ELEMENTS & COORDINATION COMPOUND


28 QUIZRR
(vi) Chelating ligands : When a bidentate or a polydentate ligand is attached through
two or more donor atoms to the same metal ion forming a ring structure, the ligand
is called chelating ligand.
The chelating ligands form more stable complexes than ordinary unidentate ligands.

I UPAC NOMENCLATURE OF COMPLEXES

The following rules are used for naming all types of complexes :
(1) In case of ionic complexes, the cationic part is named first followed by the anionic part,
irrespective of the fact, whether cation or anion or both are complex.
(2) In naming of complex ion, the ligands are namd first in the alphabetical order followed by
name of the metal ion.
(3) When there are several ligands of the same type then prefix di, tri, tetra, penta are provided
to the ligands. If in ligandÊs name any of this prefix is already present, then to avoid
confusion bis, tris etc. are provided rather than di, tri etc. and ligandÊs name is placed in
paranthesis.
(4) Name of all negative ligands ends with ÂOÊ while the name of all positively charged ligands
ends with Â-iumÊ. Neutral ligands have no special ending.

Name of negative ligands


Ligand Name Ligand Name

Fă Fluoro NH2 Amido

Clă Chloro NO2 nitro

Bră Bromo NO3 nitrato

Iă Iodo S 2O32 thiosulphato

O2ă Oxo SO24 Sulphato

O22 Peroxo CO32 Carbonato

OHă Hydroxo SCNă Thiocynato

CNă Cyano SO32 Sulphito

NCă Isocyanido CH3COOă Acetato

N3ă Nitrido Hă Hydrido

N3 azido S2ă Sulphide

TRANSITION ELEMENTS & COORDINATION COMPOUND


QUIZRR 29

Note : Negative organic ligand having ăyl suffix is NOT replaced by ă O.

Eg. CH3 · methyl

CH3 · CH 2 ethyl

· cyclopentadienyl

Name of positive ligands


NOă · nitrosonium
NO+2 · nitronium

NH2 NH+3 · hydrazinium


(5) Usually common names are provided to the neutral ligands except :
H2O ă Aqua / Auqo
NH3 ă ammine
CO ă carbonyl
NO ă Nitrosyl
(6) Oxidation state of Central metal as is represented by roman numerals including zero
immediately after metalÊs name in small bracket (paranthesis).
In case of naming of complex anion, central metal atom has suffix ă ate along with its
English/Latin name. But in case of complex cation or complex neutral molecule, central
metal atom has no specific suffix.
Usually latin names are used for those metals of which chemical symbol is derived from their
latin name except
English name Latin name
Hg Mercury Hydragesium
Sb Antimony Stibine
W Tungston Wulfrum
(7) In case of polynuclear or bridging complex compound (having 2 or more bridging compounds)
a prefix ø ă before the name of each different bridging group is added. Two or more bridging
groups of the same type are indicated by di-ø ă, tri ă ø ă etc. When a bridging ligand is
attached to more than two metal atoms or ions, this is indicated by a subscript to ø.

NH2
(a) (en)2 Co Co (en)2 (SO4)2
OH

Bis(ethylenediamine) Cobalt (III)ă øăamido ă ø ă hydroxo ă bis(ethylene diamine)cobalt


(III)sulphate

TRANSITION ELEMENTS & COORDINATION COMPOUND


30 QUIZRR

OH
(b) (H2O)4 Fe Fe (H2O)4 (SO4)2
OH

Tetraaquoiron (III) ă di ă ø ă hydroxo ă tetraaquoiron (III) sulphate

(8) For deciding the alphabetical order of ligands, the first letter of the ligandÊs name is to be
considered and prefixes di, tri, tetra, bis etc. are not considered.

Some Examples :

[Pt(NH3)6]Cl4 Hexaammineplatinum (IV) Chloride

[Co(NH3)4(H2O))Cl]Cl Tetraamineaquochlorocobalt (II) chloride

[Cr(H2O)4Cl2] + Tetraaquodichlorochromium (III) ion

[Co(NH3)5(NCS)]Cl2 Penta amineisothiocyanatocobalt (III) chloride

[Ni (CN)4]2ă Tetracyanonickelate (II) ion

Na3[CO(NO2)6] Sodium hexanitro cobaltate (III)

K3[Fe(CN)5NO] Potassium pentacyanonitrosyl ferrate (II)

Fe(CO)5 Pentacarbonyliron (0)

Ni(DMG)2 Bis (dimethylglyoximato) nickel (II)

Example 1

Give the systematic names for the following coordination compounds :


1. [Cr(NH3)3(H2O)3]Cl3
Answer :
triamminetriaquachromium(III) chloride
Solution :
The complex ion is inside the parentheses, which is a cation.
The amine ligands are named before the aqua ligands according to alphabetical order.
Since there are three chlorides binding with the complex ion, the charge on the complex ion must
be + 3 (since the compound is electrically neutral).
From the charge on the complex ion and the charge on the ligands, we can calculate the oxidation
number of the metal. In this example, all the the ligands are neutral molecules. Therefore, the
oxidation number of chromium must be same as the charge of the complex ion, + 3.

TRANSITION ELEMENTS & COORDINATION COMPOUND


QUIZRR 31

2. [Pt(NH3)5Cl]Br3
Answer : pentaamminechloroplatinum (IV) bromide
Solution :
The complex ion is a cation, the counter anion is the 3 bromides.
The charge of the complex ion must be + 3 since it bonds with 3 bromides.
The NH3 are neutral molecules while the chloride carries ă 1 charge.
Therefore, the oxidation number of platinum must be + 4.

3. [Pt(H2NCH2CH2NH2)2Cl2]Cl2
Answer : dichlorobis (ethylenediamine) platinum (IV) chloride
Solution : ethylenediamine is a bidentate ligand, the bis-prefix is used instead of di-.

4. [Co(H2NCH2CH2NH2)3]2 (SO4)3
Answer : tris(ethylenediamine)cobalt (III) sulfate
Solution :
The sulfate is the counter anion in this molecule. Since it takes 3 sulfates to bond with two
complex cations, the charge oneach complex cation must be + 3.
Since ethylenediamine is a neutral molecule, the oxidation number of cobalt in the complex ion
must be + 3.
Again, remember that you never have to indicate the number of cations and anions in the name
of an ionic compound.

5. K4[Fe(CN)6]
Answer : potassium hexacyanoferrate(II)
Solution :
potassium is the cation and the complex ion is the anion.
Since there are 4 K+ binding with a complex ion, the charge on the complex ion must be ă 4.
Since each ligand carries ă 1 charge, the oxidation number of Fe must be + 2.
The common name of this compound is potassium ferrocyanide.

6. Na2[NiCl4]
Answer : sodium tetrachloronickelate(II)
Solution :
The complex ion is the anion so we have to add the suffix ăate in the name of the metal.

TRANSITION ELEMENTS & COORDINATION COMPOUND


32 QUIZRR
7. Pt(NH3)2Cl4
Answer : diamminetetrachloroplatinum (IV)
Solution :
This is a neutral molecule because the charge on Pt+4 equals the negative charges on the four
chloro ligands.
If the compound is [Pt(NH3)2Cl2]Cl2, eventhough the number of ions and atoms in the molecule
are identical to the example, it should be named : diamminedichloroplatinum (II) chloride, a big
difference.

8. Fe(CO)6
Answer : pentacarbonyliron (0)
Solution :
Since it is a neutral complex, it is named in the same way as a complex cation. The common name
of this compound, iron carbonyl, is used more often.

9. (NH4)2[Ni(C2O4)2(H2O)2]
Answer : ammoniumdiaquabis (oxalato) nickelate(II)
Solution :
The oxalate ion is a bidentate ligand.

10. [Ag(NH3)2][Ag(CN)2]
Answer : diamminesilver(I) dicyanoargentate(I)

W ERNER’S THEORY

When aqueous ammonia is added to a solution of cobalt dichloride, CoCl2, a blue precipitate forms
of the corresponding hydroxide, Co(OH)2, which dissolves on the addtion of an excess of ammonia
to give a solution that immediately begins to absorb oxygen and turn brown. From the oxidized
solution, the following compounds can be isolated :
Composition Colour
(I) CoCl3.6NH3 Orange-yellow
(II) CoCl3.5NH3.H2O Pink
(III) CoCl3.5NH3 Purple
A great many other compounds of this type can be prepared, (i) by starting with other cobalt salts,
and (ii) by carrying out further reactions with the compounds first obtained. Among these other
compounds are two other chlorides :
(IV) CoCl3.4NH3 Violet
(V) CoCl3.4NH3 Green

TRANSITION ELEMENTS & COORDINATION COMPOUND


QUIZRR 33

At first sight, the properties of compounds (I) - (V) are very puzzling :
(i) All of them fail to give a brown precipitate of Co(OH)3 when treated with sodium hydroxide
solution - a property expected of compounds containing the Co3+ ion. Only on boiling does
a precipitate form.
(ii) All of them fail to give ammonium chloride when treated with concentrated hydrochloric acid
- a reaction that would be expected of a compound containing ammonia. Only on boiling
with sodium hydroxide is ammonia evolved.
(iii) While all of them give a precipitate of silver chloride when treated with silver nitrate in
aqueous solution, the amount that is precipitated in the cold is in some cases less than
expected, as shown in the table below. In these cases the full amount is only obtained by
leaving the solution to stand for a long time or by boiling it.
(iv) The five compounds give different numbers of ions in solution, as judged from (a) the
electrical conductivities of their solution in comparison with those of other electrolytes, (b)
the extent to which they depress the freezing point of water. The number of ions judged to
be present are shown in the table below.
Moles of AgCl Number of ions present
precipitated per mole in solutions per CoCl3
(I) CoCl3.6NH3 3 4
(II) CoCl3.5NH3.H2O 3 4
(III) CoCl3.5NH3 2 3
(IV) CoCl3.4NH3 1 2

These properties, and those of many other compounds of a similar kind, were brilliantly rationalized
by Alfred Werner in 1893. He proposed what is now referred to as his „coordination theory‰, for
which he was awarded a Nobel prize in 1913. Its principal postulates are as follows :
The first theory was called the WernerÊs theory of co-ordination compounds.
(1) Metals possess two types of valencies :
(a) Primary valency or ionizable valency : It is also referred to oxidation state.
Primary valences were obvious from the stoichiometries of simple compounds such as
NiCl2, Fe2(SO4)3 and PtCl2.
(b) Secondary valency : The valency which a metal atom or cation exercises towards
neutral molecules or negative groups (ligands) in the formation of complex ions. The
secondary valency is also called the coordination number. New materials were frequently
observed when other, independently stable substances, e.g. H2O, NH3 and KCl, were
added to these simple compounds giving, for example, NiCl2. 4H2O, Co2(SO4)3.12NH3
or PtCl2.2KCl. Such species were called complex compounds, in recognition of the
stoichiometric complications they represented, and were considered characteristic of

TRANSITION ELEMENTS & COORDINATION COMPOUND


34 QUIZRR
certain metallic elements. The number of species considered to be added to the simple
compounds give rise to the concept of secondary valence.
Example : In [Pt(NH3)6]Cl4 secondary valency of Pt is 6
(2) Primary valencies are satisfied by negative ions, secondary valencies may be satisfied by
negative ions or neutral molecules.
(3) Ligands satisfying secondary valencies are directed towards fixed positions in space giving
a definite geometry to the complex, but the primary valencies are non-directional. Six valencies
are directed towards a regular octahedron while four are directed towards either a tetrahedral
manner or square planar.
On the basis of postulates (1) and (2) & (3), Werner formulated compounds (I)ă(IV) as shown
diagrammatically below. Ordinary valency bonds are designated by black lines (ăăăăăă) and
bonds between neighbouring atoms by dotted lines (...........). The cobalt atoms have their ordinary
valency of three (as in CoF3) and are given a coordination number of six. The other atoms are
given coordination numbers to match (e.g. four for the nitrogen atom in NH3, leaving one for the
cobalt).
Cl Cl
NH3 NH3
H3N NH3 H3N NH3
Co Cl Co Cl
H3N NH3 H3N NH3
NH3 OH2
Cl Cl
CoCl3.6NH3 CoCl3.5NH3.H2O

Cl
NH3 Cl
H3N NH3 H3N NH3
Co Co Cl
H3N NH3 H3N NH3
Cl Cl
Cl
CoCl3.5NH3 CoCl3.4NH3

Now if in these formulations the black lines are taken to be ionic bonds, and the dotted and double
lines are taken to be bonds of a non-ionic character, the properties of compounds (I)ă(IV) given
above are fully explained. Thus (I) and (II) would be expected to give three Clă ions in solution,
(III) only two, and (IV) only one. This leads to the customary formulations :
(I) [Co(NH3)6]Cl3 containing the [Co(NH3)6]3+ ion;
(II) [Co(NH3)5H2O]Cl3 containing the [Co(NH5)6H2O]3+ ion;
(III) [CoCl(NH3)5]Cl2 containing the [CoCl(NH3)5]2+ ion;
(IV) [CoCl2(NH3)4]Cl containing the [CoCl2(NH3)4]+ ion.

TRANSITION ELEMENTS & COORDINATION COMPOUND


QUIZRR 35

Example 2

A, B & C are three complexes of chromium (III) with the empirical formula H12O6 Cl3Cr. All
the three complexes have water and chloride ion as ligands. Complex A does not react with
concentrated H2SO4 where as complexes B and C lose 6.75% & 13.5% of their original mass
respectively on treatment with concentrated H2SO4. Identify A, B & C.
Solution :
A = (Cr(H2O)6]Cl3
[Cr(H2O6]Cl3 + H2SO4  No reaction because in it all H2O molecules are present in coordinate
sphere.
B = [Cr(H2O)5Cl]Cl2. H2O
[Cr(H2O)5Cl]Cl2. H2O + H2SO4  One mole of H2O is removed because its is present outside
the coordination sphere)
Molecular wt of complex = 266.5

 18
% loss =  100 = 6.75%
266.5

C = [Cr(H2O)4Cl2]Cl.2H2O
[Cr(H2O)4Cl2]Cl.2H2O + H2SO4  Two mole of H2O are removed
(Because these are present outside the coordination sphere

2  18
 % loss =  100  13.50%
266.5

Example 3

A metal complex having composition Cr(NH3)4Cl2Br has been isolated in two forms A and
B. The form A reacts with AgNO3 to give a white precipitate readily soluble in dilute
aqueous ammonia, whereas B gives a pale yellow precipitate soluble in concentrated
ammonia. Write the formula of A and B and state the hybridisation of chromium in each
case. Also calculate their magnetic moments (spin - only value).
Solution :
Formula of A is [Cr(NH3)4ClBr]Cl + AgNO3  AgCl  (white)
Formula of B is [Cr(NH3)4Cl2]Br + AgNO3  AgBr  (yellow)
Cr is in + 3 oxidation state i.e. 3d 3 system
Therefore the hybridisation of Cr is sp3d 2 and spin only magnetic moment.

øS = n(n  2)BM

øS = 3(3  2)BM = 3.87 BM

TRANSITION ELEMENTS & COORDINATION COMPOUND


36 QUIZRR
VALENCE BOND THEORY
To be read from NCERT book

Example 4

The magnetic moment of [MnBr4]2ă is 5.9 B.M. What is the geometry of this complex ion?
Solution :
Since the coordination number of Mn2+ ion in this complex ion is 4, it may be either tetrahedral
(sp3 hybridisation) or square planar (dsp2 hybridisation) as shown below at (b) and (c). But the
fact that the magnetic moment of the complex ion is 5.9 BM. Shows that it should be tetrahedral
in shape rather than square-planar.

(a) Mn2+ ion 3d 4s 4p

(3d 5)

(b) [MnBr4]2ă 3d 4s 4p
ïï ïï ïï ï
(sp3 hybridisationă
tetrahedral shape) (n = 5,  = 5.9 B.M) ă ă ă ă
Br Br Br Br

sp3 hybridisation-
tetrahedral shape

3d 4s 4p
(c) [MnBr4]2ă
ïï ïï ïï ï ï
(dsp2 hybridisationă
square planar shape) Bră Bră Bră Bră
2
dsp hybridisation-
square planar shape

Example 5

How would you account for the following ?

(a) [Fe(CN)6]3ă is weakly paramagnetic while [Fe(CN6]4ă is diamagnetic.

(b) Ni(CO)4 possesses tetrahedral geometry while [Ni(CN)4]2ă is square planar.

(c) [Ni(CN)4]2ă is diamagnetic while [NiCl4]2ă is paramagnetic.

TRANSITION ELEMENTS & COORDINATION COMPOUND


QUIZRR 37

Solution :
(a) [Fe(CN)6]3ă involves d 2sp3 hybridization.

3d 4s 4p
3+ 5
Fe (d )
[Fe(CN)6]3ă
ïï ïï ïï ïï ï ï

2 3
d sp hybridization
One dăorbital is singly occupied, hence it is weakly paramagnetic in nature.
[Fe(CN)6]4ă involves also d 2sp3 hybridization but it has Fe2+ ion as central ion.

3d 4s 4p
2+ 6
Fe (d )
[Fe(CN)6]4ă ïï ïï ïï ïï ï ï

2 3
d sp hybridization
All orbitals are doubly occupied, hence it is diamagnetic in nature.

(b) In the formation of Ni(CO)4, nickel undergoes sp3 hybridization, hence it is tetrahedral in
shape.

3d 4s 4p
8 2
Ni 3d 4s
Ni(CO)4 ïï ïï ï ï

3
sp hybridization

(c) In [Ni(CN)4]2ă ion, Ni2+ undergoes dsp2 hybridization, hence it is square planar in shape.
3d 4s 4p
2+
Ni
[Ni(CN)4]2ă ïï ïï ïï ï

2
dsp hybridization
In [Ni(CN)4]2ă, all orbitals are doubly occupied, hence it is diamagnetic; while in [Ni(Cl4]2ă,
two orbitals are singly occupied, hence it is paramagnetic in nature.
3d 4s 4p

[NiCl4] ïï ïï ïï ï

3
sp hybridization

TRANSITION ELEMENTS & COORDINATION COMPOUND


38 QUIZRR

Strong field ligands like CNă, CO, en, NO2 have very strong electron donating tendency,
hence electrons of central metal ion pair up against HundÊs rule and low spin complexes are
formed.

I mportant points on Back bonding

Back bonding is more dominant than  bonding in carbonyl complexes.


Ć If back bonding increases, electrons in  orbitals of CO increases, thus its bond order decreases,
consequently bond length increases and vice versa.
Ć Shape of anti bonding orbital of CO is

C O

Therefore back bonding takes place with dry orbital metal which is identical in shape.

Example 6

In the following compounds, which has greater bond length of CO ?


[V(CO)6]ă & [Mn(CO)6]+
Solution :

V = [Ar] [4s2] [3d3]

Mn = [Ar] [4s2] [3d5]

Vă =

Mn+ =

V can easily donate the electron to CO through backbonding as compared to Mn has it has half
of orbital.
So backbonding is greater with vandium thus, CO has greater bond length in [V(CO)6]ă.

Example 7

How will you distinguish between the following isomer pairs


(a) (i) [CoBr(NH3)5]SO4 and
(ii) [Co(SO4) (NH3)5]Br
TRANSITION ELEMENTS & COORDINATION COMPOUND
QUIZRR 39

(b) (i) [Cr(H2O)6]Cl3 and


(ii) [CrCl(H2O)5]Cl2H2O
(c) (i) Cis [PtCl2(NH3)2] and
(ii) trans [PtCl2(NH3)2]
(d) The two enantiomers of [CoCl2(en)2]+
Solution :
(a) Isomer (i) gives white ppt of BaSO4 with BaCl2 whereas isomer (ii) does not form a precipitate
with BaCl2.
(b) The water molecule in isomer (ii) is lost easily on heating leading to weight loss whereas the
water molecule in isomer (i) are not easily, being coordinated to the central atom.
(c) Cis isomer (i) has dipole moment, the trans isomer (ii) does not.
(d) One isomer is dextrorotatory whereas the other is laevorotatory.

Example 8

Draw structure of
(i) ZeiseÊs salt anion [PtCl3( 2 - C2H4)]ă
(ii) [Al(CH3)3]2.
Solution :
H
H H H H
C C
Cl Me Me
C Al Al
Pt H H Me Me
C
Cl Cl
H H H

Example 9

Write down the formulae of the following coordination compounds.


(i) Hexaaqua Iron (II) sulphate
(ii) Potassium tetracyanonickelate (III)
(iii) Potassium trioxalatoaluminate (III)
(iv) Chloronitrodiammineplatinum (II)
(v) Potassium hexacyanoferrate (II)
(vi) Chlorodiammineplatinum (II) ion
(vii) Dichlorobis (enthylenediamine) cobalt (III) ion

TRANSITION ELEMENTS & COORDINATION COMPOUND


40 QUIZRR
(viii) Dichlorotetraamminecobalt (III) ion
(ix) Potassium pentacyanonitrosylcobaltate (III)
(x) Sodium ethylnediaminetetraacetato chromate (II)
Solution :
(i) [Fe(H2O)6]SO4
(ii) K[Ni(CN)4]
(iii) K3[Al(C2O4)3]
(iv) [PtCl(NO2)(NH3)2]
(v) K4[Fe(CN)6]
(vi) [PtCl(NH3)2]+
(vii) [CoCl2(en)2] +
(viii) [CoCl2(NH3)4] +
(ix) K2[Co(CN)5NO]
(x) Na2[Cr(CH3COO)4(en)]

Example 10

A solution containing 0.319 g of complex CrCl3.6H2O was passed through cation exchanger
and the solution given out was neutralised by 28.5 ml of 0.125 M NaOH. What is the correct
formula of complex ?
Solution :
The Cl atoms outside the co-ordination sphere will be ionised to produce the acid, HCl.
Thus, milliequivalent of Clă ions outside = milliequivalent of HCl formed
= milliequivalent of NaOH used
= 28.5 ï 0.125
= 3.56

0.319
mole or 1.197 millimole of complex produce 3.56 milliequivalent or millimoles of Clă. Thus,
266.5
1 mole of complex will give 3 mole of Clă, i.e. all the three Cl atoms are outside the co-ordination
sphere. Thus, the complex is [Cr(H2O)6Cl3.

Example 11
(A), (B) and (C) are three complexes of chromium (III) with the empirical formula H12O6Cl13Cr.
All the three complexes have water and chloride ions as ligands. Complex (A) does not
react with concentrated H2SO4, whereas complexes (B) and (C) lose 6.75% and 13.5% of their
original weight, respectively, on treatment with concentrated H2SO4. Identify the octahedral
complexes (A), (B) and (C).
TRANSITION ELEMENTS & COORDINATION COMPOUND
QUIZRR 41

Solution :
(A) : [Cr(H2O)6]Cl3 (Violet)
(B) : [Cr(H2O)5Cl]Cl2.H2O (Green) Molecular weight = 266.5
(C) : [Cr(H2O)4Cl2]Cl.2H2O (Dark green)
Compound (A) contains six water molecules as co-ordinated water and thus, does not lost H2O on
treatment with H2SO4. Compound (B) contains five water molecules as co-ordinated water and
one molecule as lattice water which is lost to H2SO4 showing a loss of 18 g out of 266.5 g, i.e.,
6.75% loss. Similarly, compound (C) contains four co-ordinated water-molecules and two molecules
of lattice water, which are taken out by H2SO4 to show a loss of 13.5%.

Example 12

(a) Write down the IUPAC nomenclature of the given complex along with its hybridisation
and structure
K2[Cr(NO)(NH3)(CN)4]; ø = 1.73 B.M.
(b) Draw the structures of [Co(NH3)6]3+, [Ni(CN)4]2ă and [Ni(CO)4]. Write the hybridisation
of atomic orbitals of the transition metal in each case.
Solution :
(a) Potassium amminetetracyanonitrosoniumchromate (I)
Cr is in + 1 oxidation state and possess d2sp3 hybridisation with one unpaired electron.

 n(n  2)  1(1  2)  3  1.73 BM.

(b) [Co(NH3)6]3+ : Co3+ is d2sp3 hybridised to show octahedral shape.


NH3
H3 N NH3

Co3+
H3N NH3
H3N
[Ni(CN)4]2ă : Ni2+ is dsp2 hybridised to show square planar shapae.
ă ă
NC CN

Ni2+
ă ă
NC CN
[Ni(CO)4] : Ni is sp3 hybridised to show tetrahedral shape. CO

Ni
OC CO

CO
TRANSITION ELEMENTS & COORDINATION COMPOUND
42 QUIZRR
CRYSTAL FIELD THEORY

To be done from NCERT


ISOMERISM :
Two or more substances having the same molecular formula but different structural or spatial
arrangements are called isomers. They are of two types :
isomerism

structural isomerism stereoisomerism

(i) ionization isomerism (i) Geometrical


(ii) Coordination isomerism or cis-trans isomerism
(iii) Linkage isomerism (ii) optical isomerism
(iv) Hydrate isomerism
(v) Ligand isomerism

Structural isomerism :

This type of isomerism due to the difference in structures of coordination compounds.


(i) Ionization isomerism : Complexes that have the same empirical formula and are produced
by the interchange of the position of the ligands inside the complex zone and outside the
complex zone are called ionisation isomers. They give different ions on ionization. For example


[Co(NH3)4Cl2]NO2  [Co(NH ) Cl ]+ + NO ă
3 4 2 2



[Co(NH3)4Cl(NO2)]  [Co(NH ) Cl(NO )]+ + Clă
3 4 2

(ii) Coordination isomerism :


Ionic coordination compounds that contain complex cations and anions can exist as isomers
if the ligands associated with the two metal atoms are exchanged. For example :
(1) [Pt(NH3)4] [PtCl6] and [PtCl2(NH3)4] [PtCl4]
(2) [Co(NH3)6] [Cr(CN)6] and [Cr(NH3)6] [Co(CN)6]

(iii) Linkage isomerism : Isomerism which results when a given ligand is joined to the central
atom through different atoms of the ligand. For example
[Co(NH3)5(NO2)]Cl2
and [Co(NH3)5(ONO)]Cl2
Here, the ligand NO2ă may attach with the central ion either through O-atom or through
N-atom.

(iv) Hydrate isomerism : This type of isomerism arises due to the different position of water
molecules inside and outside the coordination sphere. For example
TRANSITION ELEMENTS & COORDINATION COMPOUND
QUIZRR 43

[Cr(H2O)6]Cl3 and [Cr(H2O)5Cl]Cl2.H2O


Violet colour Blue green colour
(v) Ligand isomerism : Some ligands themselves are capable of existing as isomers, for example
diaminopropane can exist both as i.e.
C H2  C H  CH3 CH2  CH2  C H2
| | | |
NH2 NH2 and NH2 NH2
1,2 diaminopropane 1, 3-diaminopropane

when these ligands are associated to form complexes, these complexes are isomers of each
other.
e.g. [Co (1, 2-diaminopropane)2Cl2]+ and
[Co (1, 3-diaminopropane)2Cl2]+

Stereo isomerism
Stereo isomerism in coordination compounds arises due to different spatial arrangement of ligands
around the central metal ion.

Geometrical isomerism

In complexes ligands may occupy different sites. When the ligands in question occupy adjacent
positions it is called ÂCisÊ (Cis, Latin, meaning „On this side‰) and when they occupy opposite sites,
it is called ÂtransÊ (trans, Latin meaning „across‰). Therefore, geometrical isomerism is also called
ÂCis ă Trans Âisomerism. This type of isomerism is not possible for complexes of CN 2 or 3 or for
CN 4, tetrahedral complexes.
L

L
M
L·M·L L
M L

L L L
CN2 CN3 CN4
(Linear) (r PL) (Td)
In this complexes all ligand positions are adjacent not opposite. However, cis-trans isomerism is
common in square planar and octahedral complexes.
(a) Square planar
L1 L4 L1, L2
L1, L3 L2, L3
M
L2, L4 L3, L4
L2 L3 L1, L4
opposite (trans) adjacent (Cis)
TRANSITION ELEMENTS & COORDINATION COMPOUND
44 QUIZRR
(b) Octahedral
L1 L1, L2 L6, L3
L5 L4 L1, L6 L1, L3 L6, L2

M L2, L4 L1, L4 L6, L5


L3, L5 L1, L5 L6, L4
L2 L3
opposite adjacent (Cis)
L6 (trans)

(1) CNă4 Sq planar complexes


The following types of four coordinated sq. planar complexes show cis-trans isomerism.
(i) MA2B2
[Pt(NH3)2Cl2] diammine dichloro Pt (II)

H3N Cl Cl H3N

Pt Pt

H3N Cl H3N Cl
Cis trans
(Clă and NH3 ligands are adjacent) (Clă and NH3 ligands are opposite)
Cis and trans isomers may be distinguished by dipole moment measurement

(ii) MABX2
[Pt(NH3)PyCl2] (Py = pyridine C5H5N)

H3N Cl H3N Cl

Pt Pt

Py Cl Cl Py

Cis trans
(iii) [MABCD]
[PtNH3(NH2OH)Py NO2]+ Ammine hydroxylamine pyridine nitro Pt (II)
In such complexes three of the ligands (B, C or D) may be trans to A.
H3N NO2 O2N NH3 NO2 NH2OH

Pt Pt Pt

Py NH2OH Py NH2OH Py NH3


Such complexes are best denoted as [M < AB > CD], the ligands in the angular brackets are
trans to each other.

TRANSITION ELEMENTS & COORDINATION COMPOUND


QUIZRR 45

(v) Geometrical isomerism is also found in square planar complexes containing unsymmetrical
bidentate ligands e.g. [M (AB)2], gly = NH2 ă CH2 ă COOă

(2) CN ă 6 (Octahedral complexes)


Geometrical isomerism in octahedral complexes is also very common. The familiar examples
include compounds of the type :
(i) [MA4X2]
(ii) [MA4XY]
(iii) [M(AA)2X2]
(iv) [M(AA)2XY]
(v) [MA2X2Y2]
Where M = Co (III), Cr (III), Rh (III), Pt (IV) etc.
A ă A = Symmetrical bidentate ligands
X & Y = ă ive ligands.
For example
(i) [Cr(NH3)4Cl2] +
+ +
Cl Cl
H3N Cl H3N NH3

Cr Cr

H3N NH3 H3N NH3


NH3 Cl

Cis trans

TRANSITION ELEMENTS & COORDINATION COMPOUND


46 QUIZRR
(ii) [Co(en)2Cl 2] +

Cl Cl
N Cl N N

Co Co

N N N N
N Cl

Cis trans

A few complexes of the type [MA3X3] are also known e.g. [Rh(Py)3Cl3]

Cl Cl
Py Cl Py Cl

Rh Rh

Py Cl Py Py
Py Cl

Facial (fac) Meridional (mer)

Largest numbers of geometrical isomers are found for complex of the type [MABCDEF] where
each ligand is different e.g. [Pt (Py) (NH3) (NO2) ClBrI].
Unsymmetrical bidentate ligands also produce cis-trans isomers, e.g. [Cr(gly)3].

gly gly
O O
N N N N

Cr gly Cr gly

O O O O
gly N gly O
cis trans

Preparations of complex compounds

Complex compounds are generally prepared by the following methods :


(i) Substitution reaction in aq. solutions
This method involves a reaction between a metal salt in aq solution and a coordinating
agent, e.g. aq. solution of CuSO4 and excess NH3 forms a Cu2+ ammine.

TRANSITION ELEMENTS & COORDINATION COMPOUND


QUIZRR 47

[Cu(H2O)6]SO4 + 4NH3  [Cu(NH3)4]SO4 + 6H2O


(Blue) (Deep Blue)

(ii) Substitution reactions in the absence of solvent


Direct reaction between anhydrous metal salt and liquid NH3 produces metal ammines, e.g.
NiCl2 + 6NH3(l)  [Ni(NH3)6]Cl2
Yellow Violet

(iii) Redox reactions


Co(II) salt forms a Co(III) complex
4[Co(H2O)6]Cl2 + 4NH4Cl + 20NH3 + O2  4[Co(NH3)6]Cl3 + 26H2O

Stability of complex compound in solution

Complex formation can be seen as a Lewis acid ă base reaction. The Mn+ is Lewis acid and the
ligands are Lewis bases. If the interaction between Mn+ and the ligands is strong, the complex
is thermodynamically stable. The reaction between Mn+ and the ligands can be written as
Mn+ + xLmă  [MLX)p+, where (n+) + (m ă) = (p+)

The equilibrium constant k is stability constant and it is defined as

[ML X ] p
k
[Mn ][Lm ]X

The numerical value of k gives an indication of thermodynamic stability. Some stability constants
are given below :
K
Ag+ + 2NH3  [Ag(NH3)2]+ 1.6 ï 107

Co3+ + 6NH3  [Co(NH3)6]3+ 5.3 ï 1033

Cu2+ + 4CNă  [Cu(CN4)2ă]+ 2.0 ï 1027

Cu2+ + 4NH3  [Cu(NH3)4]2+ 4.5 ï 1011


The value of k for Cu2+ complexes with NH3 and CNă show that cyanocomplex is more stable than
NH3 complex.

Effective atomic number (EAN) rule

Sidwick proposed effective atomic number (EAN) to explain the stability of metal in a complex.
EAN is defined as the total number of electrons on the metal atom or ion after the complex
formation which should be equal to that of the next higher noble gas.
If Z is the atomic number, (O.N). is the oxidation number and (C.N.) is the co-ordination number
of the central metal atom or ion in the complex then
EAN = Z ă (O.N.) + 2 ï (C.N.)
TRANSITION ELEMENTS & COORDINATION COMPOUND
48 QUIZRR
If EAN of an element in a complex is equal to atomic number of a noble gas. The complex is
considered to be more stable.

Example 13

Draw the structures of [Co(NH3)6]3+, [Ni(CN)4]2ă and [Ni(CO)4]. Write the hybridisation of
atomic orbitals of the transition metal in each case.
Solution :
Hybridization of Co+3 is d2sp3, shape is octahedral.

NH3
NH3 NH3

Co

NH3 NH3
NH3

Hybridization of Ni+2 is dsp2 so shape is square planar.

NC CN

Ni

NC CN

Hybridisation of Ni is sp3 so shape is tetrahedral.

CO

Ni
CO CO CO

Colour of Transition metal compounds

A chemical substance looks coloured if it absorbs a portion of visible light ( ă 3800 ă 7800 Å) and
transmits the rest. For example, if a compound absorbs the yellow-red (6000 ă 7500 A) it appears
green (i.e. transmitted light) i.e. the colour is complementary colour. Absorption of radiation excite
electron to higher energy state. Therefore, when a substance looks coloured, it means that absorption
of visible light and transition of electron must have taken place. Most of the transition metal
compounds are coloured. It arises due to the following reasons :
(i) d-d transition
(ii) Charge transfer

TRANSITION ELEMENTS & COORDINATION COMPOUND


QUIZRR 49

T he d–d transition

The five dăorbitals of a free ion are degenerate i.e. have equal energy. But in the presence of
ligands (H2O, NH3, CNă, Fă etc.) the dăorbitals split. It depends on structure of the complex (Oh,
Td, Sq, Pl etc.).

2 2 2
dx ăy dz
dxy dyz dxz eg
t2

10DqÊ t 10Dq (or0)


Free ion
d -- orbitals
e
dx2ăy2 dz2 t2g
dxy dyz dxz
ML4(Td) ML6(Oh)

Let us consider Ti(H2O)63+. It is an Oh complex. The dăorbitals split as,

2 2 2
dxăy dz
eg

3+ 1
Ti (3d )

t2g
dxy dyz dxz

and single electron occupies t2g level. When light falls on the complex t2g electron is excited to eg
level.
(t12g e0g  t02g e1g i.e.

eg eg
hv
t2g t2g

This excitation takes place when the compound absorbs radiation corresponding to   5000 Å,
(grenish-yellow light). Therefore, transmitted wavelength corresponds violet light and the complex
looks violet. As electron transition takes place from one group of d orbitals to the other, it is called
dăd transition. The dăd transition is shown by transition metal ions of electron configurations, d1,
d2, d3, d4, d6, d7, d8, and d9.

TRANSITION ELEMENTS & COORDINATION COMPOUND


50 QUIZRR
Colour of aqua complexes

Aquo ion dăelectrons Colour

[Sc(H2O)6] 3+ O Colourless
[Ti(H2O)6]3+ d1 Violet
3+ 2
[V(H 2O) 6] d Blue
[V(H 2O) 6]2+ d3 Violet
[Cr(H2O)6]2+ d4 sky-blue
3+ 5
[Fe(H 2O) 6] d Colourless
[Fe(H 2O) 6]2+ d6 Pale-green
[Co(H 2O) 6] 2+ d7 Pink
2+ 8
[Ni(H2O)6] d Green
[Cu(H 2O) 6] 2+ d9 Blue
[Zn(H 2O) 6] 2+ d10 Colourless

Colours due to dăd transition are faint. Metal ions which have d0, d5 and d10 configurations are
colourless, if at all coloured not due to dăd transition.

Colour and charge transfer

Many transition metal ions or compounds such as MnO4ă, CrO4ă2, Cu2O, CdS etc are coloured.
They have their dăorbitals either vacant or fully filled.

Ion or Oxidation state of


dăelectrons Colour
Compound transition metal

MnO4ă Mn +7 3d0 Purple


CrO4ă2 Cr +6
3d 0
Yellow
+ 10
Cu 2O Cd 3d Red
CdS Cd+2 4d10 Yellow

Colour in such compounds is explained by charge transfer mechanism. In MnO4ă, Mn+7 is oxidizing
agent and Oă2 is reducing agent. As oxidizing agent picks up electrons and reducing agent gives
up electrons, electrons moves from Oă2 to Mn+7.

ă2
O

ă2 +7 ă2
O Mn O
e
ă2
O

TRANSITION ELEMENTS & COORDINATION COMPOUND


QUIZRR 51

Due to such movement of electrons the ion is coloured.


The colour of brown ring compound [Fe(H2O)5NO]2+, and Fe3+[Fe2+(CN)6] are also due to charge
transfer.

e e

[Fe2+(H2O)5NO] Fe3+[Fe2+(CN) ]
6
(deep brown) (deep blue)

Colours due to charge transfer are intense (deep).

Hydrolysis of Transition metal compounds

Splitting of water molecules by ions is known as hydrolysis. Cation hydrolysis takes place when
charge density on the cation is high.

Charge
Charge density =
Ionic radius

Transition metal ions on the average are small in size and the common oxidation states are + 2,
+ 3 etc. Therefore, their charge density is high and they under go hydrolysis.

H nă1 +
Mn+ :O [M(OH)] +H
H
Thus, soluble salts of transition metals from slightly acidic solution.
FeCl3 + Water  Solution, slightly acidic.
Water 3+ ă
FeCl3   Fe + 3Cl
H
3+
Fe :O Fe(OH)2+ + H+
H

H
3+
Cr :O Cr(OH)2+ + H+
H

H
2+ 2+ +
Cu :O Cu(OH) +H
H

Due to such hydrolytic reaction, cations, such as Mn+7, Cr+6, etc. are not stable in water

Mn+7 + 4H2O  MnO4ă + 8H+


Cr+6 + 4H2O  CrO4ă2 + 8H+

Covalent transition metal compounds, such as TiCl4, VCl4 etc are hydrolysed due to ability of
transition metal ion to expand coordination number by the use of empty dăorbitals.

TRANSITION ELEMENTS & COORDINATION COMPOUND


52 QUIZRR
Cl H
:O H
VCl4 + H2O V VOCl2 + 2HCl
Cl (green)
Cl
Cl (change transfer)

O RGANOMETALLIC COMPOUNDS

These compounds constitute a broad class of substances in which carbon atom is directly bonded
to a metal. Thus organic compounds in which metal atom is directly linked to carbon atom are
known as organometallic compounds.
For example, NaC  CNa is an organometallic compound as sodium is directly linked to carbon
whereas C2H5ONa, Ti(OC2H5)4 are not organometallic compounds since the metal atom is linked
to carbon through oxygen. Some representative organometallic compounds are
C2H5MgBră Ethyl magnesium bromide
(C2H5)2Zn ă Diethyl zinc
(CH3)2Cd ă Dimethyl cadmium
C6H5Li ă Phenyl lithium
(C2H5) 4Pb ă Tetraethyl lead
Alkyl or aryl magnesium halides (RMgX or ArMgX) are also called as Grignard reagents.

Classification of Organometallic Compounds

(i) Ionic compounds of electropositive metals :


These compounds are mostly formed between the electropositive metals and the carbon compounds
which are mostly acidic in nature. Thus organometallic compounds of alkali metals and alkaline
earth metals consist of ions or ion pairs.
Ră Na+ RăMg2+ Xă
Alkyl sodium Alkyl magnesium
halide

Ră C  Că Na+ Ră Zn2+ Ră
Sodium alkynide dialkyl zinc

These compounds are normally soluble in hydrocarbon solvent. They are very reactive towards air
and water. The stability of these compounds depends upon the structure of the carbon containing
part of the compound.

(ii) ăbonded complexes :


In a ăbonded complex, a metal and a carbon atom of the ligand are joined together with a sigma
bond. This means that the ligand contributes 1 electron and is therefore called one electron donor.
Tetramethyl tin, (CH3)4 Sn and trimethyl aluminium, (CH3)3Al are ăbonded oroganometallic

TRANSITION ELEMENTS & COORDINATION COMPOUND


QUIZRR 53

compounds. (CH3)3Al exists as dimer and has structure analogous to diborane. Two methyl groups
act as bridges between two aluminium atoms.

H3C CH3 CH3


Al Al
H3C CH3 CH3
Trimethyl aluminium

(iii) ăcomplexes :
These are organometallic compounds which involve the use of ăbonds present in organic
compounds. For example, ZeiseÊs salt, ferrocene and dibenzene chromium are organometallic
compounds of this type. In all these compounds the ăelectrons of the organic compound interact
with the metal ion and thus occupy one of the coordination sites. For example in ferrocene and
dibenzene chromium, the iron and chromium atoms are sandwiched between two aromatic rings.

Cl C +
K
Pt H H
Cl Cl
Zeise's salt K [Pt Cl3 (n2 ă C2H 4)]

2+
Fe
Cr

Ferrocene Fe ( 5 ă C5H5)2

[bis cyclopentadienyl iron] dibenzene chromium Cr (6 ă C6H6)2

The number of carbon atoms involved in the formation of ăcomplexes with metals is indicated
by the power of x (eta). For example, ferrocene is represented as Fe[5(C6H6)7] indicating that
5ăcarbon atoms or cyclo pentadienyl anion are involved in ăcomplexation with the metal. Similarly
one can write dibenzene chromium as [Cr(6ăC6H6)2] indicating that all the six carbons of benzene
are involved in ăcomplexation with chromium.

(iv) Metal Carbonyls :


These are the complexes where carbon of carbon monoxide donates a pair of electrons to the
metal. Nickel carbonyl and iron carbonyl are the common examples.

TRANSITION ELEMENTS & COORDINATION COMPOUND


54 QUIZRR
CO CO CO
CO CO CO

M CO Fe Ni

CO CO CO CO
CO CO
[M (CO)6]
CO CO
(M = Cr, Mo, W) [Fe (CO)5] [Ni(CO)4 ]

In metal carbonyl the oxidation state of the metal is zero. These metal carbonyls may be monomeric
bridged or polynuclear.

Bonding in Organometallic Compounds

(a) Bonding in metal carbonyls :


The metal-carbon bond in metal carbonyls has  as well as  character.
(i) ăoverlap
In a sigma bonded complex, the lone pair of electrons is present on the bonding orbital
of carbon monoxide. This bonding orbital containing lone pair interacts with the empty
dăorbital of the metal to form a metal-carbon bond as shown below :

(ii) ăoverlap :
In addition to this, the antibonding orbitals of CO can also overlap with the filled dă
orbitals of the metal resulting in back bonding as explained earlier. Thus metal carbonyls
become much more stable compounds due to multiple bonding in them.

It is important to note here that -bond is in the nodal plane of the -electrons
whereas -overlap is perpendicular to the nodal plane.

TRANSITION ELEMENTS & COORDINATION COMPOUND


QUIZRR 55

(b) Bonding of alkenes to a transition metal


The bonding of alkenes to a transition metal to form complexes has two components. First,
the ăelectron density of the alkene overlaps with a -type vacant orbital on the metal atom.
Second is the back bonding formed by the flow of electron density from a filled dăorbital on
the metal into the vacant xăantibonding molecular orbital on the carbon atom as shown.

C C
M + + +
+ C
C
 ă overlap

C + C ă
M M
ă +
C C

TRANSITION ELEMENTS & COORDINATION COMPOUND


CHEMICAL
KINETICS
2 QUIZRR

1. CH EM I CA L K I N ET I CS
Kinetics is the study of the rates of chemical processes in an effort to understand what it is that
influences these rates and to develop theories which can be used to predict them. A knowledge
of reaction rates has many practical applications, for example in designing an industrial process,
in understanding the intricate interplay of the chemical reactions that are the basis of life.
At a more fundamental level we want to understand what happens to the molecules in a chemical
reactionăthat is what happens in a single reactive encounter between two reagent molecules. By
understanding this we may be able to develop theories that can be used to predict the outcome
and rate of reactions.

2 . RA T ES, RA T E L A WS, RA T E CON ST A N T S A N D ORDER OF REA CT I ON


In this Section we will introduce the language and terms used to describe the rates of chemical
reactions. At this stage we will not be concerned with the theory of reactions or mechanisms, but
just stand back and describe the overall rates.

Concentration c
2.1 Ra t e o f r e a c t io n
The rate is defined as
c
change in concentration, c, in time t t
t
time, t
It is important to specify which species we are talking about. The rate can be positive or negative :
a positive rate means that the concentration is increasing with time e.g. a product; a negative rate
means that the concentration is falling with time e.g. a reactant.
The rate may vary with time (and concentration), so it is usual to define the rate over a very small
time, t. We think of the rate as the derivative of concentration with respect to time.

d  concentration 
rate 
dt
This derivative is the slope of a graph of concentration against time, taken at a particular time.
Symbolically, concentration is often indicated by square brackets. So [A] means the concentration
of species A, and [Br2] means the concentration of bromine, and so on.

Re l a t io n t o s t o i c h i o m e t ri c e q u a t io n s
Consider the reaction whose stoichiometric equation is
O2 + 2H2  2H2O
The stoichiometric equation shows how the number of moles of reactants and products are re-
lated; it must be balanced.
This equation says that to form two moles of water, one mole of oxygen and two moles of hydrogen
must react. It follows that the rate of consumption of hydrogen is twice that of oxygen.

d H2  d  O2 
2
dt dt

CHEMICAL KINETICS
QUIZRR 3

The rate of formation of water is twice the rate of loss of oxygen, as two moles of water are formed
from one mole of oxygen. As water is the product, the rate of change of its concentration is positive
i.e. the concentration is increasing with time; however, the rates of change of the concentrations
of both hydrogen and oxygen are negative as these concentrations are decreasing with time. In
derivative form :

d  H2 O  d O 2  1 d  H2 O  d O 2 
ă 2 or, equivalently  
dt dt 2 dt dt
With all these „rates‰, different for each species, how can we define the „rate of the reaction‰ ?
The usual procedure is to include the stoichiometric coefficients in the definition of the rate. These
coefficients are the numbers in front of the species in the balanced chemical equation. So the
stoichiometric coefficients is 1 for oxygen and 2 for both hydrogen and water.
The rate of reaction, r, is defined for any species A as

1 d A 
r [1]
vA dt

where vA is the stoichiometric coefficient of species A in the balanced chemical equation. In


addition the stoichiometric coefficients are given negative signs for reagents and positive signs for
products. This definition ensures that the rate is always positive and the same for a given reaction
no matter which species is considered.

2.2 Ord e r o f Re a c t io n
The speed of a chemical reaction, in general depends on the concentration of reading species of
the reaction. This is known as law of mass action and is stated as follows :
The ra te of a chemica l rea ction is proportiona l to the product of effective concentra tions
(a ctive ma sses) of the rea ting spec ies, ea ch ra ised to a power tha t is equa l to the corre-
sponding stoichiometric number of the substance a ppea ring in the chemica l rea ction.
Thus, for a general reaction aA + bB  cC + dD ... (2a)
We have
r  [A]a [B]b
or r = k [A]a [B]b ... (2b)
where k is the constant of proportionality.
If the rate of a reaction is determined experimentally, it is found that equation (2b) is not always
applicable. However, the experimental results can be fitted to satisfy a relation of the type of
equation (2b) where the exponents may or may not be equal to the respective stoichiometric
coefficients. In general, we may write the rate as
r = k [A]a´ [B]b´ .... (2c)
where the dimensionless exponents a´ & b´ may or may not be equal to a & b respectively. The
constants a´ & b´ may have positive, negative integral or fractional values or zero value. The
constant a´ is known as the order of the reaction with respect to A, b´ as the order of the reaction
with respect to B and so on. The sum (a´+ b´) is known as the overall order of the reaction (which

CHEMICAL KINETICS
4 QUIZRR

does not have much of significance)). If (a´ + b´) = 1, the reaction is said to be of first-order; if
(a´ + b´) = 2, the reaction is said to be second order and so on. The dependence of reaction rate
on concentration is of great use as it helps in proposing the mechanism of a reaction.
The constant k, which appears in equation (2c) is known as rate constant or more formally, the
specific reaction rate constant, since it is numerically equal to the rate of the reaction provided
all concentrations were set equal to unity. Each reaction is characterized by its own reaction rate
constant. From equation (2c), we find that the units of k is (mol lă1)n să1 where n = 1 ă (a´ + b´).
The expression of equation (2c), which relates the rate of reaction, with the concentrations of
reacting species is known as the differentia l ra te la w.
It ma y be empha sised once a gain here tha t the ra te equa tion with its ra te consta nt a nd
order of various rea cting spec ies is a n experimenta l finding a nd ca nnot be predicted
from the stoichiometry of the ba lanced rea ction.
For example, the solution reaction

N N
+ Bră
Ph Br N N+

PhCh2Br DABCO Ph

is found to have the rate law.


r = k[PhCH2Br] [DABCO]
The reaction is second order overall, and first order with respect to both PhCH2Br and DABCO.
Some reactions have very much more complex rate laws. For example, in the gas phase, the
reaction
H2 + Br2  2HBr
has the rate law
3
ka  H2  Br2  2
r
Br2   kb HBr 
For this reaction, no order with respect to Br2, and no overall order, can be defined.

2.3 Re l a t io n t o e q u i l ib r i u m c o n s t a n t s
{To be read after completing the chapter ÂÂChemical equilibriumÊÊ}
Chemical equilibrium is a dynamic state, not one in which all reaction has stopped. The forward
and back reactions continue just as they do when a system is away from equilibrium, but when
equilibrium has been reached these forward and backward rates are equal. As a result, there is
no change in the concentration of any of the species, even though the reactions are still going on.

CHEMICAL KINETICS
QUIZRR 5

Suppose we have a reaction of the form


A + B  C + D
Also suppose that rate of the forward reaction is kf[A] [B] and that the rate of the back reaction
is kr [C] [D] (these are assumptions, we cannot say from the stoichiometric equation that these
will be the rate laws). At equilibrium these two rates are equal

kf  A eq  Beq  kr Ceq  D eq

where the „eq‰ subscripts have been added to emphasize that this relation is only true at equi-
librium.
Rearranging gives
kf Ceq Deq

kr  A eq Beq
The quantity of the right is the equilibrium constant Keq, so it follows that

kf
K eq 
kr

The equilibrium constant is therefore the ratio of the rate constants of the forward and backward
reactions.

3. Unit s of k :
In general, the rate law for a nth order reaction can be taken as :

dc
 kc n
dt
where k : rate constant & n : order of reaction

dc / dt
 k
cn
units of k  (mol/L)1ăa (time)ă1

(1) For a first order reaction (n = 1)


units of k  (time)ă1
 units are : secă1, mină1, hrsă1 etc.

(2) For a second order reaction (n = 2)


Units of K  (mol/L)ă1 (time)ă1
 units are : L/mol/s.

(3) For (n = 3)
Units of k = L2 molă2 să1

CHEMICAL KINETICS
6 QUIZRR

(4) For a ÂzeroÊ order reaction (n = 0) :


Units of k  mol/L/S
general form
k = (mol/L)1ăn (time)ă1.

4 . SI M PLE RA T E L A WS
A first order rate law is one in which the rate is proportional to the concentration raised to the
power 1 (hence „first‰)
r = k1st[A]1
k1st is called a first order rate constant. The rate equation, and the reaction it describes, is said
to be first order in A or it is said that the order with respect to A is one. As the units of r are
concentration timeă1 and the units of [A] are concentration, the units of k1st are found as

r conc.time1
r  k1st  A 
1
so k1 st    time 1
 
A conc.

Hence first order rate constants have units of timeă1.


A second order reaction has the concentration raised to the power of 2.
r = k2nd[A]2
k2nd is called a second order rate constant. The rate equation, and the reaction it describes, is said to
be second order in A, or the order with respect to A is two. The units of k2nd are found to be conc.ă1
timeă1.

1) N t h Ord e r Re a c t io n
Let the reaction
A  B
th
be n order reaction then
r = k [A]n
where k is constant.
r = rate of reaction

 dA
 k A 
n

dt

 dA
  A n =  kdt
Let at t = 0, [A] = a
and t = t, [A] = a ă x

t
a x  dA
then a  An
  kdt
0

CHEMICAL KINETICS
QUIZRR 7

1  1 1 
     kt
  n  1   a  xn1 an1 
a

1  1 1 
t1 /     tó  1nă1
or 2 k1  n    a / 2  n1 a n1  a
 

1  2n1 
=  n1  1
k n  1  a 

1
Thus t1 / 
2
a 
n 1

Let in two different conditions, initial concentration be a1 & a2.

t    a 
1
/2 1
n 1
2
Then
t   a 
1
/2 2
n 1
1

where subscript 2 & 1 stands for condition 1 & 2.

t /   a
1
2
1
n 1
2

t /   a 
1
2
2
n 1
1

Thus through this equation we can know the half life time at given concentration if its half life
time at some other concentration is given.

2) Fi r st o r d e r r a t e l a w
For reaction of the form A  products, a first order rate law takes the form
r = k1st[A]1
writing the rate in the differential form this becomes

dA
  k1st  A  ........ [7]
dt
we need the minus sign as A is a reactant and so its concentration decreases
with time i.e the slope of a graph of [A] against time will be negative.
The variables [A] and t in Eq. [7] can be separated to give :
1
d[A]   k1st dt
[A]

CHEMICAL KINETICS
8 QUIZRR

which can be integrated [A]


[A]0
1
  A  d  A    k1st dt
ln [A] = ă k1st t + const. t
The constant of integration can be removed by supposing that at Plot of [A] against time
for a first order
t = 0 [A] = [A]0, the initial concentration of A; so 1n[A]0 = const. reaction.[A] falls exponen-
Substituting this gives the final form tially from [A]0 towards
zero.
ln [A] = ă k1st t + ln[A]0 .....[8]
This can also be expressed as
[A] = [A]0 exp (ă k1st t)

Fe a t u r e s o f a Fi r st Ord e r Re a c t io n
1. A first order reaction must follow above form of rate for all time instants.
This means if we are given value of a (initial concentration) and values of x at different time
instants [i.e. (a ă x) as value of reactants after t], the values of k can be calculated for different
time instants by using the above first order law. If the reaction for which the data were given
is a first order reaction, then all values of k will approximately equal to each other.
2. The time for half reaction for a first order reaction is independent of initial concentration of
reactants (i.e. a) and is given as follows :
If t1/2 be the time for half reaction, or the half life then a ă x = a/2

a
 kt1 / 2  2.303 log
a/2

 k t1/2 = 2.303 log 2 = 0.693 Ct

0.693
 t1 / 2 
k

3. The concentration of reactants in a first order reaction


decreases exponentially with time (see figure)
Note that plot of log Ct vs t is linear. It is important to note
that equation of this straight line is of the form : O t
y = mx + b. Comparing it with Ist order rate law as follows :

log Ct C0
kt  2.303 log
Ct
A
 k 
 log Ct    t  log C0 is the equation of line.
 2.303 

 k 
Note that slope of the line =   and Y intercept (OA) = log C0
 2.303 
O t
CHEMICAL KINETICS
QUIZRR 9

Ex a m p l e 1
The half life of first order decomposition of nitramide is 2.1 hour at 15ĈC.
NH2NO2 (aq)  N2O(g) + H2O(l)
If 16.2 gm of NH2NO2 is allowed to decompose, find :
(a) time taken for nitramide to decompose 99%;
(b) volume of dry N2O gas produced at this point at STP.
Solution :
(a) Using first order kinetics, we have :

C0
kt  2.303 log
Ct

0.693 100
  t  2.303 log
2.1 100  99

 t = 13.96 hours.
(b) 16.2 gm of NH2NO2  0.1 mol
& 1 mole NH2NO2  1 mole of N2O
As 99% of NH2NO2 is decomposed
 0.099 mol of NH2NO2 is decomposed
0.099 mol of N2O are produced  22.4 ï 0.099 = 2.217 L of N2O at STP.

A n a l y s is o f s o m e i m p o r t a n t f i rs t o r d e r r e a c t io n s :

1. Decomposition of Hydrogen peroxide (H 2O 2)


2 H2O2  2 H2O + O2
The rate of this first order reaction is measured by titrating a fixed volume of H2O2 (undecomposed)
against a standard solution of KMnO4. Here KMnO4 acts as oxidising agent and H2O2 as reducing
agent. The volumes of KMnO4 used for H2O2 after regular intervals of time are as follows :

Time instants t= 0 t1 t2 t3 t4 t5

Vol. of KMnO4 V0 V1 V2 V3 V4 V5

volume of KMnO4 at t = 0 corresponds to volume of H2O2 initially present


 a  V0
volume of KMnO4 at time instants t1, t2, t3, .... corresponds to volume of
H2O2 remaining after t1, t2, t3, ...
 a – x  Vt
now it being a first order reaction, follows first order kinetics, so

CHEMICAL KINETICS
10 QUIZRR

V0
kt  2.303 log
Vt

Now using the above expression, if we calculate the values of k for different time intervals t1,
t2, ... (for actual numerical data), the values of k should be same if the reaction follows first order
kinetics.

2. Decomposition of Nitrogen Pentoxide


The compond, introgen pentoxide, is a volatile solid which decomposes in the gaseous state as well
as in the from of its soluction in an insert solvent like carbon tetrachloride, chloroform etc.
according to the equation

1
N2O5  N2O 4  2NO2  O2
2

2NO2

When the reaction is Carried out in the solution, N2O4 and NO2 remain in the solution and the
volume of oxygen gas collected is noted at different internals of time It is obvious that

volume of oxygen gas Amount of N2O4



collected at any time (Vt) left decomposed (x)

i.e. substituting these values in the first order equation viz

2.303 a
k log
t ax

2.303 v
we get k= log
t v  vt

The constancy in k proves it first order reaction.

3. H y d r o l y s i s o f e s t e rs (CH 3 COOC 2 H 5 )
CH3COOC2H5) (ester) + H2O + HCl (H+)  CH3COOH + C2H5OH
The reaction rate is measured by titrating the acid (CH3COOH) produced against a standard
alkali solution. Note that when a test sample is prepared from the reacting mixture, there are two
acids : one is mineral acid H+ (HCl or any other) and second is CH3COOH produced. So volume
of alkali used gives titration value for both acids. The data is collected in the following manner.

Time instants t= 0 t1 t2 t3 t4 t

Vol. of NaOH V0 V1 V2 V3 V4 V

CHEMICAL KINETICS
QUIZRR 11

at t = 0. V0 is the voume NaOH used to neutralise the mineral acid present (H+) being used as
catalyst. (at t = 0, no CH3COOH is yet produced)
at t =  (i.e. at the end of hydrolysis), V is the volume of NaOH used to neutralise whole of
CH3COOH plus vol. of HCl present. At t = , volume CH3COOH corresponds to volume of ester
taken initially.
 a  V - V0 (as V0  vol. of HCl)
at t = t1, t2, t3, ...., V1, V2, V3, ...... corresponds to vol. of HCl plus vol. of CH3COOH being produced.
 x  Vt - V0
 a – x  (V – V0) ă (Vt - V0)

V  V0
kt  2.303 log
V  Vt

5. GA S PRESSU RE
The pressure, p, that a gas exerts is related to the volume V, the number of moles n, and the
temperature, T, by the ideal gas equation.
pV = nRT
This can be rearranged to give

n p

V RT
the fraction n/V is a measure of concentration; in SI it would be in moles mă3. The relationship
tells us that at constant temperature pressure is directly proportional to concentration.
In a reaction we will generally have a mixture of gases, and so we can talk about the partial
pressure, p, of each gas i. The partial pressure of a gas is the pressure that the gas would exert
if it occupied the volume on its own. DaltonÊs Law states that the total pressure, PT, is the sum
of the partial pressures :
PT = P1 + P2 + P3 + .....
Each partial pressure follows an ideal gas law

ni p
 i
V RT
where ni is the number of moles of gas i. Therefore the partial pressure of species i is a measure
of its concentration.
In a reaction involving gases the only measurable pressure is the total pressure exerted by the
system. However, to make kinetic measurements we need to know the concentration, that is the
partial pressure, of the individual species. We need, therefore, to find some way of relating these
two. An example of how this is done is considered here.
Consider the reaction between NO and O2 in the gas phase
2NO + O2  2NO2

CHEMICAL KINETICS
12 QUIZRR

Overall three moves of gas go to two, so as the reaction proceeds there is a measurable decreases
in the total pressure. The total pressure is the sum of the partial pressures

PT = PNO + PNO + PO
2 2

There are three unknown quantities on the right, and only one measurable quantity, the total
pressure; to solve this we are going to have to introduce some extra restrictions.
Suppose that at time zero there is no NO2 present, and that the NO and O2 are mixed in a
2 : 1 ratio, just as the stoichiometry of the reaction. If the initial partial pressure of O2 is PO ,0
2
then the initial partial pressure of NO is 2pO ,0 and so the initial total pressure, p0, is 3PO ,0. This
2 2
is the situation set out on line 1 of the Table

pO pNO pNO pT
2 2
1 pO 2pO 0 3pO  p0
2,0 2,0 2,0
2 pO ă pO 2pO ă 2pO 2pO 3pO ă pO
2,0 2 2,0 2 2 2,0 2
1 2 p ă 2p
3 p ă pO 2pO p0 ă pO
3 O 2 3
O O2 2 2

4 pT ă 2 pO
3

Suppose after time t the partial pressure of O2 has fallen by pO2; due to the stoichiometry of the
reaction the partial pressure of NO2 will fall by 2pO and the partial pressure of NO2 will
2
increase by 2pO2. This is the situation set out on line 2 of the Table.
Our aim now is to find pO2 in terms of measurable quantities. There are two steps. In going from
line 2 to line 3 in the Table we have used the fact that p0 = 3pO2,0 to substitute for pO2,0. From
line 3 we then see that pT = p0 ă pO from which it follows that pO = p0 ă pT. This value of
2 2
pO is used in going from line 3 to 4.
2
So, finally, on line 4 we have an expression for pO in terms of measurable quantities :
2

pO = pT ă 2 p0.
2 3

Ex a m p l e 2
The gas phase decomposition
2N2O5  4NO2 + O2
follows the first order rate law. At a given temperature the rate constant of the reaction
is 7.5 ï 10ă3 să1. The initial pressure of N2O5 is 0.1 atm.

CHEMICAL KINETICS
QUIZRR 13

(i) Calculate the time of decomposition of N2O5 so that the total pressure becomes 0.15
atm.
(ii) What will be the total pressure after 150 seconds?
Solution :
(i) The decomposition reaction is represented by
2N2O5 (g)  4NO2 (g) + O2 (g)
initial p0 0 0
After time t p0 ă 2x 4x x
Total pressure = p0 ă 2x + 4x + x + x = p0 + 3x = 0.15 atm
or, 3x = 0.15 ă 0.10 = 0.05

0.05
or, x  0.0166 atm
3

Pressure of N2O5 after 1 seconds,


p = p0 ă 2 ï 0.0166 = (0.1 ă 0.0332) = 0.067 atm
Now applying first order rate law

2.303 p 2.303 0.1


t log10 0  3
log10  53.4 seconds
k p 7.5  10 0.067

(ii) After 150 seconds,

2.303 0.1
150  log 10
7.5  10 3 p

0.1 150  7.5  103


or, log10   0.4885
p 2.303

0.1
p  0.03247
3.0696

p0 ă 2x = p = 0.03247
2x = (0.1 ă 0.03247)
= 0.06753
x = 0.033765
Total pressure = p0 + 3x = 0.1 + 0.101295
= 0.201 atm

CHEMICAL KINETICS
14 QUIZRR

Ex a m p l e 3
At a certain temperature the half change period for the catalytic decomposition of ammonia
were found as follows :
Pressure (Pascals) : 6667 13333 26666
Half life period in hours : 3.52 1.92 1.0
Calculate the order of reaction.
Solution :

t 
1
/2 1 a 
 2
n 1

t 
1
/2 2
 a1 
where n is order of reaction

From the given data,

n1
3.52  13333 
 (a  initial pressure)
1.92  6667 

= (2)nă1

3.52
log   n  1  log 2  0.3010   n  1
1.92

0.2632 = 0.3010 ï (n ă 1)
n = 1.87
= 2

Ex a m p l e 4
Some PH3(g) is introduced into a flask at 600ĈC containing an inert gas. PH3 proceeds to
decompose into P4(g) and H2(g) and the reaction goes to completion. Total pressure is given
below as a function of time. Find the order of the reaction and calculate the rate constant,
Time (sec) 0 60 120 
Pressure (mm Hg) 262.40 272.90 275.51 276.40
Solution :
4PH3 (g) + Inert gas  P4 (g) + 6H2 (g) + inert gas
t= 0 P Pi 0 0 0
t= t (P ă x) Pi x/4 6x/4 Pi
t =  0 Pi p/4 6p/4 Pi

CHEMICAL KINETICS
QUIZRR 15

At t = 0, 262.40 = P + Pi ...(1)
t = 60 sec 272.90 = PPH3(left) + PP4(g) + PH2(g) + Pi
t = 120 sec 275.51 = PPH3(left) + PP4(g) + PH2(g) + Pi
t=  276.40 = PPH4(left) + PH2(g) + Pi

P 6P ...(2)
276.40    Pi
4 4
276.40 ï 4 = 7P + 4Pi ... (3)
On solving equations (2) and (3), we get
P = 18.66 mm, Pi = 243.74 mm
At 60ĈC

x 6x
272.90   P  x  Pi  
4 4

7x
272.90 = 18.66 ă x + 243.74 +
4
x = 14 mm

2.303  a  2.303 18.66


k log   log
t  a  x  60 18.66  14

k = 2.32 ï 10ă2 sec


Similarly at 120 sec. k = 2.30 ï 10ă2 secă1.
Since, value of ÂkÊ are same, hence the reaction belongs to first order.

6. I N V ERSI ON OF CA N E SU GA R
The inversion of cane-sugar (sucrose) catalysed with dil HCl.

H
C12H12O11 + H2O   C6H12O6 + C6H12O6
The progress of the reaction is followed by noting the optical rotation of the reaction mixture with
the help of polarimeter at different time intervals and the change in rotation is proportional to
the amount of sugar decomposed.
Let ro, rt, r be the angle of rotation during zero, t and infinite time respectively.
Then ro ă r  a; rt ă r  a ă x

2.303  r  r 
 k log  o 
t  rt  r 

CHEMICAL KINETICS
16 QUIZRR

Ex a m p l e 5
Cane sugar is gradually converted into dextrose and laevulose by dilute acid. The rate of
inversion is observed measuring the polarization angle, at various times, when the following
results are obtained :
Time (sec) 0 10 20 30 40 100 
Angle 32.4 28.8 25.5 22.4 19.6 ă 6.1 ă 14.1
Show that the reaction is of first order. Calculate the value of t, when the solution is
optically inactive.
Solution :
In case the inversion of cane sugar is a first order change, then

2.303  A 0 2.303 r  r
k log10  log 10 0
t  
A t rt  r

(i) When t = 10, r0 = 32.4, rt = 28.8, r = 14.1

2.303 32.4    14.1 


k log10
10 28.8    14.1 

2.303 46.5
= log10
10 42.9
= 0.008 mină1
(ii) When t = 20, r0 = 32.4, rt = 25.5, r = ă 14.1

2.303 32.4    14.1 


k log10
20 25.5    14.1

2.303 46.5
= log10
20 39.6
= 0.008 mină1.
(iii) When t = 30, r0 = 32.4, rt = 22.4, r = ă 14.1

2.303 32.4    14.1


k log 10
30 22.4    14.1

2.303 46.5
= log10
30 36.5
= 0.008 mină1
Thus, the reaction is of first order as the value of k is constant.
The solution will be optically inactive when half of the cane sugar is inverted.
0.693 0.693
t1 /    86.6 min
2 k 0.008
CHEMICAL KINETICS
QUIZRR 17

Ps e u d o Fi r st Ord e r Re a c t io n
There are circumstances where a second order reaction might appear, in an experiment, to be first
order. That is when one of the reactants in the rate equation is present in great excess over the
other in the reaction mixture.
As an example, letÊs look again at the mixed second order rate law that we treated in the last
section.

d  B
We had the rate law,   k B  A  , with a reaction whose stoichiometry is, 2A + B  P + etc
dt

(P = product)
Sometimes [A]0 > > [B]0.
For example, say we prepare the system such that,
[A]0 = 2.0 M and
[B]0 = 0.001 M
Then, when the reaction has run to completion, (at time = infinity),
[A]inf = [A]0 ă 2 [B]0 = 2.0 ă 0.0001 = 1.998  2.0
So [A] is approximately constant throughout the entire reaction.
Then the rate law becomes,

d  B
  k B  A    k B A 0 ...(1)
dt

which is effectively a first order rate reaction. This rate equation has the solution,

 k A 0
B  B 0 e ...(2)

where the „effective‰ first order rate constant is k[A]0.


You can actually solve this problem as a mixed second order rate problem, as we did in the last
section, but it is lot more work.
Pseudo first order reactions are sometimes used to find the rate constant of a second order
reaction when one of your two components is very expensive and the other one is relatively cheap.
You can use an excess of the inexpensive reagent and use a small amount of the expensive one.
Notice that you can still obtain the second order rate constant by dividing the effective first order
rate constant by the concentration of the excess component.

CHEMICAL KINETICS
18 QUIZRR

Co m p a r in g r a t e c o n s t a n t s
As the half life of a first order reaction is independent of concentration it is easy simply to look
at the rate constant and get an immediate feel for how fast it is. For example, the reaction
2N2O5  4NO2 + O2
has a first order rate constant of about 2000 să1 at 400 K; the half life is therefore 0.3 ms, which
makes the reaction quite fast.
On the other hand the second order reaction
2NOBr  2NO + Br2 (gas phase)
has a rate constant at 280 K of 0.8 molă1 dm3 să1. Is this „faster‰ or „slower‰ than the first order
reaction above ? To answer this we need to know the concentration of the reactants. Suppose that
the pressure of NOBr is 0.1 atmospheres, corresponding to a concentration of 4.4 ï 10ă3 mol dmă
3
; using the above expression we find that the half life is 280 s ă so, under these conditions, the
reaction is much slower than the decomposition of N2O5.
The key point to understand here is that we cannot compare directly the rates of reactions of
different orders by comparing their rate constants; concentration has to be taken into account.

7. EL EM EN T A RY REA CT I ON A N D I T S M OL ECU L A RI T Y
Most chemical reactions proceed through a series of elementary reactions. An elementary reaction
is one that is proposed as taking place in a single step. These elementary steps are classified
according to the number of molecules, which they involve. A process in which only one molecule
is involved is known as unimolecular process. One involving two molecules is called bimolecular
and so on. It is well known that in elementary reactions, the products are formed when the
reactant molecules come close and collide together. Since the collisions in which more than three
particles come together simultaneously are very rare, the elementary process with molecularity
greater than three are not known.
If a reaction involves more than one step, the overall reaction is obtained by adding these elementary
steps. In such a case, it is wrong to decide the molecularity of the overall reaction on the basis
of its stoichiometry and rate law equation.
It was stated earlier that the order of a reaction, in general, cannot be predicted from the
stoichiometry of the overall reaction. However, the order of an elementary step can be predicted
from its molecularity. In fact, the order of an elementary step is always equal to its molecularity.
This follows from the following analysis.

7.1 U n i m o l e c u l a r Ele m e n t a r y Pro c e s s


In this process, a single reactant gives a product, A  B. This is achieved when A molecules
collide to form an activated complex (an activated complex is the one whose energy is quite high)
by transferring energies. The activated complex further decomposes independentally.

A + A  A* (activated complex of A)
A  A* (effective reaction)

CHEMICAL KINETICS
QUIZRR 19

It is obvious that the number of activated complexes produced would depend on the number or
concentration of A molecules.
A*  B

dA
r  k A 
dt
Hence, the unimolecular elementary process is necessarily a first order reaction.
1. O3  O2 + 0
2. N2O5  N2O + 2O2

7.2 B i m o l e c u l a r Ele m e n t a r y Pro c e s s


Let the elementary process be represented as
A + B  products
Now, for a molecule of A to react with a molecule of B it should come close to A and collide with
it. The rate at which collisions between A & B molecules occur is directly proportional to the
concentrations of A and B. Thus, the rate of the reaction will be given as
r = k [A] [B]
which is the expression of the second-order process. Hence, a bimolecular elementary process is
necessary a second-order reaction.
The above arguments can be extended to a trimolecular reaction. Hence a trimolecular elementary
process will follow third order rate law.
1. 2 HI  H2 + I2
2. NO + O3  NO2 + O2

7.3 T ri m o le c u l a r
2 NO + O2  2 NO2

Note (1)
For a reaction : A  B in the rate law : r = k [A]m [B]n
Neither the order of reaction (m + n) nor the molecularity of a reaction can be predicted from
stoichiometric coefficient of a balanced reaction. The order of reaction is always to be determined
experimentally and molecularity is determined theoretically after studying the reaction mechanism.
However as a theoretical idea sometime, we can have an approximate order of reaction equal to
molecularity (i.e., the number of molecules taking part in slowest elementary step for complex
reactions).

Note (2)
Order of a reaction can be fraction also. For example consider the following reaction :
(i) H2 + Br2  2 HBr
rate = k [H2] [Br2]ó (determined experimentally)
order of reaction = 1 + ó= 3/2

CHEMICAL KINETICS
20 QUIZRR

(ii) CH3CHO (g)  CH4 + CO


rate = k [CH3CHO]3/2 (determined experimentally)
order of reaction = 3/2
Also note that sum of stoichiometric coefficient (1 + 1 = 2) is not equal to the order of reaction.
Order and Molecularity of a Reaction : A Distinction
Order wrt
S.N. Chemical Equation Mole Rate Law First Second Overall
cularity Reactant Reactant

 dx 
 dt   k A   B
a b
1. aA + bB  Product a + b a b a +b
 

 dx 
 dt   k A   B 
2 0
2. aA + bB  Product a + b 2 Zero, if B is in excess 2
 

 dx 
3. P4 
2H2O2   2H2O  O2 2  dt   k  H2O2  1* ă 1
 
(Bimolecular)

 dx 
4. CH3COOC2H5 + H2O 2  dt   kCH3 COOC2 H5  1* Zero, if H2O is in excess 1
 
 (Bimolecular)
 CH3 COOH +
H

C2H5OH

 dx 
5. H
C12H22O11 + H2O   2  dt   k C12 H22O11  1* Zero, if H2Ois in excess 1
 
sucrose

C6H12O6 + C6H12O6
glucose (fructose)
 dx 
6. (CH3)3 CăCl + OHă  2    k  CH3 3 C-Cl  1 Zero, OHă does not take 1
 dt 
(Bimolecular)
(CH3)3 + OHă  part in slow step
ă
CH3OH + Cl

 dx 
 dt   kCH3 Cl OH 

7. CH3Cl + OHă  2 1 1 2
 
(Bimolelcular)
CH3OH + Clă

 dx 
8. 
C6H5N2Cl   1  dt   k  C6 H5 N 2Cl  1 ă 1
 
(Unimolecular)
C6H5Cl + N2

CHEMICAL KINETICS
QUIZRR 21

S.N. Chemical Equation Mole Rate Law First Second Overall


cularity Reactant Reactant

 dx  3

  k CH3CHO
/2
9. 
CH3CHO   1  1.5 ă 1.5
 dt 
(Unimolecular)
CH4 + CO

 dx 
 dt   k H2O 2  I 

10. H2O2 + 2Iă + 2H+ 5 1 1 (H+ is medium) 2
 
 2H2O + I2

* Pseudo ă unimolecular reactions

8. ST U DY I N G DI FFEREN T T Y PES OF REA CT I ON S


8.1 Pa r a l le l (o r Sim u l t a n e o u s ) Re a c t io n s
Rea ctions in which two products a re formed from same set of rea cta nt(s) by different
pa ths a re ca lled pa ra llel or simulta neous. For example, ăDăglucose and ăDăglucose are
formed in solution from the aldehydic form of Dăglucose following different paths as :

ăDăglucose
Aldehydic form of Dăglucose

ăDăglucose
k1 B
A
or
k2 C

The various differential rate laws are

d A
  k1  A   k2  A    k1  k2   A  .... (a)
dt

d B
 k1  A  ..... (b)
dt

d  C
 k2  A  ...... (c)
dt
Now, let us assume that at t = 0, only A was present. After a very small time, let us say, 2 second,
x mole/l of B and y mole/l of C was formed.

d  B x d C y
Then  and 
dt 2 dt 2

CHEMICAL KINETICS
22 QUIZRR

d B 
dt  x
d C y
dt

From equation (b) and (c)

d  B
dt  k1
d  C k2
dt

x k1
 
y k2
This means that at any given instant of time, the ratio of amount of B formed to that of C formed
till that instant is a constant.

Ex a m p l e 6 k1 B

k 1 = a hr ă1; k 1 : k 2 = 1 : 10 A
k2
C C
Calculate
 A  after one hour from the start of the reaction, assuming only A was present in
the beginning.
Solution :

 d A
  k1  k2   A 
dt
Separating the variables give

 d A
  k1  k2  dt
A
Integrating equation within limits,

d A t
  k1  k2   dt
At
 A  A 
0
0

 A0   k  k t
ln  
At  1 2

B  1
k1 = a hră1 and k2 = 10 a hră1
C 10 , since
CHEMICAL KINETICS
QUIZRR 23

We can write, [A0] = [At] + [B] + [C]

 A t    B   C  k  k t
ln  1 2
 At 

[C]
1  [C]   B 1 
ln 10  11a   ; k1  a hr 1 , k2  10 a hr 1 , t  1 hr 
[A t ]   C 10 

 11C 
ln  1  10 A   11a
  t
The equation can be written as

C  10 e11a
 A t  11
 1 

8.2 Se q u e n t ia l r e a c t io n s
Suppose that we have two sequential reactions which for simplicity we will assume are both first
order.

k1
A  B [1]
k
B 
2

 C [2]

We can imagine two extreme (limiting) cases :


(1) Rate constant of process [1] is very much greater than that of process [2] :
k1 >> k2
In this case A i converted rapidly into B, leading to an accumulation of B which is more
slowly converted into C. The rate at which the product C is formed depends on the rate of
process [2] ă it is said that step [2] is the rate determining step.
(2) Rate constant of process [2] is very much greater than that of process [1].
k2 >> k1.
In this case A is converted slowly into B, and then B reacts to form C almost as soon as B
is formed. The rate at which the product C is formed depends on the rate of process [1] ă step
[1] is now the rate determining step. In this limit there is very little B present at any time
as the moment it is produced it reacts.
If either of these limiting cases applies then the kinetics is greatly simplified, as the overall
rate of production of the product depends only on the step with the smallest rate constant.
The same is true for more complex schemes of sequential reactions.
k1 k2 k4k
A  B  
 C 
3
 D  
E

CHEMICAL KINETICS
24 QUIZRR

If the step from B to C has the smallest rate constant, then the rate of formation of the product
E depends only on k2.
k1 k2
A  B  
C
The differential rate equations are

 d A
Rate of disappearance of A,  k1  A  ...... (a)
dt

d B
Rate of change of B,  k1  A   k2  B  ........ (b)
dt

d C
Rate of formation of C,  k2  B  ........ (c)
dt
Integrating equation (a) within the limits we get,
t
[A]t = [A]0 eăk, ....... (d)
In order to get the value of B at any instant of time, we need to integrate equation (b). Since this
is a differentia equation, we integrate it in the following manner.
First bringing k2[B] to the left side of the equation

d  B
 k2 [B]  k1  A  ........ (e)
dt
From equation (8e), we get

d B
 k2 [B]  k1  A 0 e k1t
dt
Integration of this equation is not possible as we are unable to separate the two variables, [B] and
t. Thus, the equation is to multiplied on both sides by an integrating factor ek2t.

 d  B 
 k2  B   k1  A 0 e 2 1 
k k t
ek2t 
 dt 

d B
 k2  B  ek2t  k1  A 0 e
k2  k1 t
ek2t ..... (f)
dt

Left hand side of the equation (f) is equal to


d
dt
 B e  k2t
(a differential of [B]ek2t).

d
dt
 B  e   k  A 
k2 t
1 0
e
k2  k1  t

 
d  B ek2t  k1  A 0 e
k2  k1 
dt ..... (g)

CHEMICAL KINETICS
QUIZRR 25

Integrating equation (g) within the limits, 0 and t,

 B ek2t t

  
d  B  ek2t  k1  A 0  e k2  k1  t dt
0 0

t
 e k2  k1  t  k1
 B e
k2 t
 k1  A 0     A 0  e k2  k1 t  1
k
 2  k 1  0  2
k  k1

k1
 B   A  e k2t  e k2  k1 t  1
 k2  k1  0

k
 B   A 0  e k1t  e k2t 
 
 k2  k1  ..... (8h)

In general, the concentration of A decreases exponentially, the concentration of B initially increases


up to a maximum and then decreases thereafter, and the concentration of C increases steadily.

Time when [B] becomes maximum

d B
 k1 [A]  k2 [B]  0
dt
k1 [A] = k2 [B]
Substituting equation (8d) and equation (8h), we get

k2 k1
k1 [A]0 e k1t  [A]  e k1t  e k2t 
 k2  k1  0  

Rearranging, we get

 k2  k1   e k1t  e  k2t 
   k1 t

k2 e

k1  k  k t
1 1 e 1 2
k2

1 k
tmax  ln 1
 k1  k2  k2 ....(8i)

Maximum concentration of B
k1
[B]  [A]0  e k1t  e  k2 t 
 2 1
k  k  

CHEMICAL KINETICS
26 QUIZRR

k1   k1  k2 k1 
 B  A   k  k  1n 
 k2  k1  0 k  k 
 e 1 2  e 1 2 k2 

  k  k1 k 
k2

k1  ln  k2   k2  k2  ln 2   k1  k2  
 B  A  e  1  k
 e  1 
 k2  k1  0  
 

 k1 k2 
k1  k2   k1  k2   k2   k1  k2  
[B]max  [A]  k     
 k2  k1  0  1   k1  
.... (j)

In order to get the concentration of [C], we substitute [B] equation (8h) to (8c) and then integrate
within the limits.

d  C k2 k1
 k2  B   A 0  e k1t  e  k2t 
dt  k2  k1 

k2 k1
d C   A   e  k1t  e  k2t  dt
 k2  k1  0 

Upon integrating within the limits, 0 to t,

 C t
k2 k1
 d  C   A 0   e k1t  e k2t  dt
0  k2  k1  0

t
k k  e  k1t e  k2t 
C  2 1  A 0   
 k2  k1    k1  k2  0

k2 k1  e k1t  1   e k2t  1  
C  A      
 k2  k1  0   k1    k2  

k2 k1  1  e k1t   1  e  k2t 
C  A     
 k2  k1  0  k1   k2 

 A 0
C 
 k2   
 k2 1  e k1t  k1 1  e  k2t 
 k1     ....... (k)

CHEMICAL KINETICS
QUIZRR 27

9. RA DI OA CT I V I T Y
All radioactive decay follows first order kinetics. The only difference from chemical reactions is
that temperature does not influence the rate of radioactive decay.
In radioactive decay the term used for identifying the reaction rate is the number of nuclei rather
than concentration.
A  B

 d NA
  NA .... (a)
dt

 is similar to k in chemical reactions and is known as deca y or disintegra tion consta nt and
NA is the number of nuclei of A at a given instant of time.

N0
ln   t; Nt  N0 e t .... (b)
Nt

0.693
tó 

A term called average life time is introduced here which is given by 1/.

1
 tav 

Average lifetime means the life time of a single isolated nuclei. If NA = 1, and this nuclei decays

 dN A
in one second, then according to the equation (11a), 
dt

If a nucleus of A decays after 2 seconds, then

 dNA 1
   . The average lifetime of the nucleus is therefore 1/.
dt 2

1 tó
tav    1.44 tó
 0.693

Ac tivity of a ra dioactive element is the ra te a t which it deca ys a nd is given by  N.


A = N
Since according to equation (11b)
N t = N0 eăt
Multiplying by , we get
Nt = N0 eăt
At = A0 eăt,

CHEMICAL KINETICS
28 QUIZRR

 dNA  dNA
 q  A  q  N ( and A will not be equal)
dt dt

Specific a ctivity is the a ctivity of a unit ma ss of a ra dioactive substance.

Units of radioactivity
1 curie = 3.7 ï 1010 disintegrations per second(dps) = 3.7 ï 1010 Bq.
1 Becquerel = 1 disintegrations per second(dps)

Amount of radioactive substance left after n half-lives :


Let us start with the amount of radioactive substance in the beginning as A0.

A0
Then, amount of radioactive substance after first half-life =
21

A0
Amount of radioactive substance after second half-life =
22

A0
And amount of radioactive substance after n half-life =
2n

Ex a m p l e 7
The mean lives of a radioactive substance are 1620 years and 405 years for -emission and
-emission respectively. Find out the time during which three fourth of a sample will decay
if it is decaying both by -emission and -emission respectively.
Solution :
For successive , -emissions,

1 1  5  1
average        yr
1620 405  1620 

Following first-order kinetics,

2.303 N
 log 0
t Nt

2.303  1620 N0
t log  449.2 years
5 
N0 / 4  .

CHEMICAL KINETICS
QUIZRR 29

T h e r e a c t io n c o o r d i n a t e a n d t h e r e a c t io n p r o f i l e
The minimum energy pathway from reactants to products is called the reaction coordinate. Moving
along this pathway involves changing the positions of all of the atoms involved in the reaction in
a complex way, so the „coordinate‰ is in fact a composite of many motions.
We can plot the energy against the reaction coordinate to give a plot which is know as the reaction
profile, which you have encountered before.

potential energy
transition state

Ea

products

reactants

reaction coordinate

The transition state exists at the potential energy maxima, and the difference in energy between
this maxima and the reactants is called the activation energy E.
The fraction of molecules coming together with sufficient energy to reach the transition state is
exp(ăEa/RT). For a typical value of the activation energy of 50 kJ molă1, this means that only
about 1 in 109 of the molecules which attempt to reach the transition state actually have enough
energy to make it. We can imagine the molecules all queuing up like eager climbers at Everest
base camp (the reactants); many set off towards the summit (the transition state), but few have
the energy to make it, and so slide back down the slope.
As we commented before, actually calculating the form of the potential energy surface is a
challenging task. However, this discussion has allowed us to develop an understanding of the
origin of the energy barrier to reaction, and to introduce the important concepts of the transition
state and the reaction profile.

T h r e s h o l d En e r g y
It is the minimum amount of energy at which reaction takes place.

A c t iv a t io n e n e r g y (E a )
The energy given to the reactant to reach the threshold value of energy is called as activation
energy is shown in the graph below.

CHEMICAL KINETICS
30 QUIZRR
Threshold Energy
Threshold Energy

Eaf
Eaf Eab Eab

Potential Products
Hr = EafăEab Potential Reactants
Energy Energy
Reactants Products
Reaction Progress Reaction Progress
Endothermic Reaction
Exothermic Reaction

where
Eaf = activation energy for forward reaction
Eab = activation energy for backward reaction
Hr = Enthalpy of reaction = Eaf ă Eab

1 0. T H E COL L ISI ON T H EORY OF REA CT I ON RA T ES


We are going to look in detail at reactions which involve a collision between two species.

Species : This is a useful term covers any sort of particle you like-molecule, ion or free radical.

Reactions where a single species falls apart in some way are slightly simpler because you wonÊt
be involved in worrying about the orientation of collisions. Reactions involving collisions between
more than two species are going to be extremely uncommon.

1 0 .1 Re a c t io n s i n v o l v in g c o l li s io n s b e t w e e n t w o s p e c i e s
It is pretty that if you have a situation involving two species they can only react together if they
come into contact with each other. They first have a collide, and then they may react.
Why „may react‰? It isnÊt enough for the two species to collide-they have to collide the right way
around, and they have to collide with enough energy for bonds to break.
(The chances of all this happening if your reaction needed a collision involving more than 2
particles are remote. All three (or more) particles would have to arrive at exactly the same point
in space at the same time, with everything lined up exactly right, and having enough energy to
react. ThatÊs not likely to happen very often!)

1 0 .1 .1 T h e o r i e n t a t io n o f c o l li s io n

Consider a simple reaction involving a collision between two molecules-ethene, CH2=CH2, and
hydrogen chloride, HCl, for example. These react to give chloroethane.

CH2 = CH2 + HCl  CH3CH2Cl

CHEMICAL KINETICS
QUIZRR 31

As a result of the collision between the two molecules, the double bond between the two carbons
is converted into a single bond. A hydrogen atom gets attached to one of the carbons and a
chlorine atom to the other.
The reaction can only happen if the hydrogen end of the H-Cl bond approaches the carbon-carbon
double end. Any other collision between the two molecules doesnÊt work. The two simply bounce
off each other.

collision 1 collision 2

collision 3 collision 4

Of the collisions shown in the diagram, only collision 1 may possibly lead on to a reaction.
You may be wondering why collision 2 wonÊt work as well. The double bond has a high concentration
of negative charge around it due to the electrons in the bonds. The approaching chlorine atom is
also slightly negative because it is more eletronegative than hydrogen. The repulsion simply
causes the molecules to bounce off each other.
In any collision involving unsymmetrical species-you would expect that the way they hit each
other will be important in deciding whether or not a reaction happens.

1 0 .2 T h e e n e r g y o f t h e c o l li s io n
Ac tivation Energy
Even if the species are orientated properly, you still wonÊt get a reaction unless the particles
collide with a certain minimum energy called the a ctivation energy of the reaction.

CHEMICAL KINETICS
32 QUIZRR

Activation energy is the minimum energy


required before a reaction can occur. You can Energy
show this on an ener gy pr ofile for the
reaction. For a simple over-all exothermic Activation
reaction, the energy profile looks like this : Energy EA

Note : The only difference if the reaction was


Reactants
endothermic would be the relative positions of
the reactants and prod ucts lines. For an
endothermic change, the products would have a
Products
higher energy than the reactants, and so the
green arrow would be pointing upwards. It
makes no difference to the discussion about Progress of reaction
the activation energy.

If the particles collide with less energy than the activation energy, nothing important happens.
They bounce apart. You can think of the activation energy as a barrier to the reaction. Only those
collisions which have energies equal to or greater than the activation energy result in a reaction.
Any chemical reaction results in the breaking of some bonds (needing energy) and the making of
new ones (releasing energy). Obviously some bonds have to be broken before new ones can be
made. Activation energy is involved in breaking some of the original bonds.
Where collisions are relatively gentle, there isnÊt enough energy available to start the bond-
breaking process, and so the particles donÊt react.

1 1 . RA T E CON ST A N T S A N D T H E A RRH EN I U S EQU A T I ON


The Arrhenius equation
Rate constants and rate equations
You will remember that the rate equation for a reaction between two substances A and B looks
like this :
order of reaction order of reaction
with respect to A with respect to B

rate = a b
k[A] [B]

rate in rate constant concentrations


mol dm-3s-1 in mol dm-3

The rate equation shows the effect of changing the concentrations of the reactants on the rate of
the reaction. What about all the other things (like temperature and catalysts, for example) which
also change rates of reaction? Where does these fit into this equation?

CHEMICAL KINETICS
QUIZRR 33

These are all included in the so-called rate constant-which is only actually constant if all you are
changing is the concentration of the reactants. If you change the temperature or the catalyst, for
example, the rate constant changes.
This is shown mathematically in the Arrhenius equation.
activation energy
rate constant

E
ă A
k = Ae RT kelvin temperature

the gas consant


frequency factor
or
pre-exponential factor mathematical
quantity,e

Wha t the various symbols mea n


Starting with the easy ones...
Temperature, T
To fit into the equation, this has to be measured in kelvin.
The gas constant, R
This is a constant which comes from an equation, pV = nRT, which relates the pressure, volume
and temperature of a particular number of moles of gas. It turns up in all sort of unlikely places!
Activation energy, EA
This is the minimum energy needed for the reaction to occur. To fit this into the equation, it has
to be expressed in joules per mole-not in kJ molă1.
And then the rather trickier ones...
e
This has a value of 2.71828... and is a mathematical number, a bit like pi and is called exponent.
–(EA/ RT)
The expression, e
For reasons that are beyond the scope of any course at this level, this expressions counts the
fraction of the molecules present in a gas which have energies equal to or in excess of activation
energy at a particular temperature.
The frequency factor, A
You may also find this called the pre-exponential factor or the steric factor.
A is a term which includes factors like the frequency of collisions and their orientation. It varies
slightly with temperature, although not much. It is often taken as constant across small temperature
ranges.
You may also come across it in a different form created by a mathematical operation on the
standard one:

EA
ln k  ln A 
RT

CHEMICAL KINETICS
34 QUIZRR

1 2 . T H E EFFECT OF T EM PERA T U RE ON REA CT I ON RA T ES

1 2 .1 T h e f a c t s
What happens?
As you increase the temperature the rate of reaction increases. As a rough approximation, for
many reactions happening at around room temperature, the rate of reaction doubles for every
10ĈC rise in temperature.
You have to be careful not to take this too literally. It doesnÊt apply to all reactions. Even where
it is approximately true, it may be that the rate doubles every 9ĈC or 11ĈC or whatever. The number
of degrees needed to double the rate will also change gradually as the temperature increases.

1 2 .2 T h e e x p l a n a t i o n
1 2 .2 .1 I n c r e a s i n g t h e c o l li s io n f r e q u e n c y
Particles can only react when they collide. If you heat a substance, the particles move faster and
so collide more frequently. That will speed up the rate of reaction.
That seems a fairly straighforward explanation until you look at the numbers.
It turns out that the frequency of two-particle collisions in gases is proportional to the square root
of the kelvin temperature. If you increase the temperature from 293 K to 303 K (20ĈC to 30 &
deg. C), you will increase the collision frequency by a factor of :

303
 1.017
293
ThatÊs an increase of 1.7% for a 10Ĉ rise. The rate of reaction will probably have doubled for that
increase in temperature-in other words, an increase of about 100%. The effect of increasing
collision frequency on the rate of the reaction is very minor. The important effect is quite different.

For Example
What happens if you increase the temperature by 10ĈC from, say, 20ĈC to 30ĈC (293 K to 303 K)?
The frequency factor, A, in the equation is approximately constant for such a small temperature
change. We need to look at how e ă (EA/RT) changes - the fraction of molecules with energies equal
to or in excess of the activation energy.
LetÊs assume an activation energy of 50 kJ molă1. In the equation, we have to write that as 50000
J molă1. The value of the gas constant, R, is 8.31 J Kă1 molă1.
At 20ĈC (293 K) the value of the fraction is
E 50000
 A 
8.31  293
e RT e
= 1.21 ï 10ă9
By raising the temperature just a little bit (to 303 K), this increases :
E 50000
 A 
8.31  303
e RT e
= 2.38 ï 10ă9

CHEMICAL KINETICS
QUIZRR 35

You can see that the fraction of the molecules able to react has almost doubled by increasing the
temperature by 10ĈC. That causes the rate of reaction to almost double. This is the value in the
rule-of-thumb often used in simple rate of reaction work.
Note : This approximation (about the rate of a reaction doubling for a 10 degree rise in
temperature) only works for reactions with activation energies of about 50 kJ molă1 fairly close
to room temperature.
The rate constant goes on increasing as the temperature goes up, but the rate of increase falls
off quite rapidly at higher temperatures.

1 3 . T H E EFFECT S OF CA T A L Y ST S ON REA CT I ON RA T ES
1 3 .1 T h e f a c t s
What are catalysts?
A catalyst is a substance which speeds up a reaction, but is chemically unchanged at the end of
the reaction. When the reaction has finished, you would have exactly the same mass of catalyst
as you had at the beginning.
Some examples
Some common examples which you may need for other parts of your syllabus include :
reaction catalyst
Decomposition of hydrogen peroxide manganese (IV)
oxide, MnO2
Nitration of benzene concentrated sulphuric acid
Manufacture of ammonia by the Haber Process iron
Conversion of SO2 into SO3 during the Contact vanadium (V) oxide,
Process to make sulphuric acid V2O5

Hydrogenation of a C=C double bond nickel

1 3 .2 Ca t a l ys t s a n d a c t iv a t io n e n e r g y
To increase the rate of a reaction you need
to increase the number of successful Energy Activation Energy
collisions. One possible way of doing this without catalyst
is to provide an alternative way for the
reaction to happen which has a lower Activation Energy
activation energy. with catalyst
Adding a catalyst has exactly this effect Reactants
on activation energy. A catalyst provides
an alternative route for the reaction.
That alternative route has a lower Products
activation energy. Showing this on an
energy profile :
Progress of reaction
CHEMICAL KINETICS
36 QUIZRR

A w o rd o f c a u t i o n !
Be very careful if you are asked about this in an exam. The correct form of word is
„A catalyst provides an alternative route for the reaction with a lower activation energy.‰
It does not „lower the activation energy of the reaction‰. There is a subtle difference between the
two statements.
If particles collide with enough energy they can still react in exactly the same way as if the
catalyst wasnÊt there. It is simply that the majority of particles will react via the easier catalysed
route.

Ex a m p l e 8
If for a first order reaction, rate constant varies with temperature
according to the graph given. At 27ĈC, 1.5 ï 10ă4 per cent of the

Rate constant
reactant molecules are able to cross-over the potential barrier. At
52ĈC, the slope of this graph is equal to 0.2 kă1 secă1, calculate the
value of rate constant at 52ĈC, assuming that activation energy does
not change in this temperature range.
Solution : Temp. (K)

1 RT
1.5  10 4   e Ea /
100
= 1.5 ï 10ă6 = e ă Ea/R ï 300

 Ea = 33.43 kJ moleă1
ă Ea/RT
k= e

dk E
 a2 k
dT RT

8.314  325  325


k  0.2 
33.43  1000

k = 8.75 ï 10ă2 mină1.

Ex a m p l e 9
1 mole of gas changes lineraly from initial state (2 atm, 10 lt) to final state (8 atm, 4lt). Find
the value of rate constant, at the maximum temperature, that the gas can attain. Maximum
rate constant is equal to 20 secă1 and value of activation energy is 40 kJ moleă1, assuming
that activation energy does not change in this temperature range.
Solution :
Let the equation of straight line is
P = mV + c
Now putting the values, we get
P + V = 12

CHEMICAL KINETICS
QUIZRR 37

From the ideal gas equation,

PV PV
T 
nR R

T
12  V  V
R
For T to be maximum

dT
0
dV
V = 6 lt
P = 6 atm

36
Value of Tmax =  439 K
0.082
Now putting the values of
A = 20 secă1
Ea = 40 ï 103 J moleă1
We get
ăEa/RT
k= A e

40  103
8.314  439
k  20 e

k = 1.56 ï 10ă3 secă1.

Ex a m p l e 1 0
From the following data, show that the decomposition of hydrogen peroxide in aqueous
solution is a first order reaction. What is the value of the rate constant?

Time in minutes 0 10 20 30 40

Volume (V) in ml 25.0 20.0 15.7 12.5 9.6

where V is the number of ml of potassium permanganate required to decompose a


definite volume of hydrogen peroxide solution.
Solution :
The equation for a first order reaction is

2.303 a
k log
t ax

The volume of KMnO4 used, evidently corresponds to the undecomposed hydrogen peroxide.

CHEMICAL KINETICS
38 QUIZRR

Hence, the volume of KMnO4 used at zero time corresponds to the initial concentration ÂaÊ and the
volume used after time t corresponds to (a ă x) at that time. Inserting these values in the above
equation, we get

2.303 25
when t = 10 min., k = log = 0.022318 mină1 = 0.000373 să1
10 20.0

2.303 25
when t = 20 min., k = log = 0.023265 mină1 = 0.0003871 să1
20 15.7

2.303 25
when t = 30 min., k = log = 0.02311 mină1 = 0.000385 să1
30 12.5

2.303 25
when t = 40 min., k = log = 0.023932 mină1 = 0.0003983 să1
40 9.6

The constant value of k shows that the decomposition of H2O2 in aqueous solution is a first order
reaction. The average value of the rate constant is 0.0003879 să1.

Ex a m p l e 1 1
Two reactions (I) A  Products (II) B  Products follow first order kinetics. The rate of the
reaction (I) is doubled when temperature is raised from 300 K to 310 K. The half life for this
reaction at 310 K is 30 minute. At the same temperature B decomposes twice as fast as A.
If the energy of activation for the reaction for the reaction (II) is half that of reaction (I),
calculate the rate constant of reaction (II) at 300 K. (IIT 1992)
Solution :
(I) A  Products
(II) B  Products
 tó for (I) at 310 K = 30 minute

0.693
 K  I  at 310   0.0231 min 1 ... (1)
30
 rate = K[ ] and both reactions are of I order

K I at 310
Also given 2 ... (2)
K I at 300

K II at 310
Also given 2 ... (3)
K I at 310

E aII 1
Also we have,  ... (4)
E aI 2

CHEMICAL KINETICS
QUIZRR 39

K I at 310 EaI  310  300 


For I 2.303 log10    ... (5)
K I at 300 R  310  300 

K II at 310 EaII  310  300 


For II 2.303 log10    ... (6)
K II at 300 R  310  300 

Dividing Eqs. (5) by (6)

KI at 310
log10
KI at 300 E
 aI  2
 K at 310 EaII By eq. (4) ... (7)
log10 II
K II at 300

K I at 310  K II at 310 
or log 10  2 log10  
K I at 300  K II at 300 

2
K I at 310  K II at 310 
or log 10   ... (8)
K I at 300  K II at 300 
By Eqs. (2) and (8),

2
 K II at 310 
or   2
 K II at 300 

By Eqs. (3) and (9),

2 ï KI at 310 K = 2 (KII at 300 K)

2  K I at 310
KII at 300 K = ...(10)
2

By Eqs. (1) and (10),

KII at 300 K = 2  0.0231

KII at 300 K = 3.27 ï 10ă2 mină1


= 0.0327 mină1.

Ex a m p l e 1 2
In the Arrhenius equation for a certain reaction, the values of A and Ea (energy of activation)
are 4 ï 1013 secă1 an 98.6 kJ molă1 respectively. If the reaction is of first order, at what
temperature will its half life period be 10 minute ?

CHEMICAL KINETICS
40 QUIZRR

Solution :
Given,
K= ? A = 4 ï 1013 secă1
Ea = 98.6 ï 103 J molă1 tó = 10 ï 60 sec
ăEa/RT
 K = Ae
ï 103)/8.314 ï T
 K = 4 ï 1013 eă(98.6

0.693 0.693
Now for I order reaction, K   sec 1
tó 600

0.693 3
  4  1013 e(98.6  10 ) / 8.314  T
600

0.693 98.6  103


or log e  log e 4  1013 
600 8.314  T

0.693 98.6  103


or log10  log10 4  1013 
600 8.314  2.303  T

T = 311.35 K

Ex a m p l e 1 3
The half life period and initial concentration for a reaction are as follows. What is order
of reaction?
Initial concentration 350 540 158
tó 425 275 941
Solution :
tó (a)1ăn
For I  425  (350)1ăn ...(1)
1ăn
For II  275  (540) ...(2)
1ăn
For III  941  (158) ...(3)
By Eqs. (1) and (2),

1 n
425  350 

275  540 

Taking log,

425 350
log  1  n  log
275 540
n = 2

CHEMICAL KINETICS
QUIZRR 41

Ex a m p l e 1 4

d [A]
For the reaction A  B, the rate law expression is   K[A]ó . If initial concentration
dt
of A is A0. Calculate :
(a) integrated form of the rate expression.
(b) nature of plot of [A]ó vs. time.
(c) half life period.
Solution :

d[A]
(a)   K[A]ó
dt

dA
or   Kdt
[A]ó

on integrating   [A]ó d A  K  dt

ă 2Aó = Kt + C ...(1)
at t = 0; A = A0  C = ă 2A0ó ...(2)
By Eqs. (2) and (1),
ă 2Aó = Kt ă 2A0ó
or Kt = 2[A 0ó ă Aó] ...(3)

(b) Plot of Aó vs. t will be linear because Eq. (3) yields,


2Aó = ă Kt + 2A0ó

K
or Aó =  t  Aó
0
2

K
or Aó   t  A 0ó (similar to y = mx + C)
2
(c) If t = tó A = A0/2
 By Eq. (3)

 A  
ó
 1 
Któ  2  A 0ó   0    2A 0ó 1  
  2    2

Któ 
2[ 2  1]
2
A 0ó  2  
2 1 Aó
0

 tó 
2  2 1 A ó
0
K

CHEMICAL KINETICS
42 QUIZRR

Ex a m p l e 1 5 :
The rate law for the following reaction :

dx
Ester + H+  Acid + Alcohol, is  K (ester) [H3O+]0
dt
What would be the effect on rate if :
(a) Concentration of ester is doubled ?
(b) Concentration of H+ ion is doubled ?
Solution :
 Rate = K [ester]´ [H3O+]0
(a) r1 = K [ester]´ [H3O+]0
Let, Initial conc. of ester = a, initial conc. of H3O+ = b
 r1 = K [a] [b]0
If conc. of ester is doubled, i.e., [ester] = 2a, then
r2 = K [2 ï a]´ [b]0

r1 1
  or r2 = 2r1
r2 2

(b) r1 = K [a] [b]0


If conc. of H3O+ is doubled, i.e., [H3+O] = 2b
r3 = [a]t [2b]0

r1
  1 or r = r
r3 1 3

Ex a m p l e 1 6
The rate of a certain reaction depends on concentration according to the equation

dC k1C
  , what will be the order of reaction, where (a) concentration is very-very
dt 1 + k2 C
high, (b) very-very low.
Solution :

dC k1C k1
  
dt 1 + k2C 1
 k2
C
(a) If C is very very high 1/C is smallest and thus negligible.
d C k1
   Constant
dt k2
Thus, order of reaction is zero.

CHEMICAL KINETICS
QUIZRR 43

(b) If C is very very low 1 + k2C = k´

d C k1C
   k''C
dt k'
Thus, order of reaction is unity.

Ex a m p l e 1 7
An organic compound A decomposes following two parallel first order mechanism :
k1
B k1 1
A  and k 1 = 1.3 ï 10ă5 secă1.
k2 k2 9
C
Calculate the concentration ratio of C to A, if an experiment is allowed to start with only
A for one hour.
Solution :

k1 1

k2 9

But k1 = 1.3 ï 10ă5 secă1; k2 = 9 ï 1.3 ï 10ă5 secă1 = 11.7 ï 10ă5 secă1.
(k1 + k2) = (1.3 ï 10ă5) + (11.7 ï 10ă5) secă1 = 13 ï 10ă5 secă1 ...(1)

[B]t 1 [C]t
Also,   [B]t  ...(2)
[C]t 9 9

For parallel first order reaction :

d[A]
   k1  k2  [A]
dt

[A]t tt
d[A]
     k1  k2   dt
[A]0
[A] t0

[A]0   A t  [B]t  [C]t 


 ln   k1  k2  t; ln     k1  k2  t
[A]t  [A]t 

 Ct 
 [ A]t    Ct 
 ln  9    k1  k2  t [from eq. (2)]
  A t 
 
 

 10 Ct 
 ln 1     k1  k2  t
 9  A t 

CHEMICAL KINETICS
44 QUIZRR

 10  Ct  5
 ln 1    13  10  60  60  0.468 [from eq. (1)]
 9  A  
t 

10 Ct Ct
 1  1.5968;  0.537
9  A t [A]t

Ex a m p l e 1 8
The rate of formation of NO(g) in the reaction :
2NOBr(g)  2NO(g) + Br2(g)
was reported as 1.6 ï 10ă4 Msă1 (M = mol Lă1). What is the rate reaction and the rate of
consumption of NOBr ?
Solution :

dx 1 d  NOBr  1  d  NO d  Br 
Taking the fact    
dt 2 dt 2 dt dt
d  NO
given  1.6  10 4 Ms1
dt

dx 1 d  NO 1.6  10 4
hence rate of reaction   Ms1  8  105 Ms1
dt 2 dt 2
d  NOBr  d  NO 
and rate of consumption of NOBr    1.6  104 Ms1
dt dt

Ex a m p l e 1 9
The rate constant for the decomposition of a certain substance is
2.80 ï 10ă3 M să1 at 30ĈC and 1.38 ï 10ă2 Mă1 să1 at 50ĈC. Evaluate the Arrhenius parameters
of the reaction.
(R = 8.314 ï 10ă3 kJ molă1 Kă1)
Solution : Energy of activation Ea and are pre-exponential factor A are Arrhenius parameters
given rate constant k1 = 2.80 ï 10ă3 Mă1 să1 at 30ĈC (303 K)
k2 = 1.38 ï 10ă2 Mă1 să1 at 50ĈC (323 K)

2.303 RT1 T2 k
thus energy of activation E a  log10 2
 2 1
T  T k1

2.303  8.314  103  303  323  1.38  102 


=
 323  303 
log 10  2.80  103 
 
Ea = 64.91 kJ molă1
ă Ea/RT Ea/RT
also k = Ae  A = ke
ï 10ă3 ï 303)
A = 2.80 ï 10ă3 e64.91/(8.314
= 4.34 ï 108 Mă1 să1
CHEMICAL KINETICS
QUIZRR 45

Ex a m p l e 2 0
The acid catalysed hydrolysis of an organic compound A at 30ĈC has a time for half change
of 100 min when carried out in a buffer solution at pH 5, and 10 min when carried out at
pH 4. Both times of half-change are independent of the initial concentration of A. If the rate
 d [A]
constant K is given by  k [ A ]a [H  ]b , what are the values of a and b ?
dt
Solution :

 d A  b
The rate equation is  k [A]a H  
dt
During any experiment, pH is constant, hence

 d A
 K [A]a where K [H  ]b
dt
Since half life is independent of the initial concentration of A, hence a = 1; Consequently k´ is a
first order rate constant and is given by K´ = 0.693/T50, Therefore

b b
k2 ´ k H  2 H  2
 
 T50 1
  
 T50 2 k1 ´ k H  b H   b
 1  1

b  
 100   104   1 
 pH  log 
or    or b = 1,
 10   105   H   
 

Therefore the rate of hydrolysis has a first order dependence on [H+], so that

 d [A]
 k A  [H  ]
dt

Ex a m p l e 2 1
A reaction between substances A and B is represented stoichiometrically by A + B  C.
Observations on the rate of this reaction are obtained in three separate experiment as follows :
Initial concentrations t/h duration of Final
S.No. [A]0, M [B]0, M experiment concentration
[A]f , M
(1) 0.1000 1.0 0.50 0.0975
(2) 0.1000 2.0 0.50 0.900
(3) 0.0500 1.0 2.00 0.0450

What is the order with respect to each reactant and what is the value of the rate
constant ?

CHEMICAL KINETICS
46 QUIZRR

Solution :
  [A]
Let rate law be  k [A]a [B]b where a and b are order w.r.t. A and B respectively. First
t
  [A]
we calculate rate of disappearance of A,
t
Set  (1) (2) (3)
A ă 0.0025 ă 0.0100 ă 0.0050
t 0.50 0.50 2.00

 [A] 
   0.0050 0.0200 0.0025
 t 

[A]0 0.1000 0.1000 0.0500


[B]0 1.0 2.0 1.0

Putting these values in rate law, we have


+ 0.0050 = k [0.1000]a [1.0]b ...(1)
0.0200 = k [0.1000]a [2.0]b ...(2)
0.0025 = k [0.500]a [1.0]b ... (3)
b
on dividing eq. (2) by (1), 4 = 2 which gives b = 2
on dividing eq. (1) by (3), 2 = 2a which gives a = 1

  [A]
thus rate law is = k [A] [B]2
t
if we put the above values of any one set, say (1) rate law

[A]
t 0.0050
k   5  102 M2 h 1
[A] [B]2
 0.1000  1.0  2

Ex a m p l e 2 2
A certain reaction A + B  products is first order w.r.t. each reactant with
k = 5.0 ï 10ă3 Mă1 să1.
Calculate the concentration of A remaining after 100 s if the initial concentration of A was
0.1 M and that of B was 6.0 M. State any approximation made in obtaining your result.
Solution :
Since [B] = 6.0 M, we can consider that its concentration remains constant throughout. Thus
Rate = k [A] [B]
= k [A] 6 = 6k [A]
= k´ [A]

CHEMICAL KINETICS
QUIZRR 47

where k´ (= 6k) is new constant and reaction under this condition is supposed to follow first order
kinetics.

2.303 a
k'  log
t  a  x

a K 't 6 kt
 log  
 a  x  2.303 2.303

a 6  5.0  103  100


log 
 a  x 2.303

 a 
log    1.3026
 a  x

 a 
   20
 a  x

ax 1

a 20

a 0.1
(a ă x) =   0.005M
20 20
Hence concentration of the reactant A after 100s is 0.005 M.

CHEMICAL KINETICS
ELECTROCHEMISTRY
QUIZRR 3

ELECTROCHEMISTRY

1. INTRODUCTION
Electrochemistry is a branch of chemistry, which deals the relationship between electrical energy
and chemical changes taking place in redox reactions i.e., how chemical energy or how electrical
energy can be used to bring about a redox reaction which is otherwise not spontaneous. It has
many applications in electrolysis, energy producing cell etc. A flow of electricity through a substance
may produce a chemical reaction, it involves study of electrolysis and conductance. While a
chemical reaction causes flow of electricity through external circuit involves the measurement of
electromotive force.

2. ELECTROLYSIS
Electrolysis is a process in which chemical reactions occur at the electrodes, dipping in the electrolytes,
when voltage is applied across them. The electrode that is charged positively is called anode and
the electrode that is charged negatively is called cathode.
Let us two suitable metal electrodes, which are inserted in the electrolyte solution of MCl. The rods
are now connected to a source of electromotive force (E.M.F.) As soon as the electrodes are
connected to battery, the cations start moving towards cathode, take up electrons from the rod and
get reduced to deposit as metal while anions (Clă) move towards anode and get oxidised to release
Cl2 gas. Thus, at anode, electrons are released at the rod and go into the battery and anode
becomes positive electrode. The electrons from the battery enter cathode, making it negatively
charged. Thus, flow of electrons takes place from anode the cathode outside the cell and inside
the cell, electrons indirectly flow from cathode to anode.
The electrode at which oxidation takes place is called anode and the electrode at which reduction
takes place is called cathode.
The reactions at the two electrodes are :
At cathode : M+ + eă  M
At anode : Clă  óCl2 + eă
+ ă
eă eă

Anode (+) Cathode (ă)


Cl
M
Cl

Thus, electrolysis is a process of chemical decomposition of the electrolyte by the passage of electric
current. It is carried out in a cell called electrolytic cell.

ELECTROCHEMISTRY
4 QUIZRR
3. ELECTROLYTIC CONDUCTANCE

3.1 Conductors

Any substance, which allows the electric current to pass through it, is called an electrical conductor.
There are two types of conductors.

1. Metallic conductors
Substances, which allow the passage of electricity through them without undergoing any
chemical change.
Example : Cu, Ag, Al etc.

2. Electrolytic conductors
Substances that allow the passage of electricity through their molten state or aqueous
solutions and undergo chemical decomposition.

Difference between metallic and electrolytic conduction

Metallic Conduction Electrolytic Conduction


Passage of charge by electrons Passage of charge by ions in molten
and aqueous state.

Passage of charge brings about Brings about physical and chemical


only physical changes. Does not changes. Involves transfer of
involve transfer of matter. matter in the form of ions

Resistance increases with Resistance decreases with


temperature because of obstacles temperature because viscosity
of vibrating kernels decreases

Conducting power is high Conducting power is low

3.2 Types of Electrolytes

There are three types of electrolytes : Strong, weak and non electrolytes.

1. Strong electrolytes
A strong electrolyte is one which undergoes complete ionization when dissolved in water.
The solution contains only the ions and not molecules.

Examples : HCl, HNO3, H2SO4, NaOH, Ca(OH)2, NaCl, KCl, CH3COONa etc.

ELECTROCHEMISTRY
QUIZRR 5

HCl   H+ + Clă
NaCl   Na+ + Clă

2. Weak electrolytes
A weak electrolyte is one which undergoes partial ionization or dissociation. Here, in solution
the ions and the dissociated molecules will be in equilibrium with each other. When such a
solution is diluted, the degree of ionization increases. It becomes complete at infinite dilution.
Examples :
HCOOH, CH3COOH, NH4OH, CH3NH2, CH3COONH4, H3PO4 etc.

ă +
CH3COOH 

 CH3 COO + H

3. Non-electrolytes
A substance which doesnÊt allow the electric current to pass through is called a non electrolyte.
These substances have no ions. Therefore they do not ionize in water (Covalent compounds).
Examples :
C6H6, toluene, sugar, urea, CH4, C2H6 etc.

3 .3 Factors Influencing Electrolytic Conduction

1. The inter-ionic attraction


It is the intersection between the ions of the solute at low concentrations. It is now much
at low concentrations the inter-ionic cone is very less, but at high concentrations it is
appreciable.

+ ă

ă ă + +
ăă ă ++ +
ă +

The negative ion is pulled to the negative pole because it is surrounded by positive charges
and vice versa.

ELECTROCHEMISTRY
6 QUIZRR
2. The solvation of ions
This interaction is between the ions of the solute and the molecules of the solvent. Larger
the interaction, greater is the solvation, lower will be the mobility of ions.

3. Viscosity of the medium


Interaction between the solvent molecules. The solvent-solvent interaction also lowers the
mobility. More the viscosity less is the mobility.

4. Temperature
Ionic mobility increases with increase in temperature, thus the conductance increases.
Increase in temperature also decreases solute-solute, solute-solvent and solvent-solvent
interaction.

4. Conductance, Specific Conductance, Equivalent Conductance and Molar Conductance

The reciprocal of resistance offered by an electrolyte to the flow of electricity through it is known
as conductance. The unit of conductance is ohmă1 or mho.

1
Conductance =
Resistance

Consider the portion of the electrolyte lying between the two parallel and
equal electrodes as shown in the diagram; the area of each electrode being
ÂaÊ sq. cm and the distance between them is ÂlÊ cm. Since the electrolytes like a a
metallic conductors, also offer resistance to the flow of electricity, OhmÊs law L
can be applied to the electrolytic conductor.
The OhmÊs law relates the resistance ÂRÊ offered by a medium with the applied voltage ÂVÊ and the
passing current ÂIÊ.
V = IR
The resistance ÂRÊ is related to the dimensions of the conductors as :
R  l

1
and R
a

l l
 R ;R
a a

where R = resistance,  = specific resistance

1 1 l
or  
 R a

ELECTROCHEMISTRY
QUIZRR 7

 Specific conductance = Conductance  l


a

1 1
Just as is called conductance,  is called specific conductance denoted by ÂKÊ.
R

The solution under study is filled in a conductivity cell made of pyrex glass having two platinum
electrodes fixed parallel to each other.

l
For a given cell, is known as cell constant. Thus :
a

Specific conductance = Conductance ï Cell constant


Note : The unit of specific conductance (K) is ohmă1 cmă1 or S cmă1.

Molar Conductance ( ^ )

The molar conductance is defined as the conductance of all the ions produced by the ionisation
of 1 gm mole of an electrolyte when present in V ml of solution. It is denoted by  m

Molar conductance ( m )  K  V
where V is the volume in ml containing 1 gm mole of the electrolyte. If C is the concentration of
the solution in g mole per litre, then :

1000
K
c

Its units are ohmă1 cm2 molă1 or S cm2 molă1.

Equivalent Conductance ( eq. )

One of the factors on which the conductance of an electrolytic solution depends is the concentration
of the solution. In order to obtain comparable results for different electrolytes, it is necessary take
equivalent conductance.
Equivalent conductance is defined as the conductance of all the ions produced by one gram

equivalent of an electrolyte in a given solution. It is denoted by  eq .

At concentration C (in gm-equivalent/L i.e., normality), equivalent conductance.

 eq 
1000 specific conductance
c  or Normality 

ohm 1 cm 1 
1000  K
=
N

ELECTROCHEMISTRY
8 QUIZRR
4 .1 Equivalent Conductance at infinite Dilution

Equivalent conductance increases with the increase in dilution but after a limit it becomes constant
and does not further increase. The maximum value of equivalent conductance is known as
equivalent conductance at  dilution, denoted by  . If this increase in equivalent conductance
with dilution is only due to the increase in the degree of dissociation of the electrolyte, we can
write,

c
degree of dissociation  =


4.2 Variation of Molar Conductivity With Concentration

 m of electrolytes increases with dilution.

The variation is different for strong and weak electrolytes.

Strong Electrolytes

It is given according to the equation  cm   m  b c (Debye Huckel onsager equation)

 cm and 
m are the molar conductance at a given concentration and at infinite dilution

(respectively). b is a constant depending on the viscosity of the solvent. The graph shows that  m
decreases as the concentration increases.

c
m
<
(Ohmă1cm2molă1)

c (mol litreă1)ó

This is because at higher concentration there is greater inter ionic attraction which retards the
motion of the ions as conductance falls  cm and 
m is that conductance at infinite dilution where
the ions are far apart and there is no inter-ionic attraction. This can be obtained by extrapolation
of the graph to zero concentration.

ELECTROCHEMISTRY
QUIZRR 9

W eak Electrolytes

A weak electrolyte dissociates to a much lesser extent so its conductance is lower than that of a
strong electrolyte at the same concentration.

c
<

m
CH3COOH

c (mol litreă1)ó

The very large increase at infinite dilution is because the ionization increases and so the number
of ions in solution increases.

The value of  cm and 


m cannot be obtained by extrapolation as can be seen on the graph. It

is obtained by applying KohlrauschÊs law.

 m values for strong electrolytes is larger than weak electrolytes for the same concentration.

Increase  m for strong electrolyte is quite small as compared to that for weak electrolyte.

5. Relationship Between Molar and Equivalent Conductivities

k
According to the definition, m = C ...(i)
m

k
and  eq = C ...(ii)
eq

For a solution containing a certain mass of solute per unit volume of the solution (let us say ÂwÊ
gram per litre), we can write

w w
Cm  and Ceq 
Molar mass of the electrolyte Equivalent mass of the electrolyte

Cm Equivalent mass of the electrolyte


From these relations, we can write C  Molar mass of the electrolyte
eq

ELECTROCHEMISTRY
10 QUIZRR
We know that molar mass of an electrolyte = z ï Equivalent mass of electrolyte where z is the
number of equivalents of electrolytic charge per mole of the electrolyte. For example, n-factor of
electrolyte).

Cm 1
  ...(iii)
Ceq z

Cm  eq
Using equation (i) and (ii), we get C   ...(iv)
eq m

Then from equation (iii) and (iv), we get

Cm 1
 or  m  z   eq
Ceq z

where z can have values equal to 1, 2, 3, ...

Example 1

When a certain conductivity cell was filled with 0.020 M KCl which has a specific conductivity
of 0.2768 ohmă1 mă1, it had a resistance of 82.40  at 25ĈC. With 0.0025 M K2SO4, it has a
resistance of 326.0 . Calculate
(a) the cell constant
(b) conductivity of K2SO4 solution
(c) equivalent conductance and molar conductance of K2SO4 solution
Solution :
(a) Calculation has been done using 0.020 M KCl solution.

cell constant
specific conductance =
resistance (R)

 cell constant = specific conductance ï resistance


= 0.2768 ï 82.4
= 22.81 mă1
(b) thus specific conductance of K2SO4 solution = 22.81/326
= 0.07 ohmă1 mă1

1000  specific conductance


(c) equivalent conductance c =
normality

from (b) specific conductance = 0.07 ohmă1 mă1

ELECTROCHEMISTRY
QUIZRR 11

= 0.0007 ohmă1 cmă1


normality = 0.0025 ï 2 = 0.005 N

1000  0.0007
c =
0.005

= 140 ohmă1 cm2 equiv.ă1

1000  specific conductance


molar conductance c =
molarity

1000  0.0007
=
0.0025

= 280 ohmă1 cm2 molă1

Example 2

The resistance of a decinormal solution of a salt occupying a volume between two platinum
electrodes 1.80 cm apart and 5.4 cm2 in area was found to be 50 ohm. Calculate the equivalent
conductance of the solution.
Solution :

1000  conductance  cell constant


We know equivalent conductance  c = normality

1 1
given, conductance =  ohm 1
resistance 50

l distance
cell constant = 
a area

1.80 1 1
= = cm
5.40 3

normality = 0.1 N

1 1
1000  
 c =  50 3
0.1

= 66.66 ohmă1 cm2 equiv.ă1

ELECTROCHEMISTRY
12 QUIZRR
6. KOHLRAUSCH’S LAW
At infinite dilution an ionic specie (cation or anion) contributes a fixed value, at a given temperature,
towards equivalent conductance of the electrolyte irrespective of the other ionic species in
combination with it. There contributions are known as equivalent ionic conductance at  dilution

defined by 0c and 0a for cation and anion respectively.

Mathematically,

    y  
m for AxBy = x A  B

 
where  x  B and y B are the major conductance of A+ and Bă at infinite dilution x and y

are the ions provided by one formula unit of the electrolyte.


For example :

      
(1) m for NaCl = Na Cl

(2)    2   2  
m for BaCl2 = Ba Cl

(3) 
2  3  3 2
m for Al2 (SO4)3 = Al SO
4

Applications of Kohlrausch’s Law

Sometimes, the molar conductivity values for the ions are not available. In such cases, following
procedure is adopted.
(i) Select a series of strong electrolytes such that the sum/difference of molar conductivities of
their ions gives the molar conductivities of the ions of weak electrolyte.

(ii) Measure 
m values of these salts (strong electrolytes) at various concentrations (Cm) and

plot  m against Cm for each salt separately. Determine 


m for each salt (strong electrolyte)

by extrapolation method.

(iii) Add and/or subtract the equations to get the 


m of the weak electrolyte. Let us determine

the molar conductivity of a weak electrolyte, MA at infinite dilution. For this purpose, we

take three salts MCl, NaA and NaCl and determine their 
m values by extrapolation

method.

ELECTROCHEMISTRY
QUIZRR 13

Calculation of molar conductivity at infinite dilution (AĈ) for weak electrolytes (CH3COOH)


(CH3COOH)  

  
CH3COO  H

The value of  for KCl, CH3COOK and HCl can be obtained by extrapolation (they are strong
electrolytes).

  KCl   K
 
  Cl

  CH3 COOK     
 K
CH3COO 

  HCl         
H Cl

    
CH3 COOH  CH3COOK   HCl   KCl

Example 3

The value of   2 ă1
m for HCl, NaCl and CH3CO2Na are 426.1, 126.5 and 91 S cm mol respectively.

Calculate the value of  


m for acetic acid.

Solution :
Using KohlrauschÊs law

  
HCl       ...(i)
H Cl


CH3CO2Na  

  
Na ...(ii)
CH3CO2

  
and  NaCl   Na    Cl ...(iii)

Adding equations (i) and (ii) and subtracting equation (iii), we get


HCl  

       
NaCl =  H  Cl  CH CO    Na    Na   Cl 
CH3CO2Na  3 2

  
=  H  CH3CO2  CH3CO2H


 CH3CO2H
= (426.1 + 9.1 ă 126.5) = 390.6 S cm2 molă1

ELECTROCHEMISTRY
14 QUIZRR
Example 4

The equivalent conductance of silver nitrate solution at 250ĈC for an infinite dilution was
found to be 133.3 ohmă1 cm2 equiv.ă1. The transport number of Ag+ ions in very dilute

solution of AgNO3 is 0.464. Calculate equivalent conductance of Ag+ and NO3 at infinite
dilution.
Solution :

Given 0(Ag +) = transport number of Ag+ ï 0

Given 0(Ag +) = nAg  ï 0 (AgNO3)

= 61.9 ohmă1 cm2 equivă1


By KohlrauschÊs law


 
0 (AgNO3) = 0 Ag  0 NO3   
 
0 NO3  = 0  AgNO3   0 Ag 


= 133.3 ă 61.9
= 71.4 ohmă1 cm2 equivă1

7. APPLICATIONS OF CONDUCTANCE MEASUREMENT


Ć Conductance is measured based on the „Wheatstone Bridge‰ principle.
Ć Conductivity cell is calibrated using saturated, 1N or 0.01 N KCl solution. Their specific
conductances at a given temperature are constant values. If their conductances are known
then,

specific conductance
cell constant 
conductance

Ć Generally all measurement are made using conductivity water. If conductivity water is not
available then
True conductance = [exptl value ă conductance of water]
Ć Conductance of conductivity water is taken as zero.

7.1 Degree of Ionisation and Ionisation Constant :

If  c = equivalent conductance at concentration C gm equivalent Lă1

0 = equivalent conductance at infinite dilution

ELECTROCHEMISTRY
QUIZRR 15

c
then x=
0

For weak acid, using OswaldÊs dilution law


+ ă
HA 

 H + A

Cx2
Ka (ionisation constant of a weak acid) =
1  x 
Similarly Kb (ionisation constant of a weak base) can be calculated.

7.2 Solubility Product of a sparingly Soluble Salt :

If solute is sparingly soluble in a given solvent, its concentration is taken as its solubility in the
saturated solution. Also Lt C  0  c  0

1000  sp. conductance


Thus  c  0 
C

0 can be computed by use of KohlrauschÊs law

Thus C (which is also the solubility of the sparingly soluble salt) and hence Ksp is known.
Solute Ksp
AgCl Ksp = [Ag+] [Clă] = S2
PbI2 Ksp = [Pb2+] [Iă]2 = 4S3
Al(OH)3 Ksp = [Al3+] [OHă]3 = 27S4
AxB y Ksp = xxyy (S)x+y
S is the solubility in mol Lă1

Example 5
For a saturated solution of AgCl at 25ĈC, specific conductance is 3.41 ï 10ă6 ohmă1 cmă1 and
that of water used for preparing the solution was 1.6 ï 10ă6 ohmă1 cmă1. What is the solubility
product of AgCl ? Given :  
eqv (AgCl) = 138.3 ohm
ă1
cmă1 equivă1.
Solution :
Specific conductance of AgCl = Specific conductance of solution ă specific conductance of H2O
= (3.41 ă 1.6) ï 10ă6 = 1.81 ï 10ă6 ohmă1 cmă1.
For saturated solution of sparingly soluble salt,

 eq  
eq

ELECTROCHEMISTRY
16 QUIZRR
and concentration of AgCl = solubility of AgCl.

1000  specific conductance of AgCl


 
eq  (since molarity = normality for AgCl)
Solubility of AgCl

1000  1.81  106


 138.3 
s

1000  1.81  106


 s  1.31 10 5 mol/lit
138.3

The solubility equilibrium of AgCl is shown as

+ ă
AgCl(s) 

 Ag (aq) + Cl (aq)

 KSP = [Ag+] [Clă] = s ï s = s2


= (1.31 ï 10ă5)2 = 1.72 ï 10ă10 M2

Example 6

At 25ĈC, the equivalent conductance of propanoic acid at infinite dilution is 386.6 ohmă1
cm2 equiv.ă1. If its ionisation constant is 1.4 ï 10ă5, calculate equivalent conductance of 0.05
N propanoic acid solution at 25ĈC.
Solution :

Cx2 2
Ka =
1  x  Cx (if x < < 1)

Ka 1.4  105
x = =
C 0.05

= 0.0167

c
But x =
0

  c = x ï 0

= 0.0167 ï 386.6
= 6.47 ohmă1 equiv.ă1

ELECTROCHEMISTRY
QUIZRR 17

Example 7
Kohlrausch found from conductivity measurement on very pure water that the degree of
dissociation of water is 1.9 ï 10ă9. Determine equivalent conductance of water.


 

 0 (H  ) = 350,  0 OH  200

Solution :

c
x=
0

 c = x 0

 
= x  0 H  0 OH 
 
 
= 1.9 ï 10ă9 [350 + 200]
= 1.045 ï 10ă6 ohmă1 cm2 equiv.ă1

Example 8

What is molar conductivity, the conductivity and the resistance (in a cell with constant
0.206 cmă1) of an 0.040 M solution of acetic acid at 25ĈC ? Use Ka = 1.8 ï 10ă5
Solution :

m  CH3COOH   390.5 ohm 1 cm 2 mol1 


 

1000 ï conductivity
m =
molarity

m
but = x (degree of ionisation)
m

 m = m x

 Ka
  m = m (by Ostwald dilution Law)
C

1.8  105
= 390.5
0.04

ELECTROCHEMISTRY
18 QUIZRR
= 8.28 ohmă1 cm2 molă1

molarity  m
 conductivity =
1000

= 3.31 ï 10ă4 ohmă1 cmă1

1
but conductivity (specific conductance) = cell constant ï
resistance

cell constant
 resistance =
conductivity

0.206
= = 621.98 ohm
3.31  104

Example 9

The conductivity of the purest water is 5.5 ï 10ă8 ohmă1 cmă1. What would be the resistance
measured for a sample of this water in the conductivity cell (cell constant = 0.2063 cmă1) ?
What are the value of pKw and pH of pure water ?
(Ĉ (H+) = 349.8 ohmă1 cm2 molă1, Ĉ (OHă1) = 199.1 ohmă1 cm2 molă1)
Solution :
Conductivity of pure water = 5.5 ï 10ă8 ohmă1 cmă1

cell constant
but conductivity =
resistance

cell constant
 resistance =
conductivity

0.2063
= ohm = 3.751 ï 106 ohm
5.5  108

water is weak electrolyte

0  0
hence  0  H 2O  =  
H  H 
= 548.9 ohmă1 cm2 molă1

ELECTROCHEMISTRY
QUIZRR 19

1000  conductivity
 c (calculated) at 55.5 M H2O = molarity

1000  5.5  108


=
55.5

= 9.91 ï 10ă7

c
 x (degree of ionisation) =
0

9.91  107
=
548.9

= 2 ï 10ă9
 [H + ] = Cx = 55.5 ï 2 ï 10ă9
= 1 ï 10ă7
 pH = ă log [H+] = 7
Also [OHă] = [H+] = 1 ï 10ă7
 K w = [H+] [OHă] = 1 ï 10ă14
 pK w = ă log Kw = 14

8. QUANTITATIVE ASPECT OF ELECTROLYSIS


Decomposition of a compound into its constituents by passing electricity is called electrolysis. It
consists of an electrolytic solution in a container. Two electrodes are dipped in it and they are
connected to a battery. When the electricity is passed, the anion moves to the anode and gets
oxidized. This releases electrons, these electrons pass through the outer circuit jump through the
battery and these electrons are available at the cathode for reduction of cations.
e

^C

Cl Na

At anode, Clă  Cl + 1eă (oxidation)


At cathode, Na+ + eă  Na (reduction)

ELECTROCHEMISTRY
20 QUIZRR
9. FARADAY’S LAWS
The quantitative relationship between the amount of electricity passed through a cell and the
amount of substances discharged at the electrodes was systematised by Michael Faraday in the
form of the following laws :

9.1 First law

The amount of substance discharged (deposited or dissolved) at the electrode is proportional to the
quantity of the electricity passing through the electrolyte.

w q 

Mathematically : w  I.t  q  I.t   ...(1)
w  z.I.t 

where w is the weight of the substance discharged at an electrode in gram; q is the charge in
coulomb, t is the time of flow of electricity in second, I is the current in ampere and z is a constant
known as the Electrochemical Equivalent which is defined as the number of grams of the substance
deposited or dissolved by one coulomb of electricity.

9.2 Second law

When the same quantity of electricity is passed through different solutions, the amounts of
different substances deposited or dissolved at the electrodes in different electrolytic cells are
proportional to their equivalent weights and in an electrolytic cell, chemically equivalent amounts
of substances are discharged at both the electrodes.

Interpretation of Faraday’s Second Law

Let us now interpret the second law of Faraday in a simple manner.


1 electron reduces and deposits 1 M+ ion at an electrode (i.e., M+ + e–  M)
 1 mole of electrons shall reduce and deposit 1 mole of M+ ions
If the ion has a valency of n,
n mole of electrons shall reduce 1 mole of Mn+ ions.

1
 1 mole of electrons shall reduce mole of Mn+ ions.
n

1 1
For example : 1 mole of electrons reduces or deposits 1 mole of Ag+ or mole of Cu2+ or mole
2 3
or Al3+.

ELECTROCHEMISTRY
QUIZRR 21

Now, that (number of mole ï valency) represents number of equivalents


 1 mole of electrons shall reduce or deposit 1 equivalent of Ag+ or Cu2+ or Al3+. In general,
1 mole of electricity (electrons) liberates 1 equivalent of matter.
Again we know :
charge of 1 mole of electrons = charge of an electron ï Av. const.
= 1.6021 ï 10ă19 ï 6.022 ï 1023 coulomb
= 96487 coulomb
= 96500 coulomb
= 26.8 ampere-hour per equivalent
= 1 faraday.
Thus the essential content of FaradayÊs second law is that 1 faraday, which corresponds
to 1 mole of electrons, liberates 1 equivalent of matter.
In redox reactions, the amount of the reactant, corresponding to 1 mole of electrons
is thus its equivalent mass.

Electrochemical Equivalent and Equivalent Weight

The weight in gram of a substance liberated by 1 coulomb of electricity is called electrochemical


equivalent whereas the weight in gram liberated by 96500 (or 1 faraday or 1 mole electricity) is
called Gram Equivalent Weight of the substance.
From FaradayÊs law, we can deduce the relationship between the electrochemical equivalent and
equivalent weight.
w  Q (Ist Law)
w  E (IInd Law)

QE 1 1 
 w  F  96500 
F  

w Q
 
E F

No. of gram equivalents = No. of Faradays of electricity


 1 gm eq = any substance = 1 F of electricity
Now there are two approaches to solve a problem
First calculate the number of faradays of electricity by using :

Q It
No. of faradays = 
F 96500

ELECTROCHEMISTRY
22 QUIZRR
(i) Now by using the definition :
1 gm eq. of any substance  1 F of electricity passed
Calculate the number of gm. eq. and by using the definition of gm. eq. (gmeq. = mass/E),
determine the amount of substance deposited.
(ii) Using anodic and cathodic reactions as follows :
Let us consider a typical anode reaction
Mn+ ă ne  M
 n (eÊs)  1 molecule of M
 N0 (n eÊs)  N0 molecule of M (N0 : Avogadro number)
 N0 (n eÊs)  1 mole of M
 n F  1 mole of M (charge of N0 electrons  1 F = 96500 C)
So in this approach, first write anodic and cathodic reactions and derive the mole Vs faraday
relation.
(iii) Using the combined relation obtained from Ist and IInd Laws :
w = Z I t

EIt
 w
96500

Example 10

In the electrolysis of aq. CuSO4, a current of 2.50 Amp is allowed to flow for exactly 3.0 hr.
How many grams of Cu and lts. of O2 are produced at 25ĈC and 1 atm. pressure ?
Solution :
The electrolysis of aqueous solution CuSO4 solution takes place as follows :
CuSO4  Cu2+ + SO42ă
Cathode : Cu2+ + 2e–  Cu(s)
Anode : 2H2O  O2 + 4H+ + 4eă (4OHă  O2 + 2H2O + 4e–)

(sulphate ions and H+ ions remain in the solution to given an acidic solution)
Now from cathode : Cu2+ + 2e  Cu(s)
 2F  1 mole of Cu
calculating number of faradays passed through the cell;

It 2.5  3  3600
No. of faradays =   0.28 F
96500 96500

 2F  1 mole of Cu

ELECTROCHEMISTRY
QUIZRR 23

 0.28 F  1/2 ï 0.28 moles of Cu


 1/2 ï 0.28 ï 63.5 gm
 8.9 gm of Cu have been deposited

Now from anode : 2H2O  O2 + 4H+ + 4eă

 4F  1 mole of O2
 0.28 F  1/4 ï 0.28
 0.07 moles of O2
Now using gas equation : PV = nRT

nRT 0.07  0.0821  298


V lt    1.71 lt
P 1

Example 11

If 6.43 ï 105 Coulombs of electricity are passed through an electrolytic cell containing
NaClO3. 245 gm of NaClO4 are produced at the anode at the end of electrolysis. Determine
the anode efficiency.
Solution :

actual wt. of any substance liberated


Anode efficiency =
theoretical wt. as calculated by Faraday's Law

actual No. of faradays used up


OR Anode efficiency =
total No. of Faradays used

Let us write anode reaction first :

ClO3  ClO4

balancing by ion electron method :

ClO3  H2O  ClO4  2H  2e

 2F  1 moles of NaClO4
 1 mole of NaClO4  2 F of electricity

245 245
 moles of NaClO4  2 ï = 4 F of electricity
122.5 122.5

So for the production of 245 gm of NaClO4, 4F of electricity i.e., 4 ï 96500 C of charge is actually
consumed. But we are given that a total of 6.43 ï 105 C of electricity is passed through the cell,
so some of the charge is wasted.

ELECTROCHEMISTRY
24 QUIZRR

4  96500
 Anode efficiency =  100
6.43  105

 Anode efficiency = 60.03%

Alternative Method :

First calculate the theoretical amount of NaClO4 produced by passing 6.43 ï 105 C.
 2F  1 moles of NaClO4

6.43  105 1 6.43  105


 F   moles of NaClO4
96500 2 96500

1 6.43  105
   122.5
2 96500

 408.12 gm of NaClO4
But in actual only 245 gm of NaClO4 are produced (some of the current is lost as heat and against
the resistance to flow of ions).

245
anode efficiency =  100  60.03%
408.12

Example 12

An acidic solution of Cu2+ salt containing 0.4 gm of Cu2+ is electrolysed until the copper
is deposited. The electrolysis is continued for seven more minutes with the volume of
solution kept at 100 mL and the current at 1.2 Amp. Calculate the volume of gases evolved
at STP during the entire electrolysis.
Solution :
Assuming Cu2+ salt to be CuSO4, the reactions occuring at the electrodes will be :
Anode : H2O  2H+ + óO2 + 2eă
Cathode : Cu2+ + 2eă  Cu
After complete deposition of copper, the reactions would be
Anode : H2O  2H+ + óO2 + 2eă
Cathode : 2H2O + 2eă  H2 + 2OHă

0.4 g
Now, amount of Cu deposited =
63.6 g mol1

= 0.00629 mol

ELECTROCHEMISTRY
QUIZRR 25

1
amount of oxygen liberated = ï 0.00629 mol
2

= 0.003145 mol
quantity of electricity passed in seven minutes after the deposition of the cutire copper
= (1.2 A) (7 ï 60 s) = 504 C

504C
Amount of electrons carrying this much of electricity =  0.00522 mol
96500 C mol1

From the electrode reactions, we can say that

1
amount of oxygen liberated = ï 0.00522 mol
4

= 0.001305 mol

1
amount of hydrogen liberated = ï 0.00522 mol
2

= 0.00261 mol
Total amount of gases liberated in the entire electrolysis
= (0.003145 + 0.001305 + 0.00261) mol
= 0.00706 mol
volume of gases evolved at STP during entire electrolysis
= (0.00706 mol) (22400 mL molă1)
= 158.2 mL

Example 13

A current of 20.0 A is used to plate Ni from NiSO4 solution. Both Ni and H2 are liberated
at cathode. The current efficiency with respect to liberation of Ni is 50%.
(a) What mass of Ni is plated on cathode per hour ?
(b) What is the thickness of plating ion on both the sides of the square cathode of edge
length 4 cm ?
Given : Atomic mass of Ni = 58.79 amu and its density = 8.9 g cmă3
Solution : Since the current efficiency with respect to liberation of Ni is 50%, 10 A out of 20 A current
will be used for the liberation of Ni. Hence,
quantity of electricity available in 1 H to deposit Ni = (10 A) (60 ï 60 s) = 36000 C
Amount of electrons used in plating out

ELECTROCHEMISTRY
26 QUIZRR

36000 C
Ni = = 0.373 mol
965000 C mol1

1
Amount of Ni plated out =  0.373 mol
2

= 0.1865 mol
Mass of Ni plated out = (0.1865 mol) (58.79 g molă1)
= 10.964 gm

1
Mass of Ni plated out on either side of square cathode = ï 10.964 gm
2

= 5.482 gm

5.482
Volume of Ni plated out on either side of square cathode =
8.9 g cm 3

= 0.616 cm3

0.616 cm3
Thickness of Ni plated out on either side of square cathode =
 4 cm  4 cm 
= 0.0385 cm

Example 14

Calculate the quantity of electricity required to reduce 12.3 gm of nitrobenzene to aniline


if the current efficiency for the process is 50%. If the potential drop across the cell is 3.0
volts, how much energy is consumed ?
Solution :
Writing the ionic reaction for the reduction of nitro-benzene as follows :
C6H5NO2 + 6H+ + 6eă  C6H5NH2 + 2H2O
 6F of electricity  1 mole of nitro-benzene
Now moles of nitro-benzene = 12.3/123 = 0.1 moles
 0.1 mole  0.6 F
hence 0.6 F of electricity are used to reduce 12.3 gm of nitro-benzene if the current efficiency
is 100%. But it is given that current efficiency is 50%, so

0.6  100
No. of Faradays required =  1.2F
50

ELECTROCHEMISTRY
QUIZRR 27

Now potential difference = 3 V


The energy (E) consumed is given by : E = charge ï potential difference
E = 115800 ï 3 = 347400 J = 347.4 kJ

10. ELECTROCHEMICAL CELL


An electrochemical cell is a system consisting of electrodes that dip into an electrolyte and in which
a chemical reaction either uses or generates an electric current.
A voltaic or galvanic cell is an electrochemical cell in which a spontaneous reaction generates an
electric current.

Construction and Working

A zinc rod is dipped in ZnSO4 solution and a Cu rod in CuSO4 solution. The Zn rod is externally
connected to the copper rod through a rheostate, a galvanometer and a plug key.

Galvanometer
Plug key
1.103
( )

Salt bridge
Zn Cu

ZnSO4 CuSO4

The two aqueous solutions are interlinked through a salt bridge which is an inverted ÂuÊ tube
which is filled with saturated semi solid paste of Agar Agar saturated in KCl or KNO3 solution
[Agar agar is a carbohydrate which forms a jelly like substance when dissolved in hot water]

Working

When the circuit is completed, a deflection is observed in the (G) towards the zinc electrode
indicating that the eă are flowing from the Zn electrode to Cu electrode.
At the Zn electrode, oxidation takes place.
Zn  Zn+2 + 2eă (oxidation) ...(1)
The ÂeÊ removed or lost or retained by the metal move through the material of the electrode and
reach the Cu electrode at which they are accepted by Cu ions of the solution to form neutral
copper atoms.
Cu+2 + 2eă  Cu (reduction) ...(2)

ELECTROCHEMISTRY
28 QUIZRR
In an electrochemical cell, each electrode constitutes one half of the cell and the reaction taking
place at the electrode is called half-cell reaction. The overall cell reaction is obtained by adding
the two half-cell reactions (1) and (2).
Zn + Cu+2  Zn+2 + Cu (Overall reaction)
From this it is found that when Zn is added to CuSO4 solution, Zn displaces Cu from CuSO4 with
the liberation of heat. But in the electrochemical cell there is no direct contact between Zn and
CuSO4. Hence whatever the heat energy that would have been liberated appears in the form of
electrical energy. Hence the electrochemical cell acts as a source of current although for a short
interval.
The electrode at which oxidation takes place or the metal rod becomes negative charged is called
negative electrode.
The electrode at which reduction takes place or the metal rod becomes positive charged is called
positive electrode. Accordingly, in the above constructed electrochemical cell, zinc electrode acts as
negative electrode while Cu electrode acts as positive.

T he salt bridge

(a) acts as a link between the two aqueous solution.


(b) overcomes liquid junction potential.
(c) Maintains the electrical neutrality of the aqueous solution of the electrodes by releasing or
sending oppositely charged ions into the solution.
In general KCl salt bridge is used while KNO3 salt bridge is used when silver electrode is
involved as one of the electrodes.

11. THE STANDARD HYDROGEN ELECTRODE


The standard hydrogen electrode looks like this :

hydrogen
temperature = 298 K
at 1 bar
platinum wire

platinum foil covered in


porous platinum
dilute sulphuric acid
[H+] = 1 mol dmă3

ELECTROCHEMISTRY
QUIZRR 29

W hat is happening ?

As the hydrogen gas flows over the porous platinum, an equilibrium is set up between hydrogen
molecules and hydrogen ions in solution. The reaction is catalysed by the platinum.

2H+(aq) + 2eă 



 H2(g)

This is the equilibrium that we are going to compare all the others with.

Using the standard hydrogen electrode

The standard hydrogen electrode is attached to the electrode system you are investigatingăfor
example, a piece of magnesium in a solution containing magnesium ions.

temperature = 298 K V
high resistance
voltmeter
hydrogen
at 1 bar magnesium
platinum wire

salt
bridge

platinum foil covered in


porous platinum
dilute sulphuric acid magnesium sulphate
[H+] = 1 mol dmă3 solution
2+ ă3
[Mg ] = 1 mol dm

Cells and half cells

The whole of this set-up is described as a cell. It is a simple system which generates a voltage.
Each of the two beakers and their contents are described as half cells.

The salt bridge

The salt bridge is included to complete the electrical circuit but without introducing any more bits
of metal into the system. It is just a glass tube filled with an electrolyte like potassium nitrate
solution. The ends are „stoppered‰ by bits of control wool. This stops too much mixing of the
contents of the salt bridge with the contents of the two beakers.
The electrolyte in the salt bridge is chosen so that it doesnÊt react with the contents of either
beaker.

ELECTROCHEMISTRY
30 QUIZRR
W hat happens ?

These two equilibria are set up on the two electrodes (the magnesium and the porous platinum) :

Mg2+(aq) + 2eă 



 Mg(s)

2H+(aq) + 2eă 



 H2(g)

Magnesium has a much greater tendency to form its ions than hydrogen does. The position of the
magnesium equilibrium will be well to the left of that of the hydrogen equilibrium.
That means that there will be a much greater build-up of electrons on the piece of magnesium
that on the platinum. Stripping all the rest of the diagram out, apart from the essential bits :

Relatively few electrons


build up on the platinum
from the ionisation of the Lots of electrons build up
hydrogen from the ionisation of the
magnesium

There is a major difference between the charge on the two electrodes - a potential difference
which can be measured with a voltmeter. The voltage measured would be 2.37 volts and the
voltmeter would show the magnesium as the negative electrode and the hydrogen electrode as
being positive.
This sometimes confuses people! Obviously, the platinum in the hydrogen electrode isnÊt positive
in real terms - there is a slight excess of electrons built up on it. But voltmeters doesnÊt deal in
absolute terms - they simply measure a difference.
The magnesium has the greater amount of negativeness - the voltmeter records that as negative.
The platinum of the hydrogen electrode isnÊt as negative - it is relatively more positive. The
voltmeter records it as positive.

What if you replace the magnesium half cell by a copper one ?

This means replacing the magnesium half cell by one with a piece of copper suspended in a
solution containing Cu2+ ions with a concentration of 1 mol dmă3. You would probably choose to
use copper(II) sulphate solution.

ELECTROCHEMISTRY
QUIZRR 31

Copper forms its ions less readily than hydrogen does. Of the two equilibria :

Cu2+(aq) + 2eă 



 Cu(s)

2H+(aq) + 2eă 



 H2(g)

...the hydrogen one lies further to the left. That means that there will be less build up of electrons
on the copper than there is on the platinum of the hydrogen electrode.

Some elections build up


the platinum from the
ionisation of the hydrogen Even fewer elections build
up from the ionisation of the
copper

There is less difference between the electrical charges on the two electrodes, so the voltage
measured will be less. This time it is only 0.34 volts.

The other major change is that this time the copper is the more positive (less negative) electrode.
The voltmeter will show the hydrogen electrode as the negative one and the copper electrode as
positive.

The emf of a cell measured under standard conditions is given the symbol EĈ cell.

12. CELL CONVENTIONS

A quick way of drawing a cell

Drawing a full diagram to represent a cell takes too long. Instead, the cell in which a magnesium
electrode is coupled to a hydrogen electrode is represented like this :

ELECTROCHEMISTRY
32 QUIZRR
Square brackets show the hydrogen Single vertical lines show a boundary
flowing over the platinum. between two phases - for example, between
the magnesium ions and the solid metal.

>
>
Pt [H2(g)] | 2H+(aq) || Mg2+(aq) | Mg(s)

>
Double vertical lines show the salt
bridge. This is sometimes shown by a
single (or double) broken vertical line
(or lines).

You will often find variants on the way the hydrogen electrode is represented, such as :
Pt | H2(g) | 2H+(aq) || or Pt [H2(g)] | H+(aq) ||

>
>

Square brackets replaced by a One hydrogen ion shown


vertical line showing the rather than the 2 from
boundary between the platinum the equation.
and the hydrogen.

Cell Notation of An Electrochemical Cell

(i) Anode is written on the left side and cathode is written on the right side.
(ii) Phase boundaries are indicated by vertical bar or slash.
(iii) Concentration of the electrolytes in the anode and cathode must be written in parenthesis.
(iv) In case of a gas, the partial pressure is to be mentioned in atm or mm Hg.
(v) A comma is used to separate two chemical species present in the same solution.
(vi) A double vertical line i.e. || denotes that a salt bridge is present.
(vii) EMF of the cell is written on the extreme right of the representation.
For example :
(i) Zn(s) |ZnSO4(c1 M)|| CuSO4(c2 M)| Cu(s) ; Ecell
(ii) Pt |H2(P1 atm) | HCl (c M) AgCl(s)|Ag ; E´cell
2+ 3+ +
(iii) Pt |Fe (c1 M), Fe (c2 M)|| Ag (c M)| Ag ; E´´cell
Note : In some cell representations (as in (ii) above), the salt bridge is not indicated which implies
that the electrolyte is common to both anode and cathode compartments.

ELECTROCHEMISTRY
QUIZRR 33

Pt [H2(g)] | 2H+(aq) || Mg2+(aq) | Mg(s) EĈ cell = ă 2.37 V

Show that the magnesium (the right hand electrode) is the


negative one.

In the copper case :


Pt [H2(g)] | 2H+(aq) || Cu2+(aq) | Cu(s) EĈ cell = + 0.34 V

Shows that the copper (the right-hand electrode) is the positive


one.

Summarizing what standard electrode potentials tell

Remember that the standard electrode potential of a metal/metal ion combination is the emf
measured when that metal/metal ion electrode is coupled to a hydrogen electrode under standard
conditions.
What you are doing is comparing the position of the metal/metal ion equilibrium with the equilibrium
involving hydrogen.
Here are a few typical standard electrode potentials :
Metal/metal ion combination EĈ (volts)
Mg2+ / Mg ă 2.37
Zn2+ / Zn ă 0.76
Cu2+ / Cu + 0.34
Ag+ / Ag + 0.80

Remember that each of these is comparing the position of the metal/metal ion equilibrium with
the equilibrium involving hydrogen.
Here are the five equilibria (including the hydrogen one) :

Mg2+(aq) + 2eă 



 Mg(s)

Zn2+ (aq)
+ 2eă 

 Zn(s)

2H+(aq) + 2eă 



 H2(g)

Cu2+(aq) + 2eă 



 Cu(s)

Ag+(aq) + eă 

 Ag(s)

ELECTROCHEMISTRY
34 QUIZRR
If you compare these with the EĈ values, you can see that the ones whose positions of equilibrium
lie furthest to the left have the most negative EĈ values. That is because they form ions more
readily - and leave more electrons behind on the metal, making it more negative.
Those which donÊt shed electrons as readily have positions of equilibrium further to the right.
Their EĈ values get progressively more positive.

Half Cell Potential (Single-Electrode Potential)

When a metal is dipped into a solution containing its own ions, a half cell or a single electrode
is formed. In a half cell there are two opposing tendencies. Firstly, the metal, say M, may dissolve
in the solution or rather may go into the solution in the form of ions (M  Mn+ + ne; oxd.) and
secondly, the ions Mn+, from the solution may deposit on the electrode (Mn+ + ne  M; red.).
When one of these two tendencies dominates over the other, there develops a half cell potential
or electrode potential.
The tendency to lose electrons, i.e., to get oxidised is called oxidation potential and similarly
the tendency to gain electrons, i.e., to get reduced is called reduction potential.
Since any half cell reaction can be written as a reversible process e.g., Cu2+ + 2e  Cu, the
reduction potential and oxidation potential for a single electrode are equal in magnitude but
opposite in sign. For the electrode Cu/CuSO4 (1 M), the reduction potential, EĈ Cu2+, Cu = + 0.34
V and so its oxidation potential EĈ Cu, Cu2+ = ă 0.34 V at 25ĈC. The half cell potentials cannot be
directly determined as there is no way of isolating a single half cell reaction. The electrode
potential can be determined by coupling it with a standard hydrogen electrode (i.e., by forming
a cell). As the electrode potential of a standard hydrogen electrode has been arbitrarily fixed as
zero volt at 25ĈC, the emf of such a cell gives the single electrode potential or emf of half cell. A
standard hydrogen electrode is represented as :
Pt, H2 (1 atm), HCl ([H+] = 1 M)

Single Convention (IUPAC)

The reduction potential of a half cell is given a positive sign when the half cell reaction involves
reduction, when coupled with a standard hydrogen electrode; and a negative sign when the half
cell reaction involves oxidation, when connected with a standard hydrogen electrode.

Standard Half Cell Potential (Standard Electrode Potential) and Electrochemical Series

It will be discussed a little later that the half cell potential at a temperature depends upon the
concentration of ions of the dissipated material. If for the half cell, M | Mn+ (aq), [Mn+] = 1 M
at 25Ĉ potential is termed standard half cell potential or standard electrode potential.
Standard half cell potential, like half cell potential is also measured on standard hydrogen electrode
scale.

ELECTROCHEMISTRY
QUIZRR 35

Such a list of EĈ values of various half cells arranged in the given orders is known as
electrochemical series.
Some of the half cells or electrodes arranged in decreasing their EĈ (reduction) values are as
follows :

Standard Reduction Potentials at 298 K

Reduction half reaction Standard Reduction half Standard


reduction potential reduction reduction
EĈ (in volts) potential EĈ
(in volts)

Li+ + eă  Li ă 3.05 Sn2+ + 2eă  Sn ă 0.14

K+ + eă  K ă 2.93 Pb2+ + 2eă  Pb ă 0.13

Ba2+ + 2eă  Ba ă 2.90 2H+ + 2eă  H2 0.00

Ca2+ + 2eă  Ca ă 2.87 Sn4+ + 2eă  Sn2+ 0.013

Na+ + eă  Na ă 2.71 Cu2+ + eă  Cu+ 0.15

Mg2+ + 2eă  Mg ă 2.37 Cu2+ + 2eă  Cu 0.34

Al3+ + 3eă  Al ă 1.66 I2 + 2eă  2Iă 0.53

Mn2+ + 2eă  Mn ă 1.18 Fe3+ + eă  Fe2+ 0.77

2H2O(l) + 2eă  H2(g) ă 0.83 Ag+ +eă  Ag 0.80


+ 2Hă (aq)

Zn2+ + 2eă  Zn ă 0.76 Br2 + 2eă  2Bră 1.08

Cr3+ + 3eă  Cr ă 0.74 Cl2 + 2eă  2Clă 1.36

Fe2+ + 2eă  Fe ă 0.44 O2 (g) + 4H+ (aq) + 4eă 1.23

 2H2O

Cd2+ + 2eă  Cd ă 0.40 Au3+ + 3eă  Au 1.50

Co2+ + 2eă  Co ă 0.28 Co3+ + eă  Co2+ 1.82

Ni2+ + 2eă  Ni ă 0.25 F2 + 2eă  2Fă 2.87

ELECTROCHEMISTRY
36 QUIZRR
Note :
(i) The standard half cell potential EĈ is an intensive property, like temperature or molar
volume and so EĈ shall be the same for half cell reaction whether it is represented as
1
2X+ + 2e  X2, or X+ e  X . But unlike EĈ, GĈ (standard free energy change) is an
2 2
extensive property which depends upon the mass, that is to say, if a half reaction, 2X+ +
1
2e  X2 is represented as X+ + e  X , GĈ of the latter would be half that of the
2 2
former.
(ii) If the direction of a half cell (cell) reaction is reversed, its potential has the same magnitude
but opposite sign, e.g., if for Cu2+ + 2e  Cu; EĈ = + 0.34 V

then for Cu  Cu2+ + 2e; EĈ = ă 0.34 V

The same is also true for energy change.


(iii) Potentials are not thermodynamic functions and may not be added but the potential may
be calculated from the free energy G, using GĈ = ă nFEĈ. For example,
Fe3+ + e  Fe2+; EĈ = + 0.77 V;   GĈ = ă 1(+ 0.77) F = ă 0.77 F
Fe2+ + 2e  Fe; EĈ = ă 0.44 V;   GĈ = ă 2(ă 0.44) F = + 0.88 F
Fe3+ + 3e  Fe On adding : GĈ = + 0.11 F

G 0.11F
 EĈ for (Fe3+ + 3e  Fe) =    0.04 V
 nF  3F

but not (0.77 ă 0.44) = 0.33 V]

Characteristics of Electrochemical Series

(1) Reactive metals are placed on top (e.g. Li) and they have a great tendency to get oxidized.
Non-reactive metals like Ag and Au.
(2) Any metal above hydrogen can displace it from dilute acids.

Example : Zn + dil. 2HCl  ZnCl2 + H2

Cu + dil. HCl  no reaction

Any metal which is above another can displace that from its salt solutions.

Example : Zn + CuSO4  ZnSO4 + Cu

If two metals form a cell, the metal that is above undergoes oxidation and forms the anode
while the one below forms the cathode.

Example : Zn-Anode  Cu-Cathode

(0.76 V)  (+ 0.34 V)

ELECTROCHEMISTRY
QUIZRR 37

Calculation of Cell Potential

In an electrochemical cell, electrons flow from negative electrode to the positive electrode. This
shows that there is a potential difference between the two electrodes.
The minimum potential which causes the flow of the electrons from the negative electrode to the
positive electrode is called emf.
The emf of a cell is defined as the algebraic difference between the SRP of the positive electrode
and the SRP of the negative electrode even though oxidation is taking place at the negative
electrode.
Cell Potential (EĈ cell) = EĈ Cathode ă EĈ Anode

Standard Reduction potential Standard Reduction Potential of


of cathode anode
EĈ Cathode EĈ Anode
Or
E Right E Left

In general, for a given electrode, the magnitude of oxidation and reduction potentials remain
same but they differ with respect to their signs.
Examples :
For a cell made of a zinc electrode in ZnSO4 and copper electrode in CuSO4.
EĈ cell = EĈ Cu+2/Cu ă EĈ Zn+2/Zn
= 0.35- (ă0.76)
= 1.1 V

Example 15

The electrode potentials A, B, C and D are ă2.52 V, ă 0.16 V, + 1.3 V and ă3.01 V respectively.
On the basis of this name the following :
D ă 3.01 V 
A ă 2.52 V 
  Electro chemical series

B ă 0.16 V 

C + 1.3 V 
Solution :

(a) Most electro positive  D

(b) Most electro negative  C

(c) Highest reducing power  D


ELECTROCHEMISTRY
38 QUIZRR

(d) Highest oxidising power  C

(e) Lowest oxidising power  D

(f) Displaced by all other system  C

(g) Does not displace any other system  C

(h) displaces H2  D, A, B

(i) Always acts as negative electrode  D

(j) Always acts as positive electrode  C

(k) System which is not displaced by any other  D


(i) A and B
A is negative and B is positive
(ii) B and C
B is negative and C is positive
(iii) C and D
C is positive and D is negative
(iv) D and A
D is negative and A is positive

Example 16

The dry cell (flash light battery) used to power flashlights, clocks, radios etc follows following
reaction.
Zn(s) + 2 MnO2(s) + 8 NH4+  Zn2+ + 2 Mn3+ + 8 NH3 + 4 H2O
(a) Write anode and cathode reactions.
(b) Calculate the E0 of the dry cell if the electrode potential of cathode (E0) varies between
+ 0.49 V and + 0.74 V and of anode (E0) is ă 0.76 V.
Solution :
Note : A given electrode potential is to be taken as Reduction Potential.
Anode : Zn - 2e  Zn2+
Cathode : 2MnO2(s) + 8NH4+ + 2eă  2Mn3+ + 8NH3 + 4H2O
E0 of cathode varies between + 0.49 to + 0.74 V
E0 of cell (for E0c = 0.49 V)

ELECTROCHEMISTRY
QUIZRR 39

E0 = E0c ă E0a
E0 = 0.49 ă (ă0.76) = 1.25 V
E0 of cell (for E0c = 0.74 V)
E0 = E0c ă E0a = 0.74 ă (ă0.76) = 1.50 V
E0 of the cell varies between 1.25 V to 1.50 V

Example 17

For each of the following cells :


(a) Write the equation for cell process.
(b) Find E0 for each cell.
(c) Explain the significance of any negative answers in part (b).
1. Fe/Fe(NO3)2 (1.0 M) || Zn2+ (1.0 M)/Zn
2. Pt/Cl2(g)/KCl || Hg2Cl2(s)/Hg
3. Cd/Cd2+ (1.0 M) || AgNO3/Ag
E0(Fe) = 0.41 V; E0(Cd) = 0.40 V; E0(Zn) = 0.76 V
E0(Clă/Cl2) = ă 1.36 V; E0(Ag) = ă 0.80 V; E0 (Hg/Hg2Cl2) = ă 0.27 V
Solution :
Note : The values of electrode potential given are their oxidation potentials. For reduction potential,
change their i.e., EĈ(Fe2+/Fe) = ă 0.41 V and so on...

1. Fe + Zn2+  Fe2+ + Zn
E0 = Ec0 ă E0a
EĈ= = ă 0.76 ă (ă0.41) = ă 0.35 V
negative EMF values means that the cell will not work in the manner shown i.e., Fe as
anode and Zn as cathode. So reversing (interchanging) the anode and cathode i.e., making
Zn as anode and Fe as cathode, can make the cell work.

2. Anode : 2Clă ă 2e  Cl2


Cathode : Hg2Cl2 + 2eă  2 Hg + 2Clă
Hg2Cl2  Cl2 + 2 Hg
E0 = Ec0 ă E0a
E0 = 0.27 ă (1.36) = ă 1.09 V
negative EMF values means that the cell will not work in the manner shown i.e. Cl2/Clă as
anode and Hg2Cl2 as cathode. So reversing (interchanging) the anode and cathode i.e.,
making Hg2Cl2 (i.e., Hg/Hg2Cl2) as anode and Cl2/Clă as cathode (i.e., Clă/Cl2), can make
the cell work.

ELECTROCHEMISTRY
40 QUIZRR
3. Cd + 2 Ag+  Cd2+ + 2 Ag
E0 = E0c ă E0a = 0.8 ă (ă 0.4) = 1.20 V
EMF value is positive, hence cell will function with Cd as anode.

1 3. Cell Potential and Nernst Equation

Nernst equation is used to relate either half-cell potential or EMF of a cell with the concentration
of the involved species. Let us first consider a redox change occurring in a electrochemical cell,

xA + yB 

 zC + aD

where A, B, C and D are the species whose concentrations vary i.e. they are either gases or
solution phases. For species A, the free energy per mole of A can be given thermodynamically as
GA = GĈ A + RT In [A]
For x moles A, xGA = xGĈ A + xRT ln[A] = xGĈA + RT ln[A]x
Similarly, for all other species,
yGB = yGĈ B + RT ln [B]y
zGC = zGĈ C + RT ln [C]z
and aGD = aGĈ D + RT ln [D]a
Now, the free energy change for the overall cell reaction can be deduced as
G = (zGC + aGD) ă (xGA + yGB)
= zGĈ C + RT ln [C]z + aGĈ D + RT ln [D]a – xG°A ă RT ln [A]x ă yGĈ B ă RT ln [B]y

 Cz  D a
= (zGĈ C + aGĈ D) ă (xGĈ A + yGĈ B) + RT ln
 A x  B y

G  G + RT l n
Cz D a
...(i)
 A x B y
where GĈ is the free energy change when all the reactants and products are present at one
molar concentration.
Any spontaneous reaction occuring in a cell, occurs with a decrease in free energy. This decrease
in free energy brings in an equivalent amount of electric work obtainable from a given system
over and above any PdV energy that can be delivered to the surrounding. This can be calculated
by the total charge driven through cell and the potential difference. Thus
ă G = Total charge ï EMF of the cell
ă G = nF ï Ecell

ELECTROCHEMISTRY
QUIZRR 41

[Negative sign indicates decrease of free energy and it implies that as Ecell becomes more and
more positive, G will become more and more negative, making the reaction spontaneous]
Similarly, ă GĈ = nFEĈ cell
Therefore, equation (i) can be written as

 Cz  D a
ă nFEcell = ă nFEĈ cell + RT ln
 A x  B y
Dividing both the sides by ă nF gives,

Ecell  Ecell 
RT
ln
C  D z a

nF  A x  B y
Putting T = 298 K, R = 8.314 J/mol K, F = 96500 C, we get

z a
Ecell  Ecell 
0.059
log
 C  D 
...(ii)
n  A  x  B y
The equation (ii) is called Nernst equation, which is applicable to half-cell reactions as well as to
complete cell reactions.
Daniel cell represented as Zn(s) | Zn2+ (C1 M) || Cu2+ (C2 M)| Cu(s) assumes that Zn is the
anode and Cu is the cathode. Such an assumption would be true only if the cell potential (Ecell)
is positive.
The cell potential is given in the following three ways of which we would choose the third one
in all our problems.

Ecell = ERP(Cathode) + EOP(Anode)

or Ecell = EOP(Anode) ă EOP(Cathode)

or Ecell = ERP(Cathode) ă ERP(Anode)

ERP(Cathode) is the reduction potential of the cathode while ERP(Anode) is reduction potential of the
anode. EOP(Cathode) is the oxidation potential of the cathode while EOP(Anode) is the oxidation
potential of the anode.
Now, let us find the EMF of Daniel cell using Nernst equation. Since we need to represent the
reduction potential of cathode and anode, we first need to write the relevant reduction reactions.

For cathode : Cu2+ + 2eă  Cu

E
Cu2 |Cu
 E
Cu 2|Cu

RT
nF

log Qc or Qpc 
ELECTROCHEMISTRY
42 QUIZRR

E is the standard reduction potential of the given half reaction, R is the universal gas
Cu 2|Cu

constant, T is the absolute temperature at which cell works, F is the Faraday constant and n is
the number of mole of electrons as seen in the reaction. The expression in the log term should
be that of Kc or Kpc. This means that if reaction involves no gases, then the expression in the log
term should be that of Kc while if a gas is involved then the expression in the log term should
be that of Kpc. In these expressions, the concentration should always be in moles per liter while
the partial pressure should be in atmosphere units.

0.059 1
 E  E  log
Cu2+|Cu Cu2|Cu 2 Cu 2 
 

For anode : Zn2+ + 2eă  Zn

0.059 1
E  E  log
Zn2+|Zn Zn2 |Zn 2  Zn 2 
 

As Ecell  E E
Cu2 |Cu Zn2 |Zn

0.059 1 0.059 1
Ecell  E  log  E 2+  log
 Cu2+|Cu 2  Cu 
2  Zn |Zn 2  Zn 2 
   

Zn 2  
log  
0.059
Ecell  E 2+  E 2+ 
Cu |Cu Zn |Zn 2 Zn 
2 
 

 Zn2  
log  
0.059
 Ecell  Ecell 
2 Cu2 
 

Note : Since Ecell has been defined as ERP(Cathode) ă ERP(Anode), the Nernst expression holds good
even if the number of mole of electrons of the two half reactions are different.
For example, consider the cell,
Pt | H2 | HCl | | Cu2+| Cu

For cathode : Cu2+ + 2eă  Cu

0.059 1
E |Cu  E  log
Cu2  Cu2 |Cu 2 Cu 2 
 

ELECTROCHEMISTRY
QUIZRR 43

For cathode : H+ + eă  óH2

ó
 PH 
log  2 
0.059
E  E + 
H+|H2 H |H2 1 H 
 

ó
 PH 
log  2 
0.059 1 0.059
 Ecell  E 2+  E +  log 
Cu |Cu H |H2 2  Cu2   2  H 
   

It is also possible to balance the electrons in both the half cell reactions and then subtract
ERP(Anode) from ERP(Cathode). That is,

For anode : 2H+ + 2eă  H2

0.059 PH2
E  E  log
H+|H2 H+|H2 2 2
H 
 

ó
 H 
Ecell  E 2+  E + 
0.059
log  
Cu |Cu H |H2 2 Cu 2+  PH
  2

14. EMF-MEASUREMENTS : APPLICATIONS


Emf measurements have wide applications such as in the determination of pH, Ksp, H, S etc.

14.1 Determination of Thermodynamic Data :

Ć G can be determined
G = ă nF Ecell
Using Gibbs-Helmholtz equation :

 d  G  
G = H + T  d T  p
 

 d  Ecell  
ă nFEcell = H ă nFT  d T  p
 

ELECTROCHEMISTRY
44 QUIZRR
Ć Temp. coefficient of the emf of the cell can be determined

 d  Ecell   H  nFEcell
  =
 d T  p nFT

 d  Ecell   H E
  =  cell
 d T  p nFT T

Ć Enthalply change can be determined

 d Ecell 
H = ă nF Ecell + nFT  
 dT 

  d Ecell  
=  nF Ecell  T   
  d T  p 

Ć Entropy change can be determined


G = H ă T S

 d  G  
S =   dT  p
 

 d 
S =   dT   nF Ecell  
 p

 d Ecell 
S = nF  d T 
p

  dE   dE
(a) H  nF  T    E where is called temperature coefficient representing the change
  dT   dT

of EMF with the change of temperature, n is the number of mole of electrons involved, F
is one Faraday, E is EMF of the cell at temperature T and T is the absolute temperature.
 dE 
Depending upon the value of   , H can be negative or positive i.e., reaction can be
 dT 
exhothermic or endothermic.

ELECTROCHEMISTRY
QUIZRR 45

 dE   dE 
(b) S = nF   . When   is negative, the change in entropy would also be negative and
 d T   dT 

 dE 
when   is positive change in entropy would be favoured, i.e., S would be positive.
 dT 

Determination of Equilibrium Constant :

Let us assume that the redox change occuring in Daniel cell attains equilibrium. At equilibrium,
the reduction potential values of the two electrodes become equal and EMF of cell becomes zero.

Zn(s) + Cu2+ (aq) 



 2+
 Zn (aq) + Cu(s)

The equilibrium constant for this reaction is given as

 Zn 2  
Keq   
Cu 2 
 

Apply Nernst equation to complete cell reaction

 Zn 2 
Ecell  Ecell 
RT
ln  
nF Cu 2 
 

RT
 Ecell  l n Keq (as Ecell = 0)
nF

 nF EĈcell = RT ln Keq
ă GĈ = RT ln Keq

GĈ = ă 2.303 RT log Keq

Note : This relation is valid for many equilibrium constants like Kw, Kp, Kc, Ksp, Kf, Kd etc.

Ionisation Constants of Weak Acid

For such cases we determine [H+] using a suitable reference half cell.
Pt (H2) | H+, HA(C1) || Cu2+ (C2) | Cu
Its Ecell is measured. Thus [H+] in HA can be determined

2
H 
log  
0.0591
Ecell  Ecell 
2 Cu2  
 
ELECTROCHEMISTRY
46 QUIZRR
If [H+] is known, then by OstwaldÊs dilution law

H   K a C1
 

Determine pH of a Solution :

Pt(H2) | H+ (pH = x) || Clă (1 M) | Hg2Cl2, Hg


calomel
L.H.S. half cell H2(g)  2H+ + 2eă EĈ ox = 0.00 V
R.H.S. half cell Hg2Cl2(s) + 2e  2Hg(l) + 2Clă EĈ red = 0.2676
+ ă
Net H2(g) + Hg2Cl2(s)  2H (aq) + 2Hg(l) + 2Cl (aq) EĈ cell = 0.2676 V

K = [Clă]2 [H+]2
= [H+]2 (since [Clă] = 1 M)
0.0591
Ecell  Ecell  log [H  ]2  Ecell

 0.0591 pH
2
Ecell  EĈ cell
pH=
0.0591

Example 18
Given the overall formation constant of the [Fe(CN)6]4ă ion as 1035 and the standard
potentials for the half reactions,

Fe3+ + eă 

 Fe
2+
; EĈ = 0.77 V

[Fe(CN)6]3ă + eă 
 [Fe(CN)6] ;
 4ă
EĈ = 0.36 V
Calculate the overall formation constant of the [Fe(CN)6]3ă ion
Solution :
Let Kf be the formation constant of [Fe(CN)6]3ă ion.

Fe2+ + 6CNă 



 4ă 35
 [Fe(CN)6] ; Kf = 10 ; G10 = ă 2.303 RT log Kf = ă 199704.69 J

Fe3+ + eă 


2+
 Fe ; EĈ = 0.77 V; G02 = ă 96500 ï 0.77 = ă 74305 J

[Fe  CN 6 ]4  

 [Fe  CN 6 ]  e ; E   0.36 V ; G03 = + 96500 ï 0.36 = 34740 J
3 



Fe3+ + 6CNă  [Fe(CN)6]3ă; G 04 = G10 + G02 + G03

G 04 = ă 239269.69 J
´
G 04 = ă 2.303 RT log K f

 K´f = 8.59 ï 1041

ELECTROCHEMISTRY
QUIZRR 47

Example 19

For the reaction 4Al(s) + 3O2(g) + 6H2O + 4OHă  4[Al(OH)4ă]; EĈ cell = 2.73 V.

If Gf (OHă) = ă 157 kJ molă1 and Gf (H2O) = ă 237.2 kJ molă1, determine Gf (Al(OH)4ă).

Solution :
The GĈ and EĈ cell are related by
GĈ = ă nFEĈ cell = ă 12 ï 96500 ï 2.73 = ă 3.16 ï 103 kJ

GĈ = 4 G f (Al(OH)4ă) ă 6 G f (H2O) ă 4 G f (OHă)

(since G f of Al(s) and O2(g) are zero)


G f Al  OH 4

=  3.16  103   6   237.2   4   157 
4

= 1.30 ï 103 kJ molă1

Example 20

For the silver zinc button (miniature) cell net reaction is


Zn(s) + Ag2O(s)  ZnO(s) + 2Ag(s)

Gf (Ag2O) = ă 11.21 kJ molă1

Gf (ZnO) = ă 318.3 kJ molă1

Determine EĈcell of this button cell.

Solution :

GĈ = G f (ZnO) ă G f (Ag2O)

= ă 318.3 ă (ă 11.21)
= ă 307.09 kJ molă1 = ă 307.09 ï 103 J molă1
Also GĈ = ă nFEĈ cell
n = 2

G 307.09  103


 EĈcell = ă   1.591 V
nF 2  96500

Again note G f (element) = Zero

ELECTROCHEMISTRY
48 QUIZRR
Example 21

Find the equilibrium constant for the reaction, In2+ + Cu2+  In3+ + Cu+, at 298 K.
  
Given : ECu2  / Cu  = 0.15 V; EIn3  / In  = ă 0.42 V & EIn 2  / In  = ă 0.40 V.

Solution :

In 3  2 e  In 1 ...(1)
... (2)
In 2  e  In 1
In 2  In 3  e ..(3) [can be obtained by subtracting equation (2) from equation (1)]

ăFEĈ = ă 2F (ă 0.42) + F(ă 0.40); EĈ = ă 0.84 + 0.4 = ă 0.44 V

In+2 + Cu+2  In+3 + Cu+; EĈ = 0.15 + 0.44 = 0.59 V

0.59
ă nFEĈ = ă RT In K;  log K ; K = 1010.
0.059

Example 22
A cell contains two hydrogen electrodes. The negative electrode is in contact with a solution
of 10ă6 M hydrogen ions. The emf of the cell is 0.118 V at 25ĈC. Calculate the concentration
of hydrogen ions at the positive electrode.
Solution :
The given cell is
Pt / H2 (1 bar) | H+ (10ă6 M) || H+ (?) | H2 (1 bar) / Pt
The cell reactions are

Cathode H 
+
R
 e 
1
2
H2

Anode 1
2
 
H2  H   e
L

   
H+
 H
R

L

The cell potential is given as

H 
RT  L
Ecell  ln
F  H 
 R

106
log
which gives 0.118 V = ă (0.05915 V) H  / M
 R

ELECTROCHEMISTRY
QUIZRR 49

or 
log H   / M
 R  = log (10ă6) + 2 = ă 6 + 2 = ă 4

i.e. [H +] R = 10ă4 M

Example 23

If excess of Zn is added to 1.0 M solution of CuSO4, find the concentration of Cu2+ ions at
equilibrium. The standard reduction potential of Zn and Cu at 25Ĉ are ă 0.76 V and + 0.34 V.
Solution :
We know that at equilibrium, Ecell = 0.0 V and the reaction coefficient Q = Keq. So first let us
calculate the value of Keq. as follows :

E0 = E0c  E0a

E0 = 0.34 ă (ă 0.76) = 1.10 V

Now using E0  0.059 log K eq.


n

E0  n 1.10  2
 log K eq    37.288
0.059 0.059

 Keq. = 1.94 ï 1037


Now writing the reaction at equilibrium :
Zn + Cu2+  Zn2+ + Cu
Let x be the concentration of Cu2+ at equilibrium
Zn Cu2+ Zn 2+ Cu
initial concentration  1.0 0 0
final concentration x x 1 ă x 1 ă x (Zn is in excess, so [Zn] = ]

Zn 2 
K eq.    1x
Cu 
2  x
 

1x
  1.94  1037
x

1 1
 x 
37
1  1.94  10 1.94  1037

 x = 5.15 ï 10ă38 M Note that aZn = aCu = 1

ELECTROCHEMISTRY
50 QUIZRR
Example 24
The KSP of CuI is 1.1 ï 10ă12 M2. Determine EMF of the cell, represented as
Cu | CuI | Iă (1 M) || Cu+ (1 M) | Cu
Solution :
The anode of the given cell is of metal-insoluble metal salt-anion type half-cell while cathode is
of the type metal-metal ion. The half-cell reactions at anode are

Cu(s)  Cu+ eă ; G 1 = ă FEcu|Cu+

Cu+ + Iă 

 CuI(s) ; G 2 = 0

Net anode reaction : Cu(s) + Iă 



 CuI(s) + e
ă
; G 3 = ă FECu|CuI|Iă

According to HessÊs law, we know


G1 + G2 = G3
 G1 = G3 (as G2 = 0)
 ă FECu|Cu+ = ECu|Cu|Iă

Since, the E value of Cu  Cu+ + eă and E value of Cu + Iă  CuI + eă are same, the
representation of anode can be changed from metal-metal insoluble salt ă anion half-cell to metală
metal ion half-cell, provided the concentration of Cu+ in both the half-cells is same.
So, the complete cell can now be represented as

K 
Cu|Cu   SP  ||Cu  1 M |Cu

 [I ] 
Reactions occurring at the two electrodes are

At anode : Cu  Cu A  e


At cathode : CuC  e  Cu


Net cell reaction : CuC  Cu+A

Applying Nernst equation gives

Cu A 
In  
RT
Ecell  E   E  
Cu |Cu Cu |Cu F 
CuC 
C A
 

CuC 
Ecell = Ecell 
RT
ln  
F Cu A 
 

ELECTROCHEMISTRY
QUIZRR 51

CuC 
RT  
Ecell = F In Cu A  (as EĈcell = 0)
 

   
CuC  I 11
RT
In      0.059 log
Ecell =
F K SP 1.1  1012

Ecell = 0.705 V

15. CONCENTRATION CELLS

The cells whose EĈcell is zero are called concentration cells. This means that the two

compartments (cathode and anode) of the electrochemical cell involves same chemical species but
the concentrations of the chemical species in the two compartments are different. The concentration
cells are of basically two types.
(a) Electrode concentration cells and (b) Electrolyte concentration cells

1. Electrode Concentration Cells

In such cells, two similar electrodes at different concentrations/pressures are dipped in the same
solution with similar concentration. Let us have an electrochemical cell represented as
Pt | H2 (P1 atm) | H+ (c M) || H+ (c M) | H2 (P2 atm) | Pt
For the given cell, the reactions occurring are

At cathode : 2H+C  2e  H2 (P2 )

At anode : H2 (P1 )  2H+A  2e

Net cell reaction : H2 (P1 )  H2 (P2 )

Since the H+ concentration at the anode and cathode are same, so the net reaction is independent
of the concentration of the electrolyte. Applying Nernst equation to the net cell reaction gives

0.059 P 
Ecell  25C  E  E  log  2 
H |H2 (P2 ) H |H2 (P1 ) 2
C A  P1 

0.059 P 
Ecell  25C   Ecell  log  2 
2  P1 

ELECTROCHEMISTRY
52 QUIZRR

0.059 P 
or Ecell  25C   log  2  (since, EĈcell = 0)
2  P1 

0.059 P 
Ecell  25C   log  1 
2  P2 

The EMF of the given cell would be positive when P1 > P2 and the cell reaction would be
spontaneous.
Another example of the electrode concentration cell is that of an amalgam with two different
concentrations of the same metal dipped in same electrolyte solution.
The cell is represented as
HgăPb (c1 M) | PbSO4 (c M) | HgăPb (c2 M)
The reactions for the given cell are

At cathode : Pb2+(c) + 2eă  Pb(c2)

At anode : Pb(c1)  Pb2+(c) + 2eă

Net cell reaction : Pb(c1)  Pb(c2)

Since, the concentration of Pb2+ for the two half cells is same as the electrolyte solution for the
two compartments is same, so the net reaction is independent of the electrolyte concentration.
Applying Nernst equation to the net cell reaction gives

  0.059 c
Ecell(25ĈC) = EPb2 ( c)|Pb(c2 )  EPb2 ( c)|Pb(c1 )  log 2
2 c1

Ĉ 0.059 c
Ecell(25ĈC) = Ecell  log 2
2 c1

0.059 c
Ecell(25ĈC) =  log 2 (since EĈcell = 0)
2 c1

0.059 c
Ecell(25ĈC) =  log 1
2 c2

The net cell reaction would be spontaneous, when the EMF of the cell is positive, which is possible
only when c1 > c2.

ELECTROCHEMISTRY
QUIZRR 53

Example 25

Calculate the EMF of the electrode concentration cell


HgăZn (c1 M) | Zn2+ (c M) | HgăZn(c2 M)
at 25ĈC, if the concentration of the zinc amalgam are 2 g per 100 g of mercury and 1 g per
100 g of mercury in anode and cathode half cell respectively.
Solution :
The reactions at the two half cells are

At cathode : Zn2+(c) + 2eă  Zn(c2)

At anode : Zn(c1)  Zn2+(c) + 2eă

Net cell reaction : Zn(c1)  Zn(c2)

Applying Nernst equation to the net cell reaction gives

Ecell  Ecell 
0.059
2
c
log 2 
c1
0.059
2
c
log 1
c2
 since E Ĉ
cell 0 

0.059  2 / 65.4  3
Ecell  log    8.9  10 V
2  1 / 65.4 

2. Electrolyte Concentration Cells

In such cells, two electrodes of the same metal are dipped in solutions of metal ions of different
concentrations. Let us have an electrochemical cell represented as
Pt | H2(P atm) | HA(c1 M) || HB (c2 M) | H2 (P atm) | Pt
In such cells, HA and HB would represent strong acids, if their KaÊs are not given while they
would be weak acids, if their KaÊs are mentioned.
For the given cell, the reactions occurring are

At cathode : 2H c  c2   2e  H2 (P)

At anode : 
H2 (P)  2HA  c1   2e

Net cell reaction : 2Hc  2HA  n  2 

or Hc  HA

 n  1
The net cell reaction is independent of the pressure terms as the pressure of H2 in the two half
cells is same.

ELECTROCHEMISTRY
54 QUIZRR
Applying Nernst equation to the net cell reaction gives

HA 
E   E  
0.059
log    EĈ  0.059 log c1
Ecell(25ĈC) =  cell
H |H2
C
H |H2
A 1 HC  c2
 

c2
Ecell(25ĈC) = 0.059 log c (since EĈcell = 0)
1

The net cell reaction would be feasible spontaneously only when the EMF of the cell is positive,
which is possible only when concentration of H+ in cathode compartment (c2) is greater than the
concentration of H+ in anode compartment (c1).

Example 26

Calculate the EMF of the following galvanic cell


Zn | Zn2+ (0.01 M) || Zn2+ (0.1 M) | Zn at 298 K.
Solution :
The reactions at the two half cells are

2
At cathode : Zn C  2 e  Zn

At anode : Zn  Zn 2A  2e

Net cell reaction : Zn2C  Zn 2A

Applying Nernst equation to the net cell reaction gives

 Zn 2A 
log  
0.059
Ecell = EĈcell 
2  ZnC2 
 

ZnC 2 
0.059
log  
Ecell = 2 2 
 Zn A  (since EĈcell = 0)
 

0.059 0.1
Ecell = log  0.0295 V
2 0.01

Let us consider another electrochemical cell as


Ag | Ag2CrO4 (saturated soln.) || AgC (saturated soln.) | Ag

ELECTROCHEMISTRY
QUIZRR 55

and the solubility products of Ag2CrO4 and AgCl are (KSP) and (KSP)2 respectively. The saturated
solution of Ag2CrO4 and AgCl, each will give some [Ag+] and the [Ag+] in the two compartments
will not be the same. So, the given cell would be an electrolyte concentration cell. For the given
cell, the reactions occurring are


At cathode : Ag C  e  Ag

At anode : Ag  Zn A  e


Net cell reaction : Ag C  Ag A

Applying Nernst equation to the net cell reaction gives

 Ag A 
Ecell(25ĈC) = E   E   0.059 log  
Ag |Ag Ag |Ag 
 Ag C 
C A
 

 Ag A 
Ecell(25ĈC) = Ecell  0.059 log  
 Ag C
 


 Ag C
 
Ecell(25ĈC) = 0.059 log    (since EĈcell = 0)
Ag
 A

The [Ag+] in anode and cathode half-cells is written in terms of KSP as


  
Ag 2CrO4 (s)  2Ag  CrO4
2x x

Let ÂxÊ moles per litre be the solubility of Ag2CrO4.


 (Ksp)1 = [Ag+]2 [CrO42ă] = (2x)2x = 4x3

 3
 K sp 1
x
4

 Ag A   2 x  2  3
 K sp 1
  4

Similarly, let the solubility of AgCl be ÂyÊ moles/litre.

ELECTROCHEMISTRY
56 QUIZRR


  ă
 Ag y  Cl y
AgCl( s) 

(Ksp)2 = [Ag+] [Clă] = y2

 

y   Ag C

  K sp 2
 
Substituting the values of  Ag A  and  AgC  in the expression of Ecell gives

 K sp 2
Ecell  0.059 log
3
 K sp 1
2
4

 K sp 1
For the net cell reaction to be spontaneous,  K sp 2 has to be greater than 2  3
4
so that

the EMF of the cell would be positive.

Example 27

Calculate the potential of the cell,


Mn(s) | MnCl2 (0.001 M) | HCl (0.01 M) | O2 (0.25 atm) | Pt.
Given that EĈ = ă 1.185 V for Mn2+ | Mn couple and 1.229 V for the O2 | H2O, H+ couple.
Solution :
We can replace the cathode of the given cell by the half cell, O2|H2O, H+ because we have
already learnt in metal-insoluble salt-anion electrode that the potential of the two half cells is
same.
Thus, the cell representation becomes Mn(s) | MnCl2 (0.001 M) || H+ (0.01 M), H2O | O2 (0.25
atm) | Pt

At cathode : Mn  Mn2+ + 2eă

At anode : 2H+ + ó
O2 + 2eă  H2O

Net cell reaction : Mn + 2H+ + ó


O2  Mn2+ + H2O

Mn2 
 Ecell = E  E 2+ 
0.059
log  
O2|H2O,H 
 
Mn |Mn 2 2 ó

H  PO
  2

0.059 103
 Ecell = 1.229    1.185   log
10 
2 2
2
 0.25

Ecell = 2.37 V

ELECTROCHEMISTRY
QUIZRR 57

Example 28

By how much is the oxidising power of the MnO 4| Mn 2  couple decreased if the H+
concentration is decreased from 1 M to 10ă4 M at 25ĈC. Assume that the concentration of
other species do not change.
Solution :

In acidic medium, MnO4 acts as oxidizing agent and reduces to Mn2+ as per the reaction

MnO4  8H  5 e  Mn 2   4H2O

Mn2 
E  E 
0.059
log  
|Mn2
MnO4 |Mn 2
MnO4 8
5 MnO4  H  
  

Mn 2 
E  2 
0.059
log  
= MnO4|Mn 5 MnO4  (1)8
 

Mn2 
E´  E 
0.059
log  
|Mn2 
MnO4 |Mn 2
MnO4 8
5 MnO4  10 4 

  

 MnO4  (1)8 Mn2   


E ´  0.059      
  MnO|Mn 2  EMnO|Mn 2    log   8
 0.3776 V
 
 
5 2 
4 4
 
Mn
 MnO4  104
 
   

Thus, the oxidizing of MnO4 |Mn2 couple decreases by 0.3776 V from its standard value.

Example 29

EMF of the following cell is 0.67 at 298 K.


Pt | H2 (1 atm) | H+ (pH = X) || KCl (1 N) | Hg2Cl2(s) | Hg

Calculate pH of the anode compartment. Given : E  0.28 V


Cl |Hg 2Cl 2|Hg
Solution :
The reactions occurring in the electrochemical cell are

ELECTROCHEMISTRY
58 QUIZRR

At anode : H2  2H+ + 2eă

At Cathode : Hg2Cl2 + 2eă  2Hg + 2Clă

Net cell reaction : H2 + Hg2Cl2  2H+ + 2Hg + 2Clă

2 2
Cl  H  
log    
0.059
 Ecell  E  E 
Cl |Hg 2 Cl2|Hg H |H2 2 PH2

(Hg and Hg2Cl2 do not appear as they are pure liquid and pure solid respectively)

2
H    (1)2
log  
0.059
0.67  0.28   0.28  0.059 log  H 
2 1  

0.67 = 0.28 + 0.059 pH


 pH = 6.61

Example 30
The EMF of the cell, Hg | Mercurous nitrate (0.01 M) || Mercurous nitrate (0.1 M) | Hg was
found to be 0.0295 V at 25ĈC. Calculate the molecular formula of mercurous nitrate.
Solution :
Let the formula of mercurous nitrate be Hgn(NO3)n.
For the given cell, the reactions occurring at two electrodes are

At anode : nHg  Hg nn    A


 ne

At cathode :  Hg 
n
n
C
 ne  nHg

Net cell reaction :  Hg 


n
n
C   Hg  n
n
A

So, this is an electrolyte concentration cell for which EĈcell = 0. The Ecell will be given as

 n
 Hg n


log  A
0.059
 Ecell = 
n
 n
 Hg n

C

0.059
 Hg n
 n  

log  C 0.059 0.1
 0.0295   log
n  Hg n 
 n  
A
 n 0.01

 n = 2
Thus the formula of mercurous nitrate is Hg2(NO3)2.
ELECTROCHEMISTRY
QUIZRR 59

Example 31

It is desired to construct the following voltaic cell to have Ecell = 0.0860 V. What [Clă] must
be present in the cathode half cell to achieve this result ?
Ag(s) | Ag+ (satd. AgI (aq)) || Ag+ (satd. AgCl, x M Clă) | Ag+(s)
Ksp [AgCl = 1.8 ï 10ă10, AgI = 8.5 ï 10ă17]

E  0.80 V
Ag  / Ag

Solution :

Anode Ag(s)  Ag+ (aq) + eă E   0.80 V


Ag  / Ag+

anode

Cathode Ag+(aq) + eă  Ag(s) E   0.80 V


Ag  / Ag

cathode

Ag+ (aq)  Ag+ (aq) EĈcell  0.00 V


cathode anode
+
Ag (anode) is from AgI (satd, aq)
 AgI(s)  Ag+(aq) + Iă (aq)
Ksp = [Ag+] [Iă] = [Ag+]2

 [Ag +]AgI = [Ag+]L.H.S. = K sp

= 8.5  1017

= 0.9220 ï 10ă8 M
Ag+ (cathode) is from AgCl in presence of [Clă] = x M
AgCl(s)  Ag+(aq) + Clă (aq)
 Ksp = [Ag+] [Clă]

K sp 1.8  1010
+
[Ag ] R.H.S. = 
[Cl  ] x

 0.0591
Ecell = Ecell  log K
n

 Ag  
0.0591   L.H.S.
= 0 log 
1 [Ag ]R.H.S.

ELECTROCHEMISTRY
60 QUIZRR

0.9220 x  108
0.0860 =  0.0591 log
1.8  1010

0.0860
 = log 51.22 x
0.0591

ă 1.4562 = log 51.22 + log x


= 1.7095 + log x
 log x = ă 3.1647
x = 7 ï 10ă4 M
[Clă] = 7 ï 10ă4 M

Example 32

Calculate the emf of the following cells at 298 K.


(i) Fe | Fe2+ || Sn2+ | Sn
0.3 M 0.1 M
(ii) Pt (H2) | HCl | Pt (H2)
2 atm 10 atm

E   0.44 V , E   0.14 V
Fe 2  / Fe Sn 2  / Sn

ES.H.E.  0.00 V

Solution :

(i) Fe  Fe2+ + 2eă Eox = EFe / Fe2   0.44 V


Sn2+ + 2eă  Sn Ered = ESn 2+ / Sn   0.14 V

Sn2+ Fe  Sn + Fe2+ Ecell = 0.30 V

Fe2  
  0.3
K =  2   0.1  3
Sn
 

 Ecell = Ecell  0.0591 log K


n

= 0.30  0.0591 log 3


2

ELECTROCHEMISTRY
QUIZRR 61

0.0591
= 0.30   0.4771
2

= 0.30 ă 0.0141 = 0.2859 V


(ii) Pt (H2) | HCl | Pt (H2)
2 atm 10 atm
This is concentration cell in which hydrogen electrodes at different pressures are dipped in
HCl

at L.H.S. half cell H2 (2 atm)  2H+ + 2eă Eox = 0.00 V

at R.H.S. half cell 2H+ + 2eă  H2 (10 atm) Ered = 0.00 V


Net H2 (2 atm)  H2 (10 atm)

PH2  R.H.S. 10
K = P = 5
H2  L.H.S.  2

 0.0591
 Ecell = Ecell  log K
n

0.0591
= 0.00  log 5
2

= 0.0207 V.

Example 33

The emf of the cell is 0.788 V


Ag | AgI, 0.05 M KI || 0.05 M AgNO3 | Ag
Calculate the solubility product of AgI.

E  Eredn  0.80 V
Ag  / Ag

Solution :
KI is strong electrolyte, hence
[Iă]L.H.S. = 0.05 M
AgI(s) is sparingly soluble. If we manage to calculate Ag+ (Ag) in L.H.S. half cell, Ksp can be
calculated.

at Anode half cell Ag(s)   Ag+ (x M) + eă Eox = ă 0.80 V


anode

ELECTROCHEMISTRY
62 QUIZRR

at Cathode half cell Ag+(0.05) + eă  Ag(s) Ered = 0.80 V


cathode

Net Ag+ (0.05)  Ag+ (x M) EĈcell = 0.00 V

 Ag  
 anode x

K =   0.05
Ag
  cathode

 0.0591
 Ecell = Ecell  log K
n

0.0591  x 
0.788 = 0  log  
1  0.05 

 x 
log   = ă 13.3333 = 14.6667
 0.05 

x
= 4.6416 ï 10ă14
0.05

x = 2.321 ï 10ă15
 [Ag +] L.H.S. = 2.321 ï 10ă15 M
[I ă] L.H.S. = 0.05 M
 Ksp = [Ag+] [Iă]
= 2.321 ï 10ă15 ï 0.05
= 1.16 ï 10ă16

Example 34

When silver chloride is dissolved in a large excess of ammonia, practically all silver ion can
be assu m ed t o ex i st i n f or m of a si n gl e i on i c speci es [ Ag x (NH3)y]x+. Compute the values of
x and y using the following two cells
(A) Ag | 0.4 ï 10ă3 M AgCl, 1M NH3 || 40 ï 10ă3 M AgCl, 1 M NH3 | Ag
Ecell = 0.1185 V at 298 K
(B) Ag | 3 ï 10ă3 M AgCl, 1 M NH3 || 3.0 ï 10ă3 M AgCl, 0.1 M NH3 | Ag
Ecell = 0.1263 V at 298 K
Solution :
All the Ag+(aq) from AgCl in each of the half cells is complexed to form [Agx(NH3)y]x+

ELECTROCHEMISTRY
QUIZRR 63

hence [AgCl] = [Ag+] = [Agx(NH3)y]x+


for the given cell
Electrode Reaction EĈ

Anode x Ag + y NH3  Agx(NH3)yx+ + xeă Eox = EV


Anode Anode

Cathode Agx(NH3)yx+ + xeă  xAg + yNH3 Ered = ă E V


cathode cathode

Net Agx(NH3)yx+ + yNH3   [Agx(NH3)y]x+ + yNH3 Ecell = 0.00 V


cathode Anode Anode cathode

 Agx  NH  x  y
3 y  anode  NH3 cathode
 K =  
 Agx  NH  x  y
3 y  cathode  NH3 anode
 

(0.4  10 3 ) (1) y 1
in cell (A) K = 
 40  10  (1)
3 y 100

Ecell = 0.1185 V

 0.059
Ecell = Ecell  log K
n

0.0591  1 
0.185 = 0  log  
x  100 

0.0591  2
=
x

this give x= 1
In cell (B) using eq. (1),

3  10  (0.1)
3 y
 1 
 
y

K =
 3  10  (1)
3 y  10 

Ecell = 0.1263 V, x = 1

ELECTROCHEMISTRY
64 QUIZRR

0.0591
 0.1263 = 0  log (10) y
1

= + y ï 0.0591
y= 2
Hence complex is [Ag(NH3)2]+

Example 35

Calculate EĈ of the following half cell reaction at 298 K


Ag(NH3)2+ + eă  Ag + 2NH3

Given Ag+ + eă  Ag, EAg  / Ag = 0.80 V

Ag(NH3)2+  Ag+ + 2NH3, K = 6 ï 10ă8


Solution :
The cell uses Ag/Ag+ electrodes as reference half cell.
Half cell Reaction EĈ

L.H.S. half cell Ag(s)  Ag+ + eă E = 0.80 V


Ag / Ag 

R.H.S. half cell Ag(NH3)2+ + eă  Ag(s) + 2NH3 EĈ = x V

Net Ag(NH3)2+  Ag+ + 2NH3 = (x ă 0.80)


EĈcell

[Ag  ]  NH3 
2
 6  108
K =  Ag NH   (given)

 3 2 

at equilibrium Ecell = 0.00 V; K = Keq

0.0591
 Ecell = log K eq
n

0.0591
(x ă 0.80) = log 6  108
1

= ă 0.4268 V
x = 0.80 ă 0.4268
= 0.3732 V
Hence EĈ of the hall cell reaction
Ag(NH3)2+ + eă  Ag + 2NH3 is 0.3732 V

ELECTROCHEMISTRY
QUIZRR 65

Example 36

Find the solubility product of a saturated solution of Ag2CrO4 in water at 298 K, if the EMF
of the cell, Ag | Ag+ (Satd. Ag2CrO4 solution) || Ag+ (0.1 M) | Ag is 0.164 V at 298 K.
Solution :
For the given cell, the reactions occurring at the anode and cathode are

At anode : Ag  Ag A + eă

At cathode :  + eă
Ag C  Ag

Net cell reaction :  


Ag C  Ag A
Thus, it is an electrolyte concentration cell with EĈ cell = 0

 Ag A   Ag C
 0.059   0.059  
 Ecell  log  log
1  Ag C 1  Ag A 
   

The anode compartment have saturated solution of Ag2CrO4, supplying Ag+ ion concentration.
Let the solubility of Ag2CrO4 be x moles/litre.


  2
 2Ag  CrO4
Ag 2CrO4 (s) 
2x x

2
K SP   Ag A  CrO24     2 x x  4 x3
2
   

  Ag A   2 x  2  3 K SP
  4

0.059 0.1
Ecell  0.164  log
 1 K SP
23
4

 KSP = 2.29 ï 10ă12 M3

Example 37

An acidic solution of Cu2+ salt containing 0.4 g of Cu2+ is electrolysed until all the Cu is
deposited. The electrolysis is continued for seven more minutes with the volume of solution
kept at 100 ml and the current at 1.2 ampere. Calculate the volume of gases evolved at NTP
during entire electrolysis.
Solution :
The problem does not mention about the acidic salt of Cu2+ i.e. what kind of acidic salt is this.
ELECTROCHEMISTRY
66 QUIZRR
Does the acidic salt have chloride, sulphate or nitrate as anion against the cation Cu2+ and the
answer will solely depend on the assumption we make in the beginning about the acidic salt. First,
let us assume that the salt is cupric chloride (CuCl2).
In the I part of electrolysis, the reactions occurring at the two electrodes are

At cathode : Cu2+ + 2eă  Cu

At anode : 2Clă  Cl2 + 2eă

[The reaction occurring at anode is the oxidation of Clă in preference to OHă since the standard
oxidation potential of Clă > OHă]

0.4
Moles of Cu2+ reduced at cathode =
63.5

2  0.4
Mole of electrons required at cathode = = Mole of electrons released at anode
63.5

2  0.4  1 0.4
 Mole of Cl2 liberated at anode = 
63.5  2 63.5

0.4
Volume of Cl2 liberated at STP at anode =  22400 = 141 ml
63.5

In second part of electrolysis, when current is passed for 7 more minutes, the H+ will be reduced
at cathode since Cu2+ ions are discharged completely and OHă ions are oxidized at anode since
Clă is also completely oxidized. The reactions occurring are

At cathode : 2H+ + 2eă  H2

At anode : 2OHă  H2O + ó


O2 + 2eă

1.2  7  60
Moles of electrons passed =
96500

1.2  7  60
Volume of H2 at STP released at cathode = 96500  2  22400  58.49 ml

1.2  7  60
Volume of H2 at STP released at anode =  22400  29.245 ml
96500  4

 Total volume of gases (Cl2 + O2 + H2) liberated at STP during antire electrolysis
= 141 + 59.49 + 29.245 = 229.735 ml
Second, let us assume that the salt is that of CuSO4.

ELECTROCHEMISTRY
QUIZRR 67

In part I of electrolysis, the ions discharged at cathode and anode are Cu2+ and OHă respectively.

At cathode : Cu2+ + 2eă  Cu

At anode : 2OHă  H2O + ó


O2 + 2eă

2  0.4 0.4
 Mole of O2 liberated at anode = 63.5  4  63.5  2

0.4
 Volume of O2 at STP liberated at anode =  22400  70.55 ml
63.5  2

In part II of electrolysis, the H+ and OHă ions are discharged at cathode and anode respectively.

At cathode : 2H+ + 2eă  H2

At anode : 2OHă  H2O + ó


O2 + 2eă

1.2  7  60
Volume of H2 at STP released at cathode = 96500  2  22400  58.49 ml

Volume of O2 at STP released at anode = 29.245 ml


 Total volume of gases (H2 + O2) released at STP during entire electrolysis
= 70.55 + 58.49 + 29.245 = 158.285 ml

16. Selection of an oxidising or reducing agent

Choosing an oxidising agent

It is easier to explain this with a specific example. What could you use to oxidise iron (II) ions
to iron (III) ions ? The EĈ value for this reaction is :

Fe3+(aq) + eă 

 2+
 Fe (aq) EĈ = + 0.77 V

To change iron (II) ions into iron (III) ions, you need to persuade this equilibrium to move to the
left. That means that when you couple it to a second equilibrium, this iron EĈ value must be the
more negative (less positive one).
You could use anything which has a more positive EĈ value.
Dilute nitric acid :

NO3(aq) + 4H+(aq) + 3eă 



 NO(g) + 2H2O(I) EĈ = + 0.96 v

Acidified potassium dichromate(VI) :

Cr2O72ă(aq) + 14H+(aq) + 6eă 



 3+
 2Cr (aq) + 7H2O(I) EĈ = + 1.33 v

ELECTROCHEMISTRY
68 QUIZRR
Chlorine :

Cl2(g) + 2eă 



 ă
 2Cl (aq) EĈ = + 1.36 v

Acidified potassium managanate (VII) :

MnO4ă(aq) + 8H+(aq) + 5eă 



 2+
 Mn (aq) + 4H2O(I) EĈ = + 1.51 v

Choosing a reducing agent

Remember :
Ć Reduction is gain of electrons.
Ć A reducing agent reduces something by giving electrons to it. That means that the reducing
agent loses electrons.
You have to be a little bit more careful this time, because the substance losing electrons is found
on the right-hand side of one of theses redox equilibria. Again, a specific example makes it clearer.
For example, what could you use to reduce chromium (III) ions to chromium (II) ions ? The EĈ
value is :

Cr3+(aq) + eă 

 2+
 Cr (aq) EĈ = 0.41 v

You need this equilibrium to move to the right. That means that when you couple it with a second
equilibrium, this chromium EĈ value must be the most positive (least negative).
In principle, you could choose anything with a more negative EĈ value - for example, zinc :

Zn2+(aq) + 2eă 



 Zn(s) EĈ = ă 0.76 v

You would have to remember to start from metallic zinc, and not zinc ions. You need this second
equilibrium to be able to move to the left to provide the electrons. If you started with zinc ions,
it would already be on the left - and would have no electrons to give away. Nothing could possibly
happen if you mixed chromium (III) ions and zinc ions.
That is fairly obvious in this simple case. If you were dealing with a more complicated equilibrium,
you would have to be careful to think it through properly.

17. CELLS
For an electrochemical cell to be used as a commercial cell. It must
1. Be compact, light and rugged.
2. Voltage should not drop during use.

Battery

A number of cells connected in series forms a battery. There are three types of cells :

ELECTROCHEMISTRY
QUIZRR 69

1. Primary cells
In these the redox reactions occur only once and cannot be used again.
Example : dry cells, mercury cells.

2. Secondary cells
These can be recharged by passing current and can be used again and again.
Example : Lead storage battery, Ni-Cd storage cell.

3. Fuel cell
Energy produced by combustion of fuels like H2, CO, CH4 can be directly converted to electric
energy.

Primary cells

Dry cells
It is the compact form of the Leclanche cell.
Anode - Cylindrical Zn container.
Cathode - Central Graphite rod.
The space in between is filled with NH4Cl and ZnCl2. The graphite rod is surrounded by MnO2
and carbon.

Reactions :

 
Anode : Zn(s)  Zn2(aq )  2e

Cathode : 2 MnO2(s) + 2NH4+(aq) + 2eă  Mn2O3(s) + 2NH3 + H2O

Voltage : 1.25 to 1.5 V


It does not have a long life because the acidic NH4Cl corrodes the Zinc container.

Metal cap

(+) Pitch seal

Cardboard
cover
Zinc container
(Anode)
Graphite
NH4Cl + ZnCl2 (Cathode)
MnO2 + C
(ă)

ELECTROCHEMISTRY
70 QUIZRR
S econdary Cell

Lead storage battery


Anode - Lead
Cathode - PbO2
The electrodes are arranged alternately, separated by thin wooden or fibre glass sheet.
Electrolyte - dilute H2SO4.
Electron reaction (during discharging)

2 
Anode : Pb s  SO4  Pb SO4  2e

Cathode : PbO2  SO42  4H   2e  Pb SO4  H2O

Over all : Pb( s)  PbO2  4H   2SO42  2Pb SO4  2H2O

During charging, reverse reactions take place

PbSO4  2e  Pb(s)  SO24

PbSO 4  2H2O  PbO 2  SO 24  4H  2e

Over all 2PbSO4  2H2 O  Pb( s)  PbO2( s)  4H(aq)  2SO4(


2
aq)

Nickel Cadmium storage cell

Used mainly in calculators


Anode - Cadmium
Cathode - NiO2

Reactions

Anode : Cd(s) + 2OHă(aq)  Cd(OH)2 + 2eă

Cathode : NiO2 + 2H2O + 2eă  Ni (OH)2 + 2OHă

can be recharged.

Mercury Cell

Used in small electric devices like hearing aids and watches.


Anode ă Zinc container

ELECTROCHEMISTRY
QUIZRR 71

Cathode ă Carbon rod


Electrolyte ă moist HgO mixed with KOH

Reactions :

Anode : Zn(s) + 2OHă  ZnO(s) + H2O(I) + 2e

Cathode : HgO(s) + H2O(I) + 2e  Hg(I) + 2OHă

Overall reaction : Zn(s) + HgO(s)  ZnO(s) + Hg(I)

The cell shows a constant potential of 1.35 V throughout as it does not involve any ion whose
concentration changes.

Fuel cell

These are devices which convert the energy produced during the combustion of fuels like H2, CO
and CH4 directly into electrical energy. The most successful fuel cell is the H2-O2 fuel cell. It was
used in the Apollo space programme and the water produced used as drinking water for the
astronauts.

Working

H2 and O2 are bubbled through a porous carbon rod into concentrated NaOH. Catalysts are
present in the electrodes.

Anode : 2[H2(g) + 2OHă(aq)  2H2O + 2e]


Output
Cathode : O2(g) + 2H2O(I) + 4eă  4OHă(aq)

Overall : 2H2(g) + O2(g)  2H2O(aq) eă


(ă) (+)
H2(g) O2(g)

Anode Cathode

H2(g) O2(g)

Electrolyte
KOH (aq)

ELECTROCHEMISTRY
72 QUIZRR
A dvantages

(1) More efficient than conventional cells because the energy of the fuel is converted directly
into electric energy. Efficiency = 60-70%
(2) They are free from pollution.

Example 38
During the discharge of a lead storage battery, the density of H2SO4 falls from 1 g/cc to 2g/
c.c.H2SO4 of density of 1 g/c.c. is X% by weight and that of density of 2 g/c.c. is Y% by
weight. The battery holds Vlitre of acid before discharging. Calculate the total charge
released at anode of the battery. The reactions occurring during discharging are

At anode : Pb  SO 24  PbSO  2e 

At cathode : PbO 2  4H  + SO 24  2e   PbSO 4  2H 2 O


Solution :
Mass of acid solution before discharge of lead storage batter (LSB) = (V ï 103 ï 1) g
= (1000 ï V1) g

 X 
=  1000  V1  g
100 
Mass of H2SO4 before discharge of LSB

= (10 ï V1X)g

Net reaction during discharging : Pb + PbO2 + 2H2SO4  2PBSO4 + 2H2O

From the reaction, it is evident that the moles of electron exchanged (lost at anode and gain at
cathode) is equal to the moles of H2SO4 consumed or moles of H2O produced. Let the moles of
H2SO4 produced be x, then
Mass of H2O produced during discharge of LSB = (18x)g
Mass of H2SO4 consumed during discharge of LSB = (98x)g
 Mass of H2SO4 after discharge of LSB = [(10 V1X) ă 98x]g
Mass of acid solution after discharge of LSB = [(1000 V1) ă 98x + 18x] = [(1000 V1) ă 80x]g

Mass of H2SO4 after discharge


 % of H2SO4 after discharge of LSB =  100
Mass of acid solution after discharge

Y
(1000  V1 X)  98 x  100
1000  V1   80 x

x can be calculated as all other quantities are known.


 Total charge released at cathode, Q = nF = xF

ELECTROCHEMISTRY
QUIZRR 73

Example 39
During the discharge of a lead storage battery, the density of sulphuric acid fell from 1.294
g/m to 1.139 g/m. Sulphuric acid of density 1.294 g/m was 39% H2SO4 by wt. while acid of
density 1.139g/m contains 20% acid by wt. The battery holds 3.5 lts of acid and the value
remained practically same through the discharging. Calculate the number of amp/hr for
which the battery must have been used. The charging and discharging reactions are :

PbO 2  SO24   4H   2 e   PbSO 4  2H 2 O (discharging)

PbSO 4( s)  2 e   Pb(s) + SO 24 (charging)

Solution :
Note that density of sulphuric acid has decreased, i.e., discharging of battery takes place. First
writing both the reaction for discharging.

Anode : Pb(s) + SO24   PbSO4 (s) + 2e

Cathode : PbO2 + SO24  4H  2 e  PbSO4 + 2H2O

Overall reaction : Pb(s) + 2H2SO4 + PbO2  PbSO4(s) + 2H2O

 for the consumption of 2 molecules of H2SO4, 2e are transferred


 for the consumption of 2 moles of H2SO4, 2 moles of eÊs are transferred
 2 mole of H2SO4  2F of electricity

Now let us calculate the decrease in moles of H2SO4 from the data given.
volume of solution = 3.5 l = 3500 m
Let the mass of H2SO4 before discharging = mi
and the mass of H2SO4 after discharging = mf

39  3500  1.294
 mi = = 1766.31 gm
100

20  3500  1.139
 mf = mi  = 7973 gm
100

 m (the decrease in mass) = 1767.31 ă 797.3 = 970.01 gm

m 970.01
 moles of H2SO4 consumed =   9.898
98 98

From the equation, we get 1 mol of H2SO4  1 F of charge


 9.898 moles  1 ï 9.898 F of charge
 9.898 ï 96500
 955162.9 C of charge

ELECTROCHEMISTRY
74 QUIZRR
Now
Q = It

Q
 I=
t

955162.9
 I  265.32 amp/hr
1  60  60

Example 40

In the cell,
Tl | Tl + (0.1 M) || Sn+2 (0.01 M) | Sn
a current of 40 mA is flowing from tin to thallium electrode. An external battery of 1.2 V
emf is connected to the cell so that its polarity is opposite to the natural polarity of the cell.
If 0.38 g of thallium is deposited in one hour at 25ĈC, determine efficiency of thallium
electrode. (Atomic mass of Tl = 204)
Solution :
For reversible cells, if an external voltage of opposing polarity is attached to the cell, a current
flowing from thallium to tin electrode is given as :

E  Eb
I= ...(i)
R

where E = Applied voltage = 1.2 V, Eb = Back emf (i.e. reversible emf of cell) and R = Cell
resistance.
Calculation of R :
For cell : Tl | Tl+ (0.1 M) || Sn+2 (0.01 M) | Sn

 Tl  
Eb  Ecell  Ecell 
0.0591
log     0.340  0.140   0.059 log
0.1
1 1/ 1/
Sn 2  2
 0.01 2
 

 Eb = 0.2 V

Ecell E 0.2
I  R  cell 
R I 40  103

R = 5 
Using the value in equation (i)

1.2  0.2
I  0.2 A
5

ELECTROCHEMISTRY
QUIZRR 75

Charge passed for 1 hour = 0.2 ï 3600 C = 720 Coulomb

720
Equivalent of thallium deposited =
96500

720
Weight of thallium deposited =  204  1.52 g
96500

0.38
Efficiency of thallium electrode =  100  25%
1.52

ELECTROCHEMISTRY
SOLUTIONS
QUIZRR 3

SOLUTIONS

Introduction

A „solution‰ is defined as a homogeneous mixture of two or more substances (called as compo-


nents), the relative proportion or composition of which may vary between certain limits.

Types of Solutions

Following types of solutions are generally considered


(a) gas in gas (b) liquid in liquid
(c) gas in liquid (d) solid in liquid
The component of a particular type of solution which is present in the greater quantity is called
solvent while the other component(s) is called the solute. However, in the case of a solution
formed from a solid and a liquid, the liquid is always referred to as the solvent even though there
may be the cases when the solid is present in the larger quantity.
A solution is unsaturated if it can still dissolve more solute at the same temperature; when it
can dissolve no more of the solute at the same temperature it is said to be saturated.
It is possible to obtain supersaturated solution, i.e., solutions which contain more dissolved
solute than they should at a particular temperature, by cooling a hot concentrated solution of a
solid in a liquid in the absence of traces of solids (even dust). In order to ensure that a particular
solution is saturated but not supersaturated there must be a small amount of excess solute
present.
Ć Supersaturated solutions are not in equilibrium with the solid substance. Thus, if a small
crystal of solute (say sodium thiosulphate) is added to its supersaturated solution, the excess
immediately crystallises out, which is usually quite fast.

Kind of solution Example


1. Gas in gas Air (O2, N2, Ar and other gases)
2. Gas in liquid Carbonated water
3. Gas in solid H2 in platinum metal (adsorption)
4. Liquid in liquid Mixture of hydrocarbons (gasoline)
5. Liquid in solid Dental amalgam (Hg in Ag)
6. Solid in liquid Sea water (NaCl and other salts in water)
7. Solid in solid Metal alloys such as 14ăcarat gold (Au and Ag)

Note : Closely related to weight percent and particularly useful for dilute solutions are the
concentration units parts per million (ppm) and parts per billion (ppb).

SOLUTIONS
4 QUIZRR

mass of solute component


Parts per million (ppm) =  106
total mass of solution

mass of solute component


Parts per billion (ppb) =  109
total mass of solution

VAPOUR PRESSURE

When a liquid is placed in a closed container, it evaporates and


converts to vapour. After some time, the liquid and the vapour
reaches equilibrium, i.e., the rate of evaporation becomes equal to
the rate of condensation. At this point of time the pressure exerted
by the vapours of the liquid on the surface of the liquid and on the
walls of the container is called the vapour pressure of the liquid.
Does the vapour pressure of a liquid depend on the volume of the
vessel, shape of the vessel or the amount of liquid taken in the
container ?
These questions can be answered by invoking the concept of equilibrium constant. For the equi-

librium, liquid 
 vapour, the Kp for the equilibrium would be = Pvap, since this is the
pressure exerted by the vapours at equilibrium (by definition). We know Kp depends only on
temperature and the nature of substance. This means that vapour pressure of a liquid will also
be constant at a given temperature and will depend on temperature and the nature of liquid.
Therefore, it should be noted that whatever be the volume or shape of the vessel, or the amount
of liquid in it (assuming it to be more than the minimum amount required), the vapour pressure
of a liquid is constant at a given temperature and changes only with the change of temperature
of the liquid.
Moreover, on addition of any substance to the liquid, as long as the substance does not dissolve
in the liquid and completely covers the surface, the vapour pressure of a liquid remains constant.

Addition of a Non-Volatile Solute to a Volatile Solvent

When a non-volatile solute is added to a liquid (solvent), it is found that the vapour pressure of
the solution is less than the vapor pressure of the pure liquid. This is because of the following
two reasons
1. Attraction between the solute and solvent molecules
2. Reduction in the number of solvent molecules per unit area of the surface.
Let us understand these two reasons.
Soluteăsolvent attraction : When the solute is non-volatile, it is always invariably a solid. The
reason a solid would dissolve in a liquid is the attraction that the solvent has for the molecules/

SOLUTIONS
QUIZRR 5

ions of the solute. Therefore, there is a strong solute-solvent attraction present in the solution.
Moreover, this attraction would be dependent on the nature of solute, as different solutes will
have different extent of attraction with the same solvent. To do away with the dependency on
the nature of solute, we would like this attraction to have negligible impact on the vapour
pressure of a solution.
Reduction in the number of solvent molecules : When a non-volatile solute is dissolved in
a liquid, it distributes itself homogenously throughout the solution. Therefore, it occupies the
surface of the liquid also. This reduces the number of solvent molecules present per unit area on
the surface. This further diminishes the number of vapour molecules that are in equilibrium with
the molecules of the liquid present per unit area on the surface. Thus, the vapour pressure gets
reduced.
The first cause would be dependent on the nature of solute and the second cause is independent
of the nature of solute. To eliminate the first cause, we propose that those solutions in which the
solvent content is very high and the solute content is very low (highly dilute solutions) are the
ones in which the nature of solute plays an insignificant role towards lowering the vapour
pressure of a solution. This is because the solute amount being very small, the change in the
vapour pressure on changing the nature of the solute would also be small. Such infinitely dilute
solutions are called ideal solutions. In these solutions, the lowering of vapour pressure depends
only on the amount of solute and not on its nature.

R AOULT’S LAW

The partial vapour pressure of a component in such type of solution is given by RaoultÊs Law.
It states that the partial vapour pressure of each component in the solution is directly proportional
to its mole fraction.
Let A and B be the mole fractions of solvent (A) and solute (B) molecules in the solution and
PA and PB be the partial pressures of A and B respectively. Then according to RaoultÊs Law :
PA  A and
PB  B
From the results of various experiments, it has been observed that :
PA = PĈ A A and
PB = PĈ B B
PĈ A and PĈ B are vapour pressures of pure A and B.
Now the total vapour pressure (P) of the solution is given by DaltonÊs Law of partial pressures
as follows:
P = PA + PB
 P = PĈ A A + PĈ B B

SOLUTIONS
6 QUIZRR
For a solution of different compositions made from A and B, the plot of PA, PB against A and
B is shown as follows :

I n the graph

Two dotted lines 1 and 2 show variations in partial pressure


P PBO
3
with increasing mole fraction and the solid line 3 gives the
total vapour pressure P exerted by the solution as a whole PAO 2
at any composition.
1
PB
Note : The solutions which obey RaoultÊs Law at all compo-
PA
sitions of solute in solvent are called as Ideal Solutions.
The plot of partial pressure Vs mole fraction of such solution
is typical and in general like the graph shown here.
When solute is non-volatile : In such a solutions, the mole fraction (X)
XB = 0 XA = 0
surface molecules of only solvent will escape and contribute
to the vapour phase i.e., there is no contribution from solute in the vapour phase. The vapour
pressure of such a solution is equal to the partial vapour pressure of solvent molecules (= PA).
P = PA = PĈ A A (from RaoultÊs law)
For binary solution : A + B = 1  A= 1 ă B

PA0  PA
  B
PA0

PA0  PA
The expression is called as Rela tive Lowering of Va pour Pressure for a solution.
PA0

Another form of RaoultÊs Law can now be stated as :


Relative Lowering of vapour pressure for a solution containing non-volatile solute is equal to the
mole fraction of the solute.

Ideal Solution

Ć An ideal solution consisting of two components A and B is one in which the intermolecular
attraction A ... A, B ... B and A ... B are equal. For such cases,

Hm = 0, Vm = 0 (m stands for mixture)

In such cases there is no deviation from RaoultÊs law

Ć If attraction A ... B are greater than average of A ... A and B ... B attraction then vapour
pressure of the mixture is less than the expected value. In such cases, Vm = ă ve,

SOLUTIONS
QUIZRR 7

Hm = ăve,and there is ă ve deviation from RaoultÊs law. Boiling point in such cases is larger
than expected.

Ć If A ... B attractions are smaller than the average of A ... A and B ... B attractions
then vapour pressure of the mixture is larger than the expected value. For such cases,
Vm = + ve, Hm = + ve, and there is + ve deviation from RaoultÊs law.

Boiling point in such cases is smaller than expected.

P oA
V.P.
l
P oB P tota

PA

PB
0 XA 1
1 XB 0

 
 Vapour pressure diagram for real solutions of two liquids A and B that exhibit a negative 
 
 deviation from RaoultÊs law. The vapour pressure of both A and B are less than predicted 
 
 by RaoultÊs law. The dashed lines represent the plots for ideal solutions. 

A mixture of chloroform and acetone forms a solution with negative deviation from RaoultÊs
law. Chloroform (CHCl3) molecule is hydrogen-bonded to acetone (CH3COCH3) as shown :

CH3 Cl
C O H·C Cl
CH3 Cl

This results in decrease of escaping tendency of molecules for each component and thus
vapour pressure decreasesăa case of negative deviation.

Mixture of ethanol and acetone forms a solution with positive deviation. Ethanol molecules
are H-bonded. If acetone is added, these molecules get in between ethanol molecules and
thus H-bonding breaks. This changes the intermolecular pattern and escaping tendency of
ethanol and acetone molecules increases and thus vapour pressure increasesăa case of
positive deviation.

SOLUTIONS
8 QUIZRR
Deviation from RaoultÊs Law
Positive Negative Zero
deviation deviation deviation
H = + ve)
( (H = ă ve) (H = 0)
(i) ethanol + cyclohexane (i) acetone + chloroform (i) benzene + toluene
(ii) acetone + carbon (ii) benzene + chloroform (ii) n-hexane + n-heptane
disulphide
(iii) acetone + benzene (iii) nitric acid + chloroform (iii) ethyl bromide + ethyl iodide
(iv) ethanol + acetone (iv) acetone + aniline (iv) chlorobenzene + bromo
benzene
(v) ethanol + water (v) water + nitric acid
(vi) carbon tetrachloride + (vi) diethyl ether + chloroform
chloroform

Ptotal o
PA
V.P.
o
PB PA

PB

PB
0 XA 1
1 XB 0

 Vapour pressure diagram for real solutions of two liquids A and B that exhibit a positive deviation 
 
 from RaoultÊs law. The vapour pressure of both A and B are greater than predicted by RaoultÊs 
 
 law. 
The dashed lines represented the plots for ideal solutions.
Ć In case of + ve and ă ve deviations the boiling point composition curves have minima or
maxima. The separation of such solutions into their component canÊt be obtained by simple
or fractional distillation and these are called azeotrope mixture. An azeotropic mixture
boils at a constant temperature and distills over without undergoing any change in its
composition.
There are two types of azeotropes called as minimum boiling azeotrope and maximum
boiling azeotrope, respectively. Solutions of ethanol and water show such a large deviation
from RaoultÊs law that there is a maximum in the vapour pressure curve and hence a
minimum in the boiling point curve.
SOLUTIONS
QUIZRR 9

Example 1

How many grams of sucrose must be added to 360 g of water to lower the vapour pressure
by 1.19 mm Hg at a temperature at which vapour pressure of pure water is 25 mm Hg ?
Solution :
Given P = 1.19 mm Hg
Psolvent = 25 mm Hg
By RaoultÊs law

P
= Xsolute
Psolvent

1.19
 Xsolute =  0.0476
25

 Xsolvent = 1 ă 0.0476 = 0.9524

nsolvent
= 0.9524
ntotal

360
nsolvent =  20
18

nsolvent 20
 ntotal = =  21
0.9524 0.9524

 nsolute = 21 ă 20 = 1 = 1 ï 342 g
= 342 g

Example 2

The vapour pressure of pure liquid A at 300 K is 575 Torr and that of pure liquid B is 390
Torr. These two compounds form ideal liquid and gaseous mixtures. Consider the equilib-
rium composition of a mixture in which the mole fraction of A in the vapour is 0.35.
Calculate the total pressure of the vapour and the composition of the liquid mixture.
Solution :
A and B are volatile liquids, given

PA = 575 Torr, PB = 390 Torr

let mole fraction of A in solution = XA

SOLUTIONS
10 QUIZRR

hence, PTotal = PA XA + PB (1 ă XA)

also X´ A = mole fraction of A in the vapour = 0.35

PA X A
X´ A =  0.35
PA X A  PB 1  X A 

575 X A

= 575 X  390 1  X
A A

this gives XA = 0.27


hence, total pressure
Ptotal = 575 ï 0.27 + 390 ï 0.73
= 440 Torr
Composition of liquid mixture,
 A = 27 mol %, B = 73 mol %
Total pressure = 440 Torr

Example 3

At a certain temperature, the vapour pressure of pure ether is 646 mm and that of pure
acetone is 283 mm. Calculate the mole fraction of each component in the vapour state if
the mole fraction of ether in the solution is 0.50.
Solution :
In the given solution, both ether and acetone are volatile, so from RaoultÊs Law, we can have
vapour pressure of solution (P).
Let A : ether and B : acetone

P = PA + PB = PA0 A + PB0 B

 P = 646 ï 0.5 + 283 ï 0.5 = 464.5 mm


Now mole fraction in vapour state is given as follows :

PA P0  646  0.5
A (vapour) = = A A = = 0.695
P P 464.5

PB P 0 283  0.5
B (vapour) = = B B = = 0.305
P P 464.5

SOLUTIONS
QUIZRR 11

Colligative Properties
Certain properties of solutions depend only on the number of particles of the solute (molecular or
ions) present in definite volume of the solution and do not depend on the nature of solute. Such
properties are called colligative properties.
A few colligative properties are :
(i) Relative lowering of vapour pressure
(ii) Elevation of boiling point
(iii) Depression in freezing point
(iv) Osmotic pressure

(i) Relative lowering of vapour pressure :


The relative lowering of vapour pressure is a colligative property as it depends only on the
concentration of the solute and it is independent of its nature.
Consider a binary solution consisting of non volatile solute in a volatile solvent. The vapour
pressure of the solution (P) is equal to vapor pressure of solvent (PA) in the solution, which in
turn is directly proportional to its mole fraction in solution.
P = PA

= PA0  A ... (i)

where  A = mole fraction of solvent


If mole fraction of solute is B then
A + B = 1 or A = 1 ă B ... (ii)

From eq. (i) and (ii) PA = PA0 (1 ă B) = PA0 ă PA0 B

PA0 ă PA = PA0 B

PA0  PA
 B
PA0

PA0  PA
 B
PA0

is called the relative lowering of vapor pressure and is equal to the mole fraction of the solute.

P nB n
Now,  B   B {for dilute solutions : nA + nB  nA}
PA0 nA  nB nA

SOLUTIONS
12 QUIZRR
Calculation of molecular mass from relative lowering of vapor pressure
A known mass WB of the non-volatile solute is dissolved in a known mass (WA) of the volatile
solvent to prepare a dilute solution and relative lowering of vapor pressure is determined ex-
perimentally. Knowing the molecular mass (MA) of the solvent, molecular mass (MB) of the solute
can be determined as follows :

WA WB
n A = M , nB = M
A B

nB WB / MB
XB = =
nA  nB WA / M A  WB / MB

Relative lowering of vapor pressure is given by

P PA0  PA WB / MB
  XB 
PA0 PA0 WA / M A  WB / MB

For dilute solutions WB/MB << WA/MA and hence in the above expression WB/MB may be
neglected in the denominator as compared with WA/MA.

PA0  PA WB / MB W M
  B B
PA0 WA / M A MB WA

WBM A  PA0 
MB   0 
WA P  P 
 A A 

In this expression all the parameters are known except MB and hence MB can be calculated.

Example 4

Dry air was passed through a container having a solution of a non-volatile solute in water.
The solute weighed 26.66 g and the solvent weighed 200 g. The air then went into a solvent
container and then finally through a tube containing fused calcium chloride. The loss in
weight of water container is 0.087 g and gain in weight of CaCl2 tube is 2.036 g. Calculate
the molecular weight of the non-volatile solute present in the solution.
Solution : We can use the relation
P 0  Ps n loss in weight of solvent
 =
Ps N loss in weight of solution

26.66
0.087 M 26.66  18 26.66  18  1.949
=  ; M  53.75
2.036  0.087 200 200  M 200  0.087
18

SOLUTIONS
QUIZRR 13

ELEVATION OF BOILING POINT

The boiling point of a liquid may be defined as the temperature at which the vapor pressure of
the liquid becomes equal to the atmospheric pressure.

Atmospheric Pressure
B D
P
t
ven
l
so
P´ ure
P n
utio
Vapor Pressure

A l
So

Tb

X
Temprature T0 T1

The vapor pressure curves for the pure solvent and solution consisting of a non-volatile solute is
represented diagrammatically. AB is the vapor pressure curve for pure solvent and CD is vapor
pressure curve for the solution. T0 represents the boiling point of pure solvent and T1 represents
the boiling point of solution. In the presence of a non-volatile solute the solution has to be heated
to a slightly higher temperature so that the vapor of the solution becomes equal to atmospheric
pressure and solution begins to boil. The differences in the boiling points are represented as Tb,
which is the elevation in boiling point.
Tb = Kb m
Kb : molal boiling point elevation constant or ebullioscopic constant of solvent
m : molality of the solution

n 
Hence, Tb  K b  B  1000 
 gA 

1000 K b gB
 Tb 
gA MB

SOLUTIONS
14 QUIZRR
Boiling Point Elevation Constant (Kb ) : It is the elevation in point achieved when a solution
of a non-volatile solute of concentration 1 molal is boiled. Its boiling units are K kg molă1. Kb is
related to molar enthalpy of vaporization as

M RTb2
Kb 
Hvap  1000

where M = molar mass of solvent,


R = Universal gas constant,
Tb = boiling point of pure solvent and
H vap = enthalpy of vaporization of solvent.

Example 5

A solution containing 0.512 g of naphthalene (molar mass 128.2 g molă1) in 50 g CCl4 yields
a boiling point elevation of 0.402 K, while a solution of 0.6216 g of an unknown solute in
the same mass of the solvent gives a boiling point elevation of 0.647 K. Find the molar mass
of the unknown solute.
Solution :
Tb, naphthalene
ă Tb,CCl = Tb, naphthalene
= Kb mnaphthalene
4
Tb, unknown solute
ă Tb,CCl = Tb, unknown solute
= Kb munknown solute
4
 Tb, unknown solute
ă Tb, naphthalene
= Tb, unknown solute
ă Tb, naphthalene
= Kb ï (munknown solute
ă mnaphthalene)

Tb, naphthalene 0.402K


Kb  
mnaphthalene  
 
 0.512 g   1000 kg 1
 128.2 g mol1 
 
 50 g 

0.402  50  128.2
Kb   5.03 K kg mol 1
0.512 1000

 0.6216 g  1000 g kg 1 0.512 g  1000 kg 1 


0.647 K  0.402 K  5.03 K kg mol1   
 Mg mol1  50 g 128.2 g mol1  50 g 

Solving for M we get,


M = 96.68 g molă1.

SOLUTIONS
QUIZRR 15

DEPRESSION IN FREEZING POINT

Freezing point of a substance is defined as the temperature at which the vapor pressure of the
solid form is in equilibrium with liquid form of the substance. In other words, solid and liquid
forms of the substance has same vapor pressure. Since the vapour pressure of a solution is less
than that of pure solvent, the freezing point of the solution will be lower than that of pure
solvent.
Let Tf be the depression in freezing point, then

Tf = Tf ă T0

Tf : freezing point of solution

T0 : freezing point of pure solvent

Tf = Kf m

Kf : molal boiling point elevation constant or cryoscopic constant of solvent.

m : molality of solution

1000 K f gB
Tf 
gA MB

Final point Depression Constant (Kf) : It is the depression in freezing point achieved when
a solution of a non-volatile solute of concentration 1 molal is frozen. Its units are K kg molă1.
Kf is related to the molar enthalpy of fusion as

M RT12
Kf 
Hfusion  1000

where M = molar mass of solvent,


R = universal gas constant,
Tf = freezing point of pure solvent and
H fusion = enthalpy of fusion of solid solvent

4. OSMOTIC PRESSURE
Osmosis is defined as the spontaneous flow of solvent into solution or from a more dilute to a more
concentrated solution, when the two solutions are separated by a suitable membrane. The mem-
brane allows the free passage of the solvent but not of the dissolved substances. Membranes
possessing this property are said to be semipermeable.

SOLUTIONS
16 QUIZRR
The phenomenon of osmosis may be illustrated in a simple man-
ner by tying an animal membrane, e.g., bladder, over the end of
an inverted thistle tube, which is then partly filled with a concen-
trated solution of sucrose (cane sugar) and dipped into a beaker
Hydrostatic
of water, as shown in the given figure. The level of the liquid will
Pressure
rise in the tube until the hydrostatic pressure so produced is
sufficient to stop the flow of water. It is seen, therefore, that as
a result of osmosis, a pressure is developed which opposes the Sucrose
solution
tendency for the solvent to pass through the semipermeable
membrane into the solution. This pressure is called the osmotic Membrane
pressure of the solution. It is defined as „the excess pressure
that must be applied to a solution to prevent the passage
Water
into of solvent when the liquids are separated by a per-
fectly semipermeable membrane‰.
There has been some tendency to think pressure as a pressure produced by the solution, but this
point of view is misleading. The osmotic pressure is brought into existence only when the solution
is separated from the solvent by a semipermeable membrane. The resulting osmosis or tendency
for osmosis to occur then produces an excess pressure in the solution. When this excess pressure
attains the value of the osmosis pressure, the tendency for solvent to enter the solution is exactly
counterbalanced by the reverse tendency and a condition of equilibrium results.
For the accurate measurement of osmotic pressure, it is necessary to have a membrane that is
perfectly semipermeable. Artificial membranes made of copper ferrocyanide, Cu2Fe(CN)6, is the
best material for osmosis.
The osmotic pressure of a dilute solution of a non-volatile solute is given by V = nRT

n
 RT
V

w RT

M V

w
where  is the osmotic pressure in atmospheres, n = number of moles of solute   in V litres
M
ă1 ă1
of solution at temperature T (Kelvin), R = 0.0821 Lit atm K mol . C is the molarity of the
solution. Measurement of osmotic pressure provides a method of determining molecular weights
of proteins, polymer and other macromolecules.
If two solutions are of equal concentrations at the same temperature, they have the same osmotic
pressure and they are said to be isotonic. If two solutions have unequal osmotic pressures, the
more concentrated solution is said to be hypertonic and the more dilute solution is described as
hypo tonic. To study the contents of red blood cells, which are protected by a semipermeable

SOLUTIONS
QUIZRR 17

membrane, a technique called haemolysis is used. The red blood cells are placed in a hypo tonic
solution, when water moves into the cells. The cells swell and eventually burst releasing the
haemoglobin and other molecules.
When a bacterial cell is placed in a hypertonic solution (high concentration), the intracellular
water tends to move out of the cell by osmosis. This causes the cell to shrink.

Example 6

Find the molecular weight of the polymer having the concentration of 9.6 g/l and exerting
an osmotic pressure of 2.56 torr at a temperature of 298 K.
Solution :
      = CRT

2.56 9.6  0.0821  298



760 M

or M = 69727.5 g/mol.

Example 7

A very small amount of a non-volatile solute (that does not dissociate) is dissolved in 56.8
cm3 of benzene (density 0.889 g cmă3). At room temperature, vapour pressure of this solu-
tion is 98.88 mm Hg while that of benzene is 100 mm Hg. Find the molality of this solution.
If the freezing temperature of this solution is 0.73 degree lower than that of benzene, what
is the value of molal freezing point depression constant of benzene ?
Solution :

P   Ps w M 100  98.88 w  78  1000


       
Ps m W 98.88 m  W ï 1000

 w  1000  1.12  1000


or molality  m W   78 98.88  0.1452
   

Also T = K´f ï molality


0.73 = K´f ï 0.1452  K´f = 5.028 K molalityă1

Example 8

An aqueous solution containing 5% by weight of urea and 10% by weight of glucose. What
will be its freezing point ? K´f for H2O is 1.86Ĉ molă1 kg.
Solution :  Solution has 5% by weight urea and 10% by weight glucose

SOLUTIONS
18 QUIZRR

weight of solute
 % By weight =  100
weight of solution

 Wt. of water = 85 g; wt. of urea = 5 g; wt. of glucose = 10 g


T = TUrea + TGlucose

1000  1.86  5 1000  1.86  10


T = 
60  85 180  85

= 1.824 + 1.216 = 3.04


 Freezing point = 0 ă 3.04 = ă 3.04ĈC

Example 9

1.4 g of acetone dissolved in 100 g of benzene gave a solution which freezes at 277.12 K.
Pure benzene freezes at 278.4 K. 2.8 g of a solid (A) dissolved in 100 g of benzene gave a
solution which froze at 277.76 K. Calculate the molecular weight of (A).
Solution :
For acetone + Benzene mixture :

K´f  1000  w
T =
m W

1000  K f  1.4
(278.40 ă 277.12) =
100  58

1000  K f  1.4
or 1.28 = ...(1)
100  m

For solute (A) + Benzene mixture (Let m be the mol. weight of A)

1000  K f  2.8
(278.40 ă 277.76) =
100  m

1000  K f  2.8
or 0.64 = ...(2)
100  m

By Eqs. (1) and (2), m = 232

SOLUTIONS
QUIZRR 19

Example 10

If boiling point of an aqueous solution is 100.1ĈC. What is its freezing point ? Given latent
heat of fusion and vaporization of water are 80 cal gă1 and 540 cal gă1 respectively.
Solution :
For a given aqueous solution

Tb = K ´b ï molality

T f = K´f ï molality

Tb K´b RTb2 1000 lf


 Tf = K´f = 1000l  RT 2
v f

Tb Tb2  lf
Tf =
Tf2  lv

 Tb = 100 + 273 = 373 K


Tf = 0 + 273 = 273 K
l f = 80 cal gă1
l v = 540 cal gă1

Example 11

Two solutions of non-volatile solutes A and B having molar mass ratio of A and B (MA/MB)
as 1/3 are prepared by dissolving 5% (wt./vol.) of each in water. Calculate the ratio of
freezing point depression. If the two solutions are mixed to prepare a new solution (S1) by
mixing in the ratio 2 : 3 by volume and another new solution (S2) is obtained by mixing in
 S1
Tf
the ratio 3 : 2, find out
 Tf S2 . Assume density of solution A and B equal to one.
Solution :
For 5% solution of A :

K f  5  1000
(Tf) A = ... (1)
mA  95
For 5% solution of B :

K f  5  1000
(Tf) B = ...(2)
mB  95

SOLUTIONS
20 QUIZRR
By Eqs. (1) and (2),

 Tf A mB 3  mA 1 
   

 Tf B =
mA 1  mB 3 

For solution S1 : Let 2V mL solution of A is mixed with 3V mL solution of B.

5  2V 10V
Weight of A in 2V mL or 2V g = g = g
100 100

5  3V 15V
Weight of B in 3V mL or 3V g = g = g
100 100

Total weight of solution = 5V

 0.1V 0.15V 
K f  1000    
mA mB  1000K f 0.1mB  0.15mA 
 Tf S1 


10V 

4.9  mA  mB
 5V 
100 

For solution S2 : Similarly, have

1000  K f 0.15mB  0.10 mA   0.9898


(Tf)S =
2 4.85  mA  mB

 Tf S1 (0.1mB  0.15mA )  0.9898


Thus,
 Tf S2 =
(0.10mA  0.15mB )

(0.1 3mA  0.15mA )  0.9898


=
(0.10mA  0.15  3mA )

0.45  0.9898
=  0.81
0.55

VAN’T HOFF FACTOR ( i )

Since colligative properties depend on the number of particles of the solute, in some cases where
the solute associates or dissociates in solution, abnormal results for colligative properties are
obtained.

SOLUTIONS
QUIZRR 21

Dissociation : Let us consider the case of acetic acid dissolved in water. Let the number of mole
of acetic acid dissolved be ÂnÊ.


CH3COOH  CH COOă + H+
3

Initial no. of mole n 0 0


After dissociation n(1 ă ) n n
Total mole of solute = n(1 + )
Theroctical colligative property (for example Tb) based on the number of mole of CH3COOH
added)

n
Tb = Kb m = Kb ï  1000 , where W is the mass of solvent in gm.
W

Experimental colligative property would be given by

n(1  )
T´b = Kb m´ = Kb ï  1000
W

Experiment colligative property n 1   


  1     i, VanÊt Hoff factor
Theoretical colligative property n

Association : Let us consider the case of acetic acid dissolved in benzene. Let the number of mole
of acetic acid dissolved be ÂnÊ it forms dimer in benzene.

2CH3COOH 

 (CH3COOH)2

Initial no. of mole n 0


After dissociation n(1 ă ) n/2
Total mole of solute = n(1 ă /2)

Theoretical colligative property (for example Tb) (based on the no. of mole of CH3COOH added)

n
Tb = Kb m = Kb ï  1000 , where W is the mass of solvent in gm.
W

Experimental colligative property would be given by

 
n1  
T´b = Kb m´ = Kb ï  2
 1000
W

SOLUTIONS
22 QUIZRR

 
n 1  
Experimental colligative property  2 
  1   i, VanÊt Hoff factor
Theoretical colligative property n 2

Therefore, for dissociation i > 1 and for association i < 1. When a solute neither dissociates nor
associates, the VanÊt Hoff factor is equal to 1.

Example 12

Acetic acid associates in benzene to form a dimer. 1.65 gm of acetic acid when dissolved in
100 gm of benzene raised the B.P. by 0.36ĈC. Calculate the VanÊt Hoff factor and degree of
association of acetic acid. (k b for benzene = 2.57 ĈC/m)
Solution :
Calculation of VanÊt Hoff factor :
First calculate molecular mass from elevation in boiling point

1000 kb gB
MB =
gA  Tb

1000  2.57  1.65


=  117.8
100  0.36

Normal molecular mass


 i =
observed molecular mass

60
=  0.509
117.8
Calculation of degree of dissociation
Let x = degree of dissociation
2CH3COOH  (CH3COOH)2
1 0
1 ă x x/2
Total mass = 1 ă x/2

x
1
total mass 2
i 
initial mass 1

x
1
 0.509 = 2  x = 0.982
1

SOLUTIONS
QUIZRR 23

Example 13

When dissolved in benzene, a compound C38H30 partially dissociates by the following equa-
tion : C38H30  2 C19H15. 25.6 gm of C38H30 is dissolved in 400 gm of benzene, the freezing
point is lowered by 0.680ĈC. What % age of C38H30 molecules have dissociated ? (k f = 4.9)
Solution :
Calculate observed molecular mass (due to dissociation) as done in above example and then by
calculating i, find x.
OR Use the following approach.
Tb = i kb m

Tb  gA 0.68  400


i 
kb ( gB / MB )  1000 25.6
4.9   1000
486

 i = 1.0538
C38H30  2 C19H15
1 ă x 2x
Total mass = 1 ă x + 2x = 1 + x

total moles
i
initial moles

1 x
 1.0538 
1

 x = 0.0538
 % age of dissociation = 5.38%

Example 14

Acetic acid (CH3COOH) associates in benzene to form double molecules. 1.65 g of acetic acid
when dissolved in 100 g of benzene raised the boiling point by 0.36ĈC. Calculate the VanÊt
Hoff factor and the degree of association of acetic acid in benzene (Molal elevation con-
stant of benzene is 2.57).
Solution :
Normal molar mass of acetic acid = 60
Observed molar mass of acetic acid

wsolute 2.57  1.65  1000


M  Kb  1000   118
Wsolvent  in gms  T 100  0.36

SOLUTIONS
24 QUIZRR

Normal molar mass 60 g mol1


VanÊt Hoff factor = Observed molar mass 
118g mol1

  1    1 
= 0.508  1    1    1    1  2   1   / 2
  n    

/2 = 1 ă 0.508 = 0.492


  = ï 0.492 = 0.984
Thus acetic acid is 98.3% associated in benzene.
Now, if we include VanÊt Hoff factor in the formular for colligative properties we obtain the
normal results.

P1o  P1 n
1. Relative lowering of vapour pressure,  iX 2  i
P1o nN

2. Osmotic pressure,  = iCRT


3. Elevation in boiling point, Tb = i. 1000 ï Kb ï molality
4. Depression in freezing point, Tf = i 1000 ï Kf ï molality
Note : The value of i is taken as one when solute is non electrolyte.

Example 15
A 0.5 percent aqueous solution of Potassium Chloride was found to freeze at 0.24ĈC. Cal-
culate the VanÊt Hoff factor and degree of dissociation of the solute at this concentration
(Kf for water is 1.86).
Solution :

i  1000 K f w
T f =
W. M

i  1000  0.5  1.86


0.24 = 100  74.5

0.24  100  74.5


i =  1.923
1000  0.5  1.86

i = 1 + (n ă 1)
1.923 = 1 + (2 ă 1)
1.923 = 1 + 
 = 1.923 ă 1 = 0.923
or  = 0.923 or 92.3%

SOLUTIONS
QUIZRR 25

SOLVED EXAMPLES

Example 1

If the apparent degree of ionization of KCl (KCl = 74.5 gm molă1) in water at 290 K is 0.86.
Calculate the mass of KCl which must be made up to 1 dm3 of aqueous solution so as to
produce the same osmotic pressure as the 4.0% solution of glucose at that temperature.
Solution :
Due to ionization of KCl

KCl  K+ + Clă


Initial moles 1 0 0
Moles after dissociation 1 ă   

i
1         1  

1

From question, degree of ionization =  = 0.86


 i = 1 + 0.86 = 1.86
For Osmotic pressure of glucose,
For 4% glucose solution, Weight of glucose = 40 gm,
Volume of solution = V = 1 L = 1 dm3
Molecular weight of glucose C6H12O6 = m = 180

1
 glucose = n   RT, where glucose = osmotic pressure of glucose
V

w 1 40 1
=  RT=   RT
m V 180 1

Similarly, KCl = i  n  1 RT = 1.86  n  1  RT


V V

w1 1
= 1.86    RT
m1 V

= 1.86  w1  1  RT
74.5 1

SOLUTIONS
26 QUIZRR
As both solutions are isotonic,
 KCl = glucose

w1 40
1.86   RT =  RT
74.5 180

40 74.5
 W1 = 
180 1.86

= 8.9 gm

Example 2
1 kg of an aqueous solution of Sucrose is cooled and maintained at ă 4ĈC. How much ice
will be separated out if the initial molality of the solution is 0.75 ?
Solution :
Sucrose is a non-electrolyte, hence i = 1
Molecular weight of sucrose (C12H2O11) = m = 342 gm molă1
Molality of the solution = 0.75 m
= 0.75 mol kgă1 solvent
= 0.75 ï 342 gm Sucrose per kg solvent
= 256.5 gm Sucrose per kg solvent
Hence, weight of 1 molal solution = 1000 + 256.5 = 1256.5 gm

256.5
Sucrose present in 1 kg solution =  1000 = 204.14 gm
1256.5

Weight of solvent (H2O) present in 1 kg solution = 1000 ă 204.14 = 795.86 gm


Since depression in freezing point

w 1000
T f = Kf ï i ï ï
m W

Where W = weight of solvent


w = weight of the solute

204.14 1000
or, 4 = 1.86 ï 1 ï 
342 W

 W = 277.55 gm
i.e., weight of solvent required to maintain this solution at ă 4ĈC is W = 277.55
Hence rest weight of H2O will convert into ice.
Hence amount of ice formed = 795.86 ă 277.55 = 518.31 gm

SOLUTIONS
QUIZRR 27

Example 3

The osmotic pressure of an aqueous solution of sucrose is 2.47 atm at 303 K and the molar
volume of the water present is 18.10 cm3. Calculate the elevation in boiling point of the
solution. Given Hvap = 540 cal/g. Assume volume of solvent equal to volume of solution.
Solution :
 = CST
2.47 = C ï 0.0821 ï 303
 C = 9.93 ï 10ă2 M
Thus, 1 litre solution of sucrose contains 9.93 ï 10ă2 mole of sucrose or 9.93 ï 10ă2 ï 342 g of
sucrose.
 Volume of solution = Volume of solvent = 1000 mL

1000
 Mole of water =
18.10

1000
 Weight of water =  18 = 994.475 g
18.10

9.93  10 2
Thus, molality of solution = 3
 9.985  10 2 M
994.475  10

RTb2
Tb = Kb ï molality =  molality
1000l

2  373  373
=  9.985  10 2
1000  540

= 5.145 ï 10ă2

Example 4

The molecular weight of an organic compound is 58 g/mol. 24 g of this dissolved in 600 g


of water, calculate its boiling point when vapour pressure of water becomes 760 mm at
99.725ĈC. Kb of H2O is 0.513 K kg molă1.
Solution :
B. pt. of H2O = 99.725ĈC
Given worganic compound
= 24 g, wH = 600 g,
2O
morganic compound
= 58

SOLUTIONS
28 QUIZRR

1000  K b  w
Also, Tb =
Wm

1000  0.513  24
=  0.354
600  58

 B. pt. of solution = 99.725 + 0.354


= 100.079ĈC

Example 5

A 0.025 M solution of a monobasic acid has a freezing point of ă 0.06ĈC. Find the value of
Ka of this acid. Kf of water = 1.86 K/m. Assume that molarity is equal to molality.
Solution :
Calculate VanÊt Hoff factor using the relation :
T f = i Kf m Taking m  M

Tf 0.06
 i   1.29
Kf m 1.86  0.025

VanÊt Hoff factor is given as :


HA  H+ + Aă

1 ă   

total moles 1


i 
initial moles 1

 i = 1 +  = 1.29          = 0.29

HA  H+ + A
c ă c c c

0.025   0.29 
2
C 2
 Ka    2.96  103
1 1  0.29

Example 6

1.0 gm of monobasic acid when dissolved in 100 gm of pure waters lowers the freezing point
by 0.168ĈC. 0.2 gm of the same acid when dissolved and titrated required 15.1 ml of N/10
base. Determine the degree of dissociation of the acid. Kf of water is 1.86 K/m.

SOLUTIONS
QUIZRR 29

Solution :
Calculate the molecular mass (observed from colligative property) using the relation :

K f  gB  1000 1.86  1 1000


M0    110.6
Tf  gA 0.168  100

Now calculate the actual molecular mass from neutralisation experiment

0.2
 1000  0.1  15.1 (meq of acid = meq of base)
M /1

 M = 132.45

Mactual 132.45
i   1.19
M0 110.6

HA  H+ + Aă

1 ă   
 i = 1 +  = 1.19
  = 0.19

Example 7

30 ml of CH3OH (d = 0.7980 gm cmă3) and 70 ml of H2O (d = 9984 gm cmă3) are mixed at 25ĈC
to form a solution of density 0.9575 gm cmă3. Calculate the freezing point of the solution
Kf (H2O) is 1.86 Kg molă1 K. Also calculate its molarity.
Solution :
Weight of CH3OH (w1) = 30 cm3 ï 0.7980 gm cmă3 = 23.94 gm
Weight of solvent (H2O) (w2) = 70 cm3 ï 0.9984 gm cmă3 = 69.888 gm

w1 1000
 Molality of solution = 
m1 w2

Where m1 = molecular weight of CH3OH = 32

23.94 1000
m =   10.7046 m
32 69.88

As we know that
Depression in freezing point, Tf = Kf ï i ï molality,
For CH3OH, i = 1
Tf = 1.86 ï 10.7046ĈC = 19.91ĈC

SOLUTIONS
30 QUIZRR
Freezing point of the solution = 0 ă 19.91ĈC = ă 19.91ĈC
Weight of solution = weight of solute + weight of solvent = 23.94 + 69.888 = 93.828 gm

wt. of the solution 93.828


Volume of the solution = density of the solution  0.95751  97.99 ml

w1 1000 23.94 1000


Molarity of solution =    mol lită1 = 7.63 M
m1 V(in ml) 32 97.99

Example 8

The freezing point of an aqueous solution of KCN containing 0.1892 mol kgă1 of solvent was
found to be ă 0.704ĈC. On adding 0.095 mol of Hg (CN)2, the freezing point of the solution
was found to be ă 0.530ĈC. If the complex ă formation takes place according to the following
equation.

23
Hg(CN)2 + nKCN 

 Kn [Hg(CN)n+2] (Kf = 10 )

What is the formula of the complex ? Kf(H2O) is 1.86 kg molă1 K.


Solution :
Tf (KCN solution) = 0.704ĈC
Molality of KCN solution = 0.1892
i = (1 + x)
Tf = Kf ï m ï i

Tf 0.704
i  1  x     2.0
Kf  m 1.86  0.1892

This gives x = 1, indicating 100% ionization of KCN


Tf of the complex = 0.530
Molality of Hg (CN)2 = 0.095 mol kgă1

Kn[Hg(CN)n+2] 

 nK+ + [Hg(CN)n+2]ă

1 0 0
1 ă  n 
here i = (1 ă ) + n +  = 1 + n
  = 1
 i = 1+ n

SOLUTIONS
QUIZRR 31

T f = Kf ï i ï molality
0.503 = 1.86 ï 1 ï 0.095

0.503
i 3
1.86  0.095

1 + n = 3
or n = 2
Hence, the complex is K2[Hg(CN)4]

Example 9

The freezing point of a solution of acetic acid (mole fraction is 0.02) in benzene 277.4 K.
Acetic acid exists partly as a dimer 2A  
 A2. Calculate equilibrium constant for
dimerisation. Freezing point of benzene is 278.4 k and (Kf for benzene is 5).
Solution :
Let acetic acid = A
Benzene = B
Assume,  part of A forms dimer



2A  A2

1 0
1 ă  /2

1      / 2
 i =
1

= 1 ă /2
Mol. fraction of A = xA = 0.02
Mol. fraction of B = xB = 0.98

xA 1000
Molarity of A in B = m  X
B B

0.02 1000
= 
78 0.98

= 0.262 mol kgă1 of Benzene


Since Tf = Kf ï i ï molality

SOLUTIONS
32 QUIZRR
278.4 ă 277.4 = 5 ï i ï 0.262
or 1 = 5 ï i ï 0.262

1
i  0.262
5

= 0.763
1 ă /2 = 0.763
  = 0.47
Hence the molality of A after dimer is formed = (1 ă ) ï initial molality
= (1 ă 0.48) ï initial molality
= 0.52 ï 0.262


Molality of A2 after dimer is formed =  molality
2

0.48
=  0.262
2

= 0.24 ï 0.26
= 0.06288
The equation constant

 A 2   0.06288
Keq =
 A 2 (0.13624)2
= 3.39 kg molă1

Example 10

Find Ka , the ionization constant of tartaric acid if a 0.100 molal aqueous solution of tar-
taric acid freezes at ă 0.205ĈC. Assume that only the first ionization is of importance and
that 0.1 m = 0.1 M. Kf = 1.86 kg molă1 K.
Solution :
Assuming that the tartaric acid be a monobasic as AH. It ionizes as

AH 

 A + H+

Initially conc. C 0
Conc. after dissociation C(1 ă ) C C

SOLUTIONS
QUIZRR 33

C(1  )  C  C
Here, i = , where  = degree of dissociation
C

C(1  )
= 1
C

Molal concentration = 0.1


T f = Kf ï Cm ï 1
  = 0.1

 A   H  2
     C  C   C
Ka =
 AH C 1    1  

 0.12  0.1
Ka =  1.11  10 3
1  0.1

 Ka = 1.11 ï 10ă3.

SOLUTIONS
THE SOLID STATE
QUIZRR 3

THE SOLID STATE


Introduction

A solid is that form of matter that possesses rigidity and hence possesses a definite shape and a
definite volume.

Three are 2 types of solids

(1) Crystalline Solids

Solids with a definite geometric pattern.


Example : Iron, copper, silver, sulphur etc. are some elements which form crystalline solids.
Potassium chloride, sodium nitrate etc. are some of the compounds, which are crystalline.

(2) Amorphous Solids

Solids with particles not arranged in a regular fashion. They have only short range order or even
the particles are disordered in some cases.
Crystalline Solid Amorphous Solid
1. Definite and regular geometry with flat 1. No definite geometrical shape
faces and sharp edges
2. It breaks up into smaller crystals of the 2. Broken pieces are not generally flat
same geometrical shape
3. Sharp melting point 3. No sharp melting point
4. Anisotropic, i.e., physical properties are 4. Isotropic, i.e., physical properties are
different in different directions same in all directions.

Classification of Crystalline Solids

Ć Ionic
Ć Molecular
Ć Covalent
Ć Metallic

Type of Crystalline Solids

Ionic Metallic Covalent Molecular


Particles Anions, Metal ions in atoms Molecules
occupying cations election cloud (or atoms)
lattice points
Binding force Electrostatic Metallic Covalent Van der waals
attraction bonds bonds dipole-dipole

THE SOLID STATE


4 QUIZRR
Ionic Metallic Covalent Molecular
Properties Hard, brittle, Soft to very Very hard, Soft, poor
poor thermal hard, good poor thermal thermal and
and electrical thermal and and electrical electrical
conductors electrical conductors conductors
conductors
Example NaCl, CaBr2, Li, K, Ca, C (diamond), H2O, H2,
KNO3, etc. Cu, Na, etc. SiO2 (quartz), etc. CO2, Ar, etc.

Space Lattice and Unit Cell

The constituent particles of a crystalline solid are arranged in a definite fashion in the three
dimensional space.
One such arrangement by representing the particles with points is shown below.

c
 
a
b X

Representation of space Representation of dimensions


lattice and unit cell of a unit cell

Such a regular arrangement of the constituent particles of a crystal in a three dimensional space
is called crystal lattice or space lattice.
From the complete space lattice, it is possible to select a smallest three dimensional portion which
repeats itself in different directions produces the complete the space lattice. This is called a Unit
Cell.

Unit Cell

The smallest three-dimensional portion of a complete space lattice, which when repeated over and
again in different directions produces the complete space lattice.
The size and shape of a unit cell si determined by the lengths of the edges of the unit cell (a, b
and c) and by the angles ,  and  between the edges b and c, c and a, and a and b respectively.
If we take into consideration, the symmetry of the axial distances (a, b, c) and also the axial
angels between the edges (,  and ), the various crystals can be divided into seven systems.
These are also called crystal habits.

THE SOLID STATE


QUIZRR 5

T ypes of Symmetry in Cubic Crystals

Centre of Symmetry : It is an imaginary point within the crystal such that any line drawn
through it intersects the surface of the crystal at equal distances in both directions. There is only
one centre of symmetry in a cubic crystal.

Centre of symmetry

Plane of Symmetry : It is an imaginary plane which passes through the centre of symmetry of
a crystal and divides it into two equal portions such that one part is exactly the mirror image of
the other.

Rectangular plane of Diagonal plane of


symmetry (3) symmetry (6)

A cubic crystal has in all 3 + 6 = 9 planes of symmetry.

Axis of Symmetry : It is an imaginary straight line about which if the crystal is rotated, it will
present the same appearance more than once during a complete revolution. If the same appearance
of a crystal is repeated through an angle of 360/n, about an imaginary axis, the axis is called an
n-fold axis.

Axis of 4-fold Axis of 2-fold Axis of 2-fold


symmetry (3) symmetry (4) symmetry (6)

A cubic crystal has in all 3 + 4 + 6 = 13 axes of symmetry.

The total number of centres of symmetry, planes of symmetry and axes of symmetry possessed by
a crystal is termed elements of symmetry of the crystal. A cubic crystal possesses 23 elements
of symmetry.

THE SOLID STATE


6 QUIZRR
Centre of symmetry = 1
Places of symmetry = 3 + 6 = 9
Axes of symmetry = 3 + 4 + 6 = 13
Elements of symmetry = 1 + 9 + 13 = 23

Bravais Lattices

Bravais (1848) showed from geometrical considerations that there are only seven shapes in which
unit cells can exist.
There are :
(i) Cubic (ii) orthorhombic
(iii) Rhombohedral (v) Hexagonal
(v) Tetragonal (vi) Monoclinic and
(vii) Triclinic.
Moreover, he also showed that there are basically four types of unit cells depending on the
manner in which they are arranged in a given shape. These are : Primitive, Body Centered, Face
Centered and End Centered. He also went on to postulate that out of the possible twenty eight
unit cells (i.e. seven shapes ï four types in each shape = 28 possible unit cells), only fourteen
actually would exist. These, he postulated, based only on symmetry considerations. These fourteen
unit cells that actually exist are called Bravais Lattices

(a) Primitive unit cell (b) Body centre unit cell

(c) Face Center unit cell (d) End Center unit call

THE SOLID STATE


QUIZRR 7

The seven crystal systems and their unit cell parameters are shown as

Crystal systems Bravais lattices Unit cell parameters


Intercepts Crystal angles Examples

1. Cubic Primitive, Face centered a= b= c  =  =  = 90Ĉ


Body centered
2. Orthorhombic Primitive, Face centered, a  b  c  =  =  = 90Ĉ
Body centered, End
centered
3. Rhombohedral Primitive a= b= c  =  =   90Ĉ NaNO3, CaCO3
4. Monoclinic Primitive, End centered a  b  c  =  = 90Ĉ ,   90Ĉ K2Cr2O7,
CaSO4.2H2O
5. Triclinic Primitive a  b  c       90Ĉ CuSO45H2O
6. Tetragonal Primitive, Body centered a = b c  =  =  = 90Ĉ
7. Hexagonal Primitive a = b c  =  = 90Ĉ,  = 120Ĉ HgS, ZnO, CdS

Simple Cube Face Centered Body centred


cube (fcc) cube (Bcc)
CUBIC (a = b = c;  =  = = 90Ĉ )

Simple Body centred Simple End centred

TETRAGONAL MONOCLINIC
a=b=c a=b=c
 =  =  = 90Ĉ  =  = 90Ĉ ,  = 90Ĉ

THE SOLID STATE


8 QUIZRR

Simple Face centred Body centred End centred

ORTHORHOMBIC
a=b=c
 =  =  = 90Ĉ

Simple Simple Simple


RHOMBOHEDRAL TRICLINIC HEXAGONAL
a=b=c a=b=c a=b=c
 = = = 90Ĉ  = = = 90Ĉ  = =90Ĉ ,  = 120Ĉ

In IIT-JEE, we only have two crystal systems in syllabus, of which only four Bravais lattices exist.
Thus we have to study only
(i) Primitive or simple cubic
(ii) Body centered cubic
(iii) Face centered cubic
(iv) Hexagonal primitive
Before proceeding further, let us first understand the term rank of a unit cell and co-ordination
number. Rank of a unit cell means the number of effective atoms (or ions or ion-pairs)
in a unit cell and the co-ordination number is the number of atoms (or ions) surrounding
an atom (or ion) in a crystal lattice.

(1) PRIMITIVE OR SIMPLE CUBIC (PC/SC) UNIT CELL

In a primitive cubic unit cell, all the eight corners of the cube are occupied by atoms and they
are not present anywhere else in the cube. If all the eight atoms are of same type, the unit cell
is called ideal primitive and if they are not same, it is referred as pseudo primitive or primitive-
like.

THE SOLID STATE


QUIZRR 9

Figure (a) shows an ideal primitive cubic unit cell.

(a)
In the above figure, we are unable to see that an atom is in contact with how
many other atoms. So, in order to view this, we should look at the landscape
view of the primitive arrangement in figure (b).
In this layer, each atom is in contact with its immediate neighbours. If the
centres of these four atoms are joined, a face of the unit cell can be generated.
Now, place a layer of four other atoms over this layer in such a way that the (b)
atoms of the second layer eclipse the atoms of the first layer and join the centre of all the eight
atoms to get a primitive cubic arrangement.
In such an arrangement, each atom is in contact with six immediate
neighbouring atoms, so the co-ordination number of each atom is 6. If
ÂaÊ represents edge length of the unit cell, then a = 2r where ÂrÊ is the
radius of an atom. This is shown in figure (c).
In a lattice of this type, each corner atom is shared by eight unit cells,
four in one layer and four in the layer above it, as shown in figure (c).
Thus, the contribution of each atom placed at the corner of single cubic a
r
unit cell is 1/8. Since, there are eight corners of a cube, the effective
Simple or Primitive Cubic
1 a = 2r
number of atoms associated with a single primitive unit cell is  8  1 .
8 (c)
Thus, the rank of a primitive cubic unit cell is 1.
The volume of a cubic unit cell is not completely occupied by atoms. The ratio of volume
occupied by the effective atoms to the volume of the unit cell is called packing fraction
(PF). The fraction of volume of a unit cell, that is empty is called void fraction (VF).
Thus,
Volume occupied by the effective atoms in a unit cell
Packing fraction =
 
Volume of the unit cell a 3

and void fraction = (1 ă Packing fraction)

4 3
1 r
3 
 PF = 3
  0.523
(2r) 6

THE SOLID STATE


10 QUIZRR
Thus, the structure is relatively open since only 52.3% of the total volume is occupied by the
atoms. The remainder 47.7% of the total volume is empty space or void volume. In general, the
packing fraction is independent of the radius of atom and edge length of the unit cell and depends
only on the nature of packing. This type of structure is exhibited by polonium, Po.

(2) BODY CENTERED CUBIC (BCC) UNIT CELL

In body centered cubic unit cell, the lattice points are corners and body
centre. This implies that the atoms are present at all the corners and at the
body centered position and are not present anywhere else in the cube. If all
the nine atoms are of same size, the unit cell is referred as ideal BCC while
(a)
it is called BCCălike or pseudo BCC, if the atoms present at the corners and
the body centre are different. Figure (a) represents an ideal BCC lattice.
Here, packing consists of a base of 4 atoms. This constitutes first layer. It is
followed by a second layer in which each atom rests in the groove formed by
the atoms of first layer and the atom of second layer would also be in contact
with all the four atoms of first layer. Then a third layer of four atoms is kept
over the second layer in such a way that atoms of third layer eclipse the (b)
atoms of first layer. It must be noted here that atoms of third layer and first
layer are not in contact with each other while the atom of second layer is in contact with 8 atoms
(4 of first layer and 4 of third layer). So, the co-ordination number of each atom is 8.
In a BCC lattice, each corner atom is shared by 8 unit cells while the body centered atom is not
shared by any other unit cell. So, the number of effective atoms associated with a BCC unit cell

is   8   1  2. Thus, the rank of a BCC unit cell is 2.


1
8 

Since, the body centered atom touches all 8 corner atoms, so the body

diagonal  3a  would be equal to 4r, where ÂaÊ represents the edge


c b
length and ÂrÊ is the radius of atom. This is shown in figure (c). The
fraction of volume occupied by the atoms is given by

r a
4 3
2
r
3 3
PF = 3
  0.6802
 4r  8 (c)
 
 3

Thus, 68.20% of the volume of the until cell is occupied by atoms and
the remaining 31.98% is the void volume. This type of structure is
exhibited by K, Fe, Li, V, Cr, Mo etc.

THE SOLID STATE


QUIZRR 11

(3) FACE CENTERED CUBIC (FCC) UNIT CELL

In FCC unit cell, the lattice sites are corners and face centres. Thus. in face
centered cubic unit cell, the atoms are present at the corners and at the face
centres and are not present anywhere else in the cube. When all the 14
atoms are of same size, the unit cell is called ideal FCC while if the atoms
present at the corners and face centres are different, the unit cell is referred
(a)
as pseudo FCC or FCC-like. Figure (a) represents an ideal FCC lattice.
In this structure, two adjacent corner atoms are not i contact with each other and each face
centered atom is in contact with four corner atoms of its face. if ÂaÊ represents the edge length of

a FCC unit cell and ÂrÊ is the radius of atom, then face diagonal  
2a would be equal to 4r.

In a FCC unit cell, each corner atom is shared by 8 unit cells and each
face centered atom is shared by 2 unit cells, so the number of effective

atoms in a FCC unit cell would be   8     6   4. Thus, the rank


1 1
 8  2  b
of a FCC unit cell is 4. This is shown in figure (b).
The fraction of volume occupied by the atoms is given by
a
r
4 3
4 r
3 2 (b)
PF = 3
  0.7405
 4r  6
 
 2

Thus, 74.05% of the volume of the unit cell is occupied by atoms and the remaining 25-95% is the
empty space. Al, Ni, Ag, Fe, Pd, Cu, Au, Pb, all solid noble gases etc. exhibit this type of structure.

COORDINATION NUMBER

The number of ions of opposite charge in the immediate surrounding of a given ion is termed its
coordination number (C.N.)
C.N. and radius ratio of some crystals are summarised in Table
Limiting Radius Ratios and Structure
Limiting radius ratio C.N. Shape
(r +)/(r ă)
< 0.155 2 Linear
0.155ă0.225 3 Planar triangle
0.225ă0.414 4 Tetrahedral
0.414ă0.732 4 Square planar
0.414ă0.732 6 Octahedral
0.732ă0.999 8 Body-centred cubic

THE SOLID STATE


12 QUIZRR
Table : Characteristics of Some Typical Crystal Structure
Crystal Type of unit cell Example r +/r ă C.N. Number of
formula units of
(AB, or AB2)
per unit cell
CsCl Body-centred CsCl, CsBr, TICI 0.93 8-8 1
NaCl Face-centred AgCl, MgO 0.52 6-6 4
ZnS Face-centred ZnS 0.40 4-4 4
CaF2 Face-centred CaF2,SrF2,CdF2,ThO2 0.73 8-4 4

Unit Cell Dimensions and Density of the Material :


Let number of atoms in the unit cell = z
Edge length = a
Molecular weight or atomic weight of the species = M

Mz
Then mass of z atoms in a unit cell = N0

Volume = a3

mass Mz
Hence density d  
volume N0 a3

Note : Volume is a3 when the unit cell is cubic, it will be different for Hexagonal unit cell.

Bragg Equation

We can determine the size of the unit cell and the arrangement of atoms in a crystal from
measurement of the diffraction of X-rays by the crystal. When a beam of monochromatic X-ray
strikes two planes of atoms in a crystal at a certain angle , it is reflected. Intensity of the reflected
beam is maximum if
n = 2d sin 
This is called Bragg equation where n is the order of reflection,  the wavelength of X-ray ligh
used, d the distance between two planes.

Calculation of Number of Particles per Unit Cell of a Cubic Crystal System

Keeping the following points in mind we can calculate the number of atoms in a unit cell.
Ć An atom at the corner is shared by eight unit cells. Hence an atom at the corner contributes
1/8 to the unit cell
Ć Am atom at the face is shared by two unit cells
THE SOLID STATE
QUIZRR 13

Contribution of each atom on the face is 1/2 to the unit cell.


Ć An atom within the body of the unit cell is shared by no other unit cell
Contribution of each atom within the body is 1 to the unit cell.
Ć An atom present on the edge is shared by four unit cells
Contribution of each atom on the edge is 1/4 to the unit cell.
It can be determined by the relation

nc nf n
   i
8 2 1

nc = number of atoms at the corners of the cube


nf = number of atoms at Six faces of cube
ni = number of atoms at do the cube
By applying these rules, we can calculate the number of atoms in the different cubic unit cells
of monoatomic substances.

(a) Simple cube (b) Face centred cubic (c) Body centred cubic
Three types of unit cells for cubic crystal system

(a) Simple cube (b) Face centred cubic (c) Body centred cubic
Three dimensional view showing the number of atoms per unit cell

THE SOLID STATE


14 QUIZRR
Simple Cubic
1
In this type of unit cell, there are eight atoms at the corners and each such atom makes
8
contribution to the unit cell.

1
Hence, a simple cubic unit cell has 8 (at corners) ï = 1 atom.
8

Body Centered Cubic

This type of unit cell has eight atoms at corners and one at the body center. Each corner atom
1
makes contribution and the atom at the body center belongs only to the particular unit cell.
8
Hence, a body centered cubic unit cell has,
1
8 (at corners) ï + 1 (at body center) ï 1 = 2 atoms
8

Face Centered Cubic

A face centered cubic unit cell has one atom at each corner and one atom at each face center,
1
being shared by two unit cells and making a contribution of only to a particular unit cell.
2
Hence, a face centered cubic unit cell has,

1 1
8 (at corners) ï + 6 (at face centres) ï = 4 atoms
8 2

Example 1

An element ÂXÊ (atomic mass 150) having fcc structures has unit cell edge 400 pm. Calculate
the density of ÂXÊ and the number of unit cell for 15g of X.
Solution :
Length of the unit cell edge = 400 pm = 400 ï 10ă10 cm
Volume of unit cell = (400 ï 10ă10)3 = 6.4 ï 10ă23 cm3
Since structure is fcc the number of atoms will be 4

nM m 4 ï 150
Density = = = 15.56 g/cc
NA ï V 6.023 ï 1023 ï 6.4 ï 10-23

Mass 15
Volume of 15g of X = = = 0.9640 cc
density 15.56

THE SOLID STATE


QUIZRR 15

Volume of Substance
No. of unit cells in 0.9640 cm3 =
Unit cell volume

0.9640
= ă23
= 1.506 ï 1022 unit cells
6.4 ï 10

Example 2

The compounds A2B has ZnS structure. Its density is 3.4 g/cc. What is the edge length of
unit cell ? (Molecular weight of A = 32, B = 35).
Solution:
For ZnS structure n = 4
Molar mass of A2B (Mm) = 2 ï 32 + 35 = 99
 = 3.4 g/cc

n  Mm
 =
a3  N A

1/3
 4  99 
a
 3.4  6.023  1023 
 


a = 5.78 ï 10ă8 cm = 5.78 A

Example 3

The unit cell in a crystal of diamond belongs to a cubic crystal system but doesnÊt correspond
to the Bravais lattice. The volume of unit cell of diamond is 0.0454 nm3 and the density of
diamond is 3.52 g/cc. Find the number of carbon atoms in a unit cell of diamond ?
Solution:
Atomic mass of carbon = 12 g/mol
Volume of the unit cell of diamond = 0.0454 nm3

Atomic mass ï n
Density =
Unit cell volume ï Avoadro's no.

12  n
3.52 
0.0454  1021  6.023  1023

n = 8 atoms/unit cell
THE SOLID STATE
16 QUIZRR
Example 4

Iron occurs as BCC as well as FCC unit cell. If the effective radius of an atom of iron is 124
pm, compute the density of iron in both these structures.
Solution :
In a body centered unit cell, atoms touch each other along the cross diagonal. Hence,

4 4
4 r  3 a, a  r (124pm)  286.4pm
3 3

for this unit cell, n = 2. Hence,

n  Mm  2  55.8 
 3 N 
  23 
 7.887  106 gm 3  7.887gcm ă3
a  a  (286.4  10 12 )3  6.023  10 

In a face centered unit cell, atoms touch each other along the face diagonals. Hence,

4 4
4 r  2 a, a  r (124pm)  350.7pm
2 2

For this unit cell, n = 4. Hence,

4  55.8 
  23 
 8.59  106 gm 3  8.59 gcm 3
350.7  10 12  6.023  10 

Example 5

Use the data given below to find the type of cubic lattice to which the crystal belongs :
Fe V Pd
a/pm 286 301 388
ă3
/g cm 7.86 5.96 12.16
Solution :
The density of a cubic unit cell is given by

nM
 3
N AV  a

  a3  N AV
n 
M

THE SOLID STATE


QUIZRR 17

7.86  (2.86  108 )3  6.023  1023


For Fe, nFe   1.982  2
55.85

Hence, the cubic lattice of Fe is body-centered.

5.96  (3.01  108 )3  6.023  1023


For V, nv   1.921  2
50.94

Hence, the cubic lattice for V is body-centered.

12.16  (3.88  108 )3  6.023  1023


For Pd nPd   4.02  4
106.4

Hence, the cubic lattice for Pd face-centered.

Example 6 K1

Potassium crystallises in a bcc lattice as shown in figure K1


(a) What is the distance between nearest K1
neighbours. K1 K1
K1
(b) What is the distance between next nearest
K1 K1
neighbours ?
K2
(c) How many nearest neighbours does each K
atom have ? K1
K1
(d) How many next nearest neighbours does each
K have ? K1 5.20 A K1
Solution :

(a) The body diagonal is =  


3a  3  5.20 A  9.01 A

9.01 
The nearest neighbours along that diagonal are at the half distance apart  4.50 A ,
2
i.e., distance between K1 and K2.


(b) Distance between the next nearest neighbours = 5.20 A along the unit cell edge, i.e.,
distance between K1 and K2.
(c) Nearest neighbours having surrounded by each k atom = 8 (atom at centre is surrounded
by eight corners at the nearest), i.e., K1 surrounding K2.
(d) Next nearest neighbours having surrounded by each K atom = 6 (C.N. of each K at corner
is six), i.e., K1 surrounding K1.

THE SOLID STATE


18 QUIZRR
Example 7

Methane crystallizes in a cubic unit cell with a = 0.589 nm. If the density of liquid CH4 is
0.466 cmă3 and assuming that the density of solid is same as that of liquid at the given
temperature, predict which type of cubic unit cell will methane crystallize in.
Solution :
The cubic nit cell of methane can be simple cubic, BCC or FCC having effective number of
molecules 1, 2 or 4 respectively. Let us first calculate the density of methane when it crystallizes
in primitive (simple) cubic structure.

n M 1  16
sc  
NAV  a 3 23
6.023  10  (0.589  107 )3

sc = 0.13 g/cm3


Since in BCC lattice, the number of effective molecules is double than that in primitive cubic, so
the density of methane crystal in BCC would be double of primitive cubic.
 BCC = 2 ï 0.13 = 0.26 g/cm3
Similarly, FCC = 4 ï SC = 4 ï 0.13 = 0.52g/cm3.
The observed value of density (of liquid methane) is more closer to the density of FCC lattice, so
methane crystallizes in FCC structure.

Packing of Constituent Particles in Crystals

Assumption
It is assumed that the atoms are hard spheres of identical size. Packing is done in such a way
that they occupy maximum available space. This type of packing is called close packing.
The packing of the spheres of equal size latices place as follows :
1. Close packing of spheres in one dimension

2. Two dimensional close packing :


The rows can be combined in two different ways to form crystal places :
(a) Square close packing-particles when placed in the adjacent rows show a horizontal
as well as vertical alignment and form squares.
(b) Hexagonal close packing-Packing in every next row are placed in the depression
between the particles of the row. Hexagonal close packing with triangular voids is
more efficient.

THE SOLID STATE


QUIZRR 19

Spheres are placed in a row forming the crystal edge.

(a) Square closed packed layer (b) Hexagonal closed packed layer
Here cental sphere is in contact Here central sphere is in contact
with four other spheres with six other spheres
Note :
(1) Hexagonal close packing is more efficient as more space is accepted by the sphere in this
arrangement.
(2) In square packing, a central sphere is in contact with four other spheres, whereas in
hexagonal close packing, central sphere is in contact with six other spheres.

3. Three-dimensional packing
Let us now consider a three dimensional packing keeping a hexagonal close packed pattern for
layers.
Two types of arrangements are possible.
(a) Hexagonal close packing - AB AB type of arrangement.
(b) Cubic close packing - ABC - ABC type of arrangement.

A A A A A
A b
A b
A b
A b
A
a a a
A a
c
a
c
a
A
A b
A b
A b
A b b b
a a a a a a a a
A A A A A
A A A A A

Solid circles represent layer A


Dotted circles represent layer B

Hexagonal Close Packing

In two-dimensional packing, hexagonal close packing is more efficient. In three dimensions the
second layer spheres are placed in the A voids and B voids are unoccupied.
In the second layer there are C and D voids.
The third layer is placed such that it covers the C voids. The third layer spheres are directly over
the first layer leading to AB AB type of arrangement or hexagonal close packing.

THE SOLID STATE


20 QUIZRR
Cubic Close Packing

When the third spheres are placed on the D voids a layer different from layers A and B is
produced leading to ABC ABC....type arrangement or cubic close packing (CCP) similar to face
centered cubic. It is similar to face centered cubic packing.
Example of HCP ă Mg, Zn, Cd
Example of CCP ă Na, K, Fe, Cr,
In both the above patterns of arrangements the maximum occupied shape is 76% of the available
volume.
In both HCP and CCP the coordination number is 12 because a sphere is in contact with 6 spheres
in its own layer. It touches three spheres in the layer above and three in the layer below :

12 12
A 10 11 C 11 10

4 5 4 9
B 9 6 B 5 8
8 7 6 7

3 3 1
A 2 A
1 2
h.c.p c.c.p
ăCo-ordination number in hcp and ccp

ă(i) Hexagonal close packing

THE SOLID STATE


QUIZRR 21

ă(ii) Cubic close packing

Packing and Structure of Crystals

Packing Fraction and Density of Packing


Let radius of the atom in the packing = r
edge length of the cube = a
volume of the cube V = a 3

4 3
volume of the atom (spherical) v = r
3

4 3
r z
vz
then packing density =  3 3
V a

I. In simple cubic structure

a
r 
2

 Volume of the atom (spherical)

3
4 3 4  a a
 r    
3 3  2

r r
a3

6

 Volume of the cube = a3


Simple cubic
 Packing density

Volume of the one spherical atom in unit cell



Volume of the cube

a3 
 3
  0.5236
6a 6

THE SOLID STATE


22 QUIZRR
II. In fcc structure [hexagonal closest packed (hcp) and cubic closest-packed (ccp)]
Edge length = a
face diagonal = b = AC
In ABC
AC2 = BC2 + AB2
b2 = a2 + a2 = 2a2 G B
A
b  2a H

If radius of the spherical atom is r then


F C
b  2 a  4r a
E D
FCC structure
a
 r
2 2

4 3
Volume of the four spherical atoms of radius r in the unit cell = r  4
3

3
4  a  a3
=   4 =
3  2 2  3 2

Volume of the cube = a 3

a 3 
 Packing density = 3 =  0.7405
3 2a 3 2

III. In bcc (body centred cubic) structure


Edge length = a
and radius of the spherical atom in the unit cell = r
Face diagonal = b
and body diagonal = c
Atom at the centre touches other two atoms diagonally arranged
In EFD, FD2 = EF2 + ED2
b2 = a2 + a2

 b 2 a

THE SOLID STATE


QUIZRR 23

In AFD, AF2 = AD2 + FD2 G B


2 2 2 2 2 2
c = a + b = a + 2a = 3a A
H
 c 3a
c a
Also c = 4r F C
a b
3 a  4r
E a D

3
 r a
4
 Volume of two spherical atoms of radius r

4 3
in a unit cell = 2  r r 2r r
3

3
4  3   3 a3 c
= 2    a =
3  4  8

and volume of the cube = a3

 3 a3 3
 Packing efficiency = 3 =  0.6802
8a 8

Structure r related to a P Packing density


3
a 4 a 
r   0.52
3  2 
1. Simple
2 6

3
r
1 4  a  2
 0.74
3  2 2 
2. Face-centred cubic
2 2 6

3
3a 4  3a 3
3. Body-centered cubic r   0.68
4 3  4  8

Summary of the Four Kinds of Packing for Spheres

Structure Stacking pattern C.N. Space used (%) Unit cell


1. Simple cubic AăAăAăA 6 52 Primitive cubic
2. Body-centered cubic AăBăAăB 8 68 Body-centred cubic
3. Hexagonal AăBăAăBă 12 74 None-cubic
closest-packed
4. Cubic closest-packed AăBăCăAăBăC 12 74 Face-centred cubic

THE SOLID STATE


24 QUIZRR
Example 8

The length of the unit cell edge of a ÂbccÊ lattice metal is 352 pm. Calculate radius of the
atom of the metal.
Solution:
In a ÂbccÊ structure, the atoms touch each other along the body diagonal.

A
R
body diagonal a
2R
a
a
a R
2a
a B

Let radius of the atom = R

t hen body diagonal = R  2 R  R  4R  3 a

3 3
 R a  352 = 152.4 pm
4 4

Radius Ratio Rules

For the stability of an ionic compound, each cation should be surrounded by maximum number
of anions and vice verse. The number of oppositely charged ions surrounding each ion is called
its co-ordination number.
The larger is the size of the cation the greater is its coordination number. In other words , greater
is the radius ratio the greater is its coordination number.
The relationship between the radius and the coordination number and the structural arrangement
are called radius ratio rules. The radius ratio rule is given in the table below.

rc
Limiting radius ratio, x Co-ordination number Shape Example
ra

x < 0.155 2 Linear BeCl2


0.155 < x < 0.255 3 Triangular plannar BCl2
0.255 < x < 0.414 4 Tetrahedral ZnS, BeO
0.414 < x < 0.732 4 Square planar [PtCl4]2ă
0.414 < x < 0.732 6 Octahedral NaCl, CaO
0.732 < x < 0.999 8 Body centered cubic CsCl
x= 1 12 Ideal FCC Fe, Al, Ag

THE SOLID STATE


QUIZRR 25

 Radius of cation (r + )  r 
Radius ratio =  ă 
 
 Radius of anion (r )  r

The above relation may be understood with the following explanation.


For a most stable arrangement the anions must touch each other as well as the cation
simultaneously.
For e.g., in a planar triangular structure, the ideal arrangement is represented in the figure.

But if the size of the cation increases (Keeping the size of anions same) then the anions will no
longer be touching each others as shown in the figure.

So, readjust each other in such a way that they touch each other as well as the cation. Under
such situation four anions may be touching the cation.

THE SOLID STATE


26 QUIZRR
The leads to a tetrahedral arrangement.
Similarly in case of an octahedral arrangement, if the size of the cation decrease i.e., the radius
ratio decreases the arrangement changes to tetrahedral to become stable. But if the size of the
cation increases i.e., the radius ratio increases the arrangement changes to body centred cubic
structure to acquire stability.

Tetrahedral

r+ <0.414

Octahedral Void

r+ <0.732

Body Centre arrangement

R adius Ratio for Co-ordination Number 3 (Triangular Arrangement)

The radius ratio for the triangular arrangement of ions can be obtained as follows.

A E C

In triangular arrangement (co-ordination number 3), a cation of radius rc is surrounded by 3


anions of radius ra. The cation is touching all the three anions and anions are also in contact with
each other, an arrangement which gives limiting radius ratio for co-ordination number 3. From
figure it is obvious that

THE SOLID STATE


QUIZRR 27

AB = BC = AC = 2ra
AD = rc + ra and AE = ra
The angle BAC is 62Ĉ and the angle DAE is 30Ĉ, then by trigonometry,

AE ra
Cos 30Ĉ = 
AD rc  ra

3 ra

2 rc  ra

Taking reciprocal,

rc  ra 2

ra 3

rc 2
or 1
ra 3

rc 2 2 3
 1  0.155
ra 3 3

Radius ratio for co-ordination number 4 (tetrahedral arrangement)

In the limiting tetrahedral arrangement, a cation of radius, rc is touching all four anions of
radius, ra and the anions are also in contact with each other.
From figure, it is obvious that
a
AB = BC = CD = DA = 2ra =
2

3a
and AE = BE = CE = DE = rc + ra = B
4
A
rc  ra 3a 2
  
2ra 4 a E

rc  ra 3

1ra 2 D

C
rc 3
or 1
ra 2

rc 3
  1  0.225
ra 2
THE SOLID STATE
28 QUIZRR
R adius ratio for co-ordination number 6 (octahedral arrangement) or radius ratio for
co-ordination number 4 (square planar arrangement)

In the limiting octahedral arrangement, a cation of radius rc is in contact with six anions of radius
ra and the anions are also in contact with each other.
Draw right angle triangle passing through the centre of the three anions. The cation will lie in
the middle of the hypotenuse. From figure, it is obvious that

a
BC = AB = 2ra =
2

and AC = 2(rc + ra) = a


2rc + 2ra
2(rc  ra ) 2
  a
2ra a A

D
rc  ra
 2
ra C B

rc 2ra
or  1 2
ra Top view of octahedral arrangement

rc
 2  1  0.414
ra

Radius Ratio for Co-ordination Number 8 (Body Centered Cubic Crystal)


In the limiting cubical (body centered cubic) arrangement, a cation of radius, rc is touching all
the eight anions of radius, ra and the anions are also in contact with each other.
From the figure, it is obvious that
AB = BC = CD = 2ra = a

and AD = 2(rc + ra) = 3a


A
2(rc  ra ) 3a
 
2ra a B
E
2ra
rc
or  1 3 C
ra 2rc + 2ra
D
rc
 3  1  0.732
ra

THE SOLID STATE


QUIZRR 29

Radius ratio rules are used for finding out the type of crystal structure in which ionic solids will
crystallise, depending on their co-ordination number. In practice, the radius ratio rules are most
reliable when the cation co-ordination number is 8, less reliable with 6 co-ordinate cations and
unreliable for 4 co-ordinate cations.

Example 9

In diamond, carbon atoms occupy FCC lattice points as well as alternate tetrahedral voids.
The edge length of the unit cell is 356.7 pm. Calculate
(a) the closest distance between carbon atoms and
(b) the fraction of total volume occupied by carbon atoms.
Solution :
If a sphere is present in a tetrahedral void surrounded by same kind of spheres, then the spheres
present at corners of tetrahedron will not touch each other but the sphere in tetrahedral void
touches all spheres present at the corner of the tetrahedron.

3a 3  356.7
 2rc  , 2rc   154.45 pm
4 4

154.45
 Radius of carbon atom =  77.225 pm
2

 
4 22 3
8   77.225  1012
Fraction of total volume occupied by carbon atoms = 3 7  0.34
356.7  10 
3
12

Example 10

In a face-centered unit cell with all the lattice positions occupied by A atoms, the body-
centered octahedral hole in it is occupied by atom B of appropriate size. For such a crystal,
calculate the void space per unit volume of unit cell. Also predict the formula of the
compound.
Solution :
Let ÂaÊ be the edge length of the cube. For FCC unit cell,

4 rA  2 a

or a  2 2 rA

Now, since the atom B occupies bodyăcentered octahedral hole, it is obvious that
2rA + 2rB = a

THE SOLID STATE


30 QUIZRR

or 2rA + 2rB = 2 2 rA

or 2rB   
2  1 2rA

rB
or  2  1  1.414  1.0  0.414
rA

Volume of the cube = a3  16 2 rA3

 4 3 4 3 4
3  3  3
3 3
Volume occupied by A and B atoms =  4  rA    rA    4 rA  rA

 
Volume occupied by atoms per unit volume of unit cell is given by

4
3

 4rA3  rA3 



[4 rA3  (0.414rA )3 ]   rA3 [4  (0.414)3 ]
  0.754
Packing fraction =
16 2 rA3 12 2 rA3 12 2  rA3

 Void fraction = 1 ă 0.754 = 0.246

1  1 
Number of effective A per unit cell =   8     6   4
8  2 

Number of effective B per unit cell = 1


 Formula of the compound = A4B

TYPES OF VOIDS
In closest packing, we have observed two types of voids ÂaÊ and ÂbÊ. The voids of ÂaÊ type are called
tetrahedral voids and voids of ÂbÊ type are called octahedral voids. These tetrahedral and octahedral
voids are present only in FCC and HP unit cells.

Tetrahedral Void

A tetrahedral void is formed when an atom fits


into the depression formed by three other closest
packed atoms (forming an equilateral triangle). All
Tetrahedral void
the four atoms are in contact with each other. If
we join the centre of all four atoms, we get a
tetrahedron. The centre of tetrahedron represents
a tetrahedral void.

THE SOLID STATE


QUIZRR 31

Now let us see the location of tetrahedral void in a FCC lattice. Let us consider a FCC unit cell
and assume that corner is the origin of FCC unit cell. We have to find three other atoms with
which this corner atom is in contact with. An atom in FCC unit cell touches another atom, if they
a
ar e pr esent at a dist ance of , where a is the edge length of the unit cell. Thus, this corner atom
2
will touch the face centered atom of those faces which will meet at this corner. When the centre
of these four atoms are joined, it gives a tetrahedron and the centre of tetrahedron would be the
location of tetrahedral void.

Tetrahedral
void

Number of tetrahedral voids per FCC unit cell

The exact location of tetrahedral void in FCC unit cell can also be traced by cutting the FCC unit
cell into eight minicubes by three perpendicular cuts. The atoms in the minicube are present at
the alternate corners of the minicube and these atoms actually touch one another forming a
tetrahedron with a hole (void) in the centre. This tetrahedral void would be the exact centre of
minicube. Since, there are eight such minicubes, so there are eight tetrahedral voids in FCC unit
cell. This is twice the number of effective atoms in FCC unit cell. Thus, in general the number
of effective tetrahedral voids in a unit cell is double the number of effective atoms in that unit
cell.
Let the radius of host atom (forming FCC lattice) be rh while the
radius of foreign atom entering a tetrahedral void in an ideal
FCC lattice be rf , then the sum of radii of host and foreign atom
in terms of edge length (a) is given by

3a
 rh  rf   4
ăCentre of tetrahedral void

THE SOLID STATE


32 QUIZRR
Example 11

Compute the percentage void space per unit volume of unit cell in zinc fluoride structure.
Solution :
Anions occupy fcc positions and half of the tetrahedral holes are occupied by cations. Since there
are four anions and 8 tetrahedral holes per unit cell, the fraction of volume occupied by spheres
per unit volume of the unit cell is

4  1 4 
4   ra3    8   rc3 
   rc  
3
  3  2  3   1    
16 2ra3 3 2
  ra  

..
. for tetrahedral holes,

rc
 0.225
ra


3

2
1  (0.225)   0.7493
3

 Void volume = 1 ă 0.7493


= 0.2507/unit volume of unit cell
% void space = 25.0%

Locating Tetrahedral and Octahedral Voids

The close packed structures have both octahedral and tetrahedral voids. In a ccp structure, there
is 1 octahedral void in the centre of the body and 12 octahedral void on the edges. Each one of
which is common to four other unit cells. Thus, in cubic close packed structure.
Octahedral voids in centre of cube = 1

1
Effective number of octahedral voids located at the 12 edge of = 12  3
4

 Total number of octahedral voids = 4


In ccp structure, there are 8 tetrahedral voids. In close packed structure, there are eight spheres
in the corners of the unit cell and each sphere is in contact with three groups giving rise to eight
tetrahedral voids.

THE SOLID STATE


QUIZRR 33

o
o o
o
T T
T T
o o
o
o o
T T
T T o
o o
o

Circles labelled T represent the centers of Circles labelled O represnt centers of the
the tetrahedral interstices in the ccp octahedral interstices in the ccp arrangement of
arrangement of anions. The unit cell ÂÂownsÊÊ anions (fcc unit cell). The cell ÂownsÊÊ 4 octahedral
8 tetrahedral sites. sites.

Example 12

In a crystalline solid, having formula AB2O4, oxide ions are arranged in cubic close packed
lattice while cations A are present in tetrahedral voids and cations B are present in
octahedral voids.
(i) What percentage of the tetrahedral voids is occupied by A ?
(ii) What percentage of the octahedral voids is occupied by B ?
Solution :
In a cubic close packed lattice of oxide ions there would be two tetrahedral voids and one octahedral
void for each oxide ion.
 For four oxide ions there would be 8 tetrahedral and four octahedral voids two are occupied
by B.

1
Percentage of tetrahedral voids occupied by A =  100  12.5%
8

2
Percentage of tetrahedral voids occupied by B =  100  50%
4

Example 13

Iron (II) oxide, FeO, crystal has a cubic structure and each edge of the unit cell is 5A.
Taking density of the oxide as 4 g/cc, calculate the number of Fe2+ and O2ă ions present in
each unit cell.
Solution :
Volume of the unit cell = (5 ï 10ă8 cm)3 = 1.25 ï 10ă22 cm3.

THE SOLID STATE


34 QUIZRR
Density of FeO = 4 g cmă3
Therefore mass of unit cell = 1.25 ï 10ă22 ï 4 = 5 ï 10ă22 g.

72gmol-1
= 22 -1
= 1.195 ï 10-22 g.
6.022 ï 10 mol

5 ï 10-22
Number of FeO molecules per unit cell = 4
1.195  1022

CLASSIFICATION OF IONIC STRUCTURES

In the following structures, a black circle would denote a cation and a white circle would denote
an anion. In any solid of the type AxBy, the ratio of the coordination number of A to that of B
would be y : x.

Structure of Ionic Compounds

Simple ionic compounds are of two types i.e. AB and AB2 type. From the knowledge of close
packed structures and the voids developed there in, we can have an idea about the structures of
simple ionic compounds.
Among the two ions, constituting the binary compounds, the anions usually constitute the space
lattice with hcp or ccp type of arrangements whereas the cations, occupy the interstitial voids.
(a) If the anions (Bă) constitute the crystal lattice and all octahedral voids are occupied by
cations (A+), then the formula of the tetrahedral voids are occupied by cations, then the
formula of the solid crystal becomes A+Bă
(b) When the anions (Bă2) are constituting space lattice and all the tetrahedral voids are
occupied by the cations (A+), then the formula of the solid crystal will be A2B.

Ionic Compounds of the type AB

Ionic compounds of the type AB have three types of crystalline structures. (A) ZnS type (B) NaCl
types (C) CsCl types
1. Sodium Chloride (Rock salt) type structure
The sodium chloride structure is composed of Na+ and Clă ions. The number of sodium ions is
equal to that of Clă ions. The radii of Na+ and Clă ions 95 pm and 181 pm giving the radius ratio
of 0.524

r
Na  95
  0.524
r 181
Cl ă

THE SOLID STATE


QUIZRR 35

The radius ratio of 0.524 for NaCl suggest an octahedral void. Thus the salient features of this
structure are as follows :
(i) Chloride ions (In a typical unit cell) are arranged in cubic close packing (ccp). In this
arrangement Clă ions are present at the corners and at the centre of each face of the cube.
This arrangement is also regarded as face centred cubic arrangement (fcc).
(ii) The sodium ions are present in all the octahedral holes.
(iii) Since, the number of octahedral holes in ccp structure is equal to the number of anions,
every octahedral hole is occupied by Na+ ions. So that the formula of sodium chloride is NaCl
i.e. stoichiometry of NaCl is 1:1.

ă
Cl
+
Na

Unit cell structure of NaCl

(iv) Since there are six octahedral holes around each chloride ions, each Clă ion is surrounded
by 6 Na+ ions. Similarly each Na+ ion is surrounded by 6 Clă ions. Therefore, the coordination
number of Clă as well as of Na+ ions is six. This is called 6:6 coordination.
(v) It should be noted that Na+ ions to exactly fit the octahedral holes, the radius ration
r r  
Na 
should be equal to 0.414. However, the actual radius ration  Na  0.524  exceeds
r ă r ă 
Cl
+ ă
 Cl 
this value. Therefore to accommodate large Na ions, the Cl ions move apart slightly i.e.
they do not touch each other and form an expanded face centred lattice.
(vi) The unit cell of sodium chloride has 4 sodium and 4 chloride ions as calculated below
No of sodium ions = 12 (at edge centres) ï 1/4 + 1 (at body centre) ï 1 = 4
No of chloride ions = 8(at corner) ï 1/8 + 6 (at face centres) ï 1/2 = 4
Thus, the number of NaCl units per unit cell is 4.
(vii) The edge length of the unit cell of NaCl type of crystal is 2(r + R) (r = radii of Na+ ion)

a2 r  Na 
r
Cl ă  (R = radii of Clă ion)

THE SOLID STATE


36 QUIZRR
Thus, the distance between Na+ and Clă ions = a/2
Most of the halides of alkali metals, oxides and sulphides of alkaline earth metals have this
type of structures. Some of the common examples are NaI, KCl, Rbl, RbF, NH4Cl, NH4Br,
AgCl, AgBr and AgI.

2. Zinc blende (ZnS) type structures (sphelerite)


The zinc sulphide crystals are composed of equal number of Zn+2 and S2ă ions. The radii of the
two ions (Zn+2 = 74 pm and Să2 = 184 pm) led to the radius (r+/ră) as 0.40 which suggests a
tetrahedral arrangement.

r
Zn2 74
  0.40
r 2 184
s

The salient features of this structure are as follows :


(i) The Zinc ions are arranged in ccp arrangement, i.e. sulphide ions are present at the corners
and the centres of each face of the cube
(ii) Zinc ions occupy tetrahedral hole. Only half of the tetrahedral holes are occupied by Zn+2
so that the formula of the zinc sulphide is ZnS i.e. the stoichiometry of the compound is 1:1
(Only alternate tetrahedral holes are occupied by Zn+2)
(iii) Since the void is tetrahedral, each zinc ion is surrounded by four sulphide ions and each
sulphide ion is surrounded tetrahedrally by four zinc ions. Thus zinc sulphide has 4.4 Coă
ordination.
(iv) For exact fitting of Zn+2 in the tetrahedral holes, formed by close packing of Să2 ions, the
ratio Zn+2/Să2 should be 0.225. Actually this ratio is slightly large (0.40)
(v) There are four Zn+2 ions and four Să2 ions per unit cell as calculated below :
No. of Să2 ions = 8(at corners) ï 1/8 + 6(at face centres) ï 1/2 = 4
No. of Zn+2 ions = 4 (within the body) ï 1 = 4 Thus, the number of ZnS units per unit cell
is equal to 4. Some more examples of ionic solids having Zinc blende structures are CuC,
CuBr, CuI, AgI, beryllium sulphide.

3a
(vi) Radius Relation : r  r 2 
Zn2 s 4
S2ă
where a is the edge length.
Zn2+

THE SOLID STATE


QUIZRR 37

3. The Wurtzite Structures


It is an alternate form in which ZnS occurs in nature. The main features of this structure are
(i) Sulphide ions have HCP arrangement and zinc ions occupy tetrahedral voids.
(ii) Only half the alternate tetrahedral voids are occupied
by Zn+2 ions.
(iii) Coordinate no. Of Zn+2 ions as well as Să2 ions is 4.
Thus, this structure has 4:4 coordination.
+2
Zn
(iv) No. of Zn+2 ions per unit cell = 4(within the unit cell) ă2
S
1
ï 1 + 6 (at edge centres) ï 6
3
No. of Să2 ions per unit cell

1 1
= 12(at corners) ï  2 (at face centres) ï 3
6 2
(within the unit cell) ï 1
= 6 A unit cell representation of
wurtzite structure
Thus, there are 6 formula units per unit cell

4. Cesium Chloride (CsCl) Structure


The cesium chloride crystal is composed of equal number of cesium (Cs+) and Chloride Clă ions.

The radii of two ions (Cs+ = 169 pm and Clă = 181 pm) led to radius ratio of rCs to rCl as 0.93

which suggest a body centred cubic structure having a cubic hole.

r 169
Cs+
  0.93
r 181
Cl ă

The salient features of this structure are as follows :


(i) The chloride ion form the simple cubic arrangement and the cesium ions occupy the cubic
interstitial holes. In other words Clă ions are at the corners of a cube where Cs+ ion is at
the centre of the cube or vice versa
(ii) Each Cs+ ion is surrounded by 8 Clă ions and each Clă ion in surrounded by 8 Cs+ ions. Thus
the Co-ordination number of each ion is eight.

r
(iii) For exact fitting of Cs+ ions in the cubic voids the ratio Cs+ should be equal to 0.732.
r
Cl ă
However, actually the ratio is slightly larger (0.93). Therefore packing of Clă ions slightly
open up to accommodate Cs+ ions.

THE SOLID STATE


38 QUIZRR
(iv) The unit cell of cesium chloride has one Cs+ ion and one Clă ion
as calculated below

No. of Clă ion 8(at corners) ï 1/8 = 1

No. of Cs+ ion = 1 (at body centre) ï 1 = 1

Thus, number of CsCl units per unit cell is 1 Caesium halide structure

a 3
(v) Relation between radius of cation and anion and edge length of the cube, rCs  rCl 
2
Other common examples of this type of structure are CsBr, CsI, TICl, TIBr
Higher coordination number in CsCl(8:8) suggest that the cesium chloride lattice is more
stable than the sodium chloride lattice in which Co ă ordination number is 6:6. Actually the
cesium chloride lattice is found to be 1% more stable than the the sodium chloride lattice.
Then the question arises why NaCl and other similar compounds do not have CsCl type
lattice ă This is due to their smaller radius ratio. Any attempt to pack 8 anions around the
relatively small cation (Li+, Na+, K+, Rb+) will produce a state in which negative ions will
touch each other, sooner they approach a positive ion. This causes unstability to the lattice.

Effect of Temperature on Crystal Structure

Increase of temperature decreases the coordination of number, e.g. upon heating to 760 K, the
CsCl type crystal structure having coordination 8:8 changed to NaCl type crystal structures
having coordination 6:6

high temperature
CsCl type crystal 
 NaCl type crystal
(8:8 coordination) (6:6 Co-ordination)

Effect of Pressure on Crystal Structure

Increase of pressure increases the Co ă ordination number during crystallization e.g. by applying
pressure, the NaCl type crystal structure having 6:6 coordination number changes to CsCl type
crystal having coordination number 8:8

high pressure
NaCl type crystal 
 CsCl type crystal
(6:6 coordination) (8:8 Co-ordination)

Ionic Compound of the Type AB2

Calcium Fluoride (Fluorite) Structure


The salient features of fluorite structure are
(i) The Ca+2 ions are arranged in ccp arrangement, i.e. these ions occupy all the corners and the
centres of each face of the cube.

THE SOLID STATE


QUIZRR 39

(ii) The Fă ions all the tetrahedral holes.


(iii) Since there are two tetrahedral holes for each Ca+2 ion and Fă ions occupy all the tetrahedral
holes, there will be two Fă ions for each Ca+2 ions, thus the stoichiometry of the compound is 1:2.
(iv) Each Ca+2 ion is surrounded by 8F+ ions and each Fă ions is surrounded by 4Ca+2 ions. The
Coordination number of Ca+2 ion is eight and that of Fă ion is four, this is called 8:4 Coordination.
(v) Each unit cell has 4 calcium ions and 8 fluoride ions as explained below
No. of Ca+2 ions = 8(at corners) ï 1/8 + 6 (at face centres) ï 1/2
No. of F ions = 8 (within the body) ï 1 = 8
Thus the number of CaF2 units per unit cell is 4.
Other examples of structure are
SrF2, BaCl2, BaF2, CdF2, HgF2, CuF2, SrCl2, etc.

3a
(vi) r r  where a is the edge length
Ca2 F 4

Unit cell representation of CaF2 structure

Ionic Compound of A2B Type

The compound having A2B formula are compounds having anti fluorite structure
Anti fluorite structure is having arrangement of cations and anions opposite to the fluorite structure
Li2O has an anti fluorite structure.
(i) In the crystal structure of Li2O the Oă2 ions constitute a cubic close packed lattice (fcc structure)
and the Li+ ions occupy all the tetrahedral voids.
(ii) Each oxide ion, Oă2 ion is in contact with 8 Li+ ions and each Li+
ions having contact with 4 oxide ion. Therefore Li2O has 4:8
coordination
Examples ă Na2O, K2O, K2S, Na2S, Rb2O, Rb2S

3a
(iii) r r  where a is the length.
Li+ O2 4
THE SOLID STATE
40 QUIZRR
Normal Spinel Structure

Spinel is a mineral MgAl2O4. In it oxide ions are arranged in ccp with Mg+2 ions occupying
tetrahedral voids and Al+3 ions in a set of octahedral voids. Many ferrites (such as ZnFe2O4) also
possess spinel structure. these are very important magnetic materials and are used in telephone
and memory loops in computers.

Structure of Fe 3O 4 (Magnetite)

In Fe3O4, Fe+2 and Fe+3 ions are present in the ratio 2:1. It may be considered as having
composition FeO.Fe2O3. In Fe3O4 Oxide arranged in ccp. Fe+2 ions occupy octahedral voids while
Fe+3 ions are equally idstributed between octahedral and tetrahedral voids.
MgFe2O4 also has structure similar to magnetite. In this Mg+2 ions are present in place of Fe+2
ion in Fe3O. Magnetite has inverse spinel structure.

Summary of various structures of ionic solids


Crystal Brief description Coordination No. of Examples
Structure atoms per
unit cell
ă +
1. Rock salt Cl ions in ccp Na = 6 4 Li, Na, KI, and Rb
(NaCl ă type) Na+ ions occupy Clă = 6 halides
all octahedral voids NH4Cl, NH4Br, NH4I,
AgF, AgCl, AgBr, MgO,
CaO, TiO, FeO, NiO
2. Zinc blende Să2 ions in ccp Zn+2 = 4 4 Zns, BeS, CuCl, CuBr,
(ZnS ă types) Zn+2 ions occupy Să2 = 4 Cul, Agl, HgS
alternate
tetrahedral voids
3. Wurtzite Să2 ions in hcp Zn+2 = 4 4 ZnS, ZnO, CdS, BeO
(ZnS ă type) Zn+2 ion occupy Să2 = 4
alternate
tetrahedral voids
4. Cesium Clă ions in bcc Cs+ Cs+2 = 8 1 CsCl, CsBr, CsI, CsCN,
chloride ions in the body of Clă1 = 8 CaS
(CsCl type) cube
5. Fluorite Ca+2 ions in ccp, Ca+2 = 8 4 CaF2, SrF2, BaF2,
(CaF2 type) Fă ions occupy all Fă = 4 BaCl2, SrCl2, CdF2,
tetrahedral voids HgF2
6. Anti fluorite Oă2 ions in ccp, Li+ Li+ = 4 4 K2O, Na2O, K2S, Na2S
(Li2O ă type) ions occupy all Oă2 = 8
tetrahedral sites

THE SOLID STATE


QUIZRR 41

Example 14

The unit cell cube length for LiCl is 5.14 A . Assuming anion cation contact, calculate the
ionic radius for chloride ion.
Solution :
The distance between Li+ and Clă ion can be derived as half of the edge length of cube.

5.14 
d   2.57 A
Li+ - Cl- 2
Cl

d
Cl ă ă Cl ă
  2.572   2.572  3.63 A a
2.57 A
The radius of Clă
Cl
Li
d 2.57 A
= Cl ă ă Cl ă 3.63 
  1.815 A
2 2

Example 15

In fcc arrangement of A & B atoms, where A atoms are at corners of the unit cell, B atoms
at the face-centers, one of the atoms are missing from the corner in each unit cell then find
the percentage of void space in the unit cell.
Solution :

7
There are A atoms and 3 B atoms per unit cell
8

Also, 2 rA + 2 rB = 2a

or, a  2 rA  2 rB

 
3
Volume of unit cell = a3  2 rA  2 rB

 2 2 (rA  rB )3
Fraction of volume occupied per unit volume of the unit cell is given by

7 4 3  4  7 3
  rA    3  rB3  rA  4rB3
8 3   3  6
PE = 
2 2  rA  rB  2 2  rA  rB 
3 3

. . rA  0.414
. r
B

THE SOLID STATE


42 QUIZRR

7 3
  rA3  4  0.414 rA   rA3  7  4   0.414 3 rA3
   
6 6
2 2  rA  0.414 rA  2 2 rA3 1  0.414 
3 3

7 3
  4  0.414  
  6   0.570

2 2 1  0.414 3

So void space = 1 ă packing fraction = 1 ă 0.570 = 0.430/unit volume of unit cell


 Percentage of void space = 43%

Example 16
Prove that the void space percentage in zinc blende structure is 25%.
Solution :
Anions are in fcc positions and half of the tetrahedral holes are occupied by cations. There are
4 anions and eight tetrahedral holes per unit cell.

Here face-diagonal = 2a
4r = 2a

or, a  2 2 r
ră = radius of anion
r+ = radius of cation
a = edge length of the cell

 
3
Volume of unit cell a  2 2 r  16 2 r3

4  1 4 
4   r3    8   r3    r  
3
 3  2  3  
 Packing Fraction = 1    
 r  
3
16 2 r 3 2 

r
Since, for tetrahedral holes  0.225
r

 1   0.225 3   0.7496 / unit volume of unit cell


 PF 
3 2  

 void space = 1 ă 0.7496 = 0.2504/unit volume of unit cell


 Percentage of void space = 25.04%  25%
THE SOLID STATE
QUIZRR 43

Example 17

(a) Spinel is an important class of oxides consisting of two types of metal ions with the
oxide ions arranged in CCP pattern. The normal spinel has oneăeight of the tetrahedral
holes occupied by one type of metal ion. Such a spinel is formed by Zn2+, Al3+ and O2ă
with Zn2+ in the tetrahedral holes. Give formula of the spinel.
(b) If all the species in problem (a) touch each other, determine the fraction of the volume
occupied by ions in the unit cell.
Solution :

1  1 
(a) Number of oxide ions per unit cell =   8     6   4,
8  2 

1 
Number of Zn2+ ions per unit cell =   8   1
 8 

1 
and number of Al3+ ions per unit cell =   4   2
 2 

Thus, the formula of spinel is ZnAl2O4.

(b) For FCC lattice, 4r  2 a,


O2ă

For octahedral void, r


O2ă
r
Al3+   a2
and for tetrahedral void, r  O2?
r
Zn3+  3a
4

 a2 2r
O2ă

r r  2r
O2ă Al 3+ O2ă

 r
Al3+
  2 1 r  O2ă  0.414 rO2ă
3  2 2r 3
O2ă
and r r   r 2ă  1.225 r 2ă
O2ă Zn 2+ 4 2 O O

 r  0.225 r
Zn2 O2ă

THE SOLID STATE


44 QUIZRR


    1  43  0.225 r    2  43  0.414 r  
4 3 3 3
4  3  rO2ă O2ă O2ă
 Packing fraction = 
2 2 r 
3
O2ă

 
4 3
 r 2ă  4   0.225 3  2  0.414 3 
3 O     4.15329
  0.77
 
= 3
12 2
16  2 r
O2ă

Example 18

XY is an ionic solid which crystallizes in BCC arrangement where the radius of X+ and Yă

are respectively 1.10 A and 1.64 A . If all the X+ ions are replaced by A+ so as to form AY
(which is also in BCC arrangement), the density is found to be 3.932 g/cc per unit cell. Out
of XY and AY, which one is more ionic ? Atomic mass of A+ and Yă are respectively 22 and
30 amu.
Solution :
In case of AY,

1  22   1  30 
  3.932
6.023  1023  a3


a = 2.8 A
In BCC,


3a  2 r
A+
r
Yă 
 
3  2.8 A  2(r  1.64) A
A+


r  0.784 A
A+

Since the radius of A+ is smaller than that of X+, its polarization is more and hence AY is more
ionic than XY.

THE SOLID STATE


QUIZRR 45

Example 19

NH4Cl crystallizes in a body centered cubic lattice, with a unit cell distance of 387 pm.
Calculate
(a) the distance between the oppositely changed ions in the lattice, and

(b) the radius of the NH+4 ion if the radius of the Clă ion is 181 pm.

Solution :
(a) In a body centered cubic lattice oppositely charged ions touch each other along the cross diagonal
of the cube. Hence, we can write,

3a 3
2r   2r   3a, r   r     387 pm   335.15 pm.
2 2

(b) New, since ră = 181 pm, we have r+ = (335.15 ă 181) pm = 154.15 pm.

Example 20

r r
Na  Na 
KCl crystallizes in the same type of lattice as does NaCl. Given that,  0.5 ,  0.7
r r
Cl  K
Calculate (a) the ratio of the side of the unit cell for KCl to that for NaCl and
(b) the ratio of density of NaCl to that for KCl.
Solution :
a
NaCl crystallizes in the face centered cubic unit cell such that r r  , where a is the edge
Na  Cl  2
r  r 
Na
length of unit cell. No since, r  0.5 , Na  0.7
 r 
Cl K

r r
Na  Cl 
we will have,  1.5
r
Cl 

r 1 r + 1 rK + 0.5
K+
 ; K  ; 
r 0.7 0.5r 0.7 r  0.7
Na Cl  Cl

r r
K Cl  1.2

r 0.7
Cl 

r r
K Cl  1.2 1
Hence,  
r  r 0.7 1.5
Na Cl 

THE SOLID STATE


46 QUIZRR

aKCl
2  1.2 a n  M 
 KCl  1.143 : Now since,  = 3  
aNaCl 0.7  1.5 aNaCl a  NA 
2

3
NaCl  aKCl   M NaCl  3  58.5 
We will have,      (1.143)    1.172
KCl a M
 NaCl   KCl   74.5 

CRYSTAL DEFECTS

Till now, we have assumed that all lattice structures were perfect or ideal but in real practice,
crystal lattices have substantial number of defects. These defects affect the mechanical, electrical
and optical properties of the solids.
We will discuss about the crystal defects (lattice imperfections) in lattices made up of atoms only
and also in ionic lattices. Let us first consider lattice imperfections in lattices made up of atoms
only. The lattice imperfections are classified according to their geometric characteristics. There are
crystal defects of four types in such lattices. These are
(a) Vacancy (b) Self interstitial
(c) Substitutional impurity (d) Interstitial impurity
If an atom is missing from a lattice site, it creates a vacancy. Due to vacancy, density of the crystal
decreases. Due to absence of atoms from lattice sites, the occupancy of the lattice sites decreases
and the percentage occupancy of such crystals can be determined as

No. of atoms actually present at the lattice site


Percentage occupancy =
No. of atoms therorectically present at the lattice
site assuming no vacancy defect

Observed density
=  100
Theoretical density

Example 21

Li forms a body centered cubic lattice. If the edge length of the lattice is 3.5 ï 10ă10 m and
the observed density is 5.3 ï 102 kg mă3, calculate the percentage occupancy of lattice
points by Li metal.
Solution :

nM
Theoretical density obtained by is with the assumption that each lattice point is occupied
N AV  V
by the constituent species. But generally, some lattice points are not occupied by the constituent
particles due to some defect in the crystal lattice. These points are called vacancies. The density

THE SOLID STATE


QUIZRR 47

obtained considering vacancies is called observed density and is surely less than the theoretical
density.

2  7  103
Theoretical density =  5.42  102 kgm 3
 
3
6.023  1023  3.5  1010

Observed density
% occupancy =  100
Theoretical density

5.3  102
% occupancy =  100  97.78%
5.42  102

If an atom leaves its lattice site and occupies some interstitial space, then the defect is referred
as self interstitial. Due to this defect, density of the crystal remains unaffected.
If a foreign atom displaces or substitutes a host atom (which forms lattice) and occupies its lattice
site, the defect is called substitutional impurity. Such defects can alter the density of the crystal
depending upon the mass of the replacing atom relative to replaced atom.
If the foreign atom do not occupy a lattice site but occupies some interstitial space, then the defect
is referred as interstitial impurity. Due to such defects, the density of the crystal increases.

Self interstitial
Vacancy

Interstitial
impurity

Substitutional impurity

Figure, Type of crystal defects in a lattice made up of atoms only.


Now, let us consider these crystal imperfections in ionic crystals.
A vacancy at a cation site is mostly accompanied by a vacancy at a nearby anion site. Such paired
cationăanion vacancies are referred as Schottky defect. Such defect preserves the electrical
neutrality of the crystal but the density of crystal decreases.
When an ion (cation or anion) leaves its lattice point and occupies some interstitial space, the
defect is called Frenkel defect. This defect also preserves the electrical neutrality of the crystal
and the density of the crystal also remains unaltered.

THE SOLID STATE


48 QUIZRR
Frenkel defects arise easily when the anions are larger and the cations are smaller in size or when
the crystal structure is of an open type with large interstitial spaces.
Let us have a solution containing Ba2+ ions, contaminated with some Sr2+ ions and we want to

crystallize BaSO4 by adding SO2ă


4 ions to the solution. Since, Ba2+ and Sr2+ ions have same
charge and nearly similar size, so during the formation of BaSO4 lattice, inevitably some Sr2+ will
also be incorporated in place of Ba2+. Thus, Ba2+ is substituted by Sr2+ in BaSO4 lattice. Such
defects are called substitutional impurity defect. Due to such defects, the electrical neutrality is
preserved but the density of crystal changes. Such substitutional impurity defects are very common
in ionic compounds of the transition metals, as many of the transition metal ions are of the same
charge and are very nearly of the same size.
In a crystal lattice of titanium carbide, in which C4ă forms FCC lattice and Ti4+ occupies all
octahedral voids, when hydrogen gas is passed through such crystal, it is trapped in the tetrahedral
voids. So H2 acts as an interstitial impurity in TiC lattice. Due to such defects density of the
crystal lattice increases.
A simple anion vacancy in an ionic crystal creates an excess of positive charge. An electron can
be trapped and migrate to this anionic vacancy site, which is effect replaces the absent negative
ion. The presence of a number of such defects imparts colour to an otherwise colourless crystal.
This type of imperfection is called Făcentre (F stands for Farbe, a German word for colour).

Schottky
Frenkel defect
defect


Făcentre

Figur e, Type of cr yst al defect s in an ionic lat t ice

Example 22
Calcium crystallizes in a faceăcentered cubic unit cell with a = 0.556 nm. Calculate the
density if
(i) it contains 0.1% Frenkel defects (ii) it contains 0.1% Schottky defects.
Solution :
The density of crystal is unaffected due to Frenkel defect as the atom only occupies an interstitial
position instead of lattice site while Schottky defect changes the density of crystal.

THE SOLID STATE


QUIZRR 49

(i) The density of crystal when the crystal contained 0.1% Frenkel defects, would thus be

nM 4  40
=   = 1.5455 g/cm3
 
3 3
N AV  a 6.023  10 23
 0.556  10 7

(ii) Since the crystal contains 0.1% Schottky defect (in which the atoms are missing from lattice
site), so the mass of crystal decreases by 0.1% assuming the volume of crystal remain same.
Thus. the density of the crystal also decreases by 0.1%.

 0.1 
 = 1.5439 g/cm3
´    1   0.999   0.999  1.5455
 100 

Other Type of Defects


Metal Excess Defects
It has been observed that if a crystal of NaCl is heated in sodium vapour, it acquires a yellow
colour. This yellow colour is due to the formation of a non ă stoichiometric compound of sodium
chloride in which there is a slight excess of sodium ions. What happens in this case is that some
sodium metal gets doped into the sodium chloride crystal each atom of which gets ionised into Na+
and eă due to crystal energy. This electron occupies a site that would otherwise be filled by a
chloride ion as in figure (a).
ă ă + ă ă ă + ă
M X M X M X M X

ă + + ă + ă +
X M M X M X M

+
M

+ ă + ă + ă + ă
M X M X M X M X

ă + ă + ă + ă +
X M X M X M X M

Metal excess ă due to anion vacancies Metal excess ă due to interstitial castion
(a) (b)
Metal Deficiency Defects
In certain cases, one of the positive ions is missing from its lattice site and the extra negative
charge is balanced by some nearby metal ion acquiring two charges instead of one. There is,
evidently, a deficiency of the metal ions although the crystal as a whole is neutral. This type of
defect is generally found amongst the compounds of transition metals which can exhibit variable
valency as in figure (b).

THE SOLID STATE


50 QUIZRR
Example 23
If NaCl is doped with 10ă3 mol % SrCl2, calculate the concentration of cation vacancies ?
Solution :
Due to addition of SrCl2, each Sr2+ ion replaces two Na+ ions, but occupies only one Na+ lattice
point. This creates one cationic vacancy.
Number of moles of Sr2+ present in 100 mole of NaCl = 10ă3
Number of moles of cation vacancies in 100 mole of NaCl = 10ă3

10 3
Number of moles of cation vacancies in 1 mol =  10 5 mol
100
Total cationic vacancies = 10ă5 ï NAV = 10ă5 ï 6.02 ï 1023 = 6.02 ï 1018

Example 24
A compound AB has a rock salt type structure with A : B = 1 : 1. The formula weight
of AB is 6.023 Y amu and the closest AăB distance is Y1/3 nm.
(i) Find the density of lattice.
(ii) If the density of lattice is found to be 20 kgmă3, then predict the type of defect.
Solution :
Edge length of unit cell of AB = 2Y1/3 ï 10ă9 m

Zï M 4  6.023Y  10 3
(i) Density of AB =  kgm 3  5.0 kgm 3
 
N0 ï V 3
6.023  1023  2Y 1/3  109

(ii) Since, the observed density (20 kg mă3) of AB is higher than theoretical density, the compound
AB has either interstitial impurity defect or substitutional impurity defect.

PROPERTIES OF SOLIDS

Some of the properties of solids which are useful in electronic and magnetic devices such as,
transistor, computers, and telephones etc, are summarised below :

Electrical Properties :
Solids are classified into following classes depending on the extent of conducting nature :
Conductivity
(a) Metals : 108ă1 cmă1
(b) Insulators : 10ă12ă1 cmă1
(c) Semiconductors : 10ă5 to 106 ă1 cmă1
Ć Conductivity of the solids may be due to the movement of electrons, holes or ions.
Ć Due to presence of vacancies and other defects, solids show slight conductivity which increases
with temperature.

THE SOLID STATE


QUIZRR 51

Ć Metals show electronic conductivity.


Ć The conductivity of semiconductors and insulators is mainly governed by impurities and
defects.

Metal oxides and sulphides have metallic to insulator behaviour at different temperatures.

Conductivity
Insulator-like Insulator-to-metal Metal like
FeO, Fe2O3 Ti2O3 TiO
MnO, MnO2 V2O3 VO
Cr2O3 VO2 CrO2
CoO ReO3
NiO
CuO
V2O5

M agnetic Properties :

Based on the behaviour of substances when placed in the magnetic field, these are classfied into
five classes and summarised in Table on next page.

Dielectric Properties

When a non-conducting material is placed in an electrial field, the electrons and the nuclei in the
atom or molecule o that material are pulled in the opposite directions, and negative and possitive
charges are separated and dipoles are generated. In an electric field :
Ć These dipoles amy align themselves in the same direction, so that there is net dipole moment
in the crystal.
Ć These dipoles may align themselves in such a manner that the net dipole moment in the
crystal is zero.
Based on these facts, dielectric properties of crystals are summarised in Table 20.8.
When any material loses its resistance for electric current, then it is called superconductor.
Kammerlingh Onnes (1913) observed this phenomenon at 4K in mercury. The materials offering
no resistance to the flow of current at very low temperature (2-5K) are called superconducting
materials and phenomenon is called superconductivity.
Examples are :
(a) Nb3Ge alloy (before 1986) (b) La1.25 Ba0.15CuO4 (1986)
(c) YBa2Cu3O7 (1987)
Superconductors find applications in
(a) Electronics (b) building supermagnets
(c) levigation transport (d) power transmission.

THE SOLID STATE


52
Table : Magnetic Properties of Solids

Properties Description Alignment of Examples Application


Magnetic Dipoles
1. Diamagnetic Feebly repelled by the magnetic All paired electrons TiO2, V2O5, Insulator
fields. Non-metallic elements          NaCl, C5H6
(except O2, S) inert gases and (benzene)
species with paired electrons are
diamagnetic

2. Paramagnetic Attracted by the magnetic filed,          O2, Cu2+, Fe3+ Electronic
due to the presence of permanent at least one TiO, Ti2O3, appliances
magnetic dipoles (unpaired unpaired electron VO, VO2, CuO
electrons). In magnetic field, these
tend to orient themselves parallel
to the direction of the field and thus
QUIZRR

produce magnetism in the substance.

THE SOLID STATE


3. Ferromagnetic Permanent magnetism even in Dipoles are aligned Fe, Ni, Co, CrO2 CrO2 is used
the absence of magnetic field. in the same same in audio and
Above a temp. called Curie direction video tapes
temperature, there is no         
ferromagnetism.

4. Antiferromagnetic This arises when net dipole alignment            MnO, MnO2, Mn2O ă
is zero due to equal and opposite FeO, Fe2O3; NiO,
alignment Cr2O3, CoO, Co3O4,

5. Ferrimagnetic This arises when there is net dipole            Fe3O4, ferrites ă


moment
QUIZRR 53

Table : Dielectric Properties of Solids


Property Description Alignment Examples Applications
of Electric
Dipoles
1. Piezoelectricity when polar crystal is subjected
to a mechanical stress, electricity ă Record-players
is producedăa case of piezoel-
ectricity. Reversely if electric
field is applied, mechanical
stress is developed. Piezoelectric
Piezoelectric crystal acts as a
mechanicalăelectrical transducer.
Ć Piezoelectric crystals with      BaTiO3,
permanent dipoles are said KH2PO4
to have ferroelectricity Rochelle salt
Ć Piezoelectric crystals with      
zero dipole and said to have PbZrO3
antiferroelectricity.
2. Pyroelectricity Small electric current is product
due to heating of some of polar ă
crystal ăa case of pyroelectricity

Example 25

A strong current of trivalent gaseous boron passed through a germanium crystal decreases
the density of the crystal due to part replacement of germanium by boron and due to
interstitial vacancies created by missing Ge atoms. In one such experiment, one gram of
germanium is taken and the boron atoms are found to be 150 ppm by weight, when the
density of the Ge crystal decreases by 4%. Calculate the percentage of missing vacancies
due to germanium, which are filled up by boron atoms.
Solution :
We have to assume here that the decrease in density is not due to volume change but due to loss
of Ge atoms only. Let x by the total number of Ge atoms missing and y be the number of boron
atoms replacing germanium atoms. Assuming volume of the crystal remaining same, then 4%
decrease in density will also decrease weight of the crystal (sample) by 4%.

 72.6   11 
 1 ă x    y   0.96
 N AV   N AV 

 72.6   11 
0.04   x    y 
 N AV   N AV 

THE SOLID STATE


54 QUIZRR

 150 N AV 
where y    0.96 
 106  11
 

y  11 150  0.96
or 
N AV 106

 72.6   150  0.96 


0.04   x  
 N AV   106

(0.04  1.44  10 4 )N AV 0.04N AV


 x 
72.6 72.6

y 150 NAV  0.96  72.6


   2.376  102
x 6
10  11  0.04N AV

y
  100  2.376  102  100  2.376%
x

Example 26

Iron crystallizes in FCC lattice. The figures given below shows the iron atoms in four
crystallographic planes.

Draw the unit cell for the corresponding structure and identify these planes in the diagram.
Also report the distance between two such crystallographic planes in each case in terms
of the edge length (a) of the unit cell.
Solution :

Distance between two Distance between two Distance between two


such planes = a such planes = a such planes = a

THE SOLID STATE


QUIZRR 55

A C

3a a
Distance between two such planes  
3 3

Example 27

Calcium crystallises in a face-centred cubic unit cell with a = 0.556 nm. Calculate the
density, if
(i) it contained 0.1% Frenkel defects,
(ii) it contained 0.1% Schottky defects.
Solution :
Concept : Frenkel defects do not change the value of the theoretical density because the atom is only
occupying an interstitial position instead of lattice position. But in case of Schottky defects

 0.1 
z  4  1    3.996
 100 

Thus density can be determined using

Mz
d
N0 a3

(i) a = 0.556 mm = 0.556 ï 10ă9 m = 0.556 ï 10ă7 cm

40  4
d (with Frenkel defects) = = 1.5463 g/cm3
6.02  10 23

 0.556  10 
7 3

40  3.996
(ii) d(with Schottky defect) =
 
3
6.02  1023  0.556  107

= 1.5448 g/cm3

THE SOLID STATE


56 QUIZRR
Example 28

The effective radius of an iron atom is 1.42 A . It has a rock-salt structure. Calculate its
density (Fe = 56)
Solution :

a
Concept : Effective radius r  in ÂfccÊ structure, where a is edge length. Hence, a is known.
2 2

Density d is calculated using d  Mz


N0 a3

Due to rock-salt (fcc) structure, number of stems in a unit cell (z) = 4.

Mz
Thus, d(density) =
N0 a 3

a  2 2 r  2 2  1.42  108 cm

56  4
 d
 
3
6.02  1023  2 2  1.42  10 8

= 5.743 g/cm3

Example 29

Calculate the angle at which second-order reflection will occur in an X-ray spectrometer
when X-rays of 0.154 nm are diffracted by the atoms of a crytal having distance between
two planes as 404 pm.
Solution : n = 2a sin

n
 sin  
2a
Given, n = 2,  = 0.154 nm = 0.154 ï 10ă9 m,
 a = 404 pm = 404 ï 10ă12 m

2  0.154  10 9
 sin   = 0.3812
2  404  1012

                        = 22.41Ĉ

THE SOLID STATE


QUIZRR 57

Example 30

X-rays of wavelength 0.154 nm strike an aluminium crystal; the rays are reflected at an
angle of 19.3Ĉ. Assuming that n = 1, calculate the spacing between the planes of aluminium
atoms that is responsible for this angle of reflection.
Solution :
n = 1,  = 0.154 nm = 0.154 ï 10ă9 m,
  = 19.3Ĉ, sin  = 0.3305
 n = 2a sin

n 1  0.154  109 m
a 
2 sin  2  0.3305

= 2.33 ï 10ă10 m


= 2.33 A

Example 31

Radii of A+ and that of Xă and Yă have been given as :


A+ 1.00 pm
Xă 1.00 pm
Yă 2.00 pm
Assign structure of AX and AY crystals and also determine volume of the unit cell of AX and
AY crystals.
Solution :

r 1
A+
For AX,   1.0
r 1

r+
Since = 1.0 hence, AX has bcc structure.
r

For bcc structure, edge length a is related to r+ and ră by

a 32 r  A
r
Xă 
2 4
a [1.00  1.00] 
3 3

THE SOLID STATE


58 QUIZRR

3
 4  64
 Volume = a3      12.32 pm3
 3 3 3

r 1
A+
For AY,   0.5
r 2

r
Since, of AY is 0.5 AY has square planar or octahedral structure
r

a
 r
2

a = 2r = 2(r+ + ră) = 6 pm
 Volume = a3 = 216 pm3

Example 32

In a CPS (close packed structure) of mixed oxides, it is found that lattice has O2ă (oxide
ions), and one-half of octahedral voids are occupied by trivalent cations (A3+) and one-
eight of tetrahedral voids are occupied by divalent cations (B2+). Derive formula of the
mixed oxide.
Solution :
Number of octahedral voids per ion in Lattice = 1

Hence, number of trivalent cations A


3+
1  1 1

2 2

number of tetrahedral voids per ion in lattice = 2

Hence, number of divalent cations B2+  2    1 1



8 4

Thus, formula is A1/4B1/4O or A2BO4

Example 33

An element crystallises as face-centred cubic lattice with density as 5.20 g/cm3 and edge
length of the side of unit cell as 300 pm. Calculate mass of the element which contains
3.01 ï 1024 atoms.
Solution :
z = 4 in fcc lattice M = ? d = 5.20 g/cm3

THE SOLID STATE


QUIZRR 59

a = 300 pm = 300 ï 10ă12 m = 3 ï 10ă10 m = 3 ï 10ă8 cm


 a3 = 27 ï 10ă24 cm3
N0 = 6.02 ï 10ă23

d N0 a 3
 M=
z

5.20 g/cm 3  6.02  1023 mol ă1  27  10 24 cm 3



4

= 21.13 g molă1
Thus, 6.02 ï 1023 atoms have = 21.13g

21.13
3.01 ï 1024 atoms have = 23
 3.01  1024 g = 105.65 g
6.02  10

Example 34

The NaCl lattice has the cubic unit cell as shown KBr crystallises
in this lattice.
(a) How many K+ ions and how many Bră ions are in each unit
cell ?
(b) Aassuming the additivity of ionic radii what is a , i.e. edge
Clă
+
 
Na
l en gt h (r  1.33 A, r  1.95 A)?
K+ Br 

r
(c) What minimum value of is need to prevent, anion-anion contact in this structure?
r

Solution :
(a) Four of each :

1 ă 
 4 Br along each of 12 edges + 1 at the corner 
 

1 + 1 + 
 8 K at corners + 2 K at each of six face centres 
 

(b) a  2(r r )  2(1.33  1.95) = 6.56 A


K+ Br ă

THE SOLID STATE


60 QUIZRR
(c) Since side = r+ + ră

and diagonal = 2r  2  r  r 

 2r  2  r  r 

r  r r
2  1
r r

r
  2  1  0.414
r

Example 35

An element crystallises into a structure which may be described by a cubic type of unit cell
having one atom on each corner of the cube and two atoms on one of its diagonals. if the
volume of this unit cell is 24 ï 10ă24 cm3, and density ÂdÊ is 7.2 g cmă3, calculate the number
of atoms present in 200 g of the element
Solution :
This unit cell is a simple cubic unit cell. Thus, unit cell contains one atom of the element. Volume
of the unit cell = a3 = 24 ï 10ă24 cm3.

Mz
d
N0 a3

M1
7.2 
6.02  10 23
 24  1024

this gives M(atomic mass of elements)


= 104.02 g molă1
Number of mol in 200 g of the element

200
  1.9226
104.02

 Number of atoms in 200 g


= 1.9226 ï N0 = 1.9226 ï 6.02 ï 1023
= 11.57 ï 1023 atom

THE SOLID STATE


GENERAL
ORGANIC
CHEMISTRY
2 QUIZRR

C ontents

1. Nomenclature 3

2. Isomerism 33

3. Various Effects 75

4. Reaction Intermediateries 96

5. Acids & Bases 122

GENERAL ORGANIC CHEMISTRY


QUIZRR 3

CL A SSIFICA T I ON OF ORGA N I C COM POU N DS


Depending upon the nature of the carbon skeleton, compounds have been broadly divided into
two categories. These are :
I. Acyclic or Open chain compounds.
II. Cyclic or Closed chain compounds.
I. Open Chain or Acyclic Compounds : Compounds of carbon having open chain of carbon
atoms, branched or unbranched are caled acyclic compounds or aliphatic compounds.
Butane CH3 ă CH2 ă CH2 ă CH3
Isopentane CH3 ă CH ă CH2 ă CH3
|
CH3
Open chain compounds are also known as aliphatic compounds since the earlier compounds of this
class were obtained either from animals or vegetable fats. (Greek, aliphatos  fats).
II. Cyclic or Closed Chain or Ring Compounds : Compounds or carbon having closed chain of
carbon as well as of other atoms are called cyclic compounds. Depending upon the constitution of
the ring, these are further divided into the following categories.
1. Homocyclic or carbocyclic compounds.
2. Heterocylic compounds.
1. Carbocyclic or Homocyclic Compounds : Compounds of carbon having closed chain entirely
made up of carbon atoms are called carbocyclic or homocyclic compounds. These are further
divided into two groups.
(i) Alicyclic Compounds : Closed carbon chains except characteristic benzene ring, resembling
in properties with acyclic compounds.
For example : cycloalkanes, cycloalkenes etc.

Cyclopropane Cyclopentane Cylopenetene Cyclohexene

(ii) Aromatic Compounds : Closed chain of only carbon atoms with alternate single and
double bonds.
For example : Benzene and its derivaties.
Some important examples of monocyclic, bicyclic and triclic compounds are

GENERAL ORGANIC CHEMISTRY


4 QUIZRR
(a) Monocyclic aromatic compounds :
CH3

Benzene Toluene

(b) Bicyclic and Tricyclic aromatic compounds :

Naphthalene Anthracene
(Bicyclic) (Tricyclic)

2. Heterocyclic compounds : Compounds of carbon having closed chain made up of carbon and
other atoms. The hetero atoms commonly found in these compounds are oxygen, nitrogen and
sulphur but occasionally phosphorus, boron, silicon and some metal atoms like tin, selenium etc.
may also be present, Depending upon the chemical behaviour, these are further classified into the
following two categories.
(i) Alicyclic heterocylic compounds : Aliphatic cyclic compounds containing one or more
heteroatoms in their rings are called alicyclic heterocyclic compounds. For example.

Tetrahydro Furan Pyrrolidine


(THF)

O N
H
(ii) Aromatic heterocyclic compounds : Aromatic cyclic compounds containing one or more
heteroatoms in their molecules are called aromatic heterocyclic compounds. For example :
(ii

Pyridine Furan Pyrrole

N
O N
H

GENERAL ORGANIC CHEMISTRY


QUIZRR 5

H o m o l o g o u s Se r i e s
Organic compounds can be divided into different groups on the basis of similarity in their structure
and properties. The property by which a number of organic compounds form a homologous series is
termed as homology.
The general characteristics of a homologous series are
Ć All compounds in the series have the same functional group.
Ć All compounds in the series can be represented by the same general formula and can be
prepared by the general methods of preparation.
Ć All the homologues show a gradual gradation in their physical and chemical properties.
Ć Successive members of a homologous series differ by CH2 group and by a mass of 14 units.

Table 1.a : Homologous series of aliphatic organic compounds

Name of the homologous General formula Functional group IUPAC name


Alkanes CnH 2n+2 ă (single bond) Alkanes
Alkenes CnH 2n = (double bond) Alkenes
Alkynes CnH 2nă2  (triple bond) Alkynes
Monohydric alcohols CnH 2n+1 OH ă OH Alkanols
Aldehydes CnH 2nO ă CHO Alkanals
Ketones CnH 2nO C = 0 Alkanones
Monocarboxylic acid CnH 2nO2 ă COOH Alkanoic acids
Ethers CnH2n+2 O CăO-C Alkoxy alkanes
Primary amines CnH2n+1 NH2 ăNH2 Alkanamines
Amides CnH2n+1 CONH2 ă CONH2 Alkanamides
Esters CnH 2nO2 RăCOO ăR Alkyl alkanoate
Cyanides CnH2n+1 CN ăCN Alkane nitriles
O

Nitro compounds CnH2n+1 NO2 N Nitroalkanes


O
Acid chlorides CnH2n+1 COCl ă COCl Alkanoyl chlorides

N o m e n c l a t u r e o f Org a n i c Co m p o u n d s
Nomenclature implies assigning proper name to a particular organic compound on the basis of
certain standard rules so that the study of these compounds may become systematic. In the IUPAC
system, the name of an organic compound consists of three parts.
(i) Word root (ii) Suffix (iii) Prefix

GENERAL ORGANIC CHEMISTRY


6 QUIZRR
Wo r d Ro o t
The word root denotes the number of carbon atoms present in the chain. For example,

Chain length Word root Chain length Word root


C1 Meth C6 Hex
C2 Eth C7 Hept
C3 Prop C8 Oct
C4 But C9 Non
C5 Pent C10 Dec

Su f f i x
The word root is linked to the suffix, which may be primary, secondary or both.

Pr i m a r y s u f f i x
It indicates the nature of linkages between the carbon atoms. For example,
ane ă for single bonded compounds,
ene ă for one C = C bond
a diene ă for two C = C bonds
a triene ă for three C = C bonds
yne ă for one C  C bond
a diyne ă for two C  C bonds

Se c o n d a r y s u f f i x
It indicates the presence of functional group in the organic compound. For example,

Class of organic compound Functional group Secondary suffix


Alcohols ă OH ăol
Aldehydes ă CHO ă al
Ketones C= O ă one
Carboxylic acids ă COOH ăoic acid
Esters ă COOR alkyl..... oate
Acid amides ă CONH2 ăamide
Nitriles ă C N ănitrile
Amindes ăNH2 ăamine
Thioalcohols R ă SH ăthiol
Amides R ă CONH2 ăamide

GENERAL ORGANIC CHEMISTRY


QUIZRR 7

Pr e f i x e s
There are many groups, which are not regarded as functional groups in the IUPAC naming of the
compounds. These are regarded as substituents or side chains. These are represented as prefixes and
are placed before the word root while naming a particular compound. For example, if a compound
contains more than one functional group, then one of the functional group is regarded as principal
group and is treated as secondary suffix. The other functional groups are regarded as substituents and
are indicated by prefixes.
Substituent Prefix Substituent Prefix
ăCnH 2n+1 Alkyl ăNH2 Amino
ăF Fluoro ă NO Nitroso
ăCl Chloro ăN=Nă Diazo
ăBr Bromo ăOCH3 Methoxy
ăI ăIodo ăOC2H 5 Ethoxy
ăNO2 Nitro ăOH Hydroxy
ăCN Cyano ăCOOH Carboxy
ăNC Isocyano ăCOOR Carbalkoxy
ăCHO Formyl ăCONH2 Carbamyl
ăSH Mercapto ăCO Cl Chloroformyl
Thus, a complete IUPAC name of an organic compound may be represented as
Prefix + Word root + Primary suffix + Secondary suffix

H o w t o N a m e Org a n i c Co m p o u n d s U s in g t h e I U PA C Ru l e s
In order to name organic compounds you must first memorize a few basic names. These names are
listed within the discussion of naming alkanes. In general, the base part of the name reflects the
number of carbons in what you have assigned to be the parent chain. The suffix of the name
reflects the type(s) of functional group(s) present on (or within) the parent chain. Other groups which
are attached to the parent chain are called substituents.

Sa t u r a t e d H y d r o c a r b o n s

Number of Carbons Name


1 methane
2 ethane
3 propane
4 butane
5 pentane
6 hexane
7 heptane
8 octane
9 nonane
10 decane
11 undecane
12 dodecane

GENERAL ORGANIC CHEMISTRY


8 QUIZRR
There are a few common branched substituents which you should memorize. These are shown
below.

CH 3 CH 3 CH 3 CH 3
CH 3CH CH 3CH2CH CH 3 C CH 3 CHCH2
CH 3
isopropyl sec-butyl tert-butyl isobutyl

Ru l e s f o r Wr it in g I u p a c N a m e s
1. Identify the longest carbon chain. This chain is called parent chain.

6 4 2 7 5

3 4
1
7 5 8 6

1
3

Chain of seven carbons Chain of eight carbons 2


(wrong) (right)

Note : If more than one sets of longest possible chains are there, the selected longest chain should
have :

(1) Maximum number of side chains or

1 3 1 3 5

2 4 2 4 6

6 atoms chain with two side 6 atoms chain with one side
chains or two unbranched chain or one branched
side chains is selected side chain is rejected

GENERAL ORGANIC CHEMISTRY


QUIZRR 9

2. If more than one side chain is present, sum of the positions should be minimum while numbering
the carbon chain.

2 4 6 8 8 6 4 2

1 3 5 7 9 9 7 5 3 1

sum of positions : sum of positions :


5 + 5 + 7 = 17 3 + 5 + 5 =13
(wrong) (minimum) (right)

3. If two different substituents are at same position from ends, lowest number is assigned in order
of their alphabets.
Cl Cl

4 1
2 3
3 2
1 4

I I
(Right) (Wrong)

4. If a substituent (such as halogen, or nitro group) and a side chain are at same position from
opposite ends, lowest number is assigned to substituents

CH3 CH3

4 1
2 3
3 2
1 4

Cl Cl
(Wrong) (Right)

5. If more than two substituents and side chains are present, the sum of their number should be
lowest at the first preference irrespective of the nature of Substituents or side chains. This is
referred to as Lowest Sum Rule.

GENERAL ORGANIC CHEMISTRY


10 QUIZRR
I I

4 2
2 4
3 3
1 5 5 1

Cl CH3 Br Cl CH3 Br

1 + 2 + 3 + 5 = 11 1 + 3 + 4 + 5 = 13
(Right) (Wrong)

Note : In case when starting locant is lower from one side, then lowest sum rule is not obeyed.
CH3 CH3 CH3 CH3

4 6 10 7 5 3 1
2 8 9
7 8
1 3 5 9 6 4
10 2

CH3 CH3
(2 + 7 + 8 = 17 is accepted) (3 + 4 + 9 = 16 is rejected)
(methyl gets lower locant i.e. 2) (methyl gets higher locant i.e. 3)

6. If the same substituent occurs more then once, the location of each point on which the substituent
occurs in given. In addition, the number of times the substituent group occurs is indicated by a
prefix (di, tri, tetra, etc.)
E.g. CH3
CH3 CH CH CH3
H3C C CH3

CH3 CH3
CH3
2-3 Dimethyl butane 2-2 Dimethyl propane
In summary, the name of the compound is written out with the substituents in alphabetical order
followed by the base name (derived from the number of carbons in the parent chain). Commas are
used between numbers and dashes are used between letters and numbers. There are no space in
the name.
Here are some examples :

CH3 CH2 CH3


CH3 CH2 CH3 | |
| | CH3  CH2  CH2  CH  C  CH2  CH3
CH3  CH CH 2  CH  CH2  CH3 |
5 Ethyl  2 methylhexane CH3
4  Ethyl 3,3 dimethylheptane

GENERAL ORGANIC CHEMISTRY


QUIZRR 11

7 6 5 4 3 2 1
CH3  CH2  CH  CH  CH  CH  CH3
| | | | CH3
CH3 5' CH2 CH3 CH3
|
6'CH2 CH
|
7' CH2 CH 2 CH 2
2,3,5, Trimethyl 4 propylheptane 1-methylcyclopropane
 NOT : 2,3 dimethyl  4 sec  butylheptane 

CH3 CH3
| | 6 5 4 3 2 1
CH3  CH  CH2  CH2  CH  CH2  CH CH2  CH3 CH3  CH2  CH  CH  CH2  CH3
1 2 3 4 5| 6 7 8 9 | |
C H C H 3 CH3 CH2
| |
H3C ă CH2 CH3
2 Ethyl 4  methylhexane
5Sec  butyl 2, 7dimethylnonane

Ru l e s f o r N a m i n g A l k e n e s a n d A l k y n e s – U n s a t u r a t e d H y d ro c a r b o n s
Double bonds in hydrocarbons are indicated by replacing the suffix ăane with ăene. If there is
more than one double bond, the suffix is expanded to include a prefix that indicates the number of
double bonds present (ăadiene, ăatriene, etc.). Triple bonds are named in a similar way using the suffix
ăyne. The position of the multiple bond(s) within ethe parent chain is(are) indicated by placing the
number(s) of the first carbon of the multiple bond(s) directly in front of the base name.
Here is an important list of rules to follow :
1. The parent chain is numbered so that the multiple bonds have the lowest numbers (double
and triple bonds have priority over alkyl and halo substituents)
3 2 4
5
2 3
1
4 5
1
5 carbon chain with (Wrong)
2 unsaturated carbons or
one double bond is selected

2. When both double and triple bonds are present, numbers as low as possible are given to
double and triple bonds even though this may at times give „ăyne‰ a lower number than
„ăene‰. When there is a choice in numbering, the double bonds are given the lowest number.
2 4 6 5 3
1

1 6
3 5 4 2

(Right) (Wrong)

GENERAL ORGANIC CHEMISTRY


12 QUIZRR
3. When both double and triple bonds are present, the ăen suffix follows the parent chain directly
and the ăyne suffix follows the ăen suffix (notice that the ÂeÊ is left off, ăen instead of ă
ene). The location of the double bond(s) is(are) indicated before the parent name as before, and
the location of the triple bond(s) is(are) indicated between the ăen and ăyne suffixes.

5 4 3 2 1
H C  C CH 2  CHă CH3
2  Penten  4  yne
7 6 5 4 3 2 1
H C  C C CH2  CH2  CH=CH2
1  Hexen  5  yne

4. For a branched unsaturated acyclic hydrocarbon, the parent chain is the longest carbon chain
that contains the maximum number of double and triple bonds. If there are two or more
chains competing for selection as the parent chain (chain with the most multiple bonds), the
choice goes to :
(1) the chain with the greatest number of carbon atoms,
(2) the number of carbon atoms being equal, the chain containing the maximum number of
double bonds.
5. If there is a choice in numbering not previously covered, the parent chain is numbered to give
the substituents the lowest number at the first point of difference.
Here are some examples :

6 5 4 3 2 1 6 5 4 3 2 1 5 4 3 2 1
CH3 CH CH  CH2  CH CH2 CH C  CH CHCH CH2 CH3  CH CH  C  CH
1, 4 Hexadiene 1,3Hexadien 5 yne 3Penten 1 yne

CH2  CH3
CH3 |
6 |5 4 3 2 1 CH2 4
3
CH3  C  CH2  CH2  CH CH2 6 5 4 |3 2 1
| CH C  C  C  CH2 CH2 2
CH3 | 1
5,5Dimethyl 1 hexene CH2 CH2 CH3
3,3  Dipropyl  1,3 hexadien 5 yne 1,4,4-Trimethylcyclobutene

N a m i n g A l i c y c l ic Co m p o u n d s
Aliphatic cyclic compounds are called alicyclic compounds. These compounds are named as follows :
Ć The number of carbon atoms in the ring determines the word root.
Ć The primary suffix cyclo-is added before the word root.
Ć The primary suffix is determined by the nature of the ring (saturated or unsaturated).
For cyclic compounds in which ring contains one double bond
The suffix -ene is added to the word root.
For cyclic compounds in which ring contains a triple bond
The suffix-yne is added to the word root.

GENERAL ORGANIC CHEMISTRY


QUIZRR 13

When the ring contains two or three double or triple bonds


Suffixes like diene, triene and diyne, triyne are added to the word root.
An appropriate suffix or prefix indicates the relevant functional group or any substituent. The
position of the functional group or substituent is indicated by the serial number of the carbon atom in
the ring to which it is attached.
The lowest number to the functional group/substituent is given while numbering of the carbon
atoms in the ring.

CH2 CH2
H2C CH-CH3
H2C  CH  CH3 H2C CH
| | CH2
H2C  CH2 H2C
methylcyclobutane CH CH2 CH
CH3 cyclopentene
1,3- dimethylcyclohexane

CH 2
H2C C
CH · COOH
H2C C H2 C
cyclobutyne cyclopropanic acid

H a l o a k a n e s (a l k y l h a l id e s )
General formula : CnH2n+1 X or RX
Prefix : Halo
Haloalkanes (or alkyl halides) are monohalogen derivatives of alkanes. They are obtained by
replacing one hydrogen atom of alkane molecule by one halogen (X) atom.
RH ă H + X (= F, Cl, Br I) RX
alkane Haloalkane

In the IUPAC system of nomenclature, monohalogen derivative of alkane is named by adding the
prefix halo (fluoro, chloro or iodo) to the name of the parent alkane.

Halo + Name of parent alkane = Haloalkane

The carbon atoms are numbered, if necessary, to show the position of the halogen atom as follows :

CH3l C2H5Br CH3  CH CH3 CH3ăCH2ăCH2ăBr


|
Br

Common name : methyl iodide ethyl bromide Isopropyl bromide propyl bromide
IUPAC name : iodomethane bromomethane 2-bromopropane 1-bromopropane

GENERAL ORGANIC CHEMISTRY


14 QUIZRR
Po l y h a l o a l k a n e s
Polyhaloalkanes are those halogen derivatives of alkanes, which contain more than one halogen
atom in their molecule. The compounds containing two halogen atoms are termed dihalo and those
containing three halogen atoms as trihalo, and so on.
The general formula of dihalogen derivatives is CnH2nX2. In the IUPAC system, a dihalo derivative
is named as a dihalogen derivative of the parent alkane, and the positions of halogens atoms are
indicated as usual.
Polyhalo Parent alkane Name
compound Common IUPAC
ClH2CăCH2Cl Ethane (C2H6) Ethylene chloride 1,2-dichloroethane
CH3 ă CHBr2 Ethane (C2H6) Ethylidine bromide 1, 1 dibromoethane
CHl3 Methane (CH4) Iodoform triiodomethane

A l k a n o l s (Sa t u r a t e d M o n o h y d r i c A l c o h o l s )
General Formula : CnH2n+1 OH
Functional Group : OH
Suffix : ol
Alkanols or simply alcohols, are monohydroxy derivatives of alkanes, and are represented by the
general CnH2n+1 OH or simply as R-OH.
Alkanols or monohydric alcohols are further classified as primary, secondary and tertiary alcohols.
The classification is according to the OH group that is attached to a carbon atom, which in turn is
attached to one (or more), two or three carbon atoms respectively. For example,

H CH3
| |
H3C  C  O H H3C  C  OH
CH3CH2OH | |
CH3 CH3
primary alcohol secondary alcohol tertiary alcohol

The IUPAC name of a monohydric alcohol is obtained by replacing the terminal -e from the name
of alkane with -ol.

No. of Molecular Parent Common IUPAC


C- formula alkane name name
Atoms
1 CH3OH CH4 (methane) Methyl alcohol Methanol
2 CH3CH 2OH C2H6 (ethane) Ethyl alcohol Ethanol
3 C3H7OH C3H8 (propane) Propyl alcohol Propanol
4 C4H9OH C4H10 Butyl alcohol Butanol

GENERAL ORGANIC CHEMISTRY


QUIZRR 15

Name of alkane ă e + ol  Name of the alkanol


For example :
The -OH group can have different positions in alcohols with three or more carbon atoms, leading
to the possibility of more than one kind of alcohol. For example in the case of propyl alcohol, following
two structures are possible.

CH3  C H  CH3
CH3ăCH2ăCH2ăOH |
OH
IUPAC name : 1-propanol 2-propanol
Common : n-propyl alcohol isopropyl alcohol
(a primary alcohol) (a secondary alcohol)

The RO group obtained by removing H atom from the OH group of an alkanol is termed alkoxy
group. For example,
CH3O C2 H 5 O C3 H 7 O C4 H 9 O
Methoxy ethoxy propoxy butoxy

A l k a n e d i o l s (Di h y d r ic A l c o h o l s )
General formula : CnH2 (OH)2
Functional group : OH
Suffix : diol
In the IUPAC system, the name of dihydric alcohol is obtained by adding the suffix diol to the
name of the parent alkane and indicating the positions of the -OH groups.

Polyhalo Parent alkane Name


compound Common IUPAC
2
CH 2OHă1CH 2OH Ethane (C2H6) Ethylene glycol Ethane 1, 2-diol
3 2 1
CH3 ă CHOH CH2OH Propane (C3H8) Propylene glycol Propane 1, 2-diol

A l k o x y a l k a n e s (Et h e r s )
General formula : CnH2n+1-O- CmH2m+1
or R-O-R´
Functional group : O
Prefix : alkoxy
Ethers are alkoxy derivatives of alkanes. These are represented by the general formula R-O-R´,
where R and R´ are alkyl groups. Ether is termed as simple ether when R and R´ are the same. If the
two alkyl groups (R and R´) are different, the ether is termed as a mixed ether.

GENERAL ORGANIC CHEMISTRY


16 QUIZRR
When one hydrogen of the alkane is replaced by an alkoxy group, alkoxy-alkanes are obtained.
RHăH + OR´ RăOăR´
alkane alkoxy alkoxyalkane
In the IUPAC system of nomenclature, the large alkyl group is considered to be the alkane residue,
while the smaller alkyl group with oxygen atom is considered to be the alkoxy group. Some typical
ethers are named below.
Molecular Common Alkane Alkoxy IUPAC
formula name residue group name
Atoms
CH3OCH3 Dimethyl ether Methane Methoxy Methoxymethane
CH2OC2H 5 Ethyl methyl Ethane Methoxy* Methoxyethane
ether
C2H 5OC2H 5 Diethyl ether Ethane Ethoxy Ethoxyethne
* Here the smaller alkyl groups forms the alkoxy group

A L K A N A L S (A L DEH Y DES)
General formula : CnH2n + 1CHO or

O
R C
H

In IUPAC system of nomenclature, the name of an aldehyde is obtained by replacing the terminal
ăe of the parent alkane by the suffix-al.
IUPAC name of the aldehyde = Name of the parent alkane ă e + al = Alkanal
Names of some individual members
Molecular Acid obtained Parent Name
formula on oxidation alkane Common IUPAC
HCHO HCOOH Methane Formaldehyde Methanal
(formic acid)
CH3CHO CH3CHOOH Ethane Acetaldehyde Ethanal
(acetic acid)
C2H5CHO C2H5COOH Propane Propionaldehyde Propanal
(Propionic
(acid)

Since the ăCHO group is always present at the end of the chain in aldehydes there is no need to
designate its position 1.

GENERAL ORGANIC CHEMISTRY


QUIZRR 17

While counting the carbon atoms in the parent chain, the carbon of the ăCHO group is also
counted.
For example, the parent chain in CH3CH2CH2CHO consists of four C atoms and not three. Hence,
CH3CH2CH2CHO should be named as butanal and not 1-butanal as -CHO is always at the end of the
chain (i.e., -CHO carbon is always at position 1).

A L K A N ON ES ( K ET ON ES)
General formula : (CnH2n+1 . CO . Cm H2m+1)

Functional group : CO


or R-CO-R´
Suffix : one
The following general formula describes Ketones (or alkanones)
R
C = O where, R and R´ are alkyl groups.
R
The two alkyl groups (R and R´) may be same or different. When the two alkyl groups (R and R´)
are same, the ketone is a simple ketone. If the two are different the ketone is called a mixed ketone.
The IUPAC name of an alkanone is obtained by replacing the last -e from the name of the parent
hydrocarbon with the suffix - one. The position of the carbonyl group is indicated by a numeral
referring to the serial number of the carbon atom bearing the carbonyl group in the chain. The
numbering is done so as to give the carbonyl group the lowest number.
IUPAC name of ketone = Name of parent alkane ăe + one = Alknone

No. of Molecular Alkyl groups Parent Common IUPAC


C- formula attached to name name
Atoms the > CO group
3 CH3ă CO ă CH3 Methyl, Methyl Propane Dimethly ketone Propanone
4 CH 3ă CO ă CH 2CH 3 Methyl, Ethyl Butane Ethyl methyl Butanone-2
ketone
5 CH3CH 2ăCOăCH2CH 3 Ethyl, Ethyl Pentane Diethyl ketone Pentanone-3

Alk onic Ac ids or sat urat ed m onoc arbox ylic ac id


General formula : CnH2n COOH
+ 1

O
Functional group : R  C
OH

Suffix : oic acid

GENERAL ORGANIC CHEMISTRY


18 QUIZRR
On replacing one H atom of an alkane by one carboxylic (- COOH) group a saturated monocarboxylic
acid or alkanoic acid is obtained. Higher members of this family are generally, known as fatty acids
(obtained by the hydrolysis of fats).
In IUPAC system, the name of an alkanoic acid is obtained by replacing the last ÂeÊ from the name
of the parent alkane with -oic acid.
Name of monocarboxylic acid = Name of the parent alkane -e + oic acid
= Alkanoic acid
Typical members are named below :

No. of Molecular Parent Common IUPAC


C- formula alkane name name
Atoms
1 HCOOH Methane Formic acid Methanoic acid
2 CH3COOH Ethane Acetic acid Ethanoic acid
3 CH3CH 2COOH Propane Propionic acid Propanoic acid
4 CH3CH2CH 2COOH Butane Butyric acid Butanoic acid
5 CH3(CH 2)3COOH Pentane Valeric acid Pentanoic acid

A l k a n e d i o i c a c i d s (Sa t u r a t e d d i c a r b o x y l i c a c i d s )
General formula : Cn H2n+2 (COOH)
O
Functional grooup : C
OH
Suffix : dioic acid
A dicarboxylic acid contains two carboxylic groups linked to the same or different carbon atoms.
In the IUPAC system, the name of alkanedioic acid is obtained by adding the suffix, -dioic acid to the
name of the parent alkane.
Name of the dicarboxylic acid = Name of the parent alkane + dioic acid = Alkanedioic acid
For example : Ethanedioic acid (oxalic acid, parent alkane is ethane)

COOH
|
COOH

Propanedioic acid (Malonic acid, parent alkane is propane)

COOH

HC
2

COOH

GENERAL ORGANIC CHEMISTRY


QUIZRR 19

Butanedioic acid (Succinic acid, parent alkane is butane)

H2 C COOH

H2 C COOH

A l k a n a m i d e s (A c i d A m i d e s )
General formula : CnH2n CONH2
+ 1
where n = 0, 1, 2 ...
Functional group

O O
C R C
NH2 or NH2

Suffix : amide
In IUPAC system, the amides are named by replacing the letter e of the parent alkane by the word
amide.
IUPAC name of acid amide = Name of the parent alkane - e + amide = Alkanamide
Typical amides are named below :
Molecular Parent acid Parent alkane Common name IUPAC name
HCONH2 HCOOH (formic acid)* Methane Formamide Methanamide
CH3CONH2 CH3COOH (acetic acid)* Ethane Acetamide Ethanamide
CH3CH2CONH2 C2H5COOH Propane Propanamide Propanamide
(propionic acid)*
* Common names of the acid.

A l k a n e n i t r il e s (A l k y l Cy a n i d e s )
General formula : R-C  N
Functional group : C  N
Suffix : nitrile
The IUPAC name of an alkyl cyanide is obtained by adding the suffix - nitrile to the name of the
parent alkane. Thus, an alkyl cyanide is alkanenitrile in the IUPAC system.
Some common alkane nitriles are :

Molecular Acid formed Parent Common IUPAC


formula on hydrolysis alkane name name
HăC  N HCOOH (formic Methane Hydrogen Methanenitrile
acid) cyanide
H3ă C  N CH3COOH Ethane Methyl Ethanenitrile
(acetic acid) cyanide

GENERAL ORGANIC CHEMISTRY


20 QUIZRR
A l k a n o y l c h l o r id e s (A c i d Ch l o r id e s )
General formula : CnH2n+1COCl
where n = 0, 1, 2 ...
Functional group :

O O
C R C
Cl or Cl
Suffix : oyl chloride
Acid chlorides contain the functional group ăCOCl. This functional group is obtained by replacing
the ăOH group of a carboxyl group with a chloride atom.
The IUPAC name of an acid chloride is obtained from the IUPAC name of the parent acid by
replacing the terminal e of the parent alkane by oyl chloride.
IUPAC name of the acid chloride = Name of the parent alkane - e + oyl chloride
= Alkanoyl chloride
Some acid chlorides are named below,
Methanoyl chloride : (formyl chloride, parent acid is Methanoic acid) HCOCI
Ethanoyl chloride : (acetyl chloride, parent acid is Ethanoic acid) CH3COCI

A l k a n o i c a n h y d r id e s (a c i d a n h y d r id e s )
O O
|| ||
General formula : R  C O  C R

O O
|| ||
Functional group :  C O  C 

Suffix : oic anhydride


An acid anhydride may be considered as the condensation product of two molecule of a
monocarboxylic acid obtained by losing one molecule of water.

RCOOH RCO

- H2 O O

RCOOH RCO

IUPAC name of an acid anhydride = IUPAC name of acid parent acid-acid + anhydride = Alkanoic
anhydride
H3 C CO
O
H3 C CO
For example, Ethanoic anhydride (acetic anhydride, parent acid is ethanoic acid)

GENERAL ORGANIC CHEMISTRY


QUIZRR 21

A l k y l a k a n o a t e s (e s t e r s )
O
||
General formula : R  C OR '

O
||
Functional group :  C  OR'
Suffix : oate
An ester contains the group COOR where R is an alkyl group. The hydrolysis of the ester gives
the parent acid and so its name is based on the name of the acid.
Name of an ester = Alkyl group from the alcohol + Name of the parent acid - oic cid + oate
The names of some typical esters are

Molecular Common system IUPAC system


formula Parent acid Name of ester Parent acid Name of ester
HCOOCH3 Formic acid Methyl formate Methanoic acid Methyl methanoate
CH3COOCH3 Acetic acid Methyl acetate Ethanoic acid Methyl Ethanoate

N i t r o a l k a n e s a n d A l k y l N i t r it e s

O
General formula : R · N
O

O
Functional group : · N
O
Prefix : nitro

No. of Molecular Parent Common IUPAC


C- Atoms formula alkane name name
1 CH3NO2 Methane Nitromethane Nitromethane
2 CH3CH2NO2 Ethane Nitroethane Nitroethane
3 CH3CH2CH2NO2 Propane n-Nitropropane 1-Nitropropane

CH3 CHCH3
4 | Propane iso-Nitropropane 2-Nitropropane
NO2

Nitro compounds are named as the nitro derivatives of the parent alkane in both the systems of
nomenclature. This is done by adding the prefix nitro to the word root of the parent alkane.

GENERAL ORGANIC CHEMISTRY


22 QUIZRR
Alkyl nitrites
General formul : R-O-N=O
Functional group : ăO-N=O
Alkyl nitrites are named by adding the suffix nitrite to the alkyl group. For example
CH3-O-N=O is Methyl nitrite.
C2H5-O-N=O is Ethyl nitrite.

A l k a n a m i n e s (A m i n e s )
Amines are formed by replacing one or more hydrogen atoms of ammonia (NH3) with alkyl groups.
These are further classified as primary, secondary and teritary amines according to one, two or three
hydrogen atoms of ammonia being replaced by alkyl groups. For example,
Primary (amino, NH2) RăNH2
Secondary (imino, NH)
R
NH
R
Tertiary (nitrile,N)

R
R N
R
In IUPAC system, primary aliphatic amines are named as alkanamine. This is done by replacing
the ÂeÊ of the parent alkane by the suffix amine.
The secondary and tertiary amines are named as alkyl derivatives of the primary amine.
The higher alkyl group is considered as the residue of the parent alkane.
Examples of the amines :
No. of Molecular Parent Common name IUPAC name
C- Atoms formula alkane
H3C
NH
1* Methane Dimethylamine N-methylmethanamine
H3C

H5C2
NH
2* Ethane Ethymethylamine N-methylethanamine
H3C

H3C
H3C N
Methane Trimethylamine N,N-dimethylethamine
C2H5

* This indicates the number of C-atoms in the chain : the other groups are considered as the
substituent.

GENERAL ORGANIC CHEMISTRY


QUIZRR 23

Primary amines
No. of Molecular Parent Common IUPAC
C- Atoms formula alkane name name

1 CH3NH2 Methane Methylamine Methanamine


2 C2H5NH 2 Ethane Ethylamine Ethanamine
3 C3H7NH 2 Propane Propylamine Propanamine
4 C4H9NH 2 Butane Butylamine Butanamine

For primary amines containing three or more carbon atoms, there can be more than one isomers
of the concerned amine. For example, there are two propylamines.

H3C  CH  CH3
H3C  CH2  CH2  NH2 |
1propanamine
NH2
2 propanamine

A l k a n e n i t r il e s (A l k y l Cy a n i d e s )
Geneal formula : R-N-C  N
Functional group : C  N
Suffix : nitrile
The IUPAC name of an alkyl cyanide is obtained by adding the suffix - nitrile to the name of the
parent alkane. Thus, an alkyl cyanide is alkanentrile in the IUPAC system.
Some common alkane nitriles are :
Acid formed Parent Common IUPAC name
Molecular on alkane name
formula hydrolysis

HăC  N HCOOH (formic Methane Hydrogen Methanenitrile


acid) cyanide

H3CăC  N CH3COOH Ethane Methyl Ethanenitrile


(acetic acid) cyanide

A l k y l c a r b y l a m i n e s (A l k y l I s o c y a n i d e s )
General formula : R · N C

Functional group : · N C
Suffix : Carbylamine
In the IUPAC system, these compounds are named as carbylamines of alkyl groups attached to
the ăNC group.

GENERAL ORGANIC CHEMISTRY


24 QUIZRR

Molecular Acid formed on Common IUPAC name


formula hydrolysis name

CH3ăNC CH3COOH Methyl Methyl


(acetic acid) isocyanide carbylamine
C2H5ăNC C2H5COOH Ethyl Ethyl
(propionic acid) isocyanide carbylamine

Ru l e s f o r N a m i n g Org a n i c Co m p o u n d s Co n t a i n i n g On e o r M o re Fu n c t io n a l Gro u p s
The rules for naming an organic compound containing functional groups are exactly same as
discussed already for compounds containing double and triple bonds. In this case, the preference of
lowest number is given to carbon atom bearing the functional group. The rules are summarized
below :
(i) Select the longest continuous chain containing the carbon atom having functional group(s).
(ii) The numbering of atoms in the parent chain is done in such a way that carbon atom bearing
the functional group gets the lowest number.
(iii) If two or more same functional groups are present, these are indicated by using di, tri, tetra
as prefixed to the name of the functional group.
(iv) If the organic compound contains a functional group, multiple bonds, side chain or substituents,
the following order of preference must be followed,
Functional group > Double bond > Triple bond > Side chain.

(v) When an organic compound contains two or more functional groups, one group is regarded as
the principal functional group and the other group is treated as the secondary functional
group, which may be treated as substituent(s). The following order of preference is used for
selecting the principal functional group,

Carboxylic acids > sulphonic acids > acid anhydrides > esters > acid chlorides > amides >
nitriles > aldehydes > ketones > alcohols > amines > imines > ethers > alkenes > alkynes.

Different classes of functional groups including multiple bonded compounds and the suffix or prefix
required to name these compounds are given in the preferential decreasing order in the following table.

Class of compounds Functional group or Suffix Prefix


substituent

1. Carboxylic acids C= O Carboxylic acid/oic acid Carboxy


HO

O
||
2. Sulphonic acids  S  OH Sulphonic acid sulfo
||
O

GENERAL ORGANIC CHEMISTRY


QUIZRR 25

O O
|| ||
3. Acid anhydrides  C O  C oic anhydride ·

4. Esters C =O (R) ... carboxylate alkoxy carbonyl


RO

/(R) ... oate

5. Acid halides C= O Carbonyl halide/oyl halo carbonyl


X

halide

6. Amides C= O Carboxamide/amide carbamoyl


NH2

7. Cyanides ă CN Carbonitrile/nitrile cyano

8. Aldehydes C= O Carbaldehyde/al formyl/oxo


H

9. Ketones C= O one keto/oxo

10. Alcohols ăOH ol hydroxy

11. Amines ăNH2 amine amino

12. Imines = NH imine imino

13. Ethers ă C ă O ă C ă ă alkoxy

14. Alkenes = (double bond) ene ·

15. Alkynes  (triple bond) yne ·

For those functionalities, which have two prefixes and/or suffixes, the first one is used when carbon
atom of the functional group is not a part of the longest continuous chain and the second one is used,
when carbon atom is counted in the longest chain.

GENERAL ORGANIC CHEMISTRY


26 QUIZRR

Some Examples
(1) (HOCH2CH2O)2CHăCO2H Bis(2ăhydroxyethoxy) ethnaoic acid
C2H5
1
2
3
(2) 4 1ăEthylă4ămethylcyclohexane
CH3

F F

CH3CHăCH CH2CH3

(3) CH3(CH2)4CH2CHCH2CH(CH2)3CH3 7 ă (1, 2 ă Difluorobutyl) ă 5 ă ethyldecane

O
||
(4) CH3 CH2 C H2  C  CH2Cl 1ăchloropentană2ăone
5 4 3 2 1

2 1
(5) CH3 CH2  O  CH2 CH 2Cl 1ăChloroă2ăethoxyethane

(6) OHCCH2 CH2 CH  CH2 CHO 3ă(Formylmethyl)hexanediă1, 6ăal


|
CH2CHO

4 3 2 1
(7) OHCCH2CH2 CH  CH2 CHO Butaneă1,2,4ătricarbaldehyde
|
CHO

(8) CH3 CH2 CH  OCH2CH3 1, 1ăDiethoxypropane


|
OCH2CH3

(9) HO2CCH2 CH2 CH C H2 CH2 CO2 H Pentaneă1,3,5ătricarboxylic acid


|
CO2H

(10) NCCH2CH2CH2 CH CH2 CH2 CN Hexaneă1,3,6ătricarbonitrile


|
CN
(11) CH3CSăOăCOCH2CH 3 Propionic thioacetic anhydride

O
||
(12) CH3  C CH2 CH2CH2CO2 H 5ăOxohexanoic acid
6 5 4 3 2 1

O O
|| ||
(13) CH3CH2  C  CH2  C  CH3 Hexaneă2,4ădione
6 5 4 3 2 1
(14) CH2=CHCH2CH(OH)CH3 Pentă4ăenă2ăol
GENERAL ORGANIC CHEMISTRY
QUIZRR 27

N a m i n g A l i c y c l ic & A r o m a t i c Co m o p o u n d
Ć If alcicyclic group is attached to the long chain, the chain is numbered from the side
of the alcicyclic group.
7 5 3 1

8
6 4 2

Ć Cycloalknes with only one ring are named by attaching prefix cyclo-to the names of
the alkanes possessing the same number of carbon atoms.

cylopropane cyclopentane

Ć If only one substitutent is present, it is not necessary to designate its position.


OH Cl

cyclohexanol chlorocyclohexane
isopropylcyclohexane
Ć When two substituents are present, we number the ring beginning with the substituent
first in the alphabet. When three or more substituents are present, we begin with the
substituent that leads to the lowest set of locants.
CH3 CH3
CH2 CH3
1
2

3
4
CH2 CH3
Cl
1-ethyl-3-methylcyclohexane 1-ethy 1-3 methylcyclohexane
not 1-ethy l-5 cyclohexane not 1-ethy l-5 cyclohexane

GENERAL ORGANIC CHEMISTRY


28 QUIZRR
Ć When a single ring system is attached to a single chain with a greater number of
carbon atoms, or when more than one ring system is attached to a single chain, then
they are named as cyclocalkanes.

1-cyclobutylpentane 1,3 dicyclohexylpropane

Ć Cycloalkanes consisting of two rings only and having two or atoms in common are
named by taking the prefix bicyclo followed by the name of the alkane. In between
bicyclo and alkane, number of carbon atoms representing the bridge (except bridge
head positions) are written with in bracket. Brid ge head positions are those which
join two rings.
7 C1 and C4 are joined
4 to bride of one carbon atoms

3 5

{ 2 6
{
C1 and C2 are joined C1 and C4 are joined
to bride of two carbon atoms to bride of two carbon atoms

It is alicyclic compund with seven carbon atoms, hence it is bicycloheptane. The two rings
have two C-atoms in common, (numbered 1 and 4) in structural formula. These positions
are called the bridgehead positions. Carbon atoms 1 and 4 are tied together by three
bridges of two, two and one carbon atoms. Hence, it is bicyclo [2, 2, 1] heptane (norbornane).

7
4

1
5
3
2 6
bicyclo [2, 2, 1] hept-2-ene bicyclo [2, 2, 0] hexane bicyclo [4, 4, 0] decane

GENERAL ORGANIC CHEMISTRY


QUIZRR 29

9 2
1 3
H3C 8
4
6
7 5

bicyclo [2, 1, 1] hexane bicyclo [1, 1, 1] pentane 8-methy bicylo [4, 3, 0] nonane

1
7
8 CH3 3
6
5

4
8 methyl bicyclo [3, 2, 1] octane

Ć Spiranes are polycyclics that share only one C. Spiro is written before the alkane.

spiro [4, 3] octane spiro [5, 2] octane

Ć For naming aromatic compounds, no special rules are needed to name them, but they
are named as substituted benzene.

CH3 CH3CHCH3 CH CH2

methylbenzene ispropylbenzene vinylbenzene Cyclopropyl


(toluene) (cumene) (styrene) (benzene)

GENERAL ORGANIC CHEMISTRY


30 QUIZRR
Ć When large and complex groups are attached to the benzene ring it is common practice
to name the molecule as an alkane, alkene, etc., and benzene as side chain derivative
written as pheny 1 (C 6 H 5 , ·,Ph).

CH3CHCHO

·C CH  ·CH2 · 

phenylbenzene
or
phenylethyne 2-phenylpropanal diphenylmethane biphenyl

Fo r A r o m a t ic r i n g
X

(1)
(6) (2)
(3)
(5)
(4)

Let there is an element on position 1. We do numbering of other positions, after defining the no.
1 position.
There are specific name for different positions.
position 1, 2 and 6  Ortho (o)
position 3 and 5  meta (m)
position 4  para (p)
NO2
NO2
Cl

Cl
m-choro nitro benzene Oăcholro nitro benzene

NO2

Cl
(p-chloro-nitrobenzene)

GENERAL ORGANIC CHEMISTRY


QUIZRR 31

Ex a m p l e s

1. Phenol

CH2 CH3

2. EthylBenzene

Cl

3. Chlorobenzene

OH

4. Phenol

OH

5. 3-Chlorophenol
Cl

OH
CH2 CH CH2 CH3
6. 1-phenylButană2ăol

7. H3 C CH CH2 CH3 2-ButylBenzene

8. CH2 C CH3 1-phenylpropană2ăone

GENERAL ORGANIC CHEMISTRY


32 QUIZRR

CHO

9. Benzaldehyde

COOH

10. Benzoic acid

CHO

11. 4-aminobenzaldehyde
NH2

C OO H

12. OH 3-hydroxy benzoic acid

CO NH2

13. Benzanamide

14. N H Năphenylbenzenamine

O
15. C NH CH 3 N-methylbenzenamide

O
NH C CH2 CH3
16. N-phenylpropoanamide

GENERAL ORGANIC CHEMISTRY


QUIZRR 33

I SOM ERI SM
In the study of organic chemistry we come across many cases when two or more compounds are made
of equal number of like atoms. A molecular formula does not tell the nature of organic compound;
sometimes several organic compounds may have same molecular formula. These compounds possess the
same molecular formula but differ from each other in physical or chemical properties, are called isomers
and the phenomenon is termed isomerism (Greek, iso = equal; meros = parts). Since isomers have the
same molecular formula, the difference in their properties must be due to different modes of the combination
or arrangement of atoms within the molecule. Broadly speaking, isomerism is of two types :
I. Structural isomerism : When the isomerism is simply due to difference in the arrangement
of atoms within the molecule without any reference to space, the phenomenon is termed
structural isomerism. In other words, while they have same molecular formula they possess
different structural formulas. This type of isomerism which arises from difference in the
structure of molecules, includes :
(i) Chain or Nuclear Isomerism;
(ii) Positional Isomerism
(iii) Functional Isomerism
(iv) Metamerism and
(v) Tautomerism
II. Stereoisomerism : When isomerism is caused by the different arrangements of atoms or
groups in space, the phenomenon is called Stereoisomerism (Greek, Stereos = occupying
space). The stereoisomers have the same structural formula but differ in the spatial
arrangement of atoms or groups in the molecule. In other words, stereoisomerism is exhibited
by such compounds which have identical molecular structure but different configurations.
Stereoisomerism is of three types :
(a) Geometrical or cis-trans isomerism; and
(b) Optical Isomerism
(c) Conformational isomerism
Thus various types of isomerism could be summaried as follows :
ISOMERISM

STRUCTURAL
ISOMERISM

GEOMETRICAL OPTICAL CONFORMER


ISOMERISM ISOMERISM ISOMERISM

GENERAL ORGANIC CHEMISTRY


34 QUIZRR
CH A I N I SOM ERI SM
This type of isomerism arises from the difference in the nature, structure of the carbon chain. For
example, for butane (C4H10), the following arrangements are possible.
(a) Butane (C4H10)

H3 C  CH  CH3
|
H3 C  CH2  CH2  CH3 CH3
n-butane (butane) iso-butane  2-methylpropane 
(straight chain) (branched chain)

(b) Pentane (C5H12)

CH3
H3CHCCH2 CH3 |
| H3C  C  CH3
H3 C  CH2  CH2  CH2  CH3 CH3 |
CH3
n-pentane iso-pentane (neo-pentane
(2-methylbutane) (2,2-dimethylpropane)

(c) Butanol (butyl alcohol)

H3CHCCH2OH
|
CH3
H3CăCH2ăCH2ăCH2ăOH isobutyl alcohol
(butan-1-ol) (2-methylpropanol-1)

POSI T I ON I SOM ER I SM
This isomerism arises due to the difference in the position of the same functional group or the same
substituent while the arrangement of carbon atoms remains same. For example,

(a) Chloropropane (C3H7Cl)

H3 C  C H  CH3
H3 C  CH 2  CH2  Cl |
(n-propyl chloride) Cl
(iso-propyl chloride)
(1-chloropropane) (2-chloropropane)

(b) Butene (C4H8)

H2 C  CH  CH2  CH3 H3 C CH  CH  CH3


1 2 4 1 2 3 4
(butane-1) (butene-2)

GENERAL ORGANIC CHEMISTRY


QUIZRR 35

(c) Propanol (C3H7OH)


H3 C  CH  CH3
H3 C CH 2  C H2  O H |
n-propyl alcohol OH
iso-propyl alcohol
(propanol-1)
(propanol-2)

FU N C T I ON A L I SOM ERI SM
The compounds having the same molecular but different functional groups are said to exhibit
functional isomerism. Such compounds are termed functional isomers. For example,
(a) For C2H6O
Ether Alcohol
H3C · O · CH3 H3C · CH2 · OH
(methoxymethane) dimethyl ether (ethanol) ethyl alcohol

(b) The molecular formula C3H6O represents two functional isomers.


Aldehyde Ketone
H3C · CH2 · CHO O
(propionaldehude) propanal ||
H3C · C · CH3
(acetone) propanone

(c) Carboxylic acids and esters


CH 3 CH 2 COOH CH 3 COOCH 3
Propanoic acid Methyl ethanoate
Only Methanoic acid cannot have its isomeric ester.

(d) Dienes, allenes and alkynes


CH 2 = CHăCH = CH 2 CH 2 = C = CHăCH 3 CH 3 CH 2 C   CH
Buta ă 1, 3 ă diene Buta ă 1, 2, ă diene But ă 1 ă yne
(An allene)

(e) Cyanides and Isocyanides


RăCN RăN C
Alkyl cyanide Alkyl isocyanide
(f) Nitro alkanes and alkyl nitrites
O

R N
RăOăN=O
O Alkyl niytites
Nitroalkanes

GENERAL ORGANIC CHEMISTRY


36 QUIZRR
(g) 1Ĉ , 2Ĉ and 3Ĉ amines

CH3
CH 3 CH 2 CH 2 NH 2 CH 3 CH 2 NHCH 3 |
H3C  N  CH3

propan ă 1 ă amine N ă methyl ethanamine N, N ă Dimethyl methanamine


(1Ĉ amine) (2Ĉ amine) (3Ĉ amine)
(h) Aromatic alcohols, phenols and ethers
CH2 OH OCH3 OH
CH3

Benzyl Alcohol Anisole Oă(cresol)

Met am erism
This type of isomerism arises from the unequal distribution of carbon atoms on either side of the
functional group in the molecule of compounds belonging to the same homologous series. For example,
(a) diethyl ether and methylpropyl ether are metamers.
H5C2ăOăC 2H 5 H3CăOăC3H 7
(diethylether methylpropyl ether
(eethoxyethane) (methoxypropane)
(b) A k et on i c com pou n d h av i n g t h e m ol ecu l ar f or m u l a C 5H10O

O O
|| ||
H5C2  C  C2 H5 H3C  C  C3H7
diethyl ketone methyl propyl ketone
(pentan-3-one) (pentan-2-one)

Taut om erism
It is a special type of functional isomerism in which an -hydrogen atom is shifted from one
position (atom-1) to another (atom-3). This is referred as 1, 3-shift. Such shfits are common between
a carbonyl compound containing an -hydrogen atom and its enol form.

R' R'
| |
 R 1 C 2 C  R''
R 1 C 2 C  R'' 
| || |
H 3O 3
Keto form OH
Enol form

GENERAL ORGANIC CHEMISTRY


QUIZRR 37

H = (CăH + CăC + C=0) ă (C=C + CăO + OăH) = 359 ă 347 = 12 kcal/mol


Thus, generally keto form is more stable than enol form by 12 kcal. So, in most cases, the
equilibrium lies towards the left.
Mostly the keto form is more stable than enol form but in certain cases, enol form can become the
predominant form. The enol form is predominant in following cases :
1. Molecules in which the enolic double bond is in conjugation with another double bond/phenyl
ring. In such cases, sometimes interamolecular hydrogen bonding also stabilizes the enol.

R R
C C R´ C C R´
H O O
Keto form H
Enol form
(crossăconjujation)

CH
CH3 C CH2 C OEt CH3 C C OEt
O O O O
H
Keto form Enol form
(Crossăconjugation and
intramolecular hydrogen bonding)

CH
CH3 C CH2 C CH3 CH3 C C CH3
O O O O
H
Keto form Enol form
(Crossăconjugation and intramolecular
hydrogen bonding)

2. Molecules, which contain two bulkyl aryl groups.

Me
Ar Ar
C C H C C H where Ar = Me
Ar Ar
H O H O Me
keto form (5%) Enol form (95%)

In the keto form of 2,2ădimesitylenthanal, the ArăCăAr bond angle is 109Ĉ, whereby the
bulky aryl groups experience greater steric repulsion. This steric repulsion eases off when the
keto form transforms to enol form, where the ArăCăAr bond angle widens to 120Ĉ.

GENERAL ORGANIC CHEMISTRY


38 QUIZRR
3. When the enol is aromatic stabilized.
O OH
H
H
Keto from Enol from

The extent of enolization is also affected by the solvent, concentration and temperature. Thus,
acetoacetic ester has an enol content of 0.4% in water and 19.8% in toluene. This is because water
reduces the enol content by hydrogen bonding with the carbonyl group, making this group less available
for intramolecular hydrogen bonding. The effectiveness of intramolecular hydrogen-bonding in stabilizing
the enol, with respect to the keto form is seen on varying the solvent and particularly on transfer to
a hydroxylic solvent with MeCOCH2COMe.

Solvent % Enol Thus, the proportion of enol in the non-polar solvent (hexane) is
Gas phase 92 the same as in the gas phase and higher than in the liquid itself,
the latter acting as a polar auto-solvent. The proportion drops
Hexane 92 again in the more polar MeCN and more dramatically in water.
Liquid 76 What is happening is the increasing relative stabilization of the
keto form by solvation, this being particularly marked in water
MeCN 58 where intramolelcular hydrogen bonding of the keto formÊs C=O
H 2O 15 groups can now take place as an alternative to its enolization.

Also, the enol content of pentană2,4ădione (CH3COCH2COCH3) is found to be 95% and 45% at
27.5Ĉ and 275.5ĈC respectively.
When a strong base is added to a solution of a ketone with -hydrogen atom, both the enol and
ketone form can lose a proton. The resulting anion is same in both the cases as they differ only in the
placement of electrons. They are not tautomers but canonical forms.
R R

C C·R C C·R

R R
O O
H
H

+ ă + +
+H ăH +H ăH
ă
O
R R
ă
C C·R C C·R

R R
O
{Canonical form} {Canonical form}

GENERAL ORGANIC CHEMISTRY


QUIZRR 39

Mec hanism :
In basic medium

O H O

OH ă ă
CH3 · C · CH2 CH3 · C · CH2

OH

ă
OH + CH3 ă C = CH2 CH3 · C = CH2
H2O

ă
O

In acidic medium
+
O H O·H
+
H
CH3 · C · CH3 CH3 · C · CH2

H
+
ăH

OH

CH3 · C = CH2

Ot h e r t y p e s o f t a u t o m e r i s m
(i) PhenolăKeto tautomerism

O OH

H
Cyclohedxadienone Phenol

In this case, enol form is more stable than keto form because of the aromatic stabilization.

GENERAL ORGANIC CHEMISTRY


40 QUIZRR
(ii) NitrosoăOxime tautomerism :

R R
C N C N
R R
H O OH
Nitroso form Oxime form

This equilibrium lies far to the right and as a rule nitroso compounds are stable only when
there is no -hydrogen atom.
(iii) Nitro-Aci tautomerism
Aliphatic nitro compounds are in equilibrium with the aci forms.

R O R  O R  O
C N C N C N
R  R O R
H O OH
H
Nitro form Aci form

The nitro form is much more stable than the aci form because nitro group has resonance. Aci
form of nitro compounds is also called nitronic acids.
(iv) ImineăEnamine tautomerism/cyanideăeminine tautomerism:

R R
C C R C C R
R R
H N R NH R
Imine form Enamine form

Imine form predominates generally. Enamines are stable only when there is no hydrogen
atom attached to nitrogen.

R R
C C C C
R R
H N
.. N H

O
||
Note : Keto-enol tautomerism occurs in those compounds in which  C  bond is attached through
the carbon of CăH bond when C is saturated.

GENERAL ORGANIC CHEMISTRY


QUIZRR 41

Ex a m p l e
Which will not show tautomerism :

O
||
(a) CH 3  C  H (b) O O

CH3 O O
| || ||
(c) H3C  C  C  H (d) CH 2  CH  C  H
|
CH3

Solution :
(b), (c) & (d) will not show tautomerism.
O
||
In (b) & (d)  C  is not attached to saturated carbon or we can say that the carbon attached to
O
||
the  C  group is not sp3 hybridized.

O
||
In (c),  C  is attached to carbon which does no have any hydrogen bonded to itself.

Therefore tautomerism is not possible in this case.

Ex a m p l e
Which will have more enol : Write the order of increasing enol content.

O O
|| ||
(1) CH 3  C  CH 2  C  O CH 3
..

O O
|| ||
(2) CH 3  C  CH 2  C  CH 3

O O
(3) || ||
C· CH 2 · C· CH 3

GENERAL ORGANIC CHEMISTRY


42 QUIZRR
Solution :
(1) O H O (2) H O
O

CH3 · C · CH · C · O · CH3 CH3 · C · CH · C · CH3

OH O OH O

CH3 · C = CH · C · OCH3 CH3 · C = CH · C · CH3

(enol form) (enol form)

(3) O H O

· C · CH · C · CH3

OH O

· C = CH · C · CH3

(enol form)
..
Due to presence of lone pair in (1) on  OCH , power of carbon to withdraw electron decreases,
3
therefore its enol content is least.
In enol form of (3) extended conjugation is obtained. Thus its most stable
 order is
1< 2< 3

Ex a m p l e
Which will have more enol content ?

O
O O
|| || O
(a) CH 3  C  C  CH 3 (b)

GENERAL ORGANIC CHEMISTRY


QUIZRR 43

Solution :
enol forms are
(a)
H O O OH O

CH2 · C · C · CH3 CH2 = C · C · CH3


(enol form)

O O·H
(b)

H O O

In (a) due to repulsion between 2 O atoms, rotation occurs & hydrogen bonding gets weaker.
Rotation is not possible in (b) due to five membered ring.
 (a) > (b) {enol content}

Ex a m p l e
Write all possible structural isomers of the compound with molecular formula C5H12O.
Solution :
Total number of structural isomers = 7

(i) CH3ăCH 2CH 2CH 2ăOH (ii) CH3  CH CH2 OH


|
CH3

CH3
|
(iii) CH3  CH  CH2 CH3 (iv) H3C  C  OH
|
| CH3
OH

(v) CH3CH 2ăOăCH2CH 3 (vi) CH3ăOăCH2ăCH2ăCH 3

(vii) CH3  O  CH  CH 3
|
CH3

GENERAL ORGANIC CHEMISTRY


44 QUIZRR
Ex a m p l e
C4H8 can have so many isomers. Write their structures.
Solution :
C4H8 can have following structures :
(a) CH3ăCH2ăCH=CH2 1-butene
(b) CH3ăCH=CHăCH3 2-butene, (cis and trans)

(c) CH3  C  CH2 2-methyl-1-propene


|
CH3

(d)

CH3
(e)

ST EREOI SOM ERI SM


When isomerÊs have the same structural formula but differ in relative arrangements of atoms or groups
in space within the molecule, these are known as stereoisomers and the phenomenon as
stereoisomerism.
Stereo isomers can be configurational isomers or conformation isomers.
A particular orientation (or arrangement) of atoms in a molecule, differing from other possible
orientations by rotation around single bonds is known as conformation.
Configurational isomerÊs can be geometrical isomers (or) optical isomers.

We d g e -d a s h d i a g r a m s
Usually drawn with two bonds in the plane of the page, one infront, and one behind to give the
molecule perspective. When drawing wedge-dadh it is a good idea to visualise the tetrahedral arrangement
of the groups and try to make the diagram „fit‰ this. As a suggestion, they seem to be most effective
when the „similar‰ pairs of bonds (2-in-plane, 2-out-of-plane) are next to each other, see below :

comes forwards out of the plane of the page  infront


goes backwards out of the plane of the page  behind

GENERAL ORGANIC CHEMISTRY


QUIZRR 45

Sa w h o r s e
Sawhorse diagrams are similar to wedge-dash diagrams, but without trying to
HH CH3
use „shading‰ to denote the perspective. The representation to the right of propane
has been drawn so that we are looking at the molecule which is below us and to H HH
our left.

N e w m a n Pr o j e c t i o n s H
H CH3
These projections are drawn by looking directly along a particular bond in the
system (here a C-C bond) and arranging the substituents symmetrically around the H H
H
atoms at end of the bond. The protocol requires that the atoms within the central bond
are defined as shown below :

H
The „circle‰ defines the back H CH3 The „dot‰ defines the front
carbon atom and the groups carbon atom and the groups
attached to it. H H attached to it.
H

In order to draw a Newman projection from a wedge-dash diagram, it is useful to imagine putting
your „eye‰ in line with the central bond in order to look along it.
LetÊs work through an example, consider drawing a Newman projection by looking at the following
wedge-dash diagram of propane from the left hand side.
Ć First draw the dot and circle to represent the front and back C respectively
Ć Since the front carbon atom has an H atom in the plane of the page pointing up we can add
that first
Ć The back carbon atom has an H atom in the plane of the page pointing down
Ć Now add the other bonds to each C so that it is symmetrical
Ć The groups / bonds that were forward of the plane of the page in the original wedge-dash
diagram are now to our right
Ć Those behind the plane are now to our left

H H
CH3

H H
H

H H
H CH3 CH3 H

H H H H
H H

GENERAL ORGANIC CHEMISTRY


46 QUIZRR
Co n f o r m e rs o f c y c l o h e x a n e
The cyclohexone ring can assume many shapes due to rotation around carbon-carbon bonds.
The chair form and the boat form are the extreme cases.
1 4 1

3 2 3 2

5 6 5 6
4 Boat (less stable)
Chair (more stable)
Ć Each Carbon atom of Cyclohexane is bonded to two Hydrogens. The bond to one of
these hydrogen lies in the plane of the ring; this is called equatorial hydrogen. The
bond to the other hydrogen atom is parallel to the axis, this hydrogen atom is called
Axial Hydrogen.
Ć I n Chai r f or ms, C – H f l agpol es at C 1 and C 4 are in trans side thus minimising strain
and thus increasing stability.
Ć In boat form, there is some repulsion between C ă H flagpoles at C 1 and C 4 being on
same side and thus steric hindrance is increased and stability decreased.

N e w m a n Pr o j e c t i o n
These projections are obtained by viewing the molecule along the bond joining the two
carbon atoms. H H H

H H

H
H H
H H
H
H
Eclipsed Staggard
The various other conformers of ethane are :
Ha Hb Ha Ha H
60Ĉ
H Hb
120Ĉ
60Ĉ 60Ĉ
rotation rotation
H H
H H H
H H H
H Hb
H
Eclipsed (I) Skew or gauche (II) Eclipsed (III)

GENERAL ORGANIC CHEMISTRY


QUIZRR 47

Ha Ha H Ha

H H Hb H
180Ĉ
60Ĉ 60Ĉ
rotation rotation
Hb
H H H
H H H
H

Hb H
Staggard (IV) Eclipsed (V) Skew or gauche (VI)

1 7 .1 Co n f o r m a t i o n a l I s o m e r i s m
Conformational isomers (or conformers) are isomers formed by the rotation of bond.
(a) Conformations of Ethane
In ethane, the two carbon atoms are connected by a -bond. If one of the methyl group
in ethane molecule is kept fixed and the other is rotated about the CăC bond, infinite
number of conformers of ethane result. The conformations can be represented by
sawhorse or Newman projections.
(i) Sawhorse Projection
The sawhorse projections are represented as show in the figure. The two extreme
conformers are termed as staggered and eclipsed and the conformations lying between
them are termed as gauche or skew conformations.

H H

H H H

H H
H
H H
H H
(eclipsed) (eclipsed)

There are 2 types of strain among conformers :


(i) Torsional strain - strain due to repulsion between the bonded electron pair of CăH or
CăC bonds or other types of bonds.
(ii) Vander WaalÊs StrainăStrain due to repulsion or crowding when atoms or groups are
too close to each other due to electrostatic repulsion of the electrons.
Eclipsed form of ethane has both these strains more than in the skew form. Skew and
staggered from have similar strains, so they are equally stable

GENERAL ORGANIC CHEMISTRY


48 QUIZRR
Po t e n t i a l En e r g y Pro f i l e f o r Et h a n e Co n f o r m e r s
Energy Profile for Ethane Conformers
(I) eclipsed (III) (V) (VII)
eclipsed

PE

staggered staggered
(II) (IV) (VI)
60

120

180

240

300

360

Dihedral Angle (degrees)

(b) năButane Conformations


H H
·

1 2 3 4
CH3 · C · C · CH3
·

H H
If we consider rotation about C 2 ă C 3 bonds, we get the following conformers. There Anti
and Gauche conformations are represented by prefix A and C. These conformers represent
staggered conformations.
CH3
H3 C H 3 H
2 H H
3 CH3 Anti
A C C
CH3 H H Conformation
CH3 2 H
H CH3
H HCH3
H3 C 3
2 partially eclipsed
B C C H3C H CH3 3
2 Conformation
H CH3
CH3 H
2 H H
H
CH3
CH3
H3 C CH3
2 3 CH3 CH3 CH3
Gauche
C C C 3
2 Conformation

CH3
3 H3 CCH3
H3 C CH3
3 3
C C Eclipsed
D 2 Conformation
2

GENERAL ORGANIC CHEMISTRY


QUIZRR 49

D D

B B

C C

Dihedral Angle (degrees)

Out of the four conformations listed above, anti-conformation is the most stable since in
this conformations the two non-bonded methyl groups and the four hydrogen atoms are as far
apart as possible. The next in order of higher energy come the gauche conformations in which
the two non-bonded methyl groups are only 60Ĉ apart and hence causing crowding or steric
strain them eclipsed forms.
Next in order of higher energy fall the partially eclipsed conformations.
The fully eclipsed conformation is however, the least stable. This is due to the reason that
in this conformation, there is one methyl-methyl eclipsing interaction and two weak hydrogen-
hydrogen eclipsing interactions. Thus, the order can be sumarised as :
Anti > Gauge or skew > Partially eclipsed > fully eclipsed.

Ex a m p l e
Increasing order of stability among the three main conformations (i.e. Eclipse, Anti, Gauche)
of 2-fluoroethanol is
(a) Gauche < Eclipse < Anti (b) Eclipse < Anti < Gauche
(c) Anti < Gauche < Eclipse (d) Eclipse < Gauche < Anti
Solution :
Eclipsed form of 2-fluoroethanol is least stable due to repulsion between the F and OH groups as
well as due to torsional strain. Gauche form is most stable as the torsional as well as vander WaalÊs
strain both are considerably reduced and the intramolelcular H-bond further increases the stability.
Therefore, the increasing order of stability is Eclipse < Anti < Gauche.
 (b)

GENERAL ORGANIC CHEMISTRY


50 QUIZRR
Ge o m e t r i c a l I s o m e r i s m
Isomers which posses the same molecular and structural formula but differ in arrangement of
atoms or groups in space around the double bonds, are known as geometrical isomers and the phenomenon
is known as geometrical isomerism.
Condition for the presence of Geometrical isomerism :
(1) Rotation about C ă C bond must be hindered

C=C ăCă
ăCă (ring)

(2) On some carbon two different groups should be attached

(i) Cisătrans isomerism : When similar groups are on the same side it is cis and if same groups
are on the opposite side it is trans isomerism.

HOOC COOH
C=C HOOC H
C=C
H H H COOH
(Maleic acid)
Fumeric acid
(cis form)
(trans form)

(ii) Syn-anti isomerism :

Compounds conteining C = N ă OH units are commonly called oximes. Apart from

H
oxime of formaldehyde C = N ă OH , Oximes of any other aldehyde exibits
H
geometrical isomerism. The nomeclature of aldoximes is Syn (when H and OH present
on same side) and anti (when H and OH on opposite sides).

CH3 CH3
H H
C C

N N
OH (aldoxime) OH
(Syn) (anti)

C6 H5 N C6 H5 N
C6 H5 N N C6 H5

(Syn) (anti)

GENERAL ORGANIC CHEMISTRY


QUIZRR 51

R
Also in case of oximes of un symmetrical ketons C = N ă OH show geometrical
R'

isomerism.
R R' R R'
C C

N N
OH OH
Syn (R') anti (R')
or or
anti (R) Syn (R)

(iii) In cyclic compounds: Because of the impossibility of rotation around CăC single bond
as the conformation of cyclic compound would twist on rotation, they show isomerism.

ï ï ï

1, 2 ă cis 1, 2 ă trans 1, 3 ă cis

ï ï ï

ï ï

1, 3 ă trans 1, 4 ă cis 1, 4 ă trans

Di f f e r e n t i a t in g p r o p e r t i e s o f c i s -t r a n s i s o m e r i s m
(i) Dipole moment : Usually dipole moment of cis is larger than the trans-isomer.

Cl H Cl
H
C C

C C
H Cl Cl H
ø = 1.84D ø = 0.8D

GENERAL ORGANIC CHEMISTRY


52 QUIZRR
(ii) Melting point : The steric repulsion of the group (same) makes the cis isomer less stable than
the trans isomers hence trans form has higher melting point than cis.

E a n d Z n o m e n c l a t u r e o f g e o m e t ri c a l i s o m e r is m
If all the four groups / atoms attached to C = C doube bond are different, then Cis and trans
nomenclature fails in such cases and a new nomenclature called E and Z system of nomenclature
replace it.
H I higher priority
H II
C C

C C
Br Cl Cl Br
higher priority
higher priority
E Z

The group / atom attached to carbon-carbon double bond is given to higher rank, whose atomic
weight is higher. If the two higher ranked group are across, it is called E form (E stands for the German
word entgegen meanin thereby opposite) and if the two higher ranked groups are on the same side,
they are called Z-form (Z stands for German word Zusammen meaning thereby on the same side).

Ru l e s f o r d e c i d i n g p r i o ri t i e s a r e :
(1) Higher the atomic number of the atom attached to the double bond carbon, higher will be the
priority.
(2) If same priority group are on same side  Z
If same priority group are on different side  E

CH3 CH3
(2)
(1)
CH CH
C CH3 C
(2)
CH3 (1)
C C
H D H D
(2) (1) (2) (1)

(3) If the atom attached to the double bonded carbon are same, then priorities are decided by
next atoms attached to that atom.
(4) If the isotopes are attached to double bonded carbon atom then higher the (2) (1)
atomic mass, higher will be priority. CH3
Note : Higher priority is marked by (1) and lower by (2)
C

C
Br OH
(1) (2)

GENERAL ORGANIC CHEMISTRY


QUIZRR 53

(5) If the atom attached to the double bonded carbon continues double or triple bond then the
priorities are decided by considering the same atom twice or thrice.

(2) (1)
H3C CH2CH3
C

C
Br H
(1) (2)

N u m b e r o f Ge o m e t r i c a l I s o m e r s
(i) The number of isomers when there is only one double bond  2
(ii) If there are n alternate double bonds and their ends are not identical  2 n

For Example
CH 3 ă CH = CH ă CH = CH ă CH 2 CH 3

The isomers are


(1) (2)
H H H
CH2 CH3

CH3 C=C CH3 C=C

C=C CH2 CH3 C=C H

H H H H
cis ă cis cis ă trans

(3) (4)

H H H
CH2 CH3

H C=C H C=C

C=C H C=C CH2 CH3

CH3 CH3
H H
trans ă trans trans ă cis

Hence the compound has 2 2 = 4 isomers

GENERAL ORGANIC CHEMISTRY


54 QUIZRR
(iii) If the ends are identical, then
isomers = 2 nă1 + 2 pă1

n
where p = if (n is even)
2

n 1
p = if (n is odd)
2

For example
CH 3 ă CH = CH ă CH = CH ă CH 3
The isomers are :
(1) H H

CH3 C=C

C=C CH3

H H
cis ă cis

(2) H
CH3

CH3 C=C

C=C H

H H
cis ă trans

(3) H
CH3
H C=C

C=C H

CH3
H
trans ă trans

So, total isomers = 2 1 + 2 0 = 3

GENERAL ORGANIC CHEMISTRY


QUIZRR 55

Some students night be wondering that a trans ă cis isomer should also be considered as
done in the earlier case, but please note that here cis-trans and trans - cis isomers are identical,
hence we can write them as only one isomer :

Ex a m p l e
Find the total number of isomers of the following compounds and draw them.
(a) CH 3 ă CH = CH ă CH = C = CH CH 3
(b) CH 3 ă CH = CH ă CH = C = C = CH CH 3

CH3
C = C = C = CH · CH3
(c)
CH3

Solution :
(a) This part is an exception to the formulae used for calculating the number of isomers
with a alternate double bonds.
(i) CH C = C = CHCH3
3

C=C

H H
cis
(ii)
H CH = C = CHCH3

C=C

CH3
H
trans isomer

Note: For an isomer to exist, all the groups must be in the same plane, whether or
not the whole structure is in same plane.
Hence, for compounds like
R ă CH = C = CH ă R´, isomers do not exist because R and R´ are in perpendicular plane.

R R
C C C

H
H
[we can see that R is in the plane of paper and R´ is into the plane of paper (H is coming out
of plance)].

GENERAL ORGANIC CHEMISTRY


56 QUIZRR
(b) The isomers are :
(i) H H

CH3 C=C=C=C

C=C CH3

H H
cis ă cis

(ii) H CH3

CH3 C=C=C=C

C=C H

H H
trans ă cis

(iii) H CH3

H C=C=C=C

C=C H

CH3
H
trans ă trans

(iv) H
H

H C=C=C=C

C=C CH3

CH3
H
trans ă cis

Hence, there are 4 isomers because now the groups attached to the first and last carbon
atom are in the same plane.

GENERAL ORGANIC CHEMISTRY


QUIZRR 57

(c) CH3 H

C=C=C=C

CH3 CH3

This compound has no isomers, as the groups are not proper (two CH 3 on the same carbon
atom).

ST EREOCH EM I ST R Y T ER M I N OL OGY
Stereochemistry is all about the 3D spatial aspects of chemistry.
Stereoisomers are molecules that differ only in the arrangement of bonds in 3D space.
Superimposable Many objects (including molecules) are indistinguishable from their mirror images,
so they are superimposable.
Non-superimposable Other objects, such as your left and right hands, can be distinguished, they
are non-superimposable.
Chiral An object that has a non-superimposable mirror image is said to be „chiral‰
(Greek = „handedness”) and one that has a superimposable mirror image is „achiral‰.
If we restrict our thoughts to 2D for a second, and think about letters of the alphabet, some are
chiral while others are not. To appreciate this, print some words on a piece of paper, then look at them
in a mirror. Do they still look the same (superimposable) or are they different (non-superimposable) ?
Words like „MOM‰ are the same as the mirror image but „DAD‰ is not. ThatÊs why the word
„AMBULANCE‰ on the front hood of the vehicle looks „odd‰ unless you are looking at it in your rear
view mirror.
The most common type of „chirality‰ is observed when a carbon atom has four different groups
attached to it (so it must be sp3 hybridised). This carbon atom is them described as achirality center.

En a t i o m e r s
Ć If a pair of stereoisomers are non-superimposable mirror images of each other, then they are
enantiomers.
Ć If you donÊt know what is meant by non-superimposable mirror images then note that your
left and right hands are a pair of enantiomers.
If objects are superimposable, it means you can not tell them apart. If they are non-
superimposable, then you can always distinguish them.
Ć For example, if you look at your hands, you will see that while they are very similar, they
are essentially mirror images. They are not identical, because you can always tell which one
is which!

Me Me

H Cl H
Et Cl
(R)-2-chlorobutane Et
(S)-2-chlorobutane

GENERAL ORGANIC CHEMISTRY


58 QUIZRR
A molecule that can exist as a pair of enantiomers has the property of chirality or is described
as chiral. A molecule that lacks this property is achiral.
You should be a ble to draw the two mirror ima ges of a n a chira l molec ule a nd then see
if you rota te one of the ima ges the two molec ules a re a ctua lly the same (i.e. tha t they a re
superimposable). For exa mple, compa re 2-chloropropa ne a nd 2-chlorobuta ne systems :

CH3 CH3 CH3


b a
H CH3 H3 C H H CH3
Cl a Cl Cl b

though C a and C b are different they are indistingushable


from each other

Because the 2 alkyl groups are indistinguishable, the original molecule and itsÊ mirror image are
superimposable.

CH3 CH3 120Ĉ CH2 CH3

H CH2 CH3 H3CH2 C H H CH3


Cl Cl Cl
mirror
plane

non-superimposable structures

Because the 2 alkyl groups are distinguishable, the original molecule and itsÊ mirror image are
non-superimposable.

I m p o r t a n t p r o p e r t i e s o f e n a n t io m e r s t h a t y o u s h o u l d k n o w :
Ć They have different physical properties
Ć They have different chemical/biological properties
A mixture that contains equal quantities of enantiomers is called a racemate or a racemic
mixture.

DI A ST EREOM ER S
There are a couple of ways we can use to define diastereomers :
Ć Stereoiosmers that are not mirror images are diastereomers
Ć Stereoiosmers that are not enantiomers are diastereomers.

GENERAL ORGANIC CHEMISTRY


QUIZRR 59

Chiral molecules that possess two (or more) chirality centers (we will define this soon) have
stereoisomers that may be either enantiomers or diastereomers. Diastereomers are stereoisomers that
are not mirror images.

Br CH3
H
H
H3C Cl (S,R) - 2- bromo-3-chlorobutane

H CH3
Br
H
H3C Cl
(R,R)-2-bromo-3-chlorobutane
Diastereomers can have quite different physical and chemical properties from one another. This
is important as it allows them to be separated.

OPT I CA L I SOM ERI SM


A compound, in which an atom (say carbon) has all the four groups different, is said to have
chirality and this centre (carbon) is said to be Chiral (asymmetric) centre or stereogenic centre.
a a

x C b x C x

y y
Chiral centre achiral centre
(as all groups are different) (2 same x on the carbon atom)

A better way to determine whether a molecule is chiral is to examine its symmetry (called as
plane of symmetry). If an imagniary plane passes through the centre of the object and divides
the object so that the one half is the mirror image of the other half is having of plane of symmetry.
Any molecule that has a plane of symmetry is not chiral.

CH3 O CH3

H C H

Cl

Plane of symmetry not a plane of symmetry


(achiral) (chiral)

GENERAL ORGANIC CHEMISTRY


60 QUIZRR
OPT I CA L A CT I V IT Y
Ordinary light has vibrations in all possible planes. The ordinary light can be converted
into plane polarized light by making it pass through a nicolprism. The plane polarized light has
vibrations only in one plane. When this plane polarized light is passed through a compound,
and it turns the plane of the polarized light, then it is said to be optically active.

compound A

nicol prism Plane polarized (A is optically


light active)

Dextrorotatory ă If the compound A (isomer) rotates the plane of the polarized light to
the right, it is dextrorotatory (d)
Leavorotatory ă If the compound A (isomer) rotates the plane of polarized light to the
left, it is Laevorotatory (l)

Co n d i t i o n s f o r Op t i c a l A c t i v i t y
(1) The compound should be chiral (asymmetrical). The compound should be devoid of any
element of symmetry like plane of symmetry, centre (point) of symmetry etc.
(2) The mirror image of the compound should be non-super imposable on it.
Plane of symmetry (  ) : A plane which when passed through a molecule divides in into
two identical parts (mirror image)
b

O
a b

a b
H H

(plane of symmetry) b

Such an isomer is called meso isomer, which is optically inative due to the presence of
plane of symmetry. The optical rotation of one half of the molecule is exactly cancelled by the
other half. This is called internal compensation.
Centre of Symmetry : It is an imaginary point within the molecule and is present when
all the joining similar groups pass through a common point. This common point is called as
centre of symmetry.

GENERAL ORGANIC CHEMISTRY


QUIZRR 61
c

a b

b
i (centre of symmetry)

c
For ringed structures
Ph COOH

H H

H H

HOOC Ph

It must be noted that only even membered rings possess a centre of symmetry. It is not
found in odd-membered rings. Centre of symmetry will never exist in a molecule having only
the chiral centre.

FISH ER PROJ ECT I ON FORM U L A


Steps for drawing fisher projection :
(1) Draw horizontal cross equal to the number of a symmetric carbon atom.
(2) Carbon atoms are arranged on vertical line, C 1 being at the top or most oxidized form
at the top. E.g. CH 3 CH (OH) COOH

COOH COOH

HO H HO H

CH3 CH3

GENERAL ORGANIC CHEMISTRY


62 QUIZRR
(3) Groups present on vertical line are behind the plane and groups on horizontal are
above the plane.
COOH COOH

HO C H H C HO

CH3 CH3
This is a general method used for representation of compound in a 3-Dimentional space.
This shows that veritcal bonds are extending behind the plane whereas the horizontal bonds
project in front of the plane of page.
(a) Relative Configration : This method for designating configuration of enantiomers
uses the descriptors D and L. As we have already done, D stands for dextrorotatory and
L for leavorotatory.
Note: atoms other than C and H are called hedro atoms.
Now, for assigning the compound as D and L, we draw an arrow from H to hedro atom
through topmost element i.e. most oxidised from.

COOH COOH

H HO HO H

(dextro) (leavo)
CH3 CH3

If the arrow goes in a clockwise manner, it is called dextro rotatory and if the arrow
goes in anti clockwise, it is leavorotatory.
When the molecule contains more than one a symmetric atom, there can be differnt
configuration at different chiral carbon.

COOH COOH

L D
HO H H HO

D L
H HO HO H

COOH COOH

GENERAL ORGANIC CHEMISTRY


QUIZRR 63

COOH

L
HO H

D
H HO

COOH

(b) Absolute Configuration: optical isomers in this configuration are now written in
terms of R (rectus) and S (sinister).
Ć This is based on the actual 3-dimensional formula of the compound and not upon
its projection on a 2-dimenstional surface (as done in relative configuration).
Ć If a, b, c and d are the groups attached to the chiral atom, there on arranging
these groups in a priority sequence, configuration,
R is given to the isomer if sequence is left to right
S is given to the isomer if sequences is right to left.

Pr i o r i t y Cl a s s i f i c a t io n :
(1) Higher the atomic number, higher the priority.

Cl

H C F

Br

Br  Cl  F  H
For this sequences will be (35) (17) 9 (1)

(2) If the first atom fails to assign the priority then check for second corresponding atom
For example, while deciding priority amongst the following groups, they can be
considered as

H H

(a) H C O < > C O

O C

GENERAL ORGANIC CHEMISTRY


64 QUIZRR
N

(b) C N < > C N

N C

O O C

(c) C NH2 < > C NH2

So, while deciding the next atom in all these compounds, the structure is droken into the
structure as shown on the right hand side.
IMPORTANT NOTE : For assigning the configuration, the enantiomer is viewed along
the axis of the bond connecting the chiral centre to the group of lowest priority and from the
side of the mulecule opposite to this atom.
a

C Here d has the lowest priority


b c

Now,
Here the sequence is a  b  c  d and we can see that it is anti clock wise hence its
configuration is S.

c c

C C
a b b a

d d
(S) (R)

GENERAL ORGANIC CHEMISTRY


QUIZRR 65

Illust rat io n
Find the R & S configuration.
CHO

H3C C CH2 CH3

H
Solution :
For this case priority order is
CHO CHO
O
||
· C· H (a) Carbon attached to oxygen C C
CH3 CH2 CH3 CH3 CH2 CH3
· CH 2 CH 3 (b) Carbon to carbon
· CH 3 (c) Carbon to hydrogen. H H
(R) (S)

When the molecule contains more than one chiral centre, configuration of each centre is
determined seperately following same sequence rule.
Simplest way is to take group of lowest priority at the bottom side (we shall call it viewing
side). In this process, we always carry out even number of changes. If we take odd number of
changes, if alters its configuration).

2
COOH COOH

1 one interchange
4 H OH H3C OH

3 H
CH3 OH
second
interchange

H3C COOH

GENERAL ORGANIC CHEMISTRY


66 QUIZRR
Hence, we have here even number of changes, thus it is an R configuration.
Let us have another example,

H CHO

HO CHO H2N H

NH2 OH

NH2

OHC OH

H
(S)

An example with two chiral carbons

4
COOH

3
H OH

2
HO H

1
COOH

Here, we determine configuration of each carbon separately.

GENERAL ORGANIC CHEMISTRY


QUIZRR 67

For C 2

CH(OH)COOH OH

2 2
HO H HOOC(OH)HC COOH

COOH H
( R at C2 )

For C 3

COOH OH

3 3
H OH COOH(OH)HC COOH

CH(OH)COOH H
( R at C3 )
Thus,
4
COOH

3
H OH
is (2R, 3R) ă tartaric acid.
2
HO H

1
COOH

Meso c om pounds
In the simplest case, a compound with two chirality centers where there is the same set of four
groups at each chirality center, the combination where the four groups are arranged such that the
centers are mirror images of each other (i.e. where the molecule has an internal mirror plane) is a meso
compound.

GENERAL ORGANIC CHEMISTRY


68 QUIZRR
For example consider 2, 3-dichlorobutane. There are two chirality centers (C2 and C3), each with
ăH, ăCl, ăCH3 and the other ăCHClCH3 groups attached. Since there are two chirality centers, then,
at least in principle, there are 4 possible permutations (R, R), (R, S), (S, R) and (S, S).

H Cl H H Cl Cl Cl H
Cl Cl Cl H H H Cl
H
H3C CH3 H3C CH3 H3C CH3 H3C CH3

(R, R) (R, S) (S, R) (S, S)

If we look at the (R, S) and (S, R) as drawn above, it should be reasonably easy to recognise that
these two structures are actually the same thing because they are superimposable. This structure is the
meso isomer.
H H ClCl H H Cl Cl
Cl Cl H H Cl Cl H H
=
H3C CH3 H3C CH3 H3C CH3 H3C CH3
(R, S) (S, R)
rotate the whole
(S, R) -2, 3-dichlorobutane molecule 180Ĉ
(R, S)-2, 3- dichlorobutane (S, R)-2, 3- dichlorobutane

In the conformation shown, the mirror plane is obvious. However, lets look at other conformations
of the meso isomer and make sure we recognise why it is the meso isomer.
Important properties of meso compounds that you should know :
For simple meso compounds with 2 chirality centers :
Ć they are optically inactive
Ć they must be (R, S) configuration
Ć they are diastereomers of the (R, R) and (S, S) isomer.

Isom er t ypes
Compounds that have the same molecular formula but different chemical structures are called
isomers. Remember isomerism is a property between a pair (or more) of molecules, i.e. a molecule is an
isomer of another molecule. A similar relationship is that of brother or sister... you can only be a brother
or sister to someone else.
Depending on the nature of the difference between the structures, it is possible to classify isomers
into various sub-types. The following tree diagram should help you recognise the difference based on
a simple YES/NO question.
Isomers are compounds with the same molecular formulae but that are structurally different in
some way. It is important to be able to recognise isomers because they can have different Chemical,
Physical and biological properties.

GENERAL ORGANIC CHEMISTRY


QUIZRR 69

Do the compounds'
have the same molecular
formulae?
NO
Not isomers
YES
lsomers

Do the compounds have


the same connectivity?
NO YES

Constitutional Stereoisomers

O
OH Can the compounds be
interconverted by rotation
about single bonds ?

NO YES

Configurational Conformational

H3C H H H
Is there isomerism at a H H H
double bond ?
YES H
NO H CH3 H3C CH3

Geometric Optical

Are the compoundss


non-superimposable
mirror images?

NO YES

Diastereomers Enantiomers
Cl H Cl Cl
H3C H H3C Cl
CH 3 CH2 H H CH2 CH3
H H CH3
H3C
H3C Br H3C Br

GENERAL ORGANIC CHEMISTRY


70 QUIZRR
N u m b e r o f o p t ic a l i s o m e r s
Number of possible optical isomers in compounds containing different no. of asymmetric atoms.
1. The molecule has no symmetry
The no. of d and l ă forms a = 2n
n = no. of asymmetric atoms
The no. of meso l-forms m = 0
Total no. of optical isomers = a + m = 2n
2. The molecule has symmetry and n is even
The no. of d and l forms a = 2nă1

n
Meso forms m = 1
22

n
= 2n 1  2 2 1

Tartaric acid

COOH
·

H · C · OH
· ·

H · C · OH Plane of symmetry n = 2

COOH

a = 2nă1 = 22ă1

n
m = 1 = 21ă1 = 20 = 1
22

Total = 2 + 1 = 3

COOH COOH COOH


·

H · C · OH HO · C · H H · C · OH
·

HO · C · H H · C · OH H · C · OH
·

COOH COOH COOH


I II III

(meso forms)

One of asymmetric centre is rotating plane polarised light towards right other towards left so
total optical rotation is zero.
I & III or II & III which are not mirror image to each other are called diastereomers.

GENERAL ORGANIC CHEMISTRY


QUIZRR 71

3. The molecule has symmetry & n is odd

 n  1
n 1
The no. of d and l forms a  2 2 2

 n1 
Meso forms m  2 2

Total = a + m
= 2(nă1)
e.g. Lactic acid

CH3
|
H  C  OH
|
COOH

n = 1 no symmetry
d & l forms = 2n = 2
meso form = 0
total = 2

CH3 CH3
| |
H  C  OH OH  C  OH Mirror images are also called as enantiomers
| |
COOH COOH

Op t ic a l ly a c t iv e c o m p o u n d s h a v i n g n o c h i ra l c a r b o n
The presence of chiral carbon is neither a necessary nor a sufficient condition for optical activity,
since optical activity may be present in molecules with no chiral atom and since molecules with two or
more chiral carbon atoms are superimposable on their mirror images and hence inactive.
(i) Any molecule containing an atom that has four bonds pointing to the corners of a tertahedron
will be optically active if the four groups are different.

16
O

CH2 S CH2

1 8O

(ii) Atoms with pyramidal bonding might be expected to give rise to optical activity if the atom
is connected to three different groups, since the unshared pair of electron is analgous to a
fourth group.

Z
N
X Y

GENERAL ORGANIC CHEMISTRY


72 QUIZRR
Many attempts have been made to resolve such compounds, but until recently all failed
because of umbrella effect, also called pyramidal inversion. The umbrella effect is rapid
oscillation of the unshared pair from one side of XYZ plane to the other.
(iii) Allenes, with even number of adjacent double bonds are optically active if both sides are
dissymmetric.

H3C H H CH3
C=C=C C=C=C
H3C H H3C H
Inactive Active

Explanation :

H3C CH3
SP 2 SP
C=C=C SP 2
H3 C 1 2 3
CH3

Central carbon is sp. Therefore both the double bonds are in perpendicular direction thus in
different plane. If carbon (1) is in plane of paper then the group attached to carbon (3) are
in plane perpendicular to plane of paper. Thus no plane of symmetry exists. Thus the structure
is chiral.

CH2 CH2

(iv) CH2 is optically active.


CH2

Because both the benzene rings are in different plane, therefore structure is non-
superimposable over its image. Thus it is optically active.
(v) Biphenyls
H H

H H
It is opically inactive because the mole cule is symmetrical and plannar.

GENERAL ORGANIC CHEMISTRY


QUIZRR 73

So m e i m p o r t a n t p o i n t s :
Ć If bulky groups are present at ortho positions of both the rings, then rotation about the
CăC bond takes place, to reduce the extra energy due to repulsion and stabalize the
molecule, now the two phenyl molecules are not in same plane, hence optically active.
NO2 NO2 below plane
of paper

outside the
COOH COOH plane of paper

Ć Despite the molecule is non planar, it is optically inactive because of the presence of
plane of summetry. (plane through A & B cutting their molecules into halves and on
the other phenyl both groups are same)
A D

B D
(vi) SPYRO
In these type of compounds, the alternate molecules are in same plane.
(1) a a

b b
(optically active because of the absence of plane of symmetry)

(2)
a a

Plane of
Symmetry

b b
(Optically inactive)

GENERAL ORGANIC CHEMISTRY


74 QUIZRR
Ex a m p l e
Find whether the following compounds are optically active or not.
D
T C H

R1 R3 R1 R3
(i) (ii) R2 R4
R2 R4

CH3 Cl

(iii) H H H
H

Cl CH3
Solution :
Compound (i) is optically active because there is no plane of symmetry, which can cut the molecule
into two equal halves. Compound (ii) also does not have plane of symmetry so, it is optically active.
Compound (iii) is optically inactive because of the presence of centre of symmetry.

Ex a m p l e
Find out whether the given compounds are optically active or not.
H C H
SO3 H SO3 H 2 5
(i) (ii)
I H I CH3

CH3 C3 H7
(iii) C= C=C=C
C2 H5 CH3

Sol u t i on :
In compounds (i) and (iii), there is plane of symmetry passing through the compounds (molecular
plane). Therefore, they are optically inactive. Compound (ii) does not have any plane of symmetry as
the two phenyl rings are not in the same plane. One of the ring rotates about C-C bond axis because
of bulky substituents at O, O´ positions of two adjacent phenyl rings and the two rings are perpendicular
to each other. So, (ii) is optically active.

Op t i c a l Pu r i t y
The optical purity or the enantiomeric excess (ee%) of a sample can be determined as follows :
Optical purity = % enantiomeric excess = % enantiomer1 - % enantiomer2
= 100 []mixture / []pure sample
ee% = 100 ([major enantiomer] - [minor enantiomer]) / ([major enantiomer] + [minor enantiomer])
GENERAL ORGANIC CHEMISTRY
QUIZRR 75

where [major enantiomer] = concentration of the major enantiomer


[minor enantiomer] = concentration of the minor enantiomer
Diasteromeric substances can have different rotations both in sign and in magnitude.

Ex a m p l e
The specific rotation of Ră2ăbromooctane is ă 36Ĉ. What is percentage composition of a
mixture of enantiomers of 2-bromooctane where rotation is + 18Ĉ.
Solution : Let X = mole fraction of R,
 (1 ă X) = mole fraction S.
X (ă 36Ĉ) + (1 ă X) 36Ĉ = 18Ĉ
1
or, X
4
The mixture has 25% R and 75% S, it is 50% racemic and 50% S.

T H E B REA K I N G A N D FORM I N G OF B ON DS
Organic reactions usually involve the fission and formation of covalent bonds. The covalent bond is
often represented as a dash (·) and the movement of a pair of electrons is shown by a curved arrow,
( ), where as the movement of a single electron is a shown by fish hook arrow .
There are two ways in which a covalent bond may be broken.

H o m o l y t i c Fi s s i o n
Suppose the atoms X and Y are bonded through a covalent bond. When a bond breaks such that each
of the atoms retains one of the bonding electrons, the process is called homolytic bond cleavage (or)
homolysis. Homolytic fission results in the formation of free radicals.

XăY X+Y
Free radicals

H e t e r o l y t ic Fi s s i o n
When more electronegative atom remainÊs attached with less electronegative atom, then the bonded
pair of electrons moves towards more electronegative atom, thereby more electronegative atom gets
negative charge and less electronegative atom gets + ve charge such type of fission is known as
heterolytic fission i.e. this fission occurs by a shifting of bonded pair of electrons from less
electronegative atom to more electronegative atom.
The ions of carbon, known as carbanions and carbonium ions, are of special importance in organic
chemistry.
Homolytic chemistry is the chemistry of the odd electron; Heterolytic chemistry is the chemistry of
the electron pair. Where homolytic chemistry deals with the neutral particles called free radicals,
heterolytic chemistry deals with positive and negative charges, with cations and anions. Homolytic
reations are typically carried out in the gas phase; or in solvents whose principal function is to provide
an inert medium in which the reacting molecules can move about. Heterolytic reactions are typically

GENERAL ORGANIC CHEMISTRY


76 QUIZRR
carried out in solution in polar solvents, because of the greater ease of separation of charges therein
and very often because of the stabilization of the resultant ion pairs through solvation.

X+ Y
ă
XăY
Carbanion
Carbocation

EL ECT ROPH I LES A N D N U CL EOPH I LES

Di s c u s s i o n :
Consider the reaction of hydroxide ion (a Bronsted base) with hydrogen chloride (a Bronsted acid).

ă ă
H O H Cl H O H + Cl

The oxygen of hydroxide ion bears a formal charge of ă 1. The hydrogen of hydrogen chloride
bears a + charge because chlorine is more electronegative than hydrogen and thus the H-Cl bonding
electron pair is unequally shared. We can envision the start of the acid-base reaction between hydroxide
ion and hydrogen chloride as an electrostatic attraction between the opposite charges. As the reaction
proceeds, the oxygen atom of hydroxide ion shares a lone electron pair with the hydrogen atom of
hydrogen chloride, as shown with the curved arrow in the reaction above. This simple reaction shares
one feature in common with the majority of reactions that you will encounter in your study of elementary
organic chemistry. One species in the reaction shares an electron pair (a Lewis base) with another
species (a Lewis acid) to make a new covalent bond. Application of this electron sharing idea to any
reaction gives you an excellent chance at starting to figure out the mechanism for most common organic
reactions.
Because of the ubiquity of electron pair shares and acceptors in organic reactions, we assign special
and distinct terms to these species. A molecule or ion that accepts a pair of electrons to make a new
covalent bond is called an electrophile (from the Greek for „electron loving‰). An electrophile is the
same thing as a Lewis acid. Any molecule, ion or atom that is electron deficient in some way can
behave as an electrophile. Electron deficiency would include a formal positive change (methyl carbocation),
a partial positive charge (+), usually in conjunction with a polar bond (such as H-Cl) or an open octet
(borane). „E‰ or „E+‰ are common abbreviations for generic electrophiles.
Typical electrophiles :

H H
+ ă
H C H B H Cl
H H
methyl carbocation borane hydrogen chloride
carbon has a formal boron has an hydrogen bears
positive charge open octet a + charge

GENERAL ORGANIC CHEMISTRY


QUIZRR 77

A molecule or ion that donates a pair of electrons to form a new covalent bond is called a nuclephile
(from the Greek „nucleus loving‰). A nucleophile is the same thing as a Lewis base. Any molecule, ion
or atom that has electrons that can be shared can be a nucleophile. The most common indications that
electrons are available to be shared are formal negative charge (iodide ion), a partial negative charge
(ă), usually in conjunction with a polar bond (methyl magnesium bromide), a  bond (isobutylene) or
lone pairs (ammonia). „Nuc‰ or „Nu‰ are common abbreviations for generic nucleophiles.
Typical nucleophiles :

ă ă + H3C H
I
H3C MgBr
iodide ion C CH2 H N
carbon bears a ă charge
iodine has a formal methyl magnesium
negative charge H3C H
bromide
isobutylene ammonia
C = C  bond nitrogen lone pair

Electrophiles (Lewis Acid)

Soft Acids Hard Acids


2
(Hg 2 , Fe2  , Ag  , etc.) (Fe3  , H , Li  , Na  , C etc.)
Metals of low oxidation state Metals of relatively higher oxidation state
They have low positive charge density They have high positive charge density

Nucleophiles (Lewis bases)

Soft bases Hard bases


       
(HSO3 , HS , Cl , Br , I  etc.) (NH2 , F , H , OH etc.)
less electronegative elements having More electronegative elements having
low negative charge density high negative charge density

The study of reaction mechanisms is central to the study of organic chemistry at any level.
Therefore identification of electrophiles and nucleophiles is a critical organic chemistry survival skill.
Examination of a structure for the features discussed above is one way to identify how a molecule or

GENERAL ORGANIC CHEMISTRY


78 QUIZRR
ion might behave in a reaction. Another way is by considering the curved arrows. Because electrons
flow from an electron source to a place of electron deficiency, a curved arrow points away from a
nucleophile and to an electrophile.
Elec trons a lwa ys flow from nucleophile to elec trophile : Nucleophile : Electrophile

Ex a m p l e

Decide if each molecule or ion shown below will react as a nucleophile or electrophie, or
both.
ă 
(a) Br (b) NH2 (c) H2O
Solution :
Examine each structure for the charge distribution and electronic features discussed above.
(a) Bromide ion : This atom has four lone pairs and a formal negative charge, suggesting it is
electron-rich and can therefore function as a nucleophile. It has none of the features that
would suggest it might behave as an electrophile.
(b) Ammonium ion : This ion has a formal positive charge, suggesting it is electron-poor and can
therefore function as an electrophile. It has no lone pairs or areas of negative charges,
suggesting it will not function as a nucleophile.
(c) Water : The oxygen atom of water has two lone pairs and a -charge (oxygen is more
electronegative than hydrogen). This suggests that water can behave an a nucleophile. Each
hydrogen atom bears a + charge, so the molecule can behave as an electrophile as well.
Many molecules can be both nucleophiles and electrophiles. How they behave depends upon
what they react with. For example, if water is reacted with an electrophile, the water will
behave as a nucleophile.

Ex a m p l e :
Identify the nucleophiles and electrophiles in each mehanism step shown below.

ă CH3 CH3 CH2 CHo


CH3 S CH3 CH2 ă
C C + I
(a) S CH3
I H H

H
H  ă
CH2 + Br
(b)
Br

H OH
ă
+ H2 O + Cl
(c)
Cl

GENERAL ORGANIC CHEMISTRY


QUIZRR 79

NO2
O H NO2 
OH2
(d) N + H3O

O

Decide if each molecule or ion shown below will react as a nucleophile or electrophile or both.

CH3 H
H3 C C C O
(e) (i)
CH3 H

H
ă H C Cl
(f) H O (j)
H

O
Cl H H
C
(g) Al Cl (k) C N
Cl H H
H

(h) H·C C·H

So l u t i o n :
(a) The curved arrow that starts at the sulfur lone pairs and ends at the carbon bearing the
iodine atom indicates that the sulfur is donating a pair of electrons to form a new S-C bond.
This means CH3S- (methanethiolate) is a nucleophile in this reaction. As there are only two
reactants, the other reactants must by default be a electrophile. We can verify this by noting
that the iodoalkane is accepting a pair of electrons from the sulfur atom to form a new
covalent bond. (This is an example of the SN2 reaction).

ă CHo CHo CH2 CHo


CH3 S CHaSCH2 ă
C C + I
l H H SCHo
nuclephile electrophile

(b) In the ionization of the bromoalkane to form a carbocation and bromide ion no new bonds are
formed. Therefore this ionization process does not involve a nucleophile or electrophile. The
reverse process, reaction of the carbocation with bromide ion to form a bromoalkane, does
result in a new bond. The curved arrow that starts at the bromide ion indicates a bromine
lone pair is shared with the carbon bearing the formal positive charge resulting in a new
GENERAL ORGANIC CHEMISTRY
80 QUIZRR
C-Br bond. Therefore bromide ion is a nucleophile. The carbocation is accepting an electron
pair to make a new C-Br bond, so it is an electrophile.

H
H  ă
CH2 Br
Br
not nucleophile electrophile nucleophile
not electrophile

(c) The hydroxide ion is sharing a lone pair with the hydrogen atom, resulting in a new O-H
bond. Therefore the hydroxide ion is a nucleophile. We might also notice that hydroxide ion
bears a formal negative charge, and that ions bearing a full negative charge are electron rich
and therefore rarely act as electron acceptors (i.e., electrophiles). Because there are only two
reactants and one of these is clearly the nucleophile, the chloroalkane must therefore be the
electrophile. (This is an example of the E2 reaction).

nucleophile
ă
H OH
ă
+ H2 O + Cl
Cl
electrophile

(d) In the first step of the reaction, a  bond of benzene (C6H6) is shared with the nitronium ion
is the electrophile. The product of this electrophilic attack on the benzene ring is called an
arenium ion. In the second step, a lone pair from a water molecule is used to make a new
O-H bond with a hydrogen that is plucked from the arenium ion. Because the water is
donating the lone pair, it is the nucleophile. This means the arenium ion is the electrophile
in the second step. (This is an example of electrophilic aromatic substitution.)

nucleophile

electrophile
NO2
O H NO2 
OH2
N + H3O

O
nucleophile electrophile

(e) Tert-butyl carbocation : The central carbon of this ion bears a formal positive charge and has an
open octet. Both of these features suggest this cation will behave primarily as an electrophile.

GENERAL ORGANIC CHEMISTRY


QUIZRR 81

(f) Hydroxide ion : The oxygen atom has three lone pairs and a formal negative charge. These
features suggest this anion will behave principally as a nucleophile.
(g) Aluminium chloride : The aluminium atom has an open octet, suggesting the molecule will
behave as an electrophile. Each chlorine has three lone pairs, suggesting a possibility for
nucleophilic properties as well. Neutral halogen atoms do not share electrons very often,
however, because it would give them a formal positive charge. Because of their high
electronegativity, halogen atoms do not tolerate a positive formal charge very well. The main
exception to this rule is when a halogen can share a lone pair with a neighboring carbocation
resulting in a structure in which all atoms have complete octets.
(h) Acetylene : We predict this alkyne will behave primarily as a nucleophile because it has
two  bonds to share. A C-H bond is almost nonpolar because of the small difference in
electronegativity of carbon and hydrogen. Thus the molecule has no obviously electrophilic
sites.
(i) Formaldehyde : The lone pairs on oxygen and the C-O  bond suggest this molecule might
behave an a nucleophile. Because carbon is less electronegative than oxygen, the C=O bond
is polarized with a + charge on carbon and a - charge on oxygen. This observation gives
another reason why the molecule is a nucleophile, and also suggest it can be an electrophile.
Thus formaldehyde could be a nucleophile or electroophile, depending upon what else was in
the reaction. This dual reactivity is a common feature of many more complex molecules.
Because this dual reactivity is due to the carbonyl group (C = O), any molecule containing
this feature is expected to behave as both a nucleophile and electrophile.
(j) Methyl chloride : The lone pairs on chlorine suggest this molecule could be a nucleophile
(see answer g above for more discussion). Carbon is less electronegative than chlorine, so the
C-Cl bond is polar, with a + charge on the carbon and a - charge on chlorine. That the
carbon bears a partial positive charge suggests that the carbon atom is electrophilic. Thus this
molecule could be a nucleophile (low probability) or electrophile.
(k) Acetamide : This molecule contains a carbonyl group (C=O). As mention in answer (i) above,
this means it can be both nucleophilic and electrophilic. The nitrogen lone pair is an addition
nucleophilic site.

I n d u c t iv e e f f e c t :
Inductive effect may be defined as the permanent displacement of electrons forming a covalent
bond towards the more electronegative element or group.
The inductive effect is represented by the symbol, the arrow pointing towards the more electronegative
element or group of elements. Thus in case of n-butyl chloride inductive effect may be represented as
below.

' ' ' ' '


CH3 CH2 CH2 CH2 Cl

GENERAL ORGANIC CHEMISTRY


82 QUIZRR
Any atom or group, if attracts electrons more srongly than hydrogen, it is said to have a -I effect
(electroattracting or electron-withdrawing), viz NO2, Cl, Br, I, F, COOH OHC3, etc. while if atom or
group attracts electrons less strongly than hydrogen it is said to have + I effect (electron repelling or
electron releasing) viz,
CH3, C2H5, Me2CH and Me3C groups. The important atoms or groups which cause negative or
positive

inductive effect are arranged below in the order of decreasing effect. -I (Electron-
attracting) groups :
+
N  CH 3 3  NO2  CN > F > COOH > Cl > Br > I > CF3  OH  OCH 3  C6 H 5  H

Always remember this order.


+ I (Electron-attracting) groups :
(CH3)3 C > (CH3)2CH > CH3CH2CH2 > CH3CH2 > CH3 > H

A c i d i c s t r e n g t h o f Ca r b o x y l ic a c i d
Compound which easily donates H+ to other are good acids. So carboxylic acids which relase H+
ion more easily are better acids.
Rule
Ć ăI effect increases acidic nature
Ć +I effect reduces acidic nature
For Ex.
Ć HCOOH > CH3 COOH > CH3 CH2 COOH
(formic acid)
Formic acid is strongest among the above 3 acids because of least +I effect.
Ć Now taking another example to show ă I effect over acidic nature of acids
Cl3 C COOH > Cl2 CHCOOH > Cl CH2 COOH > HCOOH > CH3COOH
More the Cl, more is the ăI effect & hence more the acidic nature.
Ć In the case of Cl, what if Cl atoms are attached for from COOH group.
Cl CH2 COOH > Cl CH2CH2COOH > Cl CH2 CH2 CH2 COOH
The closer the ăI group more is the acidic nature. This is also known as proximity effect.
Ć As ÂsÊ character increases, the acidic nature increases as electronegativity of atom increases
i.e. sp > sp2 > sp3
(50% S (66.31) (25%)
character)
CH  C ă CH2 ă COOH > CH2 = CHCH2 COOH > CH3CH2CH2COOH
The explanation follows from the above explanation.

GENERAL ORGANIC CHEMISTRY


QUIZRR 83

Ć More the electronegativity to an atom, more the acidic nature. We will explain this with the
help of various examples
HClO4 > HClO3 > HClO2 > HO Cl
(+7) (+5) (+3) (+1)
In this case the electronegativities of Cl are in decreasing order & hence the acidic nature.
Same reasoning applies to the following
HClO4 > H2SO4 > HNO3
(+7) (+6) (+5)
We will deal with these kind of examples later.

Re a c t iv i t y o f a l k y l h a l id e s
It can be deduced by reacting substance with Ag salt (AgNO3).

RX + AgNO3  AgX

fast
NaCl + AgNO3   AgCl (Ionic reactions)

Always keep in mind

More stable is the intermediate of a chemical reaction higher is the reactivity of the initial
compound.

CH3
|
CH3  C  Cl  CH3  CH  Cl  CH3  CH  Cl  C2 H5Cl  CH3 Cl
| | |
CH3 C2H5 CH3

CH3
|
Here CH3  C would be intermediate which is most stable among other intermediates.
|
CH3
 i.e. 3Ĉ > 2Ĉ > 1Ĉ > CH3 (If R groups are differen, reactivity order)
RI > RBr > RCI is the reactivity order of halogens.
On the basis of bond dissociation energy RI is maximum reactive due to largest size of I atom and
minimum bond dissociation energy.

RESON A NCE
Whenever a molecule can be represented by two or more structures that differ only in the arrangement
of atomic nuclei ă there is resonance. The molecule is hybrid of all these structures, and cannot be
represented satisfactory any one of them. Each of these structure is said to contribute to the hybrid.

GENERAL ORGANIC CHEMISTRY


84 QUIZRR
Localized Versus Delocalized Electrons

CH3 NH2 CH3 CH CH2

localized localized
electrons electrons



CH3C delocalized
electrons

Re s o n a n c e Co n t ri b u t o r s a n d t h e Re s o n a n c e H y b ri d
1 1
2 2

resonance contributor resonance contributor

resonance hybrid

Resonance contributors are imaginary, but the resonance hybrid is real.


The resonance hybrid is more stable than any of the contributing structures.

Ru l e s f o r Dr a w in g Re s o n a n c e Co n t ri b u t o rs
1. Only electrons move
2. Only  electrons and lone-pair electrons move
3. The total number of electrons in the molecule does not change
4. The numbers of paired and unpaired electrons do not change
The electrons can be moved in one of the following ways :
1. Move  electrons toward a positive charge or toward a  bond
2. Move lone pair electrons toward a  bond
3. Move a single nonbonding electron toward a  bond
Resonance contributors are obtained by moving  electrons toward a positive charge.
 
(a) CH3CH CH CHCH3 CH3CH CH CHCH3

+ +
CH3CH CH CHCH3
  
(b) CH3CH CH CH CH CH2 CH3CH CH CH CH CH2 CH3CH CH CH CH CH2

GENERAL ORGANIC CHEMISTRY


QUIZRR 85
+ + +
CH3CH CH CH CH CH2 resonance bybrid

 
CH2 CH2 CH2 CH2 CH2
 
(c)

+
CH2

+ +

+

Moving  electrons toward a  bond

resonance contributors

resonance hybrid

ă + + ă
CH2 CH CH CH2 H 2C CH CH CH2 CH2 CH CH CH2

CH2 CH CH CH2
resonance hybrid

Moving a nonbonding pair of electrons toward a  bond

ă ă ă
O O O O O
R C R C ă C ă C ă ă C
O O O O O O
NH2 NH2
+
resonance contributors resonance contributors

ă ă
O O
R C ă C ă
O O
NH2
+
resonance hybrid resonance hybrid

GENERAL ORGANIC CHEMISTRY


86 QUIZRR
Re s o n a n c e St r u c t u r e s f o r t h e A l l yl ic Ra d i c a l a n d f o r t h e B e n zy l Ra d i c a l

CH3 CH CH CH2 CH3 CH CH=CH2


resonance contributors
 
CH3 CH CH CH2
resonance hybrid

CH2 CH2 CH2 CH2 CH2

resonance contributors

CH2
 


resonance hybrid

Im po rt an t po int s
Ć Electrons move toward an sp2 carbon but never toward an sp3 carbon
Ć Electrons are neither added to nor removed from the molecule when resonance contributors
are drawn
Ć Radicals can also have delocalized electrons if the unpaired electron is on a carbon adjacent
to an sp2 atom
Ć Resonance contributors with separated charges are less stable

O Oă
O Oă
+
R C OH R C OH ă
R C O R C O

Electron always move toward the more electronegative atom

+O ă
O O
+
CH3C CH CH2 CH3C CH CH2 CH3C CH CH2
E F G
resonance contributor obtained resonance contributor obtained
by moving  electrons away by moving  electrons toward
from the more electronegative atom the more electronegative atom

GENERAL ORGANIC CHEMISTRY


QUIZRR 87

When there is only one way to move the electrons,


CH 2 CH OCH2 CH 2 CH OCH 2

movement of the electrons away from the more electronegative atom is better than no movement
at all becauase electron delocalization makes a molecule more stable.
Features that decrease the predicted stability of a contributing resonance structure...
1. An atom with an incomplete octet
2. A negative charge that is not on the most electronegative atom
3. A positive charge that is not on the most electropositive atom
4. Charge separation

Re s o n a n c e En e r g y
Ć A measure of the extra stability a compound gains from having delocalized electrons

+ H2 HĈ = ă28.6 kcal/mol (ă120kj/mol)


experimental
cyclohexene

+ 3 H2 HĈ = ă85.8 kcal/mol (ă339 kj/mol)


calculated
cyclohexene
hypothetical

+ 3 H2 HĈ = ă49.8 kcal/mol (ă208 kj/mol)


experimental
benzene

Benzene is stabilized by electron delocalization

Su m m a r y
Ć The greater the predicted stability of a resonance contributor, the more it contributes to the
resonance hybrid.
Ć The greater the number of relatively stable resonance contributors, the greater is the resonance
energy.
Ć The more nearly equivalent the resonance contributors, the greater is the resonance energy.

GENERAL ORGANIC CHEMISTRY


88 QUIZRR
Re s o n a n c e -St a b i li ze d Ca t io n s
+ +
CH2  CHCHR CH2  CHCH2

an allylic cation the allyl cation

+ +
CHR CH 2

a benzylic cation the benzyl cation

Relative Stability of Resonating structures


All the resonating structure donÊt contribute equally to the resonance hybrid. More the stability
of resonating structure, more will be its contribution.

Stability of resonating structure (in preference order)


(1) More the no. of covalent bond in a molecule, more will be its stability.

 
CH2 = CH ă CH = CH2  CH  CH  CH  CH
2 2

first structure is more stable than second.

(2) The resonating structure in which all the atoms have complete octet will be more stable than
the resonating structure in which all the atoms have not complete octet.

 ..  ..
CH2  O  CH3  CH2 =O  CH3

(a) (b)
(b) is more stable than (a), as octet of carbon is not complete in (a)
(3) The structure in which negative charge lies on electronegative element are more stable.

ă
O O O O O

(a) (b) (c) (d) (e)

(a) & (e) are most stable as ăve charge is present on oxygen, more electronegative than carbon.

GENERAL ORGANIC CHEMISTRY


QUIZRR 89

M e s o m e r i c Ef f e c t :
Special properties are associated with systems in which a  bond is conjugated either with a second
-bond (or) with an atom which possesses a pair of electrons in a p-orbital. These are : stabilization
energy, single bond lengths which are shorter than those in non-conjugated compounds, and (in some
cases) the modification of dipolar properties.

(a) -bond- bond conjugation : The simplest example is butadiene, CH2=CHăCH=CH2. This
is symmetrical molecule and conjugation does not lead to the appearance of a dipole. In
+ 
valence bond language, contributions from the ionic structures CH2  CH  CH  CH2 and
 
CH2  CH  CH  CH2 are necessarily equal and their dipoles therefore, nullify each other.
This is not true, however, when -bonds of different types are in conjugation. For example
in an ,  unsaturated carbonyl compound such as butenal the contribution of structure (A)
is greater than that of (B) because oxygen significantly more electronegative than carbon the
carbonyl group thus polarizes the C = C bond.



H3 C O H3 C O
(A) (B)

A group such as carbonyl which withdraws electrons from an adjacent group via the -
bonding frame work is described as having a ă M effect, the ăve sign indicating electron
withdrawal and M standing for mesomeric. Other groups of ă M type include ester, nitrile and
nitro etc., and the curved arrows in the representation denote the direction of the mesomeric
effect.

C C C
+
O N N O
O

(b) -bond-p-orbital conjugation : In vinyl chloride (chloroethene) the p-orbital on the


carbon which is attached to chlorine can overlap with both the p-orbital on the second
carbon atom and one of the filled p-orbitals on chlorine thus three delocalized MOÊs are
established of which the two of lowest energy are occupied. Since the p-orbital on chlorine
is initially filled, its participation in the delocalized  system leads to the partial removal of
electrons from chlorine and the appearance of a dipole moment directed from chlorine
towards carbon. This is opposed to the dipole establishment in C ă Cl  bond as a result
of the ăI effect of chlorine, with the overall result that the dipole moment of vinyl chloride

GENERAL ORGANIC CHEMISTRY


90 QUIZRR
(1.44D) is considerably smaller than that of ethyl chloride (2.0D) in which the ă I effect is
operative. The capacity of chlorine for donating electrons into a molecular -system is
described as a + M effect.

H
H
H H
C
H Cl H2C
ă +
Cl H2C Cl
(A) (B) (C)

In VB terminology, vinyl chloride is described as a hybrid of the structures (B) and (C), the latter
symbolizing the + M effect of chlorine. Both descriptions also indicate that CăCl bond should be shorter
than in a saturated alkyl chloride, as is found.
Other elements with unshared p-electrons which take part in forming delocalized -systems include
other halogens, oxygen and nitrogen, e.g.,

CH2 = CH = OCH2 CH 2 = CH ă N (CH3)2

In each case the substituent has a + M effect.


Two further points should be noted. First, bond strength is dependent on the extent of the overlap
of the combining atomic orbitals, so that in these conjugated systems the more nearly equal in size the
p-orbitals are, the more effective is the -orbital overlap. Hence fluorine is more effective than chlorine
in conjugating with carbon, and oxygen is more effective than sulphur. Secondly, as the nuclear charge
in an atom is increased, so also is the hold of the nucleus on the surrounding electrons so that, for
comparably sized atoms, the ability to conjugate decreases as the atomic number increases. Hence the
order of + M effect is
ă NR2 > ă OR > ă F

+ M effect possessing groups are :

.. .. .. .. .. .. ..
 OH,
..   OR
.. ,  NH 2 ,  NHR,  NR 2 ,  SR
.. ,  X
.. etc.

ă M effect possessing groups are :


ă CHO, = CO, ă CN, ă NO2, ă SO3H etc.

The low reactivity of halogens bonded to unsaturated carbon is due to the + M effect of the
halogen. The CăBr bond in vinyl bromide has a partial double-bond character due to the + M effect
of bromine with consequent low reactivity of bromine.


CH 2 = CH Br CH 2 CH = Br

GENERAL ORGANIC CHEMISTRY


QUIZRR 91

The acidity of phenol is due to the + M effect of OH group. The mesomeric transfer of the lone pair
on the oxygen atom of phenol to the  electrons of the benzene ring results in several resonance
structures with positive charge on the oxygen atom. This aids the hydrogen atom of OH group to leave
as proton.

  
OH OH OH OH O

H 2O
+ H3 O+

The ionization is specially aided due to the formation of the relatively more stable phenoxide ion.
The charge delocalization in phenoxide ion affords greater stability over phenol in which charge
separation occurs in the canonical forms.


O O O O

Hence, phenol prefers to ionize, i.e. it is acidic.

Relative Stabilities of Allylic and Benzylic Cations

+
CH2  CH C R + +
| > CH2 =CHCHR > CH2 =CH CH2
R

tertiary allylic cation secondary allylic cation allyl cation

+ + +
CR > CHR > CH2

tertiary benzylic cation secondary benzylic action benzyl cation

increasing stability

GENERAL ORGANIC CHEMISTRY


92 QUIZRR
Hyperc onjugat ion :
The alkyl groups with at least one hydrogen atom on the -carbon atom, attached to an unsaturated
carbon atom, are able to release electrons by a mechanism similar to that of the electronic effect.

H
H+
ăC ă C = C ăC = C ă C

Note that the delocalization involves s and p bond orbitals (or p orbitas in case of free radicals);
thus it is also known as s-p conjugation. This type of electron release due to the presence of the system
H-C-C=C is known as hyperconjugation

H H+ H H
| | |

H  C  CH  CH2  H  C  CH  CH 2  H C  CH  CH 2  H  C  CH  CH2
| | |
H H H H+

Ex a m p l e
Which is more stable

+
(a) CH3  CH2

+
(b) CH3  CH2  CH3

Solution :
(b) is more stable than (a) as (b) has 6  hydrogen while (a) has only 3.

Ex a m p l e
Ć
How many hyperconjugative structure of CH3ăCHăCH3 are possible ?
Solution :
As there are 6  hydrogen therefore 6 hyperconjugative structures are possible.

Ex a m p l e
CH3ăCH=CH2
How many hyperconjugative structure of given molecule possible ?
Solution :
H
H

H C CH = C
H
H
As 3  hydrogen are possible, therefore 3 structures are possible.
GENERAL ORGANIC CHEMISTRY
QUIZRR 93

Note :  hydrogen is hydrogen of C-H bond where C is attached through carbon with one positive
charge/through carbon with unpaired eă/through carbon forming multiple bonds.
Note : Stability due to different effect is generally in the order.
Resonance > hyperconjugation > Inductive effect
This will help in comparing the stability of structures.
Carbocation : All three effects are considered.
Radical : Resonance & hyperconjugation predicts the stability.
Carbanion : Resonance & inductive effects predicts stability.
Note : If in resonance, nonplanar orbitals or orbitals of different energy are involved, then
inductive effect will be more dominating than resonance.

Ex a m p l e
Compare stability of :

Cl
Cl

&

NO2
NO2

(a) (b)
Here inductive effect of Cl is dominating as C (2p) and Cl (3p) have orbitals of different energies.

Therefore (b) containing NO2 at meta position will be more stable.

Ex a m p l e :
Arrange in the order of increasing stability.

   
CH2 CH2 CH2 CH2

OCH3

CH3 O ă CH3 NO2

(a) (b) (c) (d)

GENERAL ORGANIC CHEMISTRY


94 QUIZRR
Solution : (c) < (d) < (a) < (b)


CH2

Writing resonant structure of

 
CH2 CH2 CH2 CH2


..
O CH3 stabiliser +ve charge on ortho position by resonance thus most stable.

NO2 withdraws ă ve charge, thus increasing +ve charge density decreasing its stability.
CH3 donates eă by inductive effect.
..
OCH3 at meta position increases + ve charge through inductive effect and resonance is not

possible at this position. Thus


(c) < (d) < (a) < (b)

El e c t r o m e r i c Ef f e c t
This is a temporary effect and takes place between two atoms joined by a multiple bond i.e. a
double or triple bond, in the presence of an attaching reagent.
It is temporary because the molecules acquire their original electronic condition once the attacking
reagent is removed.
It also occurs as +E and ăE effect, similar to inductive effect.
Ć ăE effect ă When the transfer of electrons take place away from attacking reagent.

ă ă
C O + CN C O

(away from agent) CN

GENERAL ORGANIC CHEMISTRY


QUIZRR 95

Ć +E effect : When the transfer takes place towards the attacking agent.

+ +
C C + H C C

(towards the agent)

Interesting case
CH3 H has 2 options
(1) H can go to C(1)
C C (2) H can go to C(2)
2 1 +
H
H H

For case 1
H
CH3 H CH3
+
C C C C H
+
H
H H H

ăE effect H
(2Ĉ carbocation)

For case 2

CH3 H CH3
+
C C H C CH2
+
H
H H

+E effect H
(1Ĉ carbocation)

Case 1 will be the result, because it produces 2Ĉ carbocation which is more stable.
Note : It is a misconception that adding a nucleophile will cause ăE effect & electrophile causes
+E effect. You can see from the above question that electrophile caused ă E effect.

GENERAL ORGANIC CHEMISTRY


96 QUIZRR
FREE RA DI CA L S
There are two possible structures for simple alkyl radicals. Either a free radical may be a planar
species (A) in which the carbon atom bearing the odd electron is sp2 hybridized (as in carbocations) and
the odd electron remains in the p-orbital, or it could have a shape resembling a shallow pyramid (B)
which is neither planar nor tetrahedral and the orbital containing unpaired electron is some sort of a
hybrid between a p and an sp3 orbital.

R' R" R R
fast
C or R' C C R'
R" R"
R
(A) (B)

The question as to whether free radicals exist in a planar configuration or in a rapid equilibrium
between the two pyramidal forms (B) is difficult to answer with certainty. However, available evidence
indicates that the unpaired electron is in a p orbital stabilized free radicals certainly exist in planar
configuration.
Formation : Free radicals are often produced when a molecule is supplied with sufficient energyă
thermal or photochemicalăto cause homolysis of a covalent bond. In addition oxidation-reduction reactions
involving the gain or loss of a single electron can also generate radicals. Some typical reactions producing
free radicals are given below :

(CH3)4 Pb   4 CH3 + Pb

CH3 ă N = N ă CH3   2CH3 + N2

C6H5CO ă O ă OCOC6H5   2C6H5COO  2C6H5 + 2CO2

h
CH3COCH3   CH3 + COCH3  CH3 + CO

O O
|| anode ||
R  C O    R  C O  R  CO2
e

Stability : In general, it has been found that the stability of alkyl radicals is in the order
tertiary > secondary > primary. The reason for the stabilizing influence of alkyl groups is not far to seek
if we consider that in going from methyl to t-butyl, there is increasing tendency for electron release due
to hyperconjugation. As we have seen that the carbon atom carrying an unpaired electron has half-
filled orbital that gives the radical its characteristic unstability; the hyperconjugative mechanism tends
to fill this orbital and thus stabilizes the radical to some extent. Various hyperconjugative contributing
forms of tert-butyl radical are given below :

GENERAL ORGANIC CHEMISTRY


QUIZRR 97

CH3 CH2 H CH3 CH3

C C C H C
H3 C CH3 H3 C CH3 H3C CH2 H CH3
CH2

Allyl and benzyl radicals are particularly stable because of resonance involving  electrons of the
double bond or aromatic ring.
. .
CH2  CH  CH2  CH2  CH  CH2

CH2 CH2 CH2

Another factor that is responsible for the increased stability of tertiary radicals may be steric.
Formation of a tert-butyl radical by the removal of a hydrogen atom from isobutane, (CH3)3 CH, is
favoured as the steric repulsion between the methyl groups is relieved to a certain extent by an increase
in bond angles from 109.5Ĉ to about 120Ĉ. Clearly this steric relief should be the greatest in the
formation of a tertiary radical.

St a b i li t y o r d e r o f f r e e r a d i c a l s
CH3 CH3

CH3 < CH3 · CH2 < CH3 C < CH3 C

H CH3

< CH2 = CH · CH2 CH2

The stability can be explained on the basis of hyperconjugation and inductive effect.

C A RB OC A T I ON S
Introduction
A carbocation is molecule in which a carbon atom bears three bonds and a positive charge.
Carbocations are generally unstable because they do not have eight electrons to satisfy the octet
rule.

GENERAL ORGANIC CHEMISTRY


98 QUIZRR
H

H C +
open octet on carbon

Ca r b o c a t i o n Cl a s s i f i c a t i o n
In order to understand carbocations, we need to learn some basic carbocation momenclature. A
primary carbocation is one in which there is one carbon group attached to the carbon bearing the
positive charge. A secondary carbocation is one in which there are two carbons attached to the carbon
bearing the positive charge. Likewise, a tertiary carbocation is one in which there are three carbons
attached to the carbon bearing the positive charge.

H H
Methyl carbocations H C CH3O C
+
no C-C bonds
H H

H H
Primary (1o) carbocations CH3 C C
one C-C+ bond H H

H 
Secondary (2o) carbocations CH3 C
two C-C+ bonds CH3

CH3

Tertiary (3o) carbocations CH3 C 

three C-C+ bonds CH3

If the carbon bearing the positive charge is immediately adjacent to a carbon-carbon double bond,
the carbocation is termed an allylic carbocation. The simplest case is called the allyl carbocation.

R  R 
H2C CH2
R R

Generic allylic carbocation The allyl carbocation

GENERAL ORGANIC CHEMISTRY


QUIZRR 99

If the carbon bearing the positive charge is immediately adjacent to a benzene ring, the carbocation
is termed a benzylic carbocation. The simplest case is called the benzyl carbocation.

R H

 

R H
Generic benzylic carbocation The benzyl carbocation

If the carbon bearing the positive charge is part of an alkene, the carbocation is termed a vinylic
carbocation. The simplest case is called the vinyl carbocation. Note that the carbon bearing the
positive charge has two attachments and thus adopts sp hybridization and linear geometry.

R H
 
C C R C C H
R H
vinylic carbocation vinyl carbocation

If the carbon bearing the positive charge is part of a benzene ring, the carbocation is termed an
aryl carbocation. The simplest case is called the phenyl carbocation.

R R

R  

R R

Generic phenylic carbocation The phenyl carbocation

Ca r b o c a t i o n St a b i l i t y
The stability of carbocations is dependent on a few factors. The first factor of look at when deciding
the stability of a carbocation is resonance. Resonance is a stabilizing feature to a carbocation because
it delocalizes the positive charge and creates additional bonding between adjacent atoms. Dereasing the
electron deficiency increases the stability.

GENERAL ORGANIC CHEMISTRY


100 QUIZRR
Consider the following :
H H H

CH3 C CH3O C CH3O C

H H H
No resonance Resonance
The structure on the left does not have any resonance contributors in which electrons are donated
to the carbon with the open octet. Compare this with the carbocation that has resonance and a
delocalized positive charge. Charge delocalization imparts stability, so the structure with resonance is
lower in energy.
In the example shown above, an oxygen atom lone pair is involved in resonance that stabilizes a
carbocation. In general, any adjacent lone pair or  bond can also be involved in resonance delocalization
of a carbocation positive charge. Allylic and benzylic carbocations enjoy resonance stabilization by
delocalization of the positive charge to the adjacent  bond(s). Vinylic and aryl carbocations do not enjoy
resonance stabilization because their  electron clouds are perpendicular to the vacant p orbital of the
carbocation. (Recall that resonance requires the interacting orbitals to be parallel so they can overlap.
Without overlap there can be no resonance.)
Note the influence of inductive effect versus resonance on the energies of these molecules. The
oxygen atom that is bonded to the carbocation on the right is more electronegative than the corresponding
hydrogen atom in the left-hand structure. We would think that the inductive effect would pull electron
density away from the carbocation, making it higher in energy. In actuality, resonance usually
(but not always) outweighs other factors. In this case, carbocation stabilization by resonance
electron-donation is a more significant factor than carbocation destabilization by inductive electron
withdrawal.
Methyl and primary carbocations without resonance are very unstable, and should never be
involved in a reaction mechanism unless no other pathway is possible. More stable carbocations (secondary
or tertiary with resonance, or any carbocation with resonance) is sufficiently stable to be formed in a
mechanism under reasonable conditions.

Ex e r c i s e
Draw all significant resonance contributors for the following carbocations.


(a) (c) CH2

 OCH3 
(b) (d)

GENERAL ORGANIC CHEMISTRY


QUIZRR 101

The second factor that should be considered when thinking about carbocation stability is the
number of carbons attached to the carbon carrying the positive charge. We look at the number of
bonding electrons that are attached to the carbocation because those bonding electrons will help in
alleviating the positive charge. Bonding electrons from adjacent  bonds may overlap with the unoccupied
p orbital of the carbocation.
Hyperconjugative overlap

Adjacent H Empty pz orbital


C-H bond of carbocation
R R

R R
This phenomenon is termed hyperconjugation. Since the overlap supplies electron density to the
electron-deficient carbocation carbon, we predict that increasing the number of hyperconjugative
interactions increases carbocation stability or increasing the number of bonds adjacent to the carbocation
by increasing the number of alkyl groups attached to the carbocation carbon results in an increase in
carbocation stability. For example, a tertiary carbocation should be more stable than a secondary
carbocation.
Our simple prediction suggests that any adjacent bonding electron pair will participate in carbocation
hyperconjugation. However, only C-H and C-C bonds provide a significant level of increased stability.
When considering the importance of hyperconjugation versus resonance as the more important
stabilizing feature, resonance usually wins out. For example, a primary carbocation with resonance is
more stable than a secondary carbocation without resonance. A secondary carbocation with resonance
is usually more stable than a tertiary carbocation without resonance.
The general rules for carbocation stability can be summarized as follows :
(a) Increases substitution increases stability
+ o + o o
CH3 (methyl; least stable) < RCH2 (1 ) < R2CH (2 ) < R3C+ (3 ; most stable)
(b) Resonance is more important than substitution. For example, a secondary carbocation without
resonance is generally less stable than a primary carbocation with resonance.

Ex e r c i s e
Rank the relative stability of the three carbocations in each set.

+ 
H2 C  C  CH2 , H3C  C  CH3 
(a) | | and  CH3  C
3
H H

 
(b)

GENERAL ORGANIC CHEMISTRY


102 QUIZRR
In vinylic carbocations, the positive charge is assigned to a carbon with sp hybridization. How does
this influence the carbocationÊs stability? An sp orbital has more s character than an sp2 orbital.
Electrons in an s orbital are closer to the nucleus and therefore more tightly held than electrons in a
p orbital. This can be taken to mean that the electronegativity of carbon increases with increasing s
character. Thus : sp carbon (most s character, most electronegative) > sp2 > sp3 (least s character; least
electronegative). Electronegativity is a measure of electron attraction. So the stability of a cation is
influenced by the electronegativity of the atom bearing the positive charge. The more electronegative
the atom the less stable the cation. A vinylic carbocation carries the positive charge on an sp carbon,
which is more electronegative than an sp2 carbon of an alkyl carbocation. Therefore a primary vinylic
carbocation is less stable than a primary alkyl carbocation.
Similar reasoning explains why an aryl carbocation is less stable than a typical secondary alkyl
carbocation such as cyclohexyl carbocation.
Because of their reduced stability; vinyl and aryl carbocations are not often encountered.
The stability order of carbocations is as follows :

+ + + +
R C CH2 CH2 = CH · CH2 CH

R +
1Ĉ CH3

Ca r b o c a t i o n Fo r m a t i o n
Even though carbocation can be found in many organic reaction mechanisms, most carbocations
are formed by one of only two basic mechanism steps, ionization of a carbonăleaving group bond or
electrophilic addition to a  bond.
Ionization of a Carbon-Leaving Group Bond.
When a bond between a carbon atom and a leaving group ionizes, the leaving group accepts the
pair of electrons that used to be shared in the covalent bond. This may leave the carbon atom with an
open octet, resulting in a carbocation. The ionization is indicated with a curved arrow starting at the
bond and pointing to the leaving group atom that accepts the electron pair. Better leaving groups or
formation of a more stable carbocation result in lower activation energy and faster ionization. Carbonă
leaving group bonding ionization is illustrated using an oxonium ion.
 
(CH3)CăOH2 (CH3)3 C + OH2

Carbocation formation by ionization of a leaving group occurs in many organic reactions such as
the SN and E1 mechanisms which we will cover in later chapters.
1

GENERAL ORGANIC CHEMISTRY


QUIZRR 103

Electrophilic Addition to a  bond.


When an electrophile attacks a  bond, the  electron pair may form a new  bond to the electron-
deficient atom of the electrophile. (Not all additions to  bonds involve electrophiles or carbocations.)
This addition is indicated with a curved arrow starting at the  bond and ending at the electron
deficient atom of the electrophile. More powerful electrophiles or the formation of more stable carbocations
result in lower activation energy and faster addition. Electrophilic addition to a  bond is illustrated by
the reaction of HBr (an electrophie) wit styrene (PhCH=CH2). Note that the more stable carbocation
(secondary with resonance) is formed. This is a key mechanistic feature of MarkovnikovÊs Rule.

H Br

+ Br

Electrophilic addition to a  bond occurs in many reactions of alkenes, alkynes and benzene rings.
Note every addition reaction forms a carbocation, for example, catalytic hydrogenation or ozonolysis.

Pr o p e r t i e s o f Ca r b o c a t io n s
(i) Rearrangement
The bonding electrons of a carbocation may shift between adjacent atoms to form a more stable
carbocation. For example rearrangement will occur if a secondary carbocation can be formed from
a primary carbocation because a secondary carbocation is more stable than a primary carbocation
for e.g.
+ hydride +
CH3 CH CH2 CH3 CH CH3
shift
H more stable
2Ĉ carbocation
1Ĉ carbocation

CH3 CH3
+ methyl
CH3 C CH2 CH3 C CH2
shift +

CH3 CH3
more stable
3Ĉ carbocation
Phenyl shifts are preferable as they increase the stability by resonance effect
The migration aptitude of various groups is
(a) H ă > 3Ĉ > 2Ĉ > 1Ĉ alkyl > CH3
(b) Phă > CH3

GENERAL ORGANIC CHEMISTRY


104 QUIZRR
However a migration in reverse direction can also take place if the migration leads to formation
of benzylic compound which is highly stable.

H CH3

+ hydride +
Ph CH C CH3 Ph CH CH
shift

CH3 CH3
3Ĉ carbocation 2Ĉ carbocation
(benzylic form)

The result is highly stable due to delocalization of electrons due to resonance.


Another such example is

CH3 CH3 CH3

hydride +
CH3 C C CH3 CH3 C C CH3
+ shift

OH OH CH3
(3Ĉ carbocation)
resonating
structures

CH3

CH3 C C CH3

+ OH CH3
1. SPECIAL CASES :
Given, CH3

+ now here there 2 options, methide shift


Ph C CH2
& the other being phenyl shift

Ph

GENERAL ORGANIC CHEMISTRY


QUIZRR 105

CH3 CH3

+ yl +
meth
Ph C CH2 Ph C CH2
shift

Ph Ph (A)

phenyl shift

CH3

Here both are benzylic compounds


and are almost equally stable.
(B) Ph C CH2
+

Ph
So for major product in such cases, we go by migratory aptitude, and phenyl shift is preferable over
methide shift. So compound B will be major product.

2. Ring expansion
Rearrangement may lead to a change in ring size or formation of a ring causing greater stability
for example

+ intermediate + +
CH2 CH2
form
ă

+ +
or

(2Ĉ carbocation)

The ring stability increases as number of carbon atoms in the ring increases.

3<4<5<6

increasing order of stability

GENERAL ORGANIC CHEMISTRY


106 QUIZRR
Generally ring carrying more than 6 carbon atoms are less stable.

+ +

(1Ĉ carbocation) (2Ĉ carbocation


+ ring expansion)

+
+
methyl
shift

(3Ĉ carbocation)

(3)
+
+
H

OH

lone pair at Oxygen will react with hydrogen to complete its octet & leave the compound giving
us a 2Ĉ carbocation.
Now compound (A) has 2 option
+

ring expansion (favourable)


l
hy
et
m ift
sh

+
+

or

(3Ĉ carbocation)

GENERAL ORGANIC CHEMISTRY


QUIZRR 107

deprotonation
deprotonation

(B) (C) (D) (E) (F)


(major) (major)

(c) & (f) are the major products as more substituted alkenes are more stable.

(4) Deprotonation
Carbocations are very reactive species and are very strongly driven to dispose of the positive
charge. Even a weak base such a water can accomplish this deprotonation.

CH3 CH3

H2O
H3C C CH2 H3C C CH2
+

H H OH2

Vinylic carbocations generally do not rearrange, even if they can become more stable. For example,
the rearrangement shown below does not occur, even though a secondary carbocation would rearrange
to become more stable allylic carbocation (primary with resonance).

H H
 
H2C C CH2 X H2C C CH2

This resistance to rearrangement is probably due to orbital alignment restrictions during the
rearrangement transition state.

GENERAL ORGANIC CHEMISTRY


108 QUIZRR
Ex a m p l e
Provide a carbocation to complete each reaction. Draw the curved arrows.


H2O OH2
(a) ???

(b) H2O 
??? + H3O

1,2-alkyl shift
(c) ???

Solution :


OH2
OH2
(a)

H OH2

(b) + H3O

 1,2-alkyl shift
(c)

Ex a m p l e
Illustrate the three carbocation fates using any molecules you want. Use curved arrows and give the
products.
Solution :
Any carbocation and any other reactants are acceptable as long as the carbocation fates are
accurately illustrated.
Capture nucleophile :


H H O CH3

C CH2 + HOCH3 C CH2

H H

GENERAL ORGANIC CHEMISTRY


QUIZRR 109

Lose proton, form  bond :


H H
 
C CH2 C CH2 + H2OCH3

H HOCH3

Rearrange :
H H

C CH2 C CH3

H

CARBANIONS
Carbanions are units that contain a negative charge on a carbon atom. The negative charge gives
good nucleophilic properties to the unit that can be used in the formation of new carbon bonds.
Carbanions thus act as nucleophiles in substitution reactions, in carbonyl addition and substitution
reactions, and in 1, 4-addition (Michael) reactions.


R  C R
|
R
a carbanion

Carbanions may bear substituents that can affect the structure and reactivity of the carbanion,
and can affect the acidity of a parent C-H precursor. Halogens stabilize carbanions in the order of
Br > Cl > F. A prominent - repulsion between the F and Carbanionic centre causes some destabilization
in apha-fluorinated carbanions. The magnitude of the destabilization depends on the carbanion structure.
The destabilization maximizes as the carbanion structure approaches a planar configuration. Thus,
fluorinated carbanions possess pyramidal structures with high barriers to inversion.

Ca r b a n i o n St r u c t u r e
Carbanions are trivalent with sp3 hybridization. The lone pare of electrons occupies one of the sp3
orbitals. The geometry is thus tetrahedral. The tetrahedron can undergo inversion or retain its
stereochemistry depending on the attached substituents. A methy carbanions has a barrier to inversion
of about 2 kcal/mole.

R R
inversion
C R
C R
R R

tetrahedral carbanion

GENERAL ORGANIC CHEMISTRY


110 QUIZRR
Fo r m a t i o n :
As there is little difference between the electro negativities of carbon and hydrogen (2.5 and 2.1,
respectively) the polarity of the C-H bond is very small. The heterolytic fission of this covalent bond
to form an anion and a proton should then be a very difficult process. In other words, a hydrogen atom
bound to an sp3 carbon atom shows negligibe acidity. However, the presence of electron attracting
substituents such as nitro, cyano or carbonyl groups on the same carbon renders the hydrogen relatively
acidic. The increase in acidity is not only due to the electron-withdrawing ability of these substituents,
but also due to their ability to delocalize the negative charge of the anion. Thus hydrogens on the
carbon atom alpha to nitro, cyano or carbonyl groups have acidic character and can be removed as
protons leaving resonance stabilized anions.

ă
O O O
+ ăH
ă + +
H3C N H2C N H2C N
ă ă ă
O O O
ă ă
ăH
CH3 CN CH2 CN CH2 = C = N

ă
O O O
ă
ăH H2C C H
H3C C H H2C C H

Carboanions are also formed when a nucleophie adds to a carbon-carbon double bond.
ă
C2H5OCH2 CH CN
ă
C2 H5 O + CH2 = CH CN
ă
C2H5OCH2 C= C= N

H
St a b i l i t y :
As we have already seen, carboanions are stabilized by electron-withdrawing substituents. Generally
speaking, a carbanion is stabilized by resonance if a double bond is located  to to the anionic carbon.
This explains the stability of the allyic and benzylic carbanion.

R CH = CH CH2 R CH CH = CH2

CH2 CH2 CH2 CH2

GENERAL ORGANIC CHEMISTRY


QUIZRR 111

Ready conversion of triphenylmethane to the triphenlymethyl carbanion can be similarly explained.

Na
 C6 H5 3 CH   C6 H5 3 C 
: Na 

(C6 H5 )2 C (C6 H5 )2 C etc

The order of stability of the simple carbanion is :


methyl > primary > secondary > tertiary
Anot her fact or t hat cont r ibut es t o t he st abilit y of car banions is t he extent of s-character of the
bonding orbitals of the carboanionic carbon. Carbanion stability, thus, has been found to be in the
order : RC  CHă > R3C ă CH2ă. We have already seen that the carbon atom in acetylene is sp
hybridized (50% s character) and hence hydrogen atom attached to it should be more acidic than that
of ethylene where the carbon is sp2 hybridized (33% s character).
An interesting example is cyclopentadiene which readiy loses a proton to form the cyclolpentadienyl
anion.

CH2 Oă

H H H H

The unusual stability of cyclopentadienyl anion is explained by the fact that although it has only
four  electorns, two more electrons become available to it by the heterolysis of the C-H bond thus
forming a system of six electrons. These electrons are spread over all the five carbon atoms like the
delocalized aromatic system of benzene ring, thus conferring on it stability so characteristic of the
aromatic compounds.

Re a r r a n g e m e n t
Rearrangement does not takes place in case of carboanions.
It is due to poor migrating ability and poor leaving tendency of carboanions. Further more only
primary carboanions are stable
Exception
Ph Ph

ă
Ph C CH2 Ph C CH3
ă

(resonance stabalized)
H

Carboanion as a nucleophile will be discussed in details later on.

GENERAL ORGANIC CHEMISTRY


112 QUIZRR
C A RB EN E
A reaction in which both the groups or atoms are lost from the same carbon is called -elimination
reaction and results in the formation of carbenes.
Carbenes can be defined as neutral, divalent carbon intermediates in which a carbon is covalently
bonded to two atoms and has two non-bonded orbitals containing two electrons between them. There
are two kinds of carbenes, singlet and triplet. In the singlet state, a carbon atom is presumed to be
sp2 hybridized. Two of the three sp2 hybrid orbitals are utilized in forming two covalent bonds whereas
the third hybrid orbital contains the unshared pair of electrons. The remaining p-orbital remains
vacant. Thus, singlet carbene resembles a carbonium ion very closely. On the other hand, carbon atom
of a triplet carbene is sp hybridized and it is a linear species. These two hybrid orbitals are involved
in the bond formation with two groups and the remaining two electrons are placed, one each, in the
equivalent, mutually perpendicular py and pz orbitals. These electrons have parallel spins and a
carbene with this structure is said to be in a triplet state.

R
R

C C

R R
(singlet carbene) (triplet carbene)

At first sight, it appears that a singlet carbene has lower energy as the unshared electron pair is
in a sp2 hybrid orbital but the consideration of the electron repulsion energy that must be overcome
to pair two electrons in a single orbital places it at a higher energy level than a triplet structure. Thus,
the triplet state of a carbene is more stable than the singlet state. Triplet carbenes are not linear but
have a bond angle of 140Ĉ to 150Ĉ.
Among dihalocarbenes, the order of stability is
  CCl
Singlet carbenes : CF  
 CBr 
 CI
2 2 2 2
  CCl
  
Triplet carbenes : CF2 2  CBr2  CI2

Carbenes

CH2 CH2
triplet singlet
sp sp2
(only this one acts as a lone pair)

GENERAL ORGANIC CHEMISTRY


QUIZRR 113

Generation
1. From chloroform

Cl Cl

ă
OH ă
H C Cl ă C Cl + H2O
Or B

Cl Cl
(chloro form)
ă
CCl2 + H2O + Cl
(carbene)
2. From diazo-methane

h
CH2N2 CH2 + N2

(carbene)

Though the method is

+
CH2 N N CH2 + N ·
·N

Here N will break bond with C as it cant be pentavalent.


3. From Ketenes (Keto + alkene)

 ă +
CH2 C O CH2 + C ·
·O

4. By removal of CO2

O
ă
Cl3 C C  ă ă Cl
ă CCl3 CCl2
CO2
(carbene)

GENERAL ORGANIC CHEMISTRY


114 QUIZRR
Generally in such kind of reactions CO2 is liberated as gas, as it is stabalized due to resonance.
another example, +
N· N
O O

·
·

·
·

R · C · CH ă ă
R · C · CH
N2 CO2
ă (acyl carbene)
C·O

·
·
O
Re a c t io n s o f c a r b e n e
1. Addition :
The addition of carbenes lead to formation of cyclo compounds. Also it can be used to distinguish
singlet carbene with triplet carbene.
Generally
CH2
C C C C

CH2
Now let us see the stereochemistry behind this and how it is useful to distinguish singlet & triplet
carbene.
CH3 CH3 CH3 CH3

C C C C
CH2

H H H H
(cis product)
CH2
(assuming that this
(but sometimes due to is a triplet carbene)
collision, spin gets reversed

CH3 CH3 CH3 CH3

C C C C

H H H H
CH2 CH2

now rotation along C ăC bond will take


place, as a bond can only be formed
between e ă of opposite spins.

GENERAL ORGANIC CHEMISTRY


QUIZRR 115

CH3 CH3 CH3 CH3

C C C C

H CH2 H H3 C CH2 H

cis-1-2 dimethyl cyclopropane

CH3 CH3

C C

H3C CH2 H
trans-1-2-dimethyl cyclopropane

So with triplet we get 2 answers in the form of cis & trans forms
whereas,
in case of singlet we get only one answer for cis (we get cis) and trans (we get trans)
CH3 CH3 CH3 CH3
CH2
C C C C

H H H H
(cis )
CH2

CH3 CH3

C C

H CH2 H
2. Insertion (cis )
Carbenes can be added / inserted into a CăH bond, to increase the length of carbon chain for
example H
·

·C·H + CH2 ·C·C·H


·

H
CH2
CH2 · CH · CH3 Here 2 products are produced
·

H H
GENERAL ORGANIC CHEMISTRY
116 QUIZRR

1. CH2 · CH2 · CH3 CH2 · CH2 · CH3


·

·
H CH2
CH2

·
H

2. CH2 · CH · CH3 CH2 · CH · CH3


·

·
H H H CH2
CH2

·
H

3. Rearrangement
Alkylated and phenylated carbenes also undergo ready rearrangement to gives alkenes

R · CH
R can be alkyl, acyl, alkenyl
Note : Rearrangement in carbenes is similar to that of carbocations.

1. CH3 · CH · CH CH3 · CH ·
· CH2
·

This can have 2 products


2.
CH a. A double bond with cyclopropane
b. Ring Expansion

(a) CH CH

(b) CH
Or

GENERAL ORGANIC CHEMISTRY


QUIZRR 117

Sp e c i a l Ca s e :

H H

OH ă
+ CHCl 3
 here OH is a base which abstract H from the ring
to form carbanion and CHCl3 (chloroform) produces
carbene (:CCl2)

ă ă

Cl now since Here is a lp (lone


CCl2
C pair) it will form a double bond
Cl & simutaneous spring
ă expansion will take place as
Cl being more electronegative
can pull the electrons
Cl
C
Cl Cl

N I T REN ES
Nitrenes are electron-deficient monovalent nitrogen species in which the nitrogen atom has a
sextet of electron in its outer shell. There are two possible forms in which a nitrene can exist, the singlet
and triplet states.

R N R N
singlet triplet

Ge n e r a t i o n
1. Such compounds are formed by the elimination of a proton and a leaving group.

H
·

Br2
R · C · NH2 R · C · NH · Br
ăHBr
·
·

O O ă

R·C·N R · C · NH · Br
·
·

·
·

O O
(acyl nitrene)

GENERAL ORGANIC CHEMISTRY


118 QUIZRR
2. From isocynate

/
R·N· ·O
·C· R · N + CO
h

+ ă
·N·
·N 
R·N· R · N + N2

Nitrenes have properties like carbenes and undergo reactions like that of carbenes.
a case of rearrangement in nitrenes

+ ă
R ·C·N C·N ·C·
R·N· ·O
·
·

·
·

·
O O R (allcyl isocynate)

This reaction is also useful in generation of isocynates

ARYN ES
Arynes may be defined as aromatic compounds containing a formal carbon-carbon triple bond. The
best known aryne is benzyne which may be regarded as the aromatic counterpart of acetylene or in
other words, it is benzene minus two ortho hydrogens and can also be called as dehydrobenzene.

The benzyne bond is not like the triple bond of acetylene where the two carbons form a  bond
using sp orbitals and the remaining p orbitals are used to form  bonds. Such a structure is not possible
in benzyne because of the hexagonal geometry associated with the benzene ring. Most probably the
new bond of benzyne is formed by the overlap of sp2 orbitals to two neighbouring carbon atoms. These
sp2 orbitals are orthogonal to the  moleculr orbital of the benzene ring.

B e n zy n e
These intermediates are generated by the elimination of -proton and a leaving group (generally
halogen) from the adjacent carbons from a benzene ring system. For example,

Cl ă Cl
NH2 ăCl
ăNH3
ă
H Benzyne

GENERAL ORGANIC CHEMISTRY


QUIZRR 119
+
N2 Nă
ăN
NaNO2 /HCl decomposes

ăN + O ă C ă O +
0ă5Ĉ C ă
CO2 H C·O

·
O

Br MgBr
Mg (1 equiv.)
MgBrCl +
THF
Cl Cl

Benzynes are very reactive. Neither benzyne nor any other aryne (substituted benzyne) has yet
been isolated under ordinary conditions. The triple bond in benzynes is not identical with the formal
triple bond of alkynes because here the two -bonds are formed by the overlapping of PzăPz and
sp2 ă sp2 orbitals whereas in alkynes, they are formed by the overlap of Pz ă Pz and PyăPy orbitals.
It should be noted that the -bond (resulted from sp2 ă sp2 overlap) is a very weak bond, which
can be easily ruptured by the attack of a nucleophile (they cannot be attacked by electrophiles), thus
making benzynes very reactive. Another feature to be noted is that the extra pair of electrons does not
affect the aromaticity of benzene nucleus. The original sextet still functions as a closed ring and the
two additional electrons are merely located in a -orbital that covers only two carbons. In a reaction
where benzyne is formed, if the solution does not have sufficient nucleophile, they undergo dimerization
to give biphenylene.

Dimerization

Biphenylene

Stability : The new bond of benzyne, formed by the overlap of sp2 orbitals belonging to two
neighbouring carbon atoms in unstable, and therefore benzynes are extremely reactive chemical
species.

AROM ATI CI TY
Many cyclic, conjugated compounds possess markedly different physical and chemical properties
from those expected by comparison of their structures with acylic analogues. The simplest example is
benzene, which may be regarded as the parent compound of the aromatic series. Benzene is a planar,
cyclic compound with a cyclic cloud of delocalized  electrons above and below the plane of the ring.
Because its  electrons are delocalized, all the carbon-carbon bonds have the same length-partway
between the length of a typical single and a typical double bond. We also saw that benzene is a
particularly stable compound because it has an un-usually large resonance energy (36 kcal/mol or 15

GENERAL ORGANIC CHEMISTRY


120 QUIZRR
kJ/mol). Most compounds with delocalized electrons have much smaller resonance energies. Compounds
such as benzene with unusually large resonance energies are called aromatic compounds.

(a) (b) (c)

(a) Each carbon of benzene has a p orbital. (b) The overlap of the p orbitals forms a
cloud of  electrons above and below the plane of the benzene ring. (c) the electrostatic
potential map for benzene shows that all the carbon-carbon bonds have the same electron
density.
For a compound to be classified as aromatic, it must fulfill both of the following criteria :
1. It must have an uninterrupted cyclic cloud  electrons above and below the plane of the
moleculle (often called a  cloud). LetÊs look at what this means a little more closely.
For the  cloud to be cyclic, the molecule must be cyclic.
For the  cloud to be uninterrupted, every atom in the ring must have a p orbital.
For the  cloud to form, each p orbital must be able to overlap with the p orbitals on either
side of it. Therefore, the molecule must be planar.
2. The  cloud must contain an odd number of pairs of  electrons.
Benzene is an aromatic compound because it is cyclic and planar, every carbon in the ring has a
p orbital, and the  cloud contains three pairs of  electrons.
The German chemist Erich Huckel was the first to recognize that an aromatic compound must
have an odd number of pairs of  electrons. In 1931 he described this requirement by what was has
come to be known as HuckelÊs rule, or the 4n + 2 rule. The rule states that for a planar, cyclic compound
to be aromatic, its uninterrupted  coud must containt (4n + 2)  electrons, where n is any whole
number. According to HuckelÊs rule, then, an aromatic compound must have 2 (n =0), (n = 1), 10
(n = 2), 14 (n = 3), 18 (n = 4), etc., electrons. Because there are two electrons in a pair, HuckelÊs rule
requires that an aromatic compound must have 1, 3, 5, 7, 9, etc. pairs of electron. Thus, HuckelÊs rule
is just a mathematical way of saying that an aromatic compound must have an odd number of pairs
of  electrons.
Cyclobutadiene has two pairs of  electrons and cyclooctatetraene has four pairs of  electrons.
Therefore, these compounds are not aromatic because they have an even number of pairs of electrons.
There is an additional reason why cyclooctatraene is not aromaticăit is not planar, it is tub-shaped. We
saw that for an eight-membered ring to be planar, it must have bond angles of 135Ĉ, nd we know that
sp2 carbons have 120Ĉ bond angles. Therefore, if cyclooctatetraene were planar, it would have considerable

GENERAL ORGANIC CHEMISTRY


QUIZRR 121

angle strain. Because cyclobutadiene and cyclooctatetrene are not aromatic, they do not have the
unusual stability of aromatic compounds.

cyclobutadience benzene cycloctatetraene


[4]-annulene [6]-annulene [8]-annulene

Monocyclic hydrocarbons with alternating single and double bonds are called an-nulenes. A prefix
in brackets denotes the number of carbons in the ring.
Which of the following three-membered ring structures is aromatic? Cyclopropene is not aromatic
because it does not have an uninterrupted ring of p orbtial-bearing atoms. One of its ring atoms is sp3
hybridized, and only sp2 and sp hybridized carbons have p orbitals. Therefore, cyclopropene does not
fulfill the first criterion for aromaticity.

+ ă
cyclopropene cyclopropenyl cyclopropenly
cation anion

The cyclopropenyl cation is aromatic because it does have an uninterrupted ring of p orbital-
bearing atoms and the cloud contains one pair of delocalized electrons. The cyclopropenyl anion is
not aromatic because its  clouds has two (an even number) pairs of  electrons.
+ +

+
resonance contributors for the cyclopropenly cation

+ +

+
resonance hybrid

Cycloheptariene is not aromatic. Although it has the correct number of pairs of electrons to be
aromatic (three pairs), it does not have an uninterrupted ring of p-orbital-bearing atoms because one
of the ring atoms is sp3 hybridized. Cyclopentadiene is also not aromatic. It has an even number of
pairs of electrons (two pairs) and it does not have an uninterrupted ring of p orbital-bearing atoms.

cycloheptatriene cyclopentadiene

GENERAL ORGANIC CHEMISTRY


122 QUIZRR
The criteria for determining whether a monocyclic hydrocarbon compound is aromatic can also be
used to determine whether a polycyclic hydrocarbon compound is aromatic. Naphthalene (five pairs
of electrons), phenanthrene (seven pairs of electrons), and chrysene (nine pairs of electrons) are
aromatic.

naphthalene phenanthrene chrysene

In cyclic compounds, when an element other than carbon is present in the ring, they are called
heterocyclic compounds. They are aromatic in nature because of fulfilling both the conditions.

N N O S

H
Pyridine Pyrrole Furan Thiophene

In pyrrole, furan and thiosphene, the lone pairs are actually in sp3 hybrid orbitals to form the
aromatic sextet (4n + 2) -electrons. In pyridine, the lone pair is in sp2 hybrid orbital, which is not
involved in delocalization thus, pyridine also has 6 -electrons and is aromatic in nature.

A CI DS & B A SES
The conversion of carboxylic acids and phenols to carboxylate or phenolate ions is common because
the proton is acidic, thus the proton is easily removed to form the anion in weakly basic medium. Most
organic compounds are much less acidic than carboxylic acids, and thus need stronger basic medium
to ionize a carbon-hydrogen bond. Because of the high electronegativity of a fluorine atom, fluorinated
compounds are always more acidic than non-fluorinated compounds.
Strength of an Acid : The strength of an acid HA in water may be determined by considering
the equilibrium :


  
 H3O  A
H2 O + HA 

The equilibrium constant, in water is given by :

 H3O    A  
Ka    
HA 

GENERAL ORGANIC CHEMISTRY


QUIZRR 123

It should be emphasized that Ka, the acidity constant of the acid in water, is only approximate (as
above) if concentrations are used instead of the more correct activities. The smaller the numerical value
of pKa, the stronger the acid to which it refers.

The origin of ac idit y in organic c om pounds


Among the factors that may influence the acidity of an organic compound, HA, are :
(a) The strength of the H-A bonds.
(b) The electronegativity of A.
(c) Factors stabilising Aă compared with HA.
(d) The nature of the solvent.
Of these (a) is not normally found to be a limiting factor, but the effect of (b) is reflected in the
fact that the pKa of methanol, CH3O-H, is = 16 while that of methane, CH4 is = 43, oxygen being
considerably more electronegative than carbon. Much more important is (c) : the stabilisation is possible
in the methanoate anion compared with the undissociated methanoic acid molecule results in the pKa
of formic acid being 3.77.

O O

HC HC
ă
OăH O

+ H2O + H3O

O O
HC HC
OăH O

This is extremely effective delocalisation, with consequent stabilisation, in the methanoate anion
involving as it does two canonical structures of identical energy.
With alcohols there is no such factor ă
O O O
stabilising the alkoxide anion ROă, relative to
the ROH itself and alcohols are thus very much ă
less acidic than caroxylic acids. With phenols,
however, there is again the possibility of relatvie
stabilisation of the anion by delocalisation of its ă

negative charge through interaction with the 


orbital of the aromatic nucleus.
O

GENERAL ORGANIC CHEMISTRY


124 QUIZRR
Phenols are indeed found to be stronger acids than alcohols (the pKa of phenol itself is 9.95) but
considerably weaker than carboxylic acids. This is due to the fact that in carboxylic acids, delocalisation
of the negative charge in the carboxylate anion involves structures of identical energy and the atoms
holding the negetive charge are the highly electronegative oxygen atoms; whereas in the phenoxide
anion, the structures involving negative charge on the carbon atoms are likely to be higher energy
content than the one in which it is on oxygen and, in addition, of the centres carrying negetive charge,
only one structure is carrying highly electronegative oxygen atom.

Influenc e of t he solvent :
The prime requirement of the solvent is that it should be capable of functioning as a base; the
weaker the base, the smaller the dissociation of the acid. Thus we find that in, for example, methylbenzene
(toluene) HCl occurs as such, i.e., it is almost fully undissociated.

Fa c t o r s a f f e c t in g t h e s t r e n g t h o f a c i d s a n d b a s e s
[A] Effect of structure :
1. Field effect (ăI effect and +I effect)
2. Resonance effect
3. Periodic Table correlations
4. Steric hinderance
5. Hybridisation
6. Statistical Effects : In a symmetrical diprotic acid, the first dissocation constant is twice as
large as expected, since there are two equivalent ionizable hydrogens, while the second constant
is only half as large as expected because the conjugate base can accept a proton at two
equivalent sit es. So k1/k2 should be 4. Similar arguments hold good for basic groups.
7. Hydrogen Bonding :
p-hydroxy benzoic acid > o ă hydroxy benzoic acid
pka = 4.58 pkn = 2.98
ă5
(ka = 10 ) (kn = 10ă3)
Intra molecular hydrogen bonding between the OH and COOă groups of the conjugate base
of the ortho isomer stablizes it and results in an increased activity.

[B] The Effect of Medium or Solvent


(a) If a base is more solvated than its conjugate acid, its stability is increased relative to the
conjugate acid. e.g.
MeNH2 > NH3.
(b) Aniline in the gaseous phase is a stronger base than NH3 but its much lower basicity in
aqueous solution is caused by similar solvation effects and not by resonance and field electron
with drawing effect of a phenyl group.

GENERAL ORGANIC CHEMISTRY


QUIZRR 125

Similarly,
Pyridine and pyrrole < NH3 (In aqueous solution)
Pyridine and pyrrole > NH3 (In aqueous phase)
(c) For simple alcohols, following order of acidity is due to solvation effect.
H2O > MeCH2OH > Me2CHOH > Me3COH (in solution)
H2O > MeCH2OH < MeCHOH > Me3COH (in gaseous phases)
(d) A change from a protic to an aprotic solvent can also effect the acidity or basicity because there
is a difference in solvation of anions by a protic solvent and an aprotic one e.g.
In DMF : Picric acid is stronger than HBr
In water : HBr is stronger than picric acid

Re l a t i v e St r e n g t h o f Org a n i c A c i d s
1. % S-character
(a) Hydro carbons
Hydrogen atom attached to sp-hybridised carbon is more acidic than that on sp2 hybridised carbon
which in turn is more acidic than that on sp3.
In other words, acidity increases with percentage (%) s-character. This is because, higher the s-
character closer the electrons to the nucleus of C and farther the electrons go from H, and easily
can H be removed as H+.
Thus,

HC  CH  H2C  CH2  H3C · CH3


sp sp2 sp3
The factors responsible for influencing the acidity of an organic compound, HăX are
(a) the strength of the HăX bond
(b) the electronegativity of X
(c) factors stabilizing Xă as compared to HX and
(d) the nature of the solvent.
The most important factors among all is (c). More stable (less basic) the conjugate base is, more
acidic the compound will be. On comparing the acidic strength of two compounds, the compound having
greater acidity will have higher value of Ka and lower pKa value.
Now let us see using any of these factors, how we can predict the order of acidic strength for
organic acids.
(i) Alcohols : If we were to predict the order of acidic strength of ethanol, iso-propanol and
tertiary butanol, we can proceed as
Me Me
ă 2 ă 3 ă
Me CH2 O H CH O H C O H

Me Me

GENERAL ORGANIC CHEMISTRY


126 QUIZRR
As alkyl groups (methyl group) have a +I effect, the electron density at the oxygen atom will
increase. More the number of alkyl groups, more will be the intensity of negative charge on oxygen.
Greater the negative charge on the oxygen atom, the closer is the electron pair in the OăH bond driven
to the hydrogen atom and consequently separation of a proton becomes increasingly difficult. Thus, the
acidic strength of given alcohols will be in the order
Ethanol > iso-propanol > t-butanol
In general, electron releasing groups decreases the acidity while electron withdrawing
group increases the acidity of a compound.

Su b s t it u t e d Ph e n o l s
The effect of a ring substituent on the acid strength depends on :
(1) Substitued group is electron attracting or releasing : electron withdrawing groups increases its
acidity and vice-versa.
(2) Ability of the group to enter into resonance with the hydroxyl group and its position.
(3) Steric effect and hydrogen bonding due to the substituted group.

Illust rat io n
Compare acidity of

OH OH OH OH

CH3

, , ,
CH3

Solution : CH3

The methyl group is electron releasing and so release of a lone pair from oxygen (of the OH group
in the unionized phenol or from the Oă phenoxide ion) into the ring is opposed. This results in
diminished resonance in the contributing structures and consequently the phenoxide ion is more resonance
stabilized with respect to phenol than is the methyl phenoxide ion with respect to methylphenol. Hence,
phenol is stronger acid than all methyl phenols. Thus, the order of acidity is

OH OH OH OH

CH3
> > >

CH3

CH3

GENERAL ORGANIC CHEMISTRY


QUIZRR 127

Now among o, p, m methyl phenols, meta methyl phenol is most acidic

O O O

ă ă ă ă ă ă
CH 3

ă CH 3 ă ă

CH3
Now it is clear that meta methyl phenol are more acidic than ortho and para after the above
structure. Now in case of o-methyl phenol, methyl group is closer to O of OH group in phenol than
compared to p-methyl phenol. As a result it show more +I effect, hence decreasing the acidic character.

Illust rat io n
Compare the acidic strength of
OH OH OH OH

NO2

, , ,
NO2

Solution : NO2
The corresponding phenoxide ions obtained from the three nitrophenols would be stabilized by
delocalization as
ă
(1) O O

NO2 NO2
ă

ă
O O O O O O
N O ă N O N O

ă
ă
O
O
N O

GENERAL ORGANIC CHEMISTRY


128 QUIZRR
(2) ă
O O O

NO2 NO2 ă NO2

ă
O O

NO2 NO2

(3) ă
O O O

NO2 NO2 N

O O

ă
O O O

NO2 NO2 N
ă
O O
We can clearly see that oăand pănitrophenoxides are more extensively stabilized with respect to
m-nitrophenoxide and hence they are more acidic.
Now, m-nitrophenol is more acidic than phenol because of the ăI effect of the nitro group.
Out of oăand pănitrophenols, p-nitrophenol is slightly stronger than o-isomer as o-isomer involves
intramolecular hydrogen bonding, thus having less tendency to release the Hydrogen atom.

GENERAL ORGANIC CHEMISTRY


QUIZRR 129

O H
O
N
O

Thus the order of acidity is

OH OH OH OH

NO2
> > >

NO2

NO2

Note : The cumulative effect of the three nitro groups 2, 4, 6 positions results in a very strong
acid {2, 4, 6-trinitrophenols or picric acid}
OH

O2 N NO2

NO2

Illust rat io n
Compare the acidity of the following :

OH OH OH OH
Cl

, , &
Cl
Cl
Solution :
Since all halogens show ăI effect, it increases the acidic nature of phenols. Therefore all halophenols
are more acidic than phenols. Now, halogens cam show both +R & ăI effect. But there, ăI effect
dominates. The reasons is that for resonance to occur, there should be sufficient overlapping between
the molecules & since the size of halogens and carbon are very different, resonance does not occur
properly.
 All Halogens are weakly deactivating

GENERAL ORGANIC CHEMISTRY


130 QUIZRR
Since we are considering ăI effect, and we know that I effect decreases with distance
 Order becomes
OH OH OH OH
Cl

Cl
Cl

Illust rat io n
Arrange the following in decreasing acidic order

OH OH OH OH
OCH3

OCH3
OCH3
Solution :
Here in this question OCH3 shows ăI effect which increases the acidic nature of phenols
But, methoxy group at para position and orthoposition can show electron repelling nature. but at
ortho position we have ortho effect which makes it most acidic. And at p position methyoxy shows
repelling nature, which contradicts its ăI effect and hence making it even less acidic than phenols.

OH OH OH
now this will repel
the lone pair of OH
& hence will not
facilitate the release
OCH3 OCH3 of H.
+ OCH3 +
Hence the order becomes
OH OH OH OH
OCH3

OCH3
OCH3

A l i p h a t ic c a r b o x y l ic a c i d s :
The aliphatic carboxylic acids are much stronger acids than phenols and alcohols. This is attributed
to the fact that the carboxylate ion (obtained by the loss of proton from carboxylic acid) is relatively

GENERAL ORGANIC CHEMISTRY


QUIZRR 131

more stabilized by delocalization than the phenoxide and alkoxide ions with respect to their undissociated
molecules.


 ă +
R  C OH  R  C O  H
|| ||
0 0

The delocalization of the negative charge in the carboxylate anion involves structures of identical
energy content (as the negative charge resides on more electronegative atom, oxygen) while in phenoxide
anion, the negative charge also resides on the less electronegative atom (carbon) making these structures
of high energy content with respect to those structures in which the negative charge is on oxygen.
The presence of electron-withdrawing substitutents in simple aliphatic acids increases their acidity
while the electron-releasing substituents have reverse effect. For instance, let us compare the acidic
strength of fluoroacetic acid and acetic acid.


 ă +
CH3  C OH  CH3 ă C ă O  H
|| ||
0 0


 ă +
FăCH2  C OH  F ă CH2 ă C ă O  H
|| ||
0 0

The fluoro acetate ion is stabilized more due to strongăI effect of fluorine, with respect to acetate
ion. Hence, fluoro acetate ion is less basic than acetate ion, thereby making fluoroacetic acid stronger
than acetic acid.
Now, we can also compare the acidic strength of -and -halosubstituted acids i.e.

  
F · CH2 · CH2 · C· OH , F · CH2 · C· OH
|| ||
C C

Here the -substituted compound will have less ă I effect as compared to -substituted acid and
thus will be less acidic than b-substituted acid. Hence, the acidic strength is

F · CH2 · COOH > F · CH2 · CH2 · COOH > CH3 · COOH

If there is a doubly bonded carbon atom adjacent to the carboxyl group, the acid strength is
increased. This will be evident if we compare the acid strength of propanoic acid and propenoic (acrylic)
acid.

 O


CH2  CH ă CO2H  CH2  CH ă C  H
ă
O

 O
CH3 ă CH2 ă CO2 H 

 CH3 ă CH2 ă C  H
ă
O

GENERAL ORGANIC CHEMISTRY


132 QUIZRR
As we know that unsaturated -carbon atom in propenoate ion is sp2 hybridised, which means that
electrons are drawn closer to the carbon nucleus than in a saturated, sp3 hybridised -carbon atom,
due to greater s-contribution in the sp2 hybrid orbital. The result is that sp2 hybridised carbon atoms
are less electron-donating than saturated hybridised ones, thus, propanoate ion will be comparatively
less stabilized than propenoate ion, thereby making propenoic acid stronger than propanoic acid (but
propanoic will still be weaker than methanoic acid).
It there is a triple bonded carbon atom adjacent to carboxyl group, the acid strength is more with
respect to the presence of a doubly bonded carbon atom adjacent to ăCO2H group. This is attributed
to the fact that sp hybridized carbon atoms are less electron-releasing than sp2 hybridized carbon
atoms.
Some other trends of acidity are :
HCO2H > CH3CO2H > CH3CH2CO2H > ................
HO2CăCO2H > HO2CăCH2ăCO2H > HO2CăCH2CH2ăCO2H > ...........
These trends reflect that in any homologous series, when we move from lower members to higher
members, the acidity decreases.

A r o m a t i c c a r b o x y l ic a c i d s
It has been pointed out that replacement of the hydrogen atom in formic acid by an alkyl group
weakens the strength of the acid and the greater the +I effect of the R group, the weaker is the acid.
Phenylacetic acid, PhCH2CO2H is stronger than acetic acid and therefore the phenyl group has an
overall-I effect. On the other hand, benzoic acid is weaker than formic acid. In this case, the phenyl
group has an overall releasing effect (which is smaller than that of methyl group). These apparently
contradictory results may be explained as follows. When the carboxyl group is directly attached to the
nucleus, the resonance effect (+R) overcomes the ăI effect (in phenylacetic acid, the phenyl group is
insulated from the carboxyl group by a CH2 group and so the +R effect is not possible) :
ă
O + O O

C C + C

OH OH OH

ăI +R +R

This prevents to a large extent, the lone pair on the O atom of the OH group from entering into
resonance with the CO group. The result is a smaller positive charge on the O atom of the OH group
and so proton release is more difficult than in formic acid. The fact that benzoic acid is stronger than
acetic acid means that [ăI+(+R)] < [+I+(+R)] of the methyl group.
The same arguments may be applied to ionized benzoic acid.
O O + O
+
C + H2O H3O + C C

OH O O

ăI ăI ăR

GENERAL ORGANIC CHEMISTRY


QUIZRR 133

The orthoăeffect. This is a steric effect that is shown by o-substituents. Nearly all o-substituted
benzoic acids are stronger than benzoic acid due to this ortho effect irrespective of the polar nature of
the o-substituent.
As we have seen that benzoic acid is a resonance hybrid and so the carboxyl group is coplanar with
the ring. An o-substituent tends to prevent this coplanarity. Thus, resonance is diminished (or prevented)
and so the O atom of the OH group has a greater positive charge, resulting in increased acid strength.
It follows from this that greater the steric inhibition of resonance, the stronger is the acid. Support for
this is the following order of strengths of substituted benzoic acids.
2, 6-diăMe > 2-t-Bu > 2-Me
Here again, if we consider the stability of the anion, steric inhibition of resonance prevents the +R
effect of the ring coming into operation and since this weakens acid strength, its absence results in
increased acid strength.
OăHydroxybenzoic acid (salicylic acid) is far stronger than the corresponding m-and p-isomers.
Steric inhibition of resonance cannot explain this very large increase, since the corresponding
methoxybenzoic acids all have almost similar strengths. The carboxylate ions of o-hydroxybenzoic acid
is stabilised by intramolecular hydrogen bonding while such hydrogen bonding is not feassible in o-
methoxy benzoic acid and support for this is given by the following order of acid strength,
2, 6 ă di ă OH > 2 ă OH > benzoic acid
It can be seen that two hydrogen bonds would be expected to bring about more stabilization than
one hydrogen bonds.

ăó ăó ăó ăó ăó ăó
O O O O O O
C C H H C H
O O O

Illust rat io n
Compare acidic strength of :

COOH COOH COOH COOH


CH3

, , ,
CH3
CH3

GENERAL ORGANIC CHEMISTRY


134 QUIZRR
Solution :
Due to the ortho-effect as explained above,

COOH COOH
CH3
>

Now let us consider the effect of substituents from m-and p-positions. First let us take methyl
substituted benzoic acids. As the methyl group has +I effect, so the net result will be to increase the
+R effect in the mă or pătolyl group. Since, the inductive effect decreases with distance, so the +I effect
of CH3 from para position would be less than the +I effect of CH3 group from meta position. Thus, the
electronădensity in the OăH bond of m-methyl benzoic acid would be greater than that in p-methyl
benzoic acid. Accordingly, the p-isomer should be more acidic than the m-siomer. But in real practice,
the order is reverse. Actually, we have ignored the hyperconjugation effect of CH3 group. Due to
hyperconjugation effect of CH3 in p-isomer, the electron density in the OăH bond would be increased
greatly with respect to that in m-isomer.

O O·H O O·H O O·H


C C C

H + +
H H etc.
C·H C·H C·H

H H H

O O·H O O·H O O·H


C C C

etc.

+ +
H·C·H H·C H H·C H

H H H

Thus, the release of proton is facilitated easily in m-isomer than in p-isomer.


Thus, the final order of acidity COOH COOH COOH COOH
CH3
> > >
CH3
CH3

GENERAL ORGANIC CHEMISTRY


QUIZRR 135

Illust rat io n
Compare the acidity of

COOH COOH COOH COOH


NO2

, , ,
NO2
NO2
Solution :
It is clearly observed that due to the ortho effect, o-nitrobenzoic acid will be most acidic. As the ă
NO2 group exerts ăI and ăR effect (both reinforcing each other), the electron-density from the OăH
bond would be withdrawn, thus destabilizing the OăH bond and making mă and pă nitro benzoic acids
stronger than benzoic acid.
O O·H O O·H
C C

NO2 NO2

N+
ă
O O
As it is clear that ăNO2 group exerts its electron withdrawing resonance effect from the p-position
and not from m-position, thus withdrawal of electron density from the OăH bond is greater in p-nitro
benzoic acid than m-nitro benzoic acid. Thus, p-isomer is stronger acid than m-nitro benzoic acid.
Hence the order of acidity is :

COOH COOH COOH COOH


NO2
> > >
NO2
NO2

Illust rat io n
Compare the acidity of
COOH COOH COOH COOH
OCH3
, , ,
OCH3
OCH3

GENERAL ORGANIC CHEMISTRY


136 QUIZRR
Solution :
O-methoxy benzoic acid will be most acidic due to ortho effect.
Now, we compare the acid strength of mă and pă methoxy benzoic acid. OMe group exerts ăI and
+R effects. The electron-releasing resonance effect (+R) is operative in p-methoxy benzoic acid and not
in m-methoxy benzoic acid while ăI effect is operative in both the isomers. Thus, the electron density
is greatly increased in the OăH bond in p-isomer than in m-isomer, thereby making m-isomer stronger
acid than p-isomer.
O O·H
C

OMe

O O·H O O·H O O·H


C C C

etc.

+ +
OMe OMe OMe

As we see here that there exists high electron density on the OăH bond in the p-substituted isomer,
thus benzoic acid will also be more acidic (but less acidic-than m-isomer) than p-isomer.
Thus, the acidic strength is as follows,

COOH COOH COOH COOH


OCH3
> > >
OCH3
OCH3

Illust rat io n
Explain why acrylic (propenoic) acid is weaker acid than propynoic acid ?

Solution :
O O

CH2 = CH · C · O · H CH = C · C · O · H
sp 2 sp

Acrylic acid Propynoic acid

GENERAL ORGANIC CHEMISTRY


QUIZRR 137

If we look at these two acids, electron withdrawing effect of sp2 carbon adjacent to carbonyl group
in acrylic acid is less than the electron withdrawing effect of sp hybridized carbon. Thus, OH bond of
propynoic acid has greater polarization than the OH group of propenoic acid. Hence, propynoic acid is
a stronger acid than propenoic acid.
Note : A CH2 group flanked between two electron. withdrawing groups has acidic hydrogen.
Aldehydic group isomer is more electron withdrawing than ketonic group, which is more electron
attracting than ester groups. Thus, order of acidity of carbon acids is

H ă Că CH2 ă C ăH  CH3 ă Că CH2 ă Că CH3  CH3 O ă Că CH2 ă Că OCH3


|| || || || || ||
O O O O O O
Cl
|
ă
: C ă Cl
CHCl3 is more acidic than CHF3 because the conjugate base of CHCl3 i.e. |
Cl
ă
stabilised by ăI effect of ClÊs as well as by p-d delocalization (which is absent in :CF3 due to
the absence of d-orbital in F).

B A SI C ST REN GT H OF ORGA N I C B A SES


Strength of bases is related to the ease of accepting a proton, which in turn depends on the
availability of electron pair on the nitrogen atom (or some other basic atom). More is the availability
of electron pair, more easily the proton will be accepted and more will be the basic strength.
If we compare the basicities of NH3, MeNH 2, Me2NH and Me3N, then at a glance it would
seem like NH3 < MeNH 2 < Me2NH < Me2NH < Me3N is the basicity order but the result is quite
different in aqueous media. The correct order is Me2NH > MeNH 2 > Me3N > NH 3. This order can be
explained considering the solvation effect of the protonated amines. But in absence of a polar
protic solvent i.e. in the presence of non-polar solvent or gaseous phase, the order of basicity is
Me 3N > Me2NH > MeNH 2 > NH 3.
The effect of introducing electron withdrawing groups like Cl, NO2, RCO group close to a basic
centre decreases the basicity, due to their electron withdrawing inductive effect.
In aniline, owing to resonance, the lone pair of electrons on the nitrogen atom is less available for
co-ordinating with a proton and at the same time, small positive charge on the nitrogen atom would
tend to repel a proton. Alternatively, since there are more resonating structures possible for aniline

itself than for the cation C6 H5 NH3 , the former will be stablized with respect to the latter.
+ + +
:
:

NH 2 NH 2 NH 2 NH 2 NH 2
:
:

GENERAL ORGANIC CHEMISTRY


138 QUIZRR
Aniline is a weaker base than ammonia or cyclohexylamine. It is because of the fact that the
electron pair on nitrogen in involved in delocalization, making it less available for donation.

:
NH 2

:
NH 3

cyclohexylamine ammonia
Diphenylamine is even a weaker base than aniline due to the presence of another phenyl group
:

NH

and triphenylamine (Ph3N) is not basic at all by any means.


Introduction of alkyl group (like Me) on the nitrogen atom of aniline results in small increase in
the basic strength.
C6H5NH2 < C6H5NH2Me < C6H5NMe2
Unlike such introduction in aliphatic amines, this small increase in basic strength is progressive,
indicating that cation stabilization through hydrogenăbonded solvation, here as less influence on the
overall effect.
The effect of a ring substitutent on basicity depends on whether the substituent is electron attracting
or releasing, its ability to enter into resonance with the amino-group and its position. All the nitroanilines
are weaker bases than aniline. The nitroăgroup has a strong ăR effect and oăand pănitroaniline are
therefore, more resonance stabilised than aniline itself.

Oă Oă
+ + +
N NH2 N NH2
ă
O O

A m-nitro group cannot enter into resonance with the amino-group, but nevertheless m-nitroaniline
is a much weaker base than aniline. In this case, nitro-group has a strong-I effect. This ăI effect tends
to draw into the ring the lone pair of N, thus decreasing its basicity. Thus, the order of basicity of
nitroanilines is C6H5NH2 > m-NO2C6H4NH2 > p-NO2C6H4NH2 > o-NO2C6H4NH2.
+ +
:

O NH 2 O NH 2 O NH 2
ă
O=N O=N OăN
so on

GENERAL ORGANIC CHEMISTRY


QUIZRR 139
+ + +

:
NH 2 NH 2 NH 2 NH 2

so on

N N N N
O O O O O O O O

Tetraalkylammonium salts, e.g. R4N+Iă on treatment with moist silver oxide (AgOH) yield basic
solutions comparable in strength with the mineral alkalis. This is readily understandable as R4N+ ăOH
formed is completely ionized to give R4N+ and free OHă.
the amine Tris (trifluoro methyl) amine is found to be virtually non-basic due to the presence of
three powerful electron-withdrawing CF3 groups.

F3C
F3C N:
F3C

The amides are also found to be only very weakly basic in water because of the ăI and ăR effect
of RCO group which makes the electron pair very slightly available on nitrogen atom.

O Oă
+
R ă C · NH 2 R ă C = NH2

If two C=O groups are present, the resultant imides often become suficiently acidic to form alkali
metal salts. For example, benzeneă1, 2-dicarboximide is not basic but is acidic in nature because of the
presence of two electron-withdrawing C = O groups.
O O ă
ă
OH
N·H N ă
ăH2O

O O ă
When the ring contains an electron-releasing group, this reduces resonance of the NH2 group with
the ring and consequently should increase basicity. The methyl group raises the electron density more
at the oă and păpositions than at the măposition. Thus, the carbon atom para to the methyl group has
a high electron density and the lone pair on the nitrogen atom is therefore prevented to some extent
from entering into resonance with the ring. A methyl group raises the electron density at the m-position
is prevented less than for the p-position. Consequently, a methyl group in the ring increases the
basicity of the aniline, more so from the p-position than from the m-position.

GENERAL ORGANIC CHEMISTRY


140 QUIZRR
First drawing structure of
+ + +
NH 2 NH 2 NH 2 NH 2

CH3 CH3 CH3 CH3

+ +
NH 2 NH 2 NH 2

CH3 CH3 CH3


Now here a 3Ĉ carboanion is formed which is highly unstable and thus prevents there resonating
structures. So at para position, methyl group discourages detocalization & hence increases the basicity.
And we know that p & m methyl aniline are more basic than aniline. Now only o position is left.
At ortho position, there is ortho effect which increases the acidity and it is even weather base than
aniline.
 Order becomes
NH 2 NH 2 NH 2 NH 2
CH 3

CH 3
CH 3

Illust rat io n
Arrange the following in increasing order of basicity

NH 2 NH 2 NH 2 NH 2
OCH3

, , ,
OCH3
OCH3
Solution :
Let us see how to solve this case. Methoxy group can show both +R & ăI effect. +R effect will
increase the basicity whereas ăI effect reduces basicity.

GENERAL ORGANIC CHEMISTRY


QUIZRR 141

NH 2 NH 2 NH 2

here lp of NH2 & lp of C atom of


benzene repel each other, hence lp
of NH 2 is not delocalized, which
increases the basic nature.
OCH3 OCH3 OCH3
+ +

NH 2 NH 2 NH 2 NH 2

OCH3 OCH3 OCH3 OCH3


+ + +

In this case ăI effect of OCH3 is stronger & ăI effect reduces the basicity
NH 2

OMe
NH 2 NH 2
OCH3 OCH3
+

Here OCH3 produces the same effect as methoxy


group at para position produced, But, Ortho effect
will also be here, which reduces the basic nature. hence the order is

NH 2 NH 2 NH 2 NH 2
OCH 3

OCH 3
OCH 3

Some Important trends :


(a) Among hydrocarbons, acidity increases with the percentage of s-character. This is because
higher the s-character, closer will be the electrons to the nucleus of carbon and the electrons
further more away from H and it can be easily removed as H+. Therefore, the increasing
order of acidity is, CH3-CH3 < CH2=CH2 < CHCH
(b) Alcohols are more acidic than alkanes because oxygen is more electronegative than carbon
and consequently O-H bond will break easily to give H+ ions than C-H bond.

GENERAL ORGANIC CHEMISTRY


142 QUIZRR
(c) Carboxylic acids are more acidic than alcohols because carboxylate anion is more resonance
stabilised as compared to alkoxide ion.
(d) Phenols are more acidic than alcohols because phenoxide ion is more resonance stabilised as
compared to alkoxide ion.
(e) Phenols are less acidic than carboxylic acids. This is because in phenoxide ion some of the
resonance structures are of higher energy and this makes phenoxide ion not very stable.

O
 H5C6OH < H
OH

(f) CH3COOH > CH3CH2COOH (greater the number of CH3 added, less is the acidity due to +
I effect of CH3).
(g) Increase in electron withdrawing substituents in simple aliphatic acids increases acidity.
CH3COOH < I-CH2COOH < Br-CH2COOH < F-CH2 COOH

Cl
Cl
Cl < Cl COOH
COOH
Cl

(h) As the distance between chloro group and COOH group increases, acidity decreases.

Cl
H3C COOH H3C
COOH < < H3C < COOH
COOH Cl
Cl

(i) Benzoic acid is more acidic than carboxylic acid because benzoate anion is more resonance
stabilised.

(j) Aniline is less basic than NH3. This is because in aniline the nitrogen atom is bonded to an
sp2 hybridized carbon atom but the unshared pair on nitrogen interacts with the delocalized
orbital.

(k) Introduction of an alkyl group on the nitrogen of aniline results in small increase in pKa
value due to + I effect of alkyl groups. Therefore order of acidity is C6H5NH2 > C6H5 NHMe
> C6H5NMe2.

(l) Introduction of phenyl groups on N lowers basicity because the substituted amine becomes
more stable than the ion.

GENERAL ORGANIC CHEMISTRY


QUIZRR 143

Ex a m p l e
Compare the basic strength :

H3C CH3 H3C CH3 (x)


NH2 N N
H3C CH3 (y)

(a) (b) (c)

Solution :
In (c) due to repulsion between bulky CH3 groups, rotation occurs, thus resonance does not occur.
Therefore order of basic strength is :
(a) < (b) < (c)

Ex a m p l e
Write the order of basic strength :

NH2 O

N N N H
H
(a) (b) (c) (d) (e)

Solution :

Delocalisation of electron present on N is possible in (a) therefore (a) is least basic. N in (b) is sp2
hybridised, therefore it is less basic than (c), (d) and (e).

Oxygen in (e) withdraws electrons from N through inductive effect and in (d) CH2 groups increases
ă
e density.

Thus, the order will be :

(a) < (b) < (e) < (c) < (d)

GENERAL ORGANIC CHEMISTRY


144 QUIZRR
Ex a m p l e
Compare the basic strength :

NH2

N
N
H

(a) (b) (c)

Solution :
eă will be delocalised through resonance in (a). In (b) N is sp2 hybridised. Therefore, basic strength
is :
(a) < (b) < (c)

Points to Remember :
(1) The more stable Aă is, more stronger acid HA will be.
(2) Easier a compound is ready to give electron, more will be it basic in nature.

GENERAL ORGANIC CHEMISTRY


HYDROCARBON
QUIZRR 3

ALKANES [C nH2n+2]

Alkanes, also known as paraffins, are chemical compounds that consists only of the elements
carbon (C) and hydrogen (H) (i.e., hydrocarbons), wherein these atoms are linked together
exclusively by single bonds (i.e., they are saturated compounds) without any cyclic structure (i.e.
loops).

Each carbon atom must have 4 bonds (either C-H or C-C bonds), and each hydrogen atom must
be joined to a carbon atom (H-C bonds). A series of linked carbon atoms is known as the carbon
skeleton or carbon back-bone. In general, the number of carbon atoms is often used to define the
size of the alkane (e.g., C2-alkane).

An alkyl group is a functional group or side-chain that, like an alkane, consist solely of singly-
bonded carbon and hydrogen atoms, for example a methyl or ethyl group.

PHYSICAL PROPERTIES

Conformations of Alkanes

The carbon-carbon bonds in alkanes result from sigma overlap of two tertrahedral carbon sp3
orbitals. Sigma bonds result from the head-on overlap of two atomic orbitals, and that they are
cylindrically symmetrical. The cross section of a sigma orbital is circular. Due to this sigma bond
symmetry, there is free rotation around the carbon single bond. Bond overlap is exactly the same
for all geometric arrangements of the hydrocarbons. The different arrangements of atoms caused
by rotation about a single bond are called conformation, and a specific structure is called a
conformer (conformational isomer).

HH H
H H

H H
H H H
H

Newman projections of the two conformations of ethane : eclipsed on the left, staggered on the
right.

Staggered form is more stable than eclipsed as more repulsions are experienced between the alkyl
group in eclipsed form.

HYDRO CARBON
4 QUIZRR
Example 1

Compare the torsional strain in the gauche conformer of butane with the anti conformer.
Draw Newman projections of both and indicate the values of the dihedral angles between
MeÊs.
Solution :
The gauche conformer has the Me groups on adjacent CÊs closer to each other, leading to greater
torsional strain than in the anti conformer where they are separated as much as possible.

Me Me
H Me H H

H H H H

H Me
Gauche 60Ĉ Anti 180Ĉ

PREPARATION OF ALKANES

1. Hydrogenation of alkenes

C=C +HăH Catalyst


C C
(Pt/Pd/Ni)
H H

Hydrogenation is the chemical reaction that results from the addition of hydrogen (H2). The
process is usually employed to reduce or saturate organic compounds. The process typically
constitutes the addition of pairs of hydrogen atoms to a molecule. Catalysts are required for the
reaction to be usable; non-catalytic hydrogenation takes place only at very high temperature.
Hydrogen adds to double and triple bonds in hydrocarbons.

Ć Alkenes can be reduced to alkanes with H2 in the presence of metal catalyst such as Pt, Pd,
Ni or Rh.

Ć The two new C-H  bonds are formed simultaneously from H atoms absorbed on to the metal
surface.

Ć The reaction is stereospecific giving only the ÂsynÊ addition product.

HYDRO CARBON
QUIZRR 5

Catalytic Hydrogenation

Step 1 : Hydrogen gets absorbed onto the HăH


metal surface.

H H
Step 2 : Alkene approaches the H atoms
absorbed on the metal surface.

Step 3 : C=C reacts with the atoms on the H H


surface forming two new C-H  bonds.

H H

Note : Since hydrogenation is ÂsynÊ addition, hence we can observe the following :

CH3 CH3 CH3


D2 /Ni H OH H OH
H OH +
H D D H
CH = CH2 CH2 D CH2 D

Diastereomers

2. Wurtz reaction

Wurtz reaction, is a coupling reaction whereby two alkyl halides are reacted with sodium to
form a new carbon-carbon bond :

dry
2R  X  2Na 
 R  R  2NaX
ether

Note : Methane is not formed by this method.

HYDRO CARBON
6 QUIZRR
M echanism :

This reaction involves the radical species RĆ


One electron from sodium is transferred to the halogen to produce a sodium halide and an alkyl
radical.
+
R ă X + Na  R Ć + Na X
The alkyl radical then accepts an electron from another sodium atom to form an alkyl anion and
the sodium becomes cationic.
R Ć + Na  Ră Na+
The alkyl anion then displaces the halide in an SN2 reaction, forming a new carbon-carbon
covalent bond.
R ă Na+ + RX   R ă R + Na+ X

Example

anhydrous
2CH3I  2Na 
 CH3CH3  2NaI
ether
ethane

anhydrous
2CH3 CH2 Cl  2Na 
 CH3 CH2 CH2CH3  2NaCl
Sodium ether Sodium Chloride
n  butane

Here methyl iodide and ethyl chloride are reacted with powered sodium metal in anhydrous
ether. Alkanes containing double the number of carbon atoms are formed, i.e., methyl iodide gives
ethane, and ethyl chloride gives n-butane.
The solvent, ether in this case, must be anhydrous (free of moisture) become the alkyl anions are
so basic they readily deprotonate water to hydroxide ion, forming alcohols, and reducing the yield
of the desired product.

Limitations

(1) The wurtz reaction is limited to the synthesis of symmetric alkanes. If two dissimilar alkyl
halides are taken as reactants, then the product is a mixture of alkanes that is, often,
difficult to separate.
(2) Tertiary alkyl halides do not give wurtz reaction. (Since it follows SN2 mechanism)

Illustration

Br Cl + 2Na Cl Cl

This is formed because C-Br bond is weaker than C-Cl bond.

HYDRO CARBON
QUIZRR 7

3. Clemmensen reduction

The Clemmensen reduction is a chemical reaction described as a reduction of ketones (or


aldehydes) to alkanes using zinc amalgam and hydrochloric acid.

H H
O
Zn(Hg)
C
HCl
R1 R2 R1 R2

The Clemmensen reduction is particularly effective at reducing aryl-alkyl ketones.


With aliphatic or acyclic ketones, zinc metal reduction is much more effective.
Ć The substrate must be stable in the strongly acidic conditions of the Clemmensen reduction.
As a result of Clemmensen Reduction, the carbon of the carbonyl group involved is converted
from sp2 hybridisation to sp3 hybridisation. The oxygen atom is lost in the form of one
molecule of water.

4. Wolff-Kishner reduction

The Wolff-Kishner reduction is a chemical reaction that fully reduces a ketone (or aldehyde) to
an alkane.

NH2
O N H H
NH2 NH2
HOH
R1 R2 R1 R2 R1 R2

Reaction mechanism

The most probable mechanism involves the elimination of an alkyl anion as the final step :
H H
ă ă
H2N N N
O N N N
ăH2O OH ă
+ H2 NNH2
R R´ R R´ R R´ R R´

ă
OH
N
H2O N H H
ăN2 ă H2O

R ăH2O ă
H R H R H + OH
R´ R´ R´

HYDRO CARBON
8 QUIZRR
The mechanism first involves the formation of the hydrazone in a mechanism that is probably
analogous to the formation of an imine. Successive deprotonations eventually result in the evolution
of nitrogen. The mechanism can be justified by the evolution of nitrogen as the thermodynamic
driving force.
Ć Wolff-kishner reduction, utilizes strongly basic conditions.

5. Kolbe electrolysis

The Kolbe reaction is formally a decarboxylative dimerisation and proceed by a radical reaction
mechanism. Generally, the reaction can be observed as :

2RCOOă  R ă R + 2CO2

As an example, electrolysis of acetic acid yields ethane and carbon dioxide :

CH3COOH  CH3COOă  CH3COOĆ  CH3Ć + CO2

2CH3Ć  CH3CH3

A concentrated solution of sodium or potassium salt of a carboxylic acid or mixture of carboxylic


acids is electrolysed.

Current
RCO2 K + R´ CO2 K + 2H2O   R ă R´ + 2CO2 + H2 + 2KOH

Current
2CH3 CO2 K + 2H2O   CH3 ă CH3 + 2CO2 + 2KOH + H2
  
anode cathode

Ć If R and R´ are different, then hydrocarbons R ă R´, R ă R, R´ ă R´ are also obtained.

Ć Esters, lower alkane and alkenes are also obtained as side products.

ă +
C2H5CO2K 

 C2H5CO2 + K

C2H5CO2ă ă eă  C2H5CO2Ć (I)

C2H5CO2Ć + C2H5Ć  C2H5CO2C2H5


(I) (II) ester

Intermolecular
C2H5Ć + C2 H5Ć 
Hydrogenation
 C H + C H
2 6 2 4

HYDRO CARBON
QUIZRR 9

6. MAZINGO METHOD
In this method carbonyl compound is converted into its dithio-acetal or Ketol using ethanedithiol
in the presence of a lewis acid, dithio-acetal is hydrogenlysed over Raney Nickel.

R R S CH2
CH2SH Et2O+ăBF3
ă
C=O+ C
20Ĉ C
R CH2SH R S CH2

H2 ă Ni

R
CH2
R

7. Using orgnometallic compounds

Organometallic compounds react as electron rich or anionic carbon atoms i.e. as carbanions, which
means they will function as either base or nucleophiles.
It is reasonable to think of these organometallic compound as RăM+.

Basicity :
The following equation represents the loss of a proton from a generic hydrocarbon forming a
carbanion :

ă
RăH R H+
hydrocarbon carbanion

Ć Organolithium and organomagnesium compound are strong bases since the negative charge
is on carbon.
Ć Simple carbanions are strong bases, since the C is not very electronegative (compared to N
or O)
Ć In the presence of weak acid, RLi and RMgX protonate giving the hydrocarbon.

(a) Decomposition of Grignard Reagent

Grignard reagent (an organametallic compound) on treatment with water or dilute acid, or alcohol
is decomposed to alkanes.

X
R ă Mg ă X + H2 O RH + Mg
(Grignard
Reagent) OH

(R is alkyl or aryl X is Cl, Br, I)


HYDRO CARBON
10 QUIZRR
It is the R part of the Grignard Reagent that is converted into RH.
If heavy water D2O is used, then product is

X
R ă Mg ă X + D2 O  RD + Mg
OH

X
+ D2 O + Mg

MgX D OH

RMgX + NH3  R ă H + Mg(NH2)X

RMgX + CH3OH  RH + Mg (OCH3)X

( b) Coupling of alkyl halides with organometallic compounds corey–House Synthesis

R ă CuLi + R´X R ă R´ + RCU + LiX


Lithium Dialkylcopper

Li
RX R ă Li RCuLi + R´ X R ă R´
Alkyl Lithium
Lithium dialkyl copper

For good yields R´X should be a primary halide, the alkyl group R in the organametallic may be
primary, secondary or tertiary, for example.

Li Cul
CH3 Br CH3 ă Li CH3 ă CuLi

(CH3)CH2)6CH2I

CH3 ă CH2(CH2)6CH3

NOTE : Unlike wurtz Reaction, this reaction can be employed to prepare unsymmetrical alkanes

HYDRO CARBON
QUIZRR 11

8. Reduction by dissolving metals :

Reduction by dissolving metals e.g. zinc and acetic or hydrochloric acid, zinc and sodium hydroxide,
zinc-copper couple and ethanol, etc. In this reaction, earlier ÂnascentÊ hydrogen was considered to
be reducing agent. Now it is believed that there is an electron-transfer from the metal to the
substrate leading to the formation of carbanion, which is followed by the abstraction of a proton
from the solvent. Thus, reduction with a zinc-ethanol couple may be formulated as
Zn  Zn2+ + 2eă

e
RX + eă  Xă + R  ă
 R:

R:ă + C2H5OH  R ă H + ăOC2H5

9. Reduction by reducing agents like LiAlH4, NaBH4 etc.

Primary and secondary alkyl halides are readily reduced to alkanes by lithium aluminium hydride
LiAlH4 while reduction of tertiary halides with LiAlH4 gives mainly alkenes. On the other hand,
sodium borohydride NaBH4 reduces secondary and tertiary, but not primary, whereas triphenyltin
hydride Ph3SnH reduces all three types of alkyl halide. So each reducing agent is specific in its
action.

(a) 4R ă X + LiAlH4  4R ă H + LiX + AlX3 (X  F)

or R ă X + H :(ă)  R ă H + Xă (Hă comes from LiAlH4)

(b) R ă X + (n ă C4H9)3SnH  R ă H + (n ă C4H9)3SnX

LiAlH4 is not useful for 3Ĉ alkyl halide which is converted into alkenes. In such cases NaBH4 or
TPH is used.

LiAlH4
CH3 ă CH2 ă CH3 CH3 ă CH2 ă CH3

Cl

CH3
LiAlH4
CH3 ă C ă Cl CH3 ă CH2 ă CH3

CH3 CH3

Cl NaBH4
X (No Reaction)

HYDRO CARBON
12 QUIZRR

Cl

TPH
1Ĉ, 2Ĉ, 3Ĉ alkyl halide 
 TPH  Ph3SnH 
 Alkane

1 0. Reduction of alcohols, carbonyl compounds, acids and acid derivatives :

Alcohols, aldehydes, ketones, carboxylic acid their derivatives like acid halides and acid amides
can be reduced by HI and red phosphorous to alkanes.

red P
R ă CH2OH + 2HI   R ă CH3 + I2 + H2O

red P
R ă CHO + 4HI   R ă CH3 + 2I2 + H2O

red P
R ă CO ă R´ + 4HI   R ă CH2 + R´H + 2I2 + H2O

red P
R ă COOH + 6HI   R ă CH3 + 3I2 + 2H2O

red P
R ă COCl + 6HI   R ă CH3 + 3I2 + + HCI + H2O

red P
R ă CONH2 + 6HI   R ă CH3 + 3I2 + NH4OH

Reduction of esters with red phosphorus and HI gives two alkanes

O
|| Red Phosphorus
R  C  O  R´  HI   RH  R´H

We can use other reducing agents also reduce alkyl halide to alkanes.
P  HI
R ă X  CH3 CH3
or
Zn + HCl
or
Zn + CH3OH
or
C2H5OH + Na

HYDRO CARBON
QUIZRR 13

Example 2

Prepare
(i) Butane from chloro ethane using the Corey ă House synthesis.
(ii) Deuteropropane from isopropyl bromide.
Solution :
(i) 1. Li
CH3CH2Cl (CH3CH2)2LiCu
2. Cul
Chloro ethane Lithium diethyl cuprate

CH3 CH2 Cl

CH3 CH2 CH2 CH3

1. Li
(ii) CH3CH2Cl (CH3CH2)2LiCu
2. Cul
Chloro ethane Lithium diethyl cuprate

CH3 CH2 Cl

CH3 CH2CH2CH3

Example 3

Prepare 2-methylbutane from chloroethane and 2-chloropropane using Corey-House


synthesis.
Solution :

I. Li CH3CH2Cl
CH3CHCl ((CH3)2CH)2CuLi CH3CHCH2CH3
2. Cul

CH3 CH3 CH3


2-Chloropropane Lithium disopropyl cuprate

Example 4

Predict the products in the following reactions :

Ni
(i) Cyclopropane + H2 
120

CH3 CH3
(ii) C=C + H2 ?
D D

HYDRO CARBON
14 QUIZRR
Solution :
(1) Propane

H3C CH3
(ii) H C C H
D D
(Meso)

Chemical Reactions of Alkanes


Ć Alkanes are generally inert at room temperature as they have not any functional group
Ć They undergo free radical substitution reaction

Radical substitution Mechanism

CH4 + Cl2  CH3Cl + HCI

The organic product is chloromthane.


One of the hydrogen atoms in the methane has been replaced by a chlorine atom, so this is
substitution reaction. However, the reaction doesnÊt stop there, and all the hydrogens in the
methane can in turn be replaced by chlorine atom.

The mechanism

The mechanism involves a chain reaction. During a chain reaction, for every reactive species you
start off with, a new one is generated at the end ă and this keeps the process going.
The overall process is known as free radical substitution, or as a free radical chain reaction.

Chain initiation

The chain is initiated (started) by UV light breaking a chlorine molecule into free radicals

Cl2  2Cl •

Chain propagation reactions

These are the reactions which keep the chain going.

CH4 + ClĆ  CH3 Ć HCl

CH3Ć + Cl2  CH3CI + Cl •

HYDRO CARBON
QUIZRR 15

Chain termination reactions

These are reactions which remove free radicals from the system without replacing then by new
ones.

2ClĆ  Cl2

CH3Ć + ClĆ  CH3Cl

CH3Ć + CHĆ
3
 CH3CH3

The potential energy curve for the halogenation (chlorination) of alkane is shown as

Eact

RăH + Cl
Potential energy

R + Cl2

R ă Cl + Cl
Progress of reaction

In more complex alkanes, the abstraction of each different kind of hydrogen atom gives a different
isomeric product. Three factors determine the relative yields of isomeric product.
1. Probability Factor : This factor is based on the number of each kind of hydrogen atoms
in the alkane molecule. For example, in CH3CH2CH2CH3 there are six equivalent 1Ĉ H and
four equivalent 2Ĉ H. The probability of abstracting a 1Ĉ H to 2Ĉ H is 6 to 4, or 3 to 2.
2. Reactivity of HĆ : The order of reactivity of hydrogen atoms is 3Ĉ > 2Ĉ > 1Ĉ.
3. Reactivity of XĆ : The more reactive ClĆ is less selective and more influenced by the
probability factor. The less reactive BrĆ is more selective and less influenced by the probability
factor, as summarized by the Rea ctivity-Selec tivity Principle. If the attacking species is
more reactive, it will be less selective and the yields will be determined by the probability
factor as well as reactivity of hydrogen atoms while if the species attacking is less reactive
and more selective, the yield of the product is governed exclusively by reactivity of hydrogen
atoms.

Cl2 / h
CH3CH2CH2CH3 CH3CH2CH2CH2(Cl) + CH3CH2CH(Cl)CH3
25C
n-butane (28%) (72%)

HYDRO CARBON
16 QUIZRR

Cl 2 / h
(CH3)2CHCH3 (CH3)2CHCH2Cl + (CH3)3CăCl
25C
isobutane (64%) (36%)

Br / h
CH3CH2CH2CH3 
2 
 CH3CH2CH2CH2Br + CH3CH2CH(Br)CH3
127C

n-butane (2%) (98%)

Br / h
(CH3)2CHCH3 
2
127C

 (CH3)2CHCH2Br + (CH3)3CăBr

isobutane traces (over 99%)

In the chlorination of isobutane abstraction of one of the nine primary hydrogens leads to the
formation of isobutyl chlorides, whereas abstraction of a single tertiary hydrogen leads to the
formation of tertăbutyl chloride. The probability favours formation of isobutyl chloride by the ratio
of 9 : 1. But the experimental results show the ratio roughly to be 2 : 1 or 9 : 4.5. Evidently, about
4.5 times as many collisions with the tertiary hydrogen are successful as collisions with the
primary hydrogens. The Eact is less for abstraction of a tertiary hydrogen than for the abstraction
of a primary hydrogen.
The rate of abstraction of hydrogen atoms is always found to follow the sequence 3Ĉ > 2Ĉ > 1Ĉ.
At room temperature (25ĈC), the relative rates in chlorination are 5.0 : 3.8 : 1.0 respectively for
3Ĉ, 2Ĉ and 1Ĉ hydrogen atoms. Using these values, we can predict quite well the ratio of isomeric
chlorination products from a given alkane. For example,

Cl
CH3CH2CH2CH3   CH CH CH CH Cl + CH CH CHClCH
2
light, 25C 3 2 2 2 3 2 3

n  butyl chloride number of 1 H reactivity of 1 H 6 1.0 3 28%


     equivalent to
sec  butyl chloride number of 2 H reactivity of 2 H 4 3.8 7.6 72%

Inspite of these difference in reactivity, chlorination rarely yields a great excess of any single
isomer. In most cases, both the products are formed in considerable amounts.
The same sequence of reactivity, 3Ĉ > 2Ĉ > 1Ĉ, is found in bromination, but with enormously larger
reactivity ratios. At 127ĈC the relative rates per hydrogen atom in bromination are 1600 : 82 :
1 respectively for 3Ĉ, 2Ĉ and 1Ĉ hydrogen atoms. Here, differences in reactivity are so marked that
it outweights probability factor. Hence bromination almost exclusively gives selective product.
In bromination of isobutane at 127ĈC,

isobutyl bromide number of 1 H reactivity of 1 H


 
tert-butyl bromide number of 3 H reactivity of 3 H

9 1 9 0.5%
=   equivalent to
1 1600 1600 99.5%

HYDRO CARBON
QUIZRR 17

Hence, tert-butyl bromide happens to be the exclusive product (over 99%) with traces of isobutyl
bromide.

The reason for the higher selectivity in bromination as compared to chlorination is due to the
following explanation.

According to the general principle, for comparable reactions, the more endothermic (or less
exothermic) reaction has a transition state (TS), which more closely resembles the intermediate
and may more closely resemble the ground state (reactants). Since attack by BrĆ on alkane is
more endothermic than attack by Cl•, its TS shows more CăH bond breaking and more HăBr
bond formation. Any stabilization in the intermediate radical also occurs in the corresponding TS.
Therefore, a TS leading to a 3Ĉ RĆ, has a lower enthalpy than one leading to a 2ĈRĆ, which in
turn has a lower enthalpy than one leading to 1ĈRĆ, the relative rates of Hăabstraction by BrĆ
are 3Ĉ > 2Ĉ > 1Ĉ. The TS for Hăabstraction by ClĆ has less CăH bond breaking and less HăCl bond
formation. The nature of the incipient radical has less effect on the enthalpy of the TS and on
the rate of its formation. Hence there is less difference in the rate of formation of the three kinds
of RĆs. In the attack by the comparatively unreactive bromine atom, the transition state is
reached late in the reaction process, after the alkyl group has developed considerable radical
character. In the attack by the highly reactive chlorine atom, the transition state is reached early,
when the alkyl group has gained very little radical character. Thus bromination is more selective
than chlorination.

 
R ă H + Br R H Br R + H ă Br
low reactivity Transition state
high selectivity reached late, much
radical character

 
R ă H + Cl R H Cl R + H ă Br
Transition state
reached early, much
radical character

Ć Bromination is selective for the R–H that gives the most stable radical

Ć Chlorination is less selective.

Ć The termination steps are low probability due to the low concentration of the radical species
meaning that the chances of them colliding is very low.

HYDRO CARBON
18 QUIZRR
R adical chain mechanism for reaction of methane with Br 2

Step 1 (Initiation) : Heat or uv light cause


the Weak halogen bond to undergo homolytic
Br Br Br Br
cleavage to generate two bromine radicals and
stariting the chain process.

Step 2
(a) A bromine radical abstracts a hydrogen
to form HBr and methyl radical, then
(b) The methyl radical abstracts a bromine
atom from another molecule of Br2 to form Br H ă CH3 H ă Br ă CH3
the methyl bromide product and a nother
bromine radical, which can then itself
Br ă Br CH3 Br Bră CH3
undergo reaction 2(a) creating a cycle that
can repeat.

Step 3 (Termination) : Various reactions CH3 CH3 CH3 ă CH3


between the possible pairs of radicals allow for
the formation of ethane, Br2 or the product, Br Br Br ă Br
methyl bromide. These reactions remove
radicals and do not perpetueate the cycle. Br CH3 Br ă CH3

Free Radical Substitution of Aromatic Hydrocarbons

CH2 ă CH3 Cl
Cl2, h

Mechanism
Step 1 - Initiation

hv
 Cl   Cl 
Cl2 

Propagation

CH2 ă CH3 CH ă CH3

Cl Radical stabilized by resonance from ring.


Therefore it will be more stable.

HYDRO CARBON
QUIZRR 19

Termination :

Cl
CH ă CH3 CH ă CH3

Cl2
+ Cl

Ć Free Radicals donÊt rearrange generally.


Ć Most stable free radical is formed which decides the formation of product.
Example :

CH3 CH2 Br

(1) NBS

Me Cl Me
CH ă CH C ă CH
Me Me
Cl2 , h
(2)

Example 5

How many monochlorinated product are obtained by the chlorination of isohexane and
what is the percentage of each assuming the reactivity ratio of 3ĈH : 2ĈH : 1ĈH = 5 : 3.8 : 1.
Solution :
Isopentane on chlorination in presence of sun light gives five different monochlorinated products.

h
CH3 ă CH ă CH2 ă CH2 ă CH3 + Cl2 CH2 Cl ă CH ă CH2 ă CH2 ă CH3

CH3 CH3 (A)

CH3 ă CCl ă CH2 ă CH2 ă CH3 + CH3 ă CH ă CHCl ă CH2 ă CH3

CH3 (B) CH3 (C)

HYDRO CARBON
20 QUIZRR

CH3 ă CH ă CH2 ă CHCl ă CH3 + CH3 ă CH ă CH3 ă CH2 ă CH3Cl

CH3 (D) CH3 (E)

Product Reactivity factor x Probability factor = number of parts Percentage


(A) 1 x 6 = 6 20.6%
(B) 5 x 1 = 5 17.1%
(C) 3.8 x 2 = 7.6 26.0%
(D) 3.8 x 2 = 7.6 26.0%
(E) 1 x 3 = 3 10.3%
Total number of parts = 29.2

1. Oxidation

All alkanes readily burn in excess of air or oxygen to form carbon dioxide and water.

 3n  1  2n+2 
CnH2n+2 + O2  nCO2 ( g)    H2 O (l)
2  2 

On the other hand, controlled oxidation under various condition, leads to different products.
Extensive oxidation gives a mixture of products in which no chain fission has occurred. Under
moderate conditions mixed ketones are the major products and oxidation in the presence of boric
acid produces a mixture of secondary alcohols. The oxidation of alkanes in the vapour state occurs
via free radicals, e.g. alkyl (RĆ), alkylperoxy (ROOĆ) and alkoxy (ROĆ). Oxidising reagents such
as potassium permaganate readily oxidise a tertiary hydrogen atom to a hydroxyl group. For
example, isobutane is oxidised to t-butanol.

KMnO4
(CH3)3 CH + [O]   (CH3)3 COH

2. Sulplhonation

It is the process of replacing hydrogen atom by a sulphonic acid group, SO3H. Sulphonation of
a normal alkane from hexane onwards may be carried out by treating the alkane with oleum
(fuming sulphuric acid). The order of case of replacement of H-atoms in tertiary. Compounds is
very much easier than secondary compounds replacement of H-atoms by sulphonic is very slow
indeed. Isobutane, which contains a tertiary hydrogen atom, is readily sulphonated to give t-butyl
sulphonic acid.

(CH3)3 CH + H2SO4/SO3  (CH3)3 CSO3H + H2SO4

HYDRO CARBON
QUIZRR 21

3. Nitration

Under certain conditions alkanes react with nitric acid, when a hydrogen atom will be replaced
by a nitro-group, NO2. This process is known as nitration. Nitration of the alkanes may be carried
out in the vapour phase between 150Ĉ and 475ĈC, when a complex mixture of mononitroalkanes
is obtaines. The mixture consists of all the possible mononitroderivatives and the nitro-compounds
formed by every possibility of chain fission of the alkane. For example, propane gives a mixture
of 1-nitropropane, 2-nitropropane, nitroethane and nitromethane.

NO2
HNO3
|
CH3 CH2 CH3   CH3 CH2 CH2 NO2  CH3 C HCH3  C2 H5 NO2  CH3 NO2
400C

4. Isomerisation

It is a process by which n-alkane is converted into a branched alkane containing a methyl group
in group in the side chain by heating the n-alkane with AlCl3 ă HCl at 300ĈC.
For example,
AlCl HCl
CH3  CH3  CH2  CH3 
3
300C
 CH3  CH  CH3
n butane |
CH3
isobutane

5. Cracking : (Pyrolysis)

Cracking involves either breaking of C ă C or C ă H bonds or both.

400 ă 600Ĉ C
CH3CH2CH3 CH2 = CH2 + CH4 {C ă C fission}

400 ă 600Ĉ C
CH3 ă CH = CH2 + H2 {C ă H fission}

6. Aromatisation :

Aromatisation of n-alkanes containing six or more carbon atoms into benzene and its homologues
takes place at high temperature (600ĈC) in pressure of Cr2O3 ă Al2O3 as a catalyst.

Cr2 O3 ă Al2 O3 ă3H2


600Ĉ C
Benzene

Cr2 O3 ă Al2 O3 ă3H2


600Ĉ C

n ă octane ethyl cyclohexane


HYDRO CARBON
22 QUIZRR

ALKENES

Structure

Ć The alkene functional group consists of two sp2 hybridised C


atoms bonded to each via a  and a  bond.
R R´´
Ć The  bond is produced by the side-to-side overlap of the
C C
p-orbitals not utilised in the hybrids.
R´ R´´´
Ć The substituents are attached to the C=C unit via  bonds.
Ć The 2 C of the C=C and the 4 atoms attached directly to the
C=C are all in the same plane.

Reactivity

Ć A  bond is a region of high electron density so alkenes are typically nucleophiles.


Ć Alkenes typically undergo addition reactions in which the  bond is converted to two new
 bonds.

Stability

There are 3 factors that influence alkene stability :


1. Degree of substitution : more highly alkylated alkenes are more stable, so tetra > tri > di >
mono-substituted.
2. Stereochemistry : trans > cis due to reduced steric interactions when R groups are on
opposite sides of the double bond.
3. Conjugated alkenes are more stable than isolated alkenes.
4. Degree of Unsaturation

 2n1  2  n2 
DU =  
 2 

where n1 = no. of C-atom


n2 = no. of H atom
Note : (1) If a halogen is attached, then take halogen as H atom
(2) If an oxygen atom is present we do not count it.
(3) If nitrogen is present, for every N atom deduct 1 H atom.
(4) If degree of unsaturation = 2 then there can be :
(a) 2 double bonds
(b) 1 triple bond
(c) one double bond + one cyclic ring

HYDRO CARBON
QUIZRR 23

Example 6

A molecule for C8H12 does not show stereoisomerism. Upon hydrogenation, it absorbs
1 mole of H2. Upon reductive or oxidative ozonysis it gives some symmetrical diketone.
Deduce the structure of A.
Solution :

 2n1  2  n2 
DU =  
 2 

n1 = 8
n2 = 12

16  2  12
 DU = 3
2

Now, since it absorbs 1 mole of H2, hence only one double bond is present. And since on reductive
or oxidative ozonlysis it gives same diketone, therefore the double bond is of the form of

R R
C

R R

Hence, there is one possibility i.e.

(A)

General points regarding stability of rings :

(1)   unstable due to angle strain


(2) Stability increases with increase in the size of ring upto cyclohexane.
(3) Chair form of cyclohexane is the most stable.

HYDRO CARBON
24 QUIZRR
PREPARATION OF ALKENES
From alkynes

(1) Hydrogenation of alkynes

Reduction of alkynes is a useful method for the stereoselective synthesis of disubstituted alkene
is desired, hydrogenation in the presence of LindlarÊs catalyst is commonly used, though
hydroboration followed by hydrolysis provides an alternative approach. Reduction of the alkyne
by sodium metal in liquid ammonia gives the trans-alkene.

Lindlar's catalyst R R´
C C
2

(Pd/Baso4, quinoline)
H

H H
+

(Cis)
R·C·
· C · R´
·
R H
Na C C
Liq. NH3 H R´
(trans)

B2H6 R R´
C C
CH3 COOH
H H

R R´
Wilkinson's Catalyst
C C
Rhcl [(Ph3 P)] H H

(2) Preparation by elimination reaction

I. Dehydrohalogenation
An elimination is the loss of two atoms or groups from a molecule, which will typically result in
the formation of a new  bond.
E.g.

ă
C2H5O
H
 alc. KOH ă
CH2 CH2 ă
CH2 = CH2 + Cl + C2 H5 OH
C2H5O

Cl

IUPAC name : 1-2 elimination


commonname : -elimination

HYDRO CARBON
QUIZRR 25

T ypes of elimination Reaction

(1) E1 elimination
This is an unimolecular elimination. Which means that RDS (Rate Determining step) involves
only one molecule. Also it is the slow ionization to generate a carbocation.
The second step is the fast removal of a proton by the solvent.
In general for on E1 reaction :
Step 1 : Carbocation formation (slow, rate determining)

CH3 CH3
+ ă Slow + ă
H3C C Cl CH3 C + Cl

CH3 CH3

Step 2 : Abstraction of proton (fast)

CH3 CH3
fast
CH3 C+ C
ă
H3C CH2
C2H5 O
CH3 H

(2) Second order elimination (E2)

Generally this elimination occurs with a strong base, which elimates a proton more quickly than
the substrate can ionize.
E2 elimination is bimolecular and takes place in only one step.

C2 H5 O ă
H C2H5O ă H

CH2 CH2 CH2 CH2


ă
Cl  + Cl +
R ă OH + H
H
O
ă R·O
rate = Kr [R ă X][B ]
R
Here solvation will release solvation energy.
Solvation energy +  bond formation energy is enough to break the CăCl bond.

HYDRO CARBON
26 QUIZRR
Saytzeff’s Rule

In this type of elimination reaction the two leaving groups are lost from the adjacent carbon
atoms. One of the leaving groups is the halogen atom and the C-atom from which the halogen
and the C-atom from which the halogen is lost is usually designated as 1 or () carbon atom.
Hydrogen, the other leaving group is lost from the neighbouring carbon which is designated as
2 or () carbon atom. Now, if the alkyl halide has two or more -carbon atoms, two or more
alkenes are possible. According to saytzeffÊs rule, dehydro halogenation of alkyl halides leads to
the formation of that alkene leads the major product which has maximum number of alkyl groups

attached to C = C For example

CH3 CH3 CH3


 alc. KOH
CH3 CH CH CH3 CH3 CH = C CH3 + CH2 = CH CH CH3
 
(major) (minor)
Cl

Note : Ć Use Sayteff rule only in E2 reaction, not in E1 to predict the product.
Ć That product will be formed in major amount which has more stable alkene.

In an E1 reaction, the rate determining step is the loss of the leaving group to form the intermediate
carbocation. The more stable the carbocation is, the easier it is to form, and the faster the E1
reaction will be. Some student fall into the trap of thinking that the system with the less stable
carbocation will react fastest, but they are forgetting that it is the generation of the carbocation
that is rate determining.

Since carbocation intermediates are formed during an E1, there is always the possibility of
rearrangements (e.g. 1, 2-hydride or 1, 2-alkyl shift) to generate a more stable carbocation.

However, if the size of base is increased, it finds it relatively easier to abstract proton from a less
substituted -carbon atom than from more substituted -carbon atom of alkyl halide. Therefore,
less stable alkene becomes the major product . This is known as HoffmannÊs rule.
For example

Br CH2 CH3
CH3

Et3N
(a bulky base)

(major) (minor)

In place of haloalkanes, suphonyl derivatives of alkanes can also be used in the base catalysed
elimination reaction for the preparation of alkenes.

HYDRO CARBON
QUIZRR 27

CH3 O CH3 O
alc. KOH + ă
CH3 CH CH2 O S CH3 CH3 C = CH2 + K O S Ph CH3
isobutene
O O

Hoffman elimination also takes place when the leaving group is poor (In general strong bases
are poor leaving groups).
H

CH2 CH2

Cl leaving group

HF < HCl < HBr < HI


Weak acid
Fă > Clă > Bră > Iă
Strong base

Example 7
Predict the alkene formed in each case.

CH3
(a) Br CH3 H3C
H
but-1-ene

H3C
Br H H CH3 CH3

CH3
+ +
(b) H H3C
CH3 H H
H3C H
but-1-ene trans-but-2-ene cis-but-2-ene
(minor)

H3C H H H

(c) H3 C CH3 +
Br CH3
H CH3 CH3

trans-pent-2-ene cis-pent-2-ene

HYDRO CARBON
28 QUIZRR

CH3
H3C H H CH3
(d) H3C
CH3
Br CH3
H3C H
2-methylpent-2-ene 2-methylpent-1-ene CH3
(major)

CH3
CH3 CH3
H3 C CH3
CH3
(e) H3 C H

Br CH3 CH3
H3 C
H CH3
2, 3-dimethylbut-2-ene 2, 3-dimethylbut-1-ene
(major)

CH3
(f) H3 C CH3 CH3
H3 C CH3
CH3
H

Br CH3 H3C
CH3
H CH3
2, 3-dimethylbut-2-ene 2, 3-dimethylbut-1-ene
(major)

H3C

H3 C
CH3

H CH3

Example 8

Which of the following reaction is correct ?

CH3 CH3
Br alc.
(1)
CH3 KOH

H CH3

HYDRO CARBON
QUIZRR 29

Br
CH3
CH3 alc. KOH
(2)
CH3
CH3
H
Solution :
(1) Reaction is correct as Br and H are in anti planar planes. In (2) Br & H are in same plane.
Therefore elimination canÊt take place in this way.

Instead
Br CH3
CH3 alc. KOH
CH3 CH3

H H

Example 9

Explain which of the following reactions would provide a better synthesis of 2-pentene.

ă CH3 OH
(i) H3C CH3 + CH3 O
Br

Br

ă CH3 OH
(ii) H3C CH3 + CH3 O

Solution :

(i) H3C CH3 H3C CH3


Br Only product

Br

H3C CH3 H3C CH2 + H3C CH3


(ii)

Mixture

HYDRO CARBON
30 QUIZRR
(3) Preparation by Vicinal dehalogenation

Dehalogenation of vicinal dihalides can be effected either by NaI in acetone or by zinc in presence
of acetic acid or ethanol.

Zn dust
CH3 ă CH Br ă CH Br ă CH3 
CH3COOH or
 CH ă CH = CH ă CH
3 3
C2H5OH

NaI
CH3 ă CH Br ă CH2Br 
acetone
CH3 ă CH = CH2

The reaction mechanism involves loss of two halogen atoms in two steps. The two halogen atoms
align themselves at 180Ĉ in the same plane before they are lost.

(i) With NaI in acetone

I
+
Iă X X

ăX ă
C C C C C=C + IX

(ii) With zinc dust


Zn  Zn2+ + 2eă

X
ă
ăX ă ă
C C C C C=C +X

2e ă X X

(4) Acid Catalysed Dehydration of Alcohols

H+
H C C OH C=C +H OH

Alkenes can be prepared by dehydrogenation of alcohol in presence of protonating acid such as


H2SO4 etc.

HYDRO CARBON
QUIZRR 31

Conc. H2SO4
R ă CH = CH2 + H2O
(180Ĉ C)

H3PO4 Or P2O5
R CH2 ă CH2 ă OH R ă CH = CH2 + H2O
(200Ĉ C)

Al2O3
R ă CH = CH2 + H2O
(350Ĉ C)

with 3Ĉ alcohol, reaction always occurs by E1 mechanism.

Example 10

CH3 CH3
| + |
H
CH3  C  OH   CH3  C  CH2  H2 O
| 
CH3

Mechanism :

Reaction occurs in 3 steps

(1) Pr ot onat ion : At t ack by H + ion on ă OH group

CH3 CH3

CH3 C OH CH3 C OH2


+

CH3 CH3
+
H

(2) Dehydration : Removal of water & formation of carbocation slow step

CH3 CH3

CH3 C OH2 CH3 C + + H2 O
+

CH3 CH3

Ć Carbocation can rearrange according to stability

HYDRO CARBON
32 QUIZRR
(3) Deprotonation  Removal of H+ ion

CH3 CH3
H
CH3 C+ CH3 C = CH2
O H
CH3 H

Example 11

CH3
| H+
CH 3  CH  CH 2 OH 

 ?

Solution : CH3 CH3


+
CH3 CH CH2 OH CH3 CH CH2 OH2

ăH2O
+
H
CH3 CH3
+
CH3 C CH3 CH3 CH CH2
+ rearrangement
depronation

CH3

CH3 C = CH2

Example 12
+
OH
H
+ OăH
protonation
H

 dehydration
+

+ H2O
+
rearrangement
More stable carbocation

Ring Expansion

rearrangement + deprotonation
+

More stable
carbocation

HYDRO CARBON
QUIZRR 33

Example 13

+
OH OH2
+
H +

(a)

II
I Correct

(b) (c)
Incorrect

II path is incorrect as carbocation always rearranges to more stable carbocation as carbocation


(b) is equally stable as (a).

Note :
Ć Reaction with 1Ĉ alcohol proceeds by E2 mechanism if steric hindrance is not present.
Ć As shown in previous examples sterically hundred alcohols dehydrate by E1 mechanism.
Ć Reaction is first order as only one molecule is involved in rate determining step.

Some Important Points :

(i) Primary alcohols require the most stringent conditions to undergo dehydration i.e., use of
conc. H2SO4 and high temperature.

(ii) Tertiary alcohols can be easily dehydrated by using 25% H2SO4 at 85ĈC.

Example 14

+ +
CH2 OH CH2 (OH2 ) CH2
+
+ +
H Rearrangement ăH
 ăH2O

This evidence shows the extra stability of six membered ring.

HYDRO CARBON
34 QUIZRR
H ofmann elimination

Hofmann elimination is a process where an amine is reacted to create a tertiary amine and an
alkene by treatment with excess methyl iodide followed by treatment with silver oxide, water and
heat.

ă
H H H OH
R R Ag2O
N
MeI ă R R
R R + I R + R R
+ H2O + ăH2O R CH3
N N N
R R R R R R
CH3 CH3

After the first step, a quaternary ammonium iodide salt is created. After replacement of iodine
by an hydroxyl anion, an elimination reaction takes place to the alkene.
With unsymmetrical amines, the major product is the least substituted and generally the least
stable, an observation known as the Hofmann rule. This is in direct contrast to normal elimination
reactions where the more substituted, stable product is dominant (Saytzeff’s rule).

4. Witting Reaction

This method involves a convenient method of converting aldehydes and ketones into alkenes by
using a special class of compounds called phosphorous yields also called wittig reagent. Primary
or secondary alkyl halide is first treated with triphenyl phosphine the phosphonium halide produced
in the above reaction is converted into phosphorane by adding a strong base like C6H5 Li or
n ă C4H9 ă Li, Phosphorane is stablised by resonance.

R R

CH X + Ph3P R CH PPh3

R´ R´

The Triphenyl group of phosphorane has a strong tendency to pull oxygen atom of the aldehyde
or ketone forming alkene.

R R´´ R´´ R

C PPh3 + O = C C = C + PPh3 = O
0

R´ R´´´ R´´´ R´

(R, RÊ, RÊÊ and RÊÊ may be hydrogen or any alkyl group).

HYDRO CARBON
QUIZRR 35

5. Cleavage of Ethers

Olefins can be formed by the treatment of ethers with very strong bases such as alkylsodium,
alkylithium or sodamide.

  R´Na
C C C= C + RONa + R´H

H OR

The reaction is aided by electron withdrawing groups in the -position.

For example C2H5OCH2CH (COOC2H5)2 forms alkene just by heating without any base at all.


C2H5OCH2CH(COOC2H5)2   CH2 = C(COOC2H5)2 + C2H5OH

The mechanism probably involves a cyclic intermediate

(ă)
(ă) Ph ă CH ă O
B
Ph ă CH2 ă O ă CH2 ă CH3 CH2 = CH2 + Ph ă CH2 ă O
ăBH
H CH2

CH2

6. Pyrolysis of Esters

Thermal cleavage of an ester usually acetate involves the formations of a six membered ring as
the transition state leading to the elimination of acid leaving behind alkene.

H H
H
O
R2C O R2C O R2C
500Ĉ C
+ C
H2C C
H2C C H2C O Me
O Me
O Me

As a direct consequence of cyclic transition state, both the leaving groups namely proton and
carboxylate ion are in the cis position. This is an example of cis elimination.

HYDRO CARBON
36 QUIZRR
REACTION OF ALKENES
(1) Addition Reaction
Rules governing adding reaction.

I. (a) Markonikov addition

+
C C=C · + X Y C C C
Y X

Mechanism for the addition of hydrogen halide to an alkene involves the following two
steps :

Slow
Step 1 : C=C + H X C C
(RDS) +

fast
Step 2 : C C + Xă C C
+

H X H

The above addition follows MarkonikovÊs rule. It states that in the addition of HX to an
alkene, the hydrogen atom adds to the carbon atom of the double bond that has the greater
number of hydrogen atoms or the negative part of reagent adds to carbon having less
number of hydrogen.

For example

CH3 CH2 CH=CH2 + HI  CH3 CH2  CH  CH 3


|
I
The addition of HBr to some alkenes gives a mixture of the expected alkyl bromide and an
isomer formed by rearrangement.

HYDRO CARBON
QUIZRR 37

CH3
H+
CH2 = CH CH CH3 CH3 CH CH CH3
+

CH3

Rearrangement

CH3 CH3

CH3CH2 C CH3 CH3 CHBr CH D


+

CH3
Br ă

CH3

CH3 CH2 C CH3

Br

(b) Anti-markonikovÊs rule :

C C=C · + HBr C C C

Br H

C C C

H Br
antimarkonikov

When addition of HBr takes place in presence of peroxides (RăOăOăR or benzoyl peroxide),
addition occurs in an anti-MarkonikovÊs manner i.e., Br is added to the carbon having large
number of Hăatom.

ROOR
CH2 = CH ă CH3 + HBr   CH3 CH2 CH2 Br

Mechanism

R·O·O·R  2R · O

R · O + H · Br ROH + Br

HYDRO CARBON
38 QUIZRR

Step 1 : CH3 CH = CH2 + Br CH3 CH CH2

Br
(2 radical)

Step 2 : CH3 · CH · CH2 + HBr

CH3 ă CH2CH2Br + Br

Example 15

Why does only Bromine gives anti-MarkonikovÊs addition :


Solution :
Only for HBr, both the steps are exothermic while for HF the second step is highly endothermic,
reflecting the strength of H-F bond and the difficulty of breaking it. For HCl, it is again the
second step that is endothermic while for HI, it is the first step that is endothermic, reflecting the
fact that the energy gained in forming the weak I-C bond is not as much as that lost in breaking
the C=C  bond.

II. (a) SYN


(cis type same side)

C = C + XY C C
X Y

+ X Y H H SYS

X Y
results in cis
product

(b) ANTI
(trans type  opp side)
X
C = C + XY C C
Y

+ X Y H Y
X X
results in trans product.

HYDRO CARBON
QUIZRR 39

I. ADDITION REACTIONS

1. Catalytic Hydrogenation :
Alkenes react with hydrogen in the presence of many reagents such as :
(1) Na in liquid NH3 and alcohol
(2) H2, Pt O2/CH3COOH
(3) H2/Ni at 573 K
(4) H2/Pd-carbon in ethanol
It is a Âsyn-additionÊ reaction.

H2/Pt,Ni or pd
H H

CH3 CH3 CH3 CH3


cis-1,2-dimethyl cyclobexane

D2
H H
(duterium)

H H D D

H H
D2
CH3 · CH = CH · CH3 CH3 · CH · CH · CH3

D D

CH3 H
CH3
C D2
(Meso) D H
C Ni,
CH3 H (SYN)
D H
(Cis)
CH3

CH3 H
CH3
C D2 D H
C (SYN)
H CH3
H D
(Trans)
CH3

HYDRO CARBON
40 QUIZRR
Reducing Agents
ă H2, Ni or Pd or Pt 300Ĉ C
ă H2, PtO2/CH3COOH exclusive for alkenes cannot reduce
N
O 
 
 X 
Pd  C
ă H2 /
(methanol) or ethanol

Note :
ă PtO2/CH3COOH is used exclusively for alkenes (  also)
ă LiAlH4  reduces all except alkenes
NaBH4 / OHă

Specially for C=0

C=0 CHOH

2. Addition of halogen acid HX


3. Addition of H2O / H+
4. Addition of dil H2SO4
* governed by Markonikov

1 a HX
C·C=C· C·C·C·

X H

b HBr
C·C·C·
Peroxide
(anti markonikov) H X

2 H2O (H+ OHă )


+
C·C·C·
H (conc.)
OH H
boil
3 dil. H2SO4 (H2O)
+ ă
C·C·C·
H O SO3 H
OSO3H H

HYDRO CARBON
QUIZRR 41

1 a HX

H X
CH3 CH3

b HBr
Br H Or H H *
Peroxide

H CH3 Br CH3

2 H2O (H+ OHă )


H+ (conc.)
H OH
H CH3 boil
(H2O)
3 dil. H2SO4
+ ă
H O SO3 H
H CH3
H OSO3H

*Anti Markonikov is free radical addition Nothing is clear about cis & trans

(Nothing can be said about sterco chemistry)

Br CH3

Example 16

(1)
HBr +
attack by + Hydride
+ H+ shift
H
ring
expansion

Br + Carbocation +
Attack of Br
rearrangement

HYDRO CARBON
42 QUIZRR
+
(2) CH = CH · CH3 CH · CH2 · CH3 CHBr · CH2 · CH3

HBr

OH OH OH
Carbocation stabilized
by resonance from
benzene ring

(3) H3C CH3 H3C CH3


+
C C CH3 CH3

HBr +

ring expansion

Br
+

Bră

Note : Rearrangement may be avoid by treatment of alkene with mercuric acetate (in THF)
followed by reduction with NaBH4 / NaOH

5. Addition of halogen X2 (in CCl 4)


Note : * the reaction follow anti addition
* Reaction also follows markonikov addition

Br
anti
C · C = C · + Br2 C·C·C·

Br

Some examples :

(1) + Br2 CCl4 H Br

Br H
transă1, 2ă dibromocyclohexane

HYDRO CARBON
QUIZRR 43

(2) + Br2 CCl4 Br


(anti) (trans)
CH3 Br CH3

(3) + Br2 CCl4 CH3 Br


(anti) (trans)
CH3 CH3 Br CH3

Br

(4) CH3 · CH2 · CH = CH2 + Br2/CCl4 CH3 · CH2 · CH · CH2

Br

CH3 Br
CH3
(5) C = CH2 + Br2/CCl4 C · CH2
CH3 CH3
Br

Br

(6) CH3 · CH = CHCH3 + Br2/CCl4 CH3 · CH · *CH CH3


*
Br

NOTE :
Now the compound formed in (4)
Br

CH3 · CH2 · CH · CH2

Br

is optically active which can exist in isomers.

HYDRO CARBON
44 QUIZRR
For a compound having chiral centres, the isomers are
= 2n where n is the no of chiral centres
2n = 4 (n = 2)
CH3 CH3

H Br Br H

Br H H Br

CH3 CH3

(I) enantiomers (II)


(Resmic misture)

CH3 CH3

Br H H Br

Br H H Br

CH3 CH3

(III) (IV)
(Meso structure)
CH3
CH3 H
H Br
C Br2
enantiomers
(anti add.)
C Br H
CH3 H (cis)
CH3

CH3
CH3 H
Br H
C Br2
Meso structures
anti
C Br H
H CH3 (trans)
CH3
Br2
Note : Test of unsaturation
CCl4
(reddish brown)
(bromine water)
HYDRO CARBON
QUIZRR 45

Example 17

Predict the product. Me


H
+ Br2 ?
D
T

Solution :
ă
Br
b a
D Me
+
 + 
T H
+
Br

Here the + I effect of Me stabalizes the + charge on carbon, hence path (a) is more stable.

Example 18

Find the major product in the reaction.


Me

H
D NaCl
+ Br2 ?
T

Solution :
Me ă
Cl Me
H
D H
+ Br2


T D Br

Me Me

Br H H Br
+
T Cl Cl T

D D
Here major product will be of chlorine because Clă is a better nucleophile than Bră

HYDRO CARBON
46 QUIZRR
6. Addition of halogen X2 (in H2O) aqueous solution :

ă 
Br2 / H2 O  HO  Br 

Note : This reaction also follow anti addition and markonikovÊs addition
Br
să s+
1. HO Br
C·C=C· C·C·C·
Br2/H2O
Markonikov add OH
anti addition

2. Br2 H OH
H2O
(anti)
Br H

3. Br2 H OH
H2O
CH3 Br CH3

4. Br2 OH
CH3
H2O
CH3 CH3 Br CH3

7. O xymercuration – Demercuration (reduction)

The reagent used is :


(1) Hg(OAC)2 / THF
<
<

O
||
(CH3 · C· O)2 · Hg (polar solvent)

HYDRO CARBON
QUIZRR 47

(2) NaBH4 / OHă

NOTE : The reaction follows only Markonikov rule

Hg(OAC)2/THF/H2O
C=C C·C O

H O Hg · O · C · CH3 + CH3 · C · OH

(oxy ă mercurate) O

NaBH4/OHă

O
C·C + Hg + CH3 · C · OH
H OH

Hg(OAc) 2 /THF/H2O
C=C C·C
NaBH4 /OHă
H OH

1. Hg(OAc)2/THF/H 2O
2. NaBH4/OHă
H CH3
CH3 H OH

8. Hydroboration / Oxidation of Alkenes

1. BH3
C=C H ă C ă C ă OH
2. H2 O2 /NaOH

Ć Overall transformation : C = C to H ă C ă C ă OH
Ć Reagents (two steps) 1. BH3 or B2H6 then (2) NaOH/H2O2
Ć Anti-Markovnikov, since the B is the electrophile.
Ć Stereoselectivity : Syn since the C-B and C-H bonds form simultaneously from the BH3.
Ć The alcohol is formed over a series of steps involving the B center, with retention of
configuration at the C.

HYDRO CARBON
48 QUIZRR
R eactivity of Borane
Ć Borane reacts with alkenes via a concerted mechanism.
Ć Simultaneous making of C-B and C-H bonds as C=C and B-H break.
H
BH2
Ć Electrophilic B atom adds at the least substituted end of the alkene.
Ć Although a carbocation is not involved, the reaction occurs as if the electrophilic B atom were
making the more stable carbocation.

Mechanism :
Step 1 : The  electrons act as the nucleophilic with the electrophilic B and the H is transferred
to the C with syn stereochemistry.

2R ă CH = CH2 + B2H6  2RCH2CH2BH2

Step 2 : First step repeats twice again.

RCH2CH2BH2 + R ă CH = CH2 (R CH2CH2)2 BH

(R CH2 CH2)2 BH + R ă CH = CH2  (RCH2CH2)3B

Step 3 : Peroxide ion reacts as the nucleophile with the electrophilic B atom.

HăOăOH + HOă  HOOă + H2O

CH2 CH2R CH2 CH2R

B B
RH2CH2C CH2 CH2R RCH2H2C CH2CH2R
ă O OH
OOH

ă
CH2 CH2R OCH2CH2R
ă
OH 2HOO
B ă B
2 OH OCH2 CH2R
RH2CH2C OCH2 CH2R RH2CH2C
OH2

OCH2 CH2R ă
OCH2CH2R
ă OH
B B
RH2CH2CO OCH2 CH2R RH2CH2CO OCH2 CH2R
OH

ăRCH2CH2OH

OCH2 CH2R
2H2O
2RCH2CH2OH + H3BO3
B
RH2CH2CO OH

HYDRO CARBON
QUIZRR 49

For Example :

B2H6 H2O2 /OH H Me

Me OH H
(syn addition)

Note :
Ć If instead of H2O2/OHă we use
Ag then an ether will be formed.

CH3  CH  CH2 


3
 CH3CH2CH2 3 B 
BH Ag
 CH3 CH2 CH2  O  CH2  CH2  CH3

Ć If we use acetic acid, an alkane will be formed

CH3  CH  CH2 


3
 CH3CH2CH2 3 B 
BH 3 CH COOH
 CH3CH2CH3

Example 19

CH3 CH3
D
BD3
OH
(1) H2O2

Here D of BD3 will add to the carbon having least H atoms.

CH3 CH3
H
BD3
OD
(2) D2O2

CH3 CH3
D
Hg(OAC)2 OH
(3) NaBD4, H2O

HYDRO CARBON
50 QUIZRR
8. Hydroxylation (of 2 – OH)

NOTE : follows syn addition

Baeyers reagent
alc
C=C C·C (glycol)
KMnO4
OH OH
(Test of unsaturation)

here purple color of KMnO4 is discharged in the reaction

alc
H H
KMnO4

CH3 OH OH

(Osmium tetroxide)
1. OsO4 /Pyridine
C=C C·C
2. NaHSO3
OH OH

CH3 H
CH3
C (Meso)
(Racemic H OH
Br2 alc
mixture)
C
CCl4 KMnO4
CH3 H H OH
(cis)
CH3

CH3 H
C
Br2 alc (Racemic
(Meso)
CCl4 C mixture)
KMnO4
(anti) H CH3 (Syn)
(trans)

HYDRO CARBON
QUIZRR 51

Example 20

MCPBA
CH3 · CH = CH2 CH3 · CH · CH2
Synaddition

MCPBA = Metachloro Per oxy Acid


+ +
H (if further we add H )

H2O
+
CH3 · CH · CH2OH CH3 · CH · CH2

OH
+

OH

CH3 · CH · CH2

OH

Ć Final product is anti addition product.

9. Oxidation of Alkenes

1. Oxidation by O3 (ozone)
2. Oxidation by hot acidic KMnO4

Ozonolysis :

Ozone reacts vigorously with alkene to form unstable compound called molozonide, which rearranges
spontaneously to form a compound known as ozonide. Ozonides, themselves are unstable and
reduced directly with Zn and water. The reduction produces carbonyl compounds (aldehydes and
ketones) that can be isolated.

Ć Overall transformation : C=C to 2 ï C=O

Ć Ozonolysis implies that ozone causes the alkene to break (-lysis)

• a reducing work-up, either Zn in acetic acid or dimethyl sulfide, (CH3)2S

Ć an oxidising work-up, usually H2O2 (under these conditions, carboxylic acids are obtained
instead of aldehydes)

HYDRO CARBON
52 QUIZRR
Ć It is convenient to view the process as cleaving the alkene into two carbonyls

Ć The substituents on the C=O depend on the substituents on the original C=C

Ć The work-up determines the oxidation state of the products

Reductive work-up

Oxidative work-up

R R R R
O3
O+O
then
R H H2O2 R OH

Example 21

(1)
O
O O
O3 C·H
H3
Zn, AcOH
+ CH3 · C · H
C=O
CH3
O
O O
C · OH
O3 + H2O2
O + CH3 · C · OH

HYDRO CARBON
QUIZRR 53

(2) CH3 CH3 O


O3
CH3 · C = CH · CH3 CH3 · C = O + CH3 · C · H
Zn, AcOH

CH3 O
O3/H2O
CH3 · C = O + CH3 · C · OH

Ć In reductive ozonolysis, break double bond and each double bonded carbon will have one
oxygen atom.
Ć In oxidative ozolysis, aldehydes obtained from reductive ozonolysis will be oxidised to acids
because H2O2 formed is highly oxidising in nature.

2. Oxidation by hot acidic KMnO4

[O]
R ă CH = CH2 
 RCOOH + HCOOH
KMnO4

Here is a comparative list of oxidation by KMnO4 and ozone

(O) O3
RCOOH + HCOOH (R·CH=CH2) RCHO + HCHO
KMnO4 Zn/H2O

R
(O) O3
C = O + CO2 + H2O (R · C = CH2) R · C = O + HCHO
KMnO4 Zn/H2O
R R R

(O) O3
2 RCOOH R ·CH = CHR 2 RCHO
KMnO4 Zn/H2O
(O) O3
RCOOH + R´COOH R · CH = CHR´ RCHO + R´CHO
KMnO4 Zn/H2O

R R R
(O) O3
C = O + ROOH C = CHR C = O + RCHO
KMnO4 Zn/H2O
R R R

HYDRO CARBON
54 QUIZRR
* Terminal alkenes always give formaldehyde on ozonolysis

* dialkylated alkenes always give ketones


Note : HCOOH is easily oxidized into

HCOOH  CO2 + H2 O

* Basically products on left hand side [by KMnO4] can be achieved by further oxidising the
products achieved on right by ozonolysis.

O zonolysis of Cycloalkenes

3 O O
1

3 2
O

4 1 O O
O

O O O O

O O
O

(O)
alc . KMno4 OH
OH
O O

HYDRO CARBON
QUIZRR 55

(O) OH
alc . KMno4
OH

(O)
OH
alc . KMno4
OH
O
O

alc . KMno4
O= =O
OH OH

C ycloalkadienes

3 3
2
2 CHO +
O CHO O O
1

3
3
2 4 O
2 O +
1 5 O
O
6

3
2 4
O O +
1 5 O
O
2
6

HYDRO CARBON
56 QUIZRR
COOH
+ O O
COOH OH OH

COOH
+ O O
COOH OH OH

COOH

2 CH2

COOH

A lkadions

isomeric to alkynes & cycloalkenes

C=C·C=C
CHO COOH

CHO COOH

COOH
KMnO 4 O3
+ 2CO2 + 2CO2 C C·C C 2HCHO + CHO
COOH
CHO

2CO2 + 2H2O C C·C C 2 HCHO + H3C · C · CHO


O C O
CH3 · C · COOH ( ketoaldehyde)
 (Pyruvic acid)
(A keto acid)

HYDRO CARBON
QUIZRR 57

II. Reaction with Carbenes

For carbenes generation we need a source of carbene


We use CH2N2 (diazomethane) or CH2=C=O (Ketene)

CH 2 N2 
 CH2: + N2
(carbene)
Ć Carbene is electron deficient. It adds across the double bond.
CH2
CH3 · CH = CH2 CH3 · CH · CH2

CH2
CH2N2
light
C= C + or C C
CH2CO
CH2
Mechanism :

Step 1 : Attack on double bond

· CH
CH3 · CH · CH3 · CH · CH2 CH3 · CH · CH2
2
·

CH2 CH2 CH2

Addition in Syn

CH3 CH3

CH light CH
+ CH2N2 CH2
CH CH
CH3 H3 C
(cis)

H3 C H3 C
CH Cu2+ CH
+ CH2N2 CH2
CH CH
CH3 CH3
(Trans)

HYDRO CARBON
58 QUIZRR
Ć In presence of potassium tert-butoxide, there is addition of dichloromethylene
(dichlorocarbone), : CCl2, when CHCl3 reacts with alkene.

ă ă1
t ă BuO K
CH3 · CH = CH · CH3 + CHCl3 CH3 · CH · CH · CH3 + t · BuOH + KCl

Cl Cl

first : CCl2 is formed by -elimination (1, 1-elimination) from CHCl3 and tăBuOH

I II. Simmons Smith Reaction

Alkene reacts with CH2I2 in presence of Cu-Zn to form cycloalkane. This reaction does not
actually involve a carbene, but rather a carbenoid, a organometallic species, that reacts like a
carbene.
CH2I2 + : Zn (Cu)  IăCH2ăZnăI (carbenoid)

CuăZn
CH3 ă CH = CH2  CH2 I2  CH3 · CH · CH2

CH2

(methyl cyclopropane)

IV. Hydroxylation

There are a number of reagents that can add two OH groups to alkenes. The two OH groups can
be either added from the same side (syn hydroxylation) or from the opposite side (anti-hydroxylation)

(a) Syn Hydroxylation

Osmium tetroxide (OsO4) adds to alkene to form cyclic osmic ester (2) which can be made to
undergo ready hydrolytic of their Os-O bonds to yield the vic-dio(3).

O O
Os

Me O HO
O HO
H
Me OsO4 Me H H2O Me H
+ (HO)2OsO2
Me H Me H
H
(1) (2) (3)

cis 2ăbutene (1) thus yields the meso butan-2, 3ădiol (3), i.e. the overall hydroxylation is
stereoselectively syn, as would be expected from OsăO cleavage in a necessarily cis cyclic ester.

HYDRO CARBON
QUIZRR 59

The disadvantage of this reaction as a preparative method is expense and toxicity of OsO4.
However, this can be overcome by using it in catalytic quantities in association with H2O2 which
re-oxidises the osmic acid, (HO)2OsO2, formed to OsO4.
Alkaline permanganate, Mn18O4ă (a reagent used classically to test for unsautration), will so
effect stereoselective syn addition and this by analogy with the above, is thought to proceed via
cyclic (cis) permanganic ester. It has not proved possible actually to isolate such species but use of
Mn18O4ă, was found to lead to a vic-diol in which both oxygen atoms were O18 labeled. Thus both
were derived from MnO4ă, and neither from the solvent H2O, which provides support for a
permanganic analogue of (2) as an intermediate, provided that Mn18O4ă undergoes no O18 exchange
with the solvent H2O under these conditions. The disadvantage of MnO4ă for hydroxylation is
that the resultant 1,2ădiol is very much susceptible to further oxidation by it.

(b) Anti-Hydroxylation

Peroxyacids, RCOOH will also oxidize alkenes, e.g. trans 2ăbutene (4), by adding and oxygen
atom across the double bond to form an epoxide (5).
O
O
C
C
ă
R O
R O H
+ H O O
HăO + + RCO2H
Me Me Me
Me H H
Me H Me H
(4) (5)

Epoxides (though uncharged) have a formal resemblance to cyclic bromonium ion intermediates,
but unlike them are stable and may readily be isolated. However, they undergo nucleophilic
attack under either acid or base catalysed conditions to yield the 1, 2ădiol. In either case attack
by the nucleophile on carbon atom will be from the opposite side of the oxygen bridge in (5). Such
attack on the epoxide will involve inversion of configuration.

O H Me H Me
Oă HO
H OH ă H2O
OH OH
Me H
Me Me H Me
H OH ă
(5) (7)

H+
H OH OH
O
+
Me H Me H
H H2O ăH
+
H Me H Me
Me +
Me OH2 OH
H
(6) (7)
OH2
HYDRO CARBON
60 QUIZRR
Attack has been shown on only one of the two possible carbon atoms in (5) and (6), though on
different ones in the two cases. In each case, attack on the other carbon will lead to the same
product, the meso vicădiol (7). By comparing the configuration of (7) with that of the original
alkene by suitable choice of reagent, the hydroxylation of alkenes can be made stereoselectively
anit at will.
For example,

3CH2 = CH2 + 2KMnO4 + 4H2O 3CH2 ă CH2 + 2MnO2 + 2KOH


(dil alkaline KMnO4)
OH OH
(ethylene glycol)

V. Reaction with hot alkaline KMnO 4


O
||
alk KMnO4
CH3 ă CH2 ă CH = CH ă CH3   CH3CH2  C  OH  CH3COOH

Ć While writing the product, where double bond is present in alkene, break it from there.
O
||
Ć If CH is bond to double bond, convert it to  C OH
and other conversions are :
Rectant Product

O
= CH-  ||
 C OH

R O
= |  ||
C  C R

= CH2  CO2

Example 22

CH = CH2 COOH

KMnO4
(1) + CO2

H3C CH2 H3C O

KMnO4 COOH
(2) O + CO2

CH3 CH3

HYDRO CARBON
QUIZRR 61

O
O
O C OH
KMnO4 C OH
(3) ? +
 C OH

Stereochemistry of Addition Reactions

Alkene + X2  anti addition reaction

alkene 
dil. cold

 syn addition
alkaline KMnO4

Type of alkene Type of addition Type of product


cis syn meso
cis anti enantioners/racemic mixture
trans syn enantiomers
mixture of d & l isomers
Trans anti meso i.e., optically inactive

VI. Dimerisation :

Isobutene C4H8 in presence of conc. H2SO4 or H3PO4 is converted into a mixture of two alkenes
of molecular formula C8H16, a dimer of C4H8. This is called dimerisation.

CH3 CH3 CH3 CH3 CH3


H2SO4
CH3 · C = CH2 CH2 = C · CH2 · C · CH3 + CH3 · C = CH · C · CH3
80Ĉ C
(A) CH3 (B) CH3

(A) and (B) are position isomers and hydrogenation will form the same alkane, 2, 2, 4-trimethyl
pentane.

(1) H2SO4 , 
CH = CH2 CH = CH · CH · Ph

CH3
hv
(2) 2CH = CH2 

HYDRO CARBON
62 QUIZRR
VII. Addition of Alkanes – Alkylation

2, 2, 4-trimethyl pentane is used as aviation fuel. It can be obtained by dimersion of isobutene.


Isobutene on reaction with isobutane (alkylation) in presence of H+ gives iso-octane.

CH3 CH3
Conc. H2SO4 or HF
CH3 · C = CH2 + H · C · CH3
0 ă 10Ĉ C

CH3
CH3 CH3

CH3 · CH · CH2 · C · CH3

CH3

Mechanism

CH3 CH3
H+
Step 1 : CH3 · C = CH2 CH3 · C · CH3
+

CH3 CH3 CH3 CH3


+
Step 2 : CH3 · C = CH2 + C · CH3 CH3 · C · CH2 · C · CH3
+

CH3 CH3

Step 3 :

CH3 CH3 CH3 CH3 CH3 CH3

+
CH3 · C · CH2 · C · CH3 + H · C · CH3 CH3 · C · CH2 · C · CH3 + CH3 · C
+

CH3 CH3 CH3 CH3 CH3

VIII. Polymerisation

One of the most important reactions of simple alkenes and conjugated dienes is polymerisation.
The joining together of many small molecules to make very large molecules.

HYDRO CARBON
QUIZRR 63
O2, heat
(1) n CH2 = CH2 pressure · CH2CH2CH2CH2CH2CH2
Or
(· CH2 · CH2 ·)n
Polythelene

peroxide
(2) n CH2 = CH · CH2 · CH ·

Cl Cl n
vinyl chloride Poly vinyl chloride
(PVC)

CH3 CH3
(3)
CH2 = C · CH = CH2 · CH2 · C · CH = CH2 ·

isoprene natural rubber

Example 23

Limonene (A) is a naturally occurring C10H16 hydrocarbon that causes the odour of lemons.
When treated with excess of hydrogen in the presence of platinum catalyst, it takes up two
equivalents of hydrogen. When treated with ozone, followed by zinc in acetic acid, it forms
one mole of formaldehyde (C) and one mole of the tricarbonyl compound (B) shown below.

O O

1. O3 H3C CH3 + HCHO


Limonene
(A) 2. Zn, AcOH H (C)
(B)
O

Propose a suitable structure for limonene if it has one six membered ring.
Solution :

Limonene + 2H2  presence of two double bonds in (A)

1. O
Limonene   (B) (9 carbon) + (C) with 1 carbon
3
2. Zn.ACOH

This shows one (C = C) in an open chain, one (C = C) in a ring structure. We have following
randomness in which (A) has six membered ring. C of (C) is joined to C2 of (B) and C9, and C6
of (B) are joined by C = C bond.

 (C) is O = CH2.

HYDRO CARBON
64 QUIZRR
Hence
O CH2
1. O3
H3C H3C
O 2. Zn, AcOH
O CH3 CH3
H3C
CH3 H
(A) (B)

Thus (A) is

CH2

H3C
CH3

Example 24

Compound X consumes five moles of H2 to form

CH2CH2CH2CH3

It gives white precipitate with ethanoic AgNO3.

O
O O O
O3 CH
X Me S + H · C · C · H + H · C · C · O · H + H · C · O ·H
2 H
O
O O O
Solution :
5H2
Compound X Pt
CH2CH2CH2CH3

AgNO3
At OH

X must have five - bonds


X must have terminal alkyne

HYDRO CARBON
QUIZRR 65

Ozonolysis products of X are :

O O O O O

H · C · CH2CH2 · C · C · H + H · C · C · H + H · C · C · OH + H · C · OH

O O O

6 carbonyls  3 alkenes, 2 carboxylic acids  1 alkyne.

Hence X is

CH

HYDRO CARBON
66 QUIZRR

A LKYNES
Functional group suffix = ă yne (review)
Disubstituted alkynes, R-CC-R´, are described as „internal‰ alkynes
Monosubstituted alkynes, R-CC-H, are described as „terminal‰ alkynes.

1. Stability :

Ć As with alkenes, the more highly substituted internal alkynes are more stable.
Ć By comparing thermodynamic data of alkynes and alkenes, it can be seen that the „extra‰
 bond in an alkyne is weaker than the alkene  bond.
Ć The alkyne functional group consists of two sp hybridised C atoms bonded to each other via
one  and two  bonds.
Ć The 2  bonds are produced by the side-to-side overlap of the two pairs of p-orbital not
utilised in the hybrids.
Ć The substituents are attached to the CC via sigma bonds.
Ć The 2 C of the CC and the 2 atoms attached directly to the CC are linear.
Ć Since alkynes are linear, they cannot exist as cis-/ trans-isomers.

2. Physical Properties

Ć As with hydrocarbons in general, alkynes are non-polar and are insoluble in water but
soluble in non-polar organic solvents.
Ć Liquified acetylene or when compared, in explosive, burns with luminous smoky flame due
to high carbon content, hence used for lighting purpose.
Ć In general bond length increases in the order

CC < C = C < C ă C

3. Reactivity

Ć The  bonds are a region of high electron density so alkynes are typically nucleophiles.
Ć Alkynes typically undergo addition reactions in which one or both of the -bonds are
converted to new  bonds.
Ć Terminal alkynes, R ă C  C ă H, are quite acidic for hydrocarbons, pKa = 26
Ć Deprotonation of a terminal acetylene gives an acetylide ion.
Ć The acetylide ion is a good nucleophile and can be alkylated to give higher alkynes.

HYDRO CARBON
QUIZRR 67

4. Acidity of Terminal Alkynes

B:ă
 RăC  C:ă  B ă H
R ă C  C ă H 

Terminal alkynes are unusual for simple hydrocarbons in that they can be deprotonated (pKa =
26) using an appropriate base (typically NaNH2, pKa = 36) to generate a carbanion which can
function as a C centered nucleophile and so allow for the formation of new C-C bonds by reacting
with C centered electrophiles (such as alkyl halides).

·C·CăH C=C ·C CăH


H
p Ka = 62 p Ka = 45 p Ka = 26

In order to appreciate what makes the terminal alkyne more acidic than most other hydrocarbons,
we should look at the stability of the conjugate base (i.e., the carboanion).

ă
· C · Că sp3 C = Că ·C C sp

For each type of carbanion shown, the nature of the hybrid orbital containing the electron pair
is important. Increased s character (sp = 50%, sp2 = 33% and sp3 = 25%) implies that the alkyne
sp orbital is closer to the nucleus and so there is greater electrostatic stabilisation of the electron
pair. Therefore the conjugate base of the alkyne is the most stable and the most readily formed.
However the terminal alkyne C-H bond is not strongly acidic and a strong base, such as the amide
ion, NH2ă, is required to form the carbanion.

Preparation of Alkynes

1. Dehydrohalogenation

X X
NaNH2
·CăC· ·C C · + 2H ă X + 2NH3
in NH3
H H or alcKOH

H X
NaNH2
·CăC· ·C C · + 2H ă X + 2NH3
in NH3
H X or alc. KOH

Reaction Type : Elimination (E2)

HYDRO CARBON
68 QUIZRR
Ć The two  bonds of an alkyne can be formed using two consecutive elimination reactions.
Ć The leaving groups are usually halides (esp. Br or Cl)
Ć Reagent : usually NaNH2 (a strong base)
Ć Either geminal (1, 1-) or vicinal (1, 2-) dihalides can be used.
Ć Since 1,2-dihalides can be prepared by addition of X2 to an alkene, an alkene can be
converted into an alkyne in two steps.
Ć These reactions are typically E2 reactions and occur via an alkenyl halide.
Ć The strong base is needed in order to cause elimination of the alkenyl halide.
Ć 1,2-dihalides vicinal when treated with alcoholic KOH give alkynes by loss of two molecules
of hydrogen halide, the intermediate being vinyl halide.

Br H
alc. KOH alc. KOH
R·C·C·H R·C=C·H R·C C·H
·HBr ·HBr

H Br H Br

Ć The use of alcoholic KOH gives rise to side reactions and causes the migration of the C=C
bond to the centre of the molecule.

M echanism

H H
R H
ă ă
H·N + R·C·C·R C=C + NH3 + Br
Br R
H Br Br

R H
ă ă
C=C + N·H R·C C R + NH3 + Br
Br R
H

In this reaction elemination of the first molecule of HX results in the formation of vinyl halide
ă an alkene with halogen bonded to one of the carbons of the double bond. We take strong base
like NH2 to make the step II easier for elimination.

HYDRO CARBON
QUIZRR 69

2. Industrial source {Hydrolysis of carbides}

CaC2 + 2H2O  Ca(OH)2 + HC  CH


calcium carbide ethyne
ă ă
Calcium carbide has doubly negative carbide ion (: C  C :) which is strongly basic and reacts with
water to form acetylene.

Ć Mg2 C3 + H2O  Mg (OH2) + CH3ăCCH


propyne

Mg2 C3 contain C34  (ăC  CăC3ă)

1500C
Ć 6CH4 + 2O2 
cracking
 2CH  CH + 2CO + 1 H
2 2

3. Kolbe’s Method

Electrolysis of concentrated solution of sodium or potassium salt of maleic or tumaric acid gives
acetylene at anode.

CHCO2 Na + 2H O CH
||  2  ||| + 2CO2 + 2NaOH + H2
CHCO2Na Current
CH

4. Dehalogenation of tetrahalide or Trihalide

X H
alcohol
R · C · C · H + 2Zn R·C C · H + 2ZnX2

H X


CHX3 + 6Ag + X3CH   HC  CH + 6AgX

5. Alkylation of acetylene and Terminal Alkynes

Ć This method is used to prepare higher alkynes


Ć Terminal alkyne is one that has one ă C  CH at the end of the chain.

HC  CH + Na 
Liquid NH3  RX
 HC  C Na   HC  CR

RC  CH + Na 
Liquid NH3  R´X
 RC  C Na  RC  CR ´
terminal alkyne

HYDRO CARBON
70 QUIZRR

(i) Na Liquid NH
CH3C  CH 
3  CH C  CCH CH
3 2 3
(ii) CH3CH2I
2 pentyne

Ć Above reaction shows that acetylinic and terminal H are reactive hence salt formation is
easier.
Ć RX and R´X should be 1Ĉ alkyl halides, since higher 2Ĉ and 3Ĉ halides give mainly alkenes
when they react with sodium salt of alkynes.

CH3 C  C  Na   CH3  CH  CH2  CH3  CH3  C  CH  CH3  CH  CH  CH3  NaCl


|
Cl

Alkylation can also be done using grignard reagent

HC  CH + CH3ăMgăBr  CH  CăMgăBr + CH4

HC  CăMgăBr + CH3Br  HC  CăCH3 + MgBr2

Example 25

Outline a synthesis of propyne from isopropyl or propyl bromide. The needed vic-dihalide
is formed from propene, which is prepared from the alkyl halides.
Solution :

CH3 CH 2CH2 Br 
n-Propylbromide 
 alc. Br2 alc. NaNH2
or  CH3  CH2  CH2   CH3CHBrCH2 Br CH3 CH  CHBr   CH3C  CH
KOH KOH
CH3CHBrCH3 
Isopropylbromide 

Example 26

Synthesize the following compounds from HC  CH and any other organic and inorganic
reagents (do not repeat steps) :
(a) 1-pentyne, (b) 2-hexyne
Solution :
 +
(a) NaNH CH CH CH I
H  C  C  H 
2
 H  C  C:Na 
3 2 2  H  C  C  CH CH CH
2 2 3

+  +
(b) CH I NaNH
Na :C  C  H 
3  CH  C  C  H 
3
2
 CH3  C  C : Na
CH CH CH I

3 2 2  CH  C  C ă CH CH CH
3 2 2 3

HYDRO CARBON
QUIZRR 71

6. Alkylation of Alkynes

1.NaNH
RăC  CăH 
2  RăC  ăR'
2.R '  X

Ć Terminal alkynes are usual for simply hydrocarbons in that they can be deprotonated
(pKa = 26) using an appropriate base (typically NaNH2, pKa = 36) to generate a carbanion.
Ć 3Ĉ alkyl halides are more likely to undergo elimination.
Ć One or both of the terminal H atoms in ethylene (acetylene) H-CC-H can be substituted
providing access to monosubstituted (R-CC-H) and symmetrical (R = R´) or unsymmetrical
R  R´) disubstituted alkynes R-CC-R´.

Mechanism

Step 1 : An acid/base reaction : The amide ion acts ă


CH3 · C C·H NH2
as a base removing the acidic terminal H
to generate the acetylide ion, a carbon
nucleophile.
ă
CH3 · C C NH3
Step 2 : A nucleophilic substitution reaction. The
carbanion reacts with the electrophilic CH3CH2 · Br
+ ă
carbon in the alkyl halide with loss of the
leaving group, forming a new C-C bond. ă
CH3 · C C · CH2CH3 Br

7. Ozonolysis of alkynes

Ozonolysis of alkynes generally gives an acid anhydride or diketone product, not complete
fragmentation as for alkenes. A reducing agent is not needed for these reactions. The exact
mechanism is not completely known. If the reaction is performed in the presence of water, the
anhydride hydrolyzes to give two carboxylic acids.
O O
O3
R·C C · R´

O3 R R´

O O H2O O O

O
R R´ R OH HO R´

However, ethyne behaves differently. Ozonolysis followed by oxidative hydrolysis of ethyne gives
a mixture of glyoxal and formic acid.
1. O
HC  CH 
3 
2. H2O CHOăCHO + HCOOH

HYDRO CARBON
72 QUIZRR
8. Halogenation of Alkynes

·C C · + X2 C=C

Reaction type : Electrophilic Addition

Ć Overall transformation : CC to X-C=C-X (and potentially to X2C-CX2)


Ć Reagent : normally the halogen (e.g. Br2) in an inert solvent like methylene chloride,
CH2Cl2.
Ć Addition of X2 is not relevant since all the new bonds are the same, C-X.
Ć Reaction proceed via cyclic halonium ion.
Ć Stereoselectivity : anti since the two C-X bonds form in separate steps one from X2 the other
X ă.
Ć For HBr, care must be taken to avoid the formation of radicals as the alternate radical
addition occurs with opposite regiochemistry.

Br
HBr
|
CH3  C  CH   CH3 C  CH2  CH3 CH  CH2  Br
peroxides
major

9. Reaction of Alkynes with Hydrogen Halides

H X

·C C · + H ·X C=C

H X

·C C · + H ·X ·C·C·
excess
H X

Reaction type : Electrophilic Addition

Ć When treated with HX alkynes form vinyl halides.


Ć Hydrogen halide reactivity order : HI > HBr > HCI > HF (paralleling acidity order).
Ć Addition is predicted by MarkovnikovÊs rule with the H adding to the C with the most
H already present.

HYDRO CARBON
QUIZRR 73

Br
HBr
|
CH3  C  CH  CH3 C  CH2  CH3CH  CH2  Br
major

Ć Reaction proceeds via protonation to give the more stable carbocation intermediate.
Ć Not stereoselective since reaction proceeds via planar carbocation.
Ć In the presence of excess HX, a second addition can occur to the product alkene giving a
geminal dihalide.

Br Br
| |
HBr
CH3 C  CH2  CH3 CCH3
|
Br

Example 27
OH

dil. H2SO4
(1)
+2
Hg

OH O

dil. H2SO4
(2) +2
Hg

D2O, HgSO4 + H2SO4


(3) C CH C · CHD2

10. Hydration of Alkynes

dil H2SO4
(1) CH CH 2+
H·C=C·H
Hg
H OH
vinyl alcohol

tautomerism

CH3 · C · H

HYDRO CARBON
74 QUIZRR
dil H2SO4
(2) CH3 · C CH 2+
CH3 · C = CH2
Hg
OH enolic

tautomerism

CH3 · C · CH3

O
Ć Intermediate is enolic compound that tantomerises to an aldehyde or keto compound.

Mechanism :
..
H2O OH H
2+
Hg
R·C C·H R·C=C·H C=C

Hg
2+ R Hg+

H3O+

R OH H

C · CH3 C=C
O R H

REACTIONS

1. Hydrogenation of Alkynes

Lindlar's
·C C· HăH C=C
catalyst
H H

Reaction Type : Electrophilic Addition


Ć Alkynes can be partially reduced to cis-alkenes with H2 in the presence of poisoned catalysts.
(eg. Pd/CaCO3 / quinoline which is also known as LindlarÊs catalyst).
Ć Alkynes when reduced with lindlars catalyst gives nearly 100% cis isomer while Na in liquid
ammonia gives nearly 100% trans isomer.
Ć Alkynes can be reduced to alkanes with H2 in the presence of catalyst (Pt, Pd, Ni etc.)
Ć The new C-H  bonds are formed simultaneously from H atoms absorbed onto the metal
surface.
HYDRO CARBON
QUIZRR 75

Ć The reaction is stereospecific giving only the syn addition product.


Ć The “poisoned” catalyst prevents over-reduction, which would give the alkane by reducing
the alkene.

H H
Lindlar's catalyst
C=C
H3C CH3
(cis)
CH3 · C C · CH3 + H2
H CH3
Na in
C=C
Liq. NH3
H3C H
(trans)

2. Addition of Boron Hydrides

Diborane, the simplest hydride of boron reacts with alkyne to form trialkenylborane. Diborane
splits into two BH3 units and the addition of BH3 takes place following MarkownikoffÊs rule. The
addition continues as long as hydrogen is attached to boron atom.

2RăCCH + B2H6  2RăCH=CHăBH2

RăCCH + RăCH=CHăBH2  (RăCH=CH)2BH

R-C CH + (RăCH=CH)2BH  (RăCH=CH)3B

Trialkenylborane on hydrolysis gives alkene.

CH COOH
(RăCH=CH)3B 
3
hydrolysis
 3RăCH=CH + B(OH)
2 3

Internal alkynes give rise to alkenes where geometrical isomerism is possible. Hydroboration
followed by hydrolysis of alkynes gives cis alkene as the major product.

R R
CH3CO2H
R·C C · R + B2H6 (RCH = CR)3B C=C (Cis)

H H

HYDRO CARBON
76 QUIZRR
Oxidation of trialkenylborane with alkaline H2O2 results in the formation of carbonyl compounds.
Terminal alkynes give rise to aldehydes whereas internal alkynes give rise to ketones.

OH
|
 R  CH  CH 3 B 
H2O2 /NaOH tautomerizes

 R  CH  CH  
 R  CH2  CHO

OH O
| ||
 R  CH  CR 3 B 
H2O2 /NaOH tautomerizes

 R  CH= C  R  
 R  CH2  C  R

3. Dimerisation
Acetylene dimerises when treated with a mixture of Cu2Cl2 and NH4Cl to give vinyl acetylene.
Cu Cl  NH Cl
2CH  CH 
2 2 4  CH  CH  C  CH
2

The dimer undergoes addition reactions just like any other unsaturated hydrocarbon. The addition
reaction preferably takes place at the triple bond and not at the double bond inspite of the fact
that alkynes are less reactive than alkenes towards electrophilic addition reactions. For example,
addition of HCl to vinyl acetylene gives chloroprene.

Cl
|
CH2  CH  C  CH  HCl  CH 2  CH  C  CH2
Chloroprene 

4. Oxidation
Alkynes are oxidised by hot alkaline KMnO4, which causes cleavage of ăCCă resulting in the
formation of salts of carboxylic acids. The salts on acidification are converted into acids. Internal
alkynes give mixture of carboxylic acids while terminal alkynes give a carboxylic acid and the
terminal C-atom is oxidised to CO2 and H2O.

(i) KMnO / OH / 


CH3  C  CH 
4

 CH3COOH  CO2  H2 O
(ii ) H

(i ) KMnO / OH / 
CH3  C  C  CH2CH3 
4


 CH3COOH  CH3CH2COOH
(ii) H

5. Polymerisation Reactions
(i) Hydrochloric acid adds to acetylene in the presence of Hg2+ ion as catalyst to form vinyl
chloride. Polymerisation of vinyl chloride results in the formation of polyvinyl chloride (PVC).
 
Hg 2+  
Polymerization
HC  CH  HCl 
 CH2  CHCl 
 CH2  C H  
 | 
 Cl  n
(PVC)

HYDRO CARBON
QUIZRR 77

(ii) Addition of HCN to ethyne is catalysed by Cu2Cl2 in HCl. The product obtained is acrylonitrile,
which on polymerisation gives polyacrylonitrile (PAN).

 
CuCl2 
Polymerization 
HC  CH  HCN   CH 2  CH  CN 
  CH2  C H  
HCl
 | 
 CN  n

(PAN)
Acrylonitrile is also used in the manufacture of a synthetic rubber called BuNaăN (a copolymer
of butadiene and acrylonitrile) and a thermoplastic called ABS (a terpolymer of acrylonitrile,
butadiene and styrene).
(iii) Acetic acid adds to ethyne in the presence of Hg2+ ion to give vinyl acetate, which is used
as monomer in the preparation of polyvinyl acetate (PVA).
2+
Hg
HC CH + CH3COOH CH2 = CHOOCCH3
(vinyl acetate)

6. Reaction with hot iron tube

Ć Acetylene when passed over red hot iron tube forms benzene.

red hot
3CH CH
Fe

This reaction involves a cyclic transition state in which 8eă participate. Such compounds have
aromatic character as aromatic compounds are relatively more stable. So these transition states
are more stable.
CH
CH
CH

CH
CH
CH
CH3
e.g.
Red hot
3CH3 · C CH
Fe
CH3 CH3

HYDRO CARBON
78 QUIZRR
7. Addition of Hypohalous acid

Alkynes react with hypohalous acid in the molar ratio of 1 : 2 to give dihalo Ketones.
Acetylene forms dihaloaldehyde

HOX
R·C C · H + HOX R · C(OH) = CHX
O
ăH2O
R · C · CHX2 R · C(OH)2 · CHX2

Test of Terminal Alkynes

(1) Terminal alklynes give silver mirror in reaction with ammonium solution in silver nitrate,
i.e., ammonium silver nitrate.

CH3 · O CH + AgNO3 + NH4OH CH3 · C CAg + NH4NO3 + H2O

or silver mirror
Ag(NH3)2+ (white ppt)

Ć This test is terminal alkyne.

(2) Another test for terminal alkynes :


They give red ppt in reaction with Cummonium solution in cupurous chloride, i.e., ammonium
cupurous chloride :

CH3 · C CH + CuCl + NH4OH CH3 · C CCu + NH3Cl + H2O


red ppt

This is replaced by Cu.

Example 28

2-butyne undergoes following reactions in steps as indicated. Identify A to H.

H2 D2/Pt
CH3C CCH3 A B
Ni2B

Na/EtOH D2/Pt
C D
H2 Br2
E F
Ni2B
Br2 H2
G H
Ni

HYDRO CARBON
QUIZRR 79

Solution :
A is obtained by syn addition

H3C CH3
C C
A :
H H
Cis

B is also obtained by syn addition

H H

B : H3C · C · C · CH3

D D
meso

C is obtained by anti addition

H CH3

·C
C :
H3C H
trans

D is mixture of d ă and l ă (racemic mixture)

H D

D : H3C · C · C · CH3 d ă and l - (racemic)

D H

E is obtained by syn addition


E : as A

Allylic systems

The position adjacent to C=C often enhanced reactivity compared to simple alkanes due to the
proximity of the adjacent  system. Such positions are referred as „allylic”. Recall that the term
„vinylic” is used to described the atoms directly associated with the C=C unit.

1. Allylic carbocations

The  system of a double bond can stablise an adjacent carbocation by donating electron density
through resonance. Remember that delocalising charge is a stabilising effect. This stablilisation is
equivalent to that of two alkyl groups, so the alkyl cation has similar stability to the 2-propyl cation.

HYDRO CARBON
80 QUIZRR

+ +
1+ 1+
+ + 2 2
resonance hybrid

Note that in the two resonance forms of the allylic cation, the positive charge is located on the
terminal carbon atoms and never on the middle carbon. This is reflected in the resonance hybrid.
Note that either of the carbons with +ve charge could be attacked by a nucleophile.
Due to the stability of these allylic cations, they are readily formed as intermediates during
chemical reactions, for example SN1 reactions of allylic halides.

2. Allylic radicals

resonance hybrid

The  system of a double bond can also stabilise an adjacent radical through resonance. Remember
that delocalising the radical is a stabilising effect.
Due to the stability of these allylic radicals, they are readily formed as intermediates during
chemical reactions, for example allylic halogenation.

Reactivity

Ć Allylic bonds are often weaker and are easily broken, for example compare the bond
dissociation energies :

H
H 410 kJ/mol (98 kcal/mol)

H
H 368 kJ/mol (88 kcal/mol)

Ć The stability of the allylic radical can be utilised in the preparation of allylic halides
(esp. -Cl and -Br)
Ć Allylic halides readily undergo substitution reactions via either SN1 or SN2 pathways.

HYDRO CARBON
QUIZRR 81

3. Radical Halogenation of Allylic systems

H X
light or heat
X·X+ C C +H·X
X = Br, Cl

Reaction type : Radical Substitution


Ć When treated with Br2 or Cl2, radical substitution of allylic C-H generates the allyl halide
and HX.
Ć The process is very similar to that of alkanes.
Ć Reaction proceeds via an radical chain mechanism which involves radical intermediates.
Ć The stability of the allylic radical (due to resonance) favours substitution at the allylic
position.
Ć N-bromosuccinimide (NBS) can be used as an alternative source of Br2.

N · Br

Radical Chain Mechanism For Allylic Bromination

Step 1 (Initiation) Heat or uv light causes the weak halogen bond to undergo homolytic cleavage
to generate two bromine radicals and starting the chain process.

Br · Br Br Br

Step 2 (Propagation) :
(a) A bromine radical abstracts a hydrogen to form HBr and an allyl radical, then
(b) The allyl radical abstracts a bromine atom from another molecule of Br2 to form the allyl
bromide product and another bromine radical, which can then itself undergo reaction 2(a)
creating a cycle that can repeat.

Br H · CH2CH CH 2 H · Br CH2CH = CH 2

Br · Br CH2CH= CH2 Br Br · CH2CH = CH 2

Step 3 (Termination) : Various reactions between the possible pairs of radicals allow for the
formation of Br2 or the product, allyl bromide.

HYDRO CARBON
82 QUIZRR

Br Br Br · Br

Br CH2CH CH2 Br · CH2CH = CH2

N-Bromo succinimide (NBS)

NBS is a reagent which provides a low concentration of bromine. Hence, the most reactive
hydrogen is replaced by Br.

Example :

NBS
CH3  CH  CH2  CH2 Br  CH  CH2

Here CH3  C  CH2 is not formed, since the intermediate allylic free radical formed is very stable.
|
Br

Ć
CH2ăCH=CH2  CH2=CHăCĆH2

Other examples

CH3 CH3
Br
NBS

NBS

Br
Important : At high temperature, substitution takes place, while at low temperature addition
takes place.

Cl2
Example : CH3  CH  CH2   Cl  CH 2  CH  CH2
 high temp. 

4. Substitution Reactions of Allyl Halides

LG Nu
Nuă C C + LGă

Ć Allyl chlorides, bromides and iodides are good substrates for substitution reactions.
HYDRO CARBON
QUIZRR 83

Ć A variety of nucleophiles can used to generate a range of new functional groups.


Ć The process can be complicated by the allylic rearrangement where the nucleophile can
attack either of the deficient sites.

H2O OH H2O Cl

Cl Na2CO3 HO and Na2CO3

85% 15%

+ +

3Ĉ 1Ĉ
Ć The 3Ĉ alcohol is the major product.

Preparation of Conjugated Dienes

Ć Dienes can be prepared by elimination reactions of unsaturated alcohols and allkyl halides.
Ć The outcome of eliminations typically favours the more stable product,
Ć Since conjugated dienes are more stable than isolated dienes, the formation of the conjugated
diene is usually favoured over the isolated diene unless the structure prevents the formation
of the conjugated system.

OH Br
KHSO4 KOH
heat heat

Reaction of Dienes

In general terms, dienes undergo electrophilic addition reactions in a similar fashion to alkenes
(review)
E E
+ ă ă
+ Nu
Nu · E
Nu
However, in a little more detail :
Ć Conjugated dienes undergo addition but the proxmity of the conjugated C=C influences the
reactions.
Ć Isolated dienes react just like alkenes
Ć Cumulated dienes react more like alkynes (after all, both have sp hybridised C atoms)
Our attention here will focus on conjugated dienes.
Ć The  bonds are a region of high electron density so dienes are typically nucleophiles.
Ć Dienes react with electrophiles (e.g. H+, X+)

HYDRO CARBON
84 QUIZRR
Ć Dienes can undergo addition reactions in which one or both of the  bonds are converted to
new stronger  bonds.
Ć Overall reaction : Electrophilic addition

1. Addition of Hydrogen Halides to Dienes

Conjugated dienes undergo addition reactions in a similar manner to simple alkenes, but two
modes of addition are possible.
These differ based on the relative positions of H and X in the products :

1 1 3 X
2 +
H·X +
H H 2 4
X
Direct or 1,2-addition Conjugate or 1,4-addition

Direct H-X adds „directly‰ across the ends of a C=C


Conjugate H-X adds across the ends of the conjugated system

The numbers 1,2- and 1,4- denote the relative positions of H and X in the products.
The distribution of the products depends on the reaction conditions as shown by the example
below :

HBr
81% 19%
ă80ĈC
Br Br

HBr
44% 56%
20Ĉ C
Bră Bră

+
H
+ H + H

At low temperature, the reaction is under kinetic control (equilibrium reversible conditions) and
the major product is the more stable system (note the more highly substituted alkene). This is
supported by the fact that heating pure samples of either 3-bromo-1-butene (direct addition
product) or 1-bromo-2-butene (conjugate addition product) gives the same 15 : 85 ratio of
3-bromo-1-butene to 1-bromo-2-butene.

HYDRO CARBON
QUIZRR 85

At high temperature, the reaction is under thermodynamic control (equilibrium reversible


conditions) and the major product is the more stable system.

2. Addition of Halogens to Dienes

Like the addition of hydrogen halides to conjugate dienes, halogoens add to dienes via direct and
conjugate addition pathways :

1 1 3 X
2 +
X·X +
X X 2 4
X
Direct or 1,2-addition Conjugate or 1,4-addition

The major products are usually the more stable, conjugate addition products with the more stable
E configuration of C=C.

3. Diels-Alder Reaction

Ć The Diels-Alder reaction is a conjugate addition reaction of a conjugated diene to an


alkene (the dienophile) to produce a cyclohexene.
Ć The simplest example is the reaction of 1,3-butadiene with ethene to form cyclohexene :

+
heat

Ć The analogous reaction of 1,3-butadiene with ethyne to form 1,4-cyclohexadiene is also


known :

+
heat

Ć Since the reaction forms a cyclic product, via a cyclic transition state, it can also be described
as a “cyc loaddition”.
Ć The reaction is a concerted process :

Ć Due to the high degree of regio-and stereoselectivity (due to the concerted mechanism), the
Diels-Alder reaction is a very powerful reaction and is widely used in synthetic organic
chemistry.

HYDRO CARBON
86 QUIZRR
Ć The reaction usually thermodynamically favourable due to the conversion of 2 ăbonds into
2 new stronger ăbonds.
Ć The two reactions shown above require harsh reaction conditions, but the normal Diels-
Alder reaction is favoured by electron withdrawing groups on the electrophilic
dienophile and by electron donating groups on the nucleophilic diene.
Ć Some common examples of the components are shown below :

Dienes

O O
CO2Me
CN CO2Me CO2Me
Dienophiles
O
CO2Me
MeO2C
O CO2Me

Stereoselectivity :
Ć The Diels-Alder reaction is stereospecific with respect to both the diene and the dienophile.

Ć Addition is syn on both components (bonds form from same species at the same time)

Ć This is shown by the examples below :


cis-dienophile gives cis-substituents in the product.
trans-dienophile gives trans-substituents in the product.

CO2Me CO2Me

CO2Me CO2Me

If both substituents on the diene are Z, then both end up on the same face of the product.

CO2Me CO2Me

MeO2C CO2Me

If substituents on the diene are E and Z, then they end up on opposite faces of the product.

HYDRO CARBON
QUIZRR 87

Cyclic dienes can give stereoisomeric products depending on whether the dienophile lies under or
away from the diene in the transition state. The endo product is usually the major product (due
to kinetic control).

O
O
+ O O
O O
O endo O exo
O

O
O O O
O O O O
O
O O O

Diene and dienophile aligned directly Diene and dienophile staggered with
over each other gives the endo product respect to each other gives the exo
(dienophile under or in = endo) product (dienophile exposed or out = exo)

Example 29

1 1
2 6 2
(1) + O 6 O
3 5 3 5
4 4

Diene Dienophate

COOH
1 1 COOH
6C 6
2 2
(2) +
3 3 5
5 C
4 4 COOH

Example 30

Dehydrohalogenation of 3-bromohexane gives a mixture of cis-2-hexene and trans-2-hexene.


How can this mixture be converted to pure
(a) cis-2-hexene ? (b) trans-2-hexene ?

HYDRO CARBON
88 QUIZRR
Solution :
Relatively pure alkene geometric isomers are prepared by stereoselective reduction of alkynes.

(a) Hydrogenation of 2-hexene with LindlarÊs catalyst gives cis-2-hexene.

Br2 2NaNH2
CH3CH = CHCH2CH2CH3 CH3CH · CHCH2CH2CH3

Br Br

H3C CH2CH2CH3
H2
CH3 CCH2CH2CH3 C=C
Lindlar's catalyst
H H
cis - 2-Hexene

(b) Reduction with Na in liquid NH3 gives the trans product.

H3C H
Na
CH3C CCH2CH2CH3 C=C
NH, (1)
H CH2CH2CH3
cis - 2-Hexene

Example 31

Use HC  CH as the only organic reagent to prepare

(a) (E) ă 3ăhexene and

(b) (Z)ă3ă henene.

Solution :

 HC  C  Na 
Na
HC  CH  ...(1)

H HCl
HC  CH 
2  CH  CH  CH  CH Cl
2 2 3 2 ...(2)

Combine (1) and (2)

HYDRO CARBON
QUIZRR 89

CH  Că Na+  CH3 CH2 Cl  HC  CăCH2 ă CH3

(i) Na
(ii) CH3CH2Cl

H3CH2CC CCH2CH3

lindlar catalyst / H2 Na / liq NH3

H3C · CH2 CH2 · CH3 H CH2CH3

C C C C

H H H3CCH2 H

(Z) - 3-hexane (E) - 3-hexane

In the above reaction acetylene is the only organic reagent used.

Example 32

Deduce the structural formula of a compound, C10H10(A), that gives ÂBÊ as the only organic
compound,

HOOCCH 2 CHCH 2 COOH


| (B)
CH 2 COOH

on oxidative cleavage, C10H10 is the molecular formula of the organic compound.

Solution :
The degree of unsaturation in the compound is

Number of hydrogen
= (number of carbon) ă 1
2

10
= (10 + 1) ă
2

= 11 ă 5 = 6
This compound contains 6 bonds on oxidative cleavage it is producing only one compound, so all
the multiple bonds should be at the terminal atom.

HYDRO CARBON
90 QUIZRR
+
KMnO4 /H
HC CH · CH2 · CH · C CH HOOC · CH2 · CH · CH2COOH

(A) CH2 · C CH (B) CH2COOH

Example 33

Identify (a) the chiral compound C, C10H14, that is oxidized with alk. KMnO4 to Ph COOH,
and (b) the achiral compound D, C10H14, inert to oxidation under the same conditions.
Solution :
Given
COOH

alk. KMnO 4
C10H14
Chiral
(C)
C10H14 Inert to oxidation
(D)
achiral

Since benzoic acid is the product ÂCÊ should be monosubstituted benzene.

C4H9
CH3

H3C ă C ă CH3
H5C2 · *CH · CH3 Achiral and resistant to
oxidation

(C) (D)

HYDRO CARBON
P-BLOCK
ELEMENT
QUIZRR 3

P-BLOCK ELEMENTS
I ntroduction :

The right side of the periodic table having group number 13, 14, 15, 16, 17 and 18 are known
as p-block elements. These elements have 3, 4, 5, 6, 7 and 8 electrons in their outer most shell,
respectively. The last electron of these groupsÊ elements occupies the position in p-sub shell that
is why they are called as p-block elements. Their general configuration is ns2np1-6.

Some Important Properties of P-Block

1. Electron affinity :
Electron affinity increase from left to right along the period amongst the p-block elements and it
decreases from top to bottom. But group 15 is having exceptionally low values of electron affinity
and is due to extra stability because of the presence of exactly half filled orbital in their valence
shell. Similarly, elements of group 18 (noble gases) have zero affinities due to presence of complete
octet which provides them stability.

2. Metallic Character
The metallic character is governed by
(i) Size of atoms and
(ii) Ionization energy.
The elements having bigger size and low ionization energy has a greater metallic character. After
combining both above mentioned factors we observe that the elements with above two properties
are located in left corner of p-block and strong non-metallic elements are located at right corner
and a diagonal strip of elements thus two, having in between properties are called as metalloids.

3. Oxidation state
The p-block elements shows variety of oxidation states both positive and negative. Some of the
p-block elements show different oxidation state due to inert ă pair effect, where the lower oxidation
state is more predominant.

4. Diagonal relationship
On moving diagonally across the periodic table the element show certain simplarities.
Li Be B C

Na Mg Al St
Note : Elements of 2nd period differ from their own group elements in some of the properties. This
is due to the following reasons :

P-BLOCK ELEMENTS
4 QUIZRR
(a) Small size
(b) Absence of vacant d-orbital
(c) High IP

Some Important Characteristics of PăBlock In Tabular Form

Sr. No. Property Along period Along group


(left to right) (top to bottom)

1. Atomic radii Decreases Increases


2. Ionization potential Increases Decreases
3. Electron affinity Increases Decreases
4. Electro negativity Increases Decreases
5. Metallic character Decreases Increases
6. Oxidizing property Increases Decreases
7. Reducing property Decreases Increases

GROUP 13 ELEMENTS

Elements : B, Al, Ga, In, TI.


Group 13 elements are boron (B), aluminium (Al), Gallium (Ga), Indium (In) and thallium (TI).
Boron is the only non-metal in this group others are metal. Non-metallic character of boron is due
to its small size and high ionization energy. The general valence shell electronic configuration of
these elements is ns2np1.

General Trends in Physical Properties

Sr.No. Property Boron Aluminium Gallium Indium Thallium

1. Configuration [He]2s22p1 [Ne]3s23p1 [Ar]4s24p1 [Kr]5s25p2 [Xe]6s26p1


2. Common + 3 + 3 + 3 + 3 + 3, + 1
oxidation state
3. Atomic radius 83 143 135 167 170
(pm)
4. First ionization 801 578 579 558 589
energy (KJ/mol)
5. Electro negativity 2.0 1.5 1.6 1.7 1.8

P-BLOCK ELEMENTS
QUIZRR 5

1. Density
Generally increases down the group but aluminium has an exceptionally low density.

2. Melting point and Boiling point


B to Ga decrease then Ga to TI increase Ga has lowest M.P. (29.8ĈC) and therefore liquid at room
temperature.

3. Atomic radii and ionic radii


On moving from B to TI the size increases due to addition of new energy shells at each step down
the group but Ga is smaller than Al.

4. Ionization energy
Generally IE decreases down the group but Ga has higher IE than Al exceptionally due to smaller
in size as compared to Al.

5. Metallic character
Electropositive character increases down the group hence metalic character also increase down the
group but aluminium is having high metallic character than Gallium due to low IE than Ga.

6. Inter Pair Effect

When we go down the group in p block element, stable positive valency decreases by a factor of
2. This effect is known as inert pair effect. It arises mainly due to poor shielding effect 3d and
4d series element. This result into increase in effective nuclear charge and greater hold on valence
S electrons by nucleus. These s electrons canÊt be pulled easily out which decreases the valency
by a factor of 2.
e.g. In 14th group while carbon & silicon show + 4 valency, while Pb is mostly stable in +2 state.

7. Oxidation states and nature of bond


The group ă 13 elements show variable oxidation states of + 3 and + 1. For lighter members +
3 state is more stable than + 1. Thus for Tl, + 1 state is more stable than + 3. It is due to inert
pair effect. The 6s2 pair does not prefer to form bonds and is called inert pair. The basic reason
for such an effect is bond energy. The MăCl bond energy in MCl3 compounds are as follows :
MCl3 MăCl, energy (KJ molă1)
Ga 242
In 206
TI 153
The ionisation energies of B is very high. It is, therefore, covalent in its compounds, always
tricovalent (2s22p1 electrons).

P-BLOCK ELEMENTS
6 QUIZRR
In + 3 oxidation state Al has very high charge to radius ratio (~ 6.0). So, it is also covalent in
its anhydrous compounds. However, in aq. solution tricovalent Al (e.g. AlCl3) becomes ionic due
to high hydration energy of Al+3 (4665 KJ molă1) [Al(OH2)6]3+.3Clă. The soluble compounds are
quite acidic due to hydrolysis (high charge to radius ratio of Al+3).

[Al(OH2)6]+3 + H2O  H3O+ + [Al(H2O)5OH]+2

TI is ionic in + 1 oxidation state, TICl ionic. The TI+ ion resembles K+ and Rb+. It is because radii
of the ions are comparable (K+ ~ 1.38 Å, TI ~ 1.5 Å, Rb ~ 1.52 Å). Therefore, TIOH is a strong
base like group ă 1 hydroxides. Tricovalent compounds, MX3, are electron deficient as octet
around the central atom is not complete.

F Cl

Al
B
F F Cl Cl

(Six electrons around B or Al)

Such compounds remove their electron deficiency by :


(i) Back ă bonding
In BX3, B is sp2 and has a vacant p-orbital perpendicular at the triangular plane. X-atom
has filled p-orbital. The overlap resulting into ăbonding.

F
F
B Resonance
B
F F
F F

(ii) Coordination with Lewis bases


B has vacant orbital which accepts electron pair from Fă.
F
ă ă
B +F BF4
F F

Al(OH)3 + HOă  Al(OH) 4

(iii) Polymerisation
AlCl3 removes electron deficiency by dimerisation.

2AlCl  Al2Cl6


P-BLOCK ELEMENTS
QUIZRR 7

Dimer is made easily because Cl has lp and Al has vacant orbital.

Cl Cl Cl
Al Al
Cl Cl Cl

AlCl3 is not stabilized by back bonding. It is due to the following facts :

(i) p-orbital of Al is large, so păp overlap is not effective.


(ii) 8 electron core in Al causes larger repulsion (8e ă 8e) destabilizing back bonding effective.

BX3 can not stabilize by dimerisation. B is small in size. The dimer will have small ring.

X X X
B B
X X X

So, it is not stable.

R EACTIONS OF ELEMENTS

Pure crystalline boron is very unreactive. However, it is attacked at high temperatures by strong
oxidising agents such as a mixture of hot concentrated H2SO4 and HNO3 or Na2O2.

2B(s) + 6HNO3 (aq)  2H3BO3(aq) + 6NO2(g)

In contrast, finely divided amorphous boron (containing about 2ă5% impurities) is more reactive.
Reactions are summarised in Table below
Reactions of Group 13 (IIIA) Elements
Reaction Comment

Ć 4M + 3O2  2M2O3 Reaction at high temperature; with Al exotohermic, a


protective oxide layer is formed and TI also forms TI2O.
Ć 2M + N2  2MN Only Al and (B) form nitrides BN, a slippery white solid
with layer structure similar to graphite.
Ć 2M + 3X2  2MX3 All form MX3, Tl also form TlX. Iodide of Tl3+ (TlI3) is not
formed; but TlI reacts with I2 forming TlI3 (Tl+, I3ă)
Ć 2M + 2NaOH + 2H2O  B, Al and Ga only form NaMO2
2NaMO2 + 3H2

Ć 2M + 6NH3  2M(NH2)3+ 3H2 All form amides except B which forms BN.
2B + 2NH3  2BN + 3H2

Ć 2M + 6H+  2M3+ 3H2 Metals liberate H2, but HNO3 makes Al passive.

P-BLOCK ELEMENTS
8 QUIZRR
Ć Boron reacts directly with elements forming borides which are hard and refractory.

3Mg + 2B  Mg3B2

Borides are decomposed by acids forming boranes

Mg3B2 + 6HCl  3MgCl2 + B2H6


diborane

Ć Aluminium is far more reactive than boron but in air and water it is stable. It is due to the
formation of aluminium oxide, Al2O3 layer, that makes it passive for further attack. HNO3
also makes Al passive by oxide layer formation.
Ć Reaction of Al with O2 is exothermic and is called thermite reaction.

3
2Al(s) + O (g)  Al2O3(s), H = ă 1670 kJ
2 2

The thermite reaction involves so much energy that it can be dangerous. The Al becomes
white hot and often causes fires. For this precise reason, mixture of Al and an oxide such
as Fe2O3 or SiO2 (a source of oxygen) were used to make incendiary bombs during World
War II.

BX3

H3BO3 + NO2
MxBy
X2
M
HNO3 al
et
m

N2 O2
BN BORON B2O3

NaOH
NH3 base
S
BN + H2 Na3BO3 + H2

B2S3

Boron : Extraction

acid 
Ć Na2 B4O7.10H2O   H3BO3   H2O + B2O3
orthoboric acid


B2O3 + 6Na   2B + 3Na2O

B2O3 + 3Mg   2B + 3MgO

P-BLOCK ELEMENTS
QUIZRR 9

red hot
Ć 2BCl3 + 3H2 
filament
 2B + 6HCl

Ć Pyrolysis of BI3

red hot
2BI3 
filament
 2B + 3I2

Ć thermal decomposition of B2H6



B2H6   2B + 3H2

C ompounds of Boron

1. Borax (Na2B4O7.10H2O)
Ć The actual formula is Na2[B4O5(OH)4].8H2O
OH

O Bă O

Na2 HO · B O B · OH

O Bă O

OH
Ć Action of heat

Heat 740C
Na 2 B4 O7 .10H 2O 
 Na 2 B4 O7 
 2NaBO2  B2 O3
 10H2O
sodium Boric
metaborate
anhydrid
 
Glassy mass

Ć Various reactions of borax

Na2CO3 + H3BO3

NaOH Na2B4O7

NaBO2
4 Cl ith

NaBO2 + B2O3
Sodium H2O
NH ed w

metaborate
HCl
at
He

H3BO3 + NaOH CuO 


Heated
BN strongly NaCl + H3BO3
(also called 100Ĉ C Cu(BO2)2
inorgonic graphite) (Blue)
B2O3
HBO2 Boric anhydride

P-BLOCK ELEMENTS
10 QUIZRR
Ć Solution of Borax is alkaline in nature. This is due to hydrolysis



Na 2 B4O7  7H2O  2 NaOH + 4H3 BO3
Strong Weak
alkali acid

2. Boric acids and oxides


(i) H3BO3 or B(OH)3 orthoboric acid
(ii) HBO2 metaboric acid
(iii) H2B4O7 pyroboric acid

100Ĉ C 140Ĉ C
H3BO3 HBO2 H2B4O7

red hot

B2O3

3. Boron Trioxide, B2O3

Ć 2H3BO3  B2O3 + 3H2O

It is a white hygroscopic solid. It absorbs moisture and converted into boric acid.

B2O3 + H2O  2HBO2

HBO2 + H2O  H3BO3

Ć 2B2O3 + P4O10  4BPO4

Ć It is reduced by magnesium
Heated
B2O3 + 3Mg   2B + 3MgO

4. Boron halide

CaF2 + H2SO4 + H3BO3(or B2O3)


H2O ă
BF3 H3BO3 + BF4

180Ĉ C NaH

B2H6

P-BLOCK ELEMENTS
QUIZRR 11

5. Boron Hydrides

excess NH3
B2H6.2NH3
low temp.

excess NH3
B2H6 + NH3 (BN)x
high temp. boron nitride

ratio : 2NH3 : 1B2H6


B3N3H6
high temp. borazine

Ć B2H6 . 2NH3 is ionic and forms borazine on heating. Borazine is called inorganic benzene.

H N H

B B

N N
B
H H

Similarity between Boron and Silicon (Diagonal Relationship)


Ć Both form volatile hydrides (called boranes and silanes) which spontaneously catch fire on
exposure to air and are easily hydrolysed.
Ć Halides of both (except BF3) are easily hydrolysed :

BCl3 + 3H2O  H3BO3 + 3HCl

SiCl4 + 4H2O  H4SiO4 + 4HCl

Ć Boron forms binary compounds with several metalsăthese are called borides. Silicon form
silicides. Borides and silicides are hydrolysed by H2O forming boranes and silanes :

Mg3B2 + 6H2O  3Mg(OH)2 + B2H6

Al4Si3 + 12H2O  4Al(OH)3 + 3SiH4

Ć B2O3 and SiO2 are acidic in nature and are important constituents of glass. Borate and
silicates have tetrahedral BO4 and SiO4 structural units, respectively.
Ć Both are semi-conductors.

P-BLOCK ELEMENTS
12 QUIZRR
Ć Boric acid (H3BO3) and silicic acids (H4SiO4) are weak crystalline.
Ć They do not dissolve in cold dilute acids but do so in alkaliesăboron only in fused alkalies
and silicon in both fused and aqueous alkalies.

2B + 2NaOH + 2H2O  2NaBO2 + 3H2

Si + 2NaOH + H2O  Na2SiO3 + 2H2

C ompounds of Aluminium

1. Aluminium oxide or Alumina, Al2O3

2Al(OH)3  Al2O3 + 3H2O

Al2(SO4)3  Al2O3 + 3SO3

(NH4)2SO4 . Al2(SO4)3 . 24H2O  Al2O3 + 2NH3 + 4SO3 + 25H2O

2. Aluminium Chloride
AlCl3 exists as a dimer, thus attaining an octet of elections.

Cl Cl Cl

Al Al

Cl Cl Cl

However, when the halides dissolve in H2O, the high enthalpy of hydration is sufficient to break
the covalent dimer into [M(H2O)6]3+ and 3Xă ions.

AlCl3 + 3H2O  Al(OH)3 + 3HCl


3. Alums
Alums have general formula
M2SO4 . M´2(SO4)3 . 24H2O
where M stands for monovalent basic radicals such as Na+, K+, Rb+, Cs+, Ag+ etc. and M´ for
trivalent basic radicals such as Al3+, Cr3+, Fe3+, Mn3+, Co3+
Some examples of alums are :

Potash Alum K2SO4 . Al2(SO4)3 .24H2O


Ferric alum (NH4)2SO4.Fe2(SO4) 3

Alums are crystalline compounds. In alums each metal ion is surrounded by six water molecules

and the crystals of alums consists of [M(H2O)6]+, [M´(H2O)6]3+ and SO24 ions.

P-BLOCK ELEMENTS
QUIZRR 13

G ROUP–14

Elements of this group are p-block element. Carbon and Si are non-metals (Si slightly metallic),
Ge is metalloid, Sn and Pb are metals.
Carbon has three isotopes, they are :

Isotope No. of neutron Abundance Nature


12
C 6 98.9%
Stable isotopes
13
C 7 1.1%
Radioactive
14
C 8 Trace
tó = 5770 yrs

Physical properties of groupă14 elements

Property C Si Ge Sn Pb

Atomic no. 6 14 32 50 82
Electronic ă 2s22p2 ă 3s23p2 3d104s24p2 4d105s25p2 4f145d106s26p2
structure
Atomic radius
(A) [covalent radius] 0.77 1.17 1.22 1.40 1.44
Ionic radius (A) 0.15 0.41 0.53 0.71 0.84
Metallic radius (Å) -- -- 1.37 1.62 1.75
Ionisation
energy (ev) [I] 11.3 8.2 7.8 7.3 7.4
[II] 24.4 16.3 15.9 14.6 15.0
Electronegativity 2.5 1.9 1.8 1.7 1.7
mp (ĈC) > 3550 1410 937 232 327
bp (ĈC) 4827 2355 2830 2260 1744

{The values are not to be remembered, these are given to show the variations}
Note :
(i) The radii of Si and Ge are very similar due to d-block contraction. Also Sn and Pb have
almost similar radii due to lanthanide contraction.
(ii) Ionisation energy vary as C > Si ~ Ge > Sn < Pb. Silicon and Ge have comparable IE due
to similarity in their radii.
The radius of Sn is smaller than Pb but Pb has higher IE than Sn. It is due to higher Z* for Pb
as F ă orbital is least shielding.

P-BLOCK ELEMENTS
14 QUIZRR
O xidation states and Bonding

Groupă14 elements show variable oxidation states of + 2 and + 4. The stability of + 2 state is very
stable for Pb. It is due to inert pair effect.

C +4 +2
Si
Ge
Sn
Pb
Stability decreases Stability increases

Carbon and Si have high ionisation energy. Therefore, they are covalent in their compounds.
Tin and Pb form ionic compounds having appreciable covalent nature. It is because Sn and Pb
are not very large and have 18e core.
Carbon and Si differ widely in their bonding pattern. It is because of the following reasons :

Property C Si

Valence orbitals and 2s22p2 3s23p2 (3d0)


electrons

Number of bonds or
coordination number 4 only More than 4 (5 or 6)

-bonding Very effective No (p-p)  bonding but


(păp)  bonding (p ă d)  bonding. Using
vacant 3d orbitals

CăC bond energy 356 KJ molă1 226 KJ molă1


Low, however,
heterocatenation is
Catenation property Very high pronounced due to strong
Si ă O bond (368 KJ molă1)

The features of certain Si compounds having Si ă O and Si ă N bonds are explained by (d ă p)


 bonding. For example,

(CH3)3N triangular Pyramidal


(SiH3)3N triangular Planar
(SiH3)2O < (Si ă O ă Si)  144Ĉ

Trisilylamine, (SiH3)3N, is trogonal planar due to (d ă p)  bonding involving filled N 2pz orbital
and vacant Si 3dxy orbitals.

P-BLOCK ELEMENTS
QUIZRR 15

Si · N Si N

The large angles at O of disilyl ethers (R3Si)2O, is due both to electronic and steric factors.

C atenation

The property of self-linking is known as catenation. It depends mainly on bond energy. The
CăC bond energy is very high. Therefore, C has maximum catenation property. The bond energy
decreases down the group and so also the catenation property, i.e.

C >> Si > Ge  Sn >> Pb

Bond Bond Energy (KJ molă1)

C ă C 356
Si ă Si 210 ă 250
Ge ă Ge 190 ă 210
Sn ă Sn 105 ă 145

Sulphur also shows catenation because S ă S bond energy is relatively high. Boron, Si, P shows
heterocatenation as B ă O, Si ă O and P ă O bond energies are high.

Bond Bond Energy (KJ molă1)

S ă S 226
B ă O 560 ă 790
Si ă O 368
P ă O ~ 340

Allotropy and structure

All the elements are solid. Carbon and Sn only show allotropy.

Carbon has three crystalline allotropes :


(a) Graphite
(b) Diamond and
(c) Fullerene.

P-BLOCK ELEMENTS
16 QUIZRR
Tin is found as  ă Sn, and  ă Sn.

Element Allotrope Structure


Graphite Layer structure
C Diamond Cubic structure
Fullerence Carbon cluster

Si -- Diamond structure

Ge -- Diamond structure
 ă Sn Diamond structure
Sn  ă Sn Cubic structure
 ă Sn Simple cubic structure

Pb -- ccp

(a) Graphite

It is soft dull looking greasy substance and is electrical conductor. Graphite has two-dimensional
layer structure.
Layers are made up of sp2 hybridized C atoms. Each C is joined with three other C atoms through
 ă bonds forming hexagonal rings. Each C is left with its fourth valence electron in a p-orbital.
They overlap and form  bonds. These  bonds are delocalized. Therefore, all C ă C bond distances
in graphite are equal (1.415 Å). Sheet to sheet distance in graphite is 3.35Å. It is known as C
ă dimension. Layers are joined by weak Van der waals force.
The weak force between layers allows one layer to slide over the other. It imparts softness to
graphite. It is, therefore, used in pencils for writing. Due to delocalized  electrons graphite is a
conductor (though along the sheet not perpendicular to it). Hence, it is used as electrode.

C
C C

C C C

C C C C

C C C C

C C C

C C C

C C

P-BLOCK ELEMENTS
QUIZRR 17

(b) Diamond

It is colourless transparent hardest substance known and is insulator. Cut diamond is sparkling
solid (due to refraction). Diamond has cubic unit cell. Each C is sp3 hybridised and is joined with
four other C atoms through  ă bonds.

C
C
C

As C ă C bond energy (356 KJ molă1) is very high and diamond has a giant structure, it is very
hard. This structure is responsible for its insulator property.

Density of diamond (3.51 g/cc) is greater than graphite (2.22g/cc). This shows that to convert
graphite into diamond, pressure must be applied. There is the equilibrium.

15000 atm

Graphite  
 diamond
300 k

Graphite can be transformed into diamond at a reasonable rate at high pressure and ~ 300 K in
the presence of a transition metal (Fe, Cr, Pt) catalyst.

(c) Fullerenes

Fullerenes are carbon clusters having cage like structures.

Fullerenes C60 and C70 can be separated by chromatogrphic method over alumina. Many other
fullerenes also exist such as C32, C50, C76, C84 etc.

The fullerenes C60 is well known. It has a soccer ball structure which contains 12 five ă membered
and 20 six membered rings. The rings are unsaturated. The C60 fullerene is called Buckminster
fullerene in honour of American architect Robert Buckminster Fuller.

P-BLOCK ELEMENTS
18 QUIZRR
Structure of C60

Silicon exists only in diamond structure. A graphite like structure for Si is not possible because
it has no propensity to form (p ă p) bond.

Tin exists in three forms,  ă Sn,  ă Sn and  ă Sn.

 · Sn  · Sn  · Sn
(Grey) (White) (Brittle)
(Non ă metallic (Metallic distorted (Rhombic)
diamond structure) close packed)

Semiconductor property of Si and Ge

Silicon and to a less extent Ge are important semiconductor materials. Pure Si and Ge are non-
conductor of electricity. However, when they are droped with group ă 13 or group ă 15 elements,
they become semiconductor.

(i) Ge dopped with In


Ge ă 4s24sp2
In ă 5s25p1
When Ge is dopped with In, there is one electron less at In site. There develops a positive
hole in the lattice which makes Ge conducting. Such a semiconductor is called p ă type
(positive ă type) semiconductor.

(ii) Ge dopped with As


Ge ă 4s24p2
As ă 4s24p3
When As is dopped in Ge, there is one electron excess at As site. Due to this excess electron
Ge conducts electricity. Such a semiconductor is called n ă type (negative type).

P-BLOCK ELEMENTS
QUIZRR 19

Ge Ge Ge Ge

Ge Ge In Ge

Ge Ge Ge Ge
+In
Ge Ge Ge Ge
Ge Ge Ge Ge
Ge Ge Ge Ge

Ge Ge Ge Ge
Ge Ge Ge Ge
Ge Ge Ge Ge
+As
Ge Ge As Ge

Ge Ge Ge Ge

Ge Ge Ge Ge

M ultiple Bonding
In Group 14, carbon is the only element capable of forming comparatively stable multiple bonds
with another C atom and also with N, O etc. Examples are alkenes (> C = C <), alkynes (ă C 
C ă), imines (> C = N ă), nitriles (ă C  N ă), ketones (> C = O), etc. None of the other elements
of the group forms stable compounds containing multiple bonds. Recently, some
success has been achieved in synthesising compounds with > Si = Si < and Si = C
< double bonds, but their details are beyond the scope of this course. Stoichiometric
analogues like SiO2, SnO2 and (Me2SiO)n are known which are giant macromolecules
containing MăOăM linkages.
The tendency of formation of multiple bonds easily, in case of carbon, is because of
the high bond energy and a small atomic radius of carbon is compared to other
elements of the group. So its p-orbitals can approach the p-orbitals of the other C,
O or N atom for an effective overlap essential for -bond formation. p-orbitals
Ć Silicon also uses d-orbitals in multiple bonding, In case of N(CH3)3, geometry is
pyramidal, but in case of N(SiH3)3 it is planar. It is due to the fact that in the
latter, the lone pair of N-atom is transferred to empty d-orbitals of silicon (păd
overlapping) :

N N
CH3 CH3 SiH3 SiH3
CH3 SiH3
pyramidal planar
(no  bonding) (păd bonding)

P-BLOCK ELEMENTS
20 QUIZRR
Ć The normal valency of the elements is four, but apart from carbon, the elements can make
more than four bonds. This is because they make use of a set of d-orbitals in bonding. Thus
availability of d-orbitals (empty but electrons of ns2 and np2 can be excited into this) is
responsible for the ability of the elements, except carbon, to make complex ions such as
SiF62ă.

Energy
3d

very large
energy gap,
can not 3d
use 3d orbitals
relatively small
2p in bonding
energy gap, can
use 3d orbitals
in bonding
3p
Carbon

silicon
Fig. Energy difference between 2p and 3d energy levels in
carbon and between 3p and 3d energy levels in silicon.
Ć The main chemical differences between silicon and carbon are :
ă Si does not form stable double bonds
ă Si does not form very stable Si ă Si bonds.
ă Si has vacant 3d-orbitals in its valence shell into which it can accept electrons from
donor atoms.
Ć Some carbon compound are less reactive than the corresponding compounds of the other
members of the group. CCl4 has no interaction with H2O (hydrolysis) whereas SiCl4 does by
use of empty 3d-orbitals.
Ć All the elements give hydrides. Carbon, of course gives an immense number, but silicon and
germanium also show a wide variety.

CH 4 methane (alkane)
SnH 4 Stannane
SiH4 silane
PbH4 plumbane
GeH 4 germane
Carbon hydrides will not ignite in air unless a flame is put to them. Apart from silane, SiH4,
the silicon hydrides are less well behaved. Si3H8 is spontaneously flammable in air :

Si3H8(l) + 5O2(g)  3SiO2(s) + 4H2O(l)

P-BLOCK ELEMENTS
QUIZRR 21

Alkanes as well as silane are not hydrolysed however, traces of alkali converts silane into
hydrated silica SiO2 . nH2O.
Ć As we descend the group, acidic nature of the oxides decreases and basic nature of the oxides
increases.

Oxides Nature Oxides Nature

CO, CO2 acidic SnO, SnO2 amphoteric


SiO2 acidic PbO, PbO2 basic
GeO, GeO2 amphoteric Pb3O4 amphoteric

Si, Ge, Sn and Pb form oxides MO2 . SiO2 is infinite three-dimensional network solid of
silicon and oxygen atoms connected by single covalent bonds. GeO2, SnO2 and PbO2 are all
solids and exist in several modifications.

Example 1

Why is graphite a better electrical conductor than diamond and why does the conductivity
of graphite depend on direction ?
Solution :
Graphite has a two-dimensional sheet like structure in which each carbon atom uses sp2 hybrid
orbitals to form trigonal planar  bonds to three neighbouring carbon atoms. In addition, each
carbon atom uses its remaining p-orbital, perpendicular to the plane of the sheet, to form a  bond.
Because each carbon atom must share its  bond with its three neighbours, the  electrons are
delocalised and are free to move in the plane of the sheet. As a result, the electrical conductivity
of graphite in a direction parallel to to the sheet is about 1020 times greater than the conductivity
of diamond. The conductivity of graphite perpendicular to the sheets of carbon atoms is lower
because electrons must hop from one sheet to the next. The substance or crystal that show
different properties in different directions is called anisotropic.

Example 2

Why elemental silicon does not form graphite like structure as carbon does ? Explain.
Solution :
This is due to the reluctance of silicon to form p-p multiple bonds because of large size of silicon
atom. Hence Si exists only in diamond form.

Compounds of Carbon and Silicon

1. Carbides : Binary compounds of carbon are called carbides, SiC, CaC2, Al4C3 etc. Three types are
found :
(i) Ionic or salt like carbides,

P-BLOCK ELEMENTS
22 QUIZRR
(ii) Covalent carbides,
(iii) Interstitial carbides.

(i) Ionic carbides


Three types of ionic carbides are found :
(a) Methanides,
(b) Acetylides,
(c) Propynide.
Ionic carbides are decomposed by water (or dil. Acid) at room temperature. The liberated anions
are also immediately hydrolysed. It affords a method of classification.

(a) Methanides : (Că4 unit)


Those carbides which are hydrolysed to give CH4 are called methanide. They are supposed
to contain Că4 unit.

Be2C + 4H2O  2Be(OH)2 + CH4

Al4C3 + 12H2O  4Al (OH)3 + 3CH4

(b) Acetylides : (C2ă2 unit)


Those which are hydrolysed to C2H2 are called acetylides. They are common and formed by
Group ă 1, Group ă 2, Mg, Ca, Sr, Ba, Zn, Cd, Cu, Ag, Al and La metals.
Gr ă 1
M2C2
Cu, Ag
Gr ă 2
MC2
Zn, Cd
Al, La M2(C2) 3

CaC2 + 2H2O  Ca(OH)2 + C2H2

Al2(C2)3 + 6H2O  2Al(OH)3 + 3C2H2

(c) Propynide : (C34ă unit)


Such carbides hydrolyse to propyne

Mg2C3 + H2O  2Mg(OH)2 + CH3 ă C  CH

(ii) Covalent carbides


Elements which compares with C in size and electronegativity form such carbides, Be, B, Si etc.
Boron and Si carbides are more important.

P-BLOCK ELEMENTS
QUIZRR 23


SiO2 + 2C   Si + 2CO

High temp
Si + C   SiC
Silicon carbide (SiC) is called carborundum. It is very inert and extremely hard.
High temp
B2O3 + C   B4C
B4C is a also very hard, infusible and inert. It is used to make bulletproof clothing.
These covalent carbides (or  ă Al2O3) are comparable in hardness to diamond (MohÊs scale).

Hardness  Diamond > B4C > SiC >  ă Al2O3

(iii) Interstitial carbides


Transition metals form such carbides, in which C atoms occupy octahedral voids. Such materials
are very hard, electrically conducting and have very high mp. For example : TiC, ZrC, MoC, WC
etc.

2. Oxides

Electrons of this form oxides of the type MO and MO2. Carbon forms suboxides also, e.g., C3O2.

Oxides C Si Ge Sn Pb

CO
GeO SnO PbO
MO (neutral) X  
ampoteric
GeO2 SnO2 PbO2
MO2 CO SiO2  

ampoteric

 acidic 

(a) Carbon monoxide (CO)


It forms many important gaseous mixtures of industrial use
(i) Water gas or synthesis gas CO + H2
(ii) Producer gas CO + N2
(iii) Coal gas CO + H2 + CH4 + C O2
 CO is a neutral oxide.
 Carbon monoxide has a triple bonded C atom, one s and two p bonds (best explained
by MO model).
 CO, however, is a strong ligand towards transition metals and form variety of complexes
known as carbonyls, Ni (CO)4, Fe(CO)5 etc. They are stable due to synergic bonding.

P-BLOCK ELEMENTS
24 QUIZRR
 CO is very toxic and rapidly forms a bright red complex with haemoglobin of blood.
Thus preventing haemoglobin to carry oxygen to cells, using oxygen deficiency to
unconsciousness and then death.

(b) Carbon dioxide (CO2)


 CO2 is acidic oxide
 CO2 is biologically important
(i) Maintains pH of blood, due to the equilibrium

CO2 + H2O  HCO3 + H+

(ii) Main components of photo synthesis

Chlorophyll
6CO2  6H2 O   C6 H12 O6  6O2
Sun light
 Glucose

(iii) The reverse process of the above takes place in the process of respiration.

C6H12O6 + 6O2  6CO2 + 6H2O + Energy

 CO2 is used in the synthesis of urea


NH2

CO2 + 2NH3 H2NCO2NH4 O C + H2O
Ammonium
Carbamate NH2
Urea

 CO2 is gas at NTP and has the structure O == C == O. The (p ă p) bonding is very
effective. Therefore, even in solid state discrete CO2 molecules exist. The intermolecular
forces are weak Van der WaalsÊ force.
 Solid CO2 is called Âdry iceÊ or ÂcardiceÊ.

Silicon dioxide, SiO2, unlike CO2 is a solid. The (p ă p)  bonding is not effective in SiO2 like CO2.
It is due to larger size of Si and more core electrons (8e) in Si. Therefore, to stabilize; SiO2 contains
a continuous lattice of Si and oxygen atoms joined by covalent bonds. Each Si is sp3 and is joined
with 4 O + atoms and each O ă atom with two Si ă atoms. A macromolecule is formed, hence SiO2
is a solid.
O O

O Si O Si O

O O

O Si O Si O

O O

P-BLOCK ELEMENTS
QUIZRR 25

Halides MX2 and MX4

Group ă 14 elements form tetrahalides of the type MX4 (X = F, Cl, Br, I) except PbI4.
PbI4 does not exist due to strong oxidizing power of Pb+4 and reducing nature of Iă ion. The Pb+4
is strong oxidizing agent due to greater stability of Pb+2 (inert pair effect).
Thus, MX4, [M = C, Si, Ge, Sn, X = F, Cl, Br, I]
Germanium, Sn and Pb also for MX2. The stability of MX2 is maximum for Pb+2.
SnF2 is sparingly soluble in water and is used in fluoride containing tooth pastes.
The MX4 halides are all colourless except GeI4 and SnI4 which are bright orange solids. Colour
in these halides is due to charge transfer.

Sn+4 Iă Ge+4 Iă
e e

All tetrachlorides except CCl4 are hydrolysed by water.


Carbon tetrachloride is not hydrolysed because :

(a) carbon in CCl4 is coordinatively saturated


(b) C ă Cl bond is less polar
XC = 2.5, XCl = 3.0,  = 0.5

Silicon tetra chloride (SiCl4) is extensively hydrolysed. It is because :

(a) Si ă Cl bond is more polar


XSi = 1.8, XCl = 3.0,  = 1.2
(b) Si in SiCl4 can increase its coordination number by using 3d orbitals

The tetra halides of Si, Ge, Sn and Pb can increase their coordination numbers by using d orbitals
and can form haloanions.

SiF4 + 2Fă  SiF62

SnCl4 + 2Clă  SnCl6

The hexacoordinated halo anions are octahedral having sp3d2 hybridised central atom (Si).

Example 3

CCl4 can not be hydrolysed but SiCl4 can be. Why ?


Solution :
Vacant d orbital is not present in carbon so it can not extend its configuration like Si. Therefore
it can not be hydrolysed.

P-BLOCK ELEMENTS
26 QUIZRR
Example 4

SiCl2 is solid while SiCl4 is liquid at room temperature. Why ?


Solution :
SiCl2 is more ionic than SiCl4 by FajanÊs rule. Therefore it has greater m.p. so it is solid at room
temperature while SiCl4 is liquid.

Hydrides :
All element of this group form hydrides.
Carbon forms a large number of chain and ring compounds due to high catenation property. They
are :
Alkanes CnH 2n+2
Alkenes CnH 2n
Alkynes CnH 2nă2
Cyclo alkanes and cycloalkynes
Aromatic hydrocarbons (C6H6)
Silicon (others also) form saturated hydrides only which are called silanes, SinH2n+2 (Si ~ 8).

SiCl4 + LiAlH4  SiH4 + AlCl3 + LiCl

Silicons :
They are organosilicon polymers having empirical formula approximately R2SiO. Polymers are
very stable due to strong Si ă O and Si ă C bonds. Silicones can be prepared as oils, resins, plastics
etc. They are :

 Inert
 Water repellent
 Heat resistant
 Electrical insulator

All these properties make silicones very useful.


Hydrolysis of alkyl chlorosilanes produces products having Si ă OH bonds. They do not dehydrate
like gem diols (of C).

R Cl R OH R
Si + H2O Si Si O
R Cl (ă2HCl) R OH R
Silicates

Silicates are solids having Si ă O bonds. Most of the rocks and even soil contain silicates. Alkali
metal silicates are water soluble. They are made on a large scale as

~ 1500C
Na2CO3 + SiO2   Na4SiO4, (Na2SiO3)n etc. + CO2

P-BLOCK ELEMENTS
QUIZRR 27

They are used in detergents. The concentrated aq. solution of Na4SiO4 is solid like and as known
as water glass. Variety of silicates are found but in all the basic unit is the SiO4 tetrahedron.

Si or

Oă Oă

Each O ă atom carries one unit negative charge. In polymeric silicates SiO4 units share
O ă atoms(s). It provides a way of classification.

O rthosilicates (Mononuclear silicates)

These are silicates having discrete SiO44  ions. Metal ions are surrounded by Oăatoms.

Pyrosilicates (Si2O 7–6) :

When two SiO4 units share only one O ă atom pyrosilicates, Si2 O76 , are formed. These are also

discrete units.

Oă O Oă
Si Si
ă
O Oă
ă ă
O O

Glass

Solids are most stable in crystalline form. However, if a solid is formed rapidly (for example, when
a liquid is cooled quickly), its atoms or molecules do not have time to align themselves and may
become locked in positions other than those of a regular crystal.
If we mix sodium carbonate and calcium carbonate with sand and fuse the mixture at about
1500ĈC, we get a liquid mixture of sodium and calcium silicates. When this mixture is cooled, it
becomes viscous and eventually ceases to flow; it becomes solid ăcalled glass. Glass is thus super-
cooled liquid (solid solution). Amorphous solids, such as glass, lack regular three dimensional
arrangement of atoms. Glass thus refers to an optically transparent fusion product of inorganic
materials that has cooled to a rigid state without crystallising.

P-BLOCK ELEMENTS
28 QUIZRR


Na2CO3 + SiO2   Na2SiO3 + CO2

CaCO3 + SiO2  CaSiO3 + CO2

By varying the proportions of the three basic ingredients, and by adding other substances, we can
alter the properties of the glass as given :

Ć Calcium-alkali silicate glass ă normal domestic glass for windows


Sodium silicate ă soda glass used in cheap laboratory glassware
Ć Fining agents such as NaNO3 or As2O3 are added to remove bubbles.
Ć Decolorising agents may be added to eliminate impurities and to obtain colourless glass.
CaF2 is sometimes added to make opal glass.

Glass is attacked by HF and this property is used to make marking on the glass (etching)

Na2SiO3 + 6HF  Na2SiF6 + 3H2O


(in glass)

CaSiO3 + 6HF  CaSiF6 + 3H2O

HF, however does not attack wax. The portion of the glass on which etching is to be done is
covered with wax layer. Markings are carved on the wax layer with a pin and HF is poured on
the carved parts and left for sometime. After washing with water, wax is removed and permanent
marking is obtained.

Example 5

The standard free energies of formation of B2O3 and MgO are ă 1194 kJ molă1 and ă 569 kJ
molă1 respectively. Should it be possible to prepare boron by reacting B2O3 with magnesium ?
Solution :

B2O3 + Mg  2B + 3MgO

GĈ = Gproduct ă GĈ reactant


= 3GĈ f (MgO) ă GĈ f (B2O3)
= 3(ă 569) ă (ă 1194)
= ă 513 kJ molă1 [GĈ f (element) = 0]
The negative sign of GĈ shows that the reaction should be spontaneous. (However, the reaction
is initially endothermic when two reactants are heated.)

P-BLOCK ELEMENTS
QUIZRR 29

Example 6

(i) What type of bonding holds the layer in graphite ?


(ii) Why will graphite conduct electricity well in a direction parallel to the planes of
hexagons, but not at all well in a direction perpendicular to the planes ?
(iii) What is the name we give to crystal or substances, that show different properties in
different directions ?
Solution :
(i) Van der Waals bonding
(ii) The delocalised orbitals in graphite lie parallel to the planes and electrons can move along
the planes making use of these orbitals; conduction is good parallel to the planes. There is
no overlap of orbitals perpendicular to the planes, so conduction is poor in this direction.
(iii) They are anisotropic.

Example 7

If you had a mixture of CO and CO2, how would you find out relative proportions of the
two gases in the mixture ?
Solution :
Method I : Mixture of CO2 is run into Ca(OH)2 solution. CO2 is absorbed by Ca(OH)2 and thus
residual volume will be that of CO.

Ca(OH)2 + CO2  CaCO3 + H2O

Method II : Mixture of CO and CO2 is passed over heated charcoal :

CO2 + C  2CO

Increase in volume will be due to CO2. By measuring total volume after passing over heated
charcoal, composition of mixture of CO and CO2 can be determined.

Example 8

CO and CNă ions are highly poisonous. A sign that some one has been poisoned by either
of these two chemicals is that the personÊs blood changes colour. They both have the effect
of preventing oxygen being taken up by the blood stream. What might happen to
haemoglobin to stop oxygen being taken up ?
Solution :
CO and CNă ions are able to bond more strongly to the iron atom in haemoglobin than can
oxygen. If you were to breathe in CO for any length of time, your haemoglobin would become
saturated with CO molecules rather than oxygen. As a result, all the body processes that rely on
a supply of oxygen gradually cease. Death is the result.

P-BLOCK ELEMENTS
30 QUIZRR
Example 9

What would you expect to happen when :


(a) Tin is heated with a concentrated aqueous solution of sodium hydroxide
(b) Sulphur dioxide is passed over lead dioxide
(c) Carbon disulphide is shaken with aqueous sodium sulphide
(d) Dichlorosilane is hydrolysed by water
(e) 4 mol of ClCH2SiCl3 react with 3 mol of LiAlH4 in diethyl ether solution ?
Solution :

(a) Sn + 2OHă + 4H2O  [Sn(OH)6]2ă + 2H2

(b) PbO2 + SO2  PbSO4

(c) CS2 + Na2S  Na2CS3


sodium thiocarbonate

H OH
(d) SiH2Cl2 + 2H2O  Si +2HCI
H OH
H
Si=O + H2O
H

(e) 4ClCH2SiCl3 + 3LiAlH4  4ClCH2SiH3 + 3LiCl + 3AlCl3

Example 10

Suggest syntheses (starting from deuterium oxide as the only source of deuterium) for:
(a) B2D6 (b) B(OD)3 (c) B3N3D6 (d) B (CH2CH2D)3.
Solution :

(a) P2O5 + 3D2O  2D3PO4

Mg3B2 + 2D3PO4  Mg3(PO4)2 + B2D6

(b) B2O3 + 3D2O  2B(OD)3

(c) Mg3N2 + D2O  ND3 + Mg(OD)2


2ND3 + B2D6  [D2B(ND3)2)][BD4]   B3N3D6

(d) 6CH2 = CH2 + B2D6  2B(CH2CH2D)3

P-BLOCK ELEMENTS
QUIZRR 31

Example 11

Explain the following :


(a) Boron does not usually form a cation :
B has
(IE)1 = 801 kJ molă1, (IE)2 = 2427 kJ molă1,
(IE)3 = 3659 kJ molă1
 Thus, the total energy required to give B3+ ions is far more than that which would be
compensated by lattice energies of ionic compounds or hydration of such ions in solution.
Thus formation of cation (like B3+) is not possible.

(b) The polarity of BăX bonds is in the order : BăF > BăCl > BăBr but Lewis acidicity shows
the sequence : BF3 < BCl3 < BBr3.
 With increase in polarity of BăX bond, acidity also increases and should be thus in the
order :
BF3 > BCl3 > BBr3
But Lewis acidity is in reverse order :
BF3 < BCl3 < BBr3
There is lateral overlap of the vacant 2p orbital of B with one completely filled orbital of F leading
to p-p bonds between B and F. This BăF bond thus acquires double bond character. This also
leads to compensate electron deficiency of boron and thus Lewis acid character of BF3 is reduced.
This p-p bonding decreases going from BF3 to BBr3 and thus Lewis acidic nature increases in
the order :

BF3 < BCI3 < BBr3


excite
B 1
ground state 2p 2p2
1
2s 2s1
excited state

F F
B F B F
F F

(c) Al and Ga are trivalent in their compounds but in case of Tl, monovalent compounds
are the most stable. Why ?
 It is due to Âinert pairÊ effect due to which stability of + 3 state decreases and that of + 1
state increases while going down the group (in this case Group 13)

P-BLOCK ELEMENTS
32 QUIZRR
Al3+ > Ga3+ > In3+ > TI3+
(most stable)

Al+ < Ga+ < In+ < TI+


(most stable)

(d) CO2 is a gas but SiO2 is a solid :


 Carbon and silicon (Group 14, IVA) have four valence electrons. We might expect carbon
and silicon to form oxides with similar properties. In CO2 the ability of C and O atoms to
form  bonds through the side-wise overlap of their 2p orbitals is strong. The result is strong
CătoăO double bonds and a very stable triatomic molecule.
O=C=O
Silicon, being in third period, would have to use 3 p orbitals to form double bonds with
oxygen. The side-wise overlap of these orbitals with the 2p orbitals of oxygen is too limited
for  bond formation. From an energy stand point, a stronger bonding arrangement results
if the Si atoms from four single bonds with O atoms (bond energy : 464 kJ/mol) rather than
two double bonds (bond energy : 640 kJ/mol) since each O atom must be simultaneously
bonded to two Si atoms, the result in a network of ăSiăOăSiăbonds and thus a hard (giant)
solid :

O O

O
O S O S O

Si
O O
O O
O
O S O S O

O O
(e) SiCl4 is hydrolysed but CCl4 is inert to hydrolysis :
 SiCl4 and CCl4 are covalent compounds. Carbon has maximum covalency of four while
silicon has six. Thus CCl4 is inert towards water and is not hydrolysed but SiCl4 can
coordinate with 2 mol of H2O molecules and is thus readily hydrolysed :

SiCl4 + 2H2O  Si(OH)4 + 2HCl

Cl Cl
H Cl Cl H

H2O + Si + OH2 O Si O

H Cl Cl H
Cl Cl
Silicon with covalency six

P-BLOCK ELEMENTS
QUIZRR 33

ă2HCl

Cl
H Cl Cl H
2H2O
O Si O OH S OH

H Cl Cl H
Cl
Silicon with covalency six

OH
|
 2HCl
 HO  Si  OH
|
OH

(f) (SiH3)3N is planar while (CH3)3 N is pyramidal :

 In case of (CH3)3N, lone pair on N and three covalent bonds with CH3 groups results in sp3
hybridisation but lone pair-bond pair repulsion causes distortion hence gives pyramidal
structure :

N
CH3 CH3
CH3

But in case of (SiH3)3N, lone pair on the nitrogen is transferred to the empty d-orbitals of
silicon (p ă d overlap) thereby causing planarity of unit.

Example 12

AlF3 is insoluble in anhydrous HF but when little KF is added to the compound it becomes
soluble. On addition of BF3, AlF3 is precipitated. Write the balanced chemical equations.
Solution :

3KF + AlF3  K3AlF6

K3AlF6 + 3BF3  AlF3 + 3KBF4

P-BLOCK ELEMENTS
34 QUIZRR
Example 13

(SiH3)3N is a weaker base than (CH3)3N. Explain.


Solution :
In (SiH3)3N, the lone pair of electrons on nitrogen is involved in păd back bonding, while such
bonding is not possible (CH3)3N as d-orbitals are not present in carbon. Hence, the lone pair of
electrons on nitrogen in (CH3)3N is available for donation to Lewis acids, exactly acting as
stronger base.

Example 14

(i) A white precipitate (B) is formed when a mineral (A) is boiled with Na2CO3 solution.
(ii) The precipitate is filtered and the filtrate contains two compounds (C) and (D). The
compound (C) is removed by crystallisation and when CO2 is passed through the
mother liquor left, (D) changes to (C).
(iii) The compound (C) on strong heating gives two compounds (D) and (E).
(iv) (E) on heating with cobalt oxide produces blue coloured substance (F). Identify (A) to
(F) and give chemical reactions for the reactions at step (i) to (iv).
Solution :
(i) The mineral (A) is colemanite, Ca2B6O11. 5H2O

boiled
Ca2B6O11 + 2Na2CO3(aq)   2CaCO3 + Na2B4O7 + 2NaBO2
(B) Insoluble (C) (D)

Soluble

(ii) (C), the borax is crystallized. The mother liquor consisting sodium metaborate is treated with
CO2.

4NaBO2 + CO2  Na2B4O7 + Na2CO3

(iii) heated
Na2B4O7.10H2O   Na2B4O7 + 10H2O
Anhydrous

strongly
Na2B4O7 
heated
 2NaBO2 + B2O3

heat
(iv) CaO + B2O3   CO(BO2)2
cobalt meta borate
(blue coloured)

P-BLOCK ELEMENTS
QUIZRR 35

Example 15

PbCl4 is less stable than SiCl4. Why ?


Solution :
Pb is more stable in + 2 stable due to inert pair effect while Sn is more stable in + 4 state as
compared to Pb+4.

GROUP 15 ELEMENTS

Elements : N, P, As, Sb, Bi


Nitrogen and phosphorus are non-metals. The metallic character increases down the group due
to lower I.E. and larger size. Hence, bismuth shows metallic character. The group state electronic
configuration is ns2np3.

Trends in Chemical Reactivity

Most common oxidation states shown by group 15 elements are ă 3, + 3 and + 5. The stability
of highest oxidation state (+ 5) decreases down the group.
The covalent character goes on decreasing as we move down the group in the sequence, P > As
> Sb > Bi. This is due to increasing size of atom which refers to FajanÊs rules.
Sb and Bi are the heavier elements of the group and form M3+ cations due to decrease in
ionization enthalpy.
In contrast to nitrogen, the phosphorus atom forms p=p bonding and forms both cyclic and open
chain compounds. Hence, it shows catenation.
As we move down the group there is a decrease is covalent character.
N > P > As > Sb > Bi
Nitrogen is chemically less reactive, due to high stability of its molecule. N2 in which two nitrogen
atoms are combined through a triple covalent bond (N  N) of which one is sigma () and two
are pie () bonds, thus posses high bond strength (941.4 KJ molă1)
Nitrogen has one special feature that it can form p-p multiple bonds with itself, carbon and
oxygen due to its small size. In phosphorus rather p-p, d-p is found as in POX3.

General Trends in Physical Properties

Sr.No. Property Nitrogen Phosphorus Arsenic Antimony Bismuth


3 2 3 2 3 2 3
1. Configuration [He]2s 2p [Ne]3s 3p [Ar]4s 4p [Kr]5s 5p [Xe]6s26p3
2. Common oxidation state ă 3, + 3, + 5 ă 3, + 5, + 5 + 3, + 5 + 3, + 5 + 3
3. Atomic radius (pm) 70 110 120 140 150
4. First ionization 1403 1012 947 834 703
5. Electronegativity 3.0 2.1 2.0 1.9 1.9

P-BLOCK ELEMENTS
36 QUIZRR
Since, it is not easy from energy consideration to gain three electrons to achieve stable noble gas
configuration, the formation of the tri-negative ion (M3ă) is rare. But, nitrogen being the smallest
and the most electronegative of all the members of the group, forms trivalent anion with reactive
metals which have low ionisation energies. Thus, nitrogen forms ionic nitrides such as Li3N,
Be3N2, Mg3N2, Ca3N2. As the electronegativity of the other elements of the group, i.e., P, As, Sb
and Bi, is low, most of their compounds are covalent in nature.
Nitrogen is not able to extend its coordination number beyond four, the other elements have
coordination numbers of five as well as six. The extension of coordination number is due to the
availability of d-orbitals in P, As, Sb and Bi, using which these elements can attain sp3d and sp3d2
hybridisation, e.g. in PCl5 and PCl6ă, respectively. Thus, nitrogen does not function as a Lewis
acid whereas phosporus, arsenic antimony and bismuth do so. The hybrides of this group and
their alkyl derivatives act as Lewis bases, forming a number of addition complexes, e.g., NH3 .
BF3, CaCl2 . 4NH2, etc.
Ć Stability of + 3 state increases and that of + 5 state decreases going down the group due
to „inert pair effect‰, thus
Bi3+ > Sb3+ > As3+
Bi5+ < Sb5+ < As5+
Bi5+ salts are very rare and are good oxidising agents :

Bi5+ + 2eă  Bi3+

As3+ salts are good reducing agents :

As3+  As5+ + 2eă

Ć They form trihalide and penthalide (except N, which does not form NCl5 etc.) As we go down
the group, covalent nature decrease and ionic nature increases; thus NCl3 and PCl3 are
covalent while BiCl3 is ionic. This is also in accordance with FajanÊs rule : Smaller the size
of cation, larger the size of anion, greater the charge then greater the polarising power and
hence greater the covlalent nature. Halides are hydrolysed in water forming oxyacids or
oxychlorides :

PCl3 + 3H2O  H3PO3 + 3HCl

SbCl3 + H2O  SbOCl  + 2HCl


(orange ppt)

BiCl3 + H2O  BiOCl + 2HCl


(white ppt)

P-BLOCK ELEMENTS
QUIZRR 37

Compounds of Nitrogen

Oxides of N
(i) Dinitrogen oxide (or nitrous oxide) N2O


NH4 NO3  N2 O + 2H2 O

Structure : It is isoelectronic to CO2 and has linear structure. But unlike CO2 it has dipole
moment (0.16 D). It shows resonance

ă + + ă
N == N == O N N·O

(2) Nitric oxide (NO)

 Copper reacts with dil HNO3 (8M) to form NO. It is reduction of NO3

8HNO3 + 3Cu  3Cu(NO3)2 + 4H2O + 2NO

 Commercially, NO is obtained by catalytic oxidation of NH3 (ostwald process for HNO3)

Pt catalyst
4NH3  5O2 
 4NO + 6H2O

Nitric oxide is a good ligand and form variety of complexes with transition metals which are
known as nitrosyls, Na2[Fe(CN)5NO]

(3) Dinitrogen trioxide N2O3


It can be obtained by mixing NO and NO2 in equimolar quantities at low temperature.

Low
NO + NO2 
 N2O3
Temp.
(blue liq.)
( 30F)

In gaseous state it has the structure

The N ă N distance is very large, 1.864Å

(4) Nitrogen dioxide NO2 (N2O4)

(i) 
Cu + conc. HNO3   NO2


Cu + 4HNO3   Cu(NO3)2 + 2H2O + 2NO2

P-BLOCK ELEMENTS
38 QUIZRR
(ii) Di or trivalent metal nitrate decomposes on heating to NO2

Pb (NO3)2   PbO + 2NO2 + óO2
Structure : NO2 is an odd electron (17 electrons) molecule and is paramagnetic. It has the
structure

The unpaired electron ensures ready dimerisation to a diamagnetic colourless species N2O4
(dinitrogen tetraoxide).

2NO2  N2O4

N2O4 is planar

(5) Nitrogen pentaoxide N2O5


It is obtained by dehydration of HNO3 by P2O5

2HNO3 + P2O5   2HPO3 + N2O5

Structure : N2O5 is solid and in this state it is NO+2 .NO3 i.e. nitronium nitrate. In gaseous state
it has the structure

O O

N O N

O O

AMMONIA ă NH3
It can be prepared in many ways :
(i) Any ammonium salt on heating with alkali forms ammonia.


NH4Cl + NaOH   NH3 + NaCl + H2O

2NH4 OOCCH3 + Ca(OH)2  NH3 + Ca(CH3COO)2 + 2H2O

P-BLOCK ELEMENTS
QUIZRR 39

(ii) Ionic nitrides are hydrolysed by water to produce NH3.

Mg3N2 + 3H2O  2NH3 + 3MgO

(iii) NH+4 salts which contain non-oxidizing anions, on heating given NH3.


NH4Cl   NH3 + H2Cl

Non-oxidizing anions are Clă, SO24 , CO23 , CH3 COO , PO34 etc.

However NH+4 salts which contain oxidizing anions forms N2 or N2O on heating.

NH4NO3(l)  N2O + 2H2O

NH4NO2  N2 + 2H2O

(NH4)2 Cr2O7  N2 + Cr2O3 + 4H2O

(iv) NH3 is manufactured by :

High pressure
~ 450 C
(a) 
HaberÊs process N2  3H2  
 2NH3 (g) + Q
Fe (Fe2O3 ) catalyst
MO  Promoter 

(b) Cyanamide process

Electric
CaC2  N2 
 CaCN2  C
furnace
(~ 1100C) (Nitrolum)


CaCN2 + 3H2O   CaCO3   2NH3

Nitrous acid (HNO2)

Ć It disproportionates to HNO3 (oxidation) and NO (reduction)

3HNO2 2NO + HNO3 + H2O


+3 +2 +5
reduction

oxidation

P-BLOCK ELEMENTS
40 QUIZRR

HNO2

re age
en ng

du nt
ag izi
t

ci
id

ng
ox
Iă MnO4ă /H+
I2 Mn2+
S Fe2+ +
H2 Cr2 O72ă /H
H2O2
SO2
S Fe3+
2ă 3+
SO4 H2O Cr

Structure : H

O N
O

NO2 has planar triangular structure

N
ă
O O

Nitric acid (HNO3)


HNO3 can act as an oxidizing agent.

SO42ă S
SO2
H2S
2+
Fe S
Fe3+ HNO3 H2SO4
C I2
P

H2CO3 HIO3
H2CO3

Aqua regia

A mixture of conc. HCl and HNO3 in the ratio 3 : 1 is called aquaregia. This is a powerful
oxidizing agent and dissolves metals like Au and Pt. The two acids on mixing react as

3HCl + HNO3  NOCl + 2Cl + H2O

The presence of NOCl and Cl makes aquaregia a powerful oxidizing agent.

Au + 3Cl  AuCl3

P-BLOCK ELEMENTS
QUIZRR 41

AuCl3 + Clă  AuCl4 (or HAuCl4, chloroauric acid)

Pt + 4Clă  PtCl4

PtCl4 + 2Clă  PtCl26  (or H2PtCl6, Chloroplatinic acid)

Note : Au is not oxidized by HNO3 alone but by aquaregia.

R ed Phosphorus

Red phosporus is prepared by heating white phosphorus in the absence of air at 250ĈC for a few
hours with a trace of iodine as catalyst in a closed iron vessel fitted with a safety valve. At the
end of conversion, the product is ground under water and treated with hot caustic soda solution
to eliminate any unchanged white phosphorus. It is finally washed with hot water and dried by
heating with steam.
Another allotrope of phosphorus is black phosphorus and is obtained by heating white phosphorus
at 470 K under high pressure. A series of phases of black phosphorus are formed and one of these
phases consists of an extended layer structure in which each phosphorus is bonded to three
neighbours by single covalent bonds. Thermodynamically, black phosphorus is the most stable
form.
White phosphorus, the form produced in the industrial synthesis, is a toxic, waxy, white solid that
contains discrete tetrahedral P4 molecules. Red phosphorus, by contrast, is essentially nontoxic
and has a polymeric structure :

P P P P

P P·P P·P P·
P ·P
P P P P
White phosphorus Red phosphorus

As expected for a molecular solid that contains small, non-polar molecules, white phosphorus has
a low melting point, (44ĈC) and is soluble in non-polar solvent such as CS2. It is highly reactive,
bursting into flames when exposed to air, and is thus stored under water.

Phosphine (PH3 )

(i) When a mixture of white P (P4) and NaOH soln. is heated, PH3 is formed.

P4 + 2NaOH + 3H2O   3NaH2PO2 + PH3
In this reaction some P2H4 is also formed as side product. It is inflammable in air.
(ii) Ionic phosphides react with water to form PH3.

Ca3P2 + 6H2O  3Ca(OH)2 + 2PH3

P-BLOCK ELEMENTS
42 QUIZRR
NH3 is highly soluble in water by PH3 is insoluble in water. NH3 dissolves due to H-bonding.

H
H O H
H
O H N H O H
H
H H O H

PH3 cannot form H-bond, so it is insoluble in water.

O xides of Phosphorus

P ă oxides P4O6 and P4O10

 P4 X Stream of air

 P4 O6
600C

Excess of air
P4  P4 O10

These oxides are solids.

They differ from N oxides in stoichiometry and structure. It is due to inbality of P to form
(p ă p) bonds.

P4O6 and P4O10 both have cage structure. The structures are based upon P4 tetrahedra.

P4O 6
In this oxide P ă atoms form a regular tetrahedron and O ă atoms are present at six edges of the
tetrahedron and bridge P ă atoms.
P
O O
O
P
O P

O P O

P4O10
In this oxide P4 tetrahedron has six edgeăbridging Oăatoms and four terminal Oăatoms. The
terminal P ă O bonds have (d ă p) bonding. Therefore, there are two types of P ă O distances
in P4O10 large bridging P ă O and small terminal P ă O distances.

P-BLOCK ELEMENTS
QUIZRR 43

P
O O
O O
O P P O
O P
P
O O
O (d ă p) bonding in P4O10

O xy acids of P

The common oxy acids of P are :

Oxidation
State of P Name Formula Proticity Structure
+ 1 Hydrophosphorous
acid (phosphinic acid) H3PO2 1 Td

+ 3 Phosphorous acid H3PO3 2 Td.

Td. Polymeric
+ 5 Metaphosphoric acid (HPO3)n 1 and contain PO4
Td units

Td
+ 5 Orthophosphoric acid H3PO4 3 Highly hydrogen
bonded

Td. (two
+ 5 Pyrophosphoric acid H4P2O7 4 tetrahedra
joined through O
ă atom)

220C
H3PO4  320C
 H4P2O7   (HPO3)

Phosphorus Halides

P4 + 6X2  4PX3 (in limited supply of X2)

P4 + 10X2  4PX5 (in excess amount of X2)

P-BLOCK ELEMENTS
44 QUIZRR

Ć SO3 + PCl3  POCl3 + SO2

Ć PCl3 + H2O  H3PO3 + 3H2O

Ć PCl3 + HI  PI3 + 3HCl

Ć 3ROH + PCl  3RCl + H3PO3

Ć ROH + PCl5  RCl + POCl3 + HCl

Example 16
Pb(NO3)2

F2 H2O
F A B C + HNO3

HCI
CO 

G+H
D+E A+D+H2O

Solution :
A = NO2, N2O4, C = HNO2, D = NO, E = CO2, F = NO2F, G = NOCl, H = Cl2.

Group 16 (VIA–The Chalcogens)

Ć The first four elements are called chalcogens meaning ore forming (oxides, sulphides, selenides,
tellurides).
Ć Similar to the groups discussed earlier, the metallic character in Group 16 increases with increasing
atomic number. Oxygen and sulphur are entirely non-metallic in their chemical behaviour. Selenium
and tellurium though essentially non-metallic, assume increasing metallic character and are
termed as metalloids. Polonium is most metallic in the group.
Density, melting and boiling points covalent and ionic radii, show a regular increase from oxygen
to polonium. The large difference in melting and boiling points between oxygen and sulphur can
be explained on the basis of their structure. Oxygen exists mostly as a diatomic molecule held
together by weak van der Waals forces while others exist as polyatomic molecules, e.g., S8, Se8,
etc., where the atoms are bonded by covalent bonds associated with high dissociation energy.
Their existence as diatomic and polyatomic molecules can be explained as follows. The bond
energy of the oxygen-oxygen double bond. O=O, is 498 kJ molă1. This makes the O=O bond
more than three times as strong as the OăO bond (bond energy for OăO is 142 kJ molă1). By
comparison the S=S bond is less than twice as strong as the SăS single bond (bond energy for

P-BLOCK ELEMENTS
QUIZRR 45

S=S, 431 kJ molă1; SăS, 265 kJ molă1). This results in catenated ăOăOăOă chains begining
unstable relative to O=O, but catenated ăSăSăSă chains being stable relative to the molecule S=S.

The elements of Group 16 are characterised by high ionisation energies. Their electronegativities
decrease with increasing atomic number. Thus, in view of the fall in electronegativity, metallic
character within the group increases with increasing atomic size.

Oxygen, the second most electronegative element, fluorine being the first, has a strong tendency
to accept two electrons and give O2ă ion. Thus, almost all metal oxides are ionic and contain O2ă
ions. Usually oxygen exhibits an oxidation state ă 2 in its other compounds also. It exhibits
positive oxidation states only in a few compounds formed with fluorine, i.e., OF2 and O2F2. The
tendency for the formation of divalent anions decreases from sulphur downwards because of the
increasing size and decreasing electronegativity of the elements. Sulphur, selenium an tellurium
show a tendency for covalent with formal oxidation states + 2, + 4 and + 6 in compounds in which
they are combined with more electronegative elements such as oxygen and halogens. In the
higher oxidation states of + 4 and + 6 of these elements electrons are unpaired and promoted to
vacant d orbitals.
Ć Selenium has unique property of photoconductivity and is used in photocopying (xerox)
machines and also a decoloriser of glass. Te and Po are highly toxic. Po is higher due to its
intense radioactivity. Se and Te are found as selemides and tellurides in sulphide ores
(Cu2Se, Cu2Te). The colour of the red glass in the traffic signals is due to CdSe.
Ć With valence electronic configuration ns2 np4 the group 16 (VI A) elements are just two
electrons short of an octet configuration, and ă 2 oxidation state is therefore, a common one.
The stability of 2 state decreases with increasing metallic character as indicated by EĈ
(redox) values. This oxygen is powerful oxidising agent and H2Se and H2Te are reducing
agent (EĈ values of Se and Te are negative). Because S, Se, and Te are much less
electronegative than oxygen, they are commonly found in positive oxidation states as + 4 in
SF4, SO2 and H2SO3, and + 6 in SF6, SO3, H2SO4.
Ć H2Se and H2Te are unpleasant gaseous hydrides like H2S but less stable than H2S, order
being H2O > H2S > H2Se H2Te > H2Po.
Ć Oxoacids of S, Se and Te exist and H2SO4 and H2SeO4 and considered as strong acids.
Ć The tendency for catenation decreases markedly as we go down the group. This property is
prominently displayed by sulphur (S8). The SăS bond is important in biological system and
is found in compounds such as cysteine, some proteins and enzymes. Stable polyoxides and
polysulphides are known. Examples are :

HăOăOăH, HăSăSăH, HăSăSăSăH,


HăSăSăSăSăH.
Ć The metallic character increases as we descend the group. S is non-metal and insulator. Se
and Te are metalloids and are semiconductors. Po is a metal.

P-BLOCK ELEMENTS
46 QUIZRR
Ć Unlike oxygen, the maximum coordination number of sulphur and other elements can
exceed four and valency is not limited to two because of available d-orbitals for bonding.
Stability of + 6 state decreases and that of + 4 state increases going down the group due
to inert-pair effect. It is :
Se4+ < Te4+ < Po4+
Se6+ > Te6+ > Po6+
Ć The tendency to form multiple bonds to C, N and O decreases as we descend the group from
Se to Te. Thus S=C=S is moderately stable; Se=C=Se decomposes readily whereas Te=C=Te
does not exist.
Ć Thermal stability of the hyrdies decreases as we go down the group :
H2O > H2S > H2Se > H2Te > H2Po
Ć They form halides of different oxidation number + 1, + 2, + 4 and + 6.
Halides X Oxidation number
S2 X 2 F, Cl, Br, 1 + 1
SX 2 F, Cl + 2
SX 4 F + 4
SX 6 F + 6
Se2X2 Cl, Br + 1
SeX2, TeX2 Cl, Br + 2
SeX4, TeX4 F, Cl, Br + 4
TeX4 I + 4
SeX6, TeX6 F + 6
The stability of the halides decreases in the order F > Cl > Br > I. + 6 oxidation is observed
in fluroide, while iodide of Te(TeI4) is formed.
SF6 is thermodynamically inert, non-toxic gas. Inertness of SF6 is due to the presence of
sterically protected sulplhur atom. SF6 is used as a gaseous insulator in high voltage generators.
SF4 and SeF4 are good fluorinating agents and ăCOOH group can be converted into CF3
and > C=O and ăP=O into CF2 and ăPF2 groups.
Ć Oxides of sulphur are more stable than the corresponding oxides of other elements.

O zone (O 3)

Ć In the laboratory, O3 can be made by passing oxygen through a strong electric field. An
equilibrium is set up :
3O2(s)  2O3 (g)
Ć Some 30 km above the earthÊs surface oxygen molecules can be split apart by UV light from
the sun. Some of the atoms join with other oxygen molecules to make ozone :

P-BLOCK ELEMENTS
QUIZRR 47

O2(g) + O(g)  O3(g)

Ć O3 is an unstable, dark blue diamagnetic gas, b.p. ă 112ĈC. The colour is due to intense
absorption of red light.

NO + O3  NO2 + O2

O3 + h  O2 + O

NO2 + O  NO + O2

Net reaction : 2O3 + h  3O2


ă based on reactive chlorine species from CFC

Cl + O3  ClO + O2

O3 + hv  O + O2

ClO + O  Cl + O2

Net reaction : 2O3 + hv  3O2

Ć O3 is thermodynamically unstable, and decomposes to O2 :

2O3  3O2 G = ă 163 kJ molă1


Ć O3 is an extremely powerful oxidising agent :
O3 + 2H+ + 2eă  O2 + H2O

O3 + H2O + 2eă  O2 + 2OHă

3PbS + 4O3  3PbSO4

6NO2 + O3  3N2O5

S + H2O + O3  H2SO4

2KOH + 5O3  2KO3 + 5O2 + H2O

O3 + 2KI + H2O  I2 + 2KOH + O2

3SnCl2 + 6HCl + O3  3SnCl4 + 3H2O

2[Fe(CN)6]4ă + H2O + O3  2[Fe(CN)6]3ă + O2 + 2OHă

P-BLOCK ELEMENTS
48 QUIZRR

Potassium ozonide KO3 is an orange coloured solid and contains the paramagnetic O3 ion.

Ć The amount of O3 in a gas mixture can be determined by passing the gas into a KI solution
(at a constant pH 9.2 using borate buffer). The iodine that is liberated is titrated with
sodium thiosulphate solution.
O3 + 2Iă + H2O  I2 + 2OHă + O2
2 ă 2
I2 + 2S2O3  2I + S 4 O6
Ć O3 also adds to unsaturated organic compounds at room temperature forming ozonides
which can be cleaved to aldehydes and ketones in solution :
O

CH2 CH2
H2 O/Zn
CH2 CH2 + O3 2HCHO
O O

Ć O3 is used as a disinfectant; it is used to purify drinking water since, it can destroy bacteria
and viruses. It is better than Cl2 since, it (O3) avoids the unpleasant smell and taste of Cl2
and any excess O3 soon decomposes to O2.

H2O/Zn
CH · CH = CH · CH3  O3   2CH3 CHO

KClO3

 MnO2
electrolysis
Air O2 H2O

strong
electric field
CH2=CH2 Iă
HCHO O3 I2
H2O/Zn (HCl+
SnCl )
4ă NO 2
]
(CN) 6 o
dest zone la SnCl4
[Fe ă PbS royi y
3ă OH S H2O ng r er O2+NO2
[Fe(CN)6] eact
H2

NO2 ions
KO
S

PbSO4 H2SO4 N2O5 H2SO4 KO3

The molecules O3 is bent with an OOO angle 116.8Ĉ and equal OăO distance of 128 pm and
may be described 90Ĉ canonical forms.

O O+ O+
O 116.8Ĉ O O O O O
218pm

P-BLOCK ELEMENTS
QUIZRR 49

Allotropy and Polymorphism of Sulphur

Many different molecular species are possible for the elemental sulphur, and this accounts for the
existence of the large number of physical forms in which the element may appear. Allotropy is
more complex for the element sulphur than for any other element of group 16 (VI). Sulplhur may
exist as :

Ć rhombic sulphur (S) which has sixteen S8 rings in a unit cell

Ć monoclinic sulphur (S) which has six S8 rings in its unit cell.

Ć liquid sulphur (S ) comprised of S8 molecules. This is yellow, transparent, mobile liquid.
At 160ĈC, a remarkable transformation occurs. The S8 rings open up and join together into
long spiral-chain molecules, resulting in :

Ć liquid sulphur (Sø) which is dark in colour and very thick and viscous. The chain
length and viscosity of the liquid reach a maximum at about 180Ĉ C. At higher
temperatures the chains break up and viscosity decreases again. At 445ĈC, this liquid boils,
producing :

Ć sulphur vapours, which consists of molecules ranging from S2 to S10, but predominantly
S8. At higher temperatures, S2 predominates. S2 is paramagnetic.

Ć Plastic sulphur forms if liquid sulphur (ø) is poured into cold water. It consists of chain-
like molecules and has rubber-like properties when first formed. On standing, it becomes
brittle and finally converts to rhombic sulphur. Above transformations can be written
as :

 S8  g 
95.5C 160C 445C
S   S 
 S 

2000C 1000C
S  S2  S4  S6

Ć If rhombic sulphur is heated rapidly, it fails to convert into monoclinic sulphur and melts
at 113ĈC.

Ć In cyclo-S 6, the the ring adopts the chair form chain polymers, catena-S n are also
known.

P-BLOCK ELEMENTS
50 QUIZRR
C ompound of Sulphur

Oxide :
Sulphur forms several oxides of which sulphur dioxide (SO2) and sulphur trioxide (SO3) are
important.

Sulphur dioxide

Preparation :
4FeS2 + 11O2  2Fe2O3 + 8SO2

Properties :

1. As reducing agent
(i) Action on halogens :

SO2 + Cl2 + 2H2O  H2SO4 + 2HCl

(ii) Action on FeCl3

2FeCl3 + SO2 + 2H2O  H2SO4 + 2FeCl2 + 2HCl

2. Reaction with acidified KMnO4

2KMnO4 + 5SO2 + 2H2O  K2SO4 + 2MnSO4 + 2H2SO4

3. Reaction with acidified K2Cr2O7

K2Cr2O7 + 3SO2 + H2SO4  K2SO4 + Cr2(SO4)3 + H2O

4. Oxidising property

(i) 2H2S + SO2  2H2O + S 

(ii) SO2 + 2Mg  2MgO + S 

Sulphur trioxide

Preparation :
By dehydration of H2SO4

P O
H2SO4 
2 5  SO  H O
3 2
heat

Reaction : H2SO4 + SO3  H2S2O7

H2SO4 saturated with SO3 is called oleum.

P-BLOCK ELEMENTS
QUIZRR 51

Oxyacids of sulphur :
Based on the structural similarities, oxoacids or (oxyacids) of sulphur may be classified as :
1. Sulphoxylic acid, H2SO2
2. Sulphurous acid series
(a) Sulphurous acid, H2SO3
(b) Thiosulphurous acid, H2S2O2
(c) Dithionus acid, H2S2O4
(d) Di or pyrosulphurous acid H2S2O5
3. Sulphuric acid series
(a) Sulphuric acid, H2SO4
(b) Thiosulphuric acid, H2S2O3
(c) Di or pyrosulphuric acid, H2S2O7
4. Peroxo-acid series
Peroxomonosulphuric acid (also called CaroÊs acid, H2SO5)
Perdisulphuric acid (also called MarshallÊs acid, H2S2O8)

O HO O
S
HO HO
HO S
S O S S S OH HO S O

HO HO
O O
Sulphurous acid Thiosulphurous acid Dithionous acid Pyrosulphurous acid

HO O
HO O HO S
S
S S
O O
O OH O OH
Sulphuric acid Thiosulphuric acid O S O

OH
Pyrosulphuric acid

O O O O

HO S S OH HO S (S)n S OH

O O O O
Dithionic acid Polythionic acid

P-BLOCK ELEMENTS
52 QUIZRR
O O O

HO O S OH HO S O O S OH

O O O
Peroxomonosulphuric acid Peroxodisulphuric acid

(Structure of oxoacids of sulphur)

Sulphuric Acid (H2SO 4)

Ć As an oxidizing agent

2HBr + H2SO4  Br2 + SO2 + 2H2O

K4[Fe(CN)6] + 6H2SO4 + 6H2O  2K2SO4 + FeSO4 + 3(NH4)2SO4 + 6CO

8HI + H2SO4  H2S + 4I2 + 4H2O

Ć As a dehydrating agent
Conc. H2SO4
CH3CH2OH   CH2 = CH2 + H2O

Ć HO ă SO2 ă OH + 2PCl5  SO2Cl2 + 2POCl3 + 2HCl


Sulphuryl Chloride

O O

O S OH S
ă
O O
ă
OH O

S-atom is sp3 hybridised. SO24 ion is tetrahedral.

Example 17

H2SO4 cannot be used to obtain HBr from KBr. Why ?


Solution :
H2SO4 oxidises HBr.

Sodium thiosulphate (Na2S2O3.5H2O) :


If one of the oxygen atoms in the sulplhate ion is replaced by sulphur, the resulting ion (S2O32ă) is
known as thiosulphate.

P-BLOCK ELEMENTS
QUIZRR 53

Preparation :

(i) Sodium thiosulphate is prepared by boiling aq. solution of metal sulplhites with elemental
sulphur.

1 373K
Na 2 SO3  S8  Na 2S2O3
8 H2O

Hydrated sodium thioosulphate Na2S2O3.5H2O is known as HYPO.


(ii) SpringÊs reaction may be used for the preparation of sodium thiosulphate. It consists in
treating a mixture of sodium sulphide and sodium sulphite with calculate quantity of iodine.

Na 2 S + Na 2 SO3  I2  Na 2 S2 O3  2NaI

Properties :

(i) Reaction with dilute acids : It reacts with dilute acids to liberate sulphur dioxide gas along
with precipitate of sulphur.

Na2S2O3 + 2HCl  2NaCl + H2O + S + SO2

(ii) Reaction with BaCl2 : It gives white ppt. of barium thiosulphate.

S 2 O32  Ba 2  BaS 2 O3 


White

(iii) Reaction with silver nitrate solution : Gives white ppt. which quickly changes to yellow,
brown and finally black due to the formation of silver sulphide.

S2 O23  2Ag   Ag 2S2 O3 


White ppt.

Ag2S2O3 + H2O  Ag2S + H2SO4


With conc. solution of sodium thiosulphate, silver nitrate gives no ppt.
(iv) It reacts with silver salts to form sodium argento thiosulphate complex

AgBr + 2Na2S2O3  Na3 [Ag(S2O3)2] + NaBr


Sodium argento
thiosulphate complex

(v)  2
Thiosulphate S2O3  ion is oxidized by iodine I2 to tetrathionate S 4 O62

2Na2S2O3 + I2  Na2S4O6 + 2NaI

P-BLOCK ELEMENTS
54 QUIZRR
Example 18
Why sodium thiosulphate is used in photography ?
Solution :
It can remove the undecomposed AgBr as soluble thiosulphate complex.

2Na2S2O3 + AgBr  Na3 [Ag(S2O3)2] + NaBr

Hydrogen sulphide
Preparation :
Prepared by the action of dil. HCl or H2SO4 on iron sulphide

FeS + 2HCl (dil)  FeCl2 + H2S 

Properties :
It is a colourless, poisonous gas having the smell of rotten eggs.
1. As reducing agent
(i) Action on halogens

H2S + Cl2  2HCl + S 

(ii) Action on FeCl3

2FeCl3 + H2S  2FeCl2 + 2HCl + S 

2. Reaction with acidified KMnO4

2KMnO4 + 3H2SO4 + 5H2S  K2SO4 + 2MnSO4 + 8H2O + S 

3. Reaction with acidified K2Cr2O7

K2Cr2O7 + 4H2SO4 + 3H2S  K2SO4 + Cr2(SO4)3 + 7H2O + S 

Example 19
The two atoms of sulphur in Na2S2O3 have
(A) + 2 and + 4 oxidation state (B) ă 2 and + 6 oxidation state
(C) + 4 and + 6 oxidation state (D) same oxidation states
Solution : (B)

Example 20
Bleaching action of SO2 is due to
(A) reduction (B) oxidation
(C) hydrolysis (D) its acidic nature
Solution : (A)

P-BLOCK ELEMENTS
QUIZRR 55

Example 21

When sulphur is boiled with Na2SO3 solution, the compound formed is


(A) sodium sulphide (B) sodium sulphate
(C) sodium persulphate (D) sodium thiosulphate
Solution : (D)

Example 22

The structure of dithionate ion is


2ă 2ă
O O O O
O S S O O S S O
(A) (B)
O O O O O

2ă 2ă
O S

O S S O
S
(C) O O S O (D)
O O
O O

Solution : (B)

Example 23

When SO2 gas is passed into an acidified solution of K2Cr2O7, the oxidation state of
chromium changes
(A) from + 12 to + 6 (B) from + 3 to + 6
(C) from + 6 to + 3 (D) from + 6 to + 4
Solution : (C)

Example 24

The stability of the hydrides of oxygen, sulphur, selenium and tellurium decreases in
the order

(A) H2Te > H2Se > H2S > H2O (B) H2O > H2S > H2Se > H2Te

(C) H2O > H2S > H2Se > H2Te (D) H2O  H2S > H2Te > H2Se

Solution : (B)

P-BLOCK ELEMENTS
56 QUIZRR
Example 25

Which of the following reaction does produce SO2 ?

(A) 
Ag + H2SO4 (conc.)  
(B) CaSO4 + C 
 

(C) 
FeS2 + O2  
(D) Zn + H2SO4 
 
Solution : (D)

Example 26
In preparing a standard aqueous solution of MohrÊ salt, [FeSO4.(NH4)2SO4. 6H2O] a few millitres
of H2SO4 is added to the solution. The added H2SO4
(A) prevents reduction of the salt
(B) prevents oxidation of the salt
(C) makes the solution homogeneous by converting insoluble Fe(OH)2, which is formed by
hydrolysis, into soluble FeSO4
(D) neutralizes the ammonia formed by hydrolysis
Solution : (C)

Group 17 (Halogens)

Halogens (X2)
Ć F2 is extremely reactive, and this causes great difficulties in the preparation and handling of the
element.
ă Moissan (1886) prepared it from CaF2 :

CaF2 + H2SO4  CaSO4 + 2HF

KF + HF  KHF2

electrolysis
KHF2 
 H2  F2
at cathode at anode

ă Modern method uses graphite anode and steel cathode in a steam heated steel tank.
ă If F2 were formed by the electrolysis of aqueous fluoride solution, it would immediately
oxidise water to O2

F2 + 2H2O  4HF + O2

Ć The commercial production of Cl2 is by electrolysis, either of concentrated aqueous NaCl or of


molten salt such as NaCl(l) or MgCl2(l) :

electrolysis
2NaCl + 2H2O 
 2NaOH + H2  Cl
2

at cathode at anode

P-BLOCK ELEMENTS
QUIZRR 57

Ć Br2 is obtained from sea water and brine lakes. Sea water contains about 65 ppm Br. Br2 is
isolated in following which oxidises Bră to Br2.

Cl2 + 2Bră  Br2 + 2Clă


ă Br2 is quite volatile hence is removed by a stream of air.
ă Br2 is absorbed into aq Na2CO3 solution when NaBr and NaBO3 are formed :

3Na2CO3 + 3Br2  5NaBr + NaBrO3 + 3CO2

or 3Br2 + 6OHă  5Bră + BrO3ă + 3H2O

Ć I2 is obtainable in small quantities from dried seaweed, since certain marine plants absorb and
concentrate Iă selectively in presence of Clă and Bră. Low concentration of Iă are also found in
some natural brines (salt solutions) associated with oil fields.
Chile saltpetre is mainly NaNO3 but it contains traces of sodium iodate NaIO3 and sodium
periodate NaIO4.I2 is recovered by reducing IO3 to Iă by NaHSO3 and then oxidation of Iă to I2
by IO3 itself :

IO3 + 3HSO3  I + 3SO24 + 3H


ă +

5Iă + IO3 + 6H+  3I2 + 3H2O

Natural brine is treated with Cl2 water to oxidise Iă to I2 and the solution is passed through an
ion-exchange resin which absorbs I2 as I3 .I3 is removed from the resin by treatment with alkali.

ă Reactivity of halogens is based on bond-energy (BE) value.


F2 > Cl2 > Br2 > I2
Greater the BE, smaller the reactivity
MnO2
conc. HCl

KMnO4 conc. HCl Cl2 electrolysis NaCl (aqueous or molten)

Bră
OHă
Br2 Bră + BrOă3

I2

lO3ă
ă ă
I IO3
in chile salt petre

P-BLOCK ELEMENTS
58 QUIZRR
Ć Oxidising power is in order :
F2 > Cl2 > Br2 > I2 .

1
X 2  solid, liquid, gas   X   hydrated 
2

F2 has the most negative GĈ value, hence is strongest oxidising agent and can oxidise Clă, Bră
and Iă to respectively Cl2, Br2 and I2. F2 can also oxidise H2O to O2.

1
F2 + H2O  2HF + O2
2

Similarly Cl2 will oxidise Bră and Iă, and Br2 will oxidise Iă. In general any halogen (X2) of lower
atomic number will oxidise halide ions (Xă) of higher atomic number.
The ionic character of the MăX bond decreases in the order MăF > M ă Cl > M ă Br > M ă I.
Reaction Comment
Ć With H2O

2F2 + 2H2O  4H+ + 4Fă + O2 Vigorous reaction with F2; atmospheric O2 can
oxidise Iă to I2 hence reverse reaction
2X2 + 2H2O  4H+ + 4Xă + O2 Cl2 > Br2 > I2 (F2 does not disproportionate).

disproportionate ă +
X2 + H2O   X + HOX + H
Ć With H2

H2 + X2  2HX All the halogoens with Br2 photochemical reaction,


with I2 very slow even at high temperature
Ć With metals

2M + nX2  2MNn Most metals form halides.


Ć With CO

CO + X2  COX2 Only Cl2, Br2 from carbonyl halide


Ć With P

2P + 3X2  2PX3 For F, Cl, Br, I

2P + 5X2  2PX5 For F, Cl, Br


Ć With S

2S + X2  S2X2 Cl, Br

S + 2Cl2  SCl4

S + 3F2  SF6

P-BLOCK ELEMENTS
QUIZRR 59

Ć With H2S

H2S + X2  2HX + S All the halogens oxidise H2S (S2ă) to S


Ć With SO2

SO2 + X2  SO2X2 F and Cl


Ć With NH3

8NH3 + 3X2  N2 + 6NH4X F, Cl, Br


Ć With halogens

X2 + X´2  2XX´ Interhalogen compounds


Ć With cold NaOH

2NaOH + 2F2  2NaF + OF2 + H2O

2NaOH + X2  NaX + NaOX + H2O (X = Cl, Br) I2 does not react under the same
Cl2 reacts with dry slaked lime forming condition
bleaching powder

Ca(OH)2 + Cl2  CaCl(OCl) + H2O


Ć With hot NaOH

6NaOH + 3X2  5NaX + NaXO3 + 3H2O (X = Cl, Br, I)

Cl2 is used as a bleach or as a disinfectant (as in public water supplies). It reacts slowly with H2O
to form HCl and HOCl. The hypochlorous acid then decomposes into HCl and O radicals, which
kill bacteria.
Cl2 + H2O  HCl + HOCl

HOCl  HCl + O


These oxygen radicals are very strong oxidising as well as effective bleaching and disinfecting
agent in aqueous solution of Cl2 or hypochlorite salts.
Ć The halogens react with each other to form interhalogen compounds of the type AX, AX3, AX5 and
AX7 of which A is of higher atomic number as shown :
AX AX3 AX5 AX7
CIF CIF3 CIF5
BrF BrF3 BrF5
ICl ICl3 (I2Cl6) IF5 IF7
IBr
BrCl

P-BLOCK ELEMENTS
60 QUIZRR
Ć There are never more than two different halogens in a molecule. The bonds are covalent because
of the small electronegativity difference. Ternary combinations occur in polyhalide ions (e.g.,
IBrClă)
ă The electrical conductance of molten I2 is ascribed to self-ionisation

3I2  I+3  I3

ă The interhalogens are generally more reactive than the halogens (except F2). This is because
the AăX bond in interhalogens is weaker than the XăX bond in the halogens.
ă Hydrolysis of interhalogens gives halide and oxohalide Oxohalide is always formed from
larger halogens present.

5F  BrO3
H O
BrF5 
2
bromate ion

H2O
HCl Clă + IO3ă
iodate ion

H2O
Clă + IOă
hypoiodite ion

Anomalous Behaviour of Fluorine

Fluorine differs from other elements of the group because of its exceptionally small atomic and
ionic size and low fluorineăfluorine bond dissociation energy. The result of these differences is
that fluorine is the most electronegative element in the periodic table and is a powerful oxidant.
Some differences between flourine and other halogens are :
Ć It is more reactive than other halogens because of low FăF bond energy, also due to its high
electronegativity the bond between fluorine and other elements is very strong so its compounds
are more stable. Some of them are inert. e.g. SF6.
Ć Fluorine is almost invariably monocoordinate (coordination number = 1) and is never more
than mono covalent.
Ć Fluorides are more ionic.
Ć Fluorine forms strong hydrogen bonds resulting in the properties of hydrogen fluoride being
anomalous.
Ć Fluorine is the strongest oxidising agent and oxidises the elements to their highest oxidation
state, for example in IF7, iodine has oxidation number seven.
Ć The reactions of fluorine are also different from other halogens. Fluorine fumes in moist air
and decomposes water to give oxygen, whereas other halogens are sparingly soluble in
water and react partly to give hydrohalic and hypohalous acids.

P-BLOCK ELEMENTS
QUIZRR 61

2F2 + 2H2O  4HF + O2

X2 + H2O  HOX + HX
Fluorine reacts with alkalies to yield the oxide, F2O :

2F2 + 2NaOH  OF2 + 2NaF + H2O

The other halogens react with alkalies to yield a solution of hypohalite ions, which may
disproportionate. The oxoacid of fluorine, HOF, prepared recently is very unstable. The oxides of
fluorine are not acidic.

Cl2 + 2NaOH  NaClO + NaCl + H2O

Halogen Acids

Ć All the halogens acids (HF, HCl, HBr, HI) are known. In aqueous solution, they are typical
mineral acids whose acid strengths decrease in the direction of increasing bond energy, that
is

HI > HBr > HCl  HF


  weak acid
strong acids

Ć HX can be prepared by direct combination of H2 and X2

H2 (g) + X2(g)  2HX (g)

ă reaction between H2 and F2 is very fast,


ă reaction between H2 and Cl2 is also rapid,
ă but reaction between H2 and Br2 or I2 is very slow.

G f of HF, HCl and HBr are large and negative suggesting that for them reaction

H2 + X2  2HX

goes to completion. For HI(g), G f is small and positive suggesting that HI should dissociate

2HI(g)  H2(g) + I2 (g)

to some extent. However, because of the high activation energy the dissociation occurs only
very slowly in the absence of a catalyst. As a result, HI(g) is quite stable at room temperature.
Thus, at room temperature decomposition of HI(g) is kinetically controlled (rather than
thermodynamically controlled).

Ć 
CaF2 + H2SO4   CaSO4 + 2HF

P-BLOCK ELEMENTS
62 QUIZRR
Ć CaF2 has also SiO2 as impurity otherwise. HF formed would react with SiO2 forming SiF4
and H2SiF6.

SiO2 + 4HF  SiF4 + 2H2O

SiF4 + 2HF  H2[SiF6]

150C
Ć NaCl + H2 SO4 
 HCl + NaHSO4
conc. salt cake

550 C
NaHSO4  NaCl 
 HCl + Na 2SO4

This method is called Âsalt-cakeÊ method involves formation of NaHSO4 (salt cake)
Ć HF is only just liquid at room temperature and HCl, HBr and HI are gases. HF has
abnormally high boiling point due to intermolecular H-bonding which is not possible in other
halogen acids due to their smaller electronegativity values.
HCl < HBr < HI < HF
Some H-bonding also occurs in the gas, which consists of a mixture of cyclic (HF)6 polymers,
dimeric (HF)2 and the monomeric HF.
ă

+
+ F  +
 H 
ă
H  H
134Ĉ
Fă F

Because of the tendency to form hydrogen bond, metal fluorides are solvated by HF giving

species of the type HF2 etc. KHF2 (KF. HF), KH2F3 (KF.2HF) and KH3F4 (KF.3HF) are

known examples.
Ć HF attacks glass, other halogen acids do not. Glass is silicates of sodium and calcium and
are converted into complex fluorides :

Na2SiO3 + 6HF  Na2SiF6 + 3H2O

CaSiO3 + 6HF  CaSiF6 + 3H2O

Hence HF canÊt be stored in a glass vessel. Instead, it is stored in a wax-vessel which does
not react with HF. This unique property of HF of attacking glass is used for making
permanent marking eg. glass-wares.
Ć In the gaseous state, HX are essentially covalent. In aqueous solution, they ionise to form
H2O+ and Xă but not H+ and Xă.

H2 O  HX  H3O+  X 
base acid acid base

P-BLOCK ELEMENTS
QUIZRR 63

HF has more ionic character (due to greater electronegativity difference) than HCl, HBr and

HF  HCl < HBr < HI


HI but acid strength varies as : weak  
strong

Acid strength is dependent not on the electronegativity difference of H and X but on the
tendency of the hydrated molecule to form hydrogen ions :

HX (hydrated)  H+ (hydrated) + Xă (hydrated)

Ć Dipole moment of HF is maximum and is in order

HF > HCl > HBr > HI


1.19D 1.05D 0.80D 0.420D

Preparations of oxo acids

Most of the oxo acids and acid anions of halogens are known only in solution. They can be
obtained by reaction of the free halogens with (i) water or (ii) aq. bases :

X2 + H2O  H+ + Xă + HOX

X 2  2NaOH  NaX + NaOX + H2 O


(cold dil.)

3X 2  6NaOH  5NaX + NaOXO2  3H2 O


(conc. hot)

(X = Cl, Br, I)
All the above reactions are rapid disproportionation of halogens.
Note :
(i) Reaction of X2 and water can produce HOX only.
(ii) There is appreciable concentration of HOCl.
(iii) The HOI is negligible in a saturated solution of iodine.

Halic acids HOXO 2 (X = Cl, Br, I)


All the three halic acids are known. The acids HOClO2 and HOBrO2 are known only in solution
and as salts. Only iodic acid (HOIO2) is stable out of aq. solution. It is a white solid. It can be
prepared by oxidizing I2 with conc. HNO3 or O3.

I2 + 10HNO3  2HIO3 + 10NO2 + 4H2O

HOClO 2 and HOBrO 2


They can be made by treating barium halates with H2SO4
Ba(ClO3)2 + H2SO4  2HClO3 + BaSO4 

P-BLOCK ELEMENTS
64 QUIZRR
Ba(ClO3)2 can be prepared by passing Cl2 in Ba(OH)2 solution

6Ba(OH)2 + 6Cl2  Ba(ClO3)2 + 5BaCl2 + 6H2O

They can also be produced by reactions of X2 with hot aq. base (NaOH)

3X2 + 6HOă  XO3 + 5Xă + 3H2O

KClO3 decomposes on heating, the product depends on temperature

400 ă 500Ĉ C
(i) 2KClO3 2KCl + 3O2
[MnO 2]

150Ĉ C

Lower
(ii) 4KClO3 
 3KClO4  KCl
Temp.


Zn(ClO3 )2  2ZnO + 2Cl 2  5O2

Chlorates are used in matches and fireworks. Sodium chlorate is used as a powerful weed killer.

Perbromic acid HOBrO3

It is best prepared by oxidation of BrO3 by F2 in 5M NaOH solution.

BrO3  F2  2HO   BrO4  2F   H2 O

Solution of HBrO4 can be concentrated upto 55% (6 M), without decomposition. The hydrate
HBrO4.2H2O can be crystallized.

Periodic acid HO IO3

The common form in aq. solution of this acid is HIO4 and also H5IO6. H5IO6 is called paraperiodic
acid (or orthoperiodic acid). It is a white crystal. It is decomposes on heating as

100C ~ 200C
2H5IO6   2HIO4 
 I 2 O5 + O2 + H 2 O
  4H2O 

The IO4 ion is tetrahedral while (OH)5IO is octahedral. Periodates can be made by oxiding iodate

with Cl2 in alkaline solution.

IO3  Cl2  6HO  IO65  2Cl  3H2O

C ă OH ·C=O
HIO4
+ HIO3
· C ă OH ·C=O

P-BLOCK ELEMENTS
QUIZRR 65

Structure of oxo anions

Anions XOă XO2 XO3 XO4

Total valence electrons 14 20 26 32


No. of bonds (= no. of bps) 1 2 3 4
Electrons for bonds 8 16 24 32
Lone pairs (14 ă 8)/2 = 3 (20 ă 16)/2 = 2 (26 ă 24)/2 = 1 0
Total electrons pairs 1+ 3= 4 2+ 2= 4 3+ 1= 4 4 ă 0 = 4
Structure Td Td Td Td
Shape Linear Angular  pyramidal Td

XOă XO2ă XO3ă XO4ă


O

X X X X
O O O O
O O

O O O
(Linear) (Angular) (Pyramid) (Tetrahedral)

Structure and shape


Total
Molecules Bond Lone electron Structure Shape
pairs pairs pairs

X
XX´ 3 3 2 5 X´ T ă shaped
(tbp) X´

X´ X´
X
XX´ 5 5 1 6 Square
X´ X´ pyramid
(Oh)

X´ X´

XX´ 7 7 0 7 X Pentagonal

bipyramid

(pbp)

P-BLOCK ELEMENTS
66 QUIZRR
P seudohalogens and Pseudohalides

Many substances have properties either like X2 or Xă (X = halogen). They are called pseudohalogens
and pseudohalides. They contain two or more atoms in which one at least is N ă atom, such as
(CN)2, CNă, SCNă etc.
Some pseudohalogens and pseudohalides

Pseudohalogens Pseudohalides

Cyanide ion CNă

Cyanamide ion CN 22

Cyanogen gas (CN)2 Azid ion N3


Thiocyanogen (SCN)2 Thiocyanate ion SCNă
Cyanate ion OCNă
Fulminate ion ONCă

The cyanogen gas and CNă ion are best-known pseudohalogen and pseudohalide. They behave
like X2 and Xă, e.g.,

(a) (CN)2 disproportionate in water like X2


(CN)2 + H2O  HCN + HOCN
Cl2 + H2O  HCl + HOCl

(b) CNă behaves similar to Iă

(aq)  4CN(aq)  2CuCN +  CN 2



2Cu2+


2Cu2+
(aq)  4I  2CuI + I 2

Both the above reactions are reduction of Cu2+ to Cu+ by CNă and Iă ions.

(c) The solubility of metal halides and metal cyanides are very similar,
NaX and NaCN  Water-soluble
AgX and AgCN  Water-insoluble
(X = Cl, Br, I)
PbCl2 and Pb(CN)2  Water insoluble

(d) Like halides pseudohalides (CNă, SCNă etc.) are good ligands CuCl24 , ZnCl24 , Cu(CN)24  .

{Please note, the word ligands will be dealt in chapter co-ordination complexes}
(i) CNă complexes with transition metals and Zn, Cd and Hg.
(ii) Most of the CNă complexes are ionic [Fe(CN)6]4ă, [Ni(CN)4]2ă, [Mn(CN)6]4ă etc.
(iii) CNă can stabilize low oxidation states also, e.g [Ni(CN)4]4ă.

P-BLOCK ELEMENTS
QUIZRR 67

Bleaching Powder

The exact chemical composition of bleaching powder is not yet known but it behaves as if it
contains calcium hypochlorite Ca(OCl)2 and basic calcium chloride, Cl2.Ca(OH)2.H2O

Preparation

It is prepared by passing chlorine over slaked lime

3Ca(OH)2 + 2Cl2  Ca(OCl) 2  CaCl2



.Ca(OH)2 .H2O + H2 O
 
Bleaching Powder

Properties :

1. Reaction with dilute acids : With dilute acids, it gives chlorine which is known as
available chlorine.

CaOCl2 + 2HCl  CaCl2 + H2O + Cl2 

CaOCl2 + H2SO4  CaSO4 + H2O + Cl2 

2. When treated with water it decomposes into calcium chloride and calcium hypochlorite.

2CaOCl2 + H2O  CaCl2 + Ca(OCl)2 + H2O

3. Bleaching powder reacts with CO2 (atmospheric) and gives chlorine which accounts for its
oxidising and bleaching actions.

CaOCl2 + CO2  CaCO3 + Cl2 

4. Action of heat : On heating bleaching powder gives a mixture of chlorate and chloride.


6CaOCl2  Ca(ClO3 ) 2  5CaCl 2

Group 18 (The Noble Gases)

Ć Because of highest ionisation energy of group 18 elements in their respective periods, they
were given the name Ânoble gasesÊ and thus inert. However, at least in the case of xenon,
it is not so; it forms series of compounds.
Ć The noble gases are colourless, tasteless and odourless. In the liquid and solid states the only
forces of attraction among the atoms are very weak London or van der Waals forces.
Polarisability and interatomic interactions increase with increasing atomic number. The
attractive forces among the atoms are so small that they remain liquid at 1 atmospheric
pressure even at a temperature of 0.001 K.

P-BLOCK ELEMENTS
68 QUIZRR
Ć Helium has the lowest boiling point of any element, and its behaviour as a liquid is most
unusual. It will creep up the walls of the container in which it is placed and thus it exists
in two phases in liquid helium, one of which has an extremely low viscosity.

X eon fluorides

Xenon reacts directly with fluorine (only). The products depend on respective amounts of Xe and
F2 and temperature also. The reactions are generally carried out in seal tubes.

2:1
XeF2
/P
1:5
Xe + F2 XeF4
/P
1 : 20 XeF6
/P

  heart
P  Pressure
These three fluorides are crystalline volatile substances which sublime at room temperature but
can be kept in nickel containers.
The fluorides are strong oxidising agents.

XeF2 + 2HCl  Xe + Cl2 + 2HF

XeF4 + 4KI  Xe + 2I2 + 4KF

XeF2 + Ce2(SO4)3 + SO24  Xe + 2Ce (SO4)2 + F2


They are good fluorinating agents also.

2SF4 + XeF4  2SF6 + Xe

Pt + XeF4  Pt F4 + Xe

+ XeF2 + Xe + HF

Their reaction with water is different


(a) XeF2 is water soluble and undergoes slow oxidative hydrolysis.

2XeF2 + 2H2O  2Xe + O2 + 4HF

P-BLOCK ELEMENTS
QUIZRR 69

(b) XeF4 reacts with water violently to give XeO3

3
3XeF4 + 6H2O  2Xe + XeO3 + O2 + 12HF
2

In this reaction XeF4 disproportionates and H2O is parthy oxidized.


(c) XeF6 also reacts violently to give XeO3

XeF6 + 6H2O  XeO4 + 12HF


With small amount of water

XeF6 + H2O  XeOF4 + 2HF

Example

Xe cannot form XeF3, XeF5 type of compound.


Solution :
The electronic configuration of Xe is 5S2 5p6. So, all the electrons are paired up and when it is
excited, then either 2 or 4 or 6 etc. electrons will be unpaired. There is no scope to get odd number
of unpaired electrons in its outer most shell. For this reason it cannot form XeF3 or XeF5 type of
compound.

P-BLOCK ELEMENTS
S-BLOCK
ELEMENTS
QUIZRR 3

S-BLOCK ELEMENTS

Introduction

Shells upto (n – 1) are completely filled and differentiating (last) electron enters into ns sub-or
bit. Elements under this class are called S-Block elements
• Alk a li m et a ls forms compounds with + 1 oxidation state and those of a lka line ea r t h
met a ls + 2 oxidation state, ions having nearest noble gas configuration
• General electronic configurations is
[Inert gas] ns 1 - Alkali metals,
[Invert gas] ns 2 - Alkaline earth metals;

ALKALI METAS
Physical Properties

(a ) All the alkali elements are silvery white solids. These are soft in nature and can be cut with
the help of knife except lithium. When freshly cut, they have a bright lustre which quickly
fades due to surface oxidation. These are highly malleable and ductile.
The silvery lustre of alkali metals is due to the presence of highly mobile electrons of the
metallic lattice. There being only a single electron per atom, the metallic bonding is not so
strong. As a result of this, these metals are soft in nature. However, the softness increases
with increase of atomic number because there is continuous decrease of metallic bond strength
on account of an increase in atomic size. Bibber is the size of metal kernel weaker is the
metallic bonding.
(b) At omic a nd ionic r a d ii : Group IA atoms are the largest in their horizontal periods in
the periodic table. When the outermost electron is removed to give a positive ion, the size
decreases considerably. There are two reason for this :
(i) The outermost shell of electrons has been completely removed.
(ii) The positive charge on the nucleus is now acting on lesser number of electrons, i.e.,
attraction increases which bring contraction in size.
Atomic as well as ionic size increases for Li to Fr due to the presence of one extra shell of
electrons.
Li Na K Rb Cs
At. radii (A) 1.23 1.57 2.03 2.16 2.35
Ionic radii (A) 0.60 0.95 1.33 1.48 1.67
Atomic volume also increases as the atomic number increases.
Li Na K Rb Cs
At. volume (mL) 13.0 23.7 44.4 55.8 69.3

S-BLOCK ELEMENTS
4 QUIZRR
(c) Densit y : All are light metals. The densities are low. Lithium, sodium and potassium are
lighter than water. Density gradually increases in moving down from Li to Cs. Potassium
is, however, lighter than sodium.

Li Na K Rb Cs
Density (g/mL) 0.54 0.97 0.86 1.53 1.87

The reason for the low values is that these metals have high atomic volumes. The abnormal
value of potassium is due to unusual increase in atomic size. i.e., atomic volume.
(d) M elt ing a nd boiling point s : The energy binding the atoms in the crystal lattices of these
metals is relatively low on account of a single electron in the valency shell. Consequently,
the metals have low melting and boiling points. These decrease in moving down from Li to
Cs as the metallic bond strength decreases or cohesive force decreases.

Li Na K Rb Cs
Melting point (°C) 18 1 98 63 39 28.5
Boiling point (°C) 1347 88 0 76 6 68 8 70 5

(e) Ionisa t ion ener gies a nd electr oposit ive cha r a ct er : Due to their large size, the outermost
electron is far from the nucleus and can easily be removed. Their ionisation energies or
ionisation potentials are relatively low. Thus, the metals have a great tendency to loss the
ns 1 electron to change into M+ ions. These metals are highly electropositive in nature. As
the ionisation potential decreases from Li to Cs, the electropositive character increases, i.e.,
metallic character increases. The reactivity of the these metals increases from Li to Cs.
Li Na K Rb Cs
Ionisation potential (eV) 5.4 5.1 4.3 4.2 3.9
>
Decreases
Electropositive or
>
metallic nature Increases
>
Reactivity Increases
The ns 1 electron is so loosely held that even the low energy photons (light) can eject this
electron from the metal surface. This property is termed as phot oelect r ic effect . K and Cs
are used in photoelectric cells which are sensitive to blue light.
(f) Oxidation states : The alkali metals can lose their ns1 electron quite easily to form univalent
positive ion, M+ . The ion has a stable configuration of an inert gas.
Li + He-configura tion
+
Na Ne-configuration
K+ Ar-configura tion
+
Rb Kr-configuration
Cs + Xe-configuration
S-BLOCK ELEMENTS
QUIZRR 5

The energy required to pull out another electron from M + ion is very high, i.e. the second
ionisation potential values are high

Li Na K Rb Cs
I2 (eV) 75.6 47.4 32.0 28.0 25.0

Consequently, it is not possible for alkali metals to form M2+ ions under ordinary conditions.
These metals, thus show only one oxidation state, i.e., +1 oxidation state. These metals are
univalent in nature and show electrovalency, i.e., form electrovalent compounds.
Since the electronic configuration of M+ ions are similar to those of inert gases, these ions
have no unpaired electrons and consequently are colour less and d ia m a gnet ic in nature.

(g) H ydr a t ion of ions, hyd r a t ed r a dii hyd r a t ion ener gy : The salts of alkali metals are
ionic and soluble in water. The solubility is due to the fact cations get hydrated by water
molecules.

M   a q.  [M ( a q.)]
(Hyd r a t ed ca tion )

The smaller the cation, the greater is the degree of its hydration. Thus, the degree of
hydration of M+ ions decreases from Li + to Cs + . Consequently the radii of the hydrated ion
decreases from Li + to Cs + .

[Li(a q.)] + [Na(a q.)] + [K(a q.)] + [Rb(a q.)] + [Cs(a q.)] +
Hydrated radii (A) 3.40 2.76 2.32 2.28 2.27

The ionic conductance of these hydrated ions increases from [Li(a q.)] + to [Cs(a q.)] +
Hydration of ions is an exothermic process. The energy released when one gram mole of an
ion is dissolved in water to get it hydrated is called hydration energy. Since the degree of
hydration decreases from Li + to Cs + , the hydration energy of alkali metal ion also decreases
from Li+ to Cs+ .

(h) E lect r onega t ivit y : The tendency to attract electrons is low as the alkali metals are
electropositive. The electronegativity, thus, decreases from Li to Cs as the electropositive
character increases.

(i) C ond uct ivit y : The alkali metals are good conductors of heat and electricity. This is due
to the presence of loosely held valency electrons which are free to move throughout the
metal structure.

(j) Specific hea t s : The specific heat values decrease from Li to Cs.
Li Na K Rb Cs
Specific heat at 0°C 0.94 0.29 0.17 0.08 0.05

S-BLOCK ELEMENTS
6 QUIZRR
(k ) Hea t of a t omisa t ion : Heat of atomisation decreases from Li to Cs.
Li Na K Rb Cs
Heat of atomisation 163.6 104.2 83.2 79.5 75.7
(kJ mol –1 )
This is due to the decrease in the metallic bond strength from Li to Cs.

(l) F la m e colour a t ion : The alkali metals and their salts impart a characteristic colour to
flame.
The reason for flame colouration is that the energy of the flame causes an excitation of the
outermost electrons which on return to their original position give out the energy so absorbed
in the visible region. The energy released in minimum in the case of Li + and increases in
order Li+ , Na +, K+ , Rb+ , Cs+ . Thus, the frequency of the light emitted increases in accordance
with the formula E = hv. The frequency of light in lithium is minimum which corresponds
to red region of the spectra.

(m ) R educing na t ur e : An element, which acts as a reducing agent, must have low ionisation
energy. Alkali metals act as strong reducing agents as their ionisation energy values are
low. Since ionisation energy decreases on moving down from Li to Cs, the reducing property
increase in the same order. This, Li is weakest reducing agent while Cs is the strongest
reducing agent amongst alkali metals in free state.
The tendency of an element to lose an electron in solution is measure by its standard
O O
oxidation potential value ( E OX ). Since alkali metals have high E OX values, these are strong
reducing agents. However, it is observed that Li is the strongest reducing agent amongst
O
alkali metals in solution as E OX value of Li is maximum.

Li Na K Rb Cs
Oxidation potential (V) +3.05 +2.71 +2.93 +2.99 +2.99

It looks surprising at first sight that lithium having high value of ionisation energy amongst
alkali metals acts as strongest reducing agent in solution. This can be explained if we
understand the fact that ionisation energy is the property of an isolated atom in gaseous
state while oxidation potential is concerned when the metal atom goes into the solution.
The ionisation energy involves the change to gaseous atom to gaseous ion,

 M  ( g)  e
M ( g ) 

while oxidation potential involves the following change :

M (s )  M  (a q.)  e

S-BLOCK ELEMENTS
QUIZRR 7

The above change occurs in three steps :

(i) M (s )  M ( g ) – sublimation energy

(ii)  M  ( g ) + e–ionisation energy


M ( g ) 

(iii) M  ( g )  H 2 O  M  (a q.) + hydration energy

Sublimation energy is nearly same for all the alkali metals.


The energy required in (ii) step is the ionisation energy which is highest for lithium. Li+ ion
is smallest in size, hence it has maximum degree of hydration. Thus, in (iii) step, maximum
hydration energy is released in lithium.
Li Na K
Step (i) +122 kJ +78 kJ +61 kJ
Step (ii) +520 kJ +496 kJ +419 kJ
Step (iii) – 480 kJ –371 kJ – 299 kJ
overall 162 kJ 203 kJ 181 kJ

Thus, with great ease the following overall change occurs in lithium and it acts as a strongest
reducing agent :

M (s )  M  (a q.)  e

I t is, t her efor e, conclud ed t ha t highest r ed ucing p ower of lit hium in solut ion is d ue t o
it s la r ge hea t of hyd r a t ion.

Chemical Properties of Alkali Metals


The alkali metals are highly reactive metals and the reactivity increases down the group. The
reactivity is due to
(a ) low value of first ionization energy
(b) large size
(c) low heat of atomization.
(1) R ea ct ion wit h a ir : All alkali metals except lithium tarnish in air due to formation of an
oxide or hydroxide on the surface. These are, therefore, always kept under kerosene oil to
protect them from air.

4 M  O2  2 M 2 O

(where M = Li, Na, K, Rb, Cs)


4Li + O2 (excess)  2Li2 O (Lithium oxide)

2Na + O 2 (excess)  Na 2 O 2 (Sodium peroxide)

S-BLOCK ELEMENTS
8 QUIZRR
K + O2 (excess)  KO2 (potassium superoxide)
This difference is due to small size, Li + has a strong positive field around it which attracts
the negative charge so strongly that is does not permit the oxide onion O2– to combine with
+
another oxygen to form peroxide ion, O 2–
2 . On the other hand, Na ion because of its large
size that Li+ ion has comparatively weaker positive field around it. which cannot prevent O2
+ + +
ion to combine with another oxygen to form peroxide ion O 2–
2 . The larger K , Rb and Cs
ions have still weaker positive field around then which cannot prevent even peroxide ion,
O 2–
2 to combine with another oxygen aton to form superoxide O 2– .

(2) R ea ct ion wit h wa t er : Alkali method decompose water with the evolution of hydrogen

2M + 2H2 O  2MOH + H2 ()


Lithium decomposes water slowly. Sodium reacts with water quickly, K, Rb and Cs react
with water vigorously. The decomposition is highly exothermic and the evolved hydrogen
sometimes inflames. Thus, reactivity towards water increases from Li to Cs. This is due to
electropositive character in same order.
Alkali metals react with other compounds containing acidic hydrogen.

2Na + 2HCl  2NaCl + H2 ()

2Na + 2CH  CH  2NaC  CH + H2 ()

(3) Affinit y for non-m et a ls : Alkali methods have great affinity for non-metals.
(i) Act ion wit h hyd r ogen

2M + H2  2MH

MH + H2O  MOH + H2 ()


(ii) Act ion wit h ha logens :

2M + X2  2MX
With the exception of certain Lithium halides, the alkali metal halides are ionic compounds
(M+ X– )
Halides of potassium, rubidium and caesium have as property of combining with extra
halogen atoms forming poly halides.

KI + I2  KI 3

Note : The insolubility of LiF in water can be explained in the following manner. The
lithium ion has the highest energy of hydration as it is small in size in comparison to
other alkali metal ions and thus it should have high solubility. However, the small Li +
and F – ion interact very strongly resulting in high lattice energy of LiF which is

S-BLOCK ELEMENTS
QUIZRR 9

responsible for its insolubility (the ionic compounds, which posses low value of lattice
energy, are freely soluble in water)

(iii) Act ion wit h sulp hur a nd p hosp hor us : Alkali metals react with sulphur and
phosphorus on heating to form sulphides and phosphides

16Na + S8   8 Na 2 S
Sodium sulphide

12Na + P4   4Na 3 P
Sodium phosphide

(4) Solub ilit y in liq uid a m monia : All alkali metal dissolve in liquid ammonia giving deep
blue solutions which are conducting in nature. These solutions contain ammoniated cations
and ammoniated electrons as shown below :

M + (x + y) NH 3  M+ + e–1 (NH 3) y


The blue colour of the solution is considered to be due to ammoniated electrons which absorb
energy corresponding to red region of the visible light for the their excitation to higher
energy levels. The transmitted light is blue which imparts blue colour to the solutions. The
electrical conductivity of the solution is due to both ammoniated cations and ammoniated
electrons. The blue solution on standing slowly liberates hydrogen resulting in formation of
amide :

2M  2NH 3  2MN H 2  H 2


(Met a l a m ide)

At concentration above 3M, the solutions of alkali metals in liquid ammonia are copper-
bronze coloured. These solutions contains clusters of metal ions and hence possess metallic
lusture. The blue coloured solutions are paramagnetic due to presence of large number of
unpaired electrons, but bronze solutions are diamagnetic due to formation of electron clusters
in which ammoniated electrons with opposite spin group together
These solutions are stronger reducing agents than hydrogen and hence will react with water
to liberate hydrogen.

General Characteristic of the Compounds of the Alkali Metals

(i) O xid e a nd H yd r oxides


All the alkali metals, their oxides, peroxides and superoxides readily dissolve in water to
produce corresponding hydroxides with are strong alkalies eg
2Na + 2H 2 O  2NaOH + H 2

Na 2 O + 2H 2 O  2NaOH

S-BLOCK ELEMENTS
10 QUIZRR
Na 2 O 2 + 2H 2 O  2NaOH + H 2 O 2

2KO2 + 2H2 O  2KOH + H2 O2 + O2


Thus peroxides and superoxides also act as oxidizing agents since they react with H 2 O
forming H 2 O 2 and O 2 respectively.

The hydroxides of all the alkali metals are white crystalline solids. They are strongest of all
base and readily dissolve in water with the evolution of much heat.

(a ) Ba sic St r engt h : The Basic strength of these hydroxides increases as we move down
the group Li to Cs. The hydroxides of alkali metals behave as strong bases due to their
low ionization energies which decrease down the group. The decrease in ionization
energies leads to weakening of the bond between metal and hydroxide ion and M –
O bond in M – O – H can easily break giving M + and OH – . This results in the
increased concentration of hydroxyl ions in the solution i.e., increased basic characters.

(b) Solub ilit y a nd st a b ilit y : All these hydroxides are highly soluble in water and
thermally stable except lithium hydroxide.

2LiOH   Li 2 O  H 2 O

(ii) Nit r a t es (M NO 3 )
The nitrates do not undergo hydrolysis. With the exception of LiNO3 , other nitrates decompose
to nitrites and oxygen.

2MN O 3  2MN O 2  O 2

But, 2LiNO 3  Li 2 O  2NO 2  I 2 O 2

(iii) H a l i d e s
The alkali metals combine directly with halogens under appropriate conditions forming
halides of general formula MX. These halides can also be prepared by the action of aqueous
halogen acides (HX) on metals oxides, hydroxides or carbonate.

M2O + 2HX  2MX + H2 O

MOH + HX  MX + H2 O

M2CO3 + 2HX  2MX + CO2 + H2O (M = Li, Na, K, Rb or Cs) and (X = F, Cl, Br or I)

The fluorides are the most stable while iodides are the least stable.
The trends in meting points, boiling points and solubility of alkali metals halides can be
understood in terms of polarization effects, lattice energy and hydration of ions.

S-BLOCK ELEMENTS
QUIZRR 11

(A) P ola r iza t ion effect s


Comparison of ionic and covalent character of alkali
metal halides.
When a cation approaches and anion, the electron
cloud of the anion is attracted towards the cation
Polarizing
and hence gets distorted. This effect is called
cation Polarized anion Unpolarized anion
polarization. The power of the cation of polarize the
anion is called its polarizing power and the tendency of the anion to get polarized is called
its polarizability. The greater the polarization produced more is the concentration of the
electrons between the two atoms thereby decreasing the ionic character of increasing the
covalent character. The covalent character of any compound in general depends upon the
following factors.
(a ) Size of t he ca t ions
Smaller the cation greater is its polarizing power and hence larger is the covalent
character. The covalent character decreases as size of cation increases.
LiCl > NaCl > KCl > RbCl > CsCl
Thus LiCl is more covalent than KCl.

(b) Size of t he a nion


Larger the anion, greater is its polarizability. This explains the covalent character of
lithium halides is in order
LI > LiBr > LiCl > LiF

(c) C ha r ge of t he ion
Greater the charge on the cation greater is its polarizing power and hence larger is the
covalent character. The covalent character of some halides increase in the order
Na + Cl– < Mg+2 Cl2 < Al+3 Cl3
Similarly greater the charge on the anion, more easily it gets polarized thereby
imparting more covalent character to the compound formed eg covalent character
increase in the order : NaCl < Na 2 SO 4 < Na 3 PO 4
Thus the covalent character decreases as the charge of the anion decrease.

(d) E lect r onic configur a t ion of t he ca t ion


It two cations have the same charge and size, the one with pseudo noble gas configuration
i.e having 18 electrons in the outermost shell has greater polarizing power than a
cation with noble gas configuration i.e having 8 electrons. For example CuCl is more
covalent than NaCl.

S-BLOCK ELEMENTS
12 QUIZRR
(B) L a t t ice E ner gies
Lattice energy is defined as the amount of energy required to separate one mole of solid ionic
compound into its gaseous ions. Evidently greater the lattice energy, higher is the melting
point of the alkali metals halide and lower its solubility in water

(C ) H yd r a t ion E ner gy
It is the amount of energy released when one mole of gaseous ions combine with water to
form hydrated ions.

M+ (g) + aq  M + (aq) + hydration energy

X– (g) + aq X – (aq) + hydration energy

Higher the hydration energy of the ions greater is the solubility of the compound in water.
Further the extent of hydration depends upon the size of the ions. Smaller the size of the
ion, more highly it is hydrated and hence greater is its hydrated ionic radius and less is its
ionic mobility (Conductance).
From above arguments, the melting point and solubility in water or organic solvent of alkali
metal halides can be explained
(a ) A delicate balance between lattice enthalpy and hydration enthalpy determines the
ultimate solubility of a compound in water. For eg. Low solubility of LiF (0.27 g/100 g
H2 O) is due to its high lattice energy (–1005KJmol–1 ) whereas the low solubility of CsI
(44g/100gH 2 O) is due to smaller hydration energy of the two ions (–670 KJ/mol)
(b) The solubility of the most of alkali metal halides except those of fluorides decreases on
descending the group since the decrease in hydration energy is more than the
corresponding decrease in the lattice energy.
(c) Due to small size and high electronegativity, lithium halides except LiF are predominantly
covalent and hence are soluble in covalent solvents such as alcohol, acetone, ethyl
acetate, LiCl is also soluble in pyridine. In contrast NaCl being ionic is insoluble in
organic solvents.
(d) Due to high hydration energy of Li + ion, Lithium halides are soluble in water except
LiF which is sparingly soluble due to its high lattice energy.
(e) For the same alkali metal the melting point decreases in the order :
fluoride > chloride > bromide > iodide
because for the same alkali metal ion, the lattice energies decreases as the size of the
halide ion increases.
(f) for the same halide ion, the melting point of lithium halides are lower than those of the
corresponding sodium halides and thereafter they decrease as we move down the group
for Na to Cs.
The low melting point of LiCl (887 K) as compared to NaCl is probably because LiCl
is covalent in nature and NaCl is ionic.

S-BLOCK ELEMENTS
QUIZRR 13

Salts of oxoacids

Since the alkali metals are highly electropositive, therefore their hydroxides are very strong bases
and hence they form salts with all oxoacids (H2 CO 3 , H3 PO 4 , H2 SO4 , HNO3 , HNO2 etc). They are
generally soluble in water and stable towards heat. The carbonates (M 2 CO 3 ) of alkali metals are
remarkably stable upto 1273 K, above which they first melt and then eventually decompose to
form oxides Li 2 CO 3 , however is considerably less stable and decomposes readily.

Li2CO 3   Li 2O + CO2

This is presumably due to large size difference between Li + and C O 23  which makes the crystal
lattice unstable.
Being strongly basic, alkali metals also form solid bicarbonates. No other metals forms solid
bicarbonates though NH 4 CO3 also exists as a solid. Lithium, however does not form solid bicarbonate
though it does exist is solution. All the bicarbonate on gentle heating undergo decomposition to
form carbonates with the evolution of CO 2 .

2MHCO3   M 2CO 3 + CO2 + H2 O
All the carbonates and bicarbonates are soluble in water and their solubilities increase rapidly on
descending the group. This is due to the reason that lattice energies decrease more rapidly than
their hydration energies on moving down the group.

Anomalous Behaviour of Lithium and its Diagonal Relationship with Magnesium

The properties of lithium are quite from the properties of other alkali metals. On the other hand,
it shows greater resemblance with magnesium, which is diagonally opposite elements of it. The
main reasons for the anomalous behaviour of lithium as compared to other alkali metals are
(i) The extremely small size of lithium atom and its ion.
(ii) Greater polarizing power of lithium ion (Li + ), due to its small size which result in the
covalent character in its compounds.
(iii) Least electropositive character and highest ionization energy as compared to other alkali
meta ls.
(iv) Non availability of vacant d-orbitals in the valance shell.
The reason for resemblance of properties of lithium with magnesium is that these two
elements have almost same polarizing power.
T he following p oint s illust r a t e t he a nom a lous p r oper t ies of lit hium a nd it s d ia gona l
r ela t ionship wit h m a gnesium
(a ) The melting point and boiling point of lithium are comparatively high.
(b) Lithium is much harder than the other alkali metals. Magnesium is also hard metal.
(c) Lithium reacts with oxygen least readily to form normal oxide whereas other alkali metals
form peroxides and superoxides.
S-BLOCK ELEMENTS
14 QUIZRR
(d) LiOH like Mg(OH) 2 is weak base. Hydroxides of other alkali metals are strong bases.
(e) Due to their appreciable covalent nature, the halides and alkyls of lithium and magnesium
are soluble in organic solvents.
(f) Unlike elements of group 1 but like magnesium. Lithium forms nitride with nitrogen.

6Li + N2  2Li3 N

(g) LiCl is deliquescent and crystallizes as a hydrate, LiCl.2H2 O. Other alkali metals do not form
hydrates. MgCl 2 also forms hydrate, MgCl 2 .8H 2 O.
(h) Unlike other alkali metals lithium reacts directly with carbon to form an ionic carbide.
Magnesium also forms a similar carbide.
(i) The carbonates, hydroxides and nitrates of lithium as well as magnesium decompose on
hea ti ng.

Li2CO3  Li2 O + CO2

MgCO3  MgO + CO2

2LiOH  Li2 O + H2 O

Mg(OH) 2  MgO + H2 O

4LiNO3  2Li2 O + 4NO2 + O2

2Mg(NO3 ) 2  2Mg + 4NO2 + O2

The corresponding salts of other alkali metals are stable towards heat.
(j) Lithium nitrate, on heating, decomposes to give lithium oxide, Li 2 O whereas other alkali
metals nitrates decomposes to give the corresponding nitrite.

4LiNO 3  2Li2O + 4NO 2 + O2

2NaNO3  2NaNO2 + O 2

2KNO3  2KNO2 + O2

(k) Li2 CO3 , LiOH, LiF and Li3 PO 4 are the only alkali metal salts which are insoluble in water.
The corresponding magnesium compounds are also insoluble in water.
(l) Hydrogen carbonates of both lithium and magnesium can not be isolated in solid state.
Hydrogen carbonates of other alkali metals can be isolated in solid state.

I llustration :

T he chem ist r y of L it hium is ver y m uch sim ila r t o t ha t of m a gnesium even t hough t hey a r e
p la ced in differ ent gr oup s-Exp la in.
Solution : The ratio of their charge to size is nearly same by which they show the diagonal relationship.

S-BLOCK ELEMENTS
QUIZRR 15

COMPOUNDS OF ALKALI METALS

1. Sod ium :
Some basic reactions are :

Na2CO3+H2O Na2O2

CO2
O2
NaOH H2O NaNH2+H2
Na2O

NH3
Be+NaCl NaH
O2
BeC H2
l2
Cl 2
Al+Na2O Na NaCl
Al 2O 3
S
CO 2 P
SiO 2 Na2S
C+Na2CO3

Si + 2Na2O Na3P

C omp ound s of sodium


1. Sodium oxide, Na 2 O

Na 2 O + H 2 O  2NaOH

2. Sodium peroxide (Na 2 O 2 )


It is a pale yellow powder. It is used as an oxidizing agent

ice cold wat er


Na 2 O 2 + 2H2 O   2NaOH + H2 O 2

25 C
Na 2 O 2 + 2H2 O  2NaOH + O2

Na 2 O 2 + H 2 SO 4  Na 2 SO 4 + H 2 O 2

S-BLOCK ELEMENTS
16 QUIZRR
3. Sodium Hydroxide (Caustic soda), NaOH

NaH2PO3 + PH3
(Sodium (phosphine) Na 2CO3 Na2SO4 NaNO3
hypophosphite)
Na2S2O3+Na2S
P CO2 HNO3
4+
H 4S
2O
H2SO4 NaAlO2
Al2 O3 Sodium meta
I2
NaI+NaIO3 aluminate
(Sodium lOdate)
Co ZnO
(H entr
Br2 nc
ot at
an ed
(Cold and Na2 ZnO2
PbO
NaBr+NaBrO3 d ) dilute) (Sodium zincate)
(Sodium bromate) Cl2
I2 Br2
NaClO3+NaCl Na2PbO2
(Sodium Chlorate) NaI+NaIO NaBr+NaBrO (Sodium Plumbite)
Sodium Sodium
hypoiodite hypobromite

4. Sodium Carbonate
Preparation : (By solvays process)

NH3 + H2O + CO2 NH4HCO3

NaCl

NaHCO3 + NH4Cl

Na2CO3 + CO2

Na 2 CO 3 . 10H 2 O  Washing soda

Na 2 CO 3  Soda ash or light ash

P r op er t ies :
Na 2 CO3 + SiO2  Na 2 SiO3 + CO2 ()
Sodium silicate

S-BLOCK ELEMENTS
QUIZRR 17

Sodium silicate is called soluble glass or water glass as its is soluble in water.

Na 2 CO3 + HCl  NaHCO3 + NaCl

5. Sodium Bicarbonate (Baking soda)


It is used as a medicine to neutralize the acidity in stomach. It is also used in fire extinguishers.

2NaHCO3   Na 2 CO3 + H2 O + CO2

6. Sodium thiosulphate, Na 2 S 2 O 3 . 5H 2 O
It is also know as Hypo.

Na3[Ag(S2O3)2] + NaBr
Na2SO4+Na2S5
Ag2S + H2SO4
AgBr
H2O
Ag2S2O3 AgNO3
Na2S2O3 NaCl + SO2+S+H2O
+ 2NaNO3
H2SO4
I2
FeCl3
Na2SO4 + SO2 + S + H2O
Na2S4O6 + FeCl2 + NaCl
NaI + Na2S4O6
sodium tetrathionate

Action on copper sulphate : Cuprous thio sulphate is formed which dissolves in excess of sodium
thio sulphate to form a complex.

CuSO 4 + Na 2 S 2 O 3  CuS 2 O 3 + Na 2 SO 4

2CuS 2 O 3 + Na 2 S 2 O 3  Cu2 S 2 O 3 + Na 2 S 2 O 6

3CuS 2 O 3 + 2Na 2 S 2 O 3  Na 4 [Cu6 (S 2 O 3 ) 5 ]


Sodium cuprothiosulphate

Some more compounds of sodium


1. Glauber’s salt , Na 2 SO 4 . 10H 2 O
2. Cryolite, Na 3 AlF 6
3. Soda Felspar, NaAlSi 3 O 8

2. Potassium

S-BLOCK ELEMENTS
18 QUIZRR
1) Potassium superoxide

K + O2

H2SO4 S, C,
K2SO4 + H2O2 + O2 KO2 K2SO4 K2S + CO

H2O
CO/CO2
H2O2 + O2 + KOH

K2CO3+O2

2) Potassium carbonate, K 2 CO 3
It is also called potash or pearl ash.

K2 CO3 + 2H2 O  2KOH + H2 CO 3

3) Potassium sulphate

K2 SO4 + Al2 (SO4 ) 3 + H2 O  K2 SO 4 . Al2 (SO4 ) 3. 24H2 O


(potash alum)


K2SO4 + 4C   K2 S + 4CO ()

4) Potassium iodide, KI

AgNO 3 + KI  AgI + KNO3


(yellow)

Pb(CH3 COO) 2 + 2KI  PbI 2 + 2CH 2 COO


(yellow)

HgCl2 + 2KI  HgI2 + 2KCl


(red ppt.)

HgI2 + 2KI  K 2 HgI 4


(excess) (nessler’s regent)

S-BLOCK ELEMENTS
QUIZRR 19

ALKALINE EARTH METALS


Physical Properties

(a ) Physica l st a t e : They are all silvery white metals. They have greyish white lustre when
freshly cut, but tarnish soon after their exposure in air due to surface oxidation. They are
soft in nature but harder than alkali metals because metallic bonding is stronger than IA
elements due to possession of 2 valency electrons. However, hardness decreases with increase
in atomic number. These elements are malleable and ductile but less than alkali metals.

(b) At om ic a nd ionic r a d ii : The size of the atom increases gradually from Be to Ra, on
account of the presence of an extra energy shell at each step. The atoms are large but
smaller than corresponding IA elements since the extra charge on the nucleus attracts the
electron cloud inwards. Their ions are also large and size of the ion increases from Be2+ to
Ra 2 + .
Be Mg Ca Sr Ba Ra

Atomic radii A 1.12 1.60 1.97 2.15 2.22 —

Ionic radii (M2+ ) A 0.31 0.65 0.99 1.13 1.35 1.40

Atomic volume also increases as the atomic number increases.


Be Mg Ca Sr Ba Ra
At. vol. (ml) 4.90 13.97 25.9 35.54 36.7 38.0

(c) Densit y : These metals are denser than alkali metals in the same period because these can
be packed more tightly due to their gr ea t er nuclea r cha r ge a nd sm a ller size. The
density decreases slightly upto calcium and then increases considerably upto radium. Irregular
trend is due to the difference in the crystal structure of these elements.

Be Mg Ca Sr Ba Ra
Density (g ml –1 ) 1.84 1.74 1.55 2.63 3.62 5.5

(d) M elt ing a nd boiling point s : The melting and boiling points of these elements are higher
than corresponding alkali metals. This is due to the presence of two electrons in the valency
shell and thus, strongly bonded in the solid state. However, melting and boiling points do
not show any regular trend because atoms adopt different crystal structures.
Be Mg Ca Sr Ba Ra
m. pt. (°C) 1277 65 0 83 8 76 3 71 4 70 0
b. pt. (°C) 2770 1107 1440 1380 1640 1140

(e) I onisa t ion ener gies a nd elect r op osit ive cha r a ct er : The first and second ionisation
energies of these metals decrease from Be to Ba. The second ionisation energy in each case
is higher than the first, nearly double the first ionisation energy.

S-BLOCK ELEMENTS
20 QUIZRR
Be Mg Ca Sr Ba Ra
First I.E. (eV) 9.32 7.64 6.11 5.70 5.2 5.3
Second I.E. (eV) 18.21 15.03 11.87 11.0 10.0 10.1

The ionisation energy of last member, radium, is slightly higher than that of barium and
it is barium and it is difficult to explain this anomalous behaviour.
Although, the ionisation energies of these elements are higher than those of alkali metals,
yet these are sufficiently low to make these atoms to lose two electrons of their valency shell
to form M2+ ions and achieve the inert gas configuration. These metals are thus, strongly
electropositive in nature but less than corresponding alkali metals. The electropositive character
increases from Be to Ba. Metallic character and reactivity are directly linked with the
tendency to lose electron or electrons, i.e., with electropositive nature. Thus, these characters
increase gradually from Be to Ba.
Be Mg Ca Sr Ba Ra
Electropositive nature increase
Metallic character increases
Reactivity of the metals Increases

(f) Oxid a t ion st a t es : All show a stable oxidation state +2 in their compounds. The second
ionisation energy is nearly double the first ionisation energy for all elements. This should
cause these elements of exhibit a stable +1 oxidation state and form compounds like BaCl,
SrBr, CaI, etc., instead of BaCl 2 , SrBr 2 , CaI 2 , etc. However, the lattice energy increases as
the charge on the ion increases. The increase in the lattice energy on account of the second
electron from ns 2 is much more than the energy required (second ionisation energy) to
remove it. Hence, the stability of +2 oxidation state is due to high lattice energy. The second
factor responsible for +2 oxidation state is the hydration energy which is high for M2+ ions.
On account of the availability of energy, the process does not stop of M+ state but reach to
M2+ state readily.
Since the bivalent ions, M2+ , have an inert gas configuration, it is very difficult to remove
the third electron and hence oxidation state higher than + 2 is not possible.
Amongst alkaline earth metals, beryllium has the highest ionisation energy, i.e., least
electropositive in nature. Thus, beryllium has the minimum tendency to form Be 2+ ion and
hence a number of compounds of beryllium are covalent in nature.

(g) H yd r a t ion of ions a nd hyd r a t ion ener gy : The M2+ ions of alkaline earth metals are
extensively hydrated to form hydrated ions, [M(H 2 O) x] 2+ and during hydration a huge
amount of energy, called hydration energy, is released.

M2+ + xH 2 O  [M(H 2 O) x] 2+ + Energy

T he d egr ee of hyd r a t ion a nd t he a m ount of hyd r a t ion ener gy decr ea ses a s t he


size of t he ion incr ea ses fr om Be 2+ t o Ba 2+ .

S-BLOCK ELEMENTS
QUIZRR 21

Be 2 + Mg 2 + Ca 2 + Sr 2 + Ba 2 +
Heats of hydration 57 0 46 0 39 5 35 5 30 5
(kcal mol –1 )
The hydration energies of alkaline earth metal ions are higher than those of alkali metal
ions and thus the compounds of alkaline earth metals are more extensively hydrated than
alkali metals. Magnesium chloride and calcium chloride exist as MgCl2 .6H2 O and CaCl2 .6H2O,
respectively, while sodium chloride and potassium chloride exist as NaCl and KCl.
The ionic mobilities or ionic conductance of these ions increase from [Be(H 2 O) x] 2+ to
[Ba(H 2 O) x] 2+ because [Be(H 2 O) x] 2+ becomes heavy due to high degree of hydration.

(h ) Elect r onega t ivit y : The tendency to attract electrons is low. The electronegativity values
are thus small and decrease from Be to Ra.
Be Mg Ca Sr Ba Ra
Electronegativity 1.47 1.23 1.04 1.0 0.97 0.97

(i) C onduct ivit y : On account of the presence of two loosely bond valency electrons per atom
which can move freely through the crystal lattice, the alkaline earth metals are good conductors
of heat and electricity.

(j) Flame colour at ion : In the case of Ca, Sr, Ba and Ra, the electrons can be excited by the
supply of energy to higher energy levels. When the excited electrons return to the original
level, the energy is released in the form of light. In beryllium and magnesium, the electrons
are tightly held and hence excitation is rather difficult, thus do not show flame colouration.
Ca, Sr, Ba and Ra impart a characteristic colour to the flame.

Ca—Brick red Sr—crimson Ba—green Ra—crimson

(k) R ed ucing na t ur e : The alkaline earth metals have the tendency to lose electron and
change into bivalent cation :

M  M2+ + 2e

Hence, they act as strong reducing agents. The reducing nature increase as the atomic
number increases.
Strength of a reducing agent is linked with the value of oxidation potential. The values of
the oxidation potentials increase from Be to Ba, hence the strength as reducing agent
increases in the same order.
Be Mg Ca Sr Ba
Oxid. potential (volt) 1.85 2.37 2.87 2.89 2.90
The oxidation potentials are lower than those of the alkali metals, hence, the alkaline earth
metals are weaker reducing agents than alkali metals. The reason for the lower values of
oxidation potentials is due to high heats of atomisation (sublimation) and ionisation energies.

S-BLOCK ELEMENTS
22 QUIZRR
(l) C olour a nd m a gnet ic p r op er t y : Since, the divalent ions have noble gas configuration
with no unpaired electrons, their compounds are diamagnetic and colourless unless the
anion is coloured. The metals are also diamagnetic in nature as all the orbitals are fully filled
with spin paired electrons, e.g.
Mg 2 + = 2, 8
= 1s2, 2s2 2p 6

Chemical Properties of Alkaline Earth Elements

Table summarizes reactions of Group 2(IIA). From this, it is evident that as we go down the
group, reactivity increases.

R ea ct ions of G r oup 2 (II A)

R ea c t ion C omment
M + 2H2 O  M(OH) 2 + H 2 Be probably reacts with stream, Mg with hot water, and
Ca, Sr and Ba react rapidly with cold water.

M + 2HCl  MCl2 + H2 All the metals react with acids liberating H2 . With HNO3 ,
Be becomes passive due to formation of oxide layer.

3M + N2  M3 N2 All form nitrides at high temperatures. Stability Be > Mg


> Ca (hydrolysis to NH 3 )

M + X2  MX2 All the metals form MX2 . No polyhalides are formed.
(X2 = F2, Cl2, Br 2 , I2 )

M + 2NH3  M(NH2 ) 2 + H2 All the metals dissolve in liquid NH 3 to give deep blue
black solutions from which ammoniates [M(NH 3 ) 6 ] 2+ can
be recovered and form amides at high temperatures.

2M + O2  2MO All the metals form normal oxide. Ba also forms peroxide
(BaO 2 ).
Ba + O2  BaO2
M + H2  MH 2 Ionic salt-like hydrides formed at high temperatures by
Ca, Sr and Ba.

Be + 2H2O + 2O H –  Not with other alkaline earth metals.


[Be(OH) 4 ] 2– + H 2 (g)

M(s) 2C(s)  MC2 (s) At high temperature Be forms Be 2 C, ionic compounds

M + S  MS(s) The sulphides are insoluble, but hydrolyse if heated in water.

S-BLOCK ELEMENTS
QUIZRR 23

Some points to note :

1. Be is amphoteric, as it reacts with acid as well as with base

Be + 2NaOH + 2H2 O  Na 2 [Be(OH) 4 ] + H2


Sodium beryllate

Mg, Ca, Sr and Ba do not react with NaOH and thus are purely basic.

2. Except beryllium, all combine with hydrogen directly to form hydrides of the type MH 2 when
heated with hydrogen

M + H2  MH2

BeH 2 and MgH 2 are covalent in nature while other hydrides are ionic in nature.
BeH 2 is formed by reacting beryllium chloride with lithium aluminium hydride

2BeCl2 + LiAlH4  2BeH 2 + LiCl + AlCI3

It is polymeric. (BeH 2 ) n process hydrogen bridges. Three centre bonds are present in which a
banana shaped molecular orbital covers three atoms Be ---- H ------ Be and contains two electrons.
Hydrogen a tons lie in the plane perpendicular to the plane of molecule containing beryllium
a toms

H H
Be Be Be
H H

3. Alkaline earth metals dissolve in liquid ammonia to form coloured solutions.

M  M2+ + 2e–

2NH 3 + 2e–  2NH 2–  H 2

M2+ + 2NH 2–  M(NH2) 2

When the solution is evaporated, hex ammoniate, M(NH 3 )6 is formed. These slowly decompose to
give amides

M(NH3 ) 6  M(NH2 ) 2 + 4NH2 () + H2 ()

4. C ar bides : The carbides are ionic in nature and have NaCl type of structure with M 2+ replacing
Na + and C  C 2– replacing Cl – . Beryllium forms methanide, Be 2 C, with carbon and acetylide,
BeC2 , with acetylene. Magnesium on heating with carbon forms Mg2 C 3 also, which is an allylide
since with water it liberates allylene

S-BLOCK ELEMENTS
24 QUIZRR

CaC2 + 2H2 O  Ca(OH) 2 + C 2 H 2


carbide acetylene

Be2 C + 4H2 O  2 Be(OH)2 + CH4


methane

Mg2 C3 + 4H2 O  2 Mg(OH) 2 + CH3 – C  CH


allylide allylen e

5. C omplex for ma t ion : Generally, the alkaline earth metals do not form complexes. However, the
smaller ions have some tendency to form complexes. Beryllium forms stable complexes such as
[BeF 3 ] – , [BeF 4 ] 2– and [Be(H 2 O) 4 ] 2+

The most important complex formed by magnesium is chlorophyll in which magnesium is bonded
to the four heterocyclic nitrogen atoms. The rest of the elements from calcium to barium form
complexes only with strong complexation agents such as acetylacetone, ethylene diamine teraacetic
acid (EDTA), etc.

Anomalous Behaviour of Beryllium

Be differs form the rest of the group for three reasons.

1. It is extremely small and Fajan’s rule state that small highly charged ions tend to form
covalent compounds.

2. Be has a comparatively high electronegativity. Thus when beryllium reacts with another
atom, the difference in electronegativity is seldom large, which again favours the formation
of covalent compounds. Even BeF2 (electronegativity difference 2.5) and BeO (electronegativity
difference 2.0) show evidence of covalent character.

3. Be is in the second row of the periodic table and the outer shell can hold a maximum of eight
electrons. (The orbitals available for bonding are one 2s and three 2p orbitals. Thus Be can
form a maximum of four conventional electron pair bonds and in many compounds the
maximum coordination number of Be is 4. The later elements can have more than eight
outer electrons and may attain a coordination number of 6 using one ‘s’, three ‘p’ and two
‘d’ orbitals for bonding. Exceptions occur if multi-centre bonding occurs, as for example in
basic beryllium acetate, when higher coordination numbers are obtained.

Thus, we should expect Be to form mainly covalent compounds and commonly have a
coordination number 4. Anhydrous compounds of Be are predominantly two-covalent and
BeX 2 molecules should be linear.

S-BLOCK ELEMENTS
QUIZRR 25

Electronic structure 1s 2s 2p
of beryllium atom in
the ground state
FeăBeăF
(gas)
Electronic structure 1s 2s 2p
of beryllium atom in
excited state

Two unpaired electrons can form


bonds with two X atoms ă linear
molecule (sp hybridization)

In fact linear molecules exist only in the gas phase, as this electronic arrangement has not
filed the outer shell of electrons. In the solid state four fold coordination is always achieved.
There are several ways by which this can be achieved.
1. Two ligands that have a lone pair of electrons may form coordinate bonds using the two
unfilled orbitals in the valence shell of Be. Thus two F – ions might coordinate of BeF 2 ,
forming [BeF 4 ] 2– . Similarly diethyl ether can coordinate to Be(+II) in BeCl 2 , forming
[BeCl 2 (OEt) 2 ].
2. The BeX 2 molecules may polymerise to form chains, containing bridging halogen groups, for
example (BeF 2 ) n , (BeCl 2 ) n . Each halogen forms one normal covalent bond and uses a lone
pair to form a coordinate bond.
3. (BeMe 2 ) n has essentially the same structure as (BeCl 2 ) n , but the bonding in the methyl
compound is best regarded as three-centre two electron bonds covering one Me and two Be
a toms.
4. A covalent lattice may be formed with a zinc blende or wurtzite structure (coordination
number 4), for example by BeO and BeS.
In water beryllium salts are extensively hydrolysed to give a series of hydroxo complexes of
unknown structure. They may be polymeric and of the type:

2ă 2ă
HO OH OH HO OH OH OH

NaOH
Be Be Be Be Be [Be(OH)4]2ă
Tetrahedral
Berrylate ion
HO OH OH HO OH OH OH

If alkali is added to these solutions, the polymers break down to give the simple mononuclear
berrylate ion [Be(OH) 4 ] 2– , which is tetrahedral. Many beryllium salts contain the hydrated
ion [Be(H 2 O) 4 ] 2– , which is tetrahedral. Many beryllium salts contain the hydrated ion
[Be(H 2 O) 4 ] 2+ , rather than Be 2+ and the hydrated ion too is a tetrahedral complex ion. Note

S-BLOCK ELEMENTS
26 QUIZRR
that the coordination number is 4. Forming a hydrated complex increases the effective size
of the beryllium ion, thus spreading the charge over a larger area. Stable ionic salts such
as [Be(H 2 O) 4 ]SO 4 , [Be(H 2 O) 4 ](NO 3 ) 2 and [Be(H 2 O) 4 ]Cl 2 , are known.

Electronic structure of 1s 2s 2p
beryllium atom in the
ground state

Electronic structure of 1s 2s 2p
Be2+ ion

Electronic structure of 1s 2s 2p
Be2+ ion having gained
four electron pairs from
four oxygen atoms in
water molecules Four electron pairs tetrahedral
(sp3 hybridization)

Beryllium salts are acidic when dissolved in pure water because the hydrated ion hydrolyses
producing H3 O + . This happens because the Be–O bond is very strong and so in the hydrated
ion this weakens the O–H bonds and hence there is a tendency to lose protons. The initial
reaction is

H 2 O  [Be(H 2 O)4 ]2   [Be(H 2 O)3 (OH )]  H 3 O 

but this may be followed by further polymerization, involving hydroxobridged structures. In


alkaline solutions [Be(OH)4 ] 2– is formed. The outer group II salts do not interact so strongly
with water and do not hydrolyse appreciably.
Beryllium salts rarely have more than four molecules of water of crystallization associated
with the metal ion, because there are only four orbitals available in the second shell of
electrons, whereas magnesium can have a coordination number of 6 by using some 3d
orbitals as well as 3s and 3p orbitals.

COMPOUNDS OF ALKALI EARTH METALS

1. O xid es a nd H ydr oxid es


P r ep a r a t ion :

1. 2 Be + O2  2 BeO

2Ca + O2  2CaO


Ca CO3   CaO + CO2

Mg CO3   MgO + CO2

S-BLOCK ELEMENTS
QUIZRR 27

2. MgO + H2 O  Mg (OH) 2

CaO + H2 O  Ca (OH) 2

Note : Basic strength of hydroxides increase going down the group.


Be (OH) 2 < Mg (OH) 2 < Ca (OH) 2 < Sr (OH) 2 < Ba (OH) 2

Properties :

(1) BeO is insoluble in H 2 O but dissolves in acid and alkali

BeO + 2HCl  BeCl2 + H2 O

BeO + NaOH  Na 2 Be O2 + H2O


Sodium Beryllate

CaO combines with solid acidic oxides at high temperature

CaO + SiO2  CaSi O3

6 CaO + P4 O10  2 Ca 3 (PO4) 2

(2) Ca(OH) 2 is called lime wa t er or slaked lime and Ba(OH) 2 is called ba r yt a wa t er , Lime
water or baryta turns milky by CO 2
Ca(OH) 2 + CO2  CaCO3 + H 2 O
milky

Ba(OH) 2 + CO 2  BaCO 3 + H 2 O


milky

Milkyness disappears when excess of CO2 is passed, due to formation of soluble bicarbonates
CaCO3 + H2O + CO2  Ca(HCO3) 2
excess solu ble

2 Ca(OH) 2 + 2Cl2  CaCl2 + Ca(ClO) 2 + 2H2 O


Calcium
hypochlorite

Bicarbonates and Carbonates

• The carbonates are all ionic, but due to excessive hydration of Be 2+ , BeCO 3 is unusual
because it contains the hydrated ion [Be(H 2 O) 4 ] 2+
• BeCO 3 decomposes at low temperature and is placed in the atmosphere of CO 2

BeCO3  BeO + CO2

S-BLOCK ELEMENTS
28 QUIZRR
Thermal stability increases with increase in atomic weight down the group or with increasing
cationic size

BeCO3 < MgCO3 < CaCO3 < SrCO3 < BaCO3

• Ca(HCO3 )2 and Mg(HCO3 ) 2 exist in water causing temporary hardness which can be removed
as insoluble carbonates by boiling


Ca(HCO3 ) 2   CaCO3 () + H2 O + CO2


Mg(HCO3 )2   MgCO3 () + H2 O + CO2

• Ca(HCO 3 ) 2 and Mg(HCO 3 ) 2 are amphoteric thus neutralized by acid as well as base :

Ca(HCO3 ) 2 + H2 SO 4  CaSO4 + 2H2 O + 2W 2

Ca(HCO3 ) 2 + Ca(OH) 2  CaCO3 () + 2H2 O

S ulphates

• The solubility of the sulphates in water decreases down the group.

BeSO 4  MgSO 4 > Ca SO 4 > SrSO 4  BeSO 4


  
    
So lu b le Spa r ingly in so lub le
so lub le

High solubilities of BeSO 4 and MgSO 4 are due to the high enthalpy of hydration of the
smaller Be 2+ and Mg2+ ions, which over comes the lattice energy factors.

• The sulphates decompose into oxides and SO 3



MSO4   MO + SO3
• MgSO 4 . 7H 2 O is called Epsom Salt Although correct formulation is
[Mg (H 2 O) 6 ] SO 4 .H 2 O
150  C
MgSO 4 . 7H 2 O  MgSO 4 . H 2 O

200°C

1
O + SO2 + MgO 
st r on g
h ea t in g
 MgSO4
2 2

Not e : The reason that it first changes to monohydrate lies in the above give formulation
[Mg(H 2 O) 6 ] SO 4 .H 2 O

S-BLOCK ELEMENTS
QUIZRR 29

• CaSO 4 . 2 H 2 O is called gypsum

1
150  C 200  C
CaSO4 . 2 H2O  CaSO4 . H O
2 2  CaSO4

gypsu m plaster of anhydrite


paris

1100°C

CaO + SO3
CaSO4 + 4C  CaS + 4CO

S-BLOCK ELEMENTS
HYDROGEN
2 QUIZRR
H YDROGEN

Introduction

Hydrogen is the first element in the periodic table and is also the lightest element known. It
atomic form exists only at high temperatures. In the normal element form, it exists as a diatomic
molecule, i.e. H2.

Unique Position of Hydrogen in the Periodic Table

A proper position could not be assigned to hydrogen either in the MendeleevÊs periodic table or
Modern periodic table because of the following reason :
In some properties, it resembles alkali metals and in some propperties it resembles halogens. So
hydron can be placed both in group 1 and group 17 with alkali metals and halogen respectively.

Resemblance with Alkali Metals

1. Electronic configuration
Hydrogen contains one electron in the valence shell like alkali metals
H : 1s1
Li : [He]2s1
Na : [Ne]3s1
K : [Ar]4s1
Rb : [Kr]5s1
2. Electropositive character
Like alkali metal, hydrlogen also loses its only electron to form hydrogen ion, i.e., H+

H  H0  e –
1s 1 1s


Na  Na  e–
1s 2 2s 2 2p 6 3s1 2 2 6
1s 2s 2 p

3. Oxidation state
Like alkalil metals, hydrogen exhibits an oxidation state of +1 in its compounds.
H+ Clă Na+ Clă K+Br ă
Hydrogen chloride Sodium chloride Potassium chloride

4. Reducing agent
Alkali metals act as reducing agents because of their tendency to lose valence electron. Hydrogen
is also a very good reducing agent as evident from the following reactions :

HYDROGEN
QUIZRR 3


Fe3O4 + 4H2   3Fe + 4H2O

CuO + H2   Cu + H2O

5. Combination with electronegative elements


Just llike alkali metals hydrogen combines with electronegative elements such as halogen, oxygen,
sulphur, etc to form compounds with similar formulae
Halides Oxides Sulphides
HCl H 2O H2 S
NaCl Na2O Na2S

DIFFERENCE FROM ALKALI METALS


1. Ionization enthalpy
Ionization enthalpy of hydrogen (1312 kJ mol-1) is very high in comparison with the ionization
enthalpy of alkali metals.

2. Existence of H+
It has been established that H+ ion does not exist freely in aqueous solution. This is because of
the fact that has a very small sixe (  1.5 ï 10-3 pm) as compared to normal atomic and ionic size
(which range from 50 to 220 pm). Thus it exists in aqueous solution in the form of hydrated roton
with a formula, H9O+4. However, for the sake of simplicity hydrated proton is represented by
hydronium ion, H3O+.
On the other hand, the alkali metal ions mostly exist as hexahydrated ions.

3. Difference in halides
Hydrogen halides are different from the halides of alkali metals although they have similar
molecular formulae. For example
(i) Pur HCl is a covalent compound while NaCl is an ionic compound.
(ii) HCl is a gaseous compound while NaCl is a solid at ordinary temperature.

Resemblance with Halogens

1. Electronic configuration
Just like halogens, hydrogen needs one electron to attain the configuration of nearest noble gas.
H : 1s1
F : 1s22s22p5
Cl : 1s22s22p63s23p5

HYDROGEN
4 QUIZRR
2. Atomicity
Like halogens, hydrogen also exists in a diatomic state. The atomicty of hydrogen as well as
halogens is two.

3. Electrochemical nature
During electorlysis of LiH, CaH2, etc, in molten state hydrogen is evolved at the anode indicating
its electronegative nature. In this respect, hydrogen shows resemblance with halogens which are
also liberated at the anode during electrolysis.

4. Oxidation state
Just like halogens, hydrogen also exhibit state of -1 in some of its compounds such as metal
hydrides.
Na+1 Hă1 Na+Cl -

5. Combination with alkali metals


Just like halogen, hydrogen also combines with alkali metals to form salts with similar formulae.
NaH LiH CaH2
NaCl LiCl CaCl2

6. Combination with non-metals


Just like halogens hydrogen also react with non-metals such as carbon, silicon, germanium, etc,
to form covalent compounds.
Halides : CCl4 SiCl4 GeCl4
Hydrides : CH 4 SiH4 GeH 4

7. Ionization energy
Ionization energy of hydrogen is comparable to the ionization energies of halogens as shwon
below :
Element : H F Cl Br
Ionization energies (kJ mol-1): 1312 1681 1255 1121

Difference from Halogens

1. Less tendency of hydrideăformation


Although hydrogen forms hydride ion (H-) like halogens, yet its tendency to form hydride ion is
very less in comparison with the halogens. It is quite clear from the fact that halogens form
halides with very large number of metals but hydrogen form hydrides with only a small number
metals like sodium and calcium, etc.

HYDROGEN
QUIZRR 5

2. Absence of unshared electrons


There is no unshared pair of electron in hydrogen molecule (H2) whereas halogen molecules have
six unshared electron pairs as shwon below :

H·H Cl — Cl F—F

3. Nature of oxides
The oxides of halogen are acidic in nature whereas oxide of hydrogen is neutral
H2O Cl2O7,I2O5
Neutral oxide Acidic oxides

4. Nature of compounds
The compounds of hydrogen with halogens, i.e. hydrogen halides (HF, HCl, HBr, HI) are low
boiling covalent compounds whereas alkali metal halides (LiF, Nacl, KBr, CsI) are high melting
ionic solids.

I sotopes of Hydrogen

It has been found by mass spectrograph that hydrogen has three isotopes namely; protium,
deuterium and tritium. The relative abudance of three isotopes of hydrogen is as under.

1 2 3
Isotopes H : H : H
1 1 1

Abudance 1 : 1.5ï 10-2 : 1ï 10ă17

(a) Protium or hydrogen


It is represented by the symbol H. Its atomic number is 1 and mass number is also 1. It has
one proton (but no neutron) in its nucleas and one electron in its 1s orbital. Naturally
occuring hydrgen contains 99.985% of this isotopes.
(b) Deuterium or heavy hydrogen

It is represented by the symbol D or 12 H . ItÊs atomic number is 1 and mass number is 2. It


has proton and one neutron in its necleus and one electon in its 1s orbital. Naturally
occuring hydrogen has 0.15% of this isotopes mostly in the form of HD.
(c) Tritium

It is represented by the sympbol of T or 3


1H . Its nucleus hs one proton and 2 neutron and

there is one electron in its 1s orbital. It is an extremely rare isotope. Out of 1017 molecules
of ordinary hydrogen there is just one molecule of tritium. this isotope of hydrogen is
radioactive in nature and emits low energy  - particles (tó= 12.33  ).

HYDROGEN
6 QUIZRR

PROTIUM (11H) DUTERIUM (21H) TRITIUM (31H)


(Non ă Radioactive) (Non ă Radioactive) (Radioactive)
99.985% 0.015% 10ă15%

It may be noted that three isotopes of hydrogen have properties because of the same
electronic configuaration, 1s1. However due to different mass numbers they have different
rates of chemical reactions. For example, reaction between protium and chlorine is 13.4 times
faster than that between deuterium and chlorine. Similary electrolysis of ordinary water
(H2O) occurs more rapidly than of heavy water (D2O). Difference in properties arising due
to the difference in mass number is referred to as isotopic effect.

(d) Ortho and para hydrogen


When the spins of the nuclei are in the same direction (parallel spins), dihydrogen is called
ortho hydrogen and when the spins are in the opposite direction (anti parallel spins),
dihydrogen is called para hydrogen.

1 1
Total spin =   1 (ortho H 2 )
2 2

1 1
Total spin =   0 (para H 2 )
2 2

HYDRIDES
Binary compounds of the elements (metals or non-metals) with the hydrogen are called hydrides.
The type of hydride which an element forms depends on its electro negativity and hence the type
of bond formed. While there is no sharp divison between ionic, covalent and metallic bonding,
there are classified as metallic bonding, there are classified as :
(a) Ionic or salt like hydride
(b) Covalent or molecular hydride
(c) metallic or interstitial hydride

HYDROGEN
QUIZRR 7

I onic Hydrides

When hydrogen directly combines with any alkai metal or with alkaline earth metals Ca, Sr or
Ba
2Li(s) + H2(g)  2LiH(s)
Ca(s) + H2(g)  CaH2(s)

Covalent Hydrides

These hydrides are formed by all the true non-metals (except zero group elements) and the
elements like Al, Ga, Sn, Pb, Sb , Bi, etc., which are normally metallic in nature i.e. this class
includes the hydrides of p-block elements.
Molecular hydrides are further classified according to their relative numbers of electrons and
bonds in their lewis structures
(i) Electron dificient molecular hydrides :
Diborane (B2H6) is an example.
(ii) Electron-precise molecular hydrides :
These are formed by elements of group 14. The molecules are tetrahedral. Methane (CH4)
is an example.
(iii) Electron-rich molecular hydrides :
NH3, H2O, HF etc. are examples.

Metallic Hydrides

These hydrides are often non-stoichiometric and their composition vary with temperature and
pressure. Formula of some of the hydrides of this class are :
TiH1.73, CeH2.7, LaH2.8

Hydrogen Peroxide

Although the preparation of hydrogen peroxide given in NCERT book is sufficient for JEE, but
here we are covering only one preparation.

OH O
C2H5 O2 C2H5

H2 /Pd + H2 O2

OH O
(2 ethyl anthraquinol) (2 ethyl anthraquinol)

HYDROGEN
8 QUIZRR
Ć H2O2 is unstable and hence it disproportionates depending on the temperature and
concentration.
2H2O2 2H2O + O2

reduction
oxidation

OH
C2H5

OH

Some more points :

1. H2O2 as a bleaching agent :


Due to its oxidizing nature, it acts as a bleaching agent.

Coloured material + O  Colourless.


It bleaches meterials like silk, hair, ivory, cotton, wool etc.

HYDROGEN
QUIZRR 9

2. Oxidizing nature

(a) FeSO4
+ H2O2

(b) A solution of chromic acid in suplhuric acid or acidified potassium dictiromate is oxidized to
blue peroxide of chromium (CrO5) which is unstable, howver, it is soluble in ether and
produces blue coloured solution.

K 2Cr2O7 + H2SO4 + 4H2O2  2CrO5 + K2SO4 + 5H2O


(Blue)

Cr

(Butterfly Shape)

3. Structure of H2O2

H
94Ĉ 98'
1.48AĈ

111Ĉ 30'

H2O2 is used :
(a) as a bleaching agent
(b) as an antiseptic in surgery
(c) as a fuel for rockets

HYDROGEN
10 QUIZRR

MISCELLANEOUS EXAMPLES
Example 1


(a) Give struce O2ă, O22ă and O 2 .

(b) What is their behaviour towards H2O ?


(c) Give applications of H2O2 and KO2.
Solution
2ă 2ă
(a) O O O O O

(b) O2ă + H2O  2OH-


oxide

2-
2O 2
+ 2H2  O2 + 4OH-
peroxide

-
40 2 + 2H2O  3OH2 + 4OH-
superoxide
All the three ions are unstable towards H2O and give alkaline solution. The decomposition

of O2ă
2
and Oă2 involve oxidation and reduction. Both of these ions, especially Oă2 , are good
oxidising agents.
(c) H2O2 is used as :
ă bleacing agent
ă a substitute for Cl2 in water and sewage treatment
ă an antiseptic in curing wounds
KO2 is used in life supports in spacecrafts, submarines and emergency breathing apparatus
since it absorbs CO2 and releases O2.

4KO2 + 2CO2  2K2 CO3 + 3O2

Example 2

NaCl in earlier days used to manufacture NaOH and Cl2 involving following steps. Identify
(A) to (I) in the following :

NaCl + comc H2SO4   (A) + (B) (gas)

(B) gas 


MnO2 (C) gas


(A) + NaCl   (D) + (B) gas

HYDROGEN
QUIZRR 11

(D) + carbon + CaCO3  (E) + (F)


CaCO3   (G) + (H) gas

(G) + H2O  (I)

(I) + (E)  NaOH + CaCO3

Solution :

NaCl + conc. H2SO4  NaHSO4 + HCl


(A) (B)

HCl 
MnO2 Cl2
(B)

MnO2 + 4HCl   MnCl2 + Cl2 + 2H2O
(B) (C)

NaHSO4 + NaCl   Na2SO4 + HCI
(A) (D)

Na2SO4 + carbon + CaCO3   Na2CO3 + CaSO4
(D) (E) (F)

CaCO3   CaO + CO2
(G) (H)

CaO + H2O  Ca(OH)2


(G) (I)

Ca(OH)2 + Na2CO3  2NaOH + CaCO3


(I) (E)

Example 3

GIVE REASONS
1. Na2SO4 is soluble in water where as BaSO4 is insoluble. Why ?
Sol: The lattice energy of Na2SO4 is less than the hydration energy where as the lattice energy of the
BaSO4 (because of bivalent charge) is very high so that hydration energy released is not sufficient
to break the lattice and BaSO4 remains insoluble.
2. Why potassium carbonate can not be prepared by solvay process ?
Sol: This is due to the reason that potassium bicarbonate (KHCO3) formed asan intermediate (when
CO2 is passed through ammoniated solution of potassium chloride) is highly soluble in water and
can not be seperated by filteration.
HYDROGEN
12 QUIZRR
3. When Mg metal is burnt in air; a white powder is left behind a ash. What is the white powder ?
Sol : Mg on burning in air reacts with oxygen and nitrogen resulting in the formation of MgO and
magnesium nitride.

2Mg + O2  MgO

3Mg + N2  Mg3N2

4. In aqueous solution Li ion has the lowest mobility. Why ?


Sol : Li ions are highly hydrated in aqueous solution which result in decrease in its mobility.
5. Explain, why lithium is kept rapped in paraffin wax and not stored in kerosene oil ?
Sol: It is because lithium is a light metal and therefore it floats at the surface of kerosene oil. To
prevent its exposure to air it is kept wrapped in paraffin wax.
6. Why caesium can be used in photoelectric cell, while lithium can not be ?
Sol: Caesium has the lowest while lithium has the highest ionization energy among all the alkali
metals. Hence, caesium can lose electrone very easily while lithium cannot.
7. Which alkali metal ion has the maximum polarizing power and why ?
Sol: Li ion has the maximum polarizing power among all ther alkali metal ions. Thus is due to small
size of Li ion as result of which it has maximum charge/radius ratio.
8. Explain why
(i) Lithium on being heated in air mainly forms the monzide and not peroxide.
(ii) An aqueous solution of sodium carbonate gives alkaline tests.
Sol : (i) Li ion is smaller in size. It is stabilized more by smaller anion, oxide ion (O-2) as compared
to peroxide (O2-2).
(ii) An aqueous solution of sodium carbonate gives alkaline tests because Na2CO3 undergoes
hydrolysis forming sodium hydroxide.

Na2CO3 + H2O  NaHCO3 + NaOH

Example 4

Lithium compounds have covalent character. Explain ?


Solution :
High polarising power of Li+ ion due to its smaler size makes lithium Li
compounds (particularly lithium halides) predominantly covalent. If Li
1 is considered fully ionic compound, its dipole moment in the saseous
state would be 11.5 debye (D) but its experimental value comes out to
be 3.25 D. This is due to distortion of electron charge cloud of the Polarisation of
iodide ion by the lithium ion, which results in decrease in ionic character electron charge cloud
(hence, the decrease in dipole moment) and increase in covalent of iodide by lithium ion
character of Lil.

HYDROGEN
QUIZRR 13

Example 5

(i) An inorganic compound (A) is formed on passing a gas (B) through a concertrated liquor containing
sodium sulphide and sodium sulphite.
(ii) On adding (A) into dilute solution of silver nitrate, a white precipitate appears which quickly
changes into a black coloured compound (C).
(iii) On adding two or three drops of ferric chloride into the excess of solution of (A), a violet coloured
compound (D) is formed. This colour disappers quickly.
(iv) On adding a solution of (A) into the solution of cupric chloride, a white precipitate is first formed
which dissolves on adding excess of (A)forming a compound (E).
Identify (A) to (E) and give the chemical equations for the reactions at steps (i) to (iv).
Solution :
The reactions indicate that the compound (A) is sodium thiosulphate. It is formed in step (i) by
passing gas (B) which is either I2 or SO2.

(i) Na2S + Na2SO3 + 3SO3+ I2  Na2S2O3 + 2NaI


(B) (A)
Or

2Na2S + Na2SO3 + 3SO2  3Na2S2O3

(ii) 2AgNO3 + Na2S2O3  Ag2S2O3 + 2NaNO3


(White ppt.)

Ag2S2O3 + H2O  Ag2S + H2SO4


(C) Black

(iii) 2FeCl3 + 3Na2S2O3  Fe2(S2O3)3 + 6NaCl


(D) Violet

Fe2(S2O3)3 + Na2S2O3  2Fe(S2O3) + 6NaCl


Colourless

(iv) CuCl2 + Na2S2O3  CuS2O3 + 2NaCl

2CuS2O3 + Na2S2O3  CuS2O3 + Na2S4O6


(White ppt.)

3Cu2S2O3 + 2Na2S2O3  Na4[Cu6(S2O3)5]


(E) Soluble complex

HYDROGEN
14 QUIZRR
Example 6

A white solid is either Na2O or Na2O2. A piece of red litmus paper turns white when it is
dipped into a freshly made aqueous solution of the white solid.
(i) Identify the substance and explain with balanced equation.
(ii) Explain what would happen to the red liltmus if the white solid were the other
compound. [I.I.T. 199]
Solution :
(i) A piece of red litmus truns white when dipped into aqueous solution of white solid indicates
that the solution has bleaching action on litmus. This is due to the presence of hydrogen
peroxide in solution which is formed by action of water on sodium peroxide. Thus, the white
solidis Na2O2.

Na2O2+ 2H2O  2NaOH + H2O2

(ii) The other compound is Na2O which gives NaOH (sodium hydroxide) with water.

Na2O + H2O  2NaOH

The solution, thus, turns red litmus to blue.

Example 7

The Solvaly process can be represented by the following scheme :

Identify A, B, C, D and E. [IIT 1999]


Solution :
(A) : Ca(OH) 2

CaO + H2O  Ca(OH)2


(A)

HYDROGEN
QUIZRR 15

(B) : NH4HCO3

NH3 + H2O + CO2  NH4HCO3


(B)
(C) : Na2CO3

2NaHCO3  Na2HCO3 + H2O + CO2


(C)
(D) : NH4Cl

NH4HCO3 + NaCl  NaHCO3 + NH4Cl


(D)
(E) : CaCl2

2NH4Cl + Ca(OH)2  2NH3 + 2H2O + CaCl2


(E)

Example 8
Aqueous solution of an inorganic compound (x) shows the following reactions :
(i) It decolourises an acidified KMnO4 solution accompanied by the evolution of oxygen.
(ii) It liberates iodine from an acidifed KI solution.
(iii) It gives a brown precipitate with alkaline KMnO4 solution with evolution of oxygen.
(iv) It removes black stains from old oil paintings.
Identify, (x) and give chemical equations for the ractions at steps (i) to (iv).
[roorkee 1993]
Solution :
In reaction (i) and (iii) (x) acts as a reducing agent while in reaction (ii) it acts an oxidising agent.
Thus, these reactions indicate that (x) is H2O2 which is further confirmed from reaction (iv).

K SO + 2MnSO
(i) 2KMnO4 + 3H2SO4 + 5H2O2  2 4
 4 + 8H O + 5O
2 2
Soluble,colourless
Purple coloured

(ii) 2KI + H2SO4 + H2O2  K2SO4 + I2 + 2H2O

(iii) 2KNnO4 + 3H2O2  2KOH + 2MnO2 + 2H2O + 3O2


Brown

(iv) Pbs  4H2  PbSO4  4H 2O


Black White

HYDROGEN
16 QUIZRR
Example 9
Hydrogen peroxide acts both as an oxidising and as a reducing agent in alkaline solution
towards certain first row transition metal ions. Illustrate both these properties of H2O2
using chemical equations. [I.I.T. 1998]
Solution :
Chromium hydroxide is oxidised by H2O2 in presence of NaOH into sodium chromate

[H2O2  H2O + O] ï 3

2Cr(OH)3 + 4NaOH + 3O  2Na2CrO4 + 5H2O

2Cr(OH)3 + 4NaOH + 3H2O2  2Na2CrO4 + 8H2O

Potassium ferricynide is reduced to ferrocyanide in presence of KOH by H2O2.

2K3Fe(CN)6 + 2KOH  2K4Fe(CN)6 + H2O + O

H2O2  H2O + O2

2K3Fe(CN)6 2KOH + H2O2  2K4Fe(CN)6 + 2H2O + O2

HYDROGEN
OXIDATION
REDUCTION
QUIZRR 3

1 OXIDATION REDUCTION

1 .1 FORM A L CH A RGE
The formal charge of an atom in a molecule is the hypothetical charge the atom would have if we
could redistribute the electrons in the bonds evenly between the atoms in the bond. Formal charge (F)
is determined using

s
F = v u
2
where v = valence electrons
s = shared electrons (forming bonds)
u = unshared electrons

Cl  Br Cl
Ć In BrCl3 |
Cl
v s u Formal charge
Br 7 6 4 0
Cl 7 2 6 0

Ć In ICl4

v s u Formal charge
I 7 8 4 (one extra) ă1
Cl 7 2 6 0

Ć In NH4

v s u Formal charge
N 5 8 0 +1
H 1 2 0 0

Ox i d a t i o n Re d u c t i o n
In many cases, we can draw more than one structure for a molecule, with different arrangement
of atoms. We can use the following guidelines involving formal charge to decide between two or more
possible structures for a molecule or ion.
(a) A molecular structure that gives a Lewis structure in which all formal charges are zero is
preferable to one in which some formal charges are not zero.

OXIDATION REDUCTION
4 QUIZRR

(b) If the Lewis structure must have non-zero formal charges, the structure with the lowest
number of non zero formal charge is preferable and a Lewis structure with no more than one
large formal charge (ă 2, + 2 and so on) is preferable to a structure with several formal
charges.
(c) Lewis structures should have adjacent formal charges of zero of opposite sign.
(d) When we must choose among several Lewis structure with similar distribution of formal
charges, the structure with the negative formal charges on the more electronegative atom is
preferable.
Thiocyanate ion can be assign as three structures.
Formal Charges
N C S Total Charge

 .. ..  
I : N  C  S:  ă1 0 0 ă1
 

 .. ..  
II : C  N  S:  +1 ă2 0 ă1
 

 .. ..  
III : C  S  N:  ă1 ă2 +2 ă1
 

In all these cases total formal charge is ă 1. But we prefer structure I because it has the lowest
number of atoms with non zero formal charges (guideline (b)).

1 .2 T H E OX I DA T I ON ST A T E OF A N EL EM EN T
To describe the changes that occur in oxidationăreduction reactions and to write the correctly
balanced equations for such reactions, it is helpful to know the concept of the oxidation state (or
oxidation number) of an atom).
For monoatomic ions, the oxidation state is simply the charge on the ion. For example, oxidation
states of Na+, Mg2+, Clă and N3ă are + 1, + 2, ă 1 and ă 3 respectively. For covalently bonded substances,
the charge on an atom would be so small that it is impossible to calculate the exact charge on each atom
in the molecule. Thus, oxidation state for covalently bonded molecule is defined as „the charge an atom
would possess if all the bonds associated with that atom are broken heterolytically considering them to
be completely ionic‰. For example, in NH3 there are three NăH bonds. If we consider all three NăH
bonds to be ionic, then each H will possess a charge of + 1 while N has the charge ă 3 (because N is
more electronegative than H). In H2O2, there are two OăH bonds and one OăO bond as evident from
the structural formula HăOăOăH. Assuming each OăH bond to be ionic, each H atom would possess
+ 1 charge while each O possess ă 1 charge. The OăO bond is not ionic as the bond is between atoms
of similar electronegativity.
It is not necessary to know the structure of a molecule in order to calculate the oxidation state of
an atom in a molecule. The following set of rules are used to assign oxidation states to atoms in ions
or molecules.

OXIDATION REDUCTION
QUIZRR 5

1. Each pure element has an oxidation state of zero. This is true whether the element is a
monoatomic gas, a metallic solid or a polyatomic molecule. Thus, Fe(s), N2(g), P4(s) and S8(s)
are all in the zero oxidation state.
2. In monoatomic ions, the oxidation state of the element is equal to the charge of the ion. Thus,
oxidation state of all alkali metals is + 1 and that of alkaline earth metals is + 2 in all their
compounds. In ionic solid K2S, the oxidation stateÊs of potassium and sulfur are + 1 and ă 2
respectively. In AgCl, the oxidation state of silver and chlorine are + 1 and ă 1 respectively.
3. The oxidation state of hydrogen in any molecule in which it is combined with another element
is + 1, except in the metallic hydrides such as LiH or CaH2, where the oxidation state of
hydrogen is ă 1.
4. The oxidation state of oxygen in any molecule or ion in which it is combined with another
element is ă 2, except in the peroxides, the superoxides and in OF2. The peroxides (H2O2,
Na2O2, BaO2 etc.) are compounds in which there is an OăO covalent bond and the peroxide ion
is O22 . The oxidation state of oxygen in peroxides is ă 1. The superoxides (like KO2, CsO2 etc.)

are ionic compounds involving the superoxide ion, O2 . The oxidation state of oxygen is ă óin
superoxide in OF2, the oxidation state of oxygen is + 2 since fluorine is more electronegative
than oxygen.
5. The oxidation state of all the halogens is ă 1 in all their compounds except where they are
combined with an element of higher electronegativity or in interhalogn compounds. Oxidation
state of fluorine is always ă 1, since it is the most electronegative element.
6. In covalent compounds not involving hydrogen or oxygen, the more electronegative element is
assigned negative oxidation state while less electronegative element is assigned positive oxidation
state.
7. The algebraic sum of the oxidation numbers of all the atoms combined in a molecular or
complex ion must equal the net charge on the molecule or ion.
Until the form of radical changes, the O.N of an element in a compound remains same.
For Example
(1) O.N. of nitrogen in HNO3, NaNO3, Ca(NO3)2 is same, i.e. + 5.
(2) O.N. of sulphur in H2S, CaS, Na2S & Al2S3 is equal.
(3) Oxidation number of a metal in carbonyl complex is always zero.
e.g. Fe in Fe (CO)5
Ni in Ni (CO)4
Here the O.N. of CO is taken to be zero.
(4) Oxidation number can be zero, + ve, ă ve (integer or fraction).
e.g. O.N. of O in KO2 is ă ó
.
(5) Maximum O.N. of an element is = Group No.
(Except O & F)
(6) Minimum O.N. of an element = Group No. ă 8
(Except metals)

OXIDATION REDUCTION
6 QUIZRR

Ex a m p l e 1
Calculate the oxidation state of the underlined atoms in the given species.
(a) NO 2 (b) NO3

(c) KMnO4 (d) Cr 2 O72 


Solution :
(a) Let the oxidation state of N in NO2+ be x.
x + [2(ă 2)] = + 1
x =+ 5
Thus, oxidation state of N in NO2 is + 5.

(b) Let x be the oxidation state of N in NO3 .

x + [3(ă 2)] = ă 1
x =+ 5

Thus, oxidation state of N in NO3 is + 5.

(c) Let the oxidation state of KMnO4 be x.


[1 (+ 1)] + x + [4 (ă 2)] = 0
x =+ 7

(d) Let the oxidation state of Cr in Cr2 O72 be x.

2x + [7(ă 2] = ă 2
x =+ 6

Thus, oxidation state of C in Cr2 O72 is + 6.

Ex a m p l e 2
Determine the oxidation no. of following elements given in bold letters.
(a) H2SO5 :
Oxidation number of H = + 1
Oxidation number of O = ă 2
Oxidation number of S = a
2 ï 1 + a + 5 ï (ă 2) = 0
a = + 8 (wrong)
But this can not be true as maximum Oxidation number for S (VI gp) stands + 6. The exceptional
value is due to the fact that two O atoms in H2SO5 show peroxide linkage, i.e.,

O

H  O  O  S O  H

O
OXIDATION REDUCTION
QUIZRR 7

Thus, evaluation of oxidation number of S should be made as


2 ï 1 + a + 3 ï (ă 2) + 2 ï (ă 1) = 0
 a =+ 6
(b) H2S2O8 : Here too, O atoms form per oxide linkage. i.e.,

O O
 
H O S  O  O  S  O H
 
O O

 2 ï 1 + 2a + 6 ï (ă 2) + 2 ï (ă 1) = 0
a =+ 6
(c) NH4NO3 : 2 ï a + 4 ï 1 + 3 ï (ă 2) = 0 by rules
a = + 1 (wrong)
No doubt there are two N atoms in NH4NO3, but one N atom has negative ox. no. (attached
to H) and the other has positive oxidation number (attached to O). Therefore, evaluation should
be made separately as

oxidation number of N in NH4 and oxidation number of N in NH3

a |+ 4 ï (+ 1) = + 1 a + 3 ï (ă 1) = ă 1
 a =ă3  a =+ 5
(d) K4Fe(CN)6 : By rules, oxidation number of K= + 1
oxidation number of CNă1 = ă 1
oxidation number of Fe = a
 4 ï 1 + a + 6 ï (ă 1) = 0
 a =+ 2
(e) OSO4 : a + 4 ï (ă 2) = 0
 a =+ 8
Note : 1. The element Os and Ru show highest oxidation state i.e., + 8.
2. Recently Ba2XeO6 has been reported in which ox. no. of Xe is + 8.
(f) HCN : The evaluation can not be made directly in some cases, e.g., HCN by using rules
proposed earlier since we have no rule for oxidation number of both N and C.
In all such cases evaluation of oxidation number should be made using indirect concept or using
fundamentals by which rules have been framed.
(1) Each covalent bond contributes one unit for oxidation number
(2) Covalently bonded atoms with less electronegativity acquires positive ox. no. whereas other
with more electronegativity acquires negative oxidation number
(3) In case of co-ordinate bond, give + 2 value for oxidation number to atom from which co-
ordinate bond is directed to a more electronegative atom and ă 2 value to more electronegative
atom.

OXIDATION REDUCTION
8 QUIZRR

If co-ordinate bond is directed from more electronegative to less electronegative atom, then neglect
contribution of co-ordinate bond for both atoms in which co-ordinate bond exist.
Thus, HăC  N  Three bonds on N atom and N
1 + a + 3 ï (ă 1) = 0 is more electronegative.

a =+ 2  Ox. no. of N = 3 ï ( ă 1) = ă 3

(g) HNC : HNC


oxidation number of H = + 1
oxidation number of N = [ă 2 + (ă 1) + 0] = ă 3
for covalent [for covalent No contribution
bond with C bond with H] for co-ordinate bond
=ă3
 1 + (ă 3) + a = 0
 a =+ 2
(h) HNO3 : By rules
1 + a + 3 ï (ă 2) = 0
 a =+ 5
By fundamental approach
O
H O N
O
oxidation number of H = + 1
oxidation number of N = + 1 + (+ 2) + (+ 2) = + 5
covalent two covalent co-ordinate
bond with O bond with O bond
 N being less electronegative than O.
(i) KO2 : A super oxide of K;
oxidation number of K = + 1
oxidation number of O = a
1+2ï a =0

1
 a =ă
2
(j) Fe3O4 :
3 ï a + 4 ï (ă 2) = 0

8
 a =+
3

OXIDATION REDUCTION
QUIZRR 9

or Fe3O4 is a mixed oxide of FeO.Fe2O3.


 Fe has two oxidation no. + 2 and + 3 respectively. However, factually speaking oxidation
number of Fe in Fe3O4 is an average of two values (i.e., + 2 and + 3)

 2  2  ( 3)
Average ox. no. = 8 / 3
3

(k) KI3 : 1 + 3 ï (a)

1
a
3

or KI3 is KI + I2
 I has two oxidation no. ă 1 and 0 respectively. However, factually speaking ox. no. of I in KI3
is an average of two values ă 1 and O.

 1  2  (0) 1
Average ox. no. = 
3 3

(l) N 3H : 3ï a + 1 =0

1
a = 
3

(m) Fe0.94O : 0.94 ï a + (ă 2) = 0

200
a =
94

(n) FeSO4.(NH4)SO4.6H2O
oxidation number of Fe = a
Sum of oxidation number for H2O = 0

Sum of oxidation number for SO42 = ă 2

 a + (ă 2) + 0 + 6 ï (0) = 0
 a =+ 2
(o) NOCl : ClăN = O or use NO+Clă
oxidation number of N =  1 (for covalent bond with Cl)
oxidation number of N = + 2 (for two covalent bonds with O)
 Total ox. no.of N in NOCl = + 3
(p) NOClO4 : The compound may be written as NO+ClO4ă for ClO4ă.
For ClO4ă, let Ox. no. of Cl = a
a + 4 ï (ă 2) = ă 1
a =+ 7

OXIDATION REDUCTION
10 QUIZRR

(q) Na2[Fe(CN)5NO] : NO in iron complex has NO+ nature.


 2 ï 1 + [a + 5 ï ( ă 1) + (+ 1)] = 0
 a =+ 2
(r) [Fe(NO)(H2O)5]SO4 :
a + 1 + 5 ï 0 + (ă 2) = 0
a =+ 1
(s) Na2S4O6 : 2 ï (+ 1) + 4a + 6 ï (ă 2) = 0

5
 a
2
Here also this value is the average oxidation no. of S. The structure of Na2S4O6 is

O O
 
Na O S  S  S S  O  Na
 
O O

Thus oxidation number of each S atom forming co-ordinate bond is + 5 whereas, oxidation number
of each S atom involved in pure covalent bonding is zero.

5  5  0  0
 Average ox. no. =  5/ 2
4
(t) Na2S2O3 :
2 ï 1 + 2 ï a + 3 ï (ă 2) = 0
 a =+ 2
Here too it is the average oxidation number
The structure of Na2S2O3 is

O

Na S S  O  Na

O
The oxidation number of S involved in co-ordinate bond i.e. donor S atom is + 5. The oxidation
number of other S atom is ă 1.
(u) CrO5 or CrO(O2)2 : The structure of CrO5 is
O O
Cr
O O
O
i.e., two peroxide bonds and thus four oxygen atoms have oxidation number = ă 1 and one oxygen
atom has oxidation number = ă 2.
Thus, a + 4 ï (ă 1) + 1 ï (ă 2) = 0
a =+ 6
OXIDATION REDUCTION
QUIZRR 11

1 .3 SPECIA L M ET H OD
{If we donot know the number of peroxide linkages}
(1) Oxidation no. H2SO5 (CaroÊs Acid)
If a is the no. of peroxy linkages.
 No. of Oă atoms attached to peroxy linkage = 2a
O.N. of peroxy linkage = ă 1
Let x be the oxidation no. of S
2 ï 1 + x + (5 ă 2a) (ă 2) + 2a (ă 1) = 0
2 + x ă 2a ă 10 + 4a = 0
x + 2a = 8
a =1
x=+ 6
Peroxy linkage is formed at maximum of electronegative atom
(2) H2S2O8
2 ï 1 + 2 ï x + (za) (ă 1) + (8 ă 2a) (ă 2) = 0
2 + 2x ă 2a ă 16 + 4a = 0 O
||
2x + 2a = 14 H  O  S  O  OH
||
x+ a =7 O
a =1
x =6

O O
|| ||
H  O  S O  O  S  O  H
|| ||
O O

The compounds containing element which have highest oxidation state always act as oxidant.

+5 +6 +7 7
Example HNO3 , K 2 Cr 2 O7 , KMnO4 , K ClO4

1 .4 OX I DA T I ON - REA CT I ON
Oxidation is a process that involves the loss of electrons by the atoms or ions. Reduction is a
process that involves the gain of electrons by atoms or ions. Any reaction in which the electrons are
exchanged between atoms or ions, represents simultaneous process of oxidation and reduction is called
as Redox Reaction.
In a Redox Reaction, the species that loses electron (i.e., gets oxidised) is known as reducing
agent or reductant, (since it causes reduction of other species), the species which accepts electrons
from other reductant (i.e., gets reduced) is known as oxidising agent or oxidant (as it causes oxidation
of other species).

OXIDATION REDUCTION
12 QUIZRR

To illustrate :
Mg + Zn2+ Mg2+ + Zn is a redox reaction.
Oxidation : Mg ă 2eă  Mg2+
0

Reduction : Zn2+ + 2eă  Zn0


Mg causes reduction of Zn2+, so it is Reductant.
Zn2+ causes oxidation of Mg, so it is oxidant.
Note : A redox reaction involves two half reactions :
Oxidation and Reduction half reactions.
(1) Molecular Reaction : When one molecule gets oxidised and other gets reduced in a chemical
reaction.
reduction
+4 +7 +6
e.g. Mn O  KOH  K Mn O  K Mn O  H O
2 4 2 4 2
oxidation
Here MnO2 is getting oxidised and in KMnO4 is getting reduced (In two different molecules). So
it is intramolecular reaction.
(2) Intramolecular Reaction : When oxidation and reduction takes place in the same molecule.
3 +5 1
e.g. NH4 NO3  N 2 O + 2H2O
In this example one N is getting reduced and other is getting oxidised in the same molecule
NH4NO3. So it is a intramolecular reaction.
(3) Disproportionation Reaction : When same element gets oxidised and reduced in a reaction.
+3 +5 3
e.g. 4H3 PO3  3H3 PO4  PH3
Here only one P is oxidised as well as reduced. So it is disproportionation reaction.

Ex a m p l e 3
Which of the following are examples of disproportionation reactions ?
(a) Ag(NH3)2+ + 2H+  Ag+ + 2NH4+ (b) Cl2 + 2OHă  ClOă + Clă + H2O
(c) CaCO3  CaO + CO2 (d) 2HgO  2Hg + O2
(e) Cu2O + 2H+  Cu + Cu2+ + H2O (f) CuS + O2  Cu + SO2
(g) 2HCuCl2 + dilute with H2O  Cu + Cu2+ + 4CIă + 2H+
Solution :
The criteria for disproportionation reaction is :
A reaction in which an oxidising agent and the reducing agent is the same.
Hence among the above reactions,
(b), (e) & (g) are disproportionation reactions.

OXIDATION REDUCTION
QUIZRR 13

Ex a m p l e 4
H2SO4 acts as an oxidising agent, a dehydrating agent and an acid. Among the each of
following reactions, which behaviour is shown by H2SO4 ?
(a) C6H12O6 + H2SO4(conc.)  6C + H2O
(b) 5H2SO4(conc.) + 4zn  H2S + 4Zn2+ + 4SO42ă + 4H2O
(c) H2SO4 (dil) + Zn  Zn2+ + H2 + SO42ă
(d) H2SO4 (dil) + ZnCO3  Zn2+ + CO2 + SO42ă + H2O
Solution :
(a) as a dehydrating agent since it simply removes water from glucose.
(b) H2SO4 (S in + 6 state)  H2S (S in ă 2 state), i.e. acid is reduced. So it acts as an oxidising
agent.
(c) all strong acids liberate H2 with an active metals like Zn (i.e. it is reduced) unless SO42ă is
reduced. In this case, H+ is reduced to H2. So it acts as an oxidising agent. We can also call
it as an acidic property.
(d) simply an acid.

Ex a m p l e 5
One mole of N2H4 loses 10 mole electrons to form a new compound Y. Assuming that all the
N2 appears in new compound, what is oxidation state of N in Y? There is no change in
oxidation state of H.
Solution : N2H4  (Y) + 10e
 Y contains all N atoms
 N2ă2  (2N)a + 10e
Therefore, 2a ă (ă 4) = 10
a =+ 3

Ex a m p l e 6
In the reaction Al + Fe3O4  Al2O3 + Fe
(a) Which element is oxidized and which is reduced?
(b) Total no. of electrons transferred during the change.
Solution : 2Al0  Al2+3 + 6eă ...(1)
8e + Fe3+8/3  3Fe0
ă
...(2)
Multiplying Eq. (1) by 4 and Eq. (2) by 3, then adding
8Al0  4Al2+3 + 24e

24 e  3Fe 8 / 3  9Fe0


8Al0  3Fe3 8 / 3  4Al+3
2  9Fe
0

or 8Al + 3Fe3O4  4Al2O3 + 9Fe


Therefore, it is clear that
(a) Al is oxidized and Fe+8/3 is reduced.
(b) Total no. of electrons transferred during change = 24

OXIDATION REDUCTION
14 QUIZRR

Pro b l e m 7
The composition of a sample of wustite is Fe0.93O1.00. What percentage of iron is present in
the form of Fe III ?
Solution :

200
Oxidation no. of Fe in wustite is =  2.15
93

It is an intermediate value in between two oxidation state of Fe as, Fe (II) and (III),
Let percentage of Fe (III) be a, then
2 ï (100 ă a) + 3 ï a = 2.15 ï 100
or a = 15.05
 Percentage of Fe (III) = 15.05%

Pro b l e m 8
Identify the substance acting as oxidant or reductant reduced if any in the following:
(i) AlCl3 + 3K  Al + 3KCl
(ii) SO2 + 2H2S  3S + H2O
(iii) BaCl2 + Na2SO4  BaSO4 + 2NaCl
(iv) 3l2 + 6NaOH  NalO3 + 5Nal + 3H2O
Solution :
In a conjugate pair, oxidant has higher oxidation number
(i) For AlCl3 : Al+3 + 3e  Al0; For K : K0      K+1 + e
 Oxidant is AlCl3 and reductant is K.
(ii) For SO2 : S+4 + 4e  S0; For H2S : Să2   S0 + 2e
SO2 is oxidant H2S is reductant.
(iii) No change in oxidation number of either of the conjugate pair.
 None is oxidant or reductant.
(iv) For I2 : I20  2I+5 + 10e and I20 + 2e    2Iă1
I2 acts as oxidant and reductant both.

Pro b l e m 9
Arrange the following in order of :
(a) Increasing oxidation no. : MnCl2, MnO2, Mn(OH)3, KMnO4
(b) Decreasing oxidation no. : HXO4, HXO3, HXO2, HXO
(c) Increasing oxidation no. : I2, HI, HIO4, ICl
OXIDATION REDUCTION
QUIZRR 15

Solution :
(a) MnCl2 < Mn(OH)3 < MnO2 < KMnO4
Ox. No. +2 +3 +4 +7
(b) HXO4 > HXO3 > HXO2 > HXO4
Ox. No. +7 +5 +3 +1
(c) HI < I2 < ICl < HIO4
Ox. No. ă1 0 +1 +7

1 .5 B A L A N CI N G OF REDOX REA CT I ON S
There are 2 common and useful methods to balance redox reactions. These are
(a) Oxidation number method and
(b) Ion-electron method

Ox i d a t i o n N u m b e r M e t h o d
For balancing a redox reaction by oxidation number method, follow the order of steps as listed
below :
(i) Separate the reactants and products into two half-reactions involving the elements that
changes its oxidation number. Write the skeletal equations for each half-reaction.
(ii) For each half-reaction, deduce the oxidation numbers of the element that is changing oxidation
number.
(iii) Then balance the number of atoms of the element changing oxidation number.
(iv) Now find the total oxidation number change by determining the change per atom and
multiplying by the total number of atoms that change oxidation number. Also, decide whether
electrons are lost or gained. An increase in oxidation number is loss of electrons and a
decrease in oxidation number is gain of electrons.
(v) Add the electrons lost or gained to the half equation. Lost electrons are placed on the product
side and gained electrons are kept on the reactant side.
(vi) Now add both the half reactions after multiplying by suitable integers to make the number
of electrons lost and gained same.
(vii) Transfer the coefficient of each reactant and product to the main skeleton equation.
(viii) If the coefficients developed are not correct, then change them by inspection (Such coefficient
changes are required when an element from a compound goes in 2 different compounds, one
with the same oxidation state & the other with changed oxidation state).
(ix) Count the charges on both sides of the equation and balance the charges in the equation by
adding requisite H+ or OHă to the required side. If the reaction occurs in acidic solution, use
H+ and if it occurs in basic solution, use OHă. If the reaction occurs in a neutral solution, use
H+ or OHă on any of the side as needed i.e. in a neutral solution, if negative charges are
needed for balancing, use OHă and if positive charges are needed, use H+.
(x) Balance the hydrogens and oxygens by adding the appropriate number of H2O molecules on
the required side.

OXIDATION REDUCTION
16 QUIZRR

Ex a m p l e 1 0
Balance the following oxidation-reduction equation.
KMnO4 + KCl + H2SO4  MnSO4 + K2SO4 + H2O + Cl2
by oxidation number method.
Solution :
(i) Identify the oxidation & reduction half equations in ionic form.
Reduction half : MnO4ă1  Mn2
Oxidation half : Clă  Cl2
(ii) Balance the atoms that undergoes change in oxidation number.
Reduction half : MnO4ă1  Mn2+
Oxidation half : 2Clă1  Cl2
(iii) And the electrons lost or gained to each half equation
Reduction half : 5eă + MnO4ă1  Mn2+ ................ (A)
ă ă
Oxidation half : 2Cl  Cl2 + 2e ................. (B)
(iv) Multiply equation (A) with 2 and equation (B) with 5 and then add the two half reactions.
5e + MnO4ă1  Mn2+ ] ï 2
2Cl1  Cl2  2e ]  5
2MnO4  10Cl   5Cl2  2Mn 2
(v) Transfer the coefficients to the main equation.
2KMnO4 + 10KCl + H2SO4  2MnSO4 + K2SO4 + H2O + 5Cl2
(vi) Balance H & O atoms
2KMnO4 + 10KCl + 8H2SO4  5Cl2 + 2MnSO4 + 8H2O + K2SO4
(vii) Finally balance by inspection method
2KMnO4 + 10KCl + 8H2SO4  2MnSO4 + 6K2SO4 + 8H2O + 5Cl2

Ex a m p l e 1 1
Balance the following redox equation
K2Cr2O7 + HCl  KCl + CrCl2 + Cl2 + H2O
by oxidation number method
Solution :
(i) Identify the oxidation & reduction half equations in ionic form.
Reduction half : Cr2O72  Cr3+
Oxidation half : Cl  Cl2
(ii) Balance the atoms that undergoes change in oxidation number.
Reduction half : Cr2O72ă  2Cr3+
Oxidation half : 2Clă  Cl2

OXIDATION REDUCTION
QUIZRR 17

(iii) Add the electrons lost or gained to each half equation


Reduction half : 6eă + Cr2O72ă  2Cr3+ ............. (A)
Oxidation half : 2Clă  Cl2 + 2eă .................... (B)
(iv) Multiply equation (B) by 2 and add it to equation (B)
6eă + Cr2O72+  2Cr3+

2Cl  Cl2  2e ]  3


Cr2 O72  6Cl  2Cr3  3Cl2

(v) Transfer the coefficients to the main equation.


K2Cr2O7 + 6HCl  KCl + 2CrCl3 + 3Cl2 + H2O
(vi) By inspection, it is seen that the coefficient of HCl developed is not correct, as the
number of moles of Cl atoms on the product side are 13 while on the reactant side are
only 6. So the coefficient is changed appropriately, seeing that the net reaction is completely
balanced.
K2Cr2O7 + 14HCl  2KCl + 2CrCl3 + 3Cl2 + 7H2O

I o n -El e c t ro n M e t h o d
This method of balancing redox reaction involves following steps.
(i) Separate the reactants and products into half-reactions involving the elements that changes
its oxidation number. Write the skeleton equations for each half-reaction.
(ii) Balance each half-reaction separately involving given steps.
1. First balance the atoms of the element undergoing oxidation or reduction.
2. Then balance atoms of the elements other than hydrogen and oxygen.
3. For balancing oxygen atoms in acidic or neutral medium, add suitable number of H2O
molecules to the side deficient in O while in alkaline medium, add equal number of H2O
molecules as the excess of O on the side having excess of O atoms and add double the
number of OHă ions on the opposite side of the equation.
4. In order to balance hydrogen atom in acidic or neutral medium, add required number of
H+ to the side deficient in H while in alkaline medium, add equal number of OHă ions
as the access number of atom on the side having access H and add equal number of H2O
molecule on the opposite side of the equation.
(iii) Multiply each half-reaction by suitable integer to make the number of electrons lost and
gained same and add both the half-equations to get a completely balanced reaction.

OXIDATION REDUCTION
18 QUIZRR

Ex a m p l e 1 2
Balance the redox equation,
HNO3 + H2S  NO + S
by ion-electron method (acidic medium)
Solution :
(i) Identify the oxidation & reduction halves
Reduction half : HNO3  NO
Oxidation half : H2S  S
(ii) Atoms of the element undergoing oxidation and reduction are already balanced.
(iii) Balancing O atoms,
Reduction half : HNO3  NO + 2H2O
Oxidation half : H2S  S
(iv) Balancing H atoms,
Reducing half : 3H+ + HNO3  NO + 2H2O
Oxidation half : H2S  S + 2H+
(v) Balancing charge,
Reduction half : 3e + 3H+ + HNO3  NO + 2H2O ............... (A)
Oxidation half : H2S  S + 2H+ + 2eă ................ (B)
(vi) Multiplying equation (A) by 2 and equation (B) by 3 and then adding them
3eă + 3H+ + HNO3  NO + 2H2O] ï 2 ................ (A)

H2 S  S  2H  2e ]  3
................ (B)
2HNO3  3H2 S  2NO + 4H2 O

Ex a m p l e 1 3
Balance the following redox equation,
FeC2O4 + KMnO4 + H2SO4  Fe2(SO4)3 + CO2 + MnSO4 + K2SO4
using ion-electron method (acidic medium)
Solution :
(i) Identify the oxidation & reduction halves
Oxidation half : KMnO4  MnSO4
Reduction half : FeC2O4  Fe2(SO4)3 + CO2
(ii) In both the half-reactions, all the atoms (other then O and H) are not appearing on both sides
of the reaction. So, in the oxidation half, H2SO4 and K2SO4 are to be to added on reactant
and product side respectively.
(iii) Balancing of the atoms of the element undergoing oxidation & reduction.
Reduction half : H2SO4 + 2KMnO4  2MnSO4 + K2SO4
Oxidation half : H2SO4 + 2FeC2O4  Fe2(SO4)3 + 4CO2

OXIDATION REDUCTION
QUIZRR 19

(iv) Balancing of the atoms of elements other than O and H.


Reduction half : 3H2SO4 + 2KMnO4  2MnSO4 + K2SO4
Oxidation half : 3H2SO4 + 2FeC2O4  Fe2(SO4)3 + 4CO2
(v) Balancing O atoms,
Reducing half : 3H2SO4 + 2KMnO4  2MnSO4 + K2SO4 + 8H2O
Oxidation half : 3H2SO4 + 2FeC2O4  Fe2(SO4)3 + 4CO2
(vi) Balancing H atoms,
Reducing half : 10H+ + 3H2SO4 + 2KMnO4  2MnSO4 + K2SO4 + 8H2O
Oxidation half : 3H2SO4 + 2FeC2O4  Fe2(SO4)3 + 4CO2 + 6H+
(vii) Balancing charge,
Reduction half : 10eă + 10H+ + 3H2SO4 + 2KMnO4  2MnSO4 + K2SO4 + 8H2O.... (B)
Oxidation half : 3H2SO4 + 2FeC2O4  Fe2(SO4)3 + 4CO2 + 6H+ + 6eă.......... (A)
(viii) Multiply equation (A) by 5 and equation (B) by 3 and then adding them.
10eă + 10H+ + 3H2SO4 + 2KMnO4 + K2SO4 + 8H2O] ï 3

3H2 SO4  2FeC2O4  Fe2  SO4 3  4CO2  6H   6 e ]  5


10FeC2 O4  6KMnO4  24H2SO4  5Fe2  SO4 3  6MnSO4  3K 2SO4  20CO2  24H2O

Ex a m p l e 1 4
Balance the following redox equation,
 
AsO 33  MnO 4  AsO34  MnO2
using ion-electron method (alkaline medium).
Solution :
(i) Identify the oxidation & reduction halves

Reduction half : MnO4  MnO2

Oxidation half : AsO33  AsO34


(ii) Atoms of the element undergoing oxidation reduction are already balanced.
(iii) Balancing O atoms,

Reduction half : 2H2O + MnO 4  MnO2  4OH 

Oxidation half : 2OH   AsO33   AsO34  H2 O


(iv) Balancing H atoms,
H atoms are already balanced in both the half-reactions.
(v) Balancing charge.
Reduction half : 3eă + 2H2O + MnO4ă  MnO2 + 4OHă ...............(B)
Oxidation half : 2OHă + AsO33ă  AsO43ă + H2O + 2eă ................. (A)

OXIDATION REDUCTION
20 QUIZRR

(vi) Multiply equation (A) by 3 and equation (B) by 2 and then add (A) and (B)
3eă + 2H2O + MnO4ă  MnO2 + 4OHă] ï 2

2OH   AsO33  AsO34  H2 O+ 2e  ]  3


AsO33   2MnO4  H2O  3AsO34  2MnO2  2OH 

Mostly, the medium in which a redox reaction is to be balanced is given in the problem but if
the problem does not state the medium explicitly, then the medium is decided by looking at the
reactants or products. If an acid or base is one of the reactants or products, then the medium is the
same. If ammonia is present, the solution would be basic, for example ammonium ion is present, it
would be acidic. If metals which forms insoluble hydroxides are shown in their ionic form, the solution
is acidic.
Although, the oxidation number method and ion-electron method both lead to the correct form of
the balanced redox reaction but ion-electron method is considered superior to oxidation number method
due to following advantages :
(i) In ion-electron method, all reactants and products are completely balanced (i.e. coefficient is
developed for each one of them) whether they participate in the redox change or not, while
in oxidation number method, the balancing coefficient is developed only for the species
involved in redox change.
(ii) The balancing coefficients developed in ion-electron method are always correct and needs
no amendment while such coefficients are to be changed sometimes oxidation number
method.

Ex a m p l e 1 5
Complete and balance the following :
(a) H2O2 + I2  Iă + ?? (b) H2O2 + Sn2+  Sn4+ + ??
(c) MnO42ă + H+  Mn2+ + ?? (d) NO2 + H2O  NO + ??
Solution :
(a) I2 is reduced to Iă, so H2O2 must be oxidised, i.e. it acts as a reducing agent. Hence final
product will be O2 gas.
H2O2 + I2  Iă + O2
The balanced equation will be :
H2O2 + I2  2Iă + O2 + 2H+
(b) Sn2+  Sn4+, i.e. Sn2+ is oxidised. So H2O2 acts as an oxidising agent, which means it must
be reduced to H2O.
H2O2 + Sn2+  Sn4+ + H2O
The balanced equation will be
H2O2 + Sn2+ + 2H+  Sn4+ + 2H2O

OXIDATION REDUCTION
QUIZRR 21

(c) MnO42ă is reduced to Mn2+, i.e. it is acting as an oxidising agent. Now H+ is already in the
maximum oxidation state, so MnO42ă must also be reduced to Mn7+ state i.e. MnO4 will be
formed (disproportionation reaction)
MnO42ă + H+  Mn2+ + MnO4
The balanced equation will be :
5MnO42ă + 8H+  Mn2+ 4MnO4 + 4H2O
(d) NO2 (+ 4 state) disproportionates (like in last case) to NO (+ 2 state) and NO3ă (+ 5 state).
NO2 + H2O  NO + NO3ă
The balanced equation will be :
3NO2 + H2O  NO + 2NO3 + 2H+

Ex a m p l e 1 6
Predict the products of each of the following reactions.
(a) Na + ZnCl2  ? + ? (b) F2 + NaCl  ? + ?
(c) I2 + Fe  ? + ? (d) Iă + Fe2+  ? + ?
Solution :
(a) 2Na + ZnCl2  Zn + 2NaCl
(b) F2 + 2NaCl  2NaF + Cl2
(c) I2 + Fe  Fe2+ + 2Iă
(d) no reaction because Fe is a better reducing agent than Iă.

OXIDATION REDUCTION
IONIC EQUILIBRIA
QUIZRR 3

A CIDS AND BASES


1. The Arrhenius Concept :

Arrhenius classified a substance into an acid or a base in terms of the characteristics ions of water
which it produces in aqueous solution. Thus a n a cid is a substance which ionizes in wa ter
to produce H + (a q) or the hydronium ion; a ba se is a substance which produces OH –(a q)

HCl(aq)  H+(aq) + Clă(aq)

NaOH(aq)  Na+(aq) + OHă(aq)

Ć The strength of an acid is defined in terms of concentration of H+(aq) that is present in the
aqueous solution of a given concentration of the acid. Likewise, the strength of a base
depends upon the relative concentration of OHă(aq) in an aqueous solution of the base.
Ć A neutralisation reaction involves the combination of H+ and OHă ions to form water.

H+(aq) + OHă(aq)  H2O

Ć By Arthenius concept NH3 should not be a base. It was proposed that in aqueous solution NH3

forms the compound NH4OH, which then dissociate as a weak base into NH4 and OHă ions

H+(aq) + OHă(aq)  H2O

 ă


NH4 OH(aq)  NH4 (aq)  OH( aq)

2. Bronsted–Lowry concept of Acids and Bases

According to this concept, any hydrogen containing species ( a molecule, a cation or an anion),
which is capable of donating one or more protons to any other substance, is called an acid.
Any species (molecule, cations or anions) which is capable of accepting one or more protons from
an acid, is called a base.
Thus, according to the Bronsted-Lowry concept, an acid is proton-donor, and a base is proton-
acceptor.
The reaction of an acid with a base involves transfer of a proton from the acid to the base. So,
an acid and a base should be present simultaneously in any system. The extent of an acid-base
reaction is governed not only by the proton-donating ability of the acid, but also by the proton-
accepting tendency of the base. Acids and bases classified on the basis of this concept are termed
as Bronsted acids and bases.


  
HCl( aq)  H2 O(l )  H3O( aq)  Cl( aq)

IONIC EQUILIBRIA
4 QUIZRR
In this reaction, HCl donates its one proton to become Clă, and H2O accepts one proton to become
H3O+. Thus, HCl is Bronsted acid and H2O is a Bronsted base. For the reverse reaction., H3O+
is able to transfer its proton to Clă. So, H3O+ is a Bronsted acid and Clă is a Bronsted base.
Every acid must form a base on donating its proton, and every base must form an acid on
accepting a proton. The base that is produced when an acid donates its proton is called the
conjugate base of the acid. The acid that is produced when a base accepts a proton is called
the conjugate acid of the base. The above reaction can be written as

HCl + H2O 



 H3O  Cl ă
acid1 base 2 acid2 base1

In this Clă is the conjugate base of the acid HCl and H2O is the conjugate base of the acid H3O+.
The conjugate acid differs from conjugate base by one proton. A pair of an acid and a base which
differ from one another by a proton constitute a conjugate acid base pair. Thus,


Conjugate acid-base pair 

HCl + H2O 

 H3O  Cl ă
acid1 base 2 acid2 base1


Conjugate acid-base pair

Bronsted acid  Conjugate base of the acid + H+

Bronsted base + H+  Conjugate acid

Although the Bronsted-Lowry concept of acids and bases is better than the Arrhenius concept, it
cannot account for the acidic and basic character of compound not containing hydrogen. For
example, acidic nature of oxides such as CO2, SO2 etc., and the basic nature of the compounds
of the type CaO, Na2O etc.

R elative Strengths of Conjugate Acid–Base Pairs

A stronger Bronsted acid will have a higher tendency for donating proton to the base, hence
would tend to exist as its conjugate base. The conjugate base so formed will have very little
tendency to pick up a proton, hence would act as a weak base. Thus, there exists an interdependent
relationship between the strengths of an acid and its conjugate base.
The stronger an acid, weaker is its conjugatte base, and stronger a base, weaker is its conjugate
acid.
In water, HCl acts as a strong acid. As the reverse reaction occurs to a very small extent Clă ion
acts as a weak base. So, the anion of a strong acid is a weak base. Similarly, the cation of a weak
+
base acts as a strong acid. e.g., NH4 ion in water is a strong acid.

IONIC EQUILIBRIA
QUIZRR 5

R elative strengths of common conjugate acid-base pairs

Bronsted acid  Conjugate base of the acid + H+

HClO4 
ClO4

H2SO4 
HSO4

HI Iă

HBr Bră

HCl Clă

HNO3 
NO3

CH3COOH CH3COOă

H2CO3 
HCO3

H2 S HS ă

+
NH4 NH3

HCN CNă

H 2O OH ă

NH3 
NH 2

Amphoteric substances

Substances which can act as an acid as well as a base are called amphoteric substances. For
example, Al(OH)3, Zn(OH)2 behave both as acids and as bases in their reactions. The substance,
which can donate or accept a proton to act as an acid or a base is termed amphiprotic. For
example, the species such as HSO4 and HCO3 are amphiprotic, because of the following reactions.

HSO4ă + H2 O  H3 O + SO24  (HSO4 acting as an acid)

ă ă
HSO4ă + H2 O  OH + H2 SO4 (HSO4 acting as base)

HCO3ă + H2O  H3 O+ + CO32  (HCO3 acting as an acid)

HCO3ă + H2 O  OH  H2 CO3 (HCO3 acting as an acid)


ă

IONIC EQUILIBRIA
6 QUIZRR
Water is the most common solvent showing a unique behaviour. It can act as an acid as well as
a base. In reactions like,


 + 
H2O + NH3  NH4  OH
acid base

it acts as an acid, while it behaves like a base in a reaction of the type,


H2O + NH3 

 NH4  OH

The dual role of water molecule may thus be represented by the equation,

H 2O + H 2O 

 H3O  OH 
acid1 base 2 acid2 base1

Water is thus known as an amphiprotic solvent. Solvents which neither donate nor accept protons
are called aprotic solvents.

R elative Strength of Bronsted Acids and Bases

According to Bronsted-Lowry theory, an acid is a proton donor and base is a proton acceptor.
Therefore the strength of an acid or a base is determined by its tendency to lose or gain protons.
A strong acid is a substance which loses a proton easily to a base.

Relative strength of acids

The relative tendency of acids to transfer a proton to a common base, generally water is expressed
as the relative strength of bronsted acids. For example, HCl has a higher tendency to transfer a
proton to H2O than CH3COOH. This means HCl is a stronger acid than acetic acid. Thus,


 + 
HCl + H2O  H3O  Cl
(higher tendency for forward reaction)


 ă 
CH3COOH + H2O  CH3COO + H3O
(small tendency for forward reaction)

Similarly, a base having a higher tendency to accept a proton is stronger. For example, ammonia
accepts a proton more readily from water molecule than a water molecule from another water
molecule, and hence ammonia is more basic than water.


 + 
H2O + H2O  H3O  OH
(very little tendency for the forward reaction)

IONIC EQUILIBRIA
QUIZRR 7


  
H2O + NH3  NH4  OH

(little tendency for the forward reaction)

The ability of an acid to lose a proton (acid strength) is described by its acid ionization constant.
The larger the value of the acid ionization constant Ka, higher is the concentration of H3O+ in
the solution, stronger is the acid. Knowing the ionization constants of acids one can get the
relative strengths of different acids at a particular temperature.


  
CH3COOH + H2O  CH3COO  H3O

Acetic acid, (CH3 COOH) K a  1.85  105


H O  CH COO 
 3   3 
Ka 
CH3 COOH


  
HF + H2O  F  H3O

H O  F  
3
Hydrofluoric acid, (HF) Ka  6.7  10 4
Ka    
[HF]

Since the ionization constant of HF is larger than the ionization constant of CH3COOH, hydrofluoric
acid (HF) is a stronger acid than acetic acid, (CH3COOH).

Relative strength of bases


The ability of a base to accept a proton (basic strength) is described by its base ionization constant
(Kb). The larger the value of Kb, higher is the concentration of OHă, stronger is the base.
For example,


  
H2O + NH3  NH4  OH

 NH   OH 
Kb  
4    1.8  10 5
[NH3 ]


  
H2O + C6 H5 NH2  C6 H5 NH3  OH ,

Since the value of Kb for aniline is much less than that of ammonia, hence aniline (C6H5NH2)
is a weaker base than ammonia (NH3).

IONIC EQUILIBRIA
8 QUIZRR
Example 1

Give the conjugate acids of the following :


(a) OHă, HCO3ă, HPO4-ă2, CH3NH2, CO3ă2, NH3, CH3COOH
(b) Give the conjugate bases of the following : HSă, H3O+, H2PO4ă, HSO4ă,HF, CH3COOH,
[Al(H2O)6]3+.
Solution :
(a) OH ă : H 2O
 : H2CO3
HCO3

2 
HPO4 : H2PO4

CH3NH2 : CH3 NH3

 
CO23 : HCO3


NH3 : NH4


CH3COOH : CH3COOH2

(b) HS ă : S2ă
H 3O + : H 2O

ă 2ă
H2 PO4 : HPO4

HF : Fă
CH3COOH : CH3COOă
[Al (H2O)6]3+ : [Al(H2O)5OH]2+

3. LEWIS CONCEPT OF ACIDS AND BASES


In 1938 G.N. Lewis put forward a more generalized acid-base concept.
According to this concept, an acid is a species (charged or uncharged) which, can accept a pair
of electrons and a base is a species (charged and uncharged), which can donate a pair of electrons
towards the formation of a covalent bond. Thus, a Lewis acid is an electrophilic, and a Lewis base
is a nucleophile. For example, in the reaction: NH3 is a donor of electron pair and BF3 accepts
a pair of electrons. So, according to he Lewis concept, : NH3 is a Lewis base, and BF3 is a Lewis
acid.

IONIC EQUILIBRIA
QUIZRR 9

Lewis acids

The following types of compounds or species are Lewis acids :


Ć All molecules possessing an atom with incomplete octet of electrons or Lewis acids.
Examples : BF3, AlCl3

Ć All cations are Lewis acids,


Examples : Ag+, Cu2+, Fe2+, Fe3+ etc. are Lewis acids.

Ć All molecules in which the central atom can acquire more than eight valence electrons.
Examples : SiCl4, SiF4, etc. are Lewis acids.
Ć Molecules with multiple bonds between atoms of different electronegativities e.g. CO2, SO2
etc. are Lewis acids.

Lewis bases

The following types of compounds or species are Lewis bases.


Ć All anions are Lewis bases e.g. OHă, CNă,CH3COOă etc.
Ć All molecules having ionăpair of electrons viz : HN3, H2O, CO; and amines etc. are Lewis
bases. It may be noted that all Bronsted bases are also Lewis bases, but all Bronsted acids
are not Lewis acids.

Lewis Acid–Base Reaction

Whenever a Lewis acid reacts with a Lewis base, a covalent bond is formed.
For example, CO2 (Lewis acid) reacts with OHă(Lewis base) to give HCO3ă (bicarbonate ion), BF3
(boron trifluoride) behaves as a Lewis acid and NH3 (ammonia) as a Lewis base.
Calcium oxide (CaO) behaves as a Lewis base, and sulphur trioxide (SO3) as a Lewis acid in the
reaction between the two.
The Lewis concept of acids and bases suffers from the following limitations.
Ć It fails to account for the relative strength of acids and bases, as it is not based on ionization.
Ć An acid-base reaction being an electron-transfer reaction, should be quite fast. However,
many Lewis acid-base reactions are slow.

IONIC EQUILIBRIUM
This type of equilibrium is observed in substances that undergo ionization easily, or in polar
substances in which ionization can be induced. Ionic and polar substances are more easily soluble
in polar solvents because of the ease of ionization taking place in the solvent medium. With the

IONIC EQUILIBRIA
10 QUIZRR
dissolution of ionic and polar substances in the solvent, these solutions become rich in mobile
charge carriers (ions) and thus can conduct electricity. Substances, which are capable of conducting
electricity are called as electrolytes while those substances which are non-conducting are called as
non-electrolytes.

I onization in electrolytes

Strong electrolytes are almost completely dissociated/ionized into the constituent ions in aqueous
solution. Even at higher concentrations, very small amount of any strong electrolyte is present in
the unionized form. Weak electrolytes are however, partially ionized and a dynamic equilibrium
exist between the inonized and unionized forms.
In 1887, s. Arrhenius postulated the first theory of electrolytic ionization. The basic postulates of
his theory are.
Ć When an electrolyte is dissolved in water, it ionizes to produce ions in the solution. These
ions are free to move throughout the bulk of the solution.

Ć The total number of positive charges is equal to the total number of negative charges in any
solution of any electrolyte. The ions tend to recombine to form the unionized electrolyte.
Hence, a dynamic equilibrium exists between the ionized and unionized form of the electrolyte.
For example, an electrolyte AB ionizes in the solution to form A+ and Bă. The equilibrium
established in the solution is :

AB 

 A+ + B

The ionization constant (K) is then given by,

 A   B  
K    
[AB]

Ć The properties of an electrolyte in the solution are due to the ions it gives in the solution.
For example, a solution of copper sulphate is blue due to the presence of Cu2+(aq) ions in
it.

Ć The fraction of the total number of molecule present as free ions in the solution is known
as the degree of ionization (). This is defined as,

Number of molecules split into ions


 
Total number of molecules dissolved

The degree of ionization at any concentration (C) of AB in moles per litre is denoted as (C).

IONIC EQUILIBRIA
QUIZRR 11

OSTWALD’S DILUTION LAW


This law deals with the law of mass action to the equilibrium between the ions and unionized
molecules in aqueous solutions of weak binary electrolytes such as acetic acid, propionic acid,
HCN, NH4OH etc.
Consider a dilute solution of weak binary electrolyte AB containing C moles in 1 litre. At any
given temperature, there will be equilibrium between the unionized molecules of AB and the ions
A+ and Bă. Let () be the degree of dissociation of AB at equilibrium.

Initial Conc. AB 



 A+ + Bă
C O O
Conc. at equilibrium C (1 ă ) C C

Then according to law of mass action,

 A    B 
(C) (C) C 2
K      
[AB] C(1 ă ) (1  )

As ÂÊ is generally small, the term 1    1 .

Hence, K = C2

or  K /C = K.V

where ÂVÊ is the volume of the solution in litres containing one mole of the solute. From the above
equation it follows that if the concentration decreases, the degree of ionization () must increase.
Thus, at constant temperature, an electrolyte in solution gets ionized to a larger extent with
increasing dilution. This is called Ostwald dilution law. This law is not applicable to solutions of
strong electrolytes as they are completely dissociated and there is no equilibrium between the
dissociated and undissociated molecules.

Ionization of an acid (weak)

An acid of the type HA can undergo ionization when dissolved in water as.


  
HA + H2O  H 3O  A

If ÂnÊ moles of the acid are dissolved in ÂVÊ units of volume (litres L) and () is the degree of
ionization, then the equilibrium amounts of various species and the concentration in moles per
litre in the solution are,
Number of moles of [HA] = n (1 ă ) = C (1 ă ) mol Lă1
Number of moles of [H3O+] = n = C mol Lă1

IONIC EQUILIBRIA
12 QUIZRR
Number of moles of [Aă] = n = C mol Lă1
Where ÂCÊ is the molar concentration of the acid. The ionization constant for the above reaction
is given by,

H O+   A ă  2
Ka = 
3     (C) (C)  C
[HA] C(1  ) (1  )

If ÂÊ is amall then 1    1

Hence, Ka = C2

or   Ka / C  KaV

where ÂVÊ is the volume of the solution in litres containing one mole of the acid HA. As the degree
of ionization increases with-dilution the, the hydronium ion or hydrogen ion concentration is
given by.

Ka
[H3O ]  C  C  K aC
C

Ionization of a base (weak)

The ionization of a weak base is characterized by the equilibrium,


  
MOH  H2O (excess)  M ( aq)  OH ( aq)

If ÂnÊ moles of the base are dissolved in ÂVÊ units of volume (litres L) and  is the degree of
ionization, then the equilibrium amounts of various species and the concentration in moles per
litre in the solution are,
Number of moles of [MOH] = n (1 ă ) = C (1 ă ) mol Lă1
Number of moles of [OHă] = n = C mol Lă1
Number of moles of [M+] = n = C mol Lă1
where ÂCÊ is the molar concentration of the base. The ionization constant for the above reaction
is given by,

 M   OH ă  2
Kb      (C) (C)  C
[MOH] C(1  ) (1 ă )

If ÂÊ is small then 1    1

IONIC EQUILIBRIA
QUIZRR 13

Hence, Kb = C2

Or   Kb / C  Kb V

where ÂVÊ is the volume of the solution in litres containing one mole of the base MOH. As the
degree of ionization increase with dilution then, the hydroxide concentration is given by.

Kb
[OH ]  C  C  Kb C
C

Protic acids

Acids which contain ionizable hydrogen are called protic acids. These are classified as.

Types of protic acids

Monoprotic acids

These acids contains only one ionizable hydrogen in its molecule. It is denoted by HA.

Example : HCl, CH3COOH, HNO3 etc.

Diprotic acids

The diprotic acids contain two ionizable hydrogen in its molecule. They are denoted by H2A.
Example : H2SO4, (COOH)2, H2CO3 etc.

Triprotic acids

These acids contain three ionizable hydrogen in its molecule. It is denoted by H3A.
Example : H3PO4 etc.

In all these acids, the primary ionization constant (K1) is stronger than the secondary (K2), which
is much stronger than the tertiary (K3) ionization constant. This is because a proton (H3O+) would
be released more readily from an uncharged molecule than from a mono-negative ion, and more
readily from a mono-negative ion than from a bi-negative ion. Also the hydronium concentration
from the first ionization will suppress the subsequent ionizations due to common ion effect (this
will be discussed a little later in the chapter)
Some general observations on the behaviour of protic acids in aqueous solution are :
Ć The protic acids which have very high value of the acid dissociation or ionization constant,
ionize/dissociate almost completely in aqueous solution at ordinary dilutions. They are called
strong acids.
Example : HCl, H2SO4, and HNO3.

IONIC EQUILIBRIA
14 QUIZRR
Ć Acids, which dissociate in aqueous solution to a smaller extent at ordinary dilution, to give
low concentration of H+ ions in solution, are called weak acids.
Example : CH3COOH, (COOH)2, H3PO4, HCN, HF, etc. The Ka or K1 values of such acids
are much smaller than one.
Ć On dilution, the ionization of an acids increase. So, concentration of H+ ions also increase
on dilution. Therefore, strength of the acid increases with dilution. It is for this reason that
the acid strengths are compared at equal concentrations.
Ć For di-and tri- protic acids, first dissociation is stronger than the second, which in turn is
much stronger than the third dissociation i.e., K1  K2  K3. This is because the removal
of a proton from a negatively charged species is more difficult than from a neutral molecule.

Example 2

Which of the following acids is the strongest and which one is the weakest. Explain.
Solution :
Nitrous acid (HNO2) Ka = 5.1 ï 10ă4
Sulphurous acid (H2SO3) Ka = 1.7 ï 10ă2
Phosphoric acid (H3PO4) Ka = 7.1 ï 10ă3

 A   H  
For, HA  A  H K a    

  + 
[HA]

Thus, larger the Ka value stronger is the acid. Therefore,

H2SO3  H3PO4  HNO2


strongest weakest

Example 3

Calculate the degree of ionization and [H3O+] of a 0.1 mol/L solution of acetic acid. Given
Ka (CH3COOH)=1.8 ï 10ă5 mol/L.
Solution :
Let ÂÊ be the degree of ionization. The concentration of various species involved in the equilibrium
are as follows.


  
CH3COOH + H2O  CH3COO  H3O

At equilibrium : C (1 ă ) C C

IONIC EQUILIBRIA
QUIZRR 15

The equilibrium constant for the ionization of CH3COOH can be written as follows :

CH COOă  H O+  2
Ka = 
3   3   (C) (C)  C
[CH3COOH] C (1 ă ) (1 ă )

CH3COOH is very feebly ionized. So, ÂaÊ may be ignored in comparison to 1. Then,

Ka 1.8  10 5 mol/L
   1.34  10 3
C 0.1mol/L

Then, [H3O+] = C = (0.1 mol/L) ï (1.34 ï 10ă2)


= 1.34 ï 10ă3 mol/L

Example 4

At 298 K a 0.1 M solution of acetic acid is 1.34% ionized. What is the ionization constant
(Ka ) for the acid ?
Solution :
Acetic acid ionized as follows :



CH3 COOH  H2 O  CH3 COO   H3 O 

Initial conc/mol Lă1 0.1 0 0

Equilibrium conc/mol Lă1 0.1 (1 ă ) (0.1) (0.1)

 CH COO    H O 
 3  3  0.1   0.1 
Ka  
[CH3COOCH] 0.1 (1 ă )

1.34
   0.0134
100

0.1  2 0.1  (0.0134)2


So, Ka    1.82  105
(1  ) (1  0.0134)

IONIC EQUILIBRIA
16 QUIZRR
I onic Product of Water

The dissociation constant K of H2O, dissociating very slightly into H+ and OHă (H2O  H+ +
OHă) is given by

[H+ ] [OH ă ]
K
[H2O]

The concentration of an undissociated molecule of H2O (i.e., [H2O]) is practically equal to the
initial molar concentration of water as water dissociated feebly. [H2O] may thus be taken as
constant, the value of which is equal to 100/18, i.e., 55.54 moles per litre supposing the density
of water as 1 g per mL.
K ï [H2O] = [H+] [OHă]
or Kw = [H+] [OHă]
Kw is known as ionic product of water which increases with the increase in temperature.
At 25ĈC, Kw = 1 ï 10ă14 (mole/L)2.

At the body temperature (37ĈC), Kw = 2.5 ï 10ă14 (mole/L)2

For pure water (neutral) [H+] = [OHă]

 [OH+] = [OHă] = K w  10 14  107 mole/L

For any acidic solution, [H3O+] > [OHă]


Or [H3O+] > 10ă7 M
For any alkaline solution,
[H3O+] < [OHă]
Or [OHă] > 10ă7 M

Note : Kw is a constant irrespective of the presence of other ions in the solution.

Effect of temperature on K

The value of Kw varies with the change in temperature. The values of [H3O+] and [OH] are
always equal to each other at all temperatures but the values of Kw are different at different
temperatures. The value of Kw increases with the rise in temperature. This is because increase
in temperature will shift the equilibrium in the forward direction producing large concentrations
of [H3O+] and [OH] ions (Le ChatellerÊs principle).
Hence, Kw increases with rise in temperature.

IONIC EQUILIBRIA
QUIZRR 17

I n acidic solution

When an acidic solution of HCl is added to a pure neutral solution of water, the concentration
of [H3O+] becomes larger than 1.0 ï 10ă7 mol Lă1. The dissociation equilibrium of water shifts in
the reverse direction (Le ChatelierÊs principle). The excess [OH]ă ions combine with hydronium
ions to form undissociated water molecules so that he value of Kw remains constant in the
solution. The concentration of [OH] ions will then be equal to

Kw
[OH  ] 
H O 
 3 

The concentration of [H3O+] is more than the concentration of the [OHă] ions in acidic solution.
When a few drops of a base like NaOH is added to pure water, the concentration of [OHă]
incr eases and t hat of hydr ogen ions decr eases. The concent r at ion of [H 3O+] can be calculated
as :

Kw
[H3 O ] 
OH 
 

Thus in basic solution the concentration of [OH] will be greater than of [H3O+]
It can be concluded that the hydronium and hydroxyl ions are always present in solution whether
they are acidic or basic. However their concentrations differ.

In acidic solution : [H3O+] > [OHă]


In basic solution : [H3O+] < [OHă]
In neutral solution : [H3O+] = [OHă]

pH SCALE :
pH of solution is the negative logarithm to the base 10 of the concentration of H+ ions which it
contains (Basically P acts as a mathematical operator : p(A) = ă log10 [A])

 pH = ă log10[H+]

At 25ĈC, using this definition, a scale called pH scale is developed as follows :

For H2O, H+ = 10ă7 M


pH (H2O) = ă log (10ă7) = 7

 For a neutral solution; pH = 7

IONIC EQUILIBRIA
18 QUIZRR
(1) If [H+] > 10ă7 in a solution i.e., solution is acidic then pH < 7
(2) If [H+] < 10ă7 (or [OHă] > 10ă7) i.e. solution is basic then pH > 7.

Acidic H2O Basic


At 25ĈC 3 4 5 6 7 8 9 10 11
(298 K)

Acidity increases  pH  Basicity increases


(Neutral)

1. pH Calculation of strong acid


A strong acid completely ionises in water.

For example, HCl  H+ + Clă

Let C be the concentration of HCl.


The concentration of H+ obtained from HCl will also be C.

 pH = ă log C

This process cannot be used to calculate the pH of 10ă7 M HCl. Since, we know that any acidic
solution will have pH less than 7, we must adopt some other method.

M ethod I

We already know the concentration of H+ (or H3O+) ions from pure water is 10ă7 M. So all we
need to do is to add the H+ ion concentration from water and H+ (or H3O+) ion concentration from
HCl to get the [H+]Total.

 (H  ) T  (H  )HCl  (H  ) water  10 7 M  107 M  2  10 7 M

 pH = ă log (H+) = ă log 2 ï 10ă7 = 6.6989

Though the method looks correct but there is a mistake in it. In chemical equilibrium we have
learnt about Le-ChatelierÊs Principle, which states that when concentration of a reactant or
product is changed the reaction tends of go forward or reverse to finally reestablish equilibrium.
We can see that in the present case, by adding 10ă7 M HCl to water, we are increasing the [H+].
This should make the reaction go in the reverse direction. Therefore the [H+] from water would
become less than 10ă7 M. So the method will yield incorrect result.

IONIC EQUILIBRIA
QUIZRR 19

M ethod II

The statement given above that ÂÂthe addition of an ion to an equilibrium, having the same ion
makes the equilibrium move in a direction to consume that ion’’ is called the c ommon ion effect.
This implies that water would dissociate less in the presence of HCl. Let the amount of water
dissociated be x´ in the presence of 10ă7 M HCl.

 + 
2H2O  H3O  OH
(x´ + 10ă7) x´

At equilibrium

[H3O+] [OHă] = 10ă14

(x´ + 10ă7) (x´) = 10ă14

Calculating for x´, we get x´ = 0.618 ï 10ă7 M.

Here we can clearly see the common ion effect in operation. Water which was dissociating to give
10ă7 M H3O+ ions, has now experienced the common ion effect and has now yielded only
0.618 ï 10ă7 M H3O+ ions.

Therefore, [H3O+]T = (10ă7 + 0.618 ï 10ă7) M

 pH = ălog (1.618 ï 10ă7) = 6.7910

Now, let us answer the question as to why we take the [H+] from water into account for calculating
the pH of 10ă7 MHCl while for calculating the pH of 10ă6 M, 10ă5 M, 10ă4 M, 10ă3 M, 10ă2 M
and 10ă1 M HCl, we have not considered it. It can be seen that the H+ ions from water, decreases
due to the common ion from HCl. Greater the concentration of the common ion added, greater will
be the effect. Therefore, for concentrations higher than 10ă6 M (and inclusive of 10ă6 M), the H+
from water will be even less than 0.618 ï 10ă7 M and would be so small in comparison to the [H+]
from HCl, that we can ignore it.

Thus, it is concluded that [H3O+] from water needs to be considered only if the [H3O+]
from acid is < 10ă6 M and if [H3O+] from acid is > 10ă6 M, the [H3O+] from water can be
ignored.

IONIC EQUILIBRIA
20 QUIZRR
Example 6

While calculating the pH of 10ă7 M HCl, we have taken into account the common ion effect
of HCl on water. Why did we not take the common ion effect of water on HCl ?
Solution :
Let us assumed that the Kw of water be 10ă10 at 25ĈC. Then in pure water, x2 = 10ă10 ; x = 10ă5
Let us now calculate [H3O+] from the water in the presence of 10ă7 M HCl using the same value
of Kw.
(x ´ + 10ă7) x´ = 10ă10
 x´ = 0.995 ï 10ă5 M
5 5
The percentage decrease in the concentration of H3O+ of water is 10  0.995  10  100  0.5%
10 5
Thus, it can be concluded that larger the value of equilibrium constant of the weak electrolyte,
lesser is the common ion effect produced. So, the common ion effect is experienced more by the
weak electrolyte (having smaller equilibrium constant) than the strong electrolyte (having larger
equilibrium constant).

Example 7

Calculate the pH value of (i) 0.001 M HCl and (ii) 0.01 M NaOH
Solution :
(i) Since HCl is a strong acid, it completely ionizes and therefore, H3O+ ions concentration is equal
to that of the acid itself i.e.,
[H3O+] = [HCl] = 0.001 M = 1 ï 10ă3 M
now, pH = ă log [H3O+]
pH = ă log [1 ï 10ă3]
= ă (ă 3) log 10 = 3
(ii) Since NaOH is a strong base, it completely ionizes and therefore, OHă ions concentration is equal
to that of the base itself i.e.,
[OHă] = [NaOH] = 0.01 M = 1 ï 10ă2 M
Kw = [H3O+] [OHă]

Kw 1  1014
[H3O ]    1  1012
OH   1 10 2
 

pH = ă log [H3O+]
pH = ă log [1 ï 10ă12]
= ă (ă 12) log 10 = 12

IONIC EQUILIBRIA
QUIZRR 21

Example 8

Acid A, B, C and D have the following pKa values : A = 1.5, B = 3.5, C = 2.0, D = 5.0. Arrange
these acids in the increasing order of acid strength.
Solution :
We know that,
pKa = ă log Ka or Ka = 10ăpKa
Therefore, for the given acids,
Ka (A) = 10ă1.5 Ka (B) = 10ă3.5
Ka (C) = 10ă2.0 Ka (D) = 10ă5.0
Since, 10ă5.0 < 10ă3.5 < 10ă2.0 < 10ă1.5
Hence, the strength of acids follows the order, D < B < C < A
Weakest Strongest

Example 9

The value of Kw is 9.55 ï 10ă14 at a certain temperature. Calculate the pH of water at this
temperature.
Solution :
Kw = 9.55 ï 10ă14
For water [H 3O+] = [OHă]
If K w = [H3O+] [OHă] = 9.55 ï 10ă14 then,
[H3O+] [H 3O+] = 9.55 ï 10ă14
[H 3O+] 2 = 9.55 ï 10ă14

or [H3O ]  9.55  1014  3.09  10 7 mol L1

pH = ă log [H3O+]
pH = ă log [3.09 ï 10ă7]
= ă (log 3.09 + log 10ă7)
= ă (0.49 ă 7) = 6.51

2. pH Calculation of Weak Acid

Weak acid is that which dissociate partially in water and soon comes in equilibrium with its ions
as


  
CH3 COOH  H2 O  CH3 COO  H3 O

IONIC EQUILIBRIA
22 QUIZRR
The equilibrium constant is called Ka (dissociation or ionization constant).

[CH3COO ]  [H3O ]
Ka 
[CH3COOH]

Let the degree of dissociation of weak acid be .


  
CH3COOH  H2O  CH3COOH  H3O

Initial conc. c 0 0
Conc. at equilib. căc c c
If  < < 1 than 1 ă   1 (i.e. for  less than or equal to 0.1)

Ka
Ka  c2 or 
c

Ka
[H3O ]  c  c   Ka  c
c

Using the value of Ka and c, [H3O+] can be calculated and then pH of the solution can be
calculated.
Taking log of both the sides of above equation,
log [H3O+] = ólog Ka + ólog c
Multiplying both sides by ă 1,
ă log [H3O+] = ă ólog Ka ă ólog c

pH = ópKa ă ólog c = ó(pKa ă log c )

Example 10

The self ionisation constant for pure formic acid, K = [HCOOH2+] [HCOOă] has been estimated
as 10ă6 at room temperature. What percentage of formic acid molecules in pure formic acid
are converted to formate ion ? The density of formic acid is 1.22 g/cm3.
Solution:
Given density of formic acid = 1.22 g/cm3
 Weight of formic acid in 1 litre solution = 1.22 ï 103 g

1.22  103
Thus, [HCOOH]   26.5 M
46

IONIC EQUILIBRIA
QUIZRR 23

Since in case of auto ionisation [HCOOH2+] = [HCOOă]


and [HCOOă] [HCOOH2+] = 10ă6
[HCOOă] = 10ă3

[HCOO ă ] ï 100 103


Now % dissociation of HCOOH =   100
[HCOOH] 26.5

 C 
  C(1  )  = 0.004%
 

Example 11

Liquid ammonia ionises to a slight extent. At ă50ĈC, its self ionisation constant, KNH =
3
[NH4+] [NH2ă] = 10ă30. How many amide ions, are present per cm3 of pure liquid ammonia ?
Assume N = 6.0 ï 1023.
Solution :


  
2NH3  NH4  NH2 (self ionization)

K  [NH4 ] [NH2 ]
ă
and

[NH4 ]  [NH2 ]
ă

[NH4 ]  [NH2 ]
ă

K  1030  1015 M
 ă
[NH2 ] 

Number of amide ions in 103 cm3 = 10ă15 ï 6 ï 1023

3
10 15  6  1023
Number of amide ions in one cm =
103

= 6 ï 105 ions

IONIC EQUILIBRIA
24 QUIZRR
Example 12

Calculate the concentration of fluoroacetic acid which is required to get [H+] = 1.50 ï 10ă3 M.
Ka of acid = 2.6 ï 10ă3.

Solution : CH2 FCOOH 



 CH2 FCOO  H 

Mole before dissociation 1 0 0


Mole after dissociation (1 – )  
Given, [H+] = C = 1.5 ï 10ă3 mol litreă1

(C) (C) C 2
 Ka  
C(1  ) (1  )

1.5  103  
2.6  103 
(1 ă )

  = 0.634
Now, C = 1.50 ï 10ă3

1.50  103
 C  2.37  10 3 M
0.634

Note : Since Ka is of the order of 10ă3 M and thus it is not advisable to use Ka = C2. Because
(1 ă ) is not equal to 1 since a is not small.

Example 13

Determine degree of dissociation of 0.05 M NH3 at 25ĈC in a solution of pH = 11.


Solution : 

 NH4 OH  NH4  OH

1 0 0
(1 ă )  
Given, pH = 11
 [H + ] = 10ă11
 [OHă] = 10ă3 = C
Since, C = 0.05

10 3 10 3
  =   2  102 or 2%
C 0.05

IONIC EQUILIBRIA
QUIZRR 25

Example 14

K1 and K2 for dissociation of H2S are 4 ï 10ă3 and 1 ï 10ă5. Calculate sulphide ion
concentration in 0.1 M H2S solution. Also report [H+] and pH.

Solution : H2 S  H  HS 

[H+ ] [HS ă ]
K1   4  103
[H2S]

 [H+] = C; [Hă] = C; [H2S] = C (1 ă )

C . C C 2
or 4  103  
C(1  ) (1  )

0.1 C2
or 4  103   (1 ă  should not be neglected)
C(1  ) (1  )

  = 0.18
 [H+] = C = 0.1 ï 0.18 = 0.018 M  pH = 1.7447
[HSă] = C = 0.1 ï 0.18 = 0.018 M
[H2S] = C (1 ă ) = 0.1 (1 ă 0.18) = 0.082 M
Now HSă further dissociates to H+ and Să2; C1 = [HSă] = 0.018 M

HS  

 
 H  HS

1 0 0
(1 ă )  

[H  ] [S2 ]
 K a2  1  10 5 
[HS ]

 [H+] already in solution = 0.018 and thus, dissociation of HSă further supresses due to
common ion effect and 1 ă   1.

0.018  C11
 1  10 5   0.0018  1
C1 (1  1 )

1  105
 1  = 5.55 ï 10ă4
0.018

IONIC EQUILIBRIA
26 QUIZRR
 [Să2] = C11 = 0.018 ï 5.55 ï 10ă4 = 10ă5
[HSă] = C1 (1 ă 1) = C1 = 0.018 M

Example 15

Calculate the pH of 10ă6 M CH3COOH. Ka (CH3COOH) = 1.8 ï 10ă5.


Solution :
We can solve this problem by two methods. The methods are as follows :

Method 1 :
[H3O+] due to a weak acid is given by

[H3O ]  K a  c  1.8 105  10 6  4.24  10 6

pH = 5.37
Though it seems that we have solved the problem correctly, there is an error in this method. This
error can be highlighted by comparing it with the pH of 10ă6 M HCl. We can see that the pH
of 10ă6 M HCl would be 6. As we know that lesser pH implies higher concentration of H3O+ ions.
So a weak acid having the same concentration as that of a strong acid cannot give higher
concentration of H3O+ ions. The error in pH calculation has occurred because we have assumed
that  is very much small compared to 1 without knowing the value of . In fact, if we make this
assumption, the  we get is,

Ka 1.8  105
   4.24
c 106

This value is no possible as  cannot be more than 1, ever. Therefore, the assumption made by
us is not valid and pH of 10ă6 M CH3CO2H is to be calculated without any assumption.

Method 2 :

c 2 106   2
Ka  , 1.8  10 5 
1 1

Solving for ,  = 0.95


Note that the value of a is quite high, so it cannot be ignored with respect to 1.
 [H3O+] = c = 0.95 ï 10ă6 = 9.5 ï 10ă7
The [H3O+] from acetic acid < 10ă6 M, so [H3O+] from water should also be considered.
 (x´ + 9.5 ï 10ă7) x´ = 10ă14 (where x´ is the [H3O+] coming from water in the presence of
10ă6 M CH3COOH).

IONIC EQUILIBRIA
QUIZRR 27

x´ = 1.04 ï 10ă8
[H3O+]T = (9.5 ï 10ă7 + 1.04 ï 10ă8) = 9.6 ï 10ă7
pH = 6.01.

The above illustration shows a need to know when the assumption (that  is very small compared
to 1) is valid and when not. Thus, to calculate the pH of a weak acid, we should proceed as

Ka
First calculate the value of  from the expression  = .
c

If this value of  comes out to be less than or equal to 0.1, then the assumption is valid and

[H3O+] is calculated using the expression, [H3O+] = K a  c but if the  value obtained from

the given expression > 1, then the assumption is not valid and one has to calculate  using the

c 2
expression, K a  and then calculate [H3O+] using the expression, [H3O+] = c.
1

It was suggested that for calculating the pH of weak acid, first calculate  from the expression,

= Ka and then compare its, value with 0.1. This statement looks a little odd. How can we
C

compare the value of  (which is obtained after making the assumption) with 0.1 and then claim
that it is correct if  < 0.1 ? Let us see how the given procedure works.

2
The actual expression for calculating the value of  is K a  c . If we make the assumption
1

that  is very small compared to one and ignore it with respect to 1 in the denominator, we get
Ka = c2. By ignoring the  in the denominator, we increase the value of the denominator. This
will consequently increase the value of the numerator (since the ratio is a constant). Thus, the
value of a calculated after the assumption will be greater than the actual value of . If this
approximated value of a is less than or equal to 0.1 then it means that the actual value of  will
be even less than 0.1. Therefore, the assumption is valid and if this approximated value of  is
greater than 0.1, then the actual value of  may be greater than, equally to or less than 0.1. So
in the later case, we are unable to predict and so we do not make any assumption and calculate
c 2
actual value of  using the expression, K a  .
1

IONIC EQUILIBRIA
28 QUIZRR
Some Important Points :
Ć Relative strength of two weak acids is a measure of [H3O+]

[H3 O ]1 K1C1

 K 2C2
[H3 O ]2

Ć pH of a weak monobasic acid

pH = ă log [H3O+] = ă log K aC

1
= [ pK a  log C]
2
Total [H3O+] in a mixture of two weak acids.

[H3O ]  K1C1  K 2C2

Ć For weak bases :


  
BOH  B  OH

[B ] [OH  ] Cx2


Kb    Cx2
[BOH] (1  x)

Kb
x
C

[OHă]weak base = Cx = Kb C

Ć Relative strength of two weak bases

[OH  ]1 K1C1

 K 2 C2
[OH ]2

Ć pOH of weak monoacid base

1
pOH   log (OH )  [ pK b  log C]
2
pH = 14 ă pOH

= 14  1 [ p K b  log C]
2
Total [OHă] in a mixture of two weak bases.

[OH ]  K1C1 + K 2C2

IONIC EQUILIBRIA
QUIZRR 29

HYDROLYSIS OF SALTS

Dissolution of different salts in water does not always result in neutral solutions. For example,
aqueous solution of copper sulphate is acidic whereas aqueous solution of sodium acetate is basic
and aqueous solution of sodium chloride forms neutral solution. This is due to the dissociation of
the salt in water to form ions. This process of the reaction of anion or cation of the salt with water
to produce an acidic or an alkaline solution is called hydrolysis. Thus, hydrolysis is reverse of
neutralization.
The equilibrium constant of such a reaction is called hydrolysis constant.
Hydrolysis of anion (Aă)

BA  H2O  HA  BOH


Salt Acid Base

or A   H2O  HA  OH

[HA] [OH ]
Kh 
[A ă ]

Hydrolysis of cation (B+)

[BOH] [H3O ]
Kh 
[B + ]

The fraction of the total salt that gets hydrolysed at equilibrium is called degree of hydrolysis. It
is written as ÂhÊ.

Relation of Hydrolysis Constant and Ka and Kb

(1) Salt of weak acid and strong base


Here, the anion (Aă) is a stronger base than OHă, hence it undergoes hydrolysis to give free OHă
ions. Therefore the resulting solution will be basic in character having pH greater than 7. For
example.


  
CH3COONa + H2O  CH3COOH  Na  OH
Weak acid

Other examples of this of salts are CH3COONa, Na2CO3, Na3PO4, etc.


The aqueous solution of a salt of weak acid and strong base it alkaline. For the general reaction :



BA  H2O  BOH  HA
Salt Strong Weak
base acid

IONIC EQUILIBRIA
30 QUIZRR

or B  A   H2O 

  
 B  OH  HA

or A   H2O 

 HA  OH
ă

This is called anion hydrolysis.

H ydrolysis constant

The hydrolysis constant may be written as :

[HA] [OH ]
Kh  ...(i)
[A  ]

For weak acid HA, HA  H+ + Aă

[H+ ] [A  ]
Ka  ...(ii)
[HA]

Also, Kw = [H+] [OHă] ... (iii)


Multiplying equation (i) and (ii) and dividing by equation (iii), we get

K h . K a [HA] [OH ] [H+ ] [A  ] 1


   1
Kw [A  ] [HA] [H  ] [OH  ]

Kw
Kh 
Ka

The hydrolysis constant ÂKhÊ, of the salt is inversely proportional to the dissociation constant, ÂKaÊ
of the weak acid. Therefore, the weaker the acid, the greater is the hydrolysis constant of the salt.

Degree of hydrolysis

If the original concentration of the salt in the solution is ÂcÊ mol/litre and ÂhÊ is the degree of
hydrolysis at that concentration, then

A  H2O 

 
 OH  HA

Initial concentration c o o

Concentration of equilibrium c(1 ă h) ch ch

IONIC EQUILIBRIA
QUIZRR 31

The hydrolysis constant, [HA] [OH ]


Kh 
[A  ]

ch . ch ch2
= 
c (1  h) (1  h)

If ÂhÊ is very small as compared to 1, we can assume 1 ă h  1.


Kh = ch2

Kh
or h
c

Kw
Substituting the value of Kh, h 
K a .c

pH of the hydrolysed salt solution [OHă] = CH

Kw Kw
[H ]  
[OH ] ch

Kw Ka .c
Substituting the value of h, [H ] 
c Kw

K w .Ka
or [H ] 
c

Now, pH = ă log [H+]

1
 K . Ka 2
  log  w 
 c 

1
pH  [log K w  log K a  log c]
2

Now, ă log Kw = pKw ă log Ka = pKa

1 1 1
pH  pK w  pK a  log c
2 2 2

IONIC EQUILIBRIA
32 QUIZRR
(2) Salt of a Strong Acid and Weak Base

Let us take a salt, NH4Cl. It is a salt made from the reaction of strong acid, HCl and weak base,
NH4OH.

NH4 Cl (s)  NH4Cl( aq)  NH4 (aq)  Cl ( aq)

Out of NH4 and Clă, only NH 4 ion hydrolyses and let its degree of hydrolysis be h and Kh be

its hydrolysis constant. Let ÂcÊ M be the concentration of the salt, so the concentration of NH 4
would also be ÂcÊ M.

NH4 

 2H2 O  NH4 OH + H3 O
+

Initial conc. c 0 0
Conc. at equilib c(1 ă h) ch ch

[NH4 OH] [H3O+ ]


K eq 
[H2O]2 [NH4 ]

[NH4 OH] [H3 O+ ]


K eq [H2 O]2   Kh ... (i)
[NH4 ]

Multiplying numerator and denominator of equation (i) by OHă gives

[NH4 OH] [H3O+ ][OH  ]


 Kh
[NH4 ] [OH  ]

Kw [NH4 ] [OH  ]
 Kh  (since [H3O ] [OH  ]  K w and  Kb )
Kb [NH4 OH]

Kw ch  ch ch2
Kh   
Kb c (1  h) 1h

Assuming h to be smaller than 0.1, it can be ignored with respect to 1. So, 1 ă h  1.

K h  ch2

Kh Kw
h 
c Kb  c

IONIC EQUILIBRIA
QUIZRR 33

Kw Kw  c
 [H3O ]  ch  c  
Kb  c Kb

Taking log of both the sides


log [H3O+] = ólog Kw + ólog c ă ólog Kb
Multiplying both sides by ă 1,
ă log [H3O+] = ă ólog Kw ă ólog c + ólog Kb
 pH = ópKw ă ópKb ă ólog c = ó(pKw ă pKb ă log c)
(since, ă log Kw = pKw and log Kb = ă pKb)

 pH = 7 ă ópKb ă ólog c.
Since, two terms are to be subtracted from 7, so, the pH of such salt solutions would be less than
7 and will behave acidic in nature. ÂcÊ in the above expression represents concentration of the ion
undergoing hydrolysis and not the concentration of the salt.

(3) Salt of Weak Acid and a Weak Base


In this case both the cation and anion undergo hydrolysis to the same or different extents. The
resulting solution may be neutral, acidic or basic depending upon the relative strengths of acids
and bases. The hydrolysis may be written as :



CH3COONH4  H2O  CH3COOH  NH4OH
weak acid weak base

Some common examples are CH3COONH4, (NH4)2 CO3, AlPO4, etc.


The aqueous solution of a salt of strong acid and weak base is acidic. For the general reaction :



BA  H2 O  BOH  HA
Salt Weak Weak
base acid

or

B  A   H2O 

 BOH + HA

This involves anionic and cationic hydrolysis.

Hydrolysis constant
The hydrolysis constant may be written as :

[BOH] [HA]
Kh  ... (vi)
[B  ] [A  ]

IONIC EQUILIBRIA
34 QUIZRR
For weak acid,


  
HA  H A

[H+ ] [A ă ]
Ka 
[HA]

For weak base BOH


  
BOH  B  OH

[B+ ] [OH ă ]
Kb 
[BOH]

Also, Kw = [H+] [OHă]


Multiplying equation (vi) by equation [H+] [OHă], we get

[BOH] [HA] [H  ]
Kh 
[B ] [A  ] [H  ] [OH  ]

[BOH] [HA] [H ] [OH ]


Kh 
[B  ] [A  ] [H  ] [OH  ]

[BOH] [HA]
 . . [H ] [OH ]
[B ] [OH ] [H ][A  ]

Kw
Kh 
K a  Kb

Degree of hydrolysis

If the original concentration of the salt in the solution is ÂcÊ mol/litre and ÂhÊ is the degree of
hydrolysis at that concentration, then

B  A   H2O 

 BOH + HA

Initial concentration c c o o

Concentration at equilibrium  c (1 ă h) c (1 ă h) ch ch

IONIC EQUILIBRIA
QUIZRR 35

ch. ch. h2
The hydrolysis constant, K h  
c(1  h)  c (1  h) (1  h)2

If ÂhÊ is very small as compared to 1, we can assume 1 ă h  1.


Kh = h2

or h  Kh

Kw
Substituting the value of h 
K a . Kb

Here the degree of hydrolysis is independent of the concentration of the solution. The weaker the
acid and the base, the greater is the degree of hydrolysis of the salt.
pH of the hydrolysed salt solution

[H  ] [A  ]
Ka 
[HA]

[HA]
[H ]  K a 
[A ă ]

K a . ch K .h
[H ]   a
c (1  h) (1  h)

[H ]  K a . h

 Ka Kh

Substituting the value of ÂhÊ, [H+] = Ka Kh

Kw
or [H ]  K a
K a . Kb

Kw K a

Kb

Now, pH = ă log [H+]

1
 K .K  2
  log  w a 
 Kb 
IONIC EQUILIBRIA
36 QUIZRR

1 1 1
pH   log K w  log K a  log K b
2 2 2

1 1 1
pH  pK w  pK a  p K b
2 2 2

Knowing the molar concentration ÂcÊ of the solution, Ka, Kb and Kw, the pH of the solution can
be calculated. It is clear from the above equation that pH of the solution will depend upon the
pK value of the acid and the base.

If pKa < pKb, pH of the solution will be less than ópKw and consequently the solution will be
acidic.

If pKa > pKb, then pH of the solution will be more than ópKw and hence the solution will be
alkaline.

If pKa = pKa, pH of the solution will be equal to ópKw and hence the solution will be neutral.

(4) Salt of A Strong Acid and a Strong Base

Let us consider a salt, NaCl made by the reaction of strong acid, HCl and strong base, NaOH.

NaCl (s)  NaCl (aq)  Na  (aq)  Cl (aq)

Neither of the two ions of the salt undergoes hydrolysis. So, the solution contain only the equilibrium
of ionization of water.


  
2H2O (l)  H3O ( aq)  OH ( aq)

Thus, the pH of such salts always remain constant and the solution behaves neutral at pH = 7
at 25ĈC.

Example 16

Calculate the hydrolysis constant of the salt containing NO ă2 ions. Given Ka for HNO2
= 4.5 ï 10ă10.
Solution :

Kw 1014
KH   = 2.2 ï 10ă5
Ka 4.5  10 10

IONIC EQUILIBRIA
QUIZRR 37

Example 17

What is the pH of a 0.5 M aqueous NaCN solution ? pKb of CNă = 4.70.


Solution :
pKa for HCN = 14 ă 4.70 = 9.30

NaCN + H 2O  NaOH + HCN


1 0 0
1 ă h h h

KH Kw . C
 [OH  ]  C.h.  C  KH . C 
C Ka

1
or pOH = [ p K w  log C  pK a ]
2

1
= [14  0.3010  9.30]
2

= 2.5
 pH = 14 ă 2.5 = 11.5

Example 18

Calculate the percentage hydrolysis in 0.003 M aqueous solution of NaOCN. Ka for HOCN
= 3.33 ï 10ă4.
Solution :



NaOCN  H2O  NaOH  HCN

 KH   Kw  1014
h  C    
   K aC  3.33  10 4  0.003

h = 10ă4

104
 % hydrolysis = = 10ă2
100

IONIC EQUILIBRIA
38 QUIZRR
Problem 19

Calculate degree of hydrolysis and pH of 0.2 M solution of NH4Cl. Given Kb for NH4OH is
1.8 ï 10ă5.
Solution :

NH4 Cl  H2 O 

 NH4 OH  HCl

Before hydrolysis 1 0 0
After hydrolysis 1 ă h h h
where h is degree of hydrolysis

 KH   Kw  10 14
h  C     K .C   = 5.27 ï 10ă5
   b  1.8  105  0.2

From HCl, a strong acid

K  K 
 [H ]  C.h  C  H   ( K H . C)   w .C 
 C   Kb 

 1014  0.2 
    1.054  105
 1.8  105 
 

 pH = ă log [H+] = ă log 1.054 ï 10ă5


= 4.9771

Example 20

Find out the amount of NH4Cl dissolved in 500 mL to have pH = 4.5. Kb for NH4OH is
1.8 ï 10ă5
Solution :
Let conc. of NH4Cl be C mol litreă1
NH4Cl + H2O  NH4OH + HCl

Before hydrolysis 1 0 0
After hydrolysis (1 ă h) h h

 Kw   1014  C 
 
[H ]  (K H .C) =  . C   
 Kb   1.8  105 
 

IONIC EQUILIBRIA
QUIZRR 39

 1014  C 
10ă4.5 =  
 1.8  105 
 

C = 1.8 mol litreă1


 Wt. of NH4Cl = 1.8 ï 53.5 g/litreă1

1
= 1.8 ï 53.5 ï g/500 mL
2

= 48.15 g

Example 21

Calculate the degree of hydrolysis of a mixture of aniline & acetic acid each of them being
0.01 M. Ka of acetic acid = 1.8 ï 10ă5 & Kb (aniline) = 4.6 ï 10ă10. Also calculate pH of the
mixture.
Solution :
Now when we mix equal concentrations of aniline and acetic acid, they will neutralise each other
to form acetanilide (salt). The alt is of weak base & weak acid, so hydrolysis takes place. The
degree of hydrolysis of the salt of weak acid and weak base is given by :

h Kh

Kw 1  1014
 Kh  
K a K b (1.8  105 ) (4.6  10 10 )

K h = 1.21           h  1.21  1.09

Which is impossible.
Hence, as done in earlier example 15, here also we cannot use the approximate formula. Therefore,

h2
Use : Kh 
(1  h) 2

h2
 1.21 
(1  h)2

h
  K h  1.09   h = 0.523
1h

IONIC EQUILIBRIA
40 QUIZRR
However the pH of solution can be calculated by using the standard result :

1
pH  7   pK a  p K b 
2

pH  7 
1 
2 
 
 log 1.8  105   log 4.6  1010 

pH = 4.703
Note : Solution in this case is acidic. In such salts, pH depends upon which of two i.e. weak acid
or weak base is stronger than the other, i.e. by comparing Ka & Kb ; one can theoretically get an
ideal about the pH of solution.

COMMON ION EFFECT

The addition of an ionic salt having a common ion (anion or cation) to weak acids or weak bases,
suppresses their degree of dissociation (following LechatlierÊs principle).
Weak Acids :
Let HA be a weak acid (like CH3COOH, HCN etc.) & B+ Aă be the ionic salt containing common
anion (Aă) that is added to the acid.


  
HA aq  H aq  A aq


  
BA aq  Baq  A aq

Now, the solution has excess of Aă ions. This means increasing concentration of products, in an
equilibrium state. (Following LechatlierÊs principle) the reaction (HA  H+ + Aă) must go in
backward direction, in order to mullity the effect of added Aă ions. As a consequence, amount of
H+ in new equilibrium state will be less than before, or one can see that the degree of dissociation
of acid (HA) is decreased.

Quantitative Aspect :

HA  H+ + Aă ; its degree of dissociation =  is given by :

Ka

c

Let M molar BA be added to it & ´ be its new degree of dissociation


BA  Baq + A aq

M M (Assuming BA to be 100% ionised)

IONIC EQUILIBRIA
QUIZRR 41

HA  H+ + Aă

c ă c´ c´ c´

Now in solution : [HA] = c ă c´ ; [H+] = c´


[Aă] = [Aă]HA + [Aă]BA = c´ + M

[H  ][A  ] (c´) ( c´  M) ´( c´  M)


Ka   
[HA] (c ă ca´) 1 ă ´

 Ka = c´2 + M´ (as 1 ă ´  1)

 Ka = M´
(neglecting C´2 in comparison to M´ as ´ << 1 & ´2 <<< 1]

Ka
 ´ 
E

Note :
One similar lines you can find Ê for a weak base, BOH and adding B+ ions to it.

BUFFERS

Certain solutions, such as that of ammonium acetate, have a tendency to resist any change in its
hydronium ion concentration or pH, whenever a small amount of a strong acid or a strong base
is added to it. This property of a solution is known as buffer action.

Buffer Solution

A solution which resists any change of pH when a small amount of a strong acid or a strong base
is added to it, is called a buffer solution or simply as a buffer.
Alternatively, a buffer solution may be defined as a solution whose pH value does not change
appreciatbly upon the addition of small amounts of a strong acid, base and/or water from outside.
Thus, buffers have reserve acidity and reserve alkalinity.
Buffer solutions usually consist of a mixture of a weak acid and its salt with a strong base e.g.,
CH3COOH and CH3COONa, or that of a weak base and its salt with a strong acid e.g., NH4OH
and NH4Cl. The solution of any salt of a weak acid and weak base e.g., ammonium acetate, also
shows buffering property.

Types of Buffers

There are two types of buffers, acid buffer and basic buffer.

IONIC EQUILIBRIA
42 QUIZRR
Acid buffer

A buffer solution containing a large amounts of a weak acid, and its salt with a strong base, is
termed as an acid buffer. Such buffer solutions have pH on the acidic side i.e., pH is less than
7 at 298 K. The pH of an acid buffer is given by the equation.

[salt]
pH  pK a  log
[acid]

where Ka is the acid dissociation constant of the weak acid.

Basic buffer

A buffer solution containing relatively large amounts of a weak base and its salt with a strong
acid, it termed as a basic buffer. Such buffers have pH on the alkaline side i.e., pH is higher than
7 at 298 K.
The pH of a basic buffer is given by the equation :

[salt]
pOH  pK b  log
[base]

where Kb is the base dissociation constant of the weak base.


These equations are called Henderson equation.

Buffer-capacity and Buffer-range

The effectiveness of any buffer is described in terms of its buffer capacity. It is defined as, Âthe
number of equivalents of a strong acid (or a strong base) required to change the pH of one litre
of a buffer solution by one unit, keeping the total amount of the acid and the salt in the buffer
constantÊ.
The buffer capacity of a buffer in maximum when acid to salt or base to salt ratio is equal to 1
i.e. it contains equal number of moles of acid (or base) and the salt. All buffer solutions remain
effective over a small pH range : this pH-range is characteristic of the buffer and is termed as
the buffer-range.
For the two types of buffers, it is given by
Buffer range in pH units
Acid buffer : pKa ă 1 to pKa + 1
Basic buffer : (pKw ă pKb) ă 1 to (pKb) ă pKb) + 1
Consider acetic acid ă sodium acetate buffer, an acid buffer. the acid dissociation constant (Ka)
of acetic acid is 1.84 ï 10ă5.
Therefore, pKa for acetic acid is 4.74. Then, the buffer range of an acetic acid - sodium acetate

IONIC EQUILIBRIA
QUIZRR 43

buffer is,
pH = (pKa) ă 1 to (pKa) + 1
= 4.74 ă 1 to 4.74 + 1
= 3.74 to 5.74
Thus, the acetic acid - sodium acetate buffer will act as an effective buffer over the pH range 3.74
to 5.74.
The pH of a buffer solution depends only on the ratio of the concentrations of the salt and the
acid, or salt and the base. It does not depend on the individual concentration. Since, the ratio
remains the same even when the solution is diluted. However, at high dilutions, buffers do not
remain effective as, they are not able to resist a change in the pH value due to the addition of
a strong acid or a strong base.

Example 22

Calculate the pH of a buffer solution containing 0.2 mole of NH4Cl and 0.1 mole of NH4OH
per litre. Kb for NH4OH = 1.85 ï 10ă5.
Solution :
According to HendersonÊs equation :

[salt]
pOH = pKb + log
[base]

pKb = ă log Kb = ă log 1.85 ï 1ă5 = 4.733

0.2
pOH = 4.733 + log
0.1

= 4.733 + 0.301 = 5.034


pH = 14 ă pOH = 14 ă 5.034 = 8.966

Example 23

A buffer solution contains 0.04 mole of Na2HPO4 and 0.02 mole of NaH2PO4 per litre. (a)
Calculate pH if pKa of NaH2PO4 is 6.84. (b) If 1 ml of 1 N HCl is added to a litre of the buffer,
calculate the change in pH.
Solution :
In this mixture NaH2PO4 is the acid and Na2HPO4 is the salt.

[Salt] 0.04
pH = pKa + log = 6.84 + log = 6.84 + log 2 = 6.84 + 0.30 = 7.14
[Acid] 0.02

IONIC EQUILIBRIA
44 QUIZRR
(b) 1 ml of 1 N HCl contains 0.001 mole of H+. It will convert 0.001 mole of Na2HPO4 to 0.001
mole of NaH2PO4.

0.04  0.001 0.039


pH  pK a  log  6.84  log  7.109
0.02  0.001 0.021

The change in pH is 0.031.

Example 24

Calculate the pH of a solution made by adding 0.001 mole of NaOH to 100 ml of 0.5 M acetic
acid and 0.5 M sodium acetate solution.
Solution :

0.5
Moles of acetic acid in 100 ml of solution =  100  0.05
1000

0.5
Moles of sodium acetate in 100 ml of solution =  100  0.05
1000

When NaOH is added, acetic acid is converted into sodium acetate.


Moles of acetic acid after addition of 0.001 mole of NaOH = (0.05 ă 0.001) = 0.049
Moles of sodium acetate after addition of 0.001 mole of NaOH = (0.05 + 0.001)
= 0.051

[Salt] 0.051  10
pH  pK a  log   log (1.8  105 )  log
[Acid] 0.049  10

= 4.7447 + 0.0174 = 4.7621.

Example 25

20 ml of 0.2 M NaOH is added to 50 ml of 0.2 M acetic acid to give 70 ml of the solution. What
is the pH of the solution ? Calculate the additional volume of 0.2 M NaOH required to make
pH of the solution 4.74. The ionisation constant of acetic acid is 1.8 ï 10ă5.
Solution :
From the chemical reaction,

CH3COOH  NaOH  CH3COONa  H2O


20 ml 20 ml 20 ml

IONIC EQUILIBRIA
QUIZRR 45

It is evident that 70 ml of the product will contain


(i) 30 ml of 0.2 M unused CH3COOH
(ii) 20 ml of CH3COONa.

0.2
 No. of moles of CH3COOH in 70 ml of solution =  30  0.006 mole
1000

0.2
Similarly, No. of moles of CH3COONa in 70 ml of solution =  20  0.004 moles
1000

[Salt]
pH   log K a  log
[Acid]

Substituting the values of the various terms

0.004
pH   log 1.8  105  log
0.006

= 4.7447 ă 0.1761 = 4.5686


Calculation of the additional volume of 0.2 M NaOH required to make pH of solution 4.74.

[Salt]  Salt 
pH   log K a  log or 4.74   log 1.8  105  log  
[Acid]  Acid 

 Salt 
4.74  4.7447  log 
 Acid 
or

 Salt   Salt  1
 log   0.0047  Acid   1.011
 Acid 
or

Let x ml be the volume of additional 0.2 M NaOH added to make the pH of the solution 4.74. This
will further neutralise x ml of 0.2 M CH3COOH and produce x ml of 0.2 M sodium acetate. The
resulting solution (70 + x) will now contain
(i) (30 ă x) ml of 0.2 M acetic acid.
(ii) (20 + x) ml of 0.2 M sod. acetate.

Number of moles of acetic acid in (70 + x) ml solution  0.2  (30  x)  2  104 (30  x)
1000

0.2
Number of moles of CH3COONa in (70 + x) ml solution   (20  x)  2  10 4 (20  x)
1000

IONIC EQUILIBRIA
46 QUIZRR

[Salt] 2  104 (20  x) 20  x


Therefore,  
[Acid] 2  104 (30  x) 30  x

1 20  x

1.011 30  x

or 20.22 + 1.011 x = 30 ă x
2.011 x = 9.78 or x = 4.86
Therefore, the additional volume of 0.2 M NaOH required to make the pH of the solution 4.74
is 4.86 ml.

Example 26

500 mL of 0.2 M queous solution of acetic acid is mixed with 500 mL of 0.2 M HCl at 25ĈC.
(i) Calculate the degree of dissociation of acetic acid in the resulting solution and
pH of the solution.
(ii) If 6 g of NaOH is added to the above solution, determine the final pH. [Assume
there is no change in volume on mixing; Ka of acetic acid is 1.75 ï 10ă5 mol Lă1].
Solution :
Concentration of CH3COOH in 500 ml = 0.2 M
Concentration of HCl in 500 ml = 0.2 M
After mixing, [CH3COOH] = 0.1 M; [HCl] = 0.1 M
HCl being a strong electrolyte is completely ionised, hence concentration of H+ furnished by HCl
= 0.1 M
CH3COOH, being weak electrolyte, dissociates as below.

CH3COOH 

 CH3COOă + H+

At start c 0 0
At equilibrium c (1 ă ) c c + 0.1

[CH3COO ] [H  ] c ( c  0.1) c2  0.1 


 Ka   
[CH3COOH] c (1  ) (1  )

Since  is very very small, c2 can be neglected and 1 ă  can be taken as unity
 Ka = 0.1 

IONIC EQUILIBRIA
QUIZRR 47

Ka
or  =
0.1

1.75  105
=  1.75  104
0.1

Calculation of pH of the solution


[H+]total = 0.1 + c
since ca is very very small, it can be neglected
 [H+]total = 0.1
 pH = 1

6
(ii) No. of moles of NaOH added = = 0.15
40

No. of moles of NaOH left after reacting with 0.l mole of HCl = 0.15 ă 0.1 = 0.05 M
This 0.05 moles of NaOH will react with CH3COOH in the following way.

CH3COOH + NaOH  CH3COONa + H 2O

At start 0.1 mol 0.05 mol 0 0


At finish 0.05 mol 0 mol 0.05 mol 0.05 mol
Thus solution of CH3COOH and CH3COONa acts as an acid buffer. Hence in such cases

[Salt]
pH  pK a  log   log (1.75  105 )  log 1
[Acid]

= 4.75

Example 27

Two buffers, (X) and (Y) of pH 4.0 and 6.0 respectively are prepared from acid HA and the
salt NaH. Both the buffers are 0.50 M in HA. What would be the pH of the solution obtained
by m i x i n g equ al v ol u m es of t h e t w o bu f f er s ? (K HA = 1.0 ï 10ă5)
Solution :
Determination of concentration of salt, i.e. [Salt] in buffer X.

[Salt]
pH = pK  log
[Acid]

[Salt]
4.0 = 5.0  log
0.5

IONIC EQUILIBRIA
48 QUIZRR

[Salt]
log = ă 1
0.5

[Salt] 1
=
0.5 10

0.5
 [Salt = = 0.05 M
10

Determination of [Salt] in buffer Y

6.0 = 5.0 + log [Salt]


0.5

[Salt]
log = 1.0
0.5

[Salt] = 10 ï 0.5 = 5.0 M

0.05  5.0
 [Salt]mix =
2

= 2.525 M
Substituting the values in the relation

[Salt]
pH = pK + log
[Acid]

2.525
= 5.0 +
0.5

= 5.0 + log 5.05


= 5.0 + 0.70 = 5.7

Indicators

Indicators are substances, which indicate the completion (equivalence point or end point) of a
chemical reaction by change in colour. For example, in volumetric analysis, during the titration
of sodium hydroxide and hydrochloric acid (taken in the burette), phenolphthalein turns pink to
colourless when the whole of sodium hydroxide has been neutralized by hydrochloric acid. All
indicators show change in colour over some pH range, which varies considerably from one indicator
to another.

IONIC EQUILIBRIA
QUIZRR 49

Acid-Base Indicators

The amount of an acid (or a base), which is exactly equivalent chemically to the amount of some
standard base (or an acid), is determined by an acid-base titration. The point of equivalence is
called end point. The solution of a strong acid and strong base will be neutral at the end point
and have a pH of 7 as they are strong electrolytes. However, if either the acid or the base is a
weak electrolyte, the solution at the equivalence point will be either slightly alkaline (pH>7) or
slightly acidic (pH<7). Hence, for an acid-base titration, the correct end point will be characterized
by a definite value of H+ ion concentration (i.e. pH), which depends upon the nature of the acid
and the base and the concentration in the solution.
A large number of acid-base indicators are available which possess different colours according to
H+ ion concentration of the solution. These indicators do not change from predominantly ÂacidÊ
colour to predominantly ÂbaseÊ suddently and abruptly, but within a small interval of pH (generally
about 2 pH units). This is called colour-charge interval of the indicator. The position of the colour-
charge interval in the pH scale varies widely with different indicators. Therefore, for acid base
titrations, an indicator is chosen which shoes a distinct colour change at a pH close to that of
equivalent point.

Acid-base indicator behaviour


The mechanism of acid-base indicator was developed by Ostwald, which offered an explanation
for the colour change with change in pH. All acid-base indicators are weak organic acids or bases,
which posses different colours in unionized and ionized states. If the indicator acid is represented
as HIn and has its ionized form in, then equilibrium in aqueous solution may be represented as :


  ă
HIn  H  In
Unionized Ionized
colour colour

For a basic indicator,


  
InOH  OH  In
Unionised Ionised
colour colour

The unionized molecule has one colour while the ionized ions has another colour.
The equilibrium constant for a weak acid indicator may be written as :

[H  ] [In  ]
K Ind 
[HIn]

where, KInd is known as indicator constant.

[HIn]
or [H ]   K Ind
[In  ]

IONIC EQUILIBRIA
50 QUIZRR
Writing the above equation in logarithmic form

[HIn]
log [H  ]  log  KInd
[In  ]

[In + ]
or  log [H ]  ă log KInd + log (  pX = ă log X)
[HIn]

Thus the colour of the indicator, which is determined by the ratio

[In ă ]
is governed by the pH of the solution,
[HIn]

Similarly, for a weak organic base indicator, In OH,

[InOH]
[OH ]   KInd
[In  ]

Kw [In  ]
or [H ]  
K Ind [InOH]

Taking logarithms

Kw [In  ]
log [H  ]  log  log
K Ind [InOH]

[InOH]
or  log [H ]   log Kw  log K In  log
[In  ]

[InOH]
 pH  pKw  p K In  log
[In  ]

[InOH]
or pH  14  p K In  log at 25C
[In  ]

As the pH of the solution changes the colour of the indictor changes. This change is so minute
that it goes unnoticed by thehuman eye. therefore to see the colour change the concentration of
one of the forms should predominate. The solution will have the acid colour (i.e. of HIn form)
when the ratio [HIn] to [In] is approximately 10 and the alkaline colour (i.e. of Ină) when ratio
[Ină] to [HIn] is approximately 10.

IONIC EQUILIBRIA
QUIZRR 51

The acid colour will be visible when

[HIn]
 10
[In  ]

Only basic colour will be visible when

[In  ]
 10
[HIn]

Hence the colour interval or indicator pH range is

Indicator pH range = ă 1 + pKInd to 1 + pKInd


Indicator pH range = pKInd  1

This means that the colour change interval of an indictor lies over approximately 2 pH units.
Within this range, the change from one colour to the other another will be visible to human eye.

Acid Base Titration Curves

In acid base titrations, a base is added to an acid successively (or vice versa) and the pH of the
solution is noted after every addition. the plot of pH against the amount of base (or acid) added
is called a titration curve. The shape of the curve depends upon the ionization constants of the
acid and base used in the titration.

(1) Titration of strong acid and strong base

When we add a solution of NaOH to the solution of HCl, the pH progressively increases.

HCl( aq)  NaOH( aq)  NaCl (aq)  H2 O(l)

This is because OHă ions from the base will react with H+ ions of the acid to form water. This
decreases the concentration of H+ ions and therefore pH increases. The pH of the solution increases
only slightly in the beginning until just before the stoichiometric point, there is a sudden jump
in pH. The pH continues to increase after the stoichiometric point but then levels off because of
the presence of excess strong base in the solution. At the equivalence point the pH is 7, but has
increased sharply from 3.5 to 11 just before this point. Any indicator, which has effective range
between pH 3.5 to 11 may be used to detect the equivalence point. For example, phenolphthalein
and methyl orange are good indicators in this range.

IONIC EQUILIBRIA
52 QUIZRR
When we tirtate the base (NaOH) with the continuous addition of acid (HCl), the curve obtained
is exactly reversed. The pH decreases slowly in the beginning and near the equivalence point
there is sudden fall through pH = 7 as the concentration of H+ ions increases sharply.

14 14

12 12

10 10

8 8
Equivalence pH Equivalence
6 point 6 point
pH
4 4

2 2

0 10 20 25 30 40 50 0
Volume NaOH added Volume of HCl added (ml)
Tiration curve of strong acid Tiration curve of strong base
(HCl) with a strong base (NaOH) (NaOH) with strong acid (HCl)

(2) Titration of weak acid and a strong base

This type of titration is carried out between acetic acid and sodium hydroxide.

CH3 COOH( aq)  NaOH (aq)  CH3 COONa ( aq)  H2 O (l)

The free H+ ion from the weak acid is neutralized by OHă ions from the base and there is a small
increase in pH. Around the equivalence point large increase in pH is observed. However, the slope
of the curve is not as sleep as in the case of strong base and strong acid titration curve. Sodium
acetate formed after neutralization reacts with 14
OHă ions of water (hydrolysis) and so at the
12
equivalence point the pH is not 7. The H+ ion
concentration decreases and the pH is about 8.7 10
at the equivalence point. By the addition of small 8
amount of NaOH, pH change is observed from pH Equivalence
6 point
about 8.0 to 10.0 near the equivalence point. It is
therefore necessary to use an indicator with pH 4
range on alkaline side. Phenolphthalein is
2
commonly used. Methyl orange cannot be used
because it would give inaccurate indication of 0 5 15 25 35 45 55
Volume NaOH added (ml)
equivalence point.
The pH titration curve of weak acid (CH3COOH) and
strong base (NaOH)

IONIC EQUILIBRIA
QUIZRR 53

(3) Titration of weak base and a strong acid

This type of titration is carried out between a weak base such as ammonium hydroxide and strong
acid such as hydrochloric acid.

NH4 OH( aq)  HCl (aq)  NH4 Cl ( aq)  H2 O(l)

The equivalence point is below 7 because the salt (NH4Cl) formed at the neutralization reacts with
water to give H+ ions. The equivalence point lies at about pH 5.3. It is, therefore necessary to use
an indicator with pH range slightly on the acidic side. Methyl orange can be used. Phenolphthalein
is not suitable because its colour change occurs away from the equivalence point.

14

12

10

8
pH
6
Equivalence
4 point

0 5 15 25 35 45 55
Volume NaOH added (ml)

The pH titration curve of weak base (NH4OH) and


strong acid (HCl)

(4) Titration of weak acid and weak base

This type of titration is carried out between a weak acid such as acetic acid (CH3COOH) and weak
base such as ammonium hydroxide (NH4OH).

NH4 OH( aq)  CH3 COOH( aq)  CH3 COONH4( aq)  H2 O(l)

There is no sharp change in the pH during the titration. Hence, no sharp equivalence point can
be obtained with common indicators. However, a mixed indicator, which shows a sharp colour
change over limited pH range may be used.

IONIC EQUILIBRIA
54 QUIZRR
14

12

10

8
pH
6

0 5 15 25 35 45 55
Volume NH3 added (ml)

Titration curve of weak base (NH4OH) and


weak acid (CH3COOH)

SOLUBILITY, MOLAR SOLUBILITY AND SOLUBILITY PRODUCT


There are two ways to express a substanceÊs solubility (a) solubility and (b) molar solubility.
(a) The number of grams os solute in one litre of a saturated solution (g/litre) is termed as
solubility.
(b) The number of moles of solute in one litre of a saturated solution (moles / litre) is defined
as molar solubility.
Both these definitions refer to the concentration of saturated solutions at some given temperature
(usually 25Ĉ).
Let us consider solubility of any general salt, AxBy. Let the solubility of this salt be ÂsÊ mol per litre.


 y x
A x B y (s)  xA (aq)  yB ( aq)

xs ys
For saturated solution (at equilibrium),

[A y ] x  [B x ] y
K eq 
[A xB y ]

K eq [A xB y ]  [A y ]x [B x ] y

K sp  [A y ] x [B x ] y (since AxBy is a pure solid)

IONIC EQUILIBRIA
QUIZRR 55

where Ksp is called solubility product. The term on right hand side is called solubility product
if the solution is saturated and ionic product if the solution is not saturated (i.e., not at equilibrium).

K sp  ( xs) x  ( yx) y

K sp  x x  y y  s( x  y)

It should be noted that if the ionic product is more than KSP, the equilibrium will shift in
backward direction and some amount of salt is thrown out of solution (precipitation) and finally
an equilibrium is developed in such a way that the ionic product becomes equal to solubility
product. If ionic product is less than KSP, the solution is unsaturated and more of the salt can
be dissolved. If ionic product is equal to KSP, the solution is saturated and if ionic product is more
than KSP, the solution is called super saturated.
From the solubility product principle, we may infer that,

Ć When the ionic product of a salt in solution is equal to its solubility product, the solution is
saturated and the undissociated salt remains in equilibrium with its ions in the solution.

Ć The solubility of a salt in water depends upon other substances (ions) present is water (at
that time) and the solubility product is a constant value (at a given temperature) and is
independent of presence of any other ion present. Like other equilibrium constants such as :
Kp, Kc, Ka & Kb, it is also constant at a certain temperature & can change only when
temperature changes.

Ć The salts like AgCl are regarded as sparingly soluble salts. Some of them are : Ag2SO4,
Ag2CO3, PbCl2, PbCrO4, PbS, ZnS, Al(OH)3, Ca3(PO4)2, BaSO4, CaSO4, MnS etc.

Ksp for Ag2SO4 : Ag 2SO4  2Ag   SO24 

K sp  [2Ag  ] 2 [SO 42  ]

Ksp for Ca3(PO4)2 Ca3 (PO4 )2  3Ca 2   2PO43 

K sp  [Ca 2 ]3 [PO34 ]2

In solubilities of sparingly soluble salts, we will deal with mainly two types of problems : one to
find out soloubility in water if Ksp of a salt is given & vice-versa. Secondly, to find out solubility
of a salt in presence of other (common) ions in water.

IONIC EQUILIBRIA
56 QUIZRR
Comparison of solubility product and ionic product

Solubility product Ionic product


It is the product of the concentration of ions of It is the product of the concentration of ions of
the electrolyte each raised to the power of their the electrolyte each raised to the power of their
coefficients in the balanced chemical equation in coefficients in the balanced chemical equation in
a saturated solution a solution at any concentration.
It is applicable to only saturated solutions It is applicable to all types of solutions of any
concentration

It has a constant value for an electrolyte at a Its value is not constant and varies with change
constant temperature in concentration

Example 28

Ksp of calcium iodate Ca(IO3)3 = 6.3 ï 10ă7 at 18ĈC, find the solubility of salt (in gm/L) in
water at 18Ĉ C.
Solution :
Note : Ksp is expressed in concentration units.
In questions, solubility is generally asked (or given) to be expressed in gm/lt.
Now the following equilibrium is established.

Ca(IO3 )2  Ca 2   2IO3

Let x mole/lt be solubility of Ca(IO3)2


 [Ca 2  ]  xM & [IO 3 ]  2 xM

K sp  [Ca 2  ] [IO3 ]2

 6.3 ï 10ă7 = (x) (2x)2

 4x3 = 6.3 ï 10ă7

 x = 5.4 ï 10ă3 M
Now answer is to be expressed in gm/lt.
 solubility = x ï M0
= (5.4 ï 10ă3) ï 390 = 2.106 g/lt

IONIC EQUILIBRIA
QUIZRR 57

Example 29
The solubility of BaSO4 in water is 2.3 ï 10ă4 gm/100 mL. Calculate the % age loss in weight
when 0.2 gm of BaSO4 is washed with (a) 1 lt of water (b) 1 lt of 0.01 N H2SO4.
Solution :
(a) Solubility is in general expressed in gm/lt,
so solubility of BaSO4 = 2.3 ï 10ă3 g/lt.
Loss in weight of BaSO4 = amount of BaSO4 soluble.

2.3  10 3
 % loss   100  1.15%
0.2

2
(b) Now 0.01 N H2SO4  0.01 N SO4 ions

2
 0.005 N SO4 ions

2
Now presence of SO4 prior to washing BaSO4 will suppress the solubility of BaSO4 (due to

common ion effect). The suppression will be governed by Ksp value of BaSO4. So first calculate
Ksp of BaSO4.
Solubility of BaSO4 in fresh water = 2 ï 10ă3 g/lt

2.3  103
  Mol/lt  9.85  106 M
233.4

Ksp  [Ba 2  ] [SO42  ]

= (9.85 ï 10ă6)2 = 9.71 ï 10ă11


Now let x be solubility in mol/lt
[Ba2+] in solution = x mol/lt

& [SO24  ] in solution = (x + 0.005) mol/lt

Ionic product = [Ba2+] [SO24  ] = (x) (x + 0.005)

Ksp = Ionic Product at equilibrium (saturation)


 9.11 ï 10ă11 = (x) (x + 0.005)

 x2 + 0.005 x ă 9.71 ï 10ă11 = 0

New neglecting x2 (as x << 1)

IONIC EQUILIBRIA
58 QUIZRR

9.71  1011
 x = 1.94 ï 10ă8 mol /lt
0.005

= 1.94 ï 10ă8 ï 233.4 g/lt


= 4.53 ï 10ă8 g/lt
 4.53 ï 10ă6 gm of BaSO4 are washed away.

4.53 ï 10 ă6
 % loss = = 2.26 ï 10ă3 %
0.2

Precipitation of Salts :

For a salt (sparingly soluble) when dissolved in water : AB  A   B

at equilibrium (saturation) K sp  [A  ]  [B ]

When we mix ions or if there be two or more ions in water, we define reaction coefficient (Q),
called as ionic-product (IP) in this case, giving the products of ions in water (ions of soluble salts
& other common ions).

IP is product of ionic concentration due to ions already present in water or from a salt. It may
be & may not be equal to Ksp.

1. If Ionic Product (IP) > Ksp; precipitation takes place till I.P. equals Ksp.

2. If Ionic Product < Ksp ; a precipitate will not be formed and the solution will be unsaturated.

3. If Ionic Product = Ksp ; a precipitate will not form and the solution is saturated in that salt.

(Or we can say that solution is at a critical stage, when precipitation just begins, but actually
has not occurred yet in real sense).

Example 30

A solution contains a mixture of Ag+(0.1M) and Hg 2+


2 (0.l0 M) which are separated by
selected precipitation. Calculate maximum concentration of Iodide ion at which one of
them gets precipitated almost completely. What % age of that metal ion is precipitated ?
Solution :
First determine, which ion starts precipitating.

K sp of AgI
[Iă] required to precipitate AgI =
[Ag  ]

IONIC EQUILIBRIA
QUIZRR 59

8.5  1017
  8.5  1016 M
0.1

K sp of Hg 2 I2
[I ] for Hg 2I 2 
[Hg 2+
2 ]

2.5  1026
  5  10 13
0.1

This means that AgI will be precipitated first as [Iă] required for AgI is less.
But when [Iă] reaches 5 ï 10ă13 M then precipitation of Hg2I2 also starts.
So [Ag+] left at that stage is given as :

K sp of AgI 8.5  1017


[Ag  ]    1.7  104 M
ă 13
[I ]Hg I 5.0  10
2 2

% of Ag+ left unprecipitated

1.7  104
  100  0.17%
0.1

 % age of Ag+ = 99.83 %


This means when Hg2I2 starts precipitating, Ag+ is almost precipitated.

Example 31

Freshly precipitated Al & Mg Hydroxides are stirred vigorously in a buffer solution


containing 0.25 mol/lt of NH4Cl & 0.05 mol/lt of NH4OH. Calculate [Al3+] & [Mg2+] in solution.
Kb (NH4OH) = 1.8 ï 10ă5; Ksp (Mg(OH)2) = 8.9 ï 10ă12; Ksp (Al(OH)3) = 6 ï 10ă32.
Solution :
Note that Al (OH)3 & Mg (OH)2 are mixed with buffer which is basic. i.e. [OHă] are already
present in the solution so it will decrease the solubility of Al (OH)3 & Mg(OH)2 in the solution.
At equilibrium, [OHă] will determine the concentration of metal ions in solution.
First calculate [OHă] in the solution using basic buffer equation.

[salt]
pOH = pK b  log
[base]

IONIC EQUILIBRIA
60 QUIZRR

 0.25 
  log (1.8 ï 10 ă5 )  log  
 0.05 

pOH = 5.4431 = ă log [OHă]


 [OHă] = 3.59 ï 10ă6 M
Now this controls the following equilibria.
1. Mg(OH)2; Ksp = [Mg2+] [OHă]

8.9  10 12
 [Mg 2  ]   0.69 M
 3.59  106 
2. Al(OH)3 ; Ksp = [Al3+] [OHă]3

6  1032
 [Al3  ]   1.29  1015 M
3.59  10  6

Example 32

The solubility product of CaF2 is 3.9 ï 10ă11 at 25ĈC. What is the solubility (in grams) of this
salt in a litre of 0.1 N NaF solution ?
Solution :
Let the solubility of CaF2 in the presence of 0.1 N NaF be x.


 2 
CaF2  Ca  2F

[Ca2+] = x
[Fă] = 2x + 0.1
Ksp = [Ca2+] [Fă]2 = x (2x + 0.1]2 = 4x3 + 0.4x2 + 0.01 x
Since x is small, x2 and x3 terms would be very-very small and can be neglected.
 0.01 x = 3.9 ï 10ă11

3.9  10 11
x  3.9  10 9
0.01

The smalleness of x justifies the earlier supposition that x3 and x2 is negligible.


Molecular weight of CaF2 = (40 + 38) = 78
Mass of CaF2 per litre = 3.9 ï 10ă9 ï 78 = 3.042 ï 10ă7 g litreă1.

IONIC EQUILIBRIA
QUIZRR 61

Example 33

The solubility product (Ksp) of Ca(OH)2 at 25ĈC is 4.42 ï 10ă5. A 500 ml of saturated solution
of Ca(OH)2 is mixed with equal volume of 0.4 M NaOH. How much Ca(OH). How much
Ca(OH)2 in miligrams is precipitated ? [IIT 1992]
Solution :
Let the solubility of Ca (OH)2 in pure water = S moles / litre

Ca(OH)2  Ca 2+  2OH ă
s 2s

Then Ksp = [Ca2+] [OHă]2


4.42 ï 10ă5 = S ï (2S)2
4.42 ï 10ă5 = 4S3
S = 2.224 ï 10ă2 = 0.0223 moles litreă1
 No. of moles of Ca2+ ions in 500 ml of solution = 0.01115
Now when 500 ml of saturated solution is mixed with 500 ml of 0.4 M NaOH, the resultant
volume is 1000 ml. The molarity of OHă ions in the resultant solution would therefore be 0.2 M.

K sp 4.42  105
 [Ca 2+ ]    0.001105 M
[OH ]2 (0.2)2

Thus, No. of moles of Ca2+ or Ca(OH)2 precipitated = 0.01115 ă 0.001105 = 0.010045


Mass of Ca(OH)2 precipitated = 0.010045 ï 74 = 0.7433 g = 743.3 mg

Example 34

Given : Ag(NH3 )+2  Ag+ + 2NH3, Kc = 6.2 ï 10ă8 and Ksp of AgCl = 1.8 ï 10ă10 at 298 K. Calculate

the concentration of the complex in 1.0 M aqueous ammonia.


Solution :

For AgCl 


  
AgCl  Ag  Cl

 Ksp = [Ag+] [Că] ... (i)


Again it is given that

[Ag(NH3 ) 2 ] 

 
 Ag  2NH3 ; K c  6.2  10
8

 1 108
or Ag + + 2NH3 

 [Ag(NH3 ) 2 ] ; K f  
6.2  108 6.2

IONIC EQUILIBRIA
62 QUIZRR

[Ag(NH3 )2 ]
 Kf 
[Ag + ][NH3 ]

[Ag(NH3 )2 ]
or [Ag + ] 
K f [NH3 ]2

Since the formation constant of the complex is very high, most of the [Ag+] which dissolves
must be converted into complex and each Ag+ dissolved also requires dissolution of Clă.
 [Clă] = [Ag(NH3)2]+ and let it be c M
Equation (i) becomes

[Ag(NH3 )2 ]
Ksp =
K f [NH3 ]2

c
Ksp = c
K f [1]2

c2 = Ksp ï Kf [1]2

08
= 1.8  1010   (1)2
6.2

1.8  102
c2 = = 0.2903 ï 10ă2
6.2

c = 0.538 ï 10ă1 = 0.0538 M

Example 35

Predict whether a precipitate will be formed or not when equal volumes of 2 ï 10ă6 M BaCl2
solution and 2 ï 10ă5 M Na2SO4 solution are mixed ? The solubility product of barium
sulphate is 1 ï 10ă10.
Solution :

2
Calculation of concentrations of Ba2+ and SO4 ions before mixing.


 2 
BaCl2  Ba  2Cl

 [Ba2+] = 2 ï 10ă6 M

IONIC EQUILIBRIA
QUIZRR 63

Similarly, Na2SO4  2Na+ + SO4 


2

  SO2    2  105 M
 4 

Concentration of individual ions in the total solution which is doubled after mixing.

2  106 SO2    2  10
5
[Ba 2  ]   106 M or  105 M
2  4  2

Alternatively,
Let the volume before mixing = x litre
 The volume after mixing equal volumes of the two salts = x + x = 2x litres
Further let the molarity of the resulting solution = M2
Now applying molarity formula to Ba2+ ions.

M1 V1  M2I2
(Before mixing) (After mixing)

2 ï 10ă6 ï x = M2 ï 2x

2  106  x 2  106
M2    106
2x 2

Similarly, for SO4  ions


2

M1 V1  M2I2
(Before mixing) (After mixing)

2 ï 10ă5 ï x = M2 ï 2x

2  105  x 2  105
M2    105
2x 2

Calculation of ionic product of Ba2+ and SO4  ions


2

 Ionic product of BaSO4 = 10ă6 ï 10ă5 = 10ă11


Since here Ionic product < Solubility product
10ă11 < 10ă10
No precipitate will be formed.

IONIC EQUILIBRIA
CHEMICAL
EQUILIBRIUM
QUIZRR 3

CH EM I CA L EQU I LI B RIU M

Re v e r s i b l e Re a c t i o n s a n d Dy n a m i c Eq u i l i b r iu m
A reaction is said to be reversible if the composition of the reaction mixture on the approach of
equilibrium at a given temperature is the same irrespective of the initial state of the system, i.e.,
irrespective of the fact whether we start with the reactants or the products. Some examples of reversible
reaction are listed below:

A + B 

 C + D

H2(g) + I2(g) 



 2HI(g)

3Fe(s) + 4H2O(g) 



 Fe3O4(s) + 4H2(g)

Ag+(aq) + Fe2+(aq) 



 Fe3+(aq) + Ag(s)
It is an experimental fact that most of the processes including chemical reactions, when carried
out in a closed vessel, do not go to completion. Under these conditions, a process starts by itself or by
initiation, continues for some time at diminishing rate and ultimately appears to stop. The reactants
may still be present but they do not appear to change into products any more.

Ch a r a c t e ri s t ic s o f Ch e m i c a l Eq u i li b r i u m :
ă Chemical equilibrium, at a given temperature, is characterised by constancy of certain
observable properties such as pressure, conc., density.
ă Chemical equilibrium can be approached from either side.
ă Chemical equilibrium is dynamic in nature.
ă A catalyst does not alter the position of equilibrium. It accelerates both the forward and
reverse reactions.

T y p e s o f Eq u i l ib r i a
There are mainly two types of equilibria
(a) Homogeneous : Equilibrium is said to be homogeneous if reactants and products are in
same phase.

H2(g) + I2(g) 



 2HI(g)

N2(g) + 3H2(g) 



 2NH3(g)

N2O4(g) 

 2NO2(g)

CH3COOH(l) + C2H5OH(l) 



 CH3COOC2H5(l) + H2O(l)

CHEMICAL EQUILIBRIUM
4 QUIZRR
(b) Heterogeneous : Equilibrium is said to be heterogeneous if reactants and products are in
different phases.

CaCO3(s) 

 CaO(s) + CO2(g)

NH4HS(s) 

 NH3(g) + H2S(g)

NH2CO2NH4(s) 

 2NH3(g) + CO2(g)

Ra t e o f a Re a c t io n
The rate of a reaction is defined as the decrease in concentration per unit time of the reactants
or the increase in concentration per unit time of the products.
For example, consider
A + 2B  3C
In the atomic reaction, according to the stoichiometric coefficients, 2 moles of B disappear for every
mole of A reacting and 3 moles of C are formed for every mole of A disappears.

d[A]  1 d[B] 1 d [C]


     rate of reaction
dt 2 dt 3 dt
Here ă ve sign is put before [A] and [B] because their molar concentration decrease with time.

Illust rat ion 1


The rate of change in concentration of C in the reaction 2A + B  2C + 3D was reported
as 1 mole litreă1 secă1. Calculate the reaction rate as well as the rate of change of concen-
tration of A, B and D.
Solution :

 1 d[A]  d  B 1 d C 1 d  D
  
2 dt dt 2 dt 3 dt
= rate of reaction
d  C
 = 1 mol lă1 să1
dt

 d A d  C
   1 mol l 1 s1
dt dt

 d  B 1 d  C
  0.5 mol l 1 s1
dt 2 dt

d  D 1 d  C
  1.5 mol l 1 s1
dt 2 dt

1 d [C]
Rate of reaction =  0.5 mol l 1 s1
2 dt

CHEMICAL EQUILIBRIUM
QUIZRR 5

Fa c t o r s I n f l u e n c i n g t h e Ra t e o f a Re a c t io n
(i) Concentration of the reactant species : Greater the concentrations of the reactants, the
greater is the rate.
(ii) Temperature : The rate of a reaction increases with increase in temperature. In general
for most reactions the rate constant doubles with every 10Ĉ rise in temperature.
(iii) Nature of the reactants : Reactions which do not involve considerable bond rearrange-
ment are generally faster at room temperature than those which involve considerable bond
rearrangements.
(iv) Effect of catalyst : A reaction proceeds much faster in the presence of a catalyst. For
instance, the hydrolysis of an ester is very fast when catalyzed by H+ ions.
(v) Effect of radiation : Photochemical reactions are faster than thermal ones because in the
former all the energy of the photons is completely used in exciting the molecules while in
the latter, the energy is distributed at random translational, rotational and vibrational
modes of motion.

L a w o f M a s s A c t io n a n d Eq u i l ib r i u m Co n s t a n t
The rate at which a substance reacts is proportional to its active mass and the rate of a
chemical reaction is proportional to the product of the active masses of the reacting substance.
The reactions are generally reversible, that is, they can proceed both ways. A reaction is said to
have attained equilibrium when the rate of the forward reaction equals that of the backward reaction.
Let us consider a general case of a reversible reaction,

aA + bB 

 mM + nN
Applying the law of mass action :
Rate of the forward reaction  [A]a [B]b
or rate of the forward reaction = k1 [A]a [B]b
Rate of backward reaction  [M]m [N]n
or rate of backward reaction = k2 [M]m [N]n
At equilibrium :
Rate of forward reaction = rate of backward reaction
k1 [A]a [B]b = k2 [M]m [N]n

k1 Mm  Nn
=
k2  A a Bb

k Mm  Nn
or KC  1 = ...(1)
k2  A a Bb

CHEMICAL EQUILIBRIUM
6 QUIZRR
[ ] represents concentration in moles per litre. Here, all concentrations are at equilibrium. k1 and
k2 are known as the rate constants of the forward and backward reactions respectively.
Chemical equilibrium is dynamic in the sense that individual molecules are continually reacting,
even though the overall composition of the reaction mixture does not change. The other criteria of a
chemical equilibrium are the same equilibrium state can be attained from both sides of the reaction and
a reaction which attains equilibrium is always incomplete.

[M] & [N]


Concentrations

[A] & [B]


Eqb. is established

Time

The rate constant of any reaction is defined as the rate of the reaction when the concentration
of each reactant is unity. The rate constant of a reaction depends on the temperature and the catalyst.
kc, the ratio of the rate constants, is known as the equilibrium constant. Now, if the above reaction
is supposed to be a homogeneous gaseous one, the equilibrium constant may also represented as

m n
pM . pN
KP  ...(2)
a b
pA . pB

where p´s* represent the partial pressures at equilibrium.

Re a c t io n b e t w e e n K p & K c
The equilibrium constant Kp and Kc is known as pressure equilibrium constant. The magnitude
of Kp and Kc is a measure of the extent to which the reaction occurs. The equilibrium constants depend
only on temperature. For a given reaction Kp and Kc may be equal and may also be different
depending on the stoichiometry of the reaction. Kp and Kc and are related as
Kp = Kc (RT)n ...(3)
where, n = no. of moles of the gaseous product ă no. of moles of the gaseous reactants (in the
balanced equation and not in the reaction)
or n = (m + n) ă (a + b)
But both the equilibrium constants for a given reaction give the same information about the state
of equilibrium when the reaction at equilibrium is subjected to a change in temperature, pressure or
concentration as also expressed by Le ChatelierÊs principle. The units of Kp and Kc are not fixed and
depend on the stoichiometry of the reaction. In case the number of moles of the reactant and that of
the product are same, Kp and Kc do not have any unit.

CHEMICAL EQUILIBRIUM
QUIZRR 7

[Note : There is a third type of equilibrium constant, much less in use, called Kx, when concen-
trations are expressed in terms of mole fraction (x).]

xm . xn
Thus, Kx  M N
a b
xA . xB

Relating this expression with equations 1, 2 and 3, we get


Kp = Kx.pn = Kc(RT)n
for n = 0 : Kp = Kc = Kx
Kx does not have any unit. The mole fraction equilibrium constant Kx , unlike Kp and Kc, may
depend on pressure and volume.

I m p o r t a n t Re l a t i o n s h i p I n v o l v in g Eq u i l i b r iu m Co n s t a n t :
(a) If you reverse an equation, KC or KP is inverted i.e.

If A + B 

 C + D Kc = 10

then C + D 

 A + B K´c = 10ă1
(b) If you multiply each of the coefficient in a balanced equation by a factor m, then equilibrium
constant is raised to the same factor
1 1
If N2 + O2 

 NO, Kc = 5
2 2

then for N2 + O2 



 2NO K´c = K 2c = 52 = 25
(c) If you divide each of the coefficients in a balanced equation by the factor m, then new
equilibrium constant is mth root of the previous value.

If 2SO2 + O2 

 2SO3 Kc = 25

1 1
then for SO2 + O 

2 2  SO3 K 'c =  K c  2  25  5

(d) When you combine (i.e., add) individual equation, multiply their equilibrium constants to
obtain the equilibrium constant for the reaction.
If K1, K2, K3 are stepwise equilibrium constant for

A 

 B ...(i)

B 

 C ...(ii)

C 

 D ...(iii)

then for A 



 D [obtained by adding (i), (ii) and (iii) net equilibrium constant is,
K = K1 K2 K3.

CHEMICAL EQUILIBRIUM
8 QUIZRR
(e) Do not include concentration terms for pure solids and pure liquids in equilibrium constant
expression.

for C(s) + H2O(g) 



 CO2(g) + H2(g)

Kc 
CO H2 
 H2O 

for CaCO3(s) 



 CaO(s) + CO2(g)

Kp = PCO2 and Kp = Kc (RT)

I n t e r p r e t a t io n o f Eq u i l ib r iu m Co n s t a n t
The equilibrium constant for a reaction tells about the tendency of a reaction to proceed to
products.
(i) If the concentration of each reactant and product in a general reaction

aA + bB 

 mM + nN
are such that,

M m . Nn  K (equilibrium constant)


 A a . Bb
the reaction is at equilibrium.
(ii) If the concentration are such that,

M m . N n K
 A a .Bb
the reaction will proceed from left to right in order to attain equilibrium, or in other words,
the concentration factor or the reaction quotient increases and becomes equal to K.
(iii) And, if the concentrations are such that,

M m . N n K
 A a .Bb
the reaction will proceed from right to left in order to attain equilibrium, or in other words,
the concentration factor or the reaction quotient decreases and becomes equal to K.

CHEMICAL EQUILIBRIUM
QUIZRR 9

If the concentration factor, i.e.,


M m . N n
(not necessarily eqb. concentrations)
 A a .Bb 

is expressed as reaction quotient, Q, we have,


1. Q > K : Reaction proceeds in backward direction until equilibrium is established.
2. Q = K : Reaction is at equilibrium.
3. Q < K : Reaction proceeds in forward direction until equilibrium is established.

Sig n i f ic a n c e o f t h e M a g n i t u d e o f Eq u i l ib r i u m Co n s t a n t
(i) A very large value of KC or KP signifies that the forward reaction goes to completion or very
nearly so.
(ii) A very small value of KC or KP signifies that the forward reaction does not occur to any
significant extent.
(iii) A reaction is most likely to reach a state of equilibrium in which both reactants in which
both reactants and products are present if the numerical value of Kc or Kp is neither very
large nor very small.

Illust rat ion 2


The value of Kp for the reaction
2H2O(g) + 2Cl2(g)  4HCl(g) + O2(g)
is 0.035 atm at 400ĈC, when the partial pressure are expressed in atmosphere.
1
Calculate Kc for the reaction, O 2 ( g )  2HCl( g )  Cl 2 ( g )  H 2 O( g ) .
2
Solution :
KP = KC (RT)n
n = moles of product ă moles of reactants = 5 ă 4 = 1
R = 0.082 L atm/mol K
T = 400 + 273 = 673 K
 0.035 = Kc (0.082 ï 673)
Kc = 6.342 ï 10ă4 mol lă1
1
 K ´c for the reverse reaction would be K
c

 
1 1
 K´c  4
 1576.8 mol l 1
6.342  10

CHEMICAL EQUILIBRIUM
10 QUIZRR
When a reaction is multiplied by any number n (integer or a fraction) then Kc or Kp becomes (Kc)n
or (Kp)n of the original reaction.

1
 Kc for O2 ( g)  2HCl( g)  Cl2 ( g)  H2O( g)
2

is 1576.8  39.7 (mol 1 )ó

Ca l c u l a t i o n o f K p a n d K c
From the relation Kp = Kc (RT)n, we get two types of reaction, viz.,
(i) when n = 0, i.e., those reactions in which there is no change in the number of molecules,
e.g., H2 + I2  2HI;
n = 2 ă 2 = 0; for such a reaction, Kp = Kc
(ii) When n  0, i.e., those reactions in which there is a change in the number of molecules,
e.g., N2 + 3H2  2NH3; n = 2 ă 4 = ă 2.
For such a reaction, Kp  Kc
The calculations of Kp and Kc for the above types of reactions depend mainly on the values of
molar concentration (i.e., moles/litre) and partial pressure at equilibrium, as shown in eqns. (1) and (2).
Let us now present the methods of calculating Kp and Kc by taking the following examples :

(1 ) Fo r m a t i o n o f H I
a b 0 Initial moles
H 2(g) + I2 (g)  2HI(g)

(a ă x) (b ăx ) 2x Moles at equilibrium

( a  x) (b  x) 2x
V
Molar concentration at eqb.
V V

where x is the number of moles of H2 or I2 converted to HI at equilibrium and V is the volume


in litres of the container.

HI2   2 x / V 2
Kc =
 H 2   I2   a  x  .  b  x 
V V

4x2
=
 a  x  b  x

CHEMICAL EQUILIBRIUM
QUIZRR 11

Further, total number of moles at equilibrium


= (a ă x) + (b ă x) + 2x
=a + b
Let the total pressure be p.

 a  x . p; p 
 b  x . p
pH = and
2  a  b I2
 a  b

2x
pHI = .p
 a  b

2
pHI 4 x2

Kp = p p
H2 I 2
 a  x  b  x (substituting the value of pÊs)

(2 ) Fo r m a t i o n o f N H 3
a b 0 Initial moles
N2 (g) + 3H2 (g)  2NH3
(a ă x) (b ă 3x ) 2x Moles at equilibrium

( a  x) (b  3 x) 2x
V
Molar concentration at equilibrium
V V

where x is the number of moles of N2 which converted to NH3 at equilibrium.

2
 2x 
 NH3 2  
Kc   V
 N2  H2 3  a  x  b  3 x 
3
  
 V  V 

Further, total number of moles at equilibrium


= a ă x + b ă 3x + 2x = (a + b ă 2x)

2
 2x  
2   . p
Kp 
PNH3
  a  b  2 x  
PN .PH 3
 ax  b  3 x 
2 2
 . p . p
 a  b  2 x  a  b  2 x 

CHEMICAL EQUILIBRIUM
12 QUIZRR
(3 ) Di s s o c i a t i o n o f PCl 5
a 0 0 Initial moles
PCl5(g)  PCl3 (g) + Cl2(g)
(a ă x) x x Moles at equilibrium

( a  x) x x
Molar concentration at eqb.
V V V

Here, x is the number of moles of PCl5 converted to the products at equilibrium. But if x represents
degree of dissociation, i.e., fraction of the total number of molecules undergoing dissociation, the
equilibrium moles and equilibrium molar concentration of each species will be as follows :
a 0 0 Initial moles
PCl5(g)  PCl3 (g) + Cl2(g)

a(1 ă x) ax ax Moles at equilibrium

a(1  x) ax ax
Molar concentration at eqb.
V V V

From the values of given above, equations for Kp and Kc can be derived.

(4 ) Di s s o c i a t i o n o f H a l o g e n M o l e c u l e s
a 0 Initial moles
X2(g)  2X(g) {If x is the number of moles of X2 converted to X at eqb.}
(a ă x) 2x Moles at equilibrium

 a  x  2x 
 V   V Molar concentration at equilibrium
   

But if x is the degree of dissociation :


X2(g)  2X(g)

a(1 ă x) 2ax Moles at equilibrium

 a(1  x)   2ax 
 V   V  Molar concentration at eqb.
  

Derive Kp and Kc.

CHEMICAL EQUILIBRIUM
QUIZRR 13

Homogeneous Equilibria and Equations for Equilibrium Constant (Equilibrium Pressure is


P atm in a V L Flask)
H2 + I2  2HI N2 + 3H2  2NH3 PCl5  PCl3 + Cl2

(g) (g) (g) (g) (g) (g) (g) (g) (g)


Initial mol a b O 1 3 0 1 0 0
Equilibrium mol (a ă x) (b ă x) 2x (1 ă x) (3 ă 3x) 2x (1 ă x) x x
Total mol at eqb. (a + b) (4 ă 2x) (1 + x)

 a  x  b  x  2x  1  x 1  x  2x  1  x  x  x
Active masses  V   V   V  V  3 V   V  V  V V
                 

 a  x  b  x  2x  1  x 3 1  x x 1  x  x   x 
      2 2x
Mol fraction
 a  b  a  b  a  b   2  2  x   2  x  
1  x
 
1  x
 
1  x

 a  x  b x  2 x   1  x   3(1  x)  Px 1  x  x   x 
Partial pressure P  P  P  P  P   x)
P  P  P 
 a  b   a  b  a  b   2 (2  x   2(2  x)  (2 1  x 1  x  1 x 

2 4 x2 V 2 2
4x x
Kc
 a  x b  x 27 1  x
4
1  x V
2
16 x2  2  x 
2 2
4x Px
Kp
 a  x b  x 27 1  x P 2
4
1  x
2

Heterogeneouos Equilibria and Equation for Equilibrium Constant (Equilibrium Pressure


is P atm)
NH4HS(s)  NH3(g)+H2S(g) C(s)+CO2(g)  2CO(g) NH4CO2NH2(s)  2NH3(g)+ CO2(g)

Initial mol 1 0 0 1 1 0 1 0 0
Eqn. mol (1ăx) x x (1ă x) (1ă x) 2x (1ă x) 2x x
Total mol. at eqn. 2x (1 + x) 3x
(solidnotincluded)

x 1 1 1 x  2x  2 1
molfraction  1  1  x
2x 2 2  x   3 3

P P 1  x  2x  2P P
Partial pressure P  P 
2 2 1  x 1  x 3 3

2 2
4P x
P2 4P3
Kp
4 1 x
2
 27

CHEMICAL EQUILIBRIUM
14 QUIZRR
Re l a t io n B e t w e e n V a p o u r De n s i t y a n d De g r e e o f Di s s o c i a t i o n
Let equilibrium reaction be with x as degree of dissociation

A 

 yB.

Dd
x
given by
 y  1 d
where D is the vapour density of A before dissociation, d the vapour density of the reaction
mixture consisting of A and B, and y the no. of products.
D ï 2 = mol. wt of A
d ï 2 = mol. wt of mixture (also called abnormal mol. wt. of A)

Dd 

Thus x  for (PCl5  PCl3 + Cl2) with y = 2
d

Ex a m p l e 1
Write equilibrium constants for each :
3
(a) KClO3(s)  KCl(s) + O
2 2
(b) H2(s) + 3FeO3(s)  H2O(g) + 2Fe3O4(s)
(c) CaC2(s) + 5O2(g)  2CaCO3(s) + 2CO2(s)
(d) H2(g) + I2(g)  2HI(g)
(e) N2(g) + 3H2(g)  2NH3(g)
(f) CH3COOH(l) + CH3OH(l)  CH3COOCH3(l) + H2O(l)
Solution :
Note : 1. Write Kp for heterogeneous systems if gaseous phase is present.
2. Write KC for homogeneous solution phase.
3. Write Kp or Kc for homogeneous gaseous phase.
3/
2
(a) Kp = PO' 
 2

PH
´  0
 2O   nH2O   P 
(b) Kp =  
nH2   N 
´
PH 2

2 2
PCO
´   nCO   P  3
 2
  2
5  
.
 nO    N 
(c) Kp = 5
PO ´ 
 2  2

CHEMICAL EQUILIBRIUM
QUIZRR 15

2
PHI
' 
 
KC 
 HI
2
(d) Kp = ' or
PH  PI' 2
2
 H2   I 2 

2
PNH

' 
3 KC 
 NH3 
2

(e) Kp = or
PN
'  ' 
P
3
 N2 H2 3
 2   H2 

KC 
CH3COOCH3 H2O
(f)
CH3COOHCH3OH

Ex a m p l e 2
If a mixture of 3 mole of H2 and one mole of N2 is completely converted into NH3, what
would be the ratio of the initial and final volume at same temperature and pressure ?
Solution :
N2 + 3H2  2NH3
Initial mole 1 3 0
Final mole 0 0 2 ( complete conversion)

Initial mole
 Ratio of initial and final volumes =
Final mole

4
( V  n if P and T are constant) = 2
2

Ex a m p l e 3
Calculate the equilibrium constant for the reaction H2(g) + CO2(g)  H2O(g) + CO(g) at 1395
K, if the equilibrium constants at 1395 K for the following are :
2H2O(g)  2H2 + O2(g) K1 = 2.1 ï 10ă13
2CO2(g)  2CO(g) + O2(g) K2 = 1.4 ï 10ă12
Solution :

K1 
H2 2 O2 
For 2H2O  2H2 + O2 ...(1)
H2O2

K2 
CO2 O2 
For 2CO2  2CO + O2 ...(2)
CO2 2

CHEMICAL EQUILIBRIUM
16 QUIZRR

K
H2O2 CO
For CO2 + H2  H2O + CO ...(3)
CO2 H2 

(2)
Thus, by Eqs.
(1)

K2 CO O2   H2O


2 2

=
K1 CO2 2 H2 2 O2 

K2 CO2 H2O2  K 2
=
K1
CO2 2 H2 2

 K2   1.4  1012 
K =       2.58
13 
 K1   2.1  10 

Example 4
For the reaction A + B  3C at 25ĈC , a 3 litre vessel contains 1, 2, 4 mole of A, B and C
respectively. Predict the direction of reaction if :
(a) KC for the reaction is 10.
(b) KC for the reaction is 15.
(c) KC for the reaction is 10.66.
Solution :
A + B  3C
Before reaction [A] = 1/3
[B] = 2/3
[C] = 4/3

[C]3 43  3  3 64
Q  3   10.66
Thus, [A]  B 3  1  2 6

(a) Since KC = 10, thus Q must decrease to attain KC value and thus, [C] must decrease or [A]
and [B] should increase. Thus, reaction will occur in backward direction.
(b) Since KC = 15, thus Q must increase to attain KC value and thus, [C] must increase or [A]
and [B] should decrease. Thus, direction will occur in forward direction.
(c) Q = KC; thus, reaction is in equilibrium.

CHEMICAL EQUILIBRIUM
QUIZRR 17

Ex a m p l e 5
At temperature T, a compound AB 2 (g) dissociates according to the reaction :
2AB2(g)  2AB(g) + B2(g) with a degree of dissociation ÂxÊ which is small compared to unity.
Deduce the expression for ÂxÊ in terms of the equilibrium constant Kp and the total pressure P.
[IIT 1994]
Solution : 2AB2(g)  2AB(g) + B2(g)
Mole before dissociation 1 0 0

x
Mole after dissociation (1 ă x) x
2

Total mole at equilibrium   n  1  x  x  2x  1  2x


n
nB2   nAB 
2
 P 
Now, Kp =  
 nAB2    n 
2

1
x  
.( x)2  P 
2 
= (1  x)2  
1  x 
Kp
 2 

x3 P  x 
Kp =  x is small,  1  x  1 and 1  2  1
2  

2K p
or x = 3
P

Ex a m p l e 6
For a gaseous phase reaction, A + 2B  AB2, KC = 0.3475 litre2 molă2 at 200ĈC. When 2 mole
of B are mixed with one mole of A, what total pressure is required to convert 60% of A in
AB2?
Solution :
A + 2B  AB2

Initial mole 1 2 0
Mole at equilibrium (1 ă x) (2 ă 2x) x
Total mole at equilibrium = 1 ă x + 2 ă 2x + x = 3 ă 2x

CHEMICAL EQUILIBRIUM
18 QUIZRR
Let pressure at equilibrium be P;

'  x   1x   2  2x 
Now, PAB   P; PA'    P; PB'   P
 3  2x  3  2x   3  2x 
2

x.P
 Kp =
(1  x) 2 (2  2 x)2
(3  2 x).P P
(3  2 x) (3  2 x)2

x(3  2 x)2
Kp = ...(1)
P 2 (1  x)(2  2 x) 2

Alternate to derive Kp or Eq. (1),

n
nAB2  P 

Kp =
nA  (nB ) 2   n 
 

2
x  P 
 Kp =  
 (3  2 x) 
2
(1  x) (2  2 x)

x(3  2 x)2
= ...(1)
(1  x)(2  2 x)2 .P 2

Given that x = 0.6 and n = ă 2


n
 Kp = Kc (RT) = 0.3475 ï (0.0821 ï 473)ă 2
...(2)
By Eqs. (1) and (2),

0.6(3  1.2)2
 0.3475 ï (0.0821 ï 473)ă 2
=
P 2 (1  0.6)(2  1.2)2

0.6  (1.8)2
=
P 2 (0.4)(0.8)2

P = 181.5 atm

Ex a m p l e 7
For a gaseous phase reaction, 2HI  H2 + I2, at equilibrium 7.8 g, 203.2 g and 1638.4 g of
H2, I2 and HI respectively were found in 5 litre vessel. Calculate Kc . If all the reactants and
products are transferred to a 2 litre vessel, what will be the amount of reactants and
products at equilibrium ?

CHEMICAL EQUILIBRIUM
QUIZRR 19

Solution :
2HI  H2 + I2

1638.4 7.8 203.2


Mole at equilibrium
128 2 254
= 12.8 3.9 0.8
Let volume of container be V litre

3.9 12.8 0.8


 H2   V
; HI  V
; I 2   V

H2  I2   3.9  0.8  0.019


 KC = HI2 VV
 12.8 
2
 
 V 

 KC = 0.019
For the given reaction n = 0 and thus there will be no effect on equilibrium concentration of
reactants and products if the matter is transferred to attain equilibrium in 2 litre vessel.

Ex a m p l e 8
60 mL of H2 and 42 mL of I2 are heated in a closed vessel. At equilibrium the vessel contains
28 mL HI. Calculate degree of dissociation of HI.
Solution :
H2 + I2  2HI Given
Volume at t = 0 60 42 0  2x = 28
Volume at equilibrium (60 ă x) (42 ă x) 2x  x = 14
(60 ă 14) (42 ă 14) 28
Since at constant P and T, mole  volume of gas (By PV = nRT). Thus, volume of gases given
can be directly used as concentration. This can be done only for reactions having n = 0.
28  28 28
 KC  
46  28 46
Now for dissociation of HI; 2HI  H2 + I2
Mole at t = 0 1 0 0
Mole at equilibrium (1 ă ) (/2) /2
where  is degree of dissociation

2 1
K C1 = 
4 1   
2 KC

  46 
 =  
2(1  )  28 
  = 0.719 or 71.9%

CHEMICAL EQUILIBRIUM
20 QUIZRR
Ex a m p l e 9
KC for CO(g) + H2O(g)  CO2(g) + H2(g) at 986ĈC is 0.63. A mixture of 1 mole H2O(g) and 3 mole
CO(g) is allowed to react to come to an equilibrium. The equilibrium pressure is 2.0 atm.
(1) How many mole of H2 are present at equilibrium ?
(2) Calculate partial pressure of each gas at equilibrium.
Solution :
CO(g) + H2O(g)  CO2(g) + H2(g)
Initial mole 3 1 0 0
Mole at equilibrium (3 ă x) (1 ă x) x x
Total mole at equilibrium = 3 ă x + 1 ă x + x + x = 4

x2
Now KC =
(3  x) 1  x 

x2
 = 0.63 ( KC = 0.63)
3  x2  4 x

 x = 0.681
 Mole of H2 formed = 0.681
Total pressure at equilibrium = 2 atm
Total mole at equilibrium = 4

Pg' = PM ï mole fraction of that gas

' x.P 0.681  2


 PCO = PH  = = 0.34 atm
2 2 4 4

'
(3  x).P
PCO = = 1.16 atm
4

' (1  x).P
PH 2O
= = 0.16 atm
4

Ex a m p l e 1 0
An equilibrium mixture of CO(g) + H2O(g)  CO2(g) + H2(g) present in a vessel of one litre
capacity at 815ĈC was found by analysis to contain 0.4 mole of CO, 0.3 mole of H2O, 0.2 mole
of CO2 and 0.6 mole of H2.

CHEMICAL EQUILIBRIUM
QUIZRR 21

(a) Calculate KC.


(b) If it is derived to increase the concentration of CO to 0.6 mole by adding CO2 to the
vessel, how many mole must be added into equilibrium mixture at constant temperature
in order to get this change ?
Solution :
CO(g) + H 2O(g)  CO2(g) + H 2(g)
Mole at equilibrium 0.4 0.3 0.2 0.6

KC 
CO2 H2   0.2  0.6  1
(a) 
COH2O 0.4  0.3
( n = 0,  Volume terms are not needed.)
(b) Now it is desired to increase the conc. of CO by 0.2 at equilibrium by forcing CO2 into
equilibrium mixture. Suppose a mole of CO2 are forced in vessel at equilibrium by doing so
reaction proceeds in backward direction, i.e.,
CO2 + H2  CO + H 2O
Addition at initial
equilibrium (0.2 + a) 0.6 0.4 0.3
Mole at (0.2+aă0.2) (0.6ă0.2) (0.4+0.2) (0.3+0.2)
new equilibrium a 0.4 0.6 0.5

1 CO H2O  0.6  0.5


  =
KC CO2  H2  a  0.4
a = 0.75 mole

Ex a m p l e 1 1
A mixture of one mole of CO2 and one mole of H2 attains equilibrium at a temperature of
250ĈC and a total pressure of 0.1 atm for the change CO2(g) + H2(g) + H2O(g). Calculate Kp if
the analysis of final reaction mixture shows 0.16 volume per cent of CO.
Solution :
CO2(g) + H2(g)  CO(g) + H 2O(g)
Mole at t = 0 1 1 0 0
Mole at equilibrium (1 ă x) (1 ă x) x x
Given that Vol. % of CO = 0.16
 Mole of CO = x
Total mole at equilibrium = 1 ă x + 1 ă x + x + x = 2

CHEMICAL EQUILIBRIUM
22 QUIZRR

x 0.16
 =
2 100

 x = 0.0032

x2
Now, KC = Kp = ( n = 0, volume terms are not needed)
1  x2

 0.0032 2
Kp   1.03  10 5
1  0.0032  2

L e Ch a t e l i e r ’s Pri n c i p a l a n d Eq u i l ib r i u m Co n s t a n t
Le ChatelierÊs principle states that if a chemical reaction at equilibrium is subjected to a change
in temperature, pressure or concentration, the equilibrium shifts to that direction by which the effect
of the change is minimised.
One should note that the change in either pressure or concentration at equilibrium may shift the
equilibrium or more clearly, change the state of equilibrium, but it cannot change the equilibrium
constant Kp or Kc which depends only on temperature.
However, when a reaction (except those for which H = 0) is subjected to a change in temperature,
the equilibrium shifts to another equilibrium position at the new temperature and the equilibrium
constant also changes.
In general, for an exothermic reaction (H is negative), increase in temperature results in shifting
the equilibrium from right to left, thereby decreasing the value of the equilibrium constant. For an
endothermic reaction (H is positive), the value of equilibrium constant increases as the temperature
increases.

Ef f e c t o f T e m p e r a t u r e o n Eq u i l i b r iu m Co n s t a n t
The quantitative effect of temperature on the equilibrium constant can be determined by the
following thermodynamically derived equation, known as VanÊt Hoff equation,

H  T2  T1 
log K p  log K p   
2 1 2.303R  T1 .T2 

where K p1 and K p 2 are the equilibrium constants at temperatures T1 and T2 respectively, HĈ
is standard heat of the reaction at constant pressure.
Here, it has been assumed that H is constant in the temperature range between T2 and T1

(T2 > T1). We readily see that K p 2 > K p1 , if HĈ is positive, (endothermic) and K p 2 < K p1 , if HĈ
is negative (exothermic).

CHEMICAL EQUILIBRIUM
QUIZRR 23

Fa c t o r s I n f l u e n c i n g t h e Eq u i l i b r iu m St a t e
The effects of various factors on the equilibrium state for the different types of reactions may be
clearly understood by using the following expressions of Kp.
Type I reaction
n = 0 : A (g) + B (g) = C (g) + D (g);

moles of C  moles of D
Kp =
moles of A  moles of B

Type II reaction
n is positive : A (g) = C (g) + D (g);

moles of C  moles of D total pressure


Kp = 
moles of A total moles

n is negative : A (g) + B (g) = D (g);

moles of D total moles


Kp = 
moles of A  moles of B total pressure

1. Ef f e c t o f Pre s s u r e (o r v o l u m e )
Type of reaction Effect of Pressure (or volume)

Type I :  n = 0 Eqb. is not affected


Type II : n = +ve Increase in p (or decrease in V) shifts the eqb. to
the left
           n = ăve Increase in p (or decrease in V) shifts the eqb. to
the right.

2. Ef f e c t o f A d d i t io n o f I n e r t Ga s
Type of reaction Effect of addition of inert gas

Type I :  n = 0 Eqb. is not affected either at constant pressure or


at constant volume
Type II : n = +ve At constant volume : eqb. is not affected
At constant pressure : eqb. shifts to the right
           n = ăve At constant volume : eqb. is not affected
At constant pressure : eqb. shifts to the left

CHEMICAL EQUILIBRIUM
24 QUIZRR
3. Ef f e c t o f A d d i t io n o f Ca t a l ys t
Adding a catalyst to a reaction changes its rate but this cannot shift the equilibrium in favour
of either products or reactants. Because a catalyst affects the activation energy of both forward and
backward reactions equally, it changes both rate constants by the same factor, so their ratio Kc, does
not change. Thus, adding a catalyst to a reaction at equilibrium has no effect; it changes neither the
reaction quotient, Q, nor the equilibrium constant K, but only the time required to established equilibrium
is altered.

4. Ef f e c t o f A d d i t io n o r Re m o v a l o f a n y o f t h e Re a c t a n t s o r Pro d u c t s
If any of the reactants or products is added or removed from a system at equilibrium the equilibrium
shifts in the direction that consumes or produces the added or removed substance respectively. This is
only true if the volume of the system is kept constant. But under the constant pressure condition,
shifting of equilibrium may be in the reverse direction depending upon the value and sign of n and
number of molecules of the added or removed substance involved in the reaction.

T H ERM ODY N A M I CS OF CH EM I CA L EQU I LI B RIU M


Spontaneity
A spontaneous or natural process is a process that occurs in a system left to itself once started;
no action from outside the system (external action) is necessary to make the process continue. On the
other hand, a nonspontaneous process will not occur unless some external action is continuously
applied. We say that the rusting of iron is spontaneous.
2Fe(s) + 3O2(g)  Fe2O3(s)
Reverse reaction is not impossible, but it is certainly non spontaneous.

Gib b s Fr e e En e r g y * Ch a n g e a n d Sp o n t a n e i t y :
The Gibbs free energy ÂGÊ for system is defined as
G = H ă TS
and, for a change at constant T,
G = H ă TS
where  G = change in Gibbs free energy G
 H = change in enthalpy H
 S = change in entropy S
Sign of  G is taken as the true criterion for spontaneity.
For a process occuring at constant T and P, if
(i) G < 0 (negative) the process is spontaneous
(ii)  G > 0 (positive) the process is nonspontaneous
(iii)  G = 0 (zero) the process is at equilibrium

CHEMICAL EQUILIBRIUM
QUIZRR 25

Sign of  G, based on eqn. ( G =  H ă T  S) can be decided depending on sign of H and


S, and magnitude of T, and is explained in Table below.

Case H S G Result Example

1 ă + ă spontaneous at all temp. 2N2O(g)  2N2(g) + O2(g)


2 ă ă ă spontaneous at low temp. H2O(l)  H2O(g)
+ nonspontaneous at high temp.
3 + + + nonspontaneous at low temp. 2NH3(g)  N2(g) + 3H2(g)
ă spontaneous at high temp
4 + ă + nonspontaneous at all temp 3O2(g)  2O3(g)

5 TS = H 0 equilibrium A 
 B

Relationship of  GĈ to the Equilibrium Constant K :


 G for a reaction under any set of conditions is related to its value for standard conditions, that
is GĈ by eqn.
G = GĈ + 2.303 RT log Q
under equilibrium condition Q = Kp = Kc = K
          G = 0
GĈ = ă 2.303 RT log K
We have replaced Kp or Kc by K called thermodynamic equilibrium constant. For a general
reaction
aA + bB  cC + dD

 a C c  a D 4
K b
( aA) a  aB 

the symbol a represents the activity of the reactants and products. It is actually a ratio of the
equilibrium activity of a substance to its activity in its standard state. Thus thermodynamic equilibrium
constant is unitless quantity, a being also unitless.
ă For pure solids and liquids : a = 1
ă For gases : ideal behaviour is considered and the activity of a gas is equal to its pressure
in atm.
ă For components in solution : a = molar concentration

V a r ia t i o n o f K w it h T e m p e r a t u r e : T h e v a n ’t H o f f Eq u a t io n :
Equilibrium constant K varies with temperature T as given by vanÊt Hoff eqn.

H S
log K  
2.303 RT R

CHEMICAL EQUILIBRIUM
26 QUIZRR
where HĈ and SĈ, heat of reaction and entropy change respectively in standard state are
temperature independent. If K1 and K2 are equilibrium constants at T1 and T2 respectively, then

K2 H 1 1 
log    
K1 2.303R  T1 T2 

This equation can be used to compute HĈ

d log K H
also 
dT 2.303RT 2

log K
A graph between log K and Tă1 is a straight line of

H O ă1
slope = ă T
2.303 R

HĈ = ă 2.303 R ï slope

CHEMICAL EQUILIBRIUM
QUIZRR 27

M ISCEL L A N EOU S

Ex a m p l e 1
At 46ĈC, Kp for the reaction N2O4(g)  2NO2(g) is 0.66. Compute the percent dissociation
of N2O4 at 46ĈC and a total pressure of 0.5 atm. Also calculate partial pressure of N2O and
NO2 at equilibrium.
Solution :
Let x = fraction of N2O4 dissociated at 0.5 atm
N2O4(g)  2NO2

1 0 mol at start
ă x 2x change by reaction
1 ă x 2x mol at equilibrium

1x 2x
1x 1x
mol fraction

1 x   2x 
PN2O4  0.5   PNO = 0.5   partial pressure
1  x 2 1  x

2
 2 x  0.5 
 
P 2 NO2 1 x 
thus Kp = 0.66 = P 
N2O4 (1  x)
0.5
(1  x)

this give x = 0.5 i.e. 50% dissociation


hence partial pressure of N2O4 = 0.167 atm
and that of NO2 = 0.333 atm
Alternate method :
let PNO = P atm
2
hence PN = (0.5 ă P) atm
2O 4

2
PNO 2 P2
hence Kp  =  0.66 atm
PN2O4  0.5  P 

P2 + 0.66 P ă 0.33 = 0

this gives PNO2 = 0.333 atm, PN2O4 = 0.167 atm

CHEMICAL EQUILIBRIUM
28 QUIZRR
Ex a m p l e 2

What will be the effect on the equilibrium constant for the reaction N2 + 3H2 

 2NH3;
H = ă 22.4 kcal, when (a) pressure is increased (b) concentration of N2 is increased, and (c)
temperature is raised at equilibrium ?
Solution :
(a) No effect  As K does not depend on

(b) No effect  pressure and concentration
(c) Equilibrium constant will decrease as the temperature is increased.

Ex a m p l e 3
1
Kc for the reaction SO2 + O  SO3 at 600ĈC is 61.7. Calculate Kp . What is the unit of Kp
2 2
for the above equilibrium ? (R = 0.0821 lit. atm per deg. per mole)
Solution :
n = moles of product ă moles of reactant

 1 1
= 1  1    
 2  2

Kp = Kc (RT)n

1
Kp = 61.70.0821  (600  273) 2  7.29

pSO3 atm
Kp = 
1 1
pSO2 . pO2 atm.atm 2
2

1

Unit of K p is atmosphere 2

Ex a m p l e 4
A saturated solution of iodine in water contains 0.330 g I2/L. More than this can dissolve
in a KI solution because of the following equilibrium :
I2(aq) + Iă  I3ă
A 0.100 M Iă solution dissolves 12.5 g I2/L, most of which is converted to I3ă. Assuming that
the concentration of I2 in all saturated solutions is the same, calculate the equilibrium
constant for the above reaction.

CHEMICAL EQUILIBRIUM
QUIZRR 29

Solution :

0.330
0.330 g I2 = = 1.30 ï 10ă3 mol I2
254

12.5
12.5 g I2 = = 0.0492 mol I2
254

at equilibrium, [I2] = 1.30 ï 10ă3 M,

I3  = (0.0492 ă 1.30 ï 10ă3) = 0.0479 M


 

[Iă] = (0.100 ă 0.0479) = 0.0521 M

I3 
   0.0479
 707
K =
 
I2  I  1.30  103 (0.0521)

Ex a m p l e 5
The degree of dissociation of I2 molecule at 1000ĈC and under atmospheric pressure is 40%
by volume. Find the total pressure on the gas at equilibrium so that the dissociation is
reduced to 20% at the same temperature.
Solution :
I2(g)  2I(g)
initial mol 1 0
at equilibrium (1 ă x) 2x x = 0.40, P1 = 1 atm in case I
(1 ă y) 2 y y = 0.20, P2 = ? in case II

P12 4 x2 P1 4 y2 P2
Kp  
PI2 (1  x2 ) = (1  y2 )

4  0.16  1 4  0.04  P2
 (1  0.16)
= (1  0.04)

this gives P2 = 4.57 atm.

Ex a m p l e 6
Solid ammonium carbonate, NH4CO2NH2 dissociates completely into ammonia and carbon
dioxide when it evaporates as shown by NH4CO2NH2(s)   2NH (g) + CO (g) At 25ĈC, the
3 2
total pressure of the gases in equilibrium with the solid is 0.116 atm. What is the equilibrium
constant of the reaction? If 0.1 atm of CO2 is introduced after equilibrium is reached, will
the final pressure of CO2 be greater or less than 0.1 atm? Will the pressure of NH3 increase
or decrease?

CHEMICAL EQUILIBRIUM
30 QUIZRR
Solution :
NH4CO2NH2(s)  2NH3(g) + CO2(g)

initially : 1 0 0
at equilibrium (1 ă x) 2x x
total mol of gaseous substance at equilibrium = 2x + x = 3x
NH3 CO2

2 1
 mol fraction :
3 3

2 1
partial pressure : P P
3 3

3 2
2  2   P  4P
Kp = PNH3 .PCO2   P    
3   3 27

4
=  (0.116)3  2.31  10 4 atm 3
27

If CO2 is introduced after equilibrium is reached, then its final pressure will increase. But final
pressure of NH3 will decrease to keep Kp constant.

Ex a m p l e 7
The oxidation of sulphur dioxide is a reversible process : 2SO2(g) + O2(g)  2SO3(g). Calculate
the value of the equilibrium constant Kp at 1000 K from the following equilibrium partial
pressure.
PSO PO PSO
2 2 3
0.273 atm 0.402 atm 0.325 atm
If the above equilibrium was obtained by starting with a mixture of sulphur dioxide and
oxygen in a sealed vessel at 1000 K, what were the initial pressures of these two gases ?
Solution :
2SO2(g) + O2(g)  2SO3(g)

2
PSO (0.325) 2
Kp  3
  3.53atm 1
2
PSO P
2 O2
(0.273)2  0.402 

Initially there is no SO3, hence at equilibrium 0.325 atm pressure of SO3 would have been due
to 0.325 atm of SO2 and (0.325/2) atm of O2.

CHEMICAL EQUILIBRIUM
QUIZRR 31

hence initial pressure of SO2 = 0.273 + 0.325


= 0.598 atm

0.325
and initial pressure of O2 = 0.402 
2

= 0.565 atm.

Ex a m p l e 8
From the given data of equilibrium constants of the following reactions,
CoO(s) + H2(g)  Co(s) + H2O(g); K = 67

CoO(s) + CO(g)  Co(s) + CO2(g); K = 490


Calculate the equilibrium constant of the reaction,
CO2(g) + H2(g)  CO(g) + H2O(g)
Solution :
CoO(s) + H2(g)  Co(s) + H2O(g) K1 = 67 ...(1)

Now reversing the second reaction

1
Co(s) + CO2(g)  CoO(s) + CO(g) K2  ...(2)
490

Adding the two reactions, we get,


CO2(g) + H2(g)  CO(g) + H2O(g)

1
for which K = K1.K2 = 67   0.137
490

Ex a m p l e 9
Given that at 1000 K 2SO2(g) + O2(g)  2SO3(g) K = 261
Calculate K for the following equations :
(i) 2SO3(g)  2SO2(g) + O2(g)

1
(ii) SO3(g)  SO2(g) + O (g)
2 2

1
(iii) SO2(g) + O (g)  SO3(g)
2 2
Solution : Equation (i) is the reverse of the given equation

1
 K  0.0038 (for (i)).
261

CHEMICAL EQUILIBRIUM
32 QUIZRR
As the equation (ii) has been obtained by dividing the equation (i) by 2,

1
K for the equation (ii) will be  0.0038  2  0.0619 .

Reversing the equation (ii) we get the equation (iii), K for which will be

1
 16.155
0.0619

Ex a m p l e 1 0
Calculate the volume per cent of chlorine of equilibrium in PCl5 under a total pressure of
1.5 atm (Kp = 0.202).
Solution :
1 0 0 Initial moles
PCl5  PCl3 + Cl2

(1 ă x) x x Moles at equilibrium
Total moles at equilibrium = 1 ă x + x + x = (1 + x)

 x  x 
 . p  .p
PPCl3 .PCl2 1  x  1  x 
Kp  
PPCl5 1  x 
 .p
1  x 

moles of species
{partial pressure of a species =  total pressure }
total moles

x2
Kp = .p
1  x2

Substituting the values of Kp and p

x2
0.202 =  1.5; x  0.343
1  x2

 moles of Cl2 at equilibrium = 0.343


and total moles at equilibrium = 1+ x
= 1 + 0.343
= 1.343

CHEMICAL EQUILIBRIUM
QUIZRR 33

 volume percentage of chlorine = mole percent of chlorine

moles of Cl2
=  100
total moles

0.343
=  100
1.343
= 25.5%

Ex a m p l e 1 1
COCl2 gas dissociates according to the equation COCl2(g)  CO(g) + Cl2(g) when heated
to 724 K, density of the gas mixture at 101.325 kPa and at equilibrium is 1.162 dmă3. Calculate
(a) the degree of dissociation, (b) Kp , (c) GĈ for the reaction at 724 K.
Solution :
COCl2(g)  CO(g) + Cl2(g)
1 0 0 ...initially
(1 ă x) x x ...at equilibrium

1  x x x
 
1  x 1  x 1  x ...mole fraction

1  x  Px Px
P 
1 x 1  x 1  x ...partial pressure

PCO .PCl2 Px2


Thus Kp  
PCOCl2 1  x 2

RT
Also P ( = density)
m(observed)

RT
This gives observed molecular wt of COCl2 =
P

1.162  8.314  724


=  69 g mol1
101.325

Note :   density) g/dm3 = kg/m3

D  d 99  69
(a) x  = 0.435
d 69

CHEMICAL EQUILIBRIUM
34 QUIZRR

M 99
where D = vapour density of COCl2 before dissociation = 
2 2

69
d = vapour density of COCl2 after dissociation =
2
This can also be explained in terms of vanÊt Hoff factor.

m  theoretical 
i = 1 + (y ă 1) x = 1 + x =
m  observed 

99
(1 + x) =
69
(b) P = 101.325 kPa = 1 atm

1  0.435 
2
Px2
  0.233 atm
hence from (1) Kp = 1  x2
  [1  (0.435)2 ]

(c) GĈ = ă 2.303 RT log Kp = ă 2.303 ï 8.314 ï 724 log 0.233


= 8770 J mol
Alternatively we can compute value of x as
total mol of gaseous substances at equilibrium = (1 + x)

nRT(1  x) w RT 1  x 
hence P=  RT 1  x  =
V mV m

Pm 101.325  99
 (1 + x) =  = 1.435
RT 1.162  8.314  724
x = 0.435

Ex a m p l e 1 2
25 mL of hydrogen and 18 mL of iodine when heated in a closed container, produced 30.8
mL of HI at equilibrium. Calculate the degree of dissociation of HI at the same temperature.
Solution :
In this problem, volume of the species is proportional to their concentration. Thus
25 18 0 Initial volume
H2 + I2  2HI

(25 ă x) (18 ă x) 2x (= 30.8) Volume at equilibrium

 30.8 
or (25 ă 15.4) (18 ă 15.4) 30.8  x  2  15.4 
 

= 9.6 = 2.6

CHEMICAL EQUILIBRIUM
QUIZRR 35

Kc 
HI2  30.82  38.01
H2 I2  9.6  2.6
Now, if the dissociation of HI is carried out at the same temperature then for the reaction having
the degree of dissociation, x´, we have,
1 0 0 Initial moles
2HI  H2 + I2

x´ x´
(1 ă x´) Moles at equilibrium
2 2

1 1
Equilibrium constant K´ c = K  38.01 ...(Eqn. 5)
c

 x´  x´ 
  
 2  2   1
K´ c =
1  x´ 2 38.01

x´ = 0.245

Ex a m p l e 1 3
Variation of equilibrium constant K with temperature T is given by vanÊt Hoff eqn. log

H 
K = log A ă . A graph between log K and Tă1 was a straight line as shown (Fig.)
2.303 RT
P
and having  = tană1 (0.5) and OP =10. Calculate
(a) HĈ (standard heat of reaction) when T = 298 K
log K

(b) A (pre-exponential factor) 


(c) equilibrium constant K, at 298 K
(d) K at 798 K if HĈ is independent of temp. O T
ă1

Solution :
(a) Above eqn. represents a straight line of

H
slope =    tan   0.5
2.303R

 HĈ = 2.303 ï 8.314 ï 0.5 = 9.574 J molă1

CHEMICAL EQUILIBRIUM
36 QUIZRR
(b) also intercept, log A = OP = 10
 A = 1010

H
(c) log K = log A 
2.303 RT

9.574
= 10 
2.303  8.314  298

= 10 ă 1.68 ï 10ă3
K = 9.96 ï 109
(d) Putting values calculated in (a) and (c) into eqn.

2.303 RT1 T2 K
HĈ = log 2
T2  T1 K1

2.303  8.314  298  798 K2


log
we have 9.574 =  798  298  
9.96  109 
 K2 (equilibrium constant at 798 K) = 9.98 ï 109

Ex a m p l e 1 4
In the preparation of MgO, the reaction is
MgCO3(s)  MgO(s) + CO2(g)
Experiments carried out between 850ĈC and 950ĈC led to a set of Kp values fitting an
empirical equation
8500
log Kp = 7.310 ă
T
if the reaction is carried out in quiet air, what temperature would be predicted from the
equation for the complete decomposition of the MgCO3 ?
Solution :
If MgCO3 can decompose to yield CO2 at 1.00 atm (to push back the air), it will do so.
Kp = PCO2 = 1 atm

8500
 log Kp = log 1= 0.00 = 7.310 ă
T

8500
 T = = 1163 K = 890ĈC
7.310

CHEMICAL EQUILIBRIUM
QUIZRR 37

Ex a m p l e 1 5
For the reaction at 298 K A(g) + B(g)  C(g) + D(g)
HĈ = ă 29.8 kcal, SĈ = ă 0.100 kcal Kă1
Calculate GĈ and equilibrium constant.
Solution :
We know GĈ = HĈ ă T SĈ
= ă 29.8 ă 298 ï 0.1 = 0
since GĈ = 2.303 RT log K
hence log K = 0 which gives K = 1

Ex a m p l e 1 6
For the reaction CO(g) + 2H2(g)  CH3OH(g), hydrogen gas is introduced into a five-litre
flask at 327ĈC, containing 0.2 mole of CO (g) and a catalyst, until the pressure is 4.92 atm.
At this point 0.1 mole of CH3OH is formed. Calculate the equilibrium constants Kp and Kc .
(IIT 1990)
Solution :
Suppose x moles of H2 are introduced into the flask
0.2 x 0 Initial mole
CO + 2H2  CH3OH
(0.2 ă 0.1) (x ă 0.2) 0.1 Moles at eqb.

 0.1   x  0.2   0.1 


 5  0.02   5   5  0.02  Concentration at eqb.
     
Total moles = 0.1 + x ă 0.2 + 0.1 = x

 0.1 
PCH3OH  x  4.92 
  
2 2
 Kp = PCO  PH  0.1  x  0.2 
2
  4.92   4.9 
 x  x 

2
 x 
or Kp =   ...(1)
 ( x  0.2)4.9 
Further,

Kc =
CH3OH 
0.02  5 

2
...(2)

COH2 2  x  0.2 
0.02  
2
 x  0.2 

 5 

CHEMICAL EQUILIBRIUM
38 QUIZRR
Applying Kp = Kc (RT)n

Kp
= (RT)n = (0.0821 ï 600)ă2 ...(3)
Kc

From (1) and (2), we have,

2 2
Kp  x   x  0.2 2  x 
=       ...(4)
Kc   x  0.2  4.92   5   4.92  5 

From (3) and (4), we have

 x  2
    0.821  600  ; x  0.5
 4.92  5 

Substituting x in (1) and (2), we get


Kp = 0.1147 atmă2
Kc = 277.78 (moles/litre)ă2

Ex a m p l e 1 7
For the equilibrium CO(g) + H2O(g)  CO2(g) + H2(g) the standard enthalpy and entropy
changes at 300 K and 1200 K for the forward reaction are as follows :
HĈ 300K = ă 41.16 kJ molă1 SĈ 300K = ă 0.0424 kJ molă1
HĈ 1200K = ă 32.93 kJ molă1 SĈ 1200K = ă 0.0296 kJ molă1
In which direction will the reaction be spontaneous ?
(a) at 300 K and (b) at 1200 K, when PCO = PCO = PH = PH = 1 atm
2 2 2O
Also calculate Kp for the reaction at each temperature.
Solution :
Using the relationship, GĈ = HĈ ă T  SĈ, we have
GĈ 300K = HĈ 300K ă T  SĈ 300K
= ă 41.16 ă 300 (ă 0.0424) = ă 28.44 kJ molă1
and GĈ 1200K = HĈ 1200K ă T  SĈ 1200K
= ă 32.93 ă 1200 (ă 0.0296) = 2.59 kJ molă1
we know that a process is spontaneous if GĈ = ă ve, therefore, the reaction
CO(g) + H2O(g)  CO2(g) + H2(g)
is spontaneous at 300 K, whereas the reverse reaction is spontaneous at 1200 K.
also GĈ = ă 2.303 RT log Kp

CHEMICAL EQUILIBRIUM
QUIZRR 39
hence at 300 K
ă 28.44 = ă 2.303 ï 8.314 ï 10ă3 ï 300 log Kp
this gives Kp = 8.8 ï 104
similarly Kp (1200 K) = 0.77

Ex a m p l e 1 8
A solution is prepared having these initial concentration : [Fe3+] = [Hg22+] = 0.50 M; [Fe2+]
= Hg2+] = 0.03 M. The following reaction occurs among these ions at 25ĈC.
2Fe3+(aq) + Hg22+(aq)  2Fe2+(aq) + 2Hg2+, Kc = 9.14 ï 10ă6

What will be the ionic concentration when equilibrium is established ?


Solution :
Since all reactants and products are present initially, we do not know whether a net reaction will
occur Âto the rightÊ or Âto the leftÊ. This is where the reaction quotient Q can help us :

2 2
Fe2  Hg 2 
   0.03  (0.03)  6.48  106
2 2
  
Q = 2
(0.50)2 (0.50)
Fe3   Hg 22 
   

Since Q (6.48 ï 10ă6) is smaller than Kc (9.14 ï 10ă6), a net reaction must proceed to the right.
2Fe3+ + Hg 22+  2Fe2+ + 2Hg 2+

initial mole : 0.50 M 0.50 M 0.03 M 0.03 M

x
change conc. : ăxM  M +xM +xM
2

 x
equilibrium conc : (0.50 ă x) M  0.50  2  M (0.03 + x) M (0.03 + x) M
 

(0.03  x)2 (0.03  x) 2


 Kc = 2
 9.14  10 6
 x
(0.50  x)2  0.50  
 2

Since Kc is very small, x would be <<< 1

 x
hence (0.50 ă x)  0.50,  0.50    0.50
 2

CHEMICAL EQUILIBRIUM
40 QUIZRR

(0.03  x) 2 (0.03  x) 2
hence Kc = 2
 9.14  10 6
(0.50) (0.50)

(0.03 + x)4 = 1.1425 ï 10ă6


 (0.03 + x)2 = 1.07 ï 10ă3
(0.03 + x) = 3.27 ï 10ă2
 x = 2.7 ï 10ă3
hence ionic concentrations of
[Fe2+] = 0.03 + x = 0.03 + 2.7 ï 10ă3 = 3.27 ï 10ă2 M
[Hg2+] = 0.03 + x= 0.03 + 2.7 ï 10ă3 = 3.27 ï 10ă2 M
[Fe3+] = 0.50 ă x = 0.50 ă 2.7 ï 10ă3 = 4.973 ï 10ă1 M

x
[Fe22+] = 0.50 ă = 0.50 ă 1.4 ï 10ă3 = 4.986 ï 10ă1 M
2

Ex a m p l e 1 9
For the equilibrium :
CaCO3(s)  CaO(s) + CO2(g)

H0f (CaCO3) = ă 1207.1 kJ/mole, H0f (CaO) = ă 635.5 kJ/mole

H0f (CO2) = ă 393.5 kJ/mole

(i) How would Kp depend on temperature ?


(ii) The equilibrium constant for this reaction is much less than 1. Why, then, does heating
CaCO3(s) in an open container lead to a complete concentration to the products ?
Solution :

(a) H 0 = [ H0f (CaO) + H0f (CO2)] ă H0f (CaCO3)

= [ă 635.5 + (ă 393.5)] ă (ă 1207.1)


= + 178.1 kJ.
Because the given reaction is endothermic, the equilibrium constant will increase with
increasing temperature.
(b) On heating CaCO3, Kp increases which favours the dissociation of CaCO3. Also, in an open
container, CO2 gas escapes and more CaCO3 dissociates to replace it until no more CaCO3
remains.

CHEMICAL EQUILIBRIUM
THERMOCHEMISTRY
QUIZRR 3

THERMODYNAMICS

Introduction

The branch of chemistry, which deals with energy changes during various physical process and
and chemical reactions, is called chemical energetics.
All chemical reactions are normally accompanied by energy changes. These energy changes take
place because energy is needed to break chemical bonds, and energy is released when new bonds
are formed. In the process of chemical reaction, rearrangement of atoms in reactants take place,
to give products.

Important Thermodynamic Terms

Some common terms, which are frequently used in the discussion of energetics need to be known.

System

ÂA specified part of the universe which is under investigation is called the systemÊ. The system is
separated from the rest of the universe by a definite (real or imaginary) boundary.

Types of Systems

Systems are of various types depending upon the exchange of mass and energy and the constituent
between the system and the surroundings.

Isolated system

A system, which can neither exchange mass nor energy with surroundings, is called an isolated
system. They are also called insulated systems. Hot coffee (in contact with its vapour) in a closed
and insulated thermos is an example of isolated system. Since the vessel is closed, matter can
neither enter nor leave the vessel. Moreover, as the vessel is well insulated, heat can neither leave
the system nor enter from the surroundings.

Closed system

A system, which can exchange energy but not mass with the surroundings is called a closed
system. For example, boiling water in a closed steel vessel is an example of a closed system. The
energy can be gained or lost (through the steel walls) but not matter. Similarly, all reactions
carried out in a closed container are examples of closed systems.

Open system

A system, which can exchange matter as well as energy with the surroundings is called an open
system. All reactions carried out in open containers are examples of open system. Evaporation of

THERMOCHEMISTRY
4 QUIZRR
water in a beaker or hot coffee in a cup represents an open system. Here vapour of water or coffee
(matter) can leave the system and escape into atmosphere. The heat energy required for this
purpose is absorbed from the surroundings. All physical and chemical processes taking place in
open in our daily life are open systems because they are continuously exchanging matter and
energy with the surroundings.
Ć A system is said to be macroscopic when it consists of a large number of molecules, atoms
or ions. Properties associated with a macroscopic system are called macroscopic properties;
pressure, volume, temperature, mass, composition, surface area etc. are macroscopic properties.
Ć A system is homogeneous when it is completely uniform throughoutăa system with one
phase, pure gas, pure liquid or pure solid. A system is heterogeneous when it is not uniform
throughoutăa system with two or more phases.
ă gaseous mixture is always one-phase system
ă every solid is in different phase.
2NH3(g)  3H2(g) + N2(g)
one-phase system

CaCO3(s)  CaO (s) + CO2(g)


three-phase system

SURROUNDINGS

Matter Matter ï
OPEN ISOLATED
Energy(Work/Heat) Energy ï

SYSTEM

ï Matter Work ADIABATIC


CLOSED Heat/Work

Different type of systems


State Functions

We use the term state function to describe any property of a system


Enthalpy

that does not depend on how the system gets to the state exhibiting B
that property. The change in the state function depends on the initial I
and final values and not on the manner or path adopted to bring II
about that change. We move A to B by path I as well as by path II. A
If heat content (enthalpy) is a state function then
Progress
H1 = H2 = HB ă HA
Internal energy, entropy, free energies, etc., are state functions or state of a system.

THERMOCHEMISTRY
QUIZRR 5

State Variables

When there is any change in the macroscopic property, state of the system also changes, hence
macroscopic properties are also called state variables. For an ideal gas PV = RT (R is gas constant
and P, V and T are state variables.) If P and T are known, then V can be calculated. The two
variables, generally specified, are temperature and pressureăcalled independent variables. The
third variable generally volume is called dependent variable.
Ć Thermodynamic state of a system consisting of a single gaseous substance may be completely
defined by specifying any two of the three variablesătemperature, pressure and volume.
Ć A system in a state of thermodynamic equilibrium is one in which the macroscopic properties
do not change with time.

Extensive and Intensive Properties

Those variables whose values on division remain the same in any part of the system are called
intensive variables, e.g., temperature, pressure, concentration (molarity, normally), density, dipole
moment, viscosity, refractive index, pH, surface tension, molar volume, gas constant, specific heat
capacity, vapour pressure, specific gravity, emf of a dry cell, etc.

100g H2O 50g H2O in each


100mL H2O 50mL H2O in each
b.p. 100ĈC b.p. 100ĈC
f.p. 0Ĉ C f.p. 0Ĉ C

Those variables whose values in any part of the divided systems are different from the entire
system are called extensive variables, e.g., volume, energy, heat capacity, enthalpy, entropy, free
energy, length and mass.
mass changes on division ă an extensive property
volume changes on division ă an extensive property
b.p. and f.p. are same even on division ă intensive properties

Energy Terms in Thermodynamics

Every system has a definite amount of energy. It can exchange energy (lose or gain) with the
surroundings in a variety of ways. In chemical systems the two important modes of transference
of energy between the system and the surroundings are heat and work.

THERMOCHEMISTRY
6 QUIZRR
Heat (Q)

Energy exchanged between the system and the surroundings when they are at different
temperatures is commonly known as heat. If a system is at a higher temperature than the
surroundings, then there is a flow of heat (or energy) from system to surroundings, causing a
decline of the systems temperature and an increase in the surrounding temperature. These
process continue till the fall in temperature of the system and rise in temperature of the surroundings,
become equal.

Surroundings Surroundings
Heat Heat

Heat System colder Heat System colder


than surroundings Heat than surroundings Heat

Heat Heat
(a) Temprature of system decreases (b) Temprature of system increases

Work (W)

Work is another mode of transference of energy. Work is said to be done if the point of application
of force undergoes displacement in the direction of the force. If the system loses energy, we say
that the work is done by the system. Alternatively, if the system gains energy, we say that work
is done on the system.
For example, if a gas, enclosed in a cylinder with a piston, has a higher pressure than the
surroundings, the piston will move upward until the pressure inside and outside become equal.
The gas expands against a constant external pressure ÂPÊ and its volume changes by an amount
equal to V. The energy transfer that takes in this case is called work. At this step the work is
done by the system on the surroundings. This is given as :
Work done by the system = PV.
Alternatively, if the system is at lower pressure, piston will be pushed down until the pressure of
the system becomes equal to that of the surroundings. In this case work is done on the system
by the surroundings.
Obviously, if there is no change in volume, i.e.,
V = 0, no work is done by the system, i.e. work = 0.
In addition to these two modes, radiant energy and electrical energy are also modes of transference
of energy between the system and the surroundings.

THERMOCHEMISTRY
QUIZRR 7

Units of heat and work

The heat changes are measured in calories (cal), kilo calories (kcal), joules (J) or kilojoules (kJ).
These are related as :
1 cal = 4.184 J
1 kcal = 4.184 kJ
The S.I. unit of heat is joule or kilojoule
Work is measured in terms of ergs or joules.
1 Joule = 107 ergs
1 calorie = 4.184 ï 107 ergs
The S.I. unit of work is joule.
Sign Conventions for Heat and Work
The signs of ÂwÊ and ÂqÊ are related to the internal energy change. When ÂwÊ or ÂqÊ is positive, it
means that energy has been supplied to the system as work or as heat. The internal energy of
the system in such a case increases. On the other hand, if ÂwÊ or ÂqÊ is negative, it means that
energy has left the system as work or heat. The internal energy of the system decreases. The signs
of ÂqÊ and ÂwÊ are :
Heat absorbed by the system = q positive
Heat evolved by the system = q negative
Work done on the system = w positive
Work done by the system = w negative

Internal Energy

Every substance possesses a definite amount of energy which depends upon factors
such as composition of the substance, temperature and pressure. This is called intrinsic
or internal energy (E).
E = ET + ER + EV + EB + EE .........
where ET = Translational energy, ER = Rotational energy, EV = Vibrational energy, EB = Bonding
energy and EE = Electronic energy.
Internal energy of a system is state function and its absolute value can not be calculated but its
change can be determined. When the system undergoes a change in its state, the internal energy
change is given by
E = E2 ă E1
where E1 = Internal energy in the initial state and E2 = Internal energy in the final state.
In a chemical reaction, E = EP ă ER
where EP = Internal energy of products and ER = Internal energy of reactants.

THERMOCHEMISTRY
8 QUIZRR
Enthalpy (H)

Heat content of a system at constant pressure is called enthalpy. It is related with


internal energy by the equation,
H = E + PV.
Every substance has a definite value of enthalpy in a particular state. It is also a state function
like internal energy. The change in enthalpy accompanying a process can be determined as
H = H2 ă H1
where H1 = Enthalpy of a substance in the initial state and H2 = Enthalpy of a substance in the
final state.
In the case of a chemical reaction, H = HP ă HR
where HP = Enthalpy of products and HR = Enthalpy of reactants.

Relation Between H and E

Let us consider a chemical reaction, X  Y


then at constant pressure (P), the enthalpy change would be given as H = Hy ă Hx
we know that, H = E + PV
Let Ex and Vx be the internal energy and volume of the reactants and Ey and Vy be the respective
values of products. Then
Hx = Ex + PVx & Hy = Ey + PVy
H = (Ey + PVy) ă (Ex + PVx)
H = (Ey ă Ex) + P(Vy ă Vx)
H = E + PV ...(i)

Thermodynamic Equilibrium

A system in which the macroscopic properties do not undergo any change with time is said to be
in thermodynamic equilibrium.
Ć If there is no flow of heat from one portion of the system to another then system is said to
be in thermal equilibrium. This is only possible if temperature remains constant throughout
in all parts of the system.
Ć These are expressed by zeroth law of thermodynamics.
Ć Two objects at different temperatures in thermal contact with each other tend to move
towards the same temperature.
Ć Two objects in thermal equilibrium with the third one are in thermal equilibrium with each
other.

THERMOCHEMISTRY
QUIZRR 9

Ć If no mechanical work is done by one part of the system on another part of the system then
system is said to be in mechanical equilibrium. This is only possible if pressure remains
same throughout in all parts of the system.
Ć If composition of the various phases in the system remains the same throughout then system
is said to be in chemical equilibrium.

P rocess

The operation by which a system changes from one state to another is called a process. A process
is accompanied by change in energy (and matter also if system is open).
Ć If a process is taking place at constant temperature, it is said to be isothermal (iso-same,
therm-heat, temperature) process.
Ć If a process is taking place at constant pressure, it is said to be isobaric process.
Ć If a process is taking place at constant volume, it is said to be isochoric process.
Ć If no heat enters or leaves the system during any step of the process, it is said to be adiabatic
process. P
A(V1,T1) q1 B(V2,T1)

ăq2
C(V3,T2)
D(V4,T2)

V
In this types of processes, temperature changes because the system is not in a position to exchange
heat with the surroundings. If there is any type of expansion (due to increase in volume), it is
done at the expense of internal energy which therefore decreases hence temperature may fall. If
there is any type of contraction, internal energy is released, which therefore, increases and
temperature may increase.

Process Temperature Volume Heat exchange Type

A to B T1 constant V2 > V1 + q1 isothermal expansion


B to C T2 < T1 V3 > V1 0 adiabatic expansion,
temperature falls
C to D T2 constant V4 < V3 ă q1 isothermal compression
D to A T2 < T1 V1 < V4 0 adiabatic compression
temperature increases

THERMOCHEMISTRY
10 QUIZRR
In the above example system has undergone a series of changes and ultimately comes back to the
initial state. Such type of process is called cyclic process.
Ć If a process is carried out in such a way that the system remains virtually in a state of
equilibrium, it is saidto be quasi-static (or reversible) process. The overall change in a state
function is zero when the system returns to its original condition.

Example 1

Assuming ideal behaviour, calculate the work done when 1.6 mole of water evaporates at
373 K against the atmospheric pressure of 760 mm of Hg.
Solution :
Volume of 1.6 mole of water at 373 K in gaseous state

nRT 1.6  0.082  373


=   48.93
P 1

Volume of 1 mol = 18 g of liquid water (density = 1 g mlă1)


= 18 ï 1.6 ï 10ă3 L = 0.0288 L
Now work done (W) = ă P(V2 ă V1)
= ă 1(48.93 ă 0.0288) = ă 48.90 atm L
= ă 48.90 ï 101.325 J = ă 4954.8 J

Example 2

A system absorbs 470 J of heat and does work equivalent to 200 J on its surroundings.
Calculate the change in internal energy.
Solution :
Heat absorbed by the system = 470 J or q = 470 J
Work done by the system = 200 J or w = ă 200 J
According to first law of thermodynamics
E = q + w
E = 470 ă 200 = 270 J

Example 3

A gas absorbs 120 J of heat and expands against the external pressure of 1.10 atm from a
volume of 0.5 L to 2.0 L. What is the change in internal energy ? (1 L atm = 101.3 J)
Solution :
Work of expansion, w = PV
V = 2.0 ă 0.5 = 1.5 L, P = 836 mm of Hg

THERMOCHEMISTRY
QUIZRR 11

PV = 1.10 ï 1.5


= 1.650 L atm
= 1.650 ï 101.3 (1 L atm = 101.3 J)
= 167.1 J
Since work is done by the system w = ă 167.1 J
Heat absorbed by the system = 120 J or q = + 120 J
Now, E = q + w
= 120 J + (ă 167.1) = ă 47.1 J

E nthalpy Changes

Generally, the amount of heat, q that a system absorbs or evolves as it changes is not a state
function; it varies with the method the process has adopted. If the change occurs in such a way
that the only work done is to affect a change in volume of the system at constant pressure, then
q reflects a change in a state function of the system called enthalpy, H defined by equation
H = E + PV
We cannot determine the enthalpy of a system but we can measure change in enthalpy as the
system changes.
H = E + PV
H = E ă (ăPV)
H = E ă W
E = H + W
where W = ă PV = ă P (V2 ă V1)
Like E, H is also a state function as its change, H, is independent of the path adopted.
Ć Enthalpy of the element (in gaseous, liquid, solid or combined as H2) is zero
If PV = nRT for n moles of an ideal gas
P (V + V) = (n + ng)RT for (n + ng) moles of an ideal gas
 PV + PV = nRT + ng RT
 PV = ngRT
 H = E + ngRT = q
H is also called heat of reaction at constant pressure and E the heat of reaction at
constant volume, ng = coefficient of gaseous products ă coefficient of gaseous reactants
At constant volume (V = constant)
V = 0
 P V = ng RT = 0
 H = E = q

THERMOCHEMISTRY
12 QUIZRR
Under standard states when temperature is 298 K and pressure is 1 atm and all the species
present are at unit activity.
HĈ = EĈ + ng RT
Better way to specify standard states is to write temperature also
HĈ 298K = HĈ 298 K
+ ngR (298 K)
and HĈ, etc., is to specify the temperature other than 298 K but at 1 atm and all the species
at unit activity.

Heat Capacity

Heat capacity gives a measure of the quantity of heat absorbed by a system. It is defined as the
amount of heat required to rise the temperature of the system through 1ĈC. If the quantity of
heat required is to raise the temperature of unit mass of the substance through 1ĈC then it the
specific heat of the substance. Whereas when the whole mass of the substance is considered, it
is the heat capacity.
If ÂqÊ is the heat required by the system to raise its temperature from T1 to T2, mathematically
the heat capacity C of the system is given by the expression :

q q
C or
T2  T1 T

The value of ÂCÊ is considered over a small temperature range as heat capacity varies with temperature.
If q is the heat required to raise the temperature of the system from T to T + dT, then

q
C
dT

Heat capacity is path function and so the conditions such as constant volume or constant pressure
have to be be specified to define the path for calculating the heat capacity of a system.
The heat capacity at constant volume is represented by Cv and that at constant pressure is
represented by Cp.

Significance of Cv and Cp

For infinitesimally small changes, the expression for the first law can be written as dE = q + w
= q ă PdV
q = dE + PdV

At constant volume; At constant pressure;


or dV = 0 dV  0

THERMOCHEMISTRY
QUIZRR 13

q(v) = dE

q(v)
Now, Cv  Also H = E + PV
dT

 dH = dE + PdV

dE  E 
or Cv  or  T  or dE = dH ă PdV
dT  v

Substituting in (i)

 q( p)  d H  Pd V  PdV  d H

q( p) dH  H 
Cp   or  T 
dT dT  p

Thus, heat capacity at constant volume represents the rate of increase of internal energy of the
system with temperature, while the heat capacity at constant pressure represents the rate of
increase of enthalpy of the system with temperature.

Molar heat capacities

The heat capacities of 1 mole of a gaseous system at constant volume and at constant pressure
are called molar heat capacities.

Relationship between Cp and Cv

When the temperature of 1 mol of ideal gas is to be raised through one degree (1 K) the gas is
heated at constant volume. The gas under this condition does no external work and the heat
supplied increases the internal energy of the gas. When the gas is heated at constant pressure,
there will be an increase in its volume. The gas will expand and do some external work. Extra
heat must be supplied to the gas to enable it to perform this external work. Thus, heat capacity
at constant pressure (Cp) is larger than that at constant volume (Cv). The difference between the
two i.e., (Cp ă Cv), gives the work done by
1 mol of the gas in expansion when it is heated through one degree. This represents the work
done per mol per degree rise in temperature, i.e., R the universal gas constant. Thus,
Cp ă Cv = R
The above conclusion can be arrived mathematically as follows :

dH d E
C p  Cv   .......... (i)
dT d T

THERMOCHEMISTRY
14 QUIZRR
Also H = E + PV
For an ideal gas, PV = RT
Therefore H = E + RT

dH d E
Differentiation with respect to T   R .1
dT d T

dH dE
or  R ........ (ii)
dT d T

Comparing (i) and (ii) Cp ă Cv = R for 1 mole of ideal gas


The above relation is also called MeyerÊs relationship. It can also be written as Cp ă Cv = R = 1.99
cal Kă1 molă1 = 8.314 JKă1 molă1.

The Ratio Cp/Cv

The ratio of molar heat capacities at constant pressure to that at constant volume is represented
by . The value of  gives information about the atomicity of gases.

Cp

Cv

For monoatomic gases  = 1.67


For diatomic gases  = 1.40
For triatomic gases = 1.30

Example 4

A gas expands from a volume of 3.0 dm3 to 5.0 dm3 against constant external pressure of
3.0 atm. The work of expansion is used to heat 10.0 mol of water at temperature 290 K.
Calculate final temperature of water. (Specific heat of water = 4.184 JKă1 gă1)
Solution :
W = ă P (V2 ă V1)
= ă 3 ï (5 ă 3) = ă 6 L atm = ă 6 ï 101.25 J = 6.07 ï 102 J
Now, Q = m. C.T

Q 6.027  102 J
T    0.8 K
mC 10 ï 18(g) ï 4.184 (J g 1 K 1 )

Final temperature = T + T = 290 + 0.8 = 290.8 K

THERMOCHEMISTRY
QUIZRR 15

PRESSURE–VOLUME WORK
Isothermal Expansion
No heat flows out of or into the system in an isothermal expansion, so that temperature remains
constant throughout the process of expansion. Since for an ideal gas, the internal energy (E),
depends only on temperature, the internal energy of the gas remains constant.
i.e. E = 0
From Ist law of thermodynamics,
E = q + w
For isothermal process,
E = 0, hence q = ă w
In the case of isothermal expansion, work is done by the system at the cost of heat absorbed. The
magnitude of ÂqÊ or ÂwÊ depends on the manner in which the process of expansion is carried out
i.e. reversibly or irreversibly.
H can be calculated as follows
We know, H = E + ngRT
For isothermal process, E = 0 & T = 0 thus, H = 0

Work done in irreversible isothermal process


Suppose an ideal gas expands against external pressure P and its volume changes by an amount
dV then work done W can be given by
W = ă PdV
(a) Free expansion : W = 0, Since P = 0
(b) Expansion or compression against external pressure P
For a finite change V1 to V2,
Total work done on the system W is derived by
W = ă P (V2 ă V1)
or Wirr = ă P (V2 ă V1) ...(3)
Also U = H = 0 ...(4)
If V2 > V1 then Wirr is ă ve, i.e., expansion work or work done by the system.
The total work done during expansion of gas from V1 to V2

v2 v2 nRT
 dW = 
v1
 P dV 

v1

V
. dV

V2 V2
Wrev = ă nRT loge V = ă 2.303 nRT log10 V ...(5)
1 1

( P  1/V)

P1
Also Wrev = ă 2.303 nRT log10 P ...(6)
2

THERMOCHEMISTRY
16 QUIZRR
A diabatic Expansion

From first law of thermodynamics, E = q + w


In an adiabatic expansion, q = 0,  E = w

 dE 
The molar heat capacity at constant volume of an ideal gas is given by Cv   
 d T v

or dE = Cv. dT
and for finite changes, E = Cv ï T = w ...(x)
The value of T depends upon the nature of process (i.e. reversible or irreversible).

Work done in adiabatic reversible process

Consider a system under adiabatic conditions, showing reversible expansion of an ideal gas by a
volume dV, then from I law of thermodynamics,
..
( . dq = 0)
+ dU = dW
 dU = nCv dT
or + Cv.n ï dT = dW
This equation reveals on further treatment within temperature limits of T1 and T2, reveals
dW = Cv ï n ï dT
Cp ă Cv = R

Cp Cv R
 = C
Cv Cv v

R
Cv =
(   1)

R
Therefore, dW = + (   1)  n  d T

nR
dW = . dT
 1

nR T2
on integration  dW =
 1  T1
dT

nR
wrev =   1 [T2  T1 ] ...(7)

THERMOCHEMISTRY
QUIZRR 17

if T2 > T1 then Wrev = + ve, i.e., work done on the system


if T2 < T1 then Wrev = ă ve, i.e., work done by the system

Cp
where  is PoissonÊs ratio = ...(8)
Cv

Also H = nCp (T2 ă T1) ...(9)


If final temperature is now known :

 P 
R / Cp 
w = ă nCvT1 1   2   ...(10)
  P1  
 

Some important results of adiabatic expansions

1. PV  = constant ...(11)
2. T P1ă = constant ...(12)
ă1
3. V T = constant ...(13)

Work done in adiabatic irreversible expansion or compression

(a) Expansion against vacuum or free expansion


w = 0
(b) Expansion or compression against external pressure :

 P T  P2 T1 
w   Pext  R  1 2  ...(14)
 P1 P2 

Example 5

An insulated container is divided into two equal portions. One portion contains an ideal
gas at pressure P and temperature T. The other portion is a perfect vacuum. If a hole is
opened between two portions, calculate :
(a) the change in internal energy
(b) the change in temperature of gas.
Solution :
The system being thermally insulated and thus q = 0. The gas expands through hole in other
portion to show free expansion, i.e.


w =  PV  0 (Since P = 0)

THERMOCHEMISTRY
18 QUIZRR
Also from I law of thermodynamics
q = U + w
Since, q = 0; w = 0  U = 0

Also Internal energy U = 3 RT . Since, internal energy remains same


2

( U = 0) and thus temperature will also remain constant.

Example 6

2.8 g of N2 gas at 300 K and 20 atm was allowed to expand isothermally against a constant
external pressure of 1 atm. Calculate U, q and W for the gas.
Solution :
Initially for N2

2.8
20 ï V1 =  0.0821  300
28

 V1 = 0.123 litre
Finally for N2

2.8
1 ï V2 =  0.0821  300
28

 V2 = 2.463 litre
 W = ă P ï V [ work is done against constant P  irreversible]
= ă 1 ï (2.463 ă 0.123)
= ă 2.340 litre atm

2.340  1.987
=  calorie
0.0821

2.340  1.987  4.184


=  J
0.0821

W = ă 236.95 J
Now q = U ă W
q = 0 + 236.95 ( U = 0 for isothermal process)
q = 0 236.95 Joule

THERMOCHEMISTRY
QUIZRR 19

SECOND LAW OF THERMODYNAMICS


Spontaneous Process

In any system, a spontaneous process is one which occurs in its own without the help of
external energy. The natural changes that occur around us result for such processes. For
example, flow of heat from a body to a cold one, flow of water downhill, and the expansion
of a gas from high pressure to low pressure. Besides being spontaneous, these processes are
also unidirectional i.e., they occur own in one direction only. A spontaneous process cannot be
reversed without the aid of external work or energy. The work has to be done to move uphill
or to compress a gas.
Many common chemical reactions are spontaneous and occur in one direction although finally a
state of equilibrium is reached. Everything in nature have a tendency to move towards a state
of lowest energy, at which point equilibrium exists. The heat energy which is given out during
an exothermic reaction shows that there is a lowering in chemical energy on proceeding from the
reactants to the products. From this, we may conclude that a reduction in enthalpy provides the
necessary driving force for a reaction to occur. However, many endothermic reactions also proceed
on their own and so it looks as though a negative enthalpy change is not a reliable criterion to
judge the spontaneity of chemical reactions.
The first law of thermodynamics does not state whether a reaction is spontaneous or not and in
which direction it will occur. For example, let us take a metal bar, which is heated at one end.
Assuming no loss of heat from the bar, when heat passes from the hot end to the cold end, the
decrease in thermal energy at the hot end is equal to the increase at the cold end. This satisfies
the requirements of the law. From experience, we know that heat will not flow, on its own from
the cold end to the hot end. However, the I law does not exclude the occurrence of such non-
spontaneous processes, but only tells us that an energy decrease in one part of the system is equal
to an increase in another part.

Statement of the II Law

Second law of thermodynamics is concerned with the direction and spontaneity of processes. There
are many ways of formulating the law based on our experience of the direction or manner in
which natural processes occur. Clausius stated the law as „The transference of heat from a cold
to a hot body cannot be achieved without the performance of work.‰
Because natural or spontaneous processes occur over a period of time, they must be
thermodynamically irreversible. Another way of formulating the second law is ÂAny process occurring
on its own is thermodynamically irreversibleÊ.
The basic concept of the second law of thermodynamics is that all spontaneous processes are
unidirectional and thermodynamically irreversible.

THERMOCHEMISTRY
20 QUIZRR
E ntropy

Entropy (S) is a state function and is a measure of degree of disorder or randomness of system.
More is the disorder in system, higher is its entropy. It is normally expressed in terms of change
in entropy.
1. For a reaction entropy change (S) is given by
S = SProducts ă SReactants ...(18)

V2
nRT log e
2. S = qrev  Wrev  V1
 nR log e
V2
T T T V1

V2
= 2.303 nR log10 V ...(19)
1

P1
= 2.303 nR log10 P ...(20)
2

where, qrev is the heat supplied to a system at temperature T (K) under reversible conditions.
3. At constant P, qrev = Hrev

Hrev
S = ...(21)
T

i.e., entropy change of fusion :

H f
S f = ...(22)
T

where T is freezing point


and entropy change of vaporization :

Hv
S v = ...(23)
T

where T is boiling point


4. Entropy of one mole of a substance in pure state at one atmosphere and 25ĈC is termed as
standard entropy, SĈ.

For a reaction in standard, A  B

S  SB  SA ...(24)

THERMOCHEMISTRY
QUIZRR 21

5. At absolute zero temperature, a solid is supposed to have highly ordered state,


S = 0, i.e, lim S = 0.
T0

M eaning of Entropy

Although the concept of entropy was first applied to the performance of heat engines, pictorial
representations of entropy can be obtained from two sources and lead to the following conclusions,
(a) Entropy is the degree of disorder or randomness of a system.
(b) Change in entropy is the capacity for spontaneous change in a system.

Entropy is a measure of disorderness of a system

To illustrate the meaning of disorder or randomness, consider a substance which changes state.

Increase in
degree of
randomness

(a) High (b) Low


pressure pressure
(1) Solid (2) Liquid (3) Gas

In the gaseous state and low pressure, the molecules are free to move about unrestricted. If the
pressure is increased, the same number of molecules now occupy a smaller volume. The chances
of finding a molecule within a given volume are greater under these conditions than when it was
at lower pressure. Hence, the molecules are more ordered (or less randomly distributed) at higher
pressure. Added to this, the force of intermolecular attraction increases as the pressure increases
and this too has a restricting effect on the molecules.
In the liquid state, the molecules are much closer together than in the gaseous state, because the
volume containing the same number of molecules is much smaller. The molecules in the liquid
state are more highly ordered than a gas at high pressure. Also in the liquid state, the forces of
intermolecular attraction are greater, causing a still more highly ordered state.
In this solid state, the atoms, molecules or ions that form the crystal are usually held in fixed
crystal lattices, which can only vibrate and rotate. Hence in a solid, the atoms, molecules or ions
have little or no choice as to where they shall be i.e., they are highly ordered or have a very small
freedom.

THERMOCHEMISTRY
22 QUIZRR
This gradual decreases in the measure of disorder of the molecules is identified with entropy
changes, being high for a gas and low for a solid.
Stotal = Ssystem + Ssurroundings
For a spontaneous process Stotal must be positive, i.e.,
Stotal = Ssystem + Ssurroundings > 0
When an isolated system is in equilibrium, the entropy is maximum. The mathematical condition
for entropy (S) to be maximum is that the change in entropy (S) is zero, i.e.,
S = 0 (at equilibrium for an isolated system)
In exothermic reactions, heat released by the reaction increases the disorder of the surroundings
and overall entropy change is positive. In some exothermic reactions, entropy of the system may
also decrease due to conversion of a gas into a solid product. However, if reaction is highly
exothermic and increase in entropy of the surroundings is very high, the total entropy change will
be spontaneous. Let us take oxidation of magnesium which is highly exothermic. Conversion of
oxygen into oxide results in decrease of entropy.
2Mg (s) + O2 (g)  2MgO (s)
      rSĈ = ă 217 JKă1 molă1
      rHĈ = ă 1202 kJ molă1
Heat released will increase the entropy of the surroundings and therefore

Ssurr = 
  1202  10 3
Jmol1    4.03  10
3
JK 1mol1
298 K

   Stotal = ă 217 JKă1 molă1 + 4.03 ï 103 JKă1 molă1 = + 3.81 ï 103 JKă1 molă1
Since Stotal is positive, reaction will be spontaneous.
In endothermic reactions, reactants on conversion into products go to higher energy state and
temperature of the system falls. As a consequence, heat flows from surroundings into the system,
the entropy of the surroundings decreases. If the disorder of the system rises sufficiently high and
overall entropy increase is positive, the reaction will be spontaneous.

Entropy Change during Phase Transformations

Phase changes occur at constant temperature at a given pressure and process is reversible.
For example
H2O (s)  H2O (l); fusHĈ = 6.0 kJmolă1
(takes place at 273 K at 1 bar)
H2O(l)  H2O (vap); vapHĈ = 40.79 kJmolă1
(takes place at 373 K at 1 bar)

THERMOCHEMISTRY
QUIZRR 23

When solid melts, there is an equilibrium between the solid and the liquid at the freezing or
melting point. The process involves latent heat which is equal to standard enthalpy of fusion,
fusHĈ at constant temperature and pressure. For water, fusHĈ = 6.0 kJmolă1.

qrev  fus H
fus SĈ = 
T T

6.0 kjmol1 6.0  1000 Jmol1


=   219.9 mol1K 1
273K 273 K

Similarly, when a liquid is transformed into vapour at constant pressure and temperature.

 vap H
vapSĈ = , (T is boiling point)
T

Standard enthalpy of vaporization for water is 40.79 kJmolă1

 vap H 40.79 kJmol1 40.79  1000 Jmol1


vapSĈ =  
T 373 K 373 K

= 109.356 JKă1 molă1

Example 7
The enthalpy of vaporization of benzene (C6H6) is 308 kJ molă1 at its boiling point (80.1ĈC).
Calculate the entropy change in going from (i) liquid to vapour and (ii) vapour to liquid
at 80.1ĈC.
Solution :
Vaporization of a liquid is an endothermic process and therefore enthalpy of vaporization, vapHĈ
is positive. Contrary to this, condensation (i.e., from vapour to liquid) is exothermic process and
therefore enthalpy of condensation is negative but magnitude of enthalpy in both the case is
equal. That is,
vapHĈ = condensHĈ
For vaporization of benzene,

 vap H
vapSĈ =
T

30.8 kJmol1 30.8  1000 Jmol 1


=  = 87.3 JKă1 molă1
353 K 353 K

For condensation of benzene,

 vap H
               condensSĈ = = ă 87.3 JKă1 molă1
353 K

THERMOCHEMISTRY
24 QUIZRR
A bsolute Entropy

The entropy of a perfectly crystalline substance approaches zero as the absolute zero of temperature
is approached. This is third law of thermodynamics.
Molar entropy, the entropy per unit amount of a substance, is denoted by Sm. Standard molar
entropy, the entropy per unit amount of substance in its standard state at the specified temperature

is denoted by Sm . The standard entropy of a substance or ion is also called absolute entropy. The
definition of the entropy given by equation earlier helps in determination of entropy. The unit of
molar entropy is JKă1molă1.

Entropy Change for a Reaction

Entropy change for a chemical reaction (when reactants change completely into products) is
calculated with the help of data given for standard molar entropies at a given temperature. From
the sign of the entropy of a reaction, we can give qualitative information about the reaction.
However, for quantitative work, we require standard entropy change, rS for a reaction and it
can be determined by subtracting the standard entropies of reactants from the standard entropies
of products. Thus,

r Sm = vp Sm (products) ă vr Sm (reactants) (vp and vR are stoichiometric coefficients)

For example, in a general reaction :


aA + bB  cC + dD

r Sm = [c Sm (C) + d Sm (D)] ă [a Sm (A) + b Sm (B)]

For oxidation of iron, 4Fe(s) + 3O2 (g)  2Fe2O3 (s), we can write

r Sm = 2 Sm [Fe2O3] ă [4 ï Sm (Fe) + 3 ï Sm (O2)]

= {2 ï 87.4 ă (4 ï 27.3 + 3 ï 205.0)} JKă1molă1


= ă 549.4 JKă1 molă1
As expected, there is a large decrease in entropy during the reaction, mostly because the highly
dispersed oxygen gas reacts to form the compact solid Fe2O3. Now, we would like to understand
that inspite of the negative entropy of the reaction, why is the reaction spontaneous. As already
emphasized, Stotal = (SSystem + SSurrounding) only decides the spontaneity of a reaction. In this
case, heat evolved during the reaction increases the entropy of the surroundings.
For calculating entropy changes in the surroundings, we have to consider the heat absorbed by
the surroundings which is equal to ă rHĈ. At temperature T, the entropy change of the
surroundings is

 r H q
S(surrounding) =  
T T

THERMOCHEMISTRY
QUIZRR 25

In equation (above), because ă rHĈ is positive and so r SĈ (surroundings) is also positive,


corresponding to an increase from its initial value.

(surrounding) =

  1648  103 Jmol1  (rHĈ for the reaction is ă 1648 ï 103 J molă1)
298 K

= 5530 JKă1 molă1


This high increases in entropy of the surroundings is due to release of energy (heat) into
surroundings. Thus, total entropy change for the oxidation of iron,

r Sm (total) = rSĈ (surroundings) + rSĈ (system)

= ă rHĈ/T + rSĈ (system)


= (+ 5530 JKă1molă1) + (ă 549.4 JKă1molă1)
= 4980.6 JKmolă1
This overall increase in the entropy makes the oxidation of iron 4Fe(s) + 3O2 (g)  2Fe2O3(s)
spontaneous.

Free Energies

Gibbs free energy G, is defined as G = H ă TS


where H is the enthalpy, T is the temperature in Kelvin scale, S is the entropy. TS is the part
of systemÊs energy that is already disordered and H ă TS (= G) is the part of the systemÊs energy
and that is still ordered hence, free (available) to cause spontaneous change by becoming disordered.
Ć G is also a state function hence, change in free energy (G) for a process is independent of path
Ć G = H ă TS
Also Stotal = Ssystem + Ssurroundings

H
= S 
T

H
Since,              Ssurroundings = 
T

(we write Ssystem simply as S)


T Stotal = TS ă H
or ă TS total
= H ă T S
ă T  Stotal = G
Thus, G and Stotal have opposite signs (T is always positive); hence sign of G can also be used
to predict feasibility of a reaction at constant temperature and pressure. For a reaction to be

THERMOCHEMISTRY
26 QUIZRR
spontaneous if S > 0 then G < 0
nonspontaneous if S < 0 then G > 0
in equilibrium if S = 0 then G = 0
Ć G = H ă TS

 q
= E + PV ă TS  S = T 
 

= E ă q + PV  E = q ă W by first law


G = ă W + P V
ă G = W ă P V = Wnet

V2
ă G = Wnet = 2.303 n RT log V
1

Thus, decrease in G, (ă G), is equal to net work done = (total work ă mechanical work)

Table : Signs of Enthalpy, Entropy and Free Energy Changes and


Reaction Spontaneity at Constant Temperature and Pressure

H S G = H ă TS Reaction Spontaneity

Exothermic + ă Spontaneous at all temperatures


ă ă Spontaneous at low temperatures
+ Non-spontaneous at high temperatures
Endothermic + + Non-spontaneous at low temperatures
+ ă Spontaneous at high temperatures
ă + Non-spontaneous at all temperatures
TS = H Equilibrium

For a reaction in which a compound in its standard state is formed from its elements in their
standard states, the free energy change is the standard free energy of formation, GĈ f

GĈ f = 0 for the elements,

GĈ f = (GĈ products ă GĈ )


reactants

= HĈ ă T SĈ

Ć G is an extensive property,

Ć G changes sign when a process is reversed.

THERMOCHEMISTRY
QUIZRR 27

Energy and Electromotive Force (EMF) of a cell

Using G = GĈ + 2.303 RT log Q


G = ă nFEcell
GĈ = ă nFEĈ cell
We have ă nFEcell = ă nFEĈ cell + 2.303 RT log Q

2.303 RT
Ecell = EĈ cell  log Q
nF

This is called Nernst equation for emf of a cell.


Ecell is the emf of a cell in a given state, EĈcell is the standard emf, n the number of electrons
exchanged, F the faraday of electricity, R gas constant and T the temperature. At 298 K

2.303 RT 2.303  8.314  298


=
F 96500

= 0.0591 V

0.0591
 Ecell = EĈ cell  log Q
n

When equilibrium is attained, Ecell = 0 and Q = K

00591
 EĈ cell = log K
n

Example 8

Determine whether or not it is possible for sodium to reduce aluminium oxide to aluminium
at 298 K.
GĈ f (298 K)/kJ molă1 Al2O3(s) = ă 1582
Na2O(s) = ă 377, element = 0
Solution :
The reaction is

Al2O3(s) + 6Na(s)  3Na2O(s) + 2Al(s)

hence,   GĈ = 3GĈ f(Na2O) ă GĈ f(Al2O3)


= 3 ï (ă 377) ă (ă 1582) = + 451 kJ molă1
Evidently the reaction cannot occur since GĈ (298 K) is positive.

THERMOCHEMISTRY
28 QUIZRR
Example 9

Sulphur exists in more than one solid form. The stable form at room temperature is rhombic
sulphur. But above room temperature the following reaction occurs

S(rhombic)  S(monoclinic)

If H = 276.144 J at 298 K and 1 atm and G = 75.312 J


(a) Calculate S at 298 K
(b) Assume that H and S do not vary significantly with temperature, calculate Teq, the
temperature at which rhombic and monoclinic sulphur exist in equilibrium with each
other.
Solution :
(a) Since G = H ă T  S

H  G
S =
T

276.144  75.312
= = 0.674 J Kă1
298

(b) Under equilibrium condition G = 0


 H ă TeqS = 0

H 276.144
 Teq = =
S 0.674

= 409.7 K

Example 10

Assume HĈ and SĈ to be independent of temperature, at what temperature will the reaction
given below become spontaneous ?

N2(g) + O2(g)  2NO(g)


SĈ/JKă1molă1 191.4 204.9 210.5
HĈ = 180.8 kJ molă1
Solution :
SĈ = 2SĈ NO ă SĈ N ă SĈ O
2 2
= 2 ï 210.5 ă 191.4 ă 204.9
= 24.7 J Kă1 molă1
 GĈ = HĈ ă T SĈ
GĈ = 180.8 ă (T ï 24.7 ï 10ă3)kJ molă1
THERMOCHEMISTRY
QUIZRR 29

For spontaneity, GĈ < 0 and this occurs when


T SĈ > HĈ

 H
T>
S

180.8 ï 103
T>  7320 K
24.7

The reaction becomes spontaneous above a temperature of 7320 K.

Example 11

The enthalpy change for the transition of liquid water to steam is 41 kJ molă1 at 100ĈC.
Calculate the entropy change for the process.
Solution :

H2O(l)  H2O(vap.)

L Hvap
The equation for entropy change is S = 
T T

41  1000 41000
=   110 J mol 1 K 1
(273  100) 373

Note that S is greater than zero as there is an increase in disorder (randomness) in the system.

Example 12

3 moles of N2(g) originally at 1 atm pressure are mixed isothermally with 5 moles of H2(g)
also at 1 atm pressure to yield a mixture whose total pressure is 1 atm. Assuming ideal
behaviour, calculate entropy of mixing.
Solution :
Total pressure of mixture = 1 atm

3
Final pressure of N2 = (1) = 0.375 atm
53

Final pressure of H 2 = 1 ă 0.375 = 0.625 atm


For N2, entropy change

P2  0.325 
SN = ă nR ln P = ă 2.303 ï 3 ï 8.314 ï log    28.03 J/K
2 1  1 

THERMOCHEMISTRY
30 QUIZRR
For H2, entropy change

P2 0.675
SH = ă nR ln P = ă5 ï 8.314 ï 2.303 log = 16.34 J/K
2 1 1

Total entropy change due to mixing = SN + SH = 44.37 J/K


2 2

Example 13

Calculate the entropy change when 1 kg of water is heated from 27ĈC to 200ĈC forming
super heated steam under constant pressure. Given specific heat of water = 4180 J/kg-K and
specific heat of steam = 1670 + 0.49T J/kg-K and latent heat of vaporization = 23 ï 105 J/kg.
Solution :

T2
S = 2.303 m ï Cp ï log [where m in kg and Cp in J/kg]
T1

Entropy change for heating water from 27Ĉ to 100ĈC

1000 4180  18 373


S = 2.303  log
18 1000 300

= 910.55 J
Entropy change for heating 1 kg H2O to 1 kg steam at 100ĈC

Hv 23  105
S =   6166.21J
T 373

Entropy change for heating 1 kg steam from 373 to 473 K, m in kg.

473 nC p . d T 473 (1670  0.49T) d T


S = 
373 T
m

373 T

473 1670 dT 473


= m 373 T
 m
373
0.49 d T

= m  1670  2.303  log T 373  m  0.49  T 373


473 473

473
= 1 ï 1670 ï 2.303 log + 1 ï 0.49 ï 100
373

= 396.73 + 49 = 445.73 J
 Total entropy change = 910.55 + 6166.21 + 445.73
= 7522.50 J

THERMOCHEMISTRY
QUIZRR 31

Example 14

Titanium metal is extensively used in aerospace industry because the metal imparts strength
to structures but does not unduly add to their masses. The metal is produced by the
reduction of TiCl4(l) which in turn is produced from mineral rutile TiO2(s). Can the following
reaction for production of TiCl4(l) be carried out at 25ĈC ?

TiO2(s) + 2Cl2(g)  TiCl4(l) + O2(g)

Given that Hf for TiO2(s), TiCl4(l), Cl2 (g) and O2 (g) are ă 944.7, ă 804.2, 0.0, 0.0 kJ molă1. Also

SĈ for TiO2 (g), TiCl4(l), Cl2(g) and O2(g) are 50.3, 252.3, 233.0, 205.1 J molă1 Kă1 respectively.
Solution :
HĈ for reaction

   
= HTiCl4 (l)  HO2 ( g)  HTiO2  HCl2  2

= [ă 804.2 + 0.0 ă (ă 944.7) ă 0.0]


= 140.5 kJ
Also, SĈ for reaction

   
= STiCl4 (l)  SO2 ( g)  STiO2 (s)  SCl2 ( g)  2 

= [252.3 + 205.1 ă 50.3 ă 2 ï 233.0]


= ă 58.9 J
= -.0589 kJ
Now,    GĈ = HĈ ă TSĈ
= 140.5 ă 298.15 ï (ă 0.0589)
= 158.06 kJ

Example 15

A gas expands from 3 dm3 to 5 dm3 against a constant pressure of 3 atm. The work done
during expansion is used to heat 10 mole of water of temperature 290 K. Calculate final
temperature of water. Specific heat of water = 4.184 J gă1 Kă1.
Solution :
Since work is done against constant P and thus, irreversible
V = 5 – 3 = 2 dm 3 = 2 litre; P = 3 atm
 W = ă P. V = ă 3 ï 2 litre atm

6  4.184  1.987
= ă joule = ă 607.57 joule
0.0821
THERMOCHEMISTRY
32 QUIZRR
Now this work is used up in heating water
 W = n ï C ï T
607.57 = 10 ï 4.184 ï 18 ï T
 T = 0.81
 Final temperature = T1 + T
 = 290 + 0.81
= 290.81 K

Example 16

1 mole of an ideal gas undergoes reversible isothermal expansion from an initial volume V1
to a final volume 10 V1 and does 10 kJ of work. The initial pressure was 1 ï 107 Pa .
(a) Calculate V1.
(b) If there were 2 mole of gas, what must its temperature have been ?
Solution :

V2
W = ă 2.303 nRT log V
1

(a) Where W is work done by the system under isothermal reversible conditions, note that work
done by the system is negative.

P1
ă 10 ï 103 = ă 2.303 ï 1 ï 8.314 ï T log P ...(i)
2

Also P1V1 = P2V2 at constant temperature


7
1 ï 10 ï V1 = P2 ï 10V1

1 ï 107
 P2 =  106 Pa
10
 By Eq. (i)

107
ă 10 ï 103 = ă 2.303 ï 1 ï 8.314 ï T log
106
 T = 522.27 K
Now, using PV = nRT for 1 mole of gas;
P = 1 ï 107 Pa = 107 Nmă2
1 ï 107 ï V1 = 1 ï 8.314 ï 522.27
V = 4.34 ï 10ă4 m3
(b) If 2 mole of gas have been used, the temperature would have been

522.27
 261.13 K
2

THERMOCHEMISTRY
QUIZRR 33

Example 17

One mole of an ideal monoatomic gas is taken round cyclic P


process ABCA as shown in figure. Calculate :
(a) The work done by the gas
B(T2)
(b) The heat rejected by the gas in the path CA the heat 3P0
adsorbed by the gas in the path AB
(c) The net heat absorbed by the gas in the path BC P0 C(T2)
A(T1)
(d) The maximum temperature attained by the gas during
the cycle.
V
V0 2V0
Solution :
Path AB is isochoric (w1 = 0), path BC is isothermal (w2 = ă ve), path CA is isobaric (w3 = + ve)
Total work done by gas (w) = w1 + w2 + w3

VB
= 0 + 2.303 nRT log V  P(VC  VA )
C

VB
= 0 2.303 PBVB log  PC (VC  VA )
VC

V0
= 2.303 ï 3P0 ï V0 log + P0(2V0 ă V0)
2V0

= ă 2P0V0 + P0V0
= ă P0V0
Also w2 = ă 2 P0V0 and w3 = P0V0
Also For the path AB, i.e., isochloric

3  PB VB  PA VA 
q1 = n ï Cv ï (TB ă TA) = 1  R
2  R 

3
= [3P0V0 ă P0V0]
2

= + 3 P0V0
For the path CA, i.e., isochoric :

5  PA VA  PB VB 
q3 = n ï Cp (TA ă TB) = 1 ï R  
2  R 

THERMOCHEMISTRY
34 QUIZRR

5
= [P0V0 ă 2P0V0]
2

5
q3 = ă P V
2 0 0

5 P0 V0
Also net heat absorbed = 3P0V0 ă P0V0 =
2 2

P0 V0
 qnet =
2

P0 V0 3P0 V0
Also =
T1 T2

3P0 V0
 T2 = 3T1 =
R

Example 18

Consider a class room of dimensions 5 ï 10 ï 3 dm3 at temperature 20ĈC and pressure 1 atm.
There are 50 peoples in the room, each losing energy at the rate of 150 watt. Assuming that
the walls, ceiling, floor and furniture perfectly insulated and none of them absorbing heat,
how much time will be needed for rising the temperature of air in the room to body
7
temperature, i.e., 37ĈC ? For air Cp = R. Loss of air to the outside as the temperature rises
2
may be neglected.
Solution :
Volume of air in the room = 5 ï 10 ï 3
= 150 m3
= 150 ï 106 cm3

PV
 Mole of air = n =
RT

1 ï 150 ï 106
=
103  0.0821  293

= 6.236 ï 103

THERMOCHEMISTRY
QUIZRR 35

 H  H
Also,   p = Cp = (for 1 mole)
  T  T

 H = n.Cp.T (for n mole)

7
= 6.236 ï 103 ï ï 8.314 ï (310 ă 293)
2

= 3.085 ï 106 J
Thus, heat needed to heat the room to 37ĈC = 3.085 ï 106 J
Also, heat released by 50 peoples = 150 ï 50 J/sec
= 7500 J/sec
7500 J heat is provided in 1 sec

1  3.085  106
 3.085 ï 106 J heat will be provided in
7500

= 411.3 second

Example 19

An aeroplane weighing 63,000 kg flies up from sea level to a height of 8000 metre. Its engine
run with pure normal octane (C8H18) has a 30% efficiency. Calculate the fuel cost of the
f l i gh t , i f oct an e sel l s at Rs. 3/- per l i t r e. Gi v en den si t y of oct an e = 0.705g m L ă1, heat of
combustion of octane = 1300 kcal molă1. (g = 981 cm/sec2)
Solution :
Weight of the plane = 6.3 ï 107 g
Height of the plane = 8 ï 105 cm
 Work required to lift it to this height
= mgh
= 6.3 ï 107 ï 981 ï 8 ï 105 erg
= 4.9442 ï 1016 erg

4.9442  1016
= cal
4.18  107

= 1.1828 ï 109 cal


 Efficiency of fuel is 30%

THERMOCHEMISTRY
36 QUIZRR

1300  30
 Work obtained by 1 mole fuel = kcal
100

1300  30
 kcal work is obtained by 1 mole fuel
100

1.1828  106  100


 6
1.1828 ï 10 kcal work is obtained =  3.033  103 mol
30  1300

= 3.033 ï 103 ï 114 g


= 3.46 ï 105 g

3.46  105 3.46  105


 Volume of fuel = 
d 0.705

= 4.908 ï 105 mL = 4.908 ï 102 litre


 Cost of fuel = 490.8 ï 3 = 1472.4 Rs.

THERMOCHEMISTRY
QUIZRR 37

THERMOCHEMISTRY
It is a branch of physical chemistry that deals with energy changes accompanying chemical
transformations. It is also known as „Chemical Energetics‰. It is based on the first law of
thermodynamics.
Physicochemical changes are classified as endothermic, accompanied by adsorption of heat and
exothermic, accompanied by the evolution of heat. For example,

H2(g) + óO2(g)  H2O(g); H = ă 57.8 kcal

H2O(g)  H2(g) + óO2(g); H = 57.8 kcal

For exothermic reactions (q < 0)


At constant pressure, H = (HP ă HR) = ă ve ( HP < HR)
At constant volume, E = (EP ă ER) = ă ve ( EP < ER)
For endothermic reaction, (q > 0)
At constant pressure, H = (HP ă HR) = + ve ( HP > HR)
At constant volume, E = (EP ă ER) = + ve ( EP > ER)
Like any other transfer of heat, the heat of a chemical reaction depends upon the conditions that
hold during the process by which it is carried out.

Condition I

If the volume of the system is kept constant, no work is done on the system and the first law of
thermodynamics reduces to
E = qv
This condition is excellently approximated when the reaction is carried out in a bomb calorimeter,
where the volume remains constant.

Condition II

During the course of an experiment under ordinary bench-top conditions, the pressure is kept
constant. Many calorimeters operate at this constant atmospheric pressure. Then, we have
H = qP
The heat of reaction measured at constant pressure is exactly equal to the change in enthalpy
of the reaction system.
It is often necessary to use data obtained with bomb calorimeter which give E in order to
calculate H.
We know that, H = E + (PV)
If all the reactants and products are liquids or solids, the PV values change only slightly during
the reaction, provided the pressure is low (say 1 atm). In such cases, qP  qv.
THERMOCHEMISTRY
38 QUIZRR
For reactions in which gases are involved, the values of (PV) depends on the change inj the
number of moles of gas as a result of reaction.
   (PV) = ng(RT)
   H = E + ngRT
Constant pressure processes are much common in chemistry. Therefore, usually when ever we
speak of heat of reaction, it implies enthalpy change at constant pressure.

Heat of Reaction

It is defined as the „amount of heat evoled or absorbed when the reacting species, as represented
by a balanced chemical equation have completely reacted.‰

A + B  C + D, H = q
H = HProducts ă HReactants
= (HC + HD) ă (HA + HB)
Ć If HProducts = HReactants, H = 0
Ć If HProducts > HReactants, H = + ve, reaction is said to be endothrmic
Ć If HProducts < HReactants, H = ă ve, reaction is said to be exothermic.
Ć Enthalpy of every element in any state = Zero.

Physical States of the Reactants and Products

Since an appreciable change is involved in changing a substance from one state of matter to another,
it is important that a state symbol is attached to all the formulae given in an equation, e.g.,

1
(1) H2(g) + O (g)  H2O(l), HĈ = ă 285.9 kJ molă1
2 2

1
H2(g) + O (g)  H2O(g), HĈ = ă 241.8 kJ molă1
2 2
The difference between two HĈ values arises due to difference in physical state of water.

Thus, H2O(l)  H2O(g), HĈ = 44.1 kJ molă1


It reprsents molar enthalpy of vaporisation of water.

(2) C (graphite) + O2 (g)  CO2(g), HĈ = ă 393.5 kJ molă1

C (diamond) + O2 (g)  CO2(g), HĈ = ă 380.4 kJ molă1


The difference between two HĈ values arises due to different allotropes of carbon.

Thus C(diamond)  C (graphite), HĈ = 13.1 kJ molă1


It represents molar enthalpy of transition of carbon.

THERMOCHEMISTRY
QUIZRR 39

E nthalpies of Solution

Enthalpies of solution differ depending on whether the substances react together in dry state or
in solution. For example,

H2S(g) + I2(g)  2HI(solution) + S(s); H = ă 17.2 kcal

H2S(g) + I2(solution)  2HI (solution) + S(s); H = ă 21.93 kcal

Temperature

Enthalply of reaction also depends on temperature at which the reaction is carried out. This is due
to variation in the heat capacity of the system with temperature.
Relation between H or E and temperature are given by KirchoffÊs equation.
(H2 ă H1) = CP (T2 ă T1)
(E2 ă E1) = CV (T2 ă T1)
The above relations may be derived as follows :
For a reaction at constant pressure, H = HP ă HR
Differentiating it w.r.t. T at constant P, we get

 d H   d HP   d HR 
 dT    dT    dT    CP P   CP R  CP
 P  P  P

or d(H) = CPdT
Integrating this equation within appropriate limits, we get

T2 T2

 d  H   C  d T
T1
P
T1

HT ă HT = CP (T2 ă T1)


2 1

H T2  HT1
 CP ...(1)
 T2  T1 
This equation is used to find heat of reaction at a temperature when it is known at another
temperature.

ET2  ET1
Similarly,  Cv ...(2)
 T2  T1 

THERMOCHEMISTRY
40 QUIZRR

d  H 
In the limits as (T2 ă T1)  0, equation (1) yields the differential form  CP
dT

This shows that the rate of change of the enthalpy of a reaction is equal to the difference in heat
capacities of products and reactants.
Since actually the heat capacities themselves vary with temperature, it is sufficiently accurate to
use the average value of the heat capacity over the range of temperature considered.

Example 20

For Ag, CP (JKă1 molă1) is given by 23.43 + 0.00628 T. Calculate H if 3 mol of silver are raised
from 25ĈC to its melting point 961ĈC under 1 atm pressure.
Solution :
At constant P for 1 mole

T2 T2

H =  C dT   (23.43  0.00628T)dT
T1
P
T1

1 ă1
H = 23.43 (T2 ă T1) + (0.00628(T22  T12 ) Jmol .
2
Since, T1 = 273.15 + 25 = 298.15 K and T2 = 273.15 + 961 = 1234.15 K
(T2 ă T1) = 936 K
H = 23.43(936) + ó(0.00628) [(1234.15)2 ă (298.15)2] = 21930 + 4500
= 26430 J/mol
For 3 mol, H = 3(26430) J = 79290 J

Types of Heat (enthalpy) of Reactions

The heat or enthalpy changes taking place during the chemical reactions are expressed in different
ways depending upon the nature of the reaction. The various types of enthalpies of reactions are :
(1) Heat or Enthalpy of Formation
The heat evolved or absorbed when 1 mole of a substance is formed from its elements is
called heat of formation. It is denoted by Hf.
For example, heat of formation of carbon dioxide and methane may be expressed as :

C(s) + 2H2(g)  CH4 (g) Hf = ă 74.8 kJ


These equations should always be written for one mole as per the definition of the substance
to be formed. If for balancing, we require the coefficient 2,. 3 ... etc., in the equation then
Hf values should also be multiplied by the same number as discussed earlier.

THERMOCHEMISTRY
QUIZRR 41

Standard heat of formation

The heat of formation Hf depends upon the condition of temperature, pressure and physical state
(gas, liquid or solid) of the reactants and the products. Therefore the heat change accompanying
the formation of one mole of a compound from its elements when all the substances are in their
standard states (1 atm pressure and 298 K), is called the standard heat of formation. It is
expressed as HĈ f.
Since no heat changes are involved in the formation of elements form themselves in their standard
states, the standard enthalpy of formation of all elements is zero.
For example, the standard enthalpy of formation (HĈ f.) for H2O(l) is ă 286 kJ molă1 i.e., when
one mole of liquid water is formed from its elements H2(g) and O2(g) at 298 K and 1 atm pressure,
then 286 kJ molă1 of heat is released. The negative value of HĈ f. indicates the formation of a
stable compound.

Standard heat of reaction from standard heats of formation

The knowledge of standard heats of formation of various substances can be used to calculate the
heats of reactions under standard conditions. The standard heat of any reaction (HĈ) is equal
to the difference between the HĈ f. of all the reactants i.e.,
HĈ = Sum of the standard heats ă Sum of the standard heats of formation of products of
formation of reactants
i.e., HĈ = HĈ f (products)
ă HĈ f (reactants)
For a reaction,

aA + bB  cC + dD

HĈ = HĈ f (products)


ă HĈ f (reactants)
= [cHĈ f (C) + dHĈ f(D)] ă [aHĈ f (A) + bHĈ f(B)]
The heat of formation of all elements in their standard state is zero. On this basis, it is evident
that the heat of formation of a compound is the heat of the compound.

Elements  Compound


Heat of formation = Hf (compound) ă Hf (elements)

Example 21

The heat change for the reaction,

N 2 ( g )  3H 2 ( g )  2NH 3 ( g ) is ă 92.2 kJ. Calculate the heat of formation of ammonia.

Solution :
The heat of formation of ammonia is the heat change for the formation of 1 mole of ammonia from
its elements, i.e.,
THERMOCHEMISTRY
42 QUIZRR

1 3
N2 ( g)  H2 ( g)  NH3 (g) H = ?
2 2 f

The heat change for the reaction

N2(g) + 3H2(g)  2NH3(g)

is H = ă 92.2 kJ. This equation corresponds to formation of two moles of ammonia. Thus,

 92.2
H f    46.1 kJ mol1
2

Example 22

Calculate the heat change for the reaction CH4(g) + 2O2(g)  CO2(g) + 2H2O(l)
The heat of formation of CH4(g) and H2O are ă 74.8 kJmolă1, ă 393.5 kJmolă1 and ă 285.8 kJ
molă1 respectively.
Solution :
HĈ for the reaction

CH4(g) + 2O2(g)  CO2(g) + 2H2O(l)

is : HĈ = Hf (products)  Hf (reactants)

H [CO ( g)]  2H [H O(l)]  H [CH ( g)]  2H [O ( g)]

f 2

f 2

f 4

f 2

Hf [CO2 ( g)]   393.5 kJmol1 , Hf [H2 O(l)]   285.8 kJmol1

Hf [CH4 ( g)]   74.8 kJmol1 , Hf [O2 ( g)]  0 (by convention)

HĈ = {ă 393.5 + 2 (ă 285.8)} ă {ă 74.8 + 2 ï 0)}


= 965.1 + 74.8 = ă 890.3 kJmolă1

(2) Enthalpy of Combustion

The enthalpy of combustion of a compound is the enthalpy change at normal pressure and at
constant temperature accompanying complete combustion of one mole of the compound. It is
denoted by Hc. Combustion here means the burning of the given compound to the highest oxides
of the constituent elements in the presence of excess of oxygen.
For example, the enthalpy of combustion of benzene at 298 K is the enthalpy change of the
reaction.

15
C6H6(l) + O (g)  6CO2(g) + 3H2O(l); HĈ = ă 3268 kJ
2 2
THERMOCHEMISTRY
QUIZRR 43

Example 23

The heats of combustion of CH4 and C2H6 are ă 890.3 and ă 1559.7 kJ molă1 respectively.
Which of the two has greater efficiency of fuel per gram ?
Solution :
The fuel efficiency can be predicted from the amount of heat evolved for every gram of fuel
consumed.
(i) The combustion of methane is as follows :

CH4(g) + 2O2(g)  CO2(g) + 2H2O(l)

Hc = 890.3 kJmolă1


Molar mass of CH4 = 16

890.3
Heat produced per gram =  55.64 kJ gă1
16

(ii) The combustion of ethane is as follows :

7
C2H6(g) + O (g)  2CO2(g) + 3H2O(l)
2 2

Hc = 1559.7 kJmolă1


Molar mass of C2H6 = 30

1559.7
Heat produced per gram =  51.99 kJ gă1
30

Thus, methane has greater fuel efficiency than ethane.

Example 24

(a) A cylinder of gas supplied by a company is assumed to contain 14 kg of Butane. If a


normal family requires 20,000 kJ of energy per day for cooking, how long will the
cylinder last ?
(b) If the air supplied to the burner is insufficient, a portion of gas escapes without
combustion. Assuming that 25% of the gas is wasted due to this inefficiency, how long
will the cylinder last ? (Heat of combustion of butane = 2658 kJ/mol)
Solution :
(a) Molecular formula of butane = C4H10
Molecular mass of butane = 4 ï 12 + 10 ï 1 = 58
Heat of combustion of butane = 2658 kJ molă1
1 mole of 58 g of butane on complete combustion given heat = 2658 kJ

THERMOCHEMISTRY
44 QUIZRR

(2658)  (14)  103


14 ï 103 g of butane on complete combustion gives heat =  641586
58

The family needs 20,000 kJ of heat for cooking per day.

641586
641586 kJ of heat will be used for cooking by a family in  32 days
20000

The cylinder will last for 82 days.


(b) 25 per cent of the gas is wasted due to inefficiency. This means that only 75% of butane
is combusted. Therefore,

(641586)  75
the energy produced by 75% combustion of butane =  481190 kJ
100

481190
 The number of days the cylinder will last =  24 days
20000

Example 25

The enthalpy change involved in the oxidation of glucose is ă 2880 kJ molă1. Twenty five
percent of this energy is available for muscular work. If 100 kJ of muscular work is needed
to walk one kilometer, what is the maximum distance that a person will be able to walk
after eating 120 g of glucose.
Solution :
Hcomb of Glucose (C6H12O6) = ă 2880 kJ molă1

2880  120
Total energy available from 120 g of glucose =  1920 kJ
180

1920  25
Energy available for muscular work =  480 kJ
100

480
Distance travelled =  4.8 km
100

Example 26

Calculate the enthalpy change of combustion of cyclopropane at 298 K. The enthalpy of


formation of CO2(g), H2O(l) and propane(g) are ă 393.5, ă 285.8 and 20.42 kJmolă1 respectively.
The enthalpy of isomerisation of cyclopropane to propene is ă 33.0 kJ molă1.

THERMOCHEMISTRY
QUIZRR 45

Solution :
The required H is

9
C3H6 + O 3CO2 + 3H2O; H = ?
2 2 
cyclopropane

The given equations are :

(i) C + O2  CO2; H1 = ă 393.5 kJ

1
(ii) H2 + O H2O; H2 = ă 285.8 kJ
2 2 

(iii) 3C + 3H2  CH3CH = CH2; H3 = + 20.42 kJ

(iv) C3H6  CH2 = CHăCH3; H4 = ă 33.0 kJ


cyclopropane

Multiply equation (i) and (ii) by 3 and add them. Now subtract equation (iii) and subsequently
add equation (iv) from the resulting expression.
H = 3H1 + 3H2 ă H3 ă H4
= 3(ă 393.5) + 3(ă 285.8) ă (20.42) + (ă 33.0) = ă 2091.32 kJ

(3) Heat or Enthalpy of Neutralisation

The reaction in which an acid and a base react to give a salt and water is called neutralization
reaction. Neutralization reactions are exothermic in nature. The heat change when one gram
equivalent of an acid is completely neutralised by a base or vice versa in dilute solution, is called
heat of neutralization.
Examples of heat of neutralization are :
Neutralization of HCl with NaOH

HCl(aq) + NaOH(aq)  NaCl(aq) + H2O(l) H = ă 57.1 kJ

It is important to note that the term gram equivalent is used in the definition of heat of
neutralization. This is because neutralization involves 1 mole of H+ ions and 1 mole of OHă ions
to form 1 mole of water and 57.1 kJ of heat is liberated.

H+(aq) + OHă(aq)  H2O(l) H = ă 57.1 kJ

Now, one gram equivalent of various acids on complete dissociation liberates one mole of H+ ions.
But one mole of the acid may produce more than one mole of H+ ions in solution depending upon
its basicity; for example 1 mol of H2SO4 gives 2 mol of H+ ions and 1 mol of H3PO4 gives 3 mol

THERMOCHEMISTRY
46 QUIZRR
of H+ ions on complete dissociation. But 1 gram equivalent of both (H2SO4 or H3PO4) produces
only 1 mol of H+ ions.
Thus, it is more appropriate to use the term gram equivalent in the definition of enthalpy of
neutralization.
The average enthalpy of nuetralization of any strong acid by a strong base is found to be ă 57.7
kJ (ă 13.7 kcal) irrespective of the nature of acid or the base. This suggests that the net chemical
reaction in all neutralization reactions is the same, viz.,

H+(aq) + OHă(aq)  H2O(l) H = ă 57.1 kJ


This is because strong acid and strong base are completely ionized in aqueous solutions. The
aqueous solution of one gram equivalent of all strong acids contains the same number of H+ ions.
Similarly, aqueous solution of one gram equivalent of all strong bases also contains same number
of OHă. The neutralization reactions between strong acids and strong bases in aqueous solutions
involve simply the combination of H+ ions (from an acid) and OHă ions (from a base) to form
unionized water molecules.

Neutralization of weak acids and weak bases

The heat of neutralization of a weak acid or a weak base is less than ă 57.1 kJ and is also
different for different weak acids or bases.
For example for acetic the enthalpy of neutralization is ă 54.9 kJ.

Example 27

100 ml of 1 N of an acid and 100 ml of 1 N of a base are mixed at 298 K. During the
experiment, the temperature arose to 298.0067 K. Calculate the heat of neutralization.
Solution :
Heat capacity of solution = Mass of solution ï Specific heat capacity
Total mass of solution = 100 + 100 = 200 ml
Heat capacity of solution = 200 ï 4.2 = 840 JKă1
Heat change in the reaction = Heat capacity ï Rise in temperature
= (840 JKă1) (298.0067 ă 298)K
= 840 ï 0.0067 J = 5.63 J
Now, one gram equivalent of acid = 1 N HCl in 1000 ml
100 ml of 1 N acid on neutralization gives heat = 5.63 J

5.63
1000 ml of 1 N acid on neutralization gives heat =  1000 = 56.3 J
100

Heat of neutralization = ă 56.3 J

THERMOCHEMISTRY
QUIZRR 47

Example 28

Whenever an acid is neutralized by a base, the net reaction is :

H+(aq) + OHă(aq)  H2O(l) + 57.1 kJ

Calculate the heat evolved when 0.60 mol of HNO3 solution is mixed with 0.30 mol of KOH
solution
Solution :
According to the reacation

H+(aq) + OHă(aq)  H2O(l) + 57.1 kJ


When 1 mole of H+ ions and 1 mol of OHă ions are neutralized to form 1 mol of water, 57.1 kJ
of energy is released.
(a) Heat evolved on mixing 0.60 mol of HNO3 with 0.30 mol of KOH solution.
Since HNO3 and KOH are strong acid and bases,
0.60 mol of HNO3  0.60 mol of H+ ions
0.30 mol of KOH  0.30 mol of OHă ions
In this case, out of 0.60 mol of H+ ions (from HNO3) only 0.30 mol will be neutralised (equal
to OHă ions present) by the base. 0.3 mol of H+ ions of HNO3 will remain unreacted. The
net reaction is :

H+ (0.30 mol) + OHă (0.30 mol)  H2O (0.30 mol)

Now, heat evolved during the formation of 1 mol of H2O = 57.1 kJ


Heat evolved in the formation of 0.3 mol of H2O = 57.1 ï 0.3 = 17.13 kJ

(4) Enthalpy of solution (Hsol)

When a solute is dissolved in a solvent a solution is formed. During dissolution of a solute in any
solvent, a certain amount of heat is either absorbed or evolved. Such heat changes under constant
pressure conditions are known as the enthalpy of solution. The change in enthalpy when one
mole of a solute is dissolved in a specific quantity of a solvent at a given temperature is called
enthalpy of solution.
To avoid the amount of solvent, heat of solution is usually defined for an infinite dilute solution.
Thus, heat of solution of infinite dilution is the heat change when one mole of a substance is
dissolved in such a large quantity of solvent so that further dilution does not give any further
heat change.
For example, dissolution of sodium chloride
NaCl(s) + aq NaCl(aq)        H = Hsol = + 5.0 kJ
Here ÂaqÊ represents aqueous meaning a large excess of water.

THERMOCHEMISTRY
48 QUIZRR
(5) Enthalpy of fusion (Hfus)

The enthalpy of fusion of a substance is defined as Âthe change in enthalpy when one mole of a
solid substance is melted at its melting temperatureÊ. For example, the enthalpy change of the
reaction,

Ice (273 K)  H2O(l) (273 K)      H = 6.0kJ molă1

(6) Enthalpy of Vaporisation (Hvap)

ÂThe change in enthalpy when one mole of a liquid is converted into vapours at its boiling
temperature is called enthalpy of vaporisationÊ (Hvap).
Thus, the enthalpy change of the reaction

H2O(I), 373 K  H2O(v), 373K      H = 40.7 kJ molă1


is the enthalpy of vaporisation of water.

Example 29

Determine the value of H and E for the reversible isothermal evaporation of 90.0 g of
water at 100ĈC. Assume that water vapours behave as ideal gas and heat of evaporation of
water is 540 cal gă1.
(R = 2 cal molă1 Kă1)
Solution :

90
90 g of water = = 5.0 mol
18

Heat of evaporation of 1 g of water = 540 cal


Heat of evaporation of 90 g of water = 540 ï 90 = 48600 Cal.
H = 48600 Cal
The evaporation of 5 mol of water is represented as

5H2O(l)  5H2O(g)


n = (5 ă 0) = 5
H = E + nRT or E = H ă nRT
= 48600 ă (5) (2.0) (373) = 44870 cal

(7) Enthalpy of Sublimation (Hsub)

Sublimation is a process in which a solid substance directly changes into its vapours at any
temperature below its melting point. Enthalpy of sublimation is defined as follows :

THERMOCHEMISTRY
QUIZRR 49

The change in enthalpy when one mole of a solid substance is converted into its vapours without
melting point at a temperature below its melting point is called the enthalpy of sublimation.
For example, when one mole of solid iodine is converted into its vapours at room temperature, heat
equal to 62.4 kJ is absorbed. So, the enthalpy of sublimation of iodine is + 62.4 kJ molă1, i.e.,

I2(s)  ă1
 I2(v) = 62.4 kJ mol
Compounds, which sublime on heating are camphor, dry ice, ammonium chloride etc.
The heat of sublimation is related to heat of fusion and heat of vaporization as :
Hsublimation = Hfusion + Hvaporization

Example 30

When 1 g of liquid naphthalene (C10H8) solidifies, 149 J of heat is evolved. Calculate the
heat of fusion of naphthalene.
Solution :
The molecular mass of naphthalene is C10H8 = 10 ï 12 + 8 ï 1 = 128
Heat evolved when 1 g of naphthalene solidifies = 149 J
Heat evolved when 128 g of naphthalene solidifies = 149 ï 128 = 19072 J

If C10H8(l)  C10H8(s) H = 19072 J


For the fusion reaction,

C10H8(s)  C10H8(l)


This reaction is the reverse of the above solidification reaction so that
Hfusion = ă Hsolidification
Hfusion = 19072 J or = 19.072 kJ

(8) Enthalpy of hydration

This is defined as the heat change (evolved or absorbed) when one mole of the anhydrous salt
combines with the required number of moles of water to form the specific hydrated salt.

HESS’S LAW OF CONSTANT HEAT SUMMATION

This law states that Âthe heat change in a particular reaction is the same whether it takes place
in one step or several stepsÊ.
For example, a reactant ÂAÊ changes to a product ÂBÊ in one step and the heat change during this
process is H. If the reaction is carried out in two steps where ÂAÊ first changes to ÂCÊ an intermediate
stage and then ÂCÊ changes to ÂBÊ in the following step then let the heat change during the

THERMOCHEMISTRY
50 QUIZRR
formation of ÂAÊ to ÂCÊ be H1 and that from ÂCÊ to ÂBÊ be H2. From HessÊs law the heat change
for the reaction is given as :
H = H1 + H2

Path I
A B
H
Path II
H1 H2
C

This means that the amount of heat evolved or absorbed in a chemical reaction depends only upon
the energy of initial reactants and the final products. The heat change is independent of the path
or the manner in which the change has taken place.
The formation of carbon dioxide from carbon and oxygen can be illustrated as follows. Carbon can
be converted into carbon dioxide in two ways. Firstly solid carbon combines with sufficient amount
of oxygen to form CO2. The same reaction when carried in presence of lesser amount of oxygen
gives carbon monoxide which then gets converted to CO2 in step two in the presence of oxygen.

Path I : C(s) + O2(g)  CO2(g) H = ă 393.5 kJ

1
Path II : C(s) + O (g)  CO(g) H1 = ă 110.5 kJ
2 2

1
CO(g) + O (g)  CO2(g) H2 = ă 283.0 kJ
2 2

C(s) + O2(g)  CO2(g) H = ă 393.5 kJ


H = H1 + H2
Thus, one can conclude that thermochemical equations can be added, subtracted or multiplied like
algebraic equations to obtain the desired equation.

Example 31

Calculate the standard heat of formation of carbon disulphide (l). Given that the standard
heats of combustion of carbon (s), sulphur (s) and carbon disulphide (l) are 393.3, ă 293.72
and ă 1108.76 kJ molă1 respectively.
Solution :
The given data can be written in thermochemical equation form as :

C(s) + O2(g)  CO2(g) H = ă 393.5 kJ (i)

THERMOCHEMISTRY
QUIZRR 51

S(s) + O2(g)  SO2(g)    H = ă 293.72 kJ (ii)

CS2(I) + 3O2(g)  CO2(g) + SO2(g)  H = ă 1108.76 kJ (iii)

The required equation is :

C(s) + 2S(g)  CS2(l) H = ?

Multiplying equation (ii) by 2 and adding to equation (i) we get,

2S(s) + 2O2(g)  2SO2(g) H = ă 587.44 kJ

C(s) + O2(g)  CO2(g) H = ă 393.5 kJ

2S(s) + C(s) + O2(g)  2SO2(g) + CO2(g)

H = ă 980.74 kJ
Subtracting equation (iii) from the above equation we have,

2S(s) + C(s) + O2(g)  2SO2(g) + CO2(g) H = ă 980.74 kJ

CS2(l) + 3O2(g)  CO2(g) + SO2(g) H = ă 1108.76 kJ

C(s) + 2S(g)  CS2(l) H = ă 128.02 kJ

Born-Haber Cycle

This cycle is based on thermochemical changes taking place in the formation of a lattice. This cycle
can be used to determine lattice energy which cannot be directly measured. It is defined as that
energy released when one mole of the atomic compound (lattice) is formed from its isolated ions
in the gaseous state under standard condition.
1 q
Na(s) + Cl2(g) NaCl(s)
2

S D
2

Na(g) Cl(g) ăU

I ăeă ăE + eă

Na+(g) Clă(g)

nAm+(g) + mBnă (g)  AnBm(s)

H = ă U (lattice energy)

THERMOCHEMISTRY
52 QUIZRR
Formation of NaCl(s) lattice involves thus,

D
S+I+ EU=q
2

hence, U can be calculated.


here, S = heat of sublimation of Na(s)
I = ionisation energy of Na(g)
D = dissociation energy of Cl2
E = electron affinity of Cl(g)
U = lattice energy
q = heat of formation of NaCl(s)
If lattice is MgX2(s) then
S + (I1 + I2) + D ă 2E ă U = q
where, (I1 + I2) = total ionisation energy to form Mg2+(g).

Bond Energy or Bond Enthalpy


In chemical reactions the formation of a chemical bond is accompanied by the release of energy.
Conversely energy has to be supplied for the breaking of a bond.
Bond strengths are commonly described by their bond dissociation energy which is the energy
required to break one mole of a bond of particular type. This is a definite quantity and is
expressed in kJ molă1. For diatomic molecules the bond dissociation energy is same as bond
energy, whereas in polyatomic molecules the bond energy is taken as the mean average of the
various bond dissociation energies of the bonds of a given type.
The thermochemical data is useful in determining the bond energies of different bonds.
For example the bond energy of C-H bond in methane can be calculated form its heat of formation.
The heat for formation of methane from carbon and hydrogen has been found to beă1663 kJ molă1.

C(s) + 4H(g)  CH4(g) H = ă 1663 kJ

Methane has four C-H bonds and the energy required to break all the four C-H bonds is 1663
1663
kJ. Therefore the average C-H bond energy is  415.75 or 416 kJ molă1.
4

Example 32
Compute the average S-F bond energy in SF6. The values of standard enthalpy of formation
of SF6(g), S(g), are, 100, 275 and 80 kJ molă1 respectively.
Solution :
Consider the equation

SF6(g)  S(g) + 6F(g)

THERMOCHEMISTRY
QUIZRR 53

Hreaction = 6Hf (F) + Hf(S) ă Hf (SF6)


= 6 ï 80 + 275 ă (ă 1100) = 1855 kJ

1855
HSăF bond = = 309.17 kJ molă1
6

Example 33

The enthalpies of combustion of carbon and carbon monoxide in excess of oxygen at 298
K and constant pressure are ă 393.5 kJ/mol and 283.0 kJ/mol respectively. Calculate the
heat of formation of carbon monoxide at constant volume.
Solution :
Heat of formation of CO is written as
O2(g)  CO(g);
C(s) + ó H = ?
Now we have :
1. C(g) + O2(g)  CO2(g); H1 = ă 393.5
2. CO(g) + óO2(g)  CO2(g); H2 = ă 283
Adding appropriately (i.e., 1 - 2), we get;
C(s) + óO2(g)  CO(g)
From HessÊs Law : Hf = H1 ă H2
Hf = ă 393.5 ă (ă 283) = ă 110.5 kJ
Now calculation of the heat of formation at constant volume means that we have to calculate
change in internal energy (i.e. E). Proceeding in the following manner...
From First Law of Thermodynamics : H = E + PV
Now, PV = nRT
 PV = nRT (n = gaseous moles of products ă gaseous moles of reactant)
 H = E + n RT
 E = H ă nRT
n = 1 ă ó = ó T = 298 K R = 8.314 J/K/mol. H = ă 110.5 kJ
 E = ă 110.5 ă (ó
) ï 8.314 ï 298 = ă 111.7 kJ

Example 34

The heat of combustion of sucrose, C12H22O11(s) at constant volume is ă 1348.9 kcal molă1 at
25ĈC, then the heat of reaction at constant pressure, when steam is produced, is
(A) ă 1348.9 kcal (B) + 1342.344 kcal
(C) + 1250 kcal (D) None

THERMOCHEMISTRY
54 QUIZRR
Solution :
The combustion equation of sucrose is
C12H22O11(s) + 12O2(g)  12CO2(g) + 11H2O (l)
Here,
n = sum of gaseous product moles ă sum of gaseous reactant moles
n = 12 + 11 ă 12
n = 11
As we know,
H = E + n RT, where H = heat of reaction at constant pressure
E = heat of reaction at constant volume
Here, E = ă 1348.9 kcal
R = 2.0 cal, T = 25 + 273 = 298 K
 H = (ă 1348.9 ï 1000) + 11 ï 2 ï 298
= ă 1348900 + 6556 = ă 1342344 cal
= ă 1342.344 kcal
Hence, (A) is correct.

Example 35

In order to get maximum calorific output, a burner should have an optimum fuel to oxygen
ratio which corresponds to 3 times as much oxygen as is required theoretically for complete
combustion of the fuel. A burner which has been adjusted for methane as fuel (with x litre/
hour of CH4 and 6x litre/hour of O2) is to be readjusted for butane, C4H10. In order to get
the same calorific output, what should be the rate of supply of butane and oxygen ? Assume
that losses due to incomplete combustion etc. are the same for both fuels and that the gases
behave ideally. Heats of combustion : CH4 = 809 kJ/mol; C4H10 = 2878 kJ/mol.
Solution :
In 1.0 hour, x lts of CH4 required 6x lts of O2
H (combustion) of CH4 = 809 kJ/mol

809
 kJ/ Its at 1.0 atm and 25ĈC
24.48

H (combustion) of C4H10 = 2878 kJ/mol

2878
 k/J lts at 1.0 atm and 25ĈC
24.48

809
x lts of CH4 produces (x) kJ
24.48

THERMOCHEMISTRY
QUIZRR 55

Now this much energy has to be provided by burning of C4H10.

809
 (x) kJ will be provided by
24.48

 809   24.48 
 ( x)      (0.28 x) lts of C4H10
 24.48   2878 

C4H10 + 13/2 O2  4CO2 + 5H2O


 1 mol CH4  13/2 mol O2
 (3 ï 13/2) times of O2 is required/mol
 rate of O2 per hour = (0.28 x) ï (39/2) = (5.48 x) lts O2

Example 36

Standard enthalpy of formation of C3H7NO2(s), CO2(g) and H2O(l) are ă 133.57, ă 94.05 and
ă 68.32 kcal/mol respectively. Standard enthalpy of combustion of CH4 at 25ĈC is ă 212.8 kcal
per mole. Calculate HĈ for the reaction : 2CH4 + CO2 + 1/2 N2  C3H7NO2(s) + 1/2 H2.
Calculate E for combustion of C3H7NO2(s).
Solution :
2CH4 + CO2 + 1/2N2  C3H7NO2 : H = ?
First find Hf of CH4
Given : CH4 + 2O2  CO2 + 2H2O : H = ă 212.8
Using the definition of H, we have :
H = [HĈ f (CO2) + 2HĈ f (H2O)] ă HĈ f (CH4) (note that HĈ f O2 = 0)
 ă 212.8 = [ă 94.05 + 2(ă 68.32)] ă HĈ f (CH4)
 HĈ f (CH4) = ă 17.89 kcal/mol
Now find the H of the required equation using HĈ f (CH4)
H = [HĈ f (C3H7NO2) ă 0] ă [2 ï HĈ f (CH4) + HĈ f (CO2) + 0]
 H = (ă 133.57) ă 2(ă17.89) ă (ă 68.32) = ă 3.74 kcal/mol
Now calculate H(combustion) of C3H7NO2.
C3H7NO2(s) + 15/4O2  3CO2(g) + 1/2N2(g) + 7/2H2O(g)
  Hcomb = 3HĈ f (CO2) + 0 + 7/2HĈ f (H2O) ă HĈ f C3H7NO2 ă 0
= 3(ă 94.05) + 7/2(ă 68.32) ă (ă 133.57) = ă 387.70 kca/mol
Find E using E = H ă nRT
    E = ă 387.70 ă (ă 1/4) ï 2 ï 10ă3(298) = ă 387.72 kcal/mol

THERMOCHEMISTRY
56 QUIZRR
Example 37

Calculate the heat of reaction for the hydrogenation of acetylene to ethylene at constant
volume at 25ĈC from the following data :
(i) Enthalpy of formation of water = ă 285.8 kJ molă1
(ii) Enthalpy of combustion of acetylene = ă 1299.6 kJ molă1
(ii) Enthalpy of combustion of ethylene = ă 1410.8 kJ molă1
Solution :

(i) H2(g) + 1/2 O2(g)  H2O (l), H = ă 285.8 kJ molă1

5
(ii) C2H2(g) + O (g)  2CO2(g) + H2O (l), H = ă 1299.6 kJ molă1
2 2

(iii) C2H4(g) + 3O2 (g)  2CO2(g) + 2H2O (l), H = ă 1410.8 kJ molă1
Eqn. (ii) + Eqn. (i) ă Eqn. (iii), we get

C2H4(g) + H2(g)  C2H4(g)

H = ă 1299.6 ă 285.8 + 1410.8


= ă 174.6 kJ molă1
E = ă 174.6 ă (ă1) (8.314 ï 10ă3) (298)
= ă 172.12 kJ molă1

Example 38

From the following data, calculate the bond enthalpies of CăC and C  N bonds :
(i) Enthalpy of formation of CH3CN = 87.86 kJ molă1
(ii) Enthalpy of formation of C2H6 = ă 83.68 kJ molă1
(iii) Enthalpy of sublimation of graphite = 719.65 kJ molă1
(iv) Enthalpy of dissociation of nitrogen = 945.58 kJ molă1
(v) Enthalpy of dissociation of hydrogen = 435.14 kJ molă1
(vi) CăH bond enthalpy = 414.22 kJ molă1
Solution :

2C(s) + 3H2(g)  C2H6(g), H = ă 83.68 kJmolă1

H = [2HC(s)  C(g)


+ 3[HHăH] ă HCăC + 6HCăH]

ă 83.68 = [2 ï 719.65 + 3 (435.14)] ă [HCăC + 6 ï 414.22]

THERMOCHEMISTRY
QUIZRR 57

On solving we get, HCăC = 343.08 kJ molă1

3 1
2C(s) + H2(g) + N (g)  CH3CN(g), H = 87.86 kJ molă1.
2 2 2

3 1
H = [2HC(S)  C(g)
+ HHăH + HN=N] ă [HCăC + 3HCăH + HNN]
2 2

3 1
87.86 = [12 ï 719.65 + (435.14) + (945.58)] ă [343.08 + 3(414.22 + HNN]
2 2

On solving we get, HNN = 89.12 kJ molă1.

THERMOCHEMISTRY
GASEOUS STATE
QUIZRR 3

1. INT RODUCT ION


The classification of matter into three different states, namely solid, liquid and gaseous state is
termed as the physical classification of matter. Most properties of solid, liquid and gases that can be
observed with out sense organs are called as ÂmacroscopicÊ properties. The description of the behaviour
of the three states of matter in terms of atomic theory is called ÂmicroscopicÊ description of matter. From
the study of the observable properties of different states of matter one can understand the microscopic
nature of matter in terms of the behaviour of constituent particles.

So l id s t a t e

A solid possesses a definite size (volume) and a definite shape under ordinary conditions; and
tends to maintain these even under deforming conditions. The substances in solids are closely packed
and bound by strong inter particle attraction, making them rigid and geometrical. Some common
examples of solids are iron, silver, common salt etc.

Liquid st at e

A liquid possesses a definite volume but not definite shape. The substances in liquids have
particles, which are loosely packed and bound to each other by forces weaker than those of solids. This
makes a liquid mobile and shapeless resulting in its taking up the shape of the container in which it
is placed. A liquid also has a tendency to flow. For example, water, alcohol, milk, oil etc.

T h e Ga s e o u s St a t e

Of the three states of matter, the gaseous state is the simplest and shows greatest uniformity in
behaviour. Gases show almost similar behaviour irrespective of their chemical nature. This state is
characterized by :
Ć Gases maintain neither the volume nor the shape. The completely fill the container in which
they are placed.
Ć They expand appreciably on heating.
Ć Gases are highly compressible. The volume of the gas decreases when the pressure increases.
Ć They diffuse rapidly into space.
Ć Gases exert equal pressure in all directions.
Ć All gases are colourless except a few e.g. chlorine (greenish yellow) bromine (reddish brown),
nitrogen dioxide (reddish brown).
The behaviour of gases can be described by certain quantitative relationship called gas laws. They
give the relationship between mass, pressure, volume and temperature.

GASEOUS STATE
4 QUIZRR
M e a s u r a b l e Pro p e r t i e s o f Ga s e s

The important fundamental properties of gases are volume, pressure and temperature. These are
discussed below :

(1 ) V o l u m e

The volume of a substance is the space occupied by it. A gas occupies the entire volume of the
container available to it. The measurement of the volume of a gas only requires the measurement of
the volume of the container enclosing it.

Unit s of volum e

Generally, volume is expressed in units of litres (L), millilitre (mL) or cubic centimetres (cm3).
1 mL = 1 cm3
1 L = 103 mL or 1 dm3
The SI unit of volume is m3. As this unit is too large, volume is generally expressed in smaller
units of cubic decimetre (dm3) or cubic centimetre (cm3). These are related to each other as :
1 m3 = 103 dm3 = 106 cm3
The volume of the gas depends upon its amount, temperature and pressure V = f (amount,
temperature, pressure).

(2 ) Pr e s s u r e

Gases exert uniform pressure in all the directions on the walls of the container in which they are
confined. As pressure is force per unit area, the pressure of the gas is the force exerted by the gas per
unit area on the walls of the container.

At m ospheric pressure

Air is pulled towards the surface by gravity and this exerts pressure on the EarthÊs surface. The
pressure exerted by the gases of the atmosphere on the surface of the Earth is called atmospheric pressure.

Measurem ent of at m ospheric pressure

Atmospheric pressure is measured by an instrument known as barometer. This consists of a glass


tube closed at one end and filled with pure mercury. The tube is then inverted into an open vessel of
mercury. The mercury level in the tube drops until the pressure due to the column of mercury in the
tube becomes equal to the atmospheric pressure acting outside the tube. The region above the mercury
in the tube is almost a perfect vacuum, therefore, the pressure due to mercury.

GASEOUS STATE
QUIZRR 5

The pressure (P) is expressed as :


P = h..g
where ÂhÊ is the height of mercury column in the barometer
ÂÊ is the density of mercury.
ÂgÊ is the acceleration due to gravity.

Vacuum
Mercury

Height h = 76 cm (or 760 mm)


at 0Ĉc CORRESPONDS
Atmospheric
TO 1 ATM
pressure

Mercury barometer

U n i t s o f a t m o s p h e r ic p r e s s u r e

The maximum height of mercury, which can be supported by the atmospheric pressure provides
a measure of that pressure.
A standard pressure of one atmosphere (1 atm) is defined as the pressure that will support a
column of mercury of 76 cm height at 0Ĉ (density of mercury (13.5951 gcm3)
One atmosphere is also referred to as 760 torr. This unit is named after the scientist Torricelli,
who invented the Barometer. Thus
1 atm = 76.0 cm of mercury (cm Hg)
= 760 mm of mercury (mm Hg)
= 760 torr.
The S.I. unit of pressure is pascal (Pa), which is the pressure exerted when a force of Newton (1
N) acts on 1 m2 area. Pascal is related to atmosphere as :
1 atm = 101.326 ï 103 N mă 2

= 101.325 kPa
However, for approximate work, one atmosphere is taken to be equal to 102 kPa or 105 Pa or Nmă2.
The pressure of gases are measured by a device known as manometer that can be either open and
manometer or closed end manometers consist of ÂUÊ tube partly filled with a non-volatile liquid like
mercury. During measurement, the difference in the levels of the mercury in the two limbs gives the
difference in pressure on the two sides.

GASEOUS STATE
6 QUIZRR
(3 ) T e m p e r a t u r e

Temperature is a measure of hotness or coldness of a body. It is a measure of kinetic energy


possessed by the molecules. A hot body is said to be at a highter temperature and a cold body is said
to be a lower temperature. The devices used to measure temperature are called Thermometers. The
substance commonly used in the thermometers is mercury.
Temperature is commonly measured in Celsius scale (centigrade scale). In this scale, the freezing
point of water (0ĈC) and the boiling point of water (100ĈC) at one atmospheric pressure are taken as
reference points. The range between 0Ĉ to 100Ĉ is divided into one hundred equal parts, each division
corresponding to 1ĈC.

Unit of t em perat ure

The SI unit of temperature is degrees Kelvin (K) and this scale is known as Kelvin scale. The zero
point on the Kelvin scale is known as absolute zero, which is equal to ă 273.15ĈC.
The temperature on the Celsius scale is converted to temperature on Kelvin scale by the addition
of 273.15Ĉ. Thus,
ĈC + 273.15 = K

St a n d a r d t e m p e r a t u r e a n d p r e s s u r e

The properties of a gas depends upon the temperature and pressure. Hence, it is convenient to
specify a particular temperature and pressure for comparison of different gases. The standard conditions
of temperature and pressure are abbreviated as S.T.P. (Standard Temperature and Pressure) and
N.T.P. (Normal Temperature and Pressure).
Standard Temperature and Pressure (S.T.P. or N.T.P.) values are :
Temperature = 0ĈC or 273.15 K
Pressure = 1 atm or 760 mm Hg or 760 torr or 101.325 kPa (S.I. units)

2. GA S L A WS
P = pressure in units of atm (atmospheres) or torr. 1.00 atm = 760 torr
V = volume in units of L (liters)
n = moles of the substance

(A ) B o y l e ’s L a w

Consider the change in volume of one mole of an ideal gas with the change in pressure exerted
on the gas at constant temperature.

GASEOUS STATE
QUIZRR 7

P1 P2

piston

P2 >P1
at constant
V1 at Ideal temperature Ideal V2 at
temp1 Gas Gas temp1
cylinder

Plot of Pressure versus Volume of an


Ideal Gas at Constant Temperature
Pressure (atm)

Hyperbola

Volume (L)

Note : At any point on the curve

PV = constant When temperature is held fixed

This equation is the mathematical expression of BoyleÊs law ă at constant temperature the volume
occupied by a fixed weight of a gas is inversely proportional to the pressure exerted on it.

BoyleÊs law describes the behaviour of an ideal gas and approximates the behaviour of a real gas.
The approximation is very poor at high pressures and low temperatures.

(B ) Ch a r l e ’s L a w

Consider the change in volume of one mole of an ideal gas with the change in temperature when
the pressure is held constant.

GASEOUS STATE
8 QUIZRR
P1 P1

piston
Temp2 >Temp1

at constant
V1 at Ideal pressure Ideal V2 at
temp1 Gas Gas temp2
cylinder

Plots of Volume versus Absolute Temperature


of an Ideal Gas at Constant Pressures
Volume (L)

T (K)
in which the volume is directly proportional to the absolute temperature.
V = T ï constant when pressure is held fixed
This equation is the mathematical expression of CharleÊs law at constant pressure the volume
occupied by a fixed weight of a gas is directly proportional to the absolute temperature.
CharleÊs law describes the behaviour of an ideal gas and approximates the behaviour of a real
gas. The approximation is very poor at high pressures and low temperatures.

Ex a m p l e 1
If a 9.3 g sample of a gas occupies 12.3 L at 750 torr and 450 K. What volume will be gas
occupy at the same pressure and 25ĈC?
Solution : Since n and P are held fixed,

V1 V2
T1
= T = constant
2

GASEOUS STATE
QUIZRR 9

and V1 = 12.3 L, T1 = 450K, V2 = ?, and T2 = 25ĈC + 273 = 298 K. Substituting

V1 12.3 L V2 V2
T1
= 450 K
= = T2
298K

V2 = 8.20 L

(C ) Ga y -L u s s a c ’s l a w
In 1802 Joseph Gay - Lussac established a general relation between the pressure and temperature
of a gas. This is Gay-LussacÊs law and it states that :
„At constant volume, the pressure of a fixed mass of a gas is directly proportional to the Kelvin
temperature or absolute temperature.‰
The law may be expressed mathematically as

P1 P2
P  T or P = kT or P/T = constant or T  T
1 2

V2 > V1
V1

V2
P

(D ) A v o g a d r o ’s H y p o t h e s i s Re v i si t e d
At constant pressure and temperature the volume occupied by a gas is directly proportional to the
number of moles of the gas.
V = n ï constant when P and T are held fixed
Note : 1.00 mole of an ideal gas at 1.00 atm and 0ĈC (Standard Temperature and Pressure, STP)
occupies 22.4 L

(E) I d e a l Ga s L a w
Summary :

1
BoyleÊs : V when n and T are held fixed
P

Charles´ : V  T when n and P are held fixed


AvogadroÊs : V  n when P and T are held fixed

GASEOUS STATE
10 QUIZRR

1
Summary : V  T  n 
P

Replace  with = and the proportionality constant R


Ideal Gas Law : PV = nRT
where R is called the ideal gas constant.

Ph y s i c a l s i g n i f i c a n c e o f g a s c o n s t a n t R :

PV
R
nT

 force   force 
Pressure =  area    
   length2 

Volume = Length3

Thus R =
 force  length  (length) 
2
force  length
 amount of gas  (kelvin)  amount of gas  kelvin 

work (or energy)


=
 amount of gas  kelvin 
Thus, the dimensions of R are energy per mol per Kelvin and hence it represents the amount of
work (or energy) that can be obtained from one mole of a gas when its temperature is raised by one
Kelvin.

Values of R :
R = 0.0821 litre atm Kă1 molă1
= 8.314 ï 107 erg Kă1 molă1
= 8.314 joule Kă1 molă1
= 1.987 calorie Kă1 molă1

Ex a m p l e 2
Evaluate R if 1.00 mole of an ideal gas occupies 22.4 L at 1.00 atm and 0ĈC.
Solution :
Given P = 1.00 atm, V = 2.24 L, n = 1.00 mole, and T = 0ĈC + 273 K

P V 1.00 a t m  22.4 L  L  atm


R   0.0821
n T 1.00 m ole  273K  m ole-K

GASEOUS STATE
QUIZRR 11

R express in other units

L  atm J ca l
R  0.0821 or 8.314 or 1.987
m ole-K m ole  K m ole  K

where J (joules) and cal (calories are units of energy.

wt
The density of a substance is defined as d 
volu m e

For a gas the units of density are g/L

Ex a m p l e 3
Calculate the density of a gas (MW = 78.1 g/mole) at 740 torr and 98ĈC.
Solution :
Unknown : density (d) of the gas
Knowns : MW of the gas = 78.1 g/mole; P = (740 torr/atm) = 0.974 atm;
T = 98ĈC + 273 = 371 K
Concepts : PV = nRT; density = d = wt/volume

wt
Relationship : PV = nRT = RT
MW

 g 
wt  MW  P   (0.974a t m )
78.1
m ole 
  d
Rearranging : L  atm
V RT (0.821 )  371K 
m ole-K

g
d  2.50
L

3. CON CEPT OF I DEA L GA S


So for, we have assumed that all gases obey the gas laws under all conditions of temperature and
pressure; however, for real gases this is not true. Real gases obey these laws only under limited
condition of low pressures and high temperatures. They exhibit deviations from the gas laws and
these deviations are greater when the temperature and pressure are close to the condition at which
the gas can be condensed into a liquid. Thus BoyleÊs law, CharleÊs law and the equation of state derived
based on these laws may be regarded as approximation for real gases and are expended to be applicable
only at relatively low pressurs and moderately high temperature.

GASEOUS STATE
12 QUIZRR
3 .1 . GRA PH S

1  T, constant 
I. BoyleÊs law V   P1 V1 = P2 V2
P  n constant 

graphs are Isotherms

PV 45Ĉ P
P log nRT
log P
nRT
 [tan = nRT]
V P log V 1/V

Plot the different curves for difference values of n & V to compare.

 P, constant  V1 V2
II. CharleÊs law V T   
 n constant  T1 T2

graphs are Isobars

V V/T 45Ĉ
log V
nR nR/P
tan = log nR/P
 P
TK T log T

 V, constant  P1 P2
III. Gay LussacÊs law P T   
 n constant  T1 T2

graphs are Isochor

P P/T 45Ĉ
log P
nR nR/V
tan = log nR/V
 V
TK T log T

GASEOUS STATE
QUIZRR 13

V1 V2
IV. Gay L u ssac’s l aw V n (T, P constant) 
n1 n2

graphs are Isochor

V V/n 45Ĉ
log V
RT RT/P
tan = log RT/P
 P
n log T
n

Ex a m p l e 3
5 g of ethane are confined in a bulb of 1 litre capacity. The bulb is so weak that it will burst
if the pressure exceeds 10 atm. At what temperature will be the pressure of gas reach the
bursting value ?
Solution : w = 5 g, m = 30 for C2H6, V = 1 litre
Let the bulb burst at T Kelvin, i.e., when pressure becomes 10 atm

w
 PV = RT
m

5
10 ï 1 =  0.821  T
30

 T = 730.81 K
= 457.81ĈC

Ex a m p l e 4
An iron cylinder contains helium at a pressure of 250 kPa at 300 K. The cylinder can
withstand a pressure of 1 ï 106 Pa . The room in which cylinder is placed catches fire.
Predict whether the cylinder will blow up before it melts, or not, melting point of cylinder
= 1800 K.
Solution :
Given, P1 = 250 kPa T1 = 300 K
P2 = 1 ï 106 Pa T2 = ?
Since volume of cylinder remains constant.

P1 P2
Therefore, =
T1 T2

GASEOUS STATE
14 QUIZRR

250  10 3 1  10 6
=
300 T2

 T2 = 1200 K
The cylinder will blow up at 1200 K before its melting (m.pt. 1800 K).

Ex a m p l e 5
A gas occupies 300 mL at 27ĈC and 730 mm pressure. What would be its volume at STP?
Solution :

300 730
Given, initially V2 = litre, P2 = atm, T2 = 300 K
1000 760
At STP V1 = ?, P1 = 1 atm, T1 = 273 K

P2 V2 P1 V1
Now use, T2
= T1

V1 = 0.2622 litre
 Volume (V) at STP = 262.2 mL

Ex a m p l e 6
A gas at 0ĈC and 1 atm pressure occupies 2.5 litre. What change in temperature would be
necessary if the pressure is to be adjusted to 1.5 atm and the gas has been transferred to
a 2.0 litre container ?
Solution :
Given P1 = 1 atm, V1 = 2.5 litre, T1 = 273 K,
P2 = 1.5 atm, V2 = 2.0 litre, T2 = ?

P1 V1 P2 V2
Now use, =
T1 T2

 T2 = 327.6 K or 54.6ĈC

Ex a m p l e 7
Estimate the number of gaseous molecules left in a volume of 1 mm3 if it is pumped out to
give a vacuum of 10ă6 mm Hg at 298 K.
Solution :
P = 10ă6 mm of Hg

10 6 of Hg
=  1.315  109 atm
760 mm of Hg

GASEOUS STATE
QUIZRR 15
V = 1 mm3 = 1 ï (103 m)3
= 10ă9 m3 = 10ă9 ï (10 dm)3 = 10ă6 dm3 = 10ă6 L

PV  
1.135  109 atm 106 L 
n = RT 
 
0.0821atm L mol 1 K 1  298 K 

= 5.38 ï 10ă17
Number of molecules = 5.38 ï 10ă17 ï 6.023 ï 1023
= 3.24 ï 107

Ex a m p l e 8
When 2 g of gaseous substance A is introduced into an initially evacuated flask kept at
25ĈC, the pressure is found to be 101.325 kPa. The flask is evacuated and 3 g of B is
introduced. The pressure is found to be 50.602 kPa at 25ĈC. Calculate the ratio MA/MB.
Solution :
PV = nRT

w
PV = RT
M

PA = 101.325 kPa = 1 atm


W A = 2g, T = 298 K

2g 1 1
(1 atm) (V litres) = M g/mol  0.082 L atm mol mol K  298K.....(i)
A

50.602
PB = 50.662 kPa = atm = 0.5 atm
101.325

T = 298 K, WB = 3g

3g
 (0.5 atm) (V litres) =  0.082 L atm mol 1K 1  298K.....(ii)
MB g/mol

Dividing (i) by (ii)

1atm 2 g (MB g / mol)


0.5atm
= 3 g (M A g/mol)

M A (g / mol) 2 g 0.5 atm 1


=  
MB (g/mol) 3g 1atm 3

GASEOUS STATE
16 QUIZRR
Ex a m p l e 9
1.47 litre of a gas is collected over water at 30ĈC and 744 mm of Hg. If the gas weighs 1.98
g and vapour pressure of water at 30ĈC is 32 mm, what is the molecular weight of gas ?
Solution :
w = 1.98 g, V = 1.47 litre, T = 303 K

712
P = 744 ă 32 = 712 mm = atm
760

w
 For dry gas, PV = RT
m

712 1.98
ï 1.47 = ï 0.0821 ï 303
760 m

 m = 35.76 g

Ex a m p l e 1 0
A volume of 95 mL N2O at 27ĈC is collected in a graduated tube over mercury, the level
of Hg inside the tube being 60 mm above the outside mercury level when barometer reads
750 mm,
(a) Calculate the volume of the same mass at STP,
(b) What volume would the same mass of gas occupy at 40ĈC when the barometric pressure
being 745 mm and the level of Hg inside the tube 25 mm below the outside level ?
Solution :
(a) At STP

95
V1 = 95 mL = litre V2 = ?
1000
T1 = 300 K T2 = 273 K

690 760
P1 = 750 ă 60 = 690 mm = atm P2 = atm = 1 atm
760 760

P1 V1 P2 V2
 T1
= T
2

690  95 1  V2
=
760  1000  300 273

 V2 = 0.07848 litre
= 78.48 mL

GASEOUS STATE
QUIZRR 17

(b) V2 = ? T2 = 313 K

770
P2 = 745 + 25 = 770 mm = atm
760

P1 V1 P2 V2
Again using =
T1 T2

690  95 770  V2
=
760  300  1000 T2

V2 = 0.0888 litre
 = 88.8 mL

Ex a m p l e 1 1
An LPG (liquified petroleum gas) cylinder weighs 14.8 kg when empty; when full, it weighs
29.0 kg and shows a pressure of 2.5 atm. In the course of use at 27ĈC, the weight of full
cylinder is reduced to 23.2 kg. Find out the volume of the gas in cubic metres used up at
the normal usage conditions, and the final pressure inside the cylinder. Assume LPG to be
n-butane with normal boiling point of 0ĈC.
Solution :
Weight of butane in cylinder = 29.0 ă 14.8 = 14.2 kg = 14.2 ï 103 g
P = 2.5 atm, T = 300 K, M. wt. of butane = 58

w
 PV = RT
m

14.2  103
2.5 ï V =  0.0821  300
58

 V = 2.4 ï 103 litre = 2.4 m3


This is volume of cylinder or volume of gas.
Now the wt. of gas left after use = 23.2 ă 14.8 = 8.4 kg = 8.4 ï 103 g
The volume remains constant

w
Again using PV = RT
m

8.4  103
P ï 2.412 ï 103 =  0.0821  300
58

 Pressure (P) of gas left in cylinder = 1.50 atm

GASEOUS STATE
18 QUIZRR
Now Pressure of gas given out = 1
Wt. of gas given out = 29.0 ă 23.2
= 5.8 kg = 5.8 × 103 g
Thus, volume of gas given out under these conditions is

5.8  103
 1ï V =  0.0821  300
58

 V = 2.4 ï 103 litre = 2.4 m3

Ex a m p l e 1 2
A mixture of Na2CO3 and NaHCO3 has a mass of 22 g. Treatment with excess HCl solution
liberates 6 L of CO2 at 25ĈC and 0.947 atm pressure. Determine the percent Na2CO3 in the
mixture.
Solution :
Na2CO3 + 2HCl  CO2 + H2O + 2NaCl
NaHCO3 + HCl  CO2 + H2O + NaCl
The number of mol. of CO2 produced is equal to the number of mol of Na2CO3 plus number of
mol of NaHCO3 initially present

PV 0.947  6
n   0.232 mol
RT 0.0821  298

Let x = wt. of Na2CO3, then 22 ă x = wt. of NaHCO3. The number of mol of each reactant is equal
to the mass of divided by the formula weight.

x 22  x
  0.232
106 84

22 ă x = 9.9 g
x = 12.1 g

12.1
 100  55% Na 2CO3
22

Ex a m p l e 1 3
A jar contains a gas and a few drops of water. The pressure in the jar is 830 mm of Hg. The
temperature of the jar is reduced by 1%. The vapour pressure of water at two temperatures
are 30 and 25 mm of Hg. Calculate the new pressure in the jar.

GASEOUS STATE
QUIZRR 19

Solution :
At T K
Pgas = Pdry gas
+ Pmoisture
Pdry gas
= 830 ă 30 = 800 mm

T
Now at new temperature T1 = T ă  0.99 T
100

P P1 P2
Since, V1 = V2; constant or T  T
T 1 2

800  0.99 T
Pdry =  792 mm
gas T

Pgas = Pdry gas


+ Pmoisture
= 792 + 25
= 817 mm of Hg

Ex a m p l e 1 4
Two flasks of equal volume connected by a narrow tube (of negligible volume) are at 27ĈC
and contain 0.70 mole of H2 at 0.5 atm. One of the flask is then immersed into a bath kept
at 127ĈC, while the other remains at 27ĈC. Calculate the final pressure and the number of
mole of H2 in each flask.
Solution :

I II

Two flasks initially at 27ĈC and 0.5 atm, have same volume and 0.7 mole; thus each flask has
0.35 mole.
Let n mole of gas are diffused from II to I on heating the II flask at 127ĈC.
 Mole in I flask = (0.35 + n)
and Mole in II flask = (0.35 ă n)
Thus, if new pressure of flask is P, then
For I Pnew ï V = (0.35 + n) ï R ï 300
For II Pnew ï V = (0.35 ă n) ï R ï 400
 n = 0.05
 Mole in flask I = 0.35 + 0.05 = 0.40
GASEOUS STATE
20 QUIZRR
and Mole in flask II = 0.35 ă 0.05 = 0.30
Also 0.5 ï 2 V = 0.7 ï 0.821 ï 300 (initial condition)
 V = 17.241 litre
Thus, Pnew ï 17.241 = 0.30 ï 0.0821 ï 400
 P = 0.5714 atm

Ex a m p l e 1 5
A vessel contains 7.1 g chlorine gas at pressure P and temperature T K. On heating the
vessel to 30Ĉ higher temperature, 246 mL of chlorine at 1 atm and 27ĈC is taken out to
maintain same pressure in vessel. Calculate,
(a) the original temperature,
(b) If the gas is not allowed to escape out, the pressure increases by 0.11 atm. Calculate
the volume of vessel and initial pressure.
Solution :
(a) Mole of gas coming out on heating

PV 1  0.246
n = =  0.009987
RT 0.0821  300

7.1
Thus, mole of gas left =  0.009987
71

= 0.1 ă 0.009987 = 0.09


Also for 0.1 mole of gas PV = 0.1 RT ...(1)
for 0.09 mole of gas PV = 0.09R(T + 30) ...(2)
By Eqs. (1) and (2)  T = 270 K
(b) Also if gas is not allowed to escape, then for 0.1 mole of gas
P ï V = 0.10 ï R ï 270 ...(3)
(P + 0.11)V = 0.10 ï R ï 303 ...(4)
By Eqs. (3) and (4)  P = 0.99 atm
By Eq. (3) 0.99 ï V = 0.10 ï R ï 270
 V = 2.239 litre

Ex a m p l e 1 6
A glass capillary tube sealed at both ends is 100 cm long. It lies horizontally with the
middle 10 cm containing Hg. The two ends of the tube which are equal in length contain
air at 27ĈC and pressure of 76 cm of Hg. The tube is kept in a horizontal position such that

GASEOUS STATE
QUIZRR 21

the air column at one end is at 0ĈC, the other end is maintained at 127ĈC. Calculate the
length of the air column and its pressure which is at 0ĈC. Neglect the change in volume of
Hg and glass.
Solution :
According to problem
Initially 2L + 10 = 100
 L = 45 cm

A B A B
L L (45ăx) (45+x)
P0 P0 273 K 400 K
10cm 10 cm

Let initial pressure be P0 atm on each side. When one end is cooled and other is heated, expansion
of gas occurs at hoter end till the pressure on two sides becomes same. Let Hg column is displaced by
x cm to cooler end, i.e.,
P1 = P2 = P

P0  45  a P1 (45  x)  a
Now for end A : = ...(1)
300 400
because moles of gas remains same at 273 and 300 K; a is area of cross section of tube.

P0  45  a P2 (45  x)  a
For end B : = ...(2)
300 400
Evaluating P1 and P2 from Eqs. (1) and (2) and since
P1 = P2

273  45  P0 400  45  P0
 300   45  x = 300   45  x

 x = 8.49 cm
Thus, length of air column at 0Ĉ C = 45 ă 8.49 = 36.51 cm
length of air column at 127Ĉ C = 45 + 8.49 = 53.49 cm

273  45  76
Also pressure (P) = P1 =  85.25 cm of Hg
300  36.51

Ex a m p l e 1 7
A 10 cm column of air is trapped by a column of Hg, 8 cm long is capillary tube horizontally
fixed as shown below at 1 atm pressure. Calculate the length of air column when the tube
is fixed at same temperature.

GASEOUS STATE
22 QUIZRR
(a) Vertically with open end up
(b) Vertically with open end down
(c) At 45Ĉ from horizontal with open end up.
Solution :
(a) P1V1 = P2V2
or P1l1a = P2l2a
10 cm 8 cm
 P2 = 76 + 8 = 84 cm

P1 l1 76  10
 l2 =   9.05 cm
P2 84

(b) P1l1a = P2l2a


 P2 = 76 ă 8 = 68 cm

P1 l1 76  10
 l2 =   11.18 cm
P2 68

(c) When the tube is held at 45Ĉ with open end up, the weight of Hg is borne partially by the
gas and partially by the gas and partially by the Hg. Vertically height of Hg is a measure
of additional pressure on gas, i.e.,

8
 l =
2

Also P2l2a = P1l1a 8


a
P1 l1 76  10

 l 2 = P2 8
76 
2
a

= 9.3 cm

Ex a m p l e 1 8
A column of Hg of 10 cm in length is contained in the middle of a narrow 1 m long tube
which is closed at both ends. Both the halves of the tube contained air at a pressure 76 cm
of Hg. By what distance will the column of Hg be displaced if the tube is held vertical ?
Solution :
Let initially the length of air column on each side be L, then
2L + 10 = 100 A B

 L = 45 cm L L
P0 P0
10 cm
GASEOUS STATE
QUIZRR 23

If the tube is held vertically, let the Hg column be displaced downwards by y to attain same
pressure above and below the column of Hg. Then
PB + 10 = PA ...(1)
Pressure are taken in terms of length of Hg.
For end A : Since mole remains same on two sides

P0  L  a PA   L  y  a
= ...(2)
RT RT

LP0
 PA = L  y ...(3)
 
For end B :

P0  L  a PB   L  y  a
= ...(4)
RT RT B

(L+y)
LP0
 PB = L  y ...(5)
 
By Eqs. (1), (3) and (5)

 LP0  LP0
   10 
 L  y  L  y

(Lăy)
Putting L = 45 cm and P0 = 76 cm A
2 2
y + 684 ă (45) = 0
y = 3 cm

4. DA L T ON ’S L A W OF PA RT I A L PRESSU RE
The total pressure exerted by a mixture of gases is the sum of the partial pressures of the
individual gases. The partial pressure of a gas is the pressure that the gas would exert if it were alone
in the container at the same temperature.

PT = PA + PB +

Total Pressure
Partial Pressure of Gas B
Partial Pressure of Gas A

GASEOUS STATE
24 QUIZRR
Assuming ideal gas behavior
Total pressure P = PA + PB
 RT   RT 
PT = PA + PB = nA    nB  V  PA PB
 V   

 RT  Pressure
PT = (nA + nB)  
 V 

nA nB
PA  Pt ; PB  Pt 0 1
nA  nB nA  nB Mole fraction of B (xB)

PA = xA ï Pt and PB = xB ï Pt (where x stands for mole fraction and pronounced as ÂkiÊ).


Partial pressure of gas = its mole fraction ï Total pressure

Partial pressure of gas


% of gas in mixture =  100
Total pressure

In the laboratory, it is often convenient to collect a gas over water. When a gas is collected over
water, the observed pressure of the gas is equal to the sum of the pressure of dry gas and the pressure
of water vapour.
Therefore, Pdry gas
= Pmoist gas
ă Pwater vapour
Pressure exerted by water vapour is called aqueous tension.
Collection of a gas over water

Bottle PT = PGas + PH
2O

Gas + Gas
water vapor

Water Patm

Water

When the water level inside the bottle equals the water level outside the bottle, then
Patm = PT = PGas + PH
2O
where Patm is the atmospheric pressure, PT is the total pressure of the gases above the water
inside the bottle, PGas is the partial pressure of the collected gas, and PH2O is the vapour pressure of
water.

GASEOUS STATE
QUIZRR 25

Ex a m p l e 1 9
40.0 L sample of N2 is collected over water at 22ĈC and an atmospheric pressure of 727 torr.
Calculate the volume that the dry N2 will occupy at 1.00 atm and 0ĈC. The vapour pressure
of water is 20 torr at 22ĈC.
Solution :
Unknown : V2 = volume of the dry N2 at 1.00 atm and 0ĈC
Knowns : Patm = 727 torr, V1 = 40.0 L, temp. = 22ĈC,
PH2O = 20 torr at 22ĈC and moles of N2 are constant.
Concepts : Ideal gas law (PV = nRT), DaltonÊs law of partial pressures

Patm = 727 ton Patm = 1.0 atm

Piston
N2 N Remove H2 O
N2 H2 O 2
N2 N2 N2
V1 = 40.0L N2 N2 N2 N2 Change T & P N2 N2 V2 = ?
H2 O N2 at 0ĈC
at 22Ĉ C N2 N2 N2 N2
Cylinder
Relationships :

 PN2 1 V1 nN2 RT2


Patm = (PN )1 + PH O; nN =
2 2 2 RT1
; V2 =  PN2 2
Substituting the center equation for nN2 in the equation on the right

 (PN2 )V1   RT2  (PN2 )1 V1 T2 (Patm  PH2O )V1 T2


V2 =  RT   (P )   T (P )  T1 (PN2 )2
 1   N2 2  1 N2 2

(Patm  PH2O )V1 T2 (727 torr  20 torr)(40.0 L)(273K)


V2 =   34.4 L
T1 (PN2 ) 2 (295K)(760 torr)

Ex a m p l e 2 0
Dry air has composition 78 mol % nitrogen, 21 mol % oxygen and 1 mol % other gases.
Calculate the partial pressures of the components when the total pressure is 1 atm.

78
Solution : XN (mole fraction of N2) =  0.78
2 100

GASEOUS STATE
26 QUIZRR
because mole percent = mole fraction ï 100

21
X O2   0.21
100

1
X other gases   0.01
100

Pt = Xt Ptotal
 PN = 0.78 ï (1 atm) = 0.78 atm
2
PN = 0.21 ï (1 atm) = 0.21 atm
2
Pother gases
= (0.01) ï (1 atm) = 0.01 atm

5 . GRA H A M ’S L A W
Diffusion : When two or more non-reacting gases are kept side by side, they have the tendency
to mix with one another spontaneously and form a homogeneous mixture. This phenomenon can occur
even against gravity. This mixing ability of gases is known as diffusion.
Effusion : It is a process in which a gas is allowed to escape under pressure gradient through
a fine hole (orifice).
The difference between diffusion and effusion is that the former occurs even when there is no
pressure gradient, but the latter continues to occur only as long as there is pressure gradient. Otherwise,
both processes are essentially the same.

Ra t e o f d i f f u s i o n
It can be defined as
(a) Volume or number of moles of gas diffused per unit time
(b) Distance travelled by a gas per unit time through a tube of uniform crossăsection.

Gra h a m ’s l a w o f d i f f u s i o n o r e f f u s i o n
GrahamÊs law states that, „at constant pressure and temperature, the rate of diffusion or effusion
of a gas is inversely proportional to the square root of its density‰.

1
Rate of diffusion 
d

if r1 and r2 represent the rates of diffusion of two gases and d 1 and d 2 their respective densities,
then

r1 d2
=
r2 d1

GASEOUS STATE
QUIZRR 27

This equation can be written as :

density of gas II V.D. of gas II



r1 density of H2 V.D. of gas I
= density of gas I
r2
density of H2

2  V.D. of gas II Mol. wt. of gas II


= 
2  V.D. of gas I Mol. wt. of gas I
When a gas at a pressure P and temperature T is separated from a vacuum by a small hole, the
rate of escape of its molecules is equal to the rate at which they strike the area of the hole.
PA 0 PA 0 N A

Therefore, for a hole of area A0, rate of effusion =
 2MKT  1/ 2
 2MRT 1 / 2
Note : (1) It should be noted that the rate of diffusion or effusion actually depends on pressure
difference of the gas and not simply on its pressure. Moreover the pressure difference is to be measured
for this gas only i.e. if a container holds [He] at a pressure of 0.1 atm and if a small pin-hole is made
in the container and if the container is placed in a room, then the rate of effusion of He gas from the
container to outside depends only on its pressure difference, which is 0.1-0 (as there is no He in the
atmosphere). This implies that the diffusion of a gas is not dependent on the diffusion of any other gas.
(2) Whenever we consider the diffusion of a gas under experimental conditions, we always assume
that the gas diffuses in vaccum and during the time period for which the diffusion is studied the rate
of diffusion (or the composition of diffusing or effusing mixture of gases) remains constant.

M is c e l la n e o u s c o n c e p t s u s e d i n Ga s e o u s St a t e
(a) Bursting of containers : two concepts used depending upon type of container.
(i) Bubble type (very thin skin) cannot tolerate difference in pressure on the skin outside
pressure = inside pressure
Any change in these cause change in volume & the container burst due to maximum
stretching.
(ii) Cylinder type (thick skin) can withstand pressure difference till a limit but cannot
have volume change.
Any change cause a change in pressure & when it exceeds the limits the container
burst.
(b) Connecting containers having gases
A B

On removal of nozzel the gas from higher pressure will travel so as to have equal
pressure at both the containers... from idea of total moles & final temperature each
parameter can be calculated.

GASEOUS STATE
28 QUIZRR
Ex a m p l e 2 1
Two containers A and B of equal volume hold 3 mol of N2 and 5 mol of O2 respectively at
the same temperature. Both containers are kept separately in vacuum. Calculate the ratio
of rates of N2 to that of O2 if the area of orifice in both containers is same.
Solution :
3 mol 5 mol
N2 O2

A B
As p  n

PN2 3

PO2 5

rN2 PN2 MO2 3 32


 
rO2 PO2 MN2 5 28

3 8
=  0.6474
5 7

Ex a m p l e 2 2
A mixture containing 1.12 litres of H2 and 1.12 litres of D2 (deuterium) at N.T.P. is taken
inside a bulb connected to another bulb by a stop-cock with a small opening. The second
bulb is fully evacuated, the stop-cock is opened for a certain time and then closed. The first
bulb is now found to contain 0.05 g of H2. Determine the percentage composition by weight
of the gases in the second bulb.
Solution :

22.4  0.05
Volume of H2 left in the first bulb after diffusion = 0.56 litres at N.T.P.
2

Volume of H2 diffused = 1.12 ă 0.56 = 0.56 litres.


Let the volume of D2 diffused be v2.

v1 MO2 0.56 4
According to GrahamÊs law of diffusion, v  MH2 or v2 = (at mass of D2 being 2)
2 2

v2 = 0.396 litres at N.T.P.

GASEOUS STATE
QUIZRR 29

0.396  4
Wt. of H2 in second bulb = 0.05 g. Wt. of D2 in the second bulb =  0.07g
22.4

0.05  100
Percentage of H2 =  41.67
0.12

Percentage of D2 = 100 ă 41.67


= 58.33

6. PAY-LOAD
When a balloon is buoyant, it can take along some weight into the upper atmosphere. The maximum
weight a balloon can carry along is called its pay-load.
Pay load = weight of the air displaced ă (weight of the balloon + weight of the gas it contains)
Pay load/lifting power [based on Buoyancy]
LP of balloon = V (d ă d 1)g ă Mg
V = Volume of balloon
d = density of outside gas
d 1 = density of gas in the balloon
M = Mass of balloon

Ex a m p l e 2 3
A balloon of diameter 20 m weights 100 kg. Calculate its pay-load, if it is filled with He at
1 atm and 27ĈC. Density of air is 1.2 kgmă3.
Solution :
Weight of balloon = 100 kg = 10 ï 104 g

3
4 3 4 22  20 
Volume of balloon = r =    100  = 4190 ï 103 L
3 3 7  2 

PVm 1  4190  103  4


Weight of gas (He) in balloon =   68.13  104 g
RT 0.082  300

 Total weight of gas and balloon = 68.13 ï 104 + 10 ï 104 = 78.13 ï 104 g

1.2  4190  106


Weight of air displaced = 3
 502.8  104 g
10

Payload = weight of air displaced ă (weight of balloon + weight of gas)


Pay load = 502.8 ï 104 ă 78.13 ï 104 = 424.67 ï 104 g

GASEOUS STATE
30 QUIZRR
7. K I N ET I C T H EORY OF GA S
After knowing the experimental gas laws, it is of interest to develop a theoretical model based on
the structure of gases, which can correlates are the experimental facts, fortunately such a theory has
been developed (known as the kinetic theory of gases) it is possible to derive an expression (known as
the kinetic gas equation) from where all these gas laws can be derived. Essential postulates are :
Ć A gas consists of a large number of very small spherical tiny particles, which may be identified
with the tne molecules. The molecules of a given gas are completely identical in size, shape
and mass.
Ć The volume occupied by the molecules is negligible in comparison to the volume of the gas.
Ć The molecules are in rapid motion which is completely random. During their motion, they
collide with one another and with the sides of the vessel.
Ć The laws of classical mechanics, in particular NewtonÊs second law of motion, are applicable
to the molecules in motion.
Ć There are no force of attraction or repulsion amongst the molecules, i.e., they are moving
independent of one another.
Ć At any instant, a given molecule can have energy, ranging from a small value to a very large
value, but the average kinetic energy remains constant on a given temperature, i.e. The
average kinetic energy is proportional to the absolute temperature of the gas.

7 .1 De r iv a t io n o f t h e K in e t ic T h e o r y o f Ga s e s
An ideal gas is characterized as a gas that consists of molecules with mass and velocity but no
volume. These molecules exhibit no attractive or repulsive forces among themselves or with other
matter.
Consider a collection of N identical ideal gas molecules in a container with volume V.
V = xyz = volume of the box
N = number of identical molecules
m = mass of one molecule
z
c1 = velocity of the 1th molecule
N molecules colliding with the yz wall exert a force
y
2
mNc x
force =
3x

c12  c22  c32  ...  c12  ...  cN


2
where c2 = mean square velocity =
N

(Note : A bar over a symbol, symbol indicates the mean or average value.)

GASEOUS STATE
QUIZRR 31

Pressure is force/area

force mNc2 mNc2


P  
area 3 xyz 3V

and rearranging

mNc2
PV  (1)
3

If N is AvogadroÊs Number, NA, then n = 1 and PV = RT. Substituting RT for PV and NA for N
in eq. 1

mNA c2
RT  (2)
3

Note : If T is increased then c2 will increase and the volume V of the container must increase
if P is to remain constant.

Ex a m p l e 2 4
Calculate the pressure exerted by 1023 gas particles each of mass 10ă22 g in a container of
volume 1 dm3 . The root mean square speed is 105 cm să1.
Solution :
From the given data, we have
N = 1023
m = 10ă22 g = 10ă25 kg
V = 1 dm3 = 10ă3 m3

u2 = 105 cm să1 = 103 msă1

Therefore, from the kinetic gas equation

   
2
1 1025 kg 1023 103 ms1
P =
3 (10 3 3
m )

1
3
 
107 kgm 1s2

1
P =  107 Pa
3

GASEOUS STATE
32 QUIZRR
7 .2 De r iv a t i o n f r o m t h e K in e t ic Ga s Eq u a t i o n
The kinetic gas equation can be used to derive various gas laws and to define expressions for some
useful quantities such as the root mean square speed and the average kinetic energy. Before deriving
these, it is helpful to write this equation in the following form,
Average kinetic energy  T

1 1
mu2  T or m u2  KT
2 2

(where K is the proportionality constant)


Introducing this in equation we have

1 2 1 
PV  mNu2  N  mu2 
3 3 2 

2
PV  NKT
3

Now, we proceed to derive the various gas laws from equation.

B o y l e ’s L a w
The essential conditions for BoyleÊs law to be applicable are :
(i) Temperature (T) should remain constant.
(ii) Mass of the gas should remain constant in other words, the total number of molecules (N)
remains unchanged.
Under these conditions,

1
PV = constant or P 
V1

Which is the expression for BoyleÊs law.

Ch a r l e ’s L a w
In this case
(i) Pressure (P) remains fixed.
(ii) Mass of the gas remains unchanged, i.e., N is constant with these conditions,

 2 NK 
V  T i.e. V = (constant) T or V  T
3 P 
as required by CharleÊs law.

GASEOUS STATE
QUIZRR 33

A v o g a d r o ’s L a w
It states that under similar conditions of pressure and temperature, equal volume of all gases
contains equal number of molecules. Considering two gases, we have

2 2
P1 V1  N1 KT and P V = N KT
3 1 2 3 2 2

Since P1 = P2 and T1 = T2, therefore

P1 V1  2 / 3  N1 KT1 V N
  1  1
P2 V2  2 / 3  N2 KT2 V2 N2

If volumes are identical, obviously N1 = N2.

7 .3 K in e t ic En e r g y o f T ra n s l a t io n
The average kinetic energy of translation per molecule in a collection of AvogadroÊs Number of
molecules is

mc2
Average Kinetic Energy per Molecule =  
2

The kinetic energy of translation for AvogadroÊs Number of molecules is

mN A c 2
Kinetic Energy of Translation per Mole = ET  N A  
2

3RT
Substituting 3RT for mN A c2 , gives ET 
2

The average kinetic energy of translation per molecule is

  E T  3RT  3kT
N A 2N A 2

where k, call BoltzmannÊs constant, is the ideal gas constant per molecule.

J
k = 1.38.10ă23
molecule  K

7 .4 K in d s o f m o l e c u l a r s p e e d s
Molecular speeds are of 3 types :
(i) The RMS speed
(ii) The average speed
(iii) The most probable speed

GASEOUS STATE
34 QUIZRR
(i) The root mean square speed (Crms) : This is the type of speed we used in kinetic gas

1
equation, PV  mnc2 . It is the hypothetical speed possessed by the all the gas molecules
3
when the total kinetic energy is equally distributed amongst them.
Total kinetic energy of a sample containing n number of molecules

1 1 1
= mc12  mc22  .......  mcn2
2 2 2

If velocity possessed by all the molecules is same and that is c,

1
Total kinetic energy = n  mc2
2

1 1 1 1
n mc2 = mc12  mc22  .......  mcn2
2 2 2 2

c12  c22  .....  cn2


c2 =
n

c12  c22  .....  cn2


c=
n

Thus RMS velocity is defined as the square root of the mean of the squares of the speeds
of all the molecules present in the given sample of the gas. The value of c is determined by
using the following expressions.

3PV 3RT 3P
c  
M M d

In applying any of these equations care must be taken to use consistent units. If SI units
are used for R (= 8.31 kgm2/(s2 k mol), T(K) and M(kg/mol), the rms speed will be in meters
per second.
(ii) Average speed (Cav) : It is arithmetic mean of various speeds of the molecules.

c1  c2  .....  cn
Average speed c 
n

8RT
It is equal to
M

GASEOUS STATE
QUIZRR 35

(iii) Most probable speed (Cmp) : This is defined as the speed possessed by maximum number
of a gas at a given temperature.

2RT
It is equal to
M

The three kinds of molecular speeds are related to each other as :


Most probable speed : Average speed : RMS speed

8
= 2: : 3

Cmp : Cav : Crms : 1 : 1.128 : 1.224

M a x w e l l’s d i s t r ib u t io n o f m o l e c u l a r v e l o c i t ie s
Fraction of molecules

Most probable velocity

T1 T3 > T2 > T1
T2

T3

Velocity

From the above graph, it is clear that the fraction of molecules increases with increase in velocity,
reaches a maximum and then falls off towards zero. The important features of the curves are as
follows :
(i) The fraction of molecules with too low or too high velocities is very small.
(ii) There is certain velocity for which the fraction of molecules is maximum. This is called most
probable velocity.
The most probable velocity increases with rise in temperature. The entire distribution curve shifts
to the right with rise in temperature. The rise in temperature, therefore, increases the fraction of
molecules having high velocities considerably.

Ex a m p l e 2 5
For a gas containing 1023 molecules (each having mass 10ă22 g) in a volume of 1 dm3, calculate
the total kinetic energy of molecules if their root mean square speed is 105 cm să1. What will
be its temperature ?

GASEOUS STATE
36 QUIZRR
Solution :

1
   
1 2
2 25
Total kinetic energy (KE) = N  mu   10
23
 2 10 kg  103 ms 1 
2   

= 0.5 ï 104 kg m2 să2 = 0.5 ï 104 J


Total kinetic energy is also equal to N(3/2)RT
Thus N(3/2)kT = 0.5 ï 104 J

2 (0.5  104 J) 2 (0.5  104 J)


Hence T = 3 
kN 3 (1023 ) (1.38  1023 JK 1 )

= 2414 K

Ex a m p l e 2 6
Calculate the total kinetic energy of 0.5 mol of an ideal gas at 273 K.
Solution :

3  3
 1
Total Kinetic Energy = n  RT  = (0.5 mol)   8.314 JK mol
2  2
1 
 (273 K) 


= 1702 J

Ex a m p l e 2 7
The average speed at T1K and the most probable speed at T2 K of CO2 gas is 9 ï 104 cm secă1.
Calculate the value of T1 and T2.
Solution :

 8RT 
Average speed =  
 M 

 2RT 
Most probable speed =  
 M 

Average speed at T1 K = MP speed at T2 K for CO2

 8RT1   2RT2 
 M  =  M 
   

T1 
 T2
= ...(1)
4

GASEOUS STATE
QUIZRR 37

 2RT  4
Also for CO2 uMP =    9  10
 M 

 2  8.314  107  T2  4
    9  10
 44 
 

 T2 = 2143.37 K
 By Eq. (1) T1 = 1684.0 K

Ex a m p l e 2 8
The average speed of an ideal gas molecule at 27ĈC is 0.3 m secă1. Calculate average speed
at 927ĈC.
Solution :

 8RT 
uAV =  
 M 

Given uAV = 0.3 m secă1 at 300 K

 8R  300 
 u 1 = 0.3 =   ...(1)
 M 

at T = 273 + 927 = 1200 K

 8R  1200 
 u2 =   ...(2)
 M 

u2  1200 
 =  
0.3  300 

 u 2 = 0.6 m secă1

7 .5 De v i a t i o n f r o m I d e a l B e h a v i o u r (Re a l Ga s e s )
If measurements of pressure, volume and temperature of a gas do not confirm to the ideal gas
law (PV = RT), within the precision of measurements, the gas is said to deviate from ideal behavior and
exhibit non-ideal behaviour. Such gases are called real gases. At low pressures and moderately high
temperatures, real gases obey ideal gas equation but as the pressure is increased or the temperature
is decreased, a marked deviation from ideal behaviour is observed.

GASEOUS STATE
38 QUIZRR
To display the deviation more clearly, the ratio of the observed molar volume (Vm) to the ideal
 RT 
molar volume  Vm , ideal   is plotted as a function of pressure at constant temperature. This ratio
 P 
is called the compressibility factor (Z), which is expressed as

Vm P  Vm PV
Z  
Vm ,ideal RT nRT

For an ideal gas Z = 1 and is independent of pressure and temperature. For a real gas, Z = f
(T, P), a function of both temperature and pressure.

t = 0Ĉ C
H2

Z CH4
CO2

1.0 Ideal gas

0
100 200 300
P in atm
Above plot of Z as a function of pressure at varying temperature implies that curve for a real gas
has a tendency to coincide with that of an ideal gas at high temperature, i.e., at extremely high
temperature; the behaviour of a real gas approaches that of an ideal gas.

T1 > T2 > T3 > T4


T4

Z T3
H2
T2
T1
1.0 ideal gas

0
200 400 600
P in atm

GASEOUS STATE
QUIZRR 39

For hydrogen, Z is greater than unity (ideal value) at all pressures. For CH4 and CO2, Z is less
than unity in the lower part of the pressure ranges but is greater than unity at very high pressures.
For gases, which are easily liquefied, Z dip sharply below the ideal line, i.e., Z decreases with increase
of pressure in the lower pressure range, passes through a minimum at some pressure and then
increases continuously with pressure in the higher pressure region.

Re a s o n s f o r d e v i a t i o n s :

In order to explain the deviations from ideal behaviour, it is necessary to have a look at the kinetic
theory of gases. The following two postulates of kinetic theory do not hold good under all conditions.
Postulate no. 1 : The volume occupied by the molecules themselves is negligibly small as
compared to the total volume occupied by the gas. This can be justified when pressure is low or
temperature is high. But, if the pressure becomes too high or the temperature is too low, the volume
occupied by the molecules will no longer be negligible.
Postulate no. 2 : The forces of attraction between gas molecules are negligible. Again this
postulate is not valid at high pressures and low temperatures.

Co r re c t io n f o r Fo r c e s o f a t t r a c t io n :
Consider molecules of layer A & B. As can be seen in Fig., molecules of layer A, which are closest
to the container wall strike the wall of the container and theory exert a pressure P on it. This is the
pressure exerted by the molecules of the eral gas. The molecules of layer B attract the molecules of layer
A but do not hit the container wall. This backward pull experienced by the molecules of layer A causes
them to exert less pressures than what they would have exerted if there were no attractive forces.
AB

Therefore, if a real gas exerts pressure P, then an ideal gas would exert pressure P + p where
p is the pressure lost by the real gas due to attractive forces.
The attractive force is mutual. That is to say that this force is between layer A and layer B
molecules. Since force between two mutually attracting objects is directly proportional to the concentration
of layer A and layer B molecules, since force between two mutually attracting objective is directly
proportion to the magnitude of the cause of force, the pressure P lost by real gases proportion to the
concentration of layer A and B molecules.

GASEOUS STATE
40 QUIZRR

n
p (layer A)
V

n
p (layer B)
V

n2
p
V2

an2
p
V2
where a is the proportionality constant and is a measure of the forces of attraction between the
an2
molecules. Thus, Pideal  Preal 
V2

an2
The unit of the term will be the same as that of the pressure. Thus, the SI units of a will
V2
be Pa m6 molă2. It may be conveniently expressed in k Pa dm6 molă2.
Now, the real gas behaves same as ideal gas on removing the force of attraction. The only
difference that now remains is the molecular size.

Co r re c t io n f o r m o l e c u l a r s i ze :

The container of volume V has n mole of gas molecules each of which possess a certain volume
of their own. If the total volume blocked by 1 mole of molecules due to their sheer presence is taken
as b, then the total volume blocked by them would be nb. We can now look at the container and see
that the effective volume for the gas molecules to move is V ă nb. This means that in the volume,
V ă nb the gas molecules have zero volume of their own.
Since, Pi Vi = nRT

 an2 
 P  2   V  nb   nRT Vreal = Videal ă nb
 V 

ÂbÊ is called the excluded volume or co-volume. The numerical value of b is four times the actual
volume occupied by 1 mole of gas molecules. This can be shown as follows.
We assure that molecules are in constant motion and collide with other molecules at every instant.
Moreover bi-molecular collisions are much more frequent than tri-molecular etc. Since all molecules are
alike we shall assume, for the sake of simplicity (without introducing error) that molecules have fixed
ÂpartnersÊ for collision. That is molecule 1 & 2, 3 & 4 etc. If the total molecules are odd we can remove
one molecule (removal of one molecule would hardly make a difference) to make it even. But since we

GASEOUS STATE
QUIZRR 41

have already considered that molecules collide every instant, fixing partners for collisions would introduce
an error. This is because after one collision they would need to separate in order to collide again and
since we have fixed partners they cannot collide with other molecules. This leads to a time interval,
which is collision free. To take care of this, we now assume that molecules are stuck to each other in
pairs as shown in Fig. This creates another problem that when the molecules collide they collide in all
possible directions.
To simulate this we assume that the molecules that are stuck in pairs are rotating by keeping the
point of contact fixed. This ensures that they occupy all possible orientations. For a third molecule the
volume unavailable is the volume of the large sphere with radius 2r where r is the radius of each

4 4
  2 r   8  r 3
3
molecule. Therefore the volume unavailable is
3 3

1 2

This volume is unavailable due to two molecules. Thus, due to one molecule the volume unavailable
is 4 times its volume.

4 3
 b 4 r  6.923  1023 Thus Van der Walls equation for 1 mole of real gas is given
3

 a 
as  P  2   V  b  RT
 V 

Sig n i f ic a n c e o f a a n d b
The constant ÂaÊ is a measure of Van der Walls forces of attraction existing between the molecules
of a given gas. The greater the value of a, the greater is the strength of the Van der Waals forces. The
greater the value of a, the greater is the ease with which a gas can be liquefied.
ÂbÊ is just the incompressible volume per mole of any gas.
If temperature is high and pressure is low, the Van der Waals constants a and b become
insignificant and the Vander Waals equation becomes ideal gas equation.

GASEOUS STATE
42 QUIZRR
Ex a m p l e 2 9
 B 
One way of writing the equation of state for real gases is PV  RT 1 +  ... 
 V 
where B is a constant. Derive an approximate expression for B in terms of the van der
Waals constants Âa Ê and ÂbÊ.
Solution :
The van der Waals equation for one mole of real gas is

 a 

 P  2  V  b  RT
 V 

 a  RT
or, P  2  
  Vb
 V 
RT a
P  2
Vb V

Multiplying both the sides with V

RTV a
PV  
Vb V

 V a 
PV  RT   
 V  b VRT 

 1
b a 
PV  RT  1    
 V VRT 

1
b  b
At low pressure, V is large and <<1. Thus, the term  1   can be expanded into a power
V  V
b
series in .
V
1 2
 b b b
1   = 1     ...
 V V V

 b b
2
a 
PV  RT 1      ...  
 V V VRT 

 1  a  
PV  RT 1  b    ...
 V RT  
Comparing Eq. with the expression given in the problem
a
B  b
RT

GASEOUS STATE
QUIZRR 43

7 .6 V A N DER WA A L S EQU A T I ON
In 1873, Van der Waals modified the ideal gas equation by suggesting that the gas molecules
were not point masses but behave like rigid spheres having a certain diameter and that there exist
intermolecular forces of attraction between them. The two correction terms introduced by Van der
Waals has been described before.
The Vander Waals equation for 1 mole of a real gas is

 a 
 P  2   Vm  b   RT
 Vm 

a ab
The equation can be rewritten as PVm  Pb  V  2  RT
m Vm

a ab
or PVm = RT + Pb ă V  2
m Vm

Dividing Eq. by RT

PVm RT Pb a ab
   
RT 2
RT RT Vm RT VmRT

Pb a ab
Z = 1 + RT  V RT  2
m Vm RT

For H2 gas, the volume of a is extremely small (as it is very difficult to liquefy) so, all terms
involving ÂaÊ in the numerator of equation can be reduced to

Pb
Z = 1+
RT

It clearly indicates that with the increase of pressure, Z increases and is always greater than
unity.
For methane, and CO2, in the low pressure region, Vm would be very large and in comparison
to Vm, ÂbÊ cannot be ignored. The equation can be rewritten as

Pb  a  b 
Z = 1+ +  V RT  V  1  
RT  m  m  

b  a  b 
Since b <<Vm, so would be extremely smaller than 1. Thus, the term  V RT  V  1  
Vm  m  m  

GASEOUS STATE
44 QUIZRR
Pb
would be negative and greater in magnitude than positive term . Consequently, the Z is less than
RT
unity.

Pb
With the increase of pressure in the low pressure region, both the terms, and
RT
 a  b   a  b 
   1   increases but the increase in negative term,    1   would be more
 Vm RT  Vm    Vm RT  Vm  
 Pb 
than the increase in positive term,   , thus Z decreases continuously. This means that the difference
 RT 
in magnitude of negative term and positive term continuously increases. When this difference becomes
maximum, the minima are obtained. Thereafter, the increase in negative term is lower than the
increase in positive term. Thus, Z starts increasing but is still less than unity. Then, a time comes when
the magnitude of negative terms and positive terms become equal. Under such conditions, Z becomes
equal to unity and when the magnitude of positive term becomes more than the negative term, the
value of Z becomes greater than unity.
The nature of the curve is like this because in the low pressure region the effect of attractive
forces (ÂaÊ factor) dominates while in the high pressure region, the size effect (ÂbÊ factor) will dominate
the behaviour of the gas.
Note :
(a) At very low pressure for 1 mole gas P´ and nb can be neglected.
PV = RT
(b) At moderate pressure, nb can be neglected

 a 
 P  2   V   RT
 V 

PV a
Z= so Z = 1 
RT RTV
Z < 1
(c) At high pressure
P´ can be ignored but b cannot be ignored.
So, P (V ă b) = RT

PV Pb
Z= 1
RT RT
Z > 1
 an2 
In case of H and He due to their small size, we always ignore the internal pressure  P´ or 2  .
 V 
So, they always show Z > 1 at moderate and high pressures.

GASEOUS STATE
QUIZRR 45

Ex a m p l e 3 0
The compression factor (compressibility factor) for 1 mole of a van der WaalsÊ gas at 0ĈC
and 100 atmosphere pressure is found to be 0.5. Assuming that the volume of gas molecule
is negligible, calculate the van der WaalÊs constant a .
Solution :

PV
Z =  0.5
RT

100  V
 = 0.5
0.082  273

 V = 0.112 litre
Now using van der WaalÊs equation,

 a 
P  2   V  = RT ( b is negligible)
 V 

 a  RT
or P  2  = V
 V 

 a 
100 +  0.082  273
 2 = = 199.88

 0.112  0.112

a
 = 99.88
(0.112)2

 a = 1.253 litre2 mol2 atm

Ex a m p l e 3 1
Calculate % of the Âfree volumeÊ available in 1 mol gaseous water at 1.00 atm and 100ĈC.
Density of liquid H2O at 100ĈC is 0.958 gm/mL.
Solution :
PV = nRT

nRT
 volume occupied by 1 mol gaseous water at 100ĈC =
P

1  0.0821  373.15
= = 30.64 L
1

GASEOUS STATE
46 QUIZRR

mass 18
volume of 1 mol liquid water = 
density 0.958

= 18.79 mL = 18.79 ï 10ă3 L

18.79  10 3
% of volume occupied by liquid water =  100  0.0613%
30.64

% of free volume = 100 ă 0.0613 = 99.9386%

9. CRI T ICA L CON ST A N T S


Cr i t i c a l t e m p e r a t u r e
The critical temperature, Tc is the temperature beyond which a gas cannÊt be liquefied no matter
how gerat the pressure may be. The reason for this is that the molecules have so much thermal energy
that the intermolecular forces are not powerful enough to make the molecules stick together. Each gas
has its own critical temperature because the strengths of intermolecular forces are different in each gas.

8a
Tc 
27Rb
Normal boiling point Tb of a liquid is approximately two-thirds of its critical temperature Tc when
both are expressed in Kelvin scale.

Tb 2

Tc 3

Cr i t i c a l p r e s s u r e
The critical pressure, Pc is the pressure beyond which a liquid cannÊt be vapourised, no matter
how high is the temperature. (or)
Pc is the minimum pressure required to cause liquefaction at the temperature Tc.
a
Pc 
27b2
Cr i t i c a l v o l u m e
The critical volume, Vc is the volume occupied by one mole of a gas at critical temperature and
critical pressure.
Vc = 3/b

Boyle Tem pe rat ure


It is a particular temperature for a real gas above which the gas behave as an ideal gas over a
long range of pressure.
a
Tb 
bR
where a & b are Van der WaalÊs constants.

GASEOUS STATE
QUIZRR 47

1 0 . GA S EU D I OM ET RY
Gas Eudiometry involes determining the composition of gaseous mixtures, molecular formula for
gases and gaseous hydrocarbons and establishing volume-volume relationship among gases.
The fundamental principle involved in gases eudiometry is the AvogadroÊs law. According to this
law, volume of the gas is proportional to the no. of moles of gas at constant temperature and pressure,
i.e., volume ratio among gases is same as mole ratio at constant temperature and pressure.
In eudiometry, gaseous reactions are studied in a closed graduated tube open at one end and
closed at other end. The tube is provided with platinum terminals required to produce electric spark
through the mixture of gases. Such graduated tube is known as Eudiometer tube and hence the term
gas eudiometry is also used for gas analysis. During eudiometry, the graduated tube is first filled with
mercury. Then a known volume of the gas or gaseous mixture (which is to be studied either for
molecular formula determination or composition determination) is inserted into the eudiometer tube,
which displaces equivalent amount of mercury. Now a certain volume (in excess) of O2 is introduced
and the electric spark is produced, when the combustible material gets oxidized. In general, all elements
on oxidation with O2 forms oxide with element in highest oxidation state unless & until mentioned in
the problem. The volume of gases before combustion and after combustion is noted. Soon after the
combustion, the reaction mixture is brought to room temperature when H2O(g) condenses to H2O(l).
This leads to contraction in volume. This contraction called first contraction in volume is noted. The
volumes of CO2, other gaseous products and remaining O2 are determined by absorbing them in
suitable reagents. For example, the volume of CO2 is determined by inserting KOH (or any other alkali)
in the eudiometer tube, which absorbs all the CO2 present. The volume of remaining O2 is determined
by introducing alkaline pyrogallol, which absorbs all the left over oxygen. There is contraction in
volume noted by introducing KOH, which is referred as second contraction in volume, and contraction
in volume by inserting alkaline pyrogallol is termed as thrid contraction in volume. From the various
contractions in volume obtained, the requisite information can be derived.
O2 : absorbed by alkaline pyrogallol
O3 : absorbed by turpentine oil
NO : absorbed by FeSO4 solution
CO : absorbed by ammoniacal Cu2Cl2 solution
H2O vapour : absorbed by conc. H2SO4, fused CaCl2 (required when reaction mixture is not
brought to room temperature
Cl2 : water or alkali solution
NH3 : acid solution or CuSO4 solution
CO2 & SO2 : alkali solution (NaOH, KOH, Ca(OH) 2 , Ba(OH) 2 or ethanolamine
(HOCH2CH2NH2) etc.
Eudiometry is mainly based on AvogadroÊs law which states that „equal volumes of all gases
under similar conditions of temperature contain equal number of molecules (or equal number of moles).‰

GASEOUS STATE
48 QUIZRR
Ru l e s f o r s o l v in g p r o b l e m s b a s e d u p o n Eu d i o m e t ry

(i) The volume of a solid or liquid in gaseous mixture is neglected in comparison to the volume
of gases.
(ii) The equation represents the condition of the reaction at S.T.P.
(iii) 1 mole of gas = 22.4 l of the gas at S.T.P. = 1 volume, according to the Gay ă Lussac law
of gaseous volume.
(iv) Air is considered as a mixture of N2 and O2 (having nearly 21% of O2).
(v) N2 is mostly considered as non-reactive.
(vi) The problems may be solved through the mole concept principle.
(vii) If required, the contraction in volume can be calculated as :
 V = (Total volume of reactants ă Total volume of products)
(viii) The contraction in volume is sometimes used for the identification of the gases. This type of
contraction is observed when suitable absorbants are used.
(ix) The general equation for the combustion of a hydrocarbon is represented as,

 y y
(a) C xH y   x   O2  xCO2  H2 O
 4 2

(when hydrocarbon is saturated or unsaturated)

 3n  1 
(b) CnH2n+2 +   O2  nCO2  (n  1)H2O
 2 

(for saturated hydrocarbons)

3n
(c) CnH2n + O2  nCO2  nH2 O
2

(for alkenes)

 3n  1 
(d) CnH2n+2 +   O 2  nCO2  (n  1)H2O
 2 

(for alkyne or dienes)


(e) For nitrogen containing organic compounds, we can write :

 4x  y  y Z
CxH yN z    O2  xCO2  2 H2 O + 2 N 2
 4 

GASEOUS STATE
QUIZRR 49

Ex a m p l e 3 2
15 ml of an oxide of nitrogen was taken in a eudiometer tube and mixed with hydrogen till
the volume was 42 ml. On sparking, the resulting mixture occupied 27 ml. To the mixture,
10 ml of oxygen was added and on explosion, again the volume fell to 19 ml. Find the
formula of the oxide of nitrogen that was originally admitted in the eudiometer tube. Both
the explosions led to the formation of water.
Solution :
There are 3 methods to solve this problem.
Method 1 :
This is a method that is most commonly thought of by a student.
Let the oxide of nitrogen be represented as NxOy. On mixing with H2, the volume was 42 ml.
 The volume of H2 added was 27 ml.
On sparking, the reaction would be

x
NxOy + yH2  yH2O + N
2 2
Since on adding 10 ml of O2, combustion still takes place, it implies that hydrogen was in excess
(since N2 will not react).

x
 NxOy(g) + yH2(g)  N (g) + yH2O(l)
2 2
Initially 15 ml 27 ml 0 0

15 x
After sparking 0 (27 ă 15y)  27 ă
2

15 x
 Total volume after the first explosion = (27 ă 25y) +  27
2

15 x
  15 y  0 ;  x = 2y ...(i)
2
Do not immediately write the formula as N2O, because this is a molecule and therefore N2O is its
empirical formula and not molecular formula.
In the next explosion, hydrogen reacts with 10 ml of O2 after which the volume fell to 19 ml. Since
we do not know whether O2 or H2 is in excess, we consider all the possibilities.
(i) Hydrogen is in excess
H2(g) + óO 2 ( g )  H2O(l)
Volume initially (27 ă 5y) 10 0
Volume finally (27 ă 15y ă 20) 0 ă

GASEOUS STATE
50 QUIZRR
 Final volume of gases = volume of remaining hydrogen + nitrogen

15 x
(27 ă 15y ă 20) +  19
2

15 x
 15 y  12 this equation violates equation (i)
2
 H2 cannot be in excess.
(ii) Hydrogen is exactly consumed by O2

17
 27 ă 15y = 10; y
15
We know that x and y have to be integers. Therefore, hydrogen can not be completely consumed
by oxygen.
(iii) Oxygem is in excess
H 2(g) + óO 2 ( g )  H2O(l)
Volume initially (27 ă 5y) 10 0
Volume finally 0 10 ă ó
(27 ă 15y) ă
 Final volume = volume of oxygen remaining + nitrogen formed

 1  15 x
10  2 (27  15 y)   2  19
 
On solving x = 2 and y = 1
 Formula is N2O
Method 2 :
Proceed till you reach equation (i)
In the second explosion, H2 reacts with O2 to produce H2O.
We do not know which of these two is completely consumed. Let us therefore assume that the
amount of O2 reacted is ÂaÊ. Then the amount of H2 reacted will be Â2aÊ.
2H 2(g) + O2(g)  H2O(l)
Volume initially 27 ă 15y 10 0
Volume finally 27 ă 15y – 2a 10 ă a ă
Final volume

15 x
27 ă 25y ă 2a + 10 ă a +  19
2

15 x
Since 15y ă  0 , we get
2
37 ă 3a = 19; a = 6
Therefore, oxygen is left behind and its volume would be 4 ml.

15 x
 4  10; x = 2 and y = 1
2

GASEOUS STATE
QUIZRR 51

Method 3 :
At the end of first explosion, the total volume is 27 ml. Now 10 ml of O2 is added. The total volume
becomes 37 ml. After second explosion the volume becomes 19 ml. The volume decreased is 18 ml. This
is due to the reaction between H2 and O2 only. If H2 consumed is b ml, then O2 consumed would
be b/2 ml.

b
 b  18; b = 12 ml. This means that O reacted is 6 ml and O left is 4 ml.
2 2 2

15 x
4  19; x = 2 and y = 1
2

Ex a m p l e 3 3
A mixture of 10 ml CH4, C2H4 and C2H2 has a vapour density of 11.3. When 30 ml of oxygen
are sparked together over aqueous KOH, the volume contracts to 5.5 ml and then disappears
when pyrogallol is introduced. If volumes are measured in the same condition of pressure,
temperature and humidity; calculate the composition of the original mixture.
Solution :
Let the volume of CH4 at N.T.P. = x ml
Let the volume of C2H4 at N.T.P. = y ml
Let the volume of C2H2 at N.T.P. = z ml
From question,
x + y + z = 10 ...(1)
As we know that
Weight of CH4 + Weight of C2H4 + Weight of C2H2 = Weight of mixture

16 x 28 y 26 z 11.30
    10 ...(2)
22400 22400 22400 11200

Now,

CH4(g) + 2O2(g) 



 CO2(g) + 2H2O(l)
x ml 2x ml

C2H4(g) + 3O2(g) 



 2CO2(g) + 2H2O(l)
y ml 3y ml

5
C2H2(g) + O (g) 

 2CO2(g) + H2O(l)
2 2
5
z ml z ml
2

GASEOUS STATE
52 QUIZRR

 5 
Total volume of oxygen used up in the reaction =  2 x  3 y  z  ml
 2 

But from question


Total volume of oxygen used up = 30 ă 5.5 = 24.5 ml

5
 2x  3 y  z  24.5 ...(3)
2

Solving equations (1), (2) and (3), we get


x = 4, y = 3, z = 3
 CH4 = 4 ml, C2H4 = 3 ml, C2H2 = 3 ml

GASEOUS STATE
CHEMICAL
BONDING
QUIZRR 3

1. INTRODUCTION
A chemical bond is the physical process responsible for the attractive interactions between atoms
and molecules, and that which confers stability to diatomic and polyatomic chemical compounds.
In general, strong chemical bonding is associated with the sharing or transfer of electrons between
the participating atoms. Molecules, crystals and diatomic gasesăindeed most of the physical
environment around usăare held together by chemical bonds, which dictate the structure of
matter.
Bonds vary widely in their strength which is associated both with the energy required to break
them, and the forces they exert on the atoms they hold together. Generally covalent and ionic
bonds are often described as „strong‰, whereas hydrogen bonds and van der Waal’s bonds are
generally considered to be „weak,‰ although there exist overlaps in strength within these bond
classes.
Since opposite charges attract via a basic electromagnetic force, the negatively-charged orbiting
the nucleus and the positively-charged protons in the nucleus attract each other. Also, an electron
positioned between two nuclei will be attracted to both of them. Thus, the most stable configuration
of nuclei and electrons is one in which the electrons spend more time between nuclei, than
anywhere else in space. These electrons cause the nuclei to be attracted to each other. However,
this assembly cannot collapse to a size dictated by the volumes of these individual particles. Due
to the matter wave nature of electrons and their relatively smaller mass, they occupy a very much
larger amount of volume compared with the nuclei, and this volume occupied by the electrons
keeps the atomic nuclei relatively far apart, as compared with the size of the nuclei themselves.

1 . 1 C ause of C hemical C omb inat ion

The atoms interact with each other on account of the following reasons :
(i) Decrease in energy : It is a fundamental truth that all natural systems tend to lose
potential energy and become more stable. Other things being equal, a system that has stored
potential energy is less stable than a system that has none. It is an observed fact that a
bonded state is more stable than unbonded stable. This is due to the fact that the bonded
state has lower potential energy than unbonded state. Hence, when two atoms approach
each other, they combine only under the condition that there is a decrease in potential
energy.
When two atoms approach each other, new forces of attraction and repulsion come into play.
The forces of attraction are between the nucleus of one atom and the electrons of the other.
The forces of repulsion are between two nuclei as well as between the electrons of the two
atoms. If the net result is attraction, the total potential energy of the system decreases and
a chemical bond results. No chemical bonding is possible if net result is repulsion.
(ii) Lewis octet rule : The noble gases are known for their lack of chemical activity. There are
no known compounds of helium, neon and argon. Why are these elements so unreactive

CHEMICAL BONDING
4 QUIZRR
towards other elements ? All these elements have electronic structures that consist of filled
outermost sheels. Except for helium, whose electronic configuration is 1s2, the s-and p-
subshells of the highest energy level contain a total of eight electrons. It is, therefore,
concluded that s2p6 configuration in the outer energy level constitutes a-structure of maximum
stability and therefore, of minimum energy.
The atoms of all elements when enter into chemical combination try to attain noble gas
configuration, i.e., they try to attain either 2 electrons (when only one energy shell) or 8
electrons in their outermost energy level which is of maximum stability and hence of minimum
energy. The tendency of atoms to achieve eight electrons in their outermost shell is known
as Lewis octet rule. Octet rule was the basis of electronic theory of valency.

1 . 2 L ewis Symb ols of E lement s

Chemical bonding mainly depends on the number of electrons present in the outermost energy
level. These electrons are termed as valency electrons. The electronic configuration of sodium (Na)
is 2, 8, 1 and that of sulphur has (S) 2, 8, 6. Thus, sodium has one valency electron while sulphur
has six valency electrons.
The valency electrons in atoms are shown in terms of Lewis symbols. To write Lewis symbol for
an element, we write down its symbol surrounded by a number of dots of crosses equal to the
number of valency electrons. Paired and unpaired valency electrons are also indicated. The Lewis
symbols for hydrogen, sodium, nitrogen, oxygen and chlorine may be written as :

H Na N O Cl

Generalised, Lewis symbols for the representative elements are given in the following table :

1 2 13 14 15 16 17
Group IA IIA IIIA IVA VA VIA VIIA
Lewis symbol X X X X X X X

1 . 3 E lect r onic t heor y of Valency

The theory of valency explains chemical combination in terms of electrons. The theory was
developed independently by W. Kossel and G.N. Lewis (1916) and extended by Irving Langmuir
(in 1919).
The chemical behaviour of an atom is determined to a large extent by the number and arrangement
of electrons in the outer orbitals of the atom. Only these electrons are involved in chemical
combination and so these are called the valence electrons.

CHEMICAL BONDING
QUIZRR 5

C omp let ed E lect r on O ct et or Dup let

Group 0 of the periodic table contains the noble gases. With the exception of helium which has
a 1s2 electron arrangement others have ns2 np6 configuration in the outer orbitals.

He 1s2
Ne 1s2 2s22p6
Ar 1s2 2s2 2p6 3s2 3p6
Kr 1s2 2s2 2p6 3s2 3p6 3d 10 4s2 4p6
Xe 1s2 2s2 2p6 3s2 sp6 3d10 4s2 4p6 4d 10 5s2 5p6

Since the atoms of the noble gases were not known to form chemical bonds, it was argued that
the presence of 8 electrons (an electron octet) in the valence shell makes the atom stable. Therefore
all other atoms must undergo bonding by gaining or losing or sharing electrons so as to acquire
the electronic configuration of the nearest inert gas. The presence of 8 electrons gives the name
octet rule to this concept. In the case of the first few elements such as hydrogen, lithium and
beryllium the atoms combine in such a way as to attain the stable structure of helium with 2
electrons (duplet) in its only one valence shell. There are, however, many exceptions to the octet
rule. Also compounds of noble gases, especially xenon, have been synthesized.
Hence, bond formation can take place in 3 ways.
(1) By losing or accepting electrons as known as elec trovalent bond.
(2) By equal contribution of electrons from two atoms and these electrons are then shared
equally to form covalent bond.
(3) Contribution is made by one atom and both the electrons are shared equally by 2 atoms to
form co-ordinate bond.
Thus it can be summarised as ÂÂThe union of two or more atoms involving redistribution of
electrons in their outer shells (either by transference or sharing) in such a way so that all the
atoms acquire the stable noble gas configuration of the minimum energy is known as electronic
theory of valencyÊÊ.

2. I O NI C BO ND

An Ionic bond is a type of chemical bond that involves a metal and a non-metal ion through
electrostatic attraction. In short, it is a bond formed by the attraction between two oppositely
charged ions. The metal donates one or more electrons, forming a positively charged ion or
cation with a stable electron configuration. These electrons then enter the non metal, causing
it to form a negatively charged ion or anion which also has a stable electron configuration. The
electrostatic attraction between the oppositely charged ions causes them to come together and
form a bond.

CHEMICAL BONDING
6 QUIZRR
For example :
Sodium Chloride : The free sodium atom has one valence electron (electronic configugation
2, 8, 1) i.e. 3s1. Whereas the chlorine atom has seven valence electrons (electronic configuration
2, 8, 7) i.e. 3s2 3p5. In forming an ionic bond, the sodium atom loses its valence electron which
is accepted by chlorine atom. As a result sodium achieves noble gas configuration of Neon (2, 8)
and becomes positive ion (Na+) chlorine achieves moble gas configuration of argon (2, 8, 8) and
acquires a negative charge (Clă). The attraction between sodium ion and chloride ion is an ionic
bond.
A short hand way of showing the formation of sodium chloride using dot symbols.

ă
ï
Na Cl =
+
Na ï
Cl or Na+ Clă

(2, 8, 1) (2, 5, 7) (2, 8) (2, 8, 8)

C ond it ions for For ming E lect r ovalent or I onic Bond


The following conditions favour the formation of an electrovalent bond.
(i) Number of valency electrons : One atom should possess 1, 2 or 3 valency electrons while
the other atom should have 5, 6 or 7 valency electrons. Thus, they can also form electrovalent
bonds but do not acquire inert gas configuration always.
(ii) Difference in electronegativity : The formation of an electrovalent bond will be easier
if the difference in the electronegativities of the two atoms is high. A difference of about 2
is necessary for the formation of an electrovalent bond. The electronegativity of sodium is
0.9 and that of fluorine is 4.0. Since the difference is 3.1 both will readily form an electrovalent
bond.
(iii) Overall decrease in energy : In the formation of an electrovalent bond, there must be
overall decrease in energy, i.e., energy must be released. Energy changes are involved in
the following steps :
This is called as Born-Haber cycle.

Na(s) + 1 + ă
2 Cl2 (g)
Na Cl (s)

Sub. BE

Na(g) F(g) Lattice Energy

IE EA

Na+ + Cl ă(g)

CHEMICAL BONDING
QUIZRR 7

Where sub is the sublimation energy, BE is the bond energy, IE is ionization energy and EA is
electron affinity.
(1) Sublimation Energy : The energy required to convert one mole of a solid into gas at
constant pressure and temperature.
(2) Ionization energy : The energy required by the metal atom to release its valence electron.
For example, sodium requires small energy to give up its loosely field electron and form Na+
ion.
Ionization energy of an element with a single electron in its valence still is less than that
with two electrons. In going across a period of the periodic table from left to right, I.E.
increases and the formation of the cation is less likely. On going down a group, the outermost
electron gets further away from the nucleous, and hence is more easily removed i.e. I.E.
decreases, the formation of the cation becomes more likely lower the value of ionisation
energy of an atom, greater will be the ease of formation of the cation from it.
(3) Bond energy : The energy required required to break the molecule into atoms is called
bond energy or dissociation energy.
(4) Electron affinity : the amount of energy released when an atom takes up an electron and
forms an anion. For example, chlorine takes up an electron from the Na atom and forms Clă
ion. Non-metals of groups VIA and VIIA have high electron affinity and can form ionic
bonds.
In going across a period from left to right, electron affinity (energy released) increases and
so the formation of negative ion becomes more likely. On going down a group, electron
affinity decreases and so the formation of anion becomes less likely.
Thus Higher the value of electron affinity of the atom, greater the ease of formation of the
anion form it, i.e., other atom should have high value of electron affinity.
(5) Lattice energy : Cation and anion attract each other by electrostatic force of attraction to
give a molecule A+Bă. Since the electrostatic field of a charged particle extends in all

ă
+ ă + ă
+
ă
ă + +
ă
+
+ ă ă
+

Crystals of ionic compounds

CHEMICAL BONDING
8 QUIZRR
directions, a positive ion is surrounded by a number of negatively charged ions while each
negative ion similarly surrounded by a number of positive ions. These cations and anions
arrange systematically in an alternating cation-anion pattern as shown in Fig. This is called
a crystal lattice. This process of clustering ions increases the force of attraction and thus
potential energy decreases. The energy released when the requisite number of positive and
negative ions are condensed into crystal to form one mole of the compound is called lattice
energy.
The value of lattice energy depends on the charges present on the two ions and the distance
between them. According to CoulombÊs law, the force of attraction (F) between two oppositely
charged ions in air with charges equal to q1 and q2 and separated by a distance d is given
by,

1 q1 q2
F
4 0 K d 2

where d is equal to sum of ionic radii of the two ions and K is dielectric constant of medium.

1 q1 q2
F
4 0 K (r +  r ă )2
A B

The value of F increases if (i) q1 and q2 are high and (ii) (rA ă  rBă ) is small.

The stability of the ionic compound and the strength of the ionic bond depends on the value
of F. Higher than value of F, greater shall be the stability of the ionic compound and hence
greater shall be the strength of the ionic bond. For example, NaCl is more stable than CsCl

as (rCs ă  rCl ă ) is less than (rA ă  rBă ) . MgO is more stable than NaCl as the product q1q2

is four times more in MgO than NaCl.


Conclusions :
(i) An ionic bond is purely electrostatic in nature.
(ii) Its formation is favoured by :
(a) Low ionisation potential (I.P.) of the element that forms a cation on losing electron(s).
The element should be metal, i.e., electropositive in nature.
(b) High electron affinity (E.A.) of the element that forms an anion on gaining electron(s).
The element should be non-metal, i.e., electronegative in nature.
(c) High lattice energy (L.E.) : The energy release when isolated ions form a crystal.
The value of lattice energy depends on the charges present on the two ions and
distance between them. It shall be high if charges are high and ionic radii are small.
(d) The summation of three energies should be negative, i.e., energy is released.
I.P. + E.A. + L.E. = ăive

CHEMICAL BONDING
QUIZRR 9

(iii) Greater the difference of electronegativity between two atoms, higher will be the possibility
of ionic bond formation.
(iv) Electrovalency
The capacity of an element to form electrovalent or ionic bond is termed as electrovalency. The
capacity is measured in terms of the electrons lost or accepted. Thus, electrovalency of an element
is equal to the number of electrons lost by an atom of the element or gained by the atom of the
element as to acquire inert gas configuration. The elements which lose electron or electrons show
positive electrovalency and the elements which gain electron or electrons who negative
electrovalency. Generally positive and negative signs are not used in practice and only the
number is taken to represent electrovalency.

No. of electrons Change in electronic


Element lost or gained Electrovalency configuration
by an atom

Na 1 (lost ) 1 (Monovalent) 2, 8, 1 to 2, 8 (Na+)


K 1 (lost) 1 (Monovalent) 2, 8, 8, 1 to 2, 8, 8 (K+)
Mg 2 (lost) 2 (Divalent) 2, 8, 2 to 2, 8 (Mg++)
Ca 2 (lost) 2 (Divalent) 2, 8, 8, 2 to 2, 8, 8, (Ca++)
Al 3 (lost) 3 (Trivalent) 2, 8, 3 to 2, 8 (AI+++)
F 1 (gained) 1 (Monovalent) 2, 7, to 2, 8 (Fă)
Cl 1 (gained) 1 (Monovalent) 2, 8, 7 to 2, 8, 8 (Clă)
O 2 (gained) 2 (Divalent) 2, 6 to 2, 8 (Oă ă)
S 2 (gained) 2 (Divalent) 2, 8, 6 to 2, 8, 8 (S ă ă)
N 3 (gained) 3 (Trivalent) 2, 5 to 2, 8 (N ă ă ă)

3. Det er minat ion of lat t ice ener gy

The lattice energy of an ionic solid is determine experimentally by a process known as Born-Haber
cycle. According to this theory following steps are involved :
(i) The reactant are converted into gaseous state.
(ii) The gaseous atoms are converted into ions.
(iii) The gaseous ions are combined to form ionic compound.
For example the formation of KF can occur either directly or in steps.

Direct : 1
K( s)  F2 ( g)  KF ( s)
2

H or heat of formation = ă 562.6 kJ/mol

CHEMICAL BONDING
10 QUIZRR
Steps :
H = ă562.6kJ/mol
K(s) + 1F (g) KF(s)
2 2
Heat of Sublimation = +89.6 kJ/mol 1/2(Heat of dissocation) = 79.1 kJ/mol
(HS) (HD)
ăLattice Energy (U)
K(g) F(g)

ăeă +eă
I.P. = +419.0 kJ/mol E.A. = ă332.6 kJ/mol

K+(g) + F ă(g)

On the basis of above cycles (or constant heat summation rule)


Heat of formation = Heat of atomization + Heat of dissociation + I.P. + E.A. + Lattice Energy

HD
H = HS   I.P.  (E.A.)  ( U)
2

ă 562.6 = 89.6 + 79.1 + 419.0 ă 332.6 ă U


U = 817.7 kJ/mol
Lattice energy of KF = ă U = ă 817.7 kJ/mol

M et hod of W r it ing For mula of an I onic C omp ound

In order to write the formula of an ionic compound which is made up of two ions (simple or
polyatomic) having electrovalencies x and y respectively, the following points are followed:
(i) Write the symbol of the ions side by side in such a way that positive ion is at the left and
negative ion at the right as AB.
(ii) Write their electrovalencies in figures on the top of each symbol as AxBy.
(iii) Divide their valencies by H.C.F.

x y
(iv) Now apply crisis cross rule as i.e., formula AyBx.
A B

Examples :
2 1

Calcium chloride Ca Cl = CaCl2;

3 2

Aluminium oxide O = Al2O3;

CHEMICAL BONDING
QUIZRR 11
1 3

Potassium phosphate PO4 = K3PO4;

2 3

Magnesium nitride N = Mg3N2;

1 1

Calcium oxide O = CaO;

1 2

Ammonium sulphate SO4 = (NH4)2SO 4

G ener al C har act er ist ics of I onic C omp ound s

(i) Generally ionic compounds are hard solids. As single ions of a metal are not associated in
the solid with single ions of a non-metal, separate units of ionic compounds do not exist.
It is, therefore, wrong to talk of a molecule of an ionic compound. The formula only
indicates the ratio of number of ions and the crystal consists of a very large number of
oppositely charged ions. Thus in NaCl crystal each Na+ ion is surrounded by 6Clă ions and
vice versa (in an octahedral arrangement). The attraction between Na+ and Clă ions is
quite large.
(ii) As a good deal of thermal energy is required to overcome the large electrostatic forces of
attraction in an ionic crystal, ionic compounds have high melting and boiling points.
(iii) Ionic compounds are commonly soluble in water and other polar solvents (which separate
the ions). They are practically insoluble in organic solvents such as benzene, carbon
tetrachloride, etc., as there is no attraction between ions and the molecules of the non-
polar liquids.
(iv) Ionic compounds are electrolytes. In the presence of a ionizing solvent such as water, the
electrostatic forces between the ions are so greatly reduced that the ions got separated. (This
is due to the electrostatic attraction between the ions and the polar molecules of the solvent.)
The free ions in solution conduct electricity and on passing a current, the ionic compound
undergoes chemical decomposition (called electrolysis). When an ionic compound is melted,
the crystal lattice structure is broken and free ions are produced. It is the free movement
of ions which makes an ionic compound a conductor and to undergo electrolysis in the
molten condition.

CHEMICAL BONDING
12 QUIZRR
(v) When an ionic compound dissolves in water, the ions get solvated (in this case hydrated).
The energy released is called solvation energy. Insoluble ionic compounds (eg., sulphates,
phosphates and fluorides of Ca, Sr and Ba) have very high lattice energies and the solvation
energy of the constituent ions is insufficient to counteract the high lattice energies and make
them soluble.
(vi) The chemical properties of an ionic compound are the properties of its constituent ions. Thus
all chlorides give the characteristic reactions of the chloride ion (reactions with conc. H2SO4,
AgNO3 solution, etc.). All acids which contain H+ ions give the same reactions (change blue
litmus to red, effervesce with a carbonate, etc.).
(vii) Reactions between solutions of ionic compounds are almost instantaneous, because they are
reactions between ions (and do not involve the breaking up of bonds as in covalent compounds,
q.v.). For example, when silver nitrate solution is added to sodium chloride solution, silver
chloride is immediately precipitated. The reactions may be represented thus :

Na+ + Clă + Ag+NOă3  AgCl + Na+ + NO3ă

Var iab le E lect r ovalency

Certain elements (metals) show more than one electrovalency in their electrovalent compounds.
The atoms of these elements lose different number of electrons under different conditions, thereby
showing variable electrovalency. The following are two reasons for variable electrovalency :
(i) Instability of the core : The residue configuration left after the loss of valency electrons
(electrons present in the outermost energy level) is called a core or kernel. In the case of the
atoms of transition elements, ions formed after the loss of valency electrons do not possess
a stable core as the configuration of outermost shell is not ns2np6 but ns2 np6 d 1 to 10. The
outershell generally loses one or more electrons giving rise to metal ions of higher valencies.
The variable valencies of iron can be explained on this basis.

Fe 26 2, 8, 8 + 6, 2 3s2 3p6 3d 6, 4s2


Fe2+ 24 2, 8, 8 + 6 3s2 3p6 (Not stable)
Fe3+ 23 2, 8, 8 + 5 3s2 3p6 3d 5 (Stable)

Thus, iron shows +2 and +3 valencies. The compounds, in which iron is in trivalent state,
i.e., ferric compounds are more stable than the compounds in which iron is in divalent state
(ferrous compounds).
(ii) Inert pair effect : some of heavier representative elements of third, fourth and fifth groups
having configurations of the outermost shell ns2 np1, ns2np2 and ns2np3 show valencies with
a difference of 2, i.e., (1; 3), (2; 4), (3; 5), respectively. In the case of lower valencies, only
the electrons present on p-subshell are lost and ns2 electrons remain intact. The reluctance

CHEMICAL BONDING
QUIZRR 13

of s-electron pair to take part in bond formation is known as the inert pair effect. A partial
explanation of the inert pair effect is due to the fact that outer ns2 electrons penetrate to
some extent to penultimate orit consisting 18 electrons thereby attracted strongly towards
nucleus. Inert pair effect increases in a group as the atomic number increases.

Tl (III group) 2, 8, 18, 32, 18, 6s2 6p1


Tl + 2, 8, 18, 32, 18, 6s2 Only 6p electron is lost
Sn (IV group) 2, 8, 18, 5s2 5p2
Sn2+ 2, 8, 18, 18, 5s2 Only 5p electrons are lost
Pb (IV group) 2, 8, 18, 32, 18, 6s2 6p2
Pb+ 2, 8, 18, 32, 18, 6s2 Only 6p electrons are lost
2 3
Bi (V group) 2, 8, 18, 32, 18, 6s 6p
3+
Bi 2, 8, 18, 32, 18, 6s2 Only 6p electrons are lost

When sufficient energy is available, the s-electrons also enter into bond formation and
higher valencies are observed. This tendency to show higher valencies is less in the case of
T1, Pb and Bi but more in the case of comparatively lighter elements such as In, Sn, Sb,
etc.

4. CO VAL ENT BO NDI NG

Certain elements which have high ionisation energies are incapable of transferring electrons and
other having low electrons affinities, fail to take up electrons. The atoms of such elements share
their elections with the atoms of other elements (and sometimes among themselves) in such a
manner that both the atoms form complete outer shell. In this manner they achieve stability.
Such an association through sharing of electron pairs among atoms of different or of same kinds
is known as Covalent Bond. This was proposed by G.N. Lewis.
The covalent bonding can be achieved in two ways :

Ć Sharing electrons between atoms of same kinds, formation of H2, Cl2, O2 etc.
Ć sharing of electrons between atoms of different kinds, formation of HCl, CO2, H2O, CH4 etc.

The bond is, non-polar bond.


Covalent bond may be single, double or a triple bond. Double and triple covalent bonds are called
multiple covalent bonds. Single covalent bond is formed by sharing of only one electron pair. This
bond is represented by single dash (·). Double and triple covalent bonds are formed when atoms
bonded together shared two or three electron pair, respectively. These bonds are represented by
double dash (==) and triple dash ( ) respectively. Some examples of covalent bonding are given
below :

CHEMICAL BONDING
14 QUIZRR
1. Formation of hydrogen molecule :
In the formation of hydrogen molecule, each hydrogen atom contributes one electron and
then the pair is shared between two atoms. Both the atoms acquire stable configuration of
helium. Thus, the molecule consists of one single covalent bond.

ï
ï
H + H H H or H·H

2. For mat ion of F 2 and ot her like molecules

It is formed by combination of two F atoms by sharing one electron each. It is represented


by Lewis dot structure as follows :

ï ï ï ï
F + F F ï ïFï or FăF
ï ï

ï ï
ï
ï ï

Lewis dot structure shows only the electrons of outer most shell. Note that both of F atoms
now have 8 electrons each in their outer shell.
Note : Pair of electrons depicted as Ć Ć or xx is called as lone pair (lp) and that by x Ć is
called as shared pair or bonded pair (bp). Lone pair is the pair of electrons belonging to one
atom only.

3. For mat ion of H C l molecules

Both hydrogen and chlorine contribute one electron each and then the pair of electrons is
equally shared. Hydrogen acquires the configuration of helium and chlorine acquires the
configuration of argon.

H ï Cl H ï Cl or H · Cl
(1) (2, 8, 7)

(2) (2, 8, 8)

4. For mat ion of wat er molecule

Oxygen atom has 6 valency electrons. It can achieve configuration of neon by sharing two
electrons, one with each hydrogen atom.

CHEMICAL BONDING
QUIZRR 15

ï ï ï ï ï
H + ï Oï + H H O H or H·O·H
ï ï ï ï
(1) (2, 6) (1)

(2) (2, 8) (2)

5. Fo r m a t io n o f m e t h a n e
Carbon has four electrons in the valency shell. It can achieve stable configuration of neon
by shring four electrons with four hydrogen atoms, one with each hydrogen atom. Each
hydrogen atom acquires helium configuration.

H
H

·
ï
H ï C ï H H C H H·C·H
ï ï

·
ï
H
ï
(2, 4) H
H
(1) (2, 8) (2)

6. Fo r m a t i o n o f O 2 m o l e c u l e s
Each oxygen atom contributes two electrons and two pairs of electrons are then shared
equally. Both the atoms acquire configuration of neon.

ï ï ï ï ï ï
O + O O O or O·O
ï ï

ï ï

ï ï ï ï ï ï

(2, 6) (2, 6)
(2, 8) (2, 8)

7. For mat ion of N 2 molecules


Nitrogen atom has five valency electrons. Both nitrogen atoms achieve configuration of neon
by sharing 3 pairs of electrons, i.e., each atom contributes 3 electrons.


ï ï ï

ï ï ï

N N N N or ·N
ï ï
ï ï

(2, 5) (2, 5)
(2, 8) (2, 8)

CHEMICAL BONDING
16 QUIZRR
C ov a len cy

It is defined as the number of electrons contributed by an atom of the element for sharing with
other atoms as to achieve noble gas configuration. It can also be defined as the number of
covalent bonds formed by the atom of the element with other atoms. The usual covalency of an
element except hydrogen (which has covalency 1) is equal to (8-group number of MendeleeffÊs
table to which an element belongs). This is true for the elements belonging to IV, V, VI and VII
groups.

Element Group (8ăGroup number) Covalency


C IV 8 ă 4 = 4 4
Si IV* 8 ă 4 = 4 4
N V 8 ă 5 = 3 3
P V 8 ă 5 = 3 3
O VI 8 ă 6 = 2 2
S VI 8 ă 6 = 2 2
F VII 8 ă 7 = 1 1
Cl VII 8 ă 7 = 1 1

Generally, the covalency of an element is equal to the total number of unpaired electrons in s-
and p-orbitals of the valency shell.

1s Covalency

Hydrogen has one unpaired orbital One

Fluorine has only one unpaired orbital One

Oxygen has two unpaired p-orbitals Two

Nitrogen has three unpaired p-electrons Three

These four elements do not possess d-orbitals in their valency shell. However, the elements having
vacant d-orbitals in their valency shell like P, S, Cl, Br, I, show variable covalency by increasing
the number of unpaired electrons under excited conditions, i.e., unpairing the paired orbitals and

CHEMICAL BONDING
QUIZRR 17

shifting the electrons to vacant d-orbitals. [Such a shifting is not possible in the case of H, N, O
and F because d-orbitals are not present in their valency shell.]
Phosphorus shows 3 and 5 covalencies.
3s 3p
Phosphorus atom in
ground state

Three unpaired electrons


covalency = 3

Phosphorus atom in
excited state

Five unpaired electrons


covalency = 5

Sulphur atoms shows 2, 4 and 6 covalencies.


3s 3p 3d

Sulphur atom in
ground state

Two unpaired electrons


covalency = 2

3s 3p 3d
Sulphur atom in excited state,
(a) When p-orbital is
unpaired
(first excited state.) Four unpaired electrons
covalency = 4

(b) When s-and p-orbital


is unpaired
(first excited state).
Six unpaired electrons
covalency = 6

CHEMICAL BONDING
18 QUIZRR
Chlorine shows 1, 3, 5 and 7 covalencies.
3s 3p 3d

Chlorine atom in
ground state
One unpaired electrons
covalency = 1

Chlorine atom in excited


state,
(a) First excited state when
p-orbital is unpaired
Three unpaired electrons
covalency = 3

(b) Second excited state


when two orbitals
are unpaired
Five unpaired electrons
covalency = 5

(c) Third excited state


when one s-and two
p-orbitals are unpaired

Seven unpaired electrons


covalency = 7

Thus, variable covalency is shown by those elements whose atoms have vacant d-orbitals in their
valency shell.

G ener al C har act er ist ics of C ovalent C omp ound s

(i) In a purely covalent compound the electrons in the bond are shared equally between the
atoms linked by the bond; the resultant particles formed are not electrically charged. So,
separate molecules of the covalent compounds exist. Covalent compounds may therefore be
expected to be gases or low boiling liquids or soft, low melting solids at ordinary temperature.
In the solid state they may be amorphous or present as molecular crystals the molecules
being held together by what are called weak van der Waals forces of attraction.
(ii) Since the molecules are held together by weak van der Waals forces, covalent compounds
(except those consisting of giant molecules) have low melting and boiling points; very little
thermal energy is needed to overcome these weak intermolecular forces.

CHEMICAL BONDING
QUIZRR 19

(iii) They are non-electrolytes, i.e., they do not contain ions. Even in giant molecules such as
diamond there are no free electrons. So they are very poor conductors of electricity.

(iv) They are generally soluble in organic (non-polar) solvents such as benzene or carbon
tetrachloride but are insoluble in water or other ionizing solvents. (The solubility of covalent
compounds is very much dependent on the size of the molecules; giant molecules are
practically insoluble in nearly all solvents.)

(v) Reactions between covalent compounds are slow and often incomplete and reversible. This
is so because the reaction involves breaking and making of bonds i.e., energy considerations
are involved for reactants, activated complexes and products.

(vi) A covalent bond is a space-directed bond and it may exhibit isomerism.

Polar C ovalent Bond s – E lect r onegat ivit y

The shared pair of electrons may be shared equally between two atoms; then the covalent bond
is said to be non-polar. Equal sharing occurs between identical atoms, as in H ă H or Cl ă Cl (i.e.,
in homonuclear molecules) or between identical atoms with identical neighbours as in H3CăCH3.
When the two bonded atoms are dissimilar (i.e., in heteronuclear molecules) the sharing is unequal.
For example a chlorine atom has a greater electron attracting power than a hydrogen atom; so
in H ă Cl, the shared pair of electrons are drawn more towards chlorine and away from hydrogen.
The result is separation of charges within the molecule, the chlorine end acquiring a slight
+ 
negative charge and the hydrogen end a slight but equal positive charge H Cl . Such covalent
bonds are said to be polar (i.e. bonds formed by sharing a pair of electrons between two atoms
but displaced towards the nucleus of one of the bonded atoms).
The net tendency of a bonded atom in a covalent to attract the shared pair of electrons
towards itself is known as electronegativity. (This word does not mean the actual content
of the electric charge, but just the tendency to acquire it in a molecule). Thus F is highly
electronegative, but Fă which has already an extra electron is not.
To assess the tendency of an atom of a given element to attract electrons towards itself in a
covalent bond, relative electronegativity values are used.
(i) Electronegativity value increase across a period and decrease down a group.
(ii) Smaller atoms have greater electronegativity than larger ones and so they attract electrons
more towards them than larger ones. Alkali metals have low electronegativities and halogens
high electronegativities.
(iii) Atoms with nearly filled shells of electrons (e.g., halogens) have greater electronegativity
than those with sparsely occupied shells.

CHEMICAL BONDING
20 QUIZRR
(iv) Elements with low electronegativity values such as Cs (0.8) and Rb (0.8) tend to form
positive ions, i.e., these are metals. Elements with high electronegativity values such as
F(4.0) and O (3.5) tend to form negative ions, i.e., these are non-metals.
(v) Electronegativity value may be used to make rough predictions of the type of bonding to be
found in a compound. The larger the difference between electronegativity values of two
combining atoms, the more polar the covalent bond. If the difference is greater than 2, the
greater the chance for ionic bonding (i.e., the chance of covalent bond assuming 100% ionic
character). From this point of view ionic may be considered to be an extreme case of a polar
bond (with total separation of charges).
If the difference between the electronegativities of the combining atoms is zero or small, the
bond is essentially non-polar.
Let XA and XB represent the electronegativities of two atoms A and B. If XB ă XA = 1.7, the
covalent bond A ă B is said to have 50% ionic character. On the basis, the % ionic character
in some typical bonds are calculated (Table). These calculations are very qualitative.

% Ionic Character of Bonds

C ă H N ă H O ă H F ă H
4% 10% 39% 60%

C ă F C ă Cl C ă Br C ăl
43% 11% 3% 0%

5. THE LEWIS THEORY

The octet rule : The Lewis theory gave the first explanation of a covalent bond (in terms of
electrons) that was generally accepted. If two electrons are shared between two atoms, this
constitutes a bond and binds the atoms together. For many light atoms, a stable arrangement is
attained when the atom is surrounded by eight electrons.
This octet can be made up from some electrons which are totally owned and some electrons which
are ÂsharedÊ. Thus atoms continue to form bonds until they have made up an octet of electrons.
This is called the Âoctet ruleÊ. The octet rule explains the observed valencies in a large number
of cases. For example nitrogen atom has 5 outer electrons and in NH3 it shares three of these,
forming three bonds and thus attaining an octet, hydrogen has only one electron and by sharing
one electron of two electrons.

N + 3[H*] H N H
* *
*
H

CHEMICAL BONDING
QUIZRR 21

E xcep t ion t o oct et r ule

It is observed that atoms in some molecules could exist with some other number of electrons in
their valence shells, rather than 8 electrons without affecting the stability. We are discussing
some of them.

BF 3

Boron atom has only six electrons in its outershell even after making three single bonds with
three F atoms (i.e. it completes only sixet).

BeC l 2

Beryllium has only four electrons in its outershell even after making two single bonds with two
Cl atoms.
CI Be CI

PC l5

Phosphorus after making five single bonds with five Cl atoms has ten electrons in its outershell.

CI CI
P
CI CI
CI

SF 6

Sulphur makes six single bonds with six F atoms and thus has 12 electrons in its outershell.

L ewis St r uct ur es of M olecules

The formula of a molecule shows the number of atoms of each element but does not show the
bonding arrangement of the atoms. To represent the bonding pattern in a molecule, the electron

CHEMICAL BONDING
22 QUIZRR
dot symbols of the elements are arranged such that the shared pairs and unshared pairs (called
lone pairs) are shown and the octet rule (or duet for hydrogen) is satisfied. For example, a
molecule of fluorine is shown as

.. .. .. ..
:F : :F : and a molecule of hydrogen fluoride is shown as H :F: or H  F:
.. .. .. ..

Arrangement of dot symbols used to represent molecules are called Lewis Structures. Lewis
structures do not convey any information regarding the shape of the molecule. Usually, the
shared pairs of electrons are represented by lines between atoms and any unshared pairs are
shown as dot pairs.
Lewis structures are written by fitting the element dot symbols together to show shared electron
pairs and to satisfy the octet rule. For example,

.. ..
(i) In water (H2O), one H and two .O : complete their duet and octet respectively as : O  H
. |
H

 
(ii) In ammonia (NH3), three H and one N fit together and satisfy their duet and octet


respectively as H N H
|
H

 
(iii) In carbon tetrachloride (CCl4), four :Cl and  C  complete their octet as



: Cl:
|
Cl  C  Cl
|
: Cl:


For the given molecules, we have adopted hit & trial to fit the dot symbols together and satisfy
the octet rule. But remember that hydrogen form one bond, oxygen forms two bonds, nitrogen
three bonds and carbon forms four bonds. For simpler molecules, the hit & trial method works
perfectly but for slightly complicated polyatomic species, this may give us more than one possible
structure. Thus, a systematic approach is needed to design the Lewis structures of such polyatomic
species. But before proceeding further, let us understand the limitation of this approach.

CHEMICAL BONDING
QUIZRR 23

L imit at ions of L ewis T heor y of Dr awing St r uct ur e

(i) This method would be applicable to only those molecules/species, which follow octet rule
except hydrogen.
(ii) This method would not be applicable to species, which have more than one central atom (like
N2O4, N2H4 etc.).
(iii) This method is also not suitable for species, which contains transition metal atom as the
central atom.

There are three kinds of molecules/species, which do not follow octet rule.

(a) Molecules, which have contraction of octet. Such molecules are electron deficient. For example,
BH3, BF3, BCl3, AlCl3, GaCl3 etc.
(b) Molecules, which have expansion of octet. Such species have more than eight electrons in
their outermost shell. This is possible in those molecules, which have vacant d-orbitals, thus
they can expand their octet. For example, PCl5, SF6 etc.
(c) Molecules containing odd number of electrons (in total) cannot satisfy octet rule. Such
species are called odd electron species and are paramagnetic in nature due to presence of
unpaired electron. For example, NO, NO2 and ClO2.

M et hod of Dr awing L ewis St r uct ur es

To draw the Lewis structures of polyatomic species, follow the given sequence.
(i) First calculate n1.
n1 = Sum of valence electron of all the atoms of the species  net charge on the species.
For a negatively charged species, electrons are added while for positively charged species,
the electrons are subtracted. For an uninegatively charged species, add 1 to the sum of
valence electrons and for a dinegatively charged species, and 2 and so on.
(ii) Then calculate n2.
n2 = (8 ï number of atoms other than H) + (2 ï number of H atoms)
(iii) Subtract n1 from n2, which gives n3.
n3 = n2 ă n1 = number of electrons shared between atoms = number of bonding electrons.

n3 n2  n1
 = number of shared (bonding) electron pairs = number of bonds.
2 2

(iv) Subtracting n3 from n1 gives n4.


n4 = n1 ă n3 = number of unshared electrons or non-bonding electrons

n4 n1  n3
 = number of unshared electron pairs = number of lone pairs.
2 2

CHEMICAL BONDING
24 QUIZRR
(v) Identify the central atom. Generally, the central atom is the one, which is least electronegative
of all the atoms, when the other atoms do not contain hydrogen. When the other atoms are
hydrogen only, then the central atom would be the more electronegative atom.
(vi) Now around the central atom, place the other atoms and distribute the required number of
bonds (as calculated in step (iv), keeping in mind that every atom gets an octet of electrons
except hydrogen.
(vii) Then calculate the formal charge on each atom of the species.
Formal charge is the difference between the valence electrons in an isolated atom and the
number of electrons assigned to that atom in a Lewis Structure.
Formal charge on an atom = number of valence electrons of the atom ă (number of shared
electrons of that atom + number of unshared electrons of that atom).
Formal charge on an atom = number of valence electrons of the atom ă number of bonds
formed by that atom ă number of unshared electrons (2 ï lone pairs) of that atom.
For every electron of an atom that is shared in a bond, the „number of bonds formed by the
atom‰ is one. Therefore if an atom forms only one bond (A ă B), one electron of the bond
is that of A and other is that of B. So the „number of bonds‰ of A and B each is one. But
if the bond were a co-ordinate bond (A  B), then two electrons of A are involved in it. This
makes the number of bonds of A to be 2 and that of B to be zero.
(viii) When two adjacent atoms get opposite formal charges, then charges can be removed by
replacing the covalent bond between the atoms by a dative (co-ordinate) bond. This bond
will have the arrowhead pointing towards the atom with positive formal charge. It is not
mandatory to show the dative bonds unless required to do so.
(ix) The given Lewis structure should account for the factual aspects of the molecule like resonance
(delocalization), bond length, p-d back bonding etc.
Sometimes, there are more than one acceptable Lewis structure for a given species. In such
cases, we select the most plausible Lewis structure by using formal charges and the following
guidelines :
Ć For neutral molecules, a Lewis structure in which there are no formal charges is
preferable to one in which formal charges are present.
Ć Lewis structures with large formal charges (+ 2, + 3 and/or ă 2, ă 3 and so on) are less
plausible than those with small charges.
Ć Among Lewis structures having similar distributions of formal charges, the most plausible
structure is the one in which negative formal charges are placed on the more
electronegative atoms.

CHEMICAL BONDING
QUIZRR 25

E xamp le 1

Determine Lewis structure of NO2 .


Solution :
(i) n1 = 5 + (6 ï 2) ă 1 = 16
(ii) n2 = (3 ï 8) = 24
(iii) n3 = n2 ă n1 = 24 ă 16 = 8

8
 Number of bonds = 4
2

(iv) n4 = n1 ă n3 = 16 ă 8 = 8

8
 Number of lone pairs =  4
2

(v) nitrogen is the central atom (as it is less electronegative than O). Arranging two O atoms
around it and distributing 4 bonds and 4 lone pairs as

O N O
(a) (b)

(vi) Calculating formal change on each atom.


Formal charge on N = 5ă 4 ă 0= + 1
Formal charge on O (a) = 6 ă 2 ă 4 = 0
Formal charge on O (b) = 6 ă 2 ă 4 = 0
Thus, the structures can now be shown as

+1
O N O
(a) (b)

E xamp le 2

Determine Lewis structure of CNă ion.


Solution :
(i) n1 = 4 + 5 + 1 = 10
(ii) n2 = (2 ï 8) = 16
(iii) n3 = n2 ă n1 = 16 ă 10 = 6

6
 Number of bonds = 3
2

CHEMICAL BONDING
26 QUIZRR
(iv) n4 = n1 ă n3 = 10 ă 6 = 4

4
 Number of lone pairs = 2
2

(v) Carbon is the central atom (C is less electronegative than N) and arrange N, number of
bonds and number of lone pairs around it as
(vi) Formal charge on C = 4 ă 3 ă 2 = ă 1
Formal charge on N = 5 ă 3 ă 2 = 0
Thus, final Lewis structure of CNă would be
ă
C N

E xamp le 3

Draw Lewis structure for NH4+ ion.


Solution :
(i) n1 = 5 + (4 ï 1) ă 1 = 8
(ii) n2 = (8 ï 1) + (2 ï 4) = 16
(iii) n3 = n2 ă n1 = 16 ă 8 = 8

8
 Number of bonds = 4
2

(iv) n4 = n1 ă n3 = 8 ă 8 = 0
 Number of lone pairs = 0
(v) Nitrogen being the central atom, distributing other atoms (H) around it, and 4 bonds with
the 4 H atoms, the structure looks like

H (b)

H N H(c)
(a)

H (d)

(vi) Formal charge on N = 5 ă 4 ă 0 = + 1


Formal charge on H(a)/H(b)/H(c)/H(d) = 1 ă 1 ă 0 = 0
Thus, final Lewis structure of NH4+ would be

H
+
H N H

CHEMICAL BONDING
QUIZRR 27

6. COO RDINATE BOND

It is a special type of covalent bond in which both the shared electrons are contributed by one
atom only. It may be defined as ÂÂa covalent bond in which both electrons of the shared pair are
contributed by one of the two atoms’’. Such a bond is also called as dative bond. A coordinate or
a dative bond is established between two such atoms, one of which has a complete octet and
possesses a pair of valency electrons while the other is short of a pair of electrons.

ïï ïï
A + B ïï A B or A B

ïï
ïï ïï

This bond is represented by an arrow ().


The atom which contributes electron pair is called the donar while the atom which accepts it is
called acceptor.
Note : Coordinate bond after formation is indistinguishable from a covalent bond.
The formation of a coordinate bond can be looked upon as a combination of electrovalent and
covalent bonds. The formation may be assumed to have taken place in two steps:
(i) The donor atom loses one electron and transferred to acceptor atom. As a result donor atom
acquires a positive charge and the acceptor atom acquires a negative charge.

+ ă
A + B A B
ï ï

ï ï

ï ï

(ii) These two charged particles now contribute one electron each and this pair is shared by both
the atoms.
+ ï ï ă ï ï
A + B A + B
ï ï

ï ï

ï ï ï ï

As the coordinate bond is a combination of one electrovalent bond and one covalent bond, it is also
termed as semi pola r bond.
The compound consisting of the coordinate bond is termed coordinate compound. Some examples
of coordinate bond formation are given below :
(1) Formation of ammonium ion : The ammonia molecule has a lone pair of electrons i.e. an
unshared pair. The hydrogen ion H+, has empty s orbital. The lone pair comes to be shared
between the nitrogen and hydrogen atoms.
+
H
H H
+
H N + H H N H or H N H
H H
H

CHEMICAL BONDING
28 QUIZRR
Nitrogen atom is called the donor and H+, the acceptor. NH3 is a neutral molecules. H+ carries
a unit positive charge, so NH+4 ion carries positive charge. Here, all the NăH bond become
identical.
(2) Formation of Aluminium Chloride, Al2 Cl6

Cl Cl Cl

Al Al

Cl Cl Cl

(3) Formation of ozone : Oxygen molecule consists of two oxygen atoms linked by a double
covalent bond. Each oxygen atom has two lone pairs of electrons. When one lone pair of
electrons is donated to a third oxygen atom which has six electrons, a coordinate bond is
formed.

ï
O O + O O O O
ïï

C har act er ist ics of C oor d inat e C omp ound s

The properties of coordinate compounds are intermediate between the properties of electrovalent
compounds and covalent compounds. The main properties are described below :
(i) Physical state : These exist as gases, liquids and solids under ordinary conditions.
(ii) Melting and boiling points : Their melting and boiling points are higher than purely
covalent compounds and lower than purely ionic compounds.
(iii) Solubility : these are sparingly soluble in polar solvents like water but readily soluble in
non-polar (organic) solvents.
(iv) Stability : These are a stable as the covalent compounds. The addition compounds are,
however, not every stable. It is also a strong bond because the paired electrons cannot be
separated easily.
(v) Conductivity : Like covalent compounds, these are also bad conductors of electricity. The
solutions or fused mass do not allow the passage of electricity.
(vi) Molecular reactions : These undergo molecular reactions. The reactions are slow.
(vii) Isomerism : The bond is rigid and directional. Thus, coordinate compounds show isomerism.
(viii) Dielectric constant : The compounds containing coordinate bond possess high values of
dielectric constants.

CHEMICAL BONDING
QUIZRR 29

7. DIPOLE MOMENTS

A dipole consists of a positive and an equal negative charge separated by a distance within a
molecule. The degree of polarity of a bond is given by the dipole moment (ø), which is the product
of either charge (e) and the distance (d) between them. ø = de. ÂeÊ is of the order of magnitude
of the electronic charge, i.e. about 10ă10 esu and d is the distance between the atomic centres,
i.e., about 10ă8 cm. Hence dipole moment may be expected to have values around 10ă10 ï 10ă8
= 10ă18 esu-cm. It is however, general practice to express dipole moments in Debye units (D),
1 D = 10ă18 esu-cm.

Electric pole Electric pole

+ ă

If the charge is in SI units (Coulombs) and d in metre ø will coulomb-metre (C.m) units.
1D = 3.336 ï 10ă30 C. m.
Dipole moment is a vector quantity, i.e., it has both magnitude as well as direction. Thus the
overall value of the dipole moment of a polar molecule depends on its geometry and shape i.e.,
vectorial addition of dipole moment of constituent bonds.
Any covalent bond which has a certain degree of polarity will have a corresponding dipole
moment, though it does not follow that compounds containing such bonds will have dipole moments,
for the polarity of the molecule as a whole is the vector sum of the individual bond moments. For
example, CO2 has zero dipole moment, although the C = O bond is a polar bond. This shows that
CO2 is a linear molecule, O = C = O, so that the dipole moments of the two C = O bonds cancel
out. The C  Cl bond has a definite polarity and a definite dipole moment but carbon tetrachloride
has zero dipole moment because it is a tetrahedral molecule, and the resultant of the 4C ă Cl bond
moments is zero. On the contrary CH3Cl, CH2Cl2 and CHCl3 have definite dipole moments.

Ap p licat ions of Dip ole M oment

(i) To decide polarity of the molecule : Molecules having zero dipole moment are said to
be non-polar molecules and those having R  0 are polar in nature.
(ii) To determine geometry of molecules : The values of dipole moments provide valuable
information about the structure of molecules.
(a) CO2, CS2 molecules are linear as values of their dipole moments are zero.
(b) H2O is not a linear molecule as it has dipole moment. Actually, it has V-shaped
structure and the bond angle is 105Ĉ. Similarly, SO2 has a bent structure.
(c) In ammonia, three hydrogen atoms do not lie symmetrically with respect to nitrogen
as it has dipole moment. It has pyramidal structure.

CHEMICAL BONDING
30 QUIZRR
(iii) To distinguish cis and tra ns forms of geometrical isomers : Experimental values of
dipole moment of the isomers (cis and trans) are determined. The trans isomer usually
possesses either zero dipole moment or very low value in comparison to cis form.

H C Cl H C Cl

H C Cl Cl C H

Cis-1, 2, dichloroethene Trans-1, 2 dicholroethene


ø = 1.9 D ø 0

It should be noted that if two groups have opposite inductive character then trans isomer
will have greater dipole moment of,

H Cl Cl H

C C

C C

CH3 H CH3 H

Trans Cis

Trans > cis

(iv) To determine orientation in benzene ring : Dipole moment is useful to ascertain the
orientation of substituents. The greater the dipole moment, the greater is the asymmetry. In
general, dipole moment follows the order :

Ortho > meta > para

Cl Cl Cl
Cl

Cl
Cl
Ortho Meta Para
ø = 2.54 D ø = 1.48 D ø=0

CHEMICAL BONDING
QUIZRR 31

E xamp le 4

Both CO2 and N2O are linear but dipole moment of CO2 in zero but for N2O it is non-zero,
why ?
Solution :
The answer lies in the structure of these molecules. CO2 is a symmetrical molecule while N2O is
unsymmetrical. Thus for N2O, dipoles do not cancel each other, leaving the molecule with a
resultant dipole moment, while the bond moment of CO2 cancel each other, so CO2 has no net
dipole moment.

N N O ; O C O

E xamp le 5

Compare the dipole moment of NH3 and NF3.


Solution :
LetÊs draw the structure of both the compounds and then analyse their dipole directions.

N N
H H H F F F

The structure of both NH3 and NF3 are pyramidal with three bond pairs and one lone pair. In
NH3, as N is more electronegative than hydrogen, so the resultant bond dipole is towards N,
which means that both the lone pair and bond pair dipoles are acting in the same direction and
are summed up. In case of NF3, the bond dipole (of NăF bonds) is acting towards fluorine, (as
fluorine is more electronegative than N) so in NF3 the lone pair and bond pair dipoles are acting
in opposition, resulting in a decreased dipole moment. Thus, NH3 has higher dipole moment than
NF3.

Dip ole M oment and Per cent age I onic C har act er

The measured dipole moment of a substance may be used to calculate the percentage ionic
character of a covalent bond in simple molecules.
1 unit charge = Magnitude of electronic charge = 4.8 ï 10ă10 e.s.u.
1 D = 1 ï 10ă18 e.s.u.ăcm

Observed dipole moment


 % ionic character = Theoretical dipole moment

CHEMICAL BONDING
32 QUIZRR
Theoretical dipole moment is confined to when we assume that the bond is 100% ionic and it is
broken into ions while observed dipole moment is with respect to fractional charges on the atoms
of the bond.

E xamp le 6
The dipole moment of LiH is 1.964 ï 10ă29 coulomb meter, distance between Li and H is 1.596
Å. Find the % ionic character in the molecule.
Solution :
ø of 100% ionic molecule (Li+Hă) = 1.6 ï 10ă19 C ï 1.596 ï 10ă10 m = 2.554 ï 10ă29 Cm

1.9  10 29
% ionic character =  100
2.554  1029

= 74.4%

T r ansition Fr om I onic to C ovalent Bond–FAJ ANS’ R UL E

Just as a covalent bond may have partial ionic character an ionic bond may also show a certain
degree of covalent character. When two oppositely charged ions approach each other closely, the
cation would attract the electrons in the outer shell of the anion and simultaneously repel its
nucleus. This produces distortion or polarisation of the anion, which is accompanied by some
sharing of electrons between the ions, i.e., the bond acquires a certain covalent character.

Cation anion Cation anion

separate Polarised anion

The ability of a cation to polarise the near by anion is called its polarising power and the
tendency of an anion to get distorted or deformed or polarised by the cation is called its
polarisability.

Fact or s I nfluencing I on – Defor mat ion or I ncr easing C ovalent C har act er

(i) Large charge on the ions :


The greater the charge on the cation, the more strongly will it attract the electrons of the
anion. For example, Al3+ can distort Clă ion more than Na+ ion. So aluminium chloride is
a covalent compound whereas NaCl, AlF3, AgF are ionic.

CHEMICAL BONDING
QUIZRR 33

(ii) Small cation and large anion :


For a small cation, the electrostatic force with which its nucleus will attract the anion will
be large. Moreover a large anion cannot hold the electrons in its outermost shell, especially
when they are attracted by a neighbouring cation. Hence there will be increased covalence
with a small cation and a large anion, as in AgI.
(iii) Cation with a non-inert gas type of electronic configuration :
A cation with a 18 electron outermost shell such as Ag+ ([Kr] 4d10) polarizes anions more
strongly than a cation with a 8 electron arrangements as in K+. The ÂdÊ electrons in Ag+ do
not screen the nuclear charge as effectively as the ÂsÊ and ÂpÊ electron shell in K+. Thus AgI
is more covalent than KI, although Ag+ and K+ ions are nearly of the small size. Cuprous
and mercurous salts are covalent.
The above statements regarding the factors which influence covalent character are called
FajansÊ rules. It can thus be seen easily that there is nothing like a purely ionic compound
or a purely covalent compound.

8. SHAPES OF MOLECULES
The shapes or geometry of a molecule is quite accurately predicted by VSEPR (valence shell
electron-pair repulsion) theory. According to this theory all valence shell electron pairs
surrounding the central atom arrange themselves in such a manner as to be as far away from each
other as possible. By separating the electrons from each other, electrostatic repulsion, (the cause
of higher energy) is minimised. As a result each molecules tends to acquire a state of lowest
energy.
The basic ideas can be summarized as follows :
Valence shell pairs of electrons are arranged about the central atom so that repulsions among
them are minimized, or so that there is maximum separation among the regions of high electron
density (bond pairs) about the atom. For instance, two regions of high electron density would be
most stable on opposite sides of the central atom (the linear arrangement), while three regions
would be most stable when they are arranged at the corners of an equilateral triangle (the
trigonal planar arrangement). The resulting arragement of these regions is referred to as the
electronic geometry of the central atom.
Number of Region of High Electronic Geometry Bond Angles
Electron Density (bpÊs)
2 linear 180Ĉ
3 trigonal planar 120Ĉ
4 tetrahedral 109Ĉ 28
5 trigonal bipyramidal 90Ĉ, 120Ĉ, 180Ĉ
6 octahedral 90Ĉ, 180Ĉ

We will discuss this table detail a little later.

CHEMICAL BONDING
34 QUIZRR
Some imp or t ant p oint s :

(1) a covalent bond is formed by overlapping of atomic orbitals of avalency shell of the two
atoms. As a result of overlapping, there is maximum electron density some where between
the two atoms.
(2) Greater the overlapping, higher is the strength of chemical bond.
(3) Electrons which are already paired in valency shell can enter into bond formation if they
can be unpaired first and shifted to vacant orbitals of slightly higher energy of the same
main energy shell. (Valency shell).
This point explains the trivalency of boron, tetravalency of carbon, pentavalency of
phosphorus, hexavalency of sulphur inspite of the fact that these atoms have paired orbitals
in the valency shell.

2s 2p 2s 2p

Boron carbon

Excited State

One electron is shifted to p-orbital

3s 3p 3d

Phosphorus

One electron shifted to d-orbital

(4) Between 2 orbitals of same stability (i.e. having same energy) one more directionally
concentrated would form a stronger bond. Dumb-bell shaped p-orbitals will form a stronger
bond as compared to spherically symmetrical s-orbital. It is formed by head on or axial
overlap.

CHEMICAL BONDING
QUIZRR 35

The types of bonds can be formed an account of overlapping.


(1) Sigma Bond : A bond formed between two atoms by the overlap of singly occupied orbitals
along their axes (end to end overlap) is called sigma () bond. The bond formed by this type
of over lapping is very strong as the extent of overlapping is sufficiently high. In sigma
bond, the electron density accumulates between the centres of the atoms being bounded and
lies on the imaginary line joining the nuclei of bonded atoms.
Sigma bonds are formed by three types of overlapping.
(i) s-s overlapping :
For example, hydrogen molecule.
Each hydrogen atom has one electron in is orbital which is spherical. Is orbital of both
the hydrogen atoms approach each other closely and when they reach a point of
maximum attraction by the two nuclei, they overlap and form a sigma bond.

s-s overlap

1s 1s

ï
Molecular axis

Formation of H2 molecule by s-s overlapping

The bond has two electrons which have opposite spins. The probability of finding these
electrons is maximum in the region between the two nuceli on the molecular axis. The
electron density of the bond is distributed symmetrically about the molecular axis.
(ii) s–p overapping (Formation of HF, H2O molecules) :
(a) In the formation of HF molecule the 1s-orbital of hydrogen overlaps with the
p-orbital of fluorine containing unpaired electron.

1s 2p 2s 1p

Hydrogen atom z y x
Fluorine atom

Formation of HF molecule by s-p overlapping

CHEMICAL BONDING
36 QUIZRR
(b) Formation of water molecule : Oxygen atom has the configuration of valency
shell i.e., it has two orbitals singly occupied. These two orbitals overlap with 1 s-
orbital of two hydrogen atoms forming sigma bonds.

2 ăpx

Hydrogen atoms
(1s1)
py
Oxygen atom s-p overlap

H O
104.5Ĉ

M.O.
H

Formation of water molecule by s-p overlapping

Since the two orbitals of oxygen are at right angle to each other an angle of 90Ĉ
is expected between two sigma bonds but actual bond angle observed is 104.5Ĉ.

(iii) p-p overlapping (Formation of fluorine molecule) : This is illustrated by the


formation of fluorine molecule. The electronic configuration of fluorine atom is
1s2 2s2 2 px2 2 p2y 2 p1z , i.e., one orbital is singly occupied. When p-orbitals of two fluorine
atoms approach each other with their heads directly towards one another, they overlap
and form a sigma bond.

2s 2p 2p 2s
p-p overlap

Fluorine atom Fluorine atom

Head on
overlap
p p p-p overlap M.O.
z z

Formation of F2 molecule by p-p overlapping

CHEMICAL BONDING
QUIZRR 37

(2) ) Bond
Pi (
-bonds are formed by the sidewise or lateral overlapping of p-orbitals. The overlapping
takes place at the side of two lobes and hence, the extent of overlapping is relatively smaller.
Thus, -bond is a weaker bond in comparison to sigma bond. The molecular orbital is
oriented above and below the plane containing nuclear axis.

p p p-p overlapping  M.O.

Formation of -bond

Formation of oxygen molecule : Oxygen atom has two p-orbitals singly occupied in the
valency shell. When two oxygen atoms approach each other, one set of p-orbitals experiences
head on overlap forming a sigma bond while other set of p-orbitals overlaps sidewise to form
a -bond. Thus, oxygen molecule has one -and one -bond.
-bond

2s 2p 2p 2s
p-p overlap

(Head on)

px py pz px py pz
-bond
Oxygen atom Oxygen atom
p-p overlap
(sidewise)
Sidewise
overlap

pz pz 
 M.O.
py py
 M.O.
Head on
overlap

Formation of O2 molecule

CHEMICAL BONDING
38 QUIZRR
Similarly, the formation of nitrogen molecule can be explained. It has one sigma bond and
two -bonds.
All single bonds are sigma bonds. A double bond consists of one sigma and one -one while a triple
bond consists of one sigma and two -bonds.
(i) Bond energy increases from a single bond to a trip bond.
Bond energy : Single bond < Double bond < Triple bond
(ii) Bond strength increases from a single bond to a triple bond.
Bond strength : Single bond < Double bond < Triple bond
(iii) Bond length of a multiple bond (double or triple) is always shorter than the corresponding
single bond.
Bond length : Single bond > Double bond > Triple bond
(iv) Reactivity of a multiple bond is always more than the single bond. This is due to the fact
that -electrons are mobile in nature.
Reactivity : Single bond < Double bond < Triple bond.

9. HYBRIDIZATION

The valence bond theory (overlapping concept) explains satisfactorily the formation of various
molecules but it fails to account the geometry and shapes of various molecules. It does not give
explaination why BeCl2 is linear, BF3 is planar. CH4 is tetrahedral, NH3 is pyramidal and water
is V-shaped molecule. In order to explain these cases, the valence bond theory has been
supplemented by the concept of hybridization. This is a hypothetical concept and has been introduced
by Pauling and Slater. According to this concept any number of atomic orbitals of an atom which
differ in energy slightly may mix with each other to form new orbitals called hybrid orbitals. The
process of mixing or amalgamation of atomic orbitals of nearly same energy to produce a set of
entirely new orbitals of equivalent energy is known as hybridization. The following are the rules
of hybridization :
(i) Only orbitals (atomic) of nearly same energy belonging to same atom or ion can take part
in hybridization.
(ii) Number of the hybrid orbitals formed is always equal to number of atomic orbitals which
have taken part in the process of hybridization.
(iii) Most of the hybrid orbitals are similar but they are not necessarily identical in shape. They
may differ from one another in orientation in space.
(iv) Actually the orbitals which undergo hybridization and not the electrons. For example, for
orbitals of nitrogen atom (2s2 2 p1x 2 p1y 2 p1z ) belonging to valency shell when hybridize, form
four hybrid orbitals, one of which has two electrons (as before) and other three have one
electron each.

CHEMICAL BONDING
QUIZRR 39

(v) Hybrid orbitals only form sigma bonds.


It is concept not only predicts the correct shapes but also explains the actual microscopic
observations of the geometry of molecules. The different cases of hybridisation arises by
mixing of s,pd orbitals; few of them are discussed below.

Sp 3 H yb r id isat ion

This type of hybridisation results from mixing of one s, three p orbitals of outer (valence) shell
of an atom. Let us take the example of CH4 molecule.
Carbon atom has configuration 1s2, 2s2 2px1 2py1. In ground state, it has two unpaired orbitals
which can form only two covalent bonds. To get tetravalency, 2s-orbital is unpaired and the
electron is shifted to 2p-orbital. Now in excited state the four unpaired orbitals undergo hybridization
giving rise to four hybrid orbitals which are 109Ĉ28´ apart. The four hybrid orbitals overlap with
s-orbital of each of the four hydrogen atoms forming four sigma bonds. The molecule formed is
tetrahedral.

2s 2p

Carbon atom in ground state

Carbon atom in excited state


sp3 hybridization

H H

C C
109
H H H Ĉ 28´ H

H H

Formation of CH4 molecule

CHEMICAL BONDING
40 QUIZRR
2
Sp H yb r id isat ion

To understand it let us take the example of BF3 molecule.


BF3 molecule : Boron atom has configuration 1s2, 2s2 2p1. In ground state, it has one unpaired
orbital which can form only one covalent bond. To get trivalency, the 2s-orbital is unpaired and
the electron is shifted to 2p-orbital. Now in excited state the three unpaired orbitals undergo

F
F

120

B

12

Ĉ
120Ĉ

B Atom F F
F F

Formation of BF3 molecule

hybridization giving rise to three hybrid orbitals which are 120Ĉ apart. The three hybrid orbitals
overlap with three p-orbitals from three fluorine atoms forming three sigma bonds. The molecule
formed is triangular and planar.

2s 2p

Boron atom in ground state 1s2 , 2 s2 2 p1x

Boron atom in excited state 1s2 , 2 s1 2 p1x 2 p1y


sp2 hybridization

Sp H yb r id isat ion

An sp hybrid orbital is formed when in an atom one s and one p orbital mix with each other to
form two equivalent orbitals. Two such orbitals i.e. sp hybrid orbitals are most stable when they
adopt an angle of 180Ĉ between them so that they are at the maximum distance apart. Hence,
the shape is linear.
BeF2 molecule explains sp hybridisation Beryllium atom has the configuration 1s2, 2s2. Since
there are no unpaired electrons in the valence shell, it cannot form any covalent bond. Thus,
2s-orbital is first unpaired and an electron is shifted to 2p-orbital.

CHEMICAL BONDING
QUIZRR 41

2s 2p

Ground state

Excited state
sp hybridization

These s and p orbitals overlap with p-orbital of fluorine atoms forming two sigma bonds.
The molecule formed is linear with a bond angle 180Ĉ.

F Be F 180Ĉ
F F
Be
Hybrid orbitals

Formation of BeF2 molecule

Sp 3 d hyb r id isat ion

PCl5 molecule : P-atom has configuration 1s2, 2s2 2 p6 , 3s2 3 p1x 3 p1z . In ground state, it can
form three covalent bonds as three unpaired orbitals are present in the valency shell. To get
pentavalency, 3s-orbital is unpaired and the electron is shifted to 3d-orbital. Now in the excited
state the five orbitals involving one să, one dă and three p-orbitals

Cl

Cl

P
Cl

Cl

Cl

Trigonal bipyramid

undergo hybridization giving birth to five hybrid orbitals which overlap with five chlorine atoms
forming five sigma bonds. Out of five -bonds, three bonds which are located at 120Ĉ angle are
equitorial and the remaining two are axial. Axial bond length is greater.

CHEMICAL BONDING
42 QUIZRR
3s 3p 3d

Ground state of P-atom

Excited state of P-atom

sp3d hybridization

Sp 3 d 2 H yb r id isat ion

SF6 Molecule :
3s 3p 3d

Ground state of S-atom

Excited state of S-atom

sp3 d2 hybridization

3s-and paired 3p-orbital are unpaired and electrons are shifted to d-orbitals. After hybridization
six hybrid orbitals directed towards the corners of a regular octahedron come into existence which
overlap with six fluorine atoms. The SF6 molecule has octahedral structure.

F F

F F

Note : Please note that sp3d and sp3d 2 hybridisation cannot be shown by elements of 2nd period
(e.g. Nitrogen, oxygen, Fluorine etc.) because of the absence of low-energy d-orbitals available for
hybridisation.

CHEMICAL BONDING
QUIZRR 43

H yb r id isat ion and G eomet r y of some molecules C ont aining lone p air s of E lect r ons

Sp3 Hybridisation
(a) NH3 molecule : Nitrogen atom undergoes sp3 hybridization forming four hybrid orbitals.

N N N

H H H H

H H
Hybrid orbitals of N Overlapping with Ammonia
hydrogen atoms

Three of the hybrid orbitals contain one electron each while the fourth one has a pair of
electrons. Three hybrid orbitals having one electron each overlap with three hydrogen atoms
forming three sigma bonds while the lone pair of fourth hybrid orbitals remains unused.

2s 2p

Ground state of N-atom

sp3 hybridization

The expected bond angle should be 109Ĉ 28´ but the actual bond angle is 106Ĉ 45´ because
of the repulsion between lone pair and bonded pairs due to which contraction occurs. Thus,
ammonia molecule is pyramidal in shape.

(b) H2O molecule : Oxygen atom undergoes sp3 hybridization forming four hybrid orbitals.
Two of the hybrid orbitals contain one electron each while other two a pair of electrons each.
The hybrid orbitals having one electron each overlap with hydrogen atoms forming two
sigma bonds while the lone pairs of the other hybrid orbitals remain unused.

2s 2p

Ground state of O-atom

sp3 hybridization

CHEMICAL BONDING
44 QUIZRR
The expected bond angle is 109Ĉ28´ but the actual bond angle is 104Ĉ35´. This is due to the
presence of two lone pairs which repel each other and the bonded pairs more strongly and
cause them to come closer and thereby reducing the bond angle form 109Ĉ 28´ to 104Ĉ35´.
Similarly, the geometry of PH3, PCl3, NF3, H2S, etc., can be explained. It is clear from the
above two structures that higher the number of lone pairs present on a central atom, the
greater is the contraction caused in the bond angle. The bond angle is also decreased as the
size of the central atom increases. The bond angle in PH3 is 93Ĉ20´.

O O

H H

H H
Hybrid orbitals of O Overlapping with Water molecule
hydrogen atoms (V-shaped)

G eomet r y and hyb d r isat ion of M olecules having mult ip le b ond s

(i) Ethylene molecule (C2H4) : Ethylene molecule is formed as a result of sp2 hybridization
of carbon. Each carbon atom in excited state undergo sp2 hybridization giving rise to three
hybrid orbitals each. These hybrid orbitals lie in the xy plane while the fourth unhybridized
orbital lies at right angle to the hybridized orbitals. In the formation of ethylene two hybrid
orbitals, i.e., one from each carbon atom form a sigma bond by head on overlap while the
remaining overlap with hydrogen atoms. The unhybridized p-orbitals undergo sidewise
overlap to form a -bond.

2s 2p

Ground state of carbon atom

Excited state of carbon atom


sp2 hybridization

CHEMICAL BONDING
QUIZRR 45

-bond
H H H H
-bond
Sigma Sigma
C C 120Ĉ
bond bond

H H H H
Unhybridized orbital

Formation of Ethylene molecule

Acet ylene molecule (C 2 H 2 ) :

It is formed as a result of sp hybridisation of carbon. Each carbon atom in excited state undergoes
sp hybridisation giving rise to two hybrid orbitals each. Each carbon atom is left with two
unhybridized p-orbitals.

2s 2p

Ground state

Excited state
sp hybridization

The two hybrid orbitals of each carbon atom are used up in forming C ă C and C ă H sigma bonds.
The unhybridized orbitals overlap sidewise to form two -bonds.

-bond

-bond
H H
H C C H
-bond

-bond -bond

Acetylene is linear structure.

1 0 . VSEPR THEORY

As already explained, VSEPR theory say that ÂÂall valence shell electron pairs surrounding the
central atom arrange themselves in such a manner as to be as for away from each other as

CHEMICAL BONDING
46 QUIZRR
possibleÊÊ. Thus, the mutual interaction among the electrons orient the orbitals in space to an
equilibrium position where repulsion becomes minimum. The extent of repulsive interaction then
follows the order lonepair-lone pair > lone pair bond pair > bond pair-bond pair.

E xamp le 7

Why the bond angle of H · C · H in methane (CH4) is 109Ĉ 28´ while H · N · H bond angle
in NH3 is 107Ĉ though both carbon and nitrogen are sp 3 hybridized ?
Solution :
In CH4, there are 4 lone pair of electrons while in NH3 are 3 bond pair of electrons and 1 lone
pair of electrons since bond pair-bond pair repulsion is less than lone pair-bond pair repulsion, in
NH3 bond angle is reduced of 109Ĉ 20´ to 107Ĉ.

E xamp le 8

The bond angle of H2O is 104Ĉ while that of F2O is 102Ĉ. Why ?
Solution :
Concept : ÂÂIf the electronegativity of the peripheral atoms is more, then the bond angle will be
less’’.
Here, both F2O and H2O have a lone pair of electrons. But Fluorine is highly electronegative thus
electrons are drawn more towards F in F2O, whereas in H2O it is drawn towards O. So there will
be more repulsion in H2O molecule between the adjacent bond pairs. Hence the bond angle will
be more.

more repulsion
O O

H F
H F
Less repulsion

E xamp le 9

Out of H2O and H2S which is having greater bond angle and why ?
Solution :
Concept : ÂÂIf the electronegativity of central atom is more, then bond angle will be moreÊÊ
Here since Oxygen is more electronegative than sulphur, hence oxygen draws the shared pair
more towards itself. Hence bond pair repulsion is more in case of H2O.

CHEMICAL BONDING
QUIZRR 47

PREDICTION OF SHAPE OF COVALENT MOLECULES


Geometry can be easily predicted if the state of hybridization of the central atom is known. Steps
for knowing the state of hybridization of central atom.
(1) Detect the central atom
(2) Count the number of -bonds
(3) Count the number of lone pair of electrons on central atom.
(4) Adding the two, if total is 4, the hybridization is sp3. If this total is 3, the hybridization is
sp2 and if total is 2, hybridization is sp.
Note : This is a shortkut method and is not applicable to all molecules.

Some E xamp les :

1. NCl3
 bonds = 3
lone pair on N = 1 N
3
hybridization = sp Cl
Cl
shape = pyramidal
Cl
2. CH 4
-bonds = 4 H

lone pair on C = 0 H C H
3
hybridization = sp
H
shape = Tetratedral

3. CH2O
-bonds = 2
lone pair on O = 2 O

Hybridization = sp3 H H
shape = V-shape (bent)

4. BBr3
 bond = 3 Br
lone pair = 0 B
hybridization = sp2 Br Br
shape = trigonal planar

CHEMICAL BONDING
48 QUIZRR
5. SnCl2
-bonds = 2
lone pair = 1 Sn
hybridization = sp2 Cl Cl
shape = bent

6. Be Cl2
-bonds = 2
lone pair = 0 Cl Be Cl
hybridization = sp
shape = linear

7. PCl5 F
-bonds = 5 F

lone pair = 0 P F
3
hybridization = sp d
F
shape = Trigonal bipyramidal F
F
8. SF4
F F
-bonds = 4
F S S
lone pair = 1
F F
hybridization = sp3d F F
shape = Trigonal bipyramidal (A) (B)
Since it is trigonal bipyramidal structure hence, 2 structures are possible.
Here, the lone pair is placed at different positions. In [B], it is at equational position and in [A],
it is at axial position.
Now, when a lone pair is at equatorial position, the repulsion is minimized.
So structure [B] is correct :
Note : Whenever there are lone pairs in TBP geometry, they should be placed in equational
position so that repulsion is minimum.

9. ClF3 F
-bonds = 3
lone pair = 2
Cl F
hybridization = sp3d
shape = T-shaped.
F

CHEMICAL BONDING
QUIZRR 49

Please note that the lone pairs are placed at equatorial position for repulsion to be minimized.
Now, students may be wondering that the hybridization is sp3d so shape should be TBP. But if
we see only the bonds, it gives us the impression of T-shape.

F
10. XeF2
-bonds = 2
lone pair = 3 Xe
3
hybridization = sp d
shape = linear
F
11. PF2Br3 F
-bonds = 5 Br

lone pair = 0 P Br
hybridization = sp3d Br
shape = Trigonal bipyramidal F

Here we see that fluorine is placed in axial position whereas bromine is placed in equatorial
position. This is because, fluorine being more electronegative pulls away bonded electron towards
itself more than it is done by bromine atom which results in decrease in bond pair-bond pair
repulsion and hence, it is placed in axial position.

12. SF6 F
F F
-bonds = 6
lone pairs = 0 S

hybridization = sp3d 2 F F
F
shape = octahedral/square bipyramidal

13. IF5 F
F F
-bonds = 5
lone pair = 1 I

hybridization = sp3d 2 F F
shape = square pyramidal

14. XeF4
-bonds = 4
lone pairs = 2
hybridization = sp3d 2
shape = square planar

CHEMICAL BONDING
50 QUIZRR
Here 3 structures are possible.

F F
F F
F F
Xe Xe Xe

F F F F
F F

(A) (B) (C)

Now [A] and [B] are same, they can be inter converted by simple rotation of molecule. The basic
difference between [B] and [C] is that in [B] the lone pair is present in the anti-positions which
minimize the repulsion which is not possible in structure [C] where the lone pairs are adjacent.
So, in a octahedral structure, the cone pairs must be placed at the axial positions to minimize
repulsion. So both structure [A] and [B] are correct.

15. IF7 F
-bonds = 7 F F
I F
lone pair = 0
F F
hybridization = sp3d 3
F
shape = pentagonal bipyramidal

Alt er nat e M et hod


Now, it becomes very difficult if the compound is unknown and students do not know the number
of -bonds and -bonds. Also, in cases of ionic compounds, calculating the hybridization and shape
is difficult. Hence, to solve such problems we can apply another method which will be applicable
everywhere.
Steps to be employed :
(1) Identify the central atom along with peripheral atoms.
(2) Count the total valence electrons of the Central and peripheral atoms.
(3) Add the electrons in case net of anionic species and subtract the electrons in case of cationic
species.
(4) If the above value is less than or equal to 8, divide it by 2 and if the value is greater than
8, divide it by 8.
(5) The quotient plus (remainder/2) gives us the total number of typrid orbitals.

Some examples

1. HCN
Total valence electrons = 1 + 4 + 5 = 10

CHEMICAL BONDING
QUIZRR 51

Divide by 8
so, N = 1 + 1
Hence, hybridization = sp H · C N
shape = linear


2. NO3 O

Total valence electrons = 5 + 3 ï 6 + 1 = 24


N
So, N = 3 + 0
ă
hence, hybridization = sp2 O O
shape = Trigonal planar


3. NO2

Total valence electrons = 5 + 12 + 1 = 18


So, N = 2 + 1 N
ă
hence, hybridization = sp2 O O
shape = angular / bent

 O
4. SO24
ă ă
Total valence electrons = 6 + 6 ï 4 + 2 = 32 O S O
So, N = 4 + 0
hybridization = sp3 O

shape = Tetrahedral


5. ClO3

Total valence electrons = 7 + 6 ï 3 + 1 = 26


So, N = 3 + 1 Cl
ă
hybridization = sp3 O O
O
shape = trigonal pyramidal


6. NH2

Total valence electrons = 5 + 2 + 1 = 8


So, N = 4 + 0 N
3
hybridization = sp
H
shape = bent H

CHEMICAL BONDING
52 QUIZRR
7. XeO2F2 O
F
Total valence electrons = 8 + 6 ï 2 + 7 ï 2 = 34
So, N = 4 + 1 Xe

hybridization = sp3d O
shape = distorted TBP (sea ă saw geometry) F


8. I3 I

Total valence electrons = 3 ï 7 + 1 = 22


I
So, N = 2 + 3
hybridization = sp3d
I
shape = linear

9. XeOF4 O
Total valence electrons = 8 + 6 + 4 ï 7 = 42 F F
So, N = 5 + 1 Xe
3 2
hybridization = sp d
F F
shape = square pyradinal

M olecular Shap es
No. of electron Type of Geometry of molecules No. of No. of Actual shape Examples
pairs over hybridisation as per VSEPR theory bonded atoms lone pairs of molecules
central atom

2 sp Linear, 180Ĉ 2 0 Linear CO2, HgCl2,BeF2 , ZnCl2 ,


MgCl2, C2H2, HCN

3 sp2 Trignal planar, 120Ĉ 3 0 Trigonal planar BF3, AlCl3, SO3, C2H4,
NO3ă, CO32ă, HCHO, C6H6
< 120Ĉ 2 1 V-shape (bent) NO2ă, SO2, SnCl2

4 sp3 Tetrahedral, 109.5Ĉ 4 0 Tetrahedral CH4, SiH4, SO42ă, SnCl4,


ClO4ă, BF4ă, NH4+
< 109.5Ĉ 3 1 Trigonal pyramid NH3, PCl3, PH3, AsH3,
ClO3ă
104.5Ĉ 2 2 V-shaped (bent) H2O, H2S, PbCl2, OF2,
NH2ă, ClO2ă

5 sp3 d Trigonal bipyramid 5 0 Trigonal pyramid PCl5, SbCl5


4 1 TeCl4, SF4
3 2 T-shaped ClF3, IF3
2 3 Linear I3ă, XeF2

CHEMICAL BONDING
QUIZRR 53

No. of electron Type of Geometry of molecules No. of No. of Actual shape Examples
pairs over hybridisation as per VSEPR theory bonded atoms lone pairs of molecules
central atom

6 sp3d2 Octahedral 6 0 Octahedral SF6, PF6ă, SnCl6ă,


5 1 Square ICl5, BrF5, IF5
pyramidal
4 2 square planar XeF4, ICl4ă

7 s3d3 Pentagonal bipyramid 7 0 Pentagonal IF7


bipyramid
6 1 distorted
octahedral XeF6

E xamp le 4

Considering x-axis as the internuclear axis, which out of the following will form a sigma
bond ?
(a) 1s and 1s
(b) 1s and 2p x
(c) 2p y and 2p y
(d) 2p x and 2p y
(e) 1s and 2s
Solution :
The sigma bond is formed by axial overlap and is present in the following cases :
(a) 1s and 1s
(b) 1s and 2px
(c) 1s and 2s

1 1 . MOLECULAR ORBITAL THEORY


The valence bond theory is based on the assumption that the formation of a molecule involves an
interaction between the electron waves of only those atomic orbitals of the prticipating atoms
which are half filled. These atomic orbitals mix with one another to form a new orbital of greater
stability while all other orbitals on the atoms remain undisturbed or maintain their individual
identity. But this cannot be correct because the nucleus of one approaching atom is bound to
affect the electron waves of nearly all the orbitals of the other atom. Besides this in valence bond
theory fails to explain the formation of coordinate bond, the paramagnetic character of O2 molecule
and the formation of odd electron molecules or ions such as H+2 ion where no pairing of electron
occurs.
Molecular orbital theory of chemical bonding is more rational and more useful in comparison to
valence bond theory. This theory was put forward by Hund and Mulliken. According to this

CHEMICAL BONDING
54 QUIZRR
theory, all the atomic orbitals of the atoms participating in molecule formation get disturbed when
the concerned nuceli approach nearer. They all get mixed up to give rise to an equivalent number
of new orbitals that belong to the molecule now. These are called molecular orbitals. The electrons
belonging originally to the participating atoms are now considered to be moving along the molecular
orbitals under the influence of all the nuclei.
Linear combination of atomic orbitals (LCAO); Formation of bonding and antibonding
molecular orbitals : Molecular orbitals of a molecule are obtained by the linear combination of
atomic orbitals of the bonded atoms. The electron waves that describe the atomic orbitals have
positive and negative phase or amplitude just as there are positive (upward) and negative
(downward) amplitudes associated with standing wave. When wave are combined, they may
interact either constructively or destructively. If the two identical waves are added, they combine
constructively to produce the wave with double the amplitude and same wavelength. Conversely,
if they are subtracted, they combine destructively to produce the wave with zero amplitude.



+ a
ă 2a

(a)
+ a
ă Amplitude 2a

+ a
ă

(b) Amplitude zero

Likewise when two atomic orbitals overlap they can be in phase (added) or out of phase (subtracted).
If they overlap in phase, constructive interaction occurs in the region between two nuceli and a
bonding orbital is produced. The energy of the bonding orbital is always lower (more stable) than
the energies of the combining atomic orbitals. When they overlap out of phase, destructive
interference reduces the probability of finding an electron in the region between the nuceli and
antibonding orbital is produced. The energy of an antibonding orbital is higher (less stable) than
the energies of the combining atomic orbitals. The antibonding orbitals are represented by subscript
asterisks, i.e., * , * , * , etc. Thus, the number of molecular orbitals formed from atomic orbitals

CHEMICAL BONDING
QUIZRR 55

is equal to the number of atomic orbitals responsible for their formation. The formation can be
represented graphically.

Antibonding M.O.
Subtraction
Energy Repulsive

Atomic Atomic
orbital orbital

Addition
Bonding M.O.
Attractive

The different notations for representing responding and anti-bonding Molecular orbitals are

Atomic orbitals s and s Pz and Pz Px and Px Py and Py


that are mixed

Bonding M.O. s (pz) (px) (py)

Antibonding M.O. s * (pz) *(px) *(py)

We have assumed the two pz orbitals to overlap end to end, so that the M.O. formed is of the ÂÊ
type (similar to the ÂÊ bond in V.B. theory); then the two Px atomic orbitals, as also the two py
orbitals will overlap laterally to give M.OÊs of the  type.
Ć When electrons are successively placed in the M.OÊs, Aufbau principle, HundÊs rule and
PauliÊs principle are followed, as in the case of the atomic orbitals.
Aufbau Principle : M.OÊs are occupied in the order of increasing energy. The following is the
general arrangement of M.OÊs in the order of increasing energy.
(1s) < * (1s) < (2s) < * (2s) <  (2pz) < (2px) = (2py) < * (2px) = * (2py) < * (2pz)
... etc.
The above is only a general order and slight variations often occur due to interaction
between s and p orbitals. For example, sometimes, (2px) = (2py) < (2pz).
Ć HundÊs rule of maximum multiplicity :
The degenerate M.OÊs are occupied singly, before any pairing could occur. The maximum
capacity for each M.O. is 2 electrons.
Ć Only atomic robitals of equal or nearly equal energies combine to give the M.OÊs. In the case
of homonuclear diatomic molecules, energies of corresponding A.OÊs of the two atoms are
equal so the above condition of combination of A.OÊs assumes special significance in the case
of Hetero nuclear diatomic molecules and it has to be used with caution.

CHEMICAL BONDING
56 QUIZRR
Formation of molecular orbitals.
(1) From s-atomic orbitals.

+ *1s
ion
ă
tract
sub
+ Energy
add
itio
n
Is Is + 1s

(2) From p orbitals

ă + ă + * 2px
tion
btrac
su
+ Energy
2px 2px add
itio
n 2px
ă + ă

+ ă y* 2py
on
tracti
sub ă +
+ Energy
add
itio +
n
2py 2py y 2py

CHEMICAL BONDING
QUIZRR 57

E ner gy level d iagr am for M olecular O r b it als

 *2px *2px

2p *2py *2pz 2p 2p *2py *2pz 2p


2px 2py 2pz

Increasing Energy
2py 2pz 2px

2s *2s 2s 2s *2s 2s
2s 2s

1s *1s 1s 1s *1s 1s
1s 1s

Atomic Molecular Atomic Atomic Molecular Atomic


Orbitals Orbitals Orbitals Orbitals Orbitals Orbitals

For elements with Z < 7 For elements with Z > 7


Molecular orbital energy level diagram Molecular orbital energy level diagram

BOND ORDER AND BOND STABILITY

No. of electrons in bonding No. of electron in antibonding


·
Molecular orbitals molecular orbitals
Bond order =
2

Ć Greater the bond order, the more stable it will be. Hence,
(i) Bond energy  Bond order

1
(ii) Bond length 
Bond order

Ć When bond order is zero or less than zero, the species is unstable and doesnot exist.
Ć A bond order of +1 indicates a stable single bond.
Ć A bond order of +2 indicates a stable double bon.
Ć A bond order of +3 is a stable triple bond.

CHEMICAL BONDING
58 QUIZRR
Ć A bond order of + óshows that the species is unstable but it exists.
Ć If the molecule contains unpaired electrons, it will be paramagnetic and hence it is attracted
in a magnetic field.
Ć If the molecule has all paired electrons, the molecule will be diamagnetic and hence it is
repelled in the magnetic field.

Note : Fractional bond order indicates ion.

M olecular O r b it al C onfigur at ion of Some homonuclear d iat omic sp ecies

1. H2 : It is formed from 1s atomic orbitals of two atoms.

*1s

1s2 1s 1s2


Atomic Atomic
orbitals orbitals
Molecular
orbitals

Hence, M.O. configuration of H2 is  1s2

2ă0
B.O. =  1
2

Hence it is stable and diamagnetic

2. He2 : Electronic configuration of He atom : 1s2

*1s1

1s2 1s2 1s2

2ă2
B.O. =  O
2

Hence He2 is not stable.

CHEMICAL BONDING
QUIZRR 59

+
3. He 2

*1s1

1s2

the structure is 1s2, *1s1

2ă1
B.O. =  ó
2

Hence, He+2 is stabler than He2

4. Li2 (Li atom : 1s2 2s1)

2s1 *2s0 2s1

2s2

1s2 *1s2 1s2

1s2

4ă2
B.O. =  1
2

hence, Li2 is stable and is found to exist to some extent in lithium vapour.

CHEMICAL BONDING
60 QUIZRR
5. N2 (N atom : 1s2 2s2 2p 3)

*2px

2p 2p
*2py *2pz
2px

2py 2pz

2s2 2s2
*2s2

2s2

1s2 1s2
*1s2

1s2

1
B.O. = (10  4)  3
2

Thus molecule is diamagnetic and has a triple bond.

CHEMICAL BONDING
QUIZRR 61

6. O2 : [O : 1s2 2s2 2p 4]

*2px

2p 2p
*2py *2pz

2py 2pz

2px

10  6
B.O. = 2
2

Due to the presence of unpaired electrons in the two antibonding orbitals, O2 is paramagnetic.

Let us now compare the bond strength of O2, O2 , O2 , O22 .

7. For O2 : one electron from the antibonding M.O. has been removed.

83
 B.O.   2.5
2

For O2 one electron is added to the antibonding M.O.

85
 B.O.   1.5
2

2 86
For O2 B.O.  1
2

 the bond is strongest in O2 and the bond length the least.

HETERONUCLEAR DIATOMIC SPECIES

 N: 1s2 2s2 2 p3 
(1) NO  
2 2 4
 O: 1s 2s 2 p 

Total valence electrons = 5 + 6 = 11

CHEMICAL BONDING
62 QUIZRR
Please note that since Oxygen is present, hence we take molecular energy diagram for z > 7
Hence,

*2px

*2py *2pz

2py 2pz

2px

*2s2

2s2 2s2

2s2

83
B.O.   2.5
2

Due to the presence of an unpaired electron in an antibonding orbital, NO is paramagnetic.


Let us compare the B.Os in NO, NO+, NO2+, NOă,

For NO : B.O. = 2.5

83
For NO+ : B.O. =  2.5 = (note that NO+ is isoelectronic with N )
2 2

72
For NO2+ : B.O.   2.5
2

[NO2+ is formed by removing 1eă from an antibonding orbital and 1eă from a bonding orbital of
NO]

CHEMICAL BONDING
QUIZRR 63

(2) Carbon monoxide molecule (CO)


Total valence electrons = 4 + 6 = 10
The distribution of electrons in various molecular orbitals of CO molecule is done in the following
manner :

*
(2s)2 ( 2s)2 (2 p y )2 (2 pz)2 (2 px )2

1
Bond order  (8  2)  3
2

It represents one sigma bond and two -bonds. The molecule is stale and diamagnetic in nature.

E xa m p le 11
Find the bond orders in the species, CN, CNă, CN+, F2 and FO.
Solution :
(a) CN. On the basis of the simple treatment of A.O overlap, the nine valence electrons (4 + 5)
in CN are distributed as
2s 2, 2s 2, 2p()4, 2p()1

72
B.O.   2.5
2

(b) CNă (isoelectronic with N2). B.O. = 3


(c) CN+ (isoelectronic with C2). B.O. = 2
(d) F2 : The valence (14) electrons are distributed as 2s 2, 2s 2, 2s 2, 2p()4, 2p()*4

86
B. O.  1
2

(e) FO : Number of valence electrons = 7 + 6 = 13.


The distribution may be represented as 2s 2, 2s 2, 2s 2, 2p4, 2p3

85
B.O.   1.5
2

CHEMICAL BONDING
64 QUIZRR
E xamp le 12
Compare the bond energies of N2, N 2 & N 2 .
Solution :
The configuration of N2 is

 * 
 2 py  2  
12s 1*2s 22 s *2
2s   2 px 2 py 
 2 pz   
 2 pz 

Now N 2 means removal of an electron from a bonding M.O. This will decrease the B.O.

 B.O. of N 2 = ó(5 ă 0) = 2.5



Now again for N 2 bond order is ó(6 ă 1) = 2.5

So from the bond order it may seem that both N 2 & N 2 may have the same bond energy. But
removal of an electron from a diatomic species tend to decrease the inter electronic repulsion and
thereby shortens the bond lengths. So the bond energy becomes more than compared to N 2 .

 N2 > N 2 > N 2

E xamp le 13

Compare the bond length of OăO in following molecules :


KO2, O2, O2[AsF6]
Solution :

KO2 has O2

O2[AsF6] has O2 Bond order = 2.5

O2 : Bond order = 2
Bond length in increasing order.

O2 < O2 < O2

E xamp le 14

Which diatomic molecule of second period besides O2 should be paramagnetic?


Solution :
B2ă (1s)2 ( 1s)2 ( 2s)2 ( 2px)2 (2py)2 = (2pz)2 ( 2pz)1 = ( 2py)1 ( 2px)0
As, paramagnetism arises due to unpaired electron. Therefore B2 is paramagnetic molecule.

CHEMICAL BONDING
QUIZRR 65

1 2 . INTERMOLECULAR FORCES
Forces operating between different molecules are called intermolecular forces.
Types : The following are the main types of intermolecular forces :
(1) Dipole ă dipole interaction
(2) Dipole ă induced dipole interaction
(3) Induced dipole - induced dipole interaction
(4) Ion-dipole interaction
(5) Hydrogen bonding
Forces operating different molecules due to the separation of charge is called Vander Waals forces.
It is a weak force. The first three forces are basically the manifestation of vander waals force in
different types of chemical systems.
(1) Dipole-dipole : Consider a sample of HBr. The electronegativity difference between Hydrogen
and bromine is quite appreciable. Thus, a considerable amount of charge separation takes
place resulting in the formation of a dipole H+ ă Bră. This leads to a force of attraction
between the molecules (which are dipoles in this case).

+ ă + ă
H Br H Br
ă
Br
+
H

(2) Dipole-induced dipole interaction : The word „induced‰ indicates that a molecule which
originally did not have charge separation, is forced to have it under the influence of another
molecule having centres of positive and negative charge (dipole) to take for distance the
dissolution of the H2O.
H2O having centres of partial +ve charge (H) and partial negative charge (O) forces Cl2 to
undergo the electron cloud of one of the Cl atoms. Thus, it will develop a partial positive
charge. On the other hand, hydrogen atom will attract electrons cloud, thus chlorine atom.

+ ă +
Cl Cl H Hydrogen will attract eă1s
+ ă
O Cl Cl
Oxygen will repel eă1s
H

(3) Induced dipole-induced dipole interaction (Dispersive or london force) : These type
of interactions take place between neutral molecules due to instantaneous charge separation.

CHEMICAL BONDING
66 QUIZRR
Consider a sample of Cl2. The electron cloud around chlorine atoms is not stationary. It
vibrates so, sometimes, one chlorine atom may attract it more than the other, leading to
sudden appearance of partial positive and negative charges. Now, one molecule is surrounded
by millions of other molecules. Whenever the instantaneous dipoles and suitable oriented
(positive pole of one close to negative pole of other), it leads to weak force of interaction
which are called induced dipole-induced dipole interaction.

(4) Ion-dipole interaction : This type of interaction can be suitably explained using the
dissolution of NaCl in H2O. The +vely charged Na+ ions are attracted by the oxygen atoms
and Clă by the hydrogen atom of H2O. It is called hydration.

+ +
H H

+ O ă +
H  H
ă
O Na + O

+ +
H ă H
O Hydration of cation

+ +
H H

ă
O

+ +
H H
+ +
H H

ăO ă
Cl
ă
O

+ +
H H Hydration of anion
+ +
H H

ă
O

In hydration, energy is liberated due to attraction among oppositively charged ions and energy
of the system is lowered. This energy is called heat of hydration or hydration energy.
Cation having smaller size will have higher hydration energy because charge density will
be higher. Relative magnitude of the above forces.
Induced dipole-induced dipole < Dipole-induced dipole < Dipole ă dipole < Ion
ă dipole

CHEMICAL BONDING
QUIZRR 67

1 3 . HYDROGEN BONDING
It is an electrostatic attractive force between covalently bonded hydrogen atom of one molecule
or a part of a molecule and an electro negative atom (such as F, O, N) of another molecule or
another part of the same molecule. It is very weak (strength about 2-10 k ca molă1 or 8.4 ă 42
kJ molă1) as compared to a covalent bond (strength 50ă100 kcal molă1 or 209ă418.4 kJ molă1).

C ond it ions for H yd r ogen Bond ing


1. The molecule must possess a highly electronegative atom linked to hydrogen atom.
2. The size of the electronegative atom should be small.

T yp es of H yd r ogen Bond ing

1. Inter-Molecular Hydrogen Bonding : In this, two molecules of the same compound join
to form aggregates. For example,

H H H

N H N H N H

H H H

H H H

O H O H O H

Intermolecular hydrogen bonding increases the boiling point of the compound and also its
solubility in water.

2. Intramolecular Hydrogen Bonding : In this hydrogen occurs between atoms of the same
molecules present on different sites. This leads to intramolecular ring formation and this
phenomenon is called chelation. Some examples are :

H
O H O O
Cl N
O

CHEMICAL BONDING
68 QUIZRR
9 . 3 C onseq uences of hyd r ogen b ond ing

(a) High boiling and melting points of NH3, H2O and HI in comparison to hydrides of other
elements of V, VI and VII groups to which N, O and F belong respectively are due to
hydrogen bonding.
(b) Unique properties of water and ice are due to hydrogen bonding. Density of ice is less than
water and water contracts when heated between 0ĈC to 4ĈC. These two unique properties
are explained on account of open cage structure of ice due to hydrogen bonding.
(c) Strength of certain acids and bases can be explained on the basis of hydrogen bonding eg.
when we compare the acidic strength of 0 - m - and p - hydroxy benzoic acid.

OH OH OH
COOH

COOH
COOH

Ka = 10.5 ï 10ă5 Ka = 8.3 ï 10ă5 Ka = 2.9 ï 10ă5

The abnormally high dissociation constant of o - hydroxybenzoic acid is due to the fact that
the conjugate base is stablised by hydrogen bonding.
(d) Solubility : The organic compounds like alkane, alkenes, alkynes are insoluble in water due
to absence of H-bonding whereas alcohols, organic acids, amines are soluble in water due
to H-bonding.
(e) State of matter : H2O exists in liquid state whereas H2S in gaseous state because hydrogen
bonding exist in water and no H-bonding exists in H2S.

INERT PAIR EFFECT

Heavier p-block and d-block elements show two oxidation states. One is equal to group number
and second is group number minus two. Fore example.
Pb (5s2, 5p2) shows two oxidation states, + II and + IV. Here + II is more stable than +IV. Here
+II is more stable than +IV which arises after loss of all four valence electrons. Reason for more
stability of +II O.S. is that 5s2 electrons are reluctant to participate in bonding because bond
energy released after bond formation is loss than that required to unpair these electrons.

BACK BO NDI NG

Sometimes, in order to make up electron deficiency in the central atom, the surrounding atom
forms a -bond which is one side, i.e. shared electrons are contributed by surrounding atom only

CHEMICAL BONDING
QUIZRR 69

which is having lone pair of electrons . Over lap involves vacant orbital on the central atom and
filled orbital on surrounding atom.

B F

RESONANCE
It is generally observed that a single valence bond structure of a molecule cannot correctly
account for the properties of the molecule. In such cases, the concept of resonance is introduced.
According to this concept if two or more alternate valence bond structures can be written for a
molecule, the actual structure is said to be a resonance or mesomeric hybrid of all these alternate
structures. For example, carbon dioxide molecule can be represented by the following three
structures :

+ ă ă +
O C O O C O O C O
I II III

The calculated values of bond distances between carbon and oxygen in C == O and C  O are
1.22 Å and 1.10 Å, respectively but the observed bond distance between carbon and oxygen in
carbon dioxide is 1.15 Å. Thus, none of the above structures correctly explains the observed bond
length. It is, thus, said that a hybrid form of these structures can provide the exact explanation.
The various structures of which the molecule is a resonance hybrid are known as canonical forms
or mesomeric forms. Actually resonance by brid does not oscillate between the canonical forms of
a mixture of these forms but it is a definite form and has a definite structure which cannot be
written on paper.

Rules for Selecting Canonical Forms or Mesomeric Forms :


(i) The relative position of all the atoms in each of the canonical forms must be the same. They
should differ only in the position of electrons.
(ii) The number of unpaired and paired electrons in each of the canonical forms must be same.
(iii) The contributing structures should not differ much in energy.
(iv) The contributing structures should be such that negative charge resides on more
electronegative and positive charge on the electropositive. Like charges should not reside on
atoms close together in the canonical forms.
Formal charges on the atoms in the molecule help us in choosing the most appropriate
resonance structure. For example, nitrous oxide molecule is represented by two resonance

CHEMICAL BONDING
70 QUIZRR
electron dot structures, one of which has a negative formal charge on the oxygen atom and
the other of which has a negative charge on the terminal nitrogen atom.

+ ă ă +
N N O N N O

Since oxygen is a more electronegative element than nitrogen, the structure that places a
negative formal charge on oxygen is probably lower in energy than the structure that has
a negative formal charge on nitrogen.
+ ă
Thus, the actual structure of N2O is N N O

(v) As a result of resonance, the bond order changes in many molecules or ions.

Total number of bonds between two


atoms in all the structures
Bond order =
Total number of resonating structures

e.g., (a) 2+1


B.O. = = 1.5
2

O Oă Oă
(b)
C C C
Oă Oă O Oă Oă O

2+1+ 1
B.O. =  1.33
3

R esonance E ner gy

It has been observed that the molecule which shows resonance has greater heat of formation than
the calculated heat of formation of any one of canonical forms. The difference is represented by
E and its called resonance energy.

 Calculated value of heat 


 Experimental value   
E =     of formation of most 
 of heat of formation   
 stable canonical form 

On account of E, the hybrid form is more stable than any of the canonical forms.

CHEMICAL BONDING
QUIZRR 71

Energy difference between most stable resonating structure and resonance hybrid structure is
called resonance energy.

Some E xamp les showing R esonance

(i) Nitric oxide molecule : It is a resonance hybrid of following two structures :

ï +

ï
ï
ï ï
ï N O ï N O

(ii) Nitrogen dioxide molecule : It is a resonance hybrid of the following structures :

ï ï
N N

O O O O

(iii) Carbonate ion : it is the resonance hybrid of the following forms :

O Oă Oă

C C C
ă
O Oă O Oă ă
O O

CHEMICAL BONDING
72 QUIZRR

MISCELLANEOUS EXAMPLES

E xamp le 1

Ethyl alcohol (C2H5OH) has higher boiling point than dimethyl ether (CH3-O-CH3) although
the molecular weight of both are same.
Solution :
Though ethyl alcohol and dimethyl ether have the same molecular weight but in ethyl alcohol the
hydrogen of the O-H groups forms intermolecular hydrogen bonding with the OH group in
another molecule. But in case of ether the hydrogen is linked to C is not so electronegative to
encourage the hydrogen to form hydrogen bonding.

C2H5 C2H5

O H O H

Due to intermolecular H-bonding, ethyl alcohol remains in the associated form and therefore boils
at a higher temperature compared to dimethyl ether.

E xamp le 2
Explain unusual stability of chlorohydrate through a compound with two or moreăOH
groups present on one carbon atom is usually unstable.
Solution :
The unusual stability of chloral hydrate has been attributed to ăI effect of chlorine and to the
formation of interamolecular hydrogen bonds.

H
O
Cl
Cl C CH
Cl
O
H

E xamp le 3
The salt KHF2 is known but KHCl2 is not known. Explain.
Solution :

The formation of KHF2 involves reaction of HF2 with KOH. Similar is the case with KHCl2.

So the main factor is the formation of HF2 or HCl2 ion.

H  F  F   F  H......F 

CHEMICAL BONDING
QUIZRR 73

H  Cl  Cl   Cl  H......Cl  (not possible)

Due to higher electronegativity and small size of fluorine, it is capable of forming H-bond resulting

in the formation of HF2 and thereby KHF2 exists. But with chlorine, there is no possibility of
H-bonding, so there is no possibility of existence of KHCl2.

E xamp le 4

o-hydroxy benzaldehyde is more volatile than p-hydroxy benzaldehyde.


Solution :
More volatility means compound has lower boiling point. p-hydroxy benzaldehyde remains associated
through intermolecular hydrogen bonding. But in o-hydroxy benzaldehyde, intramolecular H-
bonding takes places, as a result of which there is no association. So p-hydroxy benzaldehyde,
which remains as an associated species has got higher boiling and so less volatile while o-hydroxy
benzaldehyde is highly volatile.

E xamp le 5

Though the electronegativities of nitrogen and chlorine are same, NH3 exists as liquid
whereas HCl as gas. Why ?
Solution :
The size of nitrogen is less than the size of chlorine. Therefore, electron density in nitrogen is more
than that of chlorine. So, nitrogen forms hydrogen bonding leading to association of molecules.
Hence, NH3 is liquid. Hydrogen bonding is not possible with chlorine.

E xamp le 6

Explain giving reason :

ClF2 linear, but the ion ClF2 is bent.

Solution :

In ClF2 central chlorine atom involves sp3d hybridization, to have minimum electronic repulsion
three lone pairs should be in equatorial position as follows; giving linear shape to the ion.

Whereas, in case of ClF2 ion central atom Cl involves sp3 hybridization having two lone pairs,

resulting in bent shape for the ion, (bond angle less than 109Ĉ28´ due to repulsion of bond pair
by lone pair).

CHEMICAL BONDING
74 QUIZRR

+
ă
120Ĉ Cl

F F
Bent shape
F

E xamp le 7

Arrange in order of increasing dipole moment BF3, H2S, H2O


Solution :
BF3 has zero dipole moment because of symmetry, H2S has a lower dipole moment than H2O
because of much lower bond polarity of HăS bond compared to HăO bond. Hence the order is BF3
< H2S < H2O.

E xamp le 8

Explain with reason :


(i) Two different bond lengths are observed in PF5 but only one bond length observed
in SF6.
(ii) O-hydroxy benzaldehyde is liquid at room temperature while p-hydroxy
benzaldehyde is high melting solid.
Solution :
(i) PF5 has trigonal pyramidal structures (sp3d hybridization of central atom) in which bond
angles are 90Ĉ and 120Ĉ respectively and there are two types of bond-axial and equatorial
(axial being longer than equatorial). In case of SF6 the structure is octahedral (sp3d2
hybridization of the centra atom-S) resulting only one type of bond, bond angle (90Ĉ) and
one type of bond length.
F

F F

F F
P (PF5 ) (SF6 )
F F S
F F
Bond angle ă 90Ĉ
F F
(Bond angle 90Ĉ & 120Ĉ )

CHEMICAL BONDING
QUIZRR 75

(ii) There is intramolecular H bonding in o-hydroxy benzaldehyde while intermolecular hydrogen


bonding in p-hydroxy benzaldehyde.

CH O H O
CH O H

O-hydroxy benzaldehyde CHO


O H OCH

OH

E xamp le 9

CO2 is a gas whereas SiO2 is solid at room temperature. Explain.


Solution :
O =C = O
In CO2, p-p overlap is possible due to shorter C ă O bond length and similarity in energy
between the orbitals of carbon and oxygen. But in SiO2 -overlap is not possible and thereby to
complete the octet Si has to go for intermolecular multiplicity, i.e. to form 4-sigma bonds with
oxygen, hence it has high meting point.

E xamp le 10

Explain the following :


(a) NF3 is inert towards hydrolysis, but NC3 undergoes hydrolysis rapidly, why ?
(b) Syn-form of pyridine-2-carbaxaldoxime is less stable than that of the anti-form.
Why?
Solution :
(a) For hydrolysis, either the central atom, or the atom around the central must contain vacant
d-orbitals.
In NF3, neither N nor F contains vacant d-orbitals, therefore it will not undergo hydrolysis.
But in NCl3, Cl atom contains vacant d-orbital. Therefore, it will undergo hydrolysis.
 NCl3, + 4H2O  NH4OH + 3HCl
(b) Anti-form of pyridine-2-carbaxaldoxime, has intramolecular H-bond, but the syn-form of
pyridine-2-carbaxaldoxime does not have intramolecular H-bond. Hence, syn form is more
stable.

CHEMICAL BONDING
76 QUIZRR
E xamp le 11

In the equation, A + 2B + H2O  C + 2D (A = HNO2, B = H2SO3, C = NH2OH), identify D.


Draw the structures of A, B, and D.
Solution :

HNO2  2H2SO3  H2O  NH2OH  2H2SO4


(A) (B) (C) (D)

Structures· (A) HNO2; HO · N == O,

(B) H2 SO3 ; HO · S  O,
|
OH

(C) NH 2OH; H · N · OH
|
H

O
(D) H2 SO4 ; HO · S
| O
OH

E xamp le 12
Which of the two peroxide ion or superoxide ion has larger bond length ?
Solution :
The bond length in a molecule is dependent on bond order. The higher is the bond order, smaller
will be the bond length.


Peroxide ion, O 2

2ă * * *
O2 : KK ( 2s)2 ( 2s)2 (2 px )2 (2 py )2 (2 pz )2 ( 2 p y )2 ( 2 pz )2

8ă6
Bond order = 1
2

ă
Superoxide ion, O 2

ă * * *
O 2 : KK ( 2s)2 ( 2s)2 (2 px )2 (2 py )2 (2 pz )2 ( 2 p y )2 ( 2 pz )1

CHEMICAL BONDING
QUIZRR 77

8ă5
Bond order =  1.5
2

Bond order of superoxide ion is higher than peroxide ion, hence bond length of peroxide ion is
larger.

E xamp le 13

Which d-orbital is involved in dsp 2 ă hybridization?


Solution :

d This is because its four lobes lie along the x-axis and y-axis. The two p-orbitals cn combine
x2  y2 .

along these axes.

E xamp le 14

(i) Arrange the following as indicated:


(a) NH3, PH3, AsH3, SbH3 (increasing bond angle)
(b) MgCO3, BaCO3, CaCO3, BeCO3 (increasing stability)
Solution :
(i) (a) NH3 < PH3 < AsH3 < SbH3
(b) BeCO3 < MgCO3 < CaCO3 < BaCO3

E xamp le 15

Give the structures of compounds and hybridization of central atom in each of the following
species.

(a) NF4 (b) N 2 F3 (c) NH2OH (d) SPCl3 (e) PF3Cl2

Solution :
F
(a) Tetrahedral +
N (sp3)
F F
F

+
(b) Planar at each nitrogen atom F N N (sp 2)

CHEMICAL BONDING
78 QUIZRR
H

(c) Pyramidal at N and bent at O N O (sp 3)

H H

Cl Cl
(d) Tetrahedral S P (sp 3)

Cl

F
Cl

(e) Trigonal bipyramidal P F (sp3d)

Cl
F

E xamp le 16
(a) Why is the bond energy of PăCl bond different in PCl3 and PCl5 ?
(b) Out of CH3Cl and CHCl3, which one will have higher dipole moment and why ?
(c) Bond angle in NH3 is greater than NF3 whereas the bond angle in PH3 is lesser
than PF3. Why ?
Solution :
(a) I n PCl 3, P exhibits sp3 hybridization and molecule is triangular pyramidal. But in PCl5, P
exhibits sp3 d hybridization. Therefore, it is trigonal bipyramidal. Three P-Cl bonds are on
the plane while two P-Cl bonds are above and below the plane which are different from
remaining three P-Cl bonds.
(b) CH3Cl is having higher dipole moment in comparison to CHCl3. Because in CHCl3, each Că
Cl bond dipole induces an opposite dipole on the other two C-Cl bonds. Each C-Cl moment
is thereby reduced, ultimately lowering the overall dipole moment of the molecule.
(c) In case of NF3, F atoms are highly electronegative because of which the charge cloud is
displaced further away from the central atom and repulsion decreases, thereby reducing the
bond angle.

N N

H H F F
H F

CHEMICAL BONDING
QUIZRR 79

In case of PF3, the valence shell of phosphorus atom is incompletely filled. The high charge
density of electron on the F atom causes a partial-shift of its lone pairs to the phosphorus
atom via pi-bond formation. The P-F bonds attain some double bond character resulting in
increased bp-bp repulsion thereby increasing F-P-F bond angle. This is not possible in case
of NF3 because of non-availability of vacant d-orbitals on nitrogen.

E xamp le 17

Why H2O is liquid while H2S is a gas at ordinary temperature ?


Solution :
Oxygen has high electronegativity than sulphur. As a result, H2O forms hydrogen bonding.
Consequently, molecules of water come nearer to each other through hydrogen bonding. This
results in higher boiling point of water and hence it is a liquid.

E xamp le 18

The density of ice is less than that of water or ice floats over water. Explain.
Solution :
In ice, water molecule is associated with four other molecules through hydrogen bonding in a
tetrahedral manner, i.e., it has open-cage like structure in which lesser molecules are packed per
mL. When ice melts, the molecules come closer to one another. Consequently the density of water
in liquid state is more than in solid state. Thus, ice floats over water.

E xamp le 19

Both carbon-oxygen bonds in the methanoate ion, HCOOă, have the same length (127 pm).
What does this tell about bonding ?
Solution :
This is due to the presence of resonance in the molecule.

ă
O O

H C H C

Oă O

Each bond has part of the character of single bond and part of character of a double bond. The
electrons are delocalised over the three atoms.

O
H C ă

CHEMICAL BONDING
80 QUIZRR
E xamp le 20

The dipole moment of LiH is 1.964 ï 10ă29 cm and the interatomic distance between Li and
H in this molecule is 596 A. What is the per cent ionic character in LiH ?
Solution :
The dipole moment of 100% ionic molecule (Li+Hă) = (1 electronic charge) (interatomic distance)
= (1.602 ï 10ă19 C) (1.596 ï 10ă10 m)
+
= 2.557 ï 10ă29 Cm O H
Fractional ionic character

C Oă
Exp. value of dipole moment
=
Theoretical value of dipole moment
H
(o-hydroxybenzaldehyde)
1.964 ï 10 ă29 Intrahydrogen bonding
=  0.768
2.557 ï 10 ă29

The bond in LiH is 76.8% ionic.

E xamp le 21

Discuss the hybridization of carbon atoms in allene (C3H4) and show the p-orbital overlaps.
[I.I.T. 1999]
Solution :

I II III
Allene is CH2 == C == CH2 carbon atoms I and III are in sp2 hybridized state while carbon II

is in sp hybridized state. Two unhybridized orbitals of carbon II overlap sidewise with unhybridized
orbital of each of carbon I and carbon III to form -bonds besides it also forms a sigma bond with
each of the carbon atoms using hybridized orbitals. Two hybridized orbitals of each of carbon I
and III form sigma bonds with hydrogen atoms.

I II III
sp2 -bond
sp 2

-bond
- sp2 sp sp2
bo sp
nd
sp2
2
sp
-bond
-bond
-bond
nd
bo
-

CHEMICAL BONDING
PERIODIC TABLE
& PROPERTIES
QUIZRR 3

PERIODIC PROPERTIES

Introduction

The basic object of classification is to arrange the facts regarding elements and compounds in such
a way so that we may have greatest control over their characteristics with least possible efforts.
Such a classification is called as Periodic table.
Some of the earlier attempts by scientists are discussed below.

Dobereiner’s Triads

In 1829, J. Dobereiner made an important first step towards a systematic classification by arranging
elements into groups of three (triads). He observed that the atomic weight of the middle element
of the group was midway between the atomic weights of the other two. He also concluded that
the same midpoint relation held true for the physical properties of these elements.
Some of his triads were (Li, Na, K), (CI, Br, I), (Ca, Sr, Ba), (S, Se,Te)

Newland’s law of Octave

The first attempt to classify all the known elements was made by Newland in 1864.
He proposed that if the elements are arranged according to the increasing atomic weights (beginning
with lightest element, excluding Hydrogen), the chemical and physical properties of a particular
element would be similar to those of the elements seven places before and seven places after it.
For example, Li, the second element in NewlandÊs list, had properties similar to Na, the ninth
element in the list and K, the sixteenth element. Thus these were similar elements represented
by numbers 2, 9, 16 which show interval of seven.

Mendeleev’s Periodic Law and Classification

In 1869, Mendeleev classified the then known 56 elements on the basis of their physical and
chemical properties by giving a law known as the periodic law. The name periodic law stems from
the fact that the properties of the elements orderly recur in a cyclic fashion. His periodic law states
that „the physical and chemical properties of the elements are periodic function of
their atomic masses.‰ This means that when the elements are arranged in the order of their
increasing atomic masses, the elements with similar properties recur at regular intervals. Based
on this law all the known elements were arranged in the form of a table known as periodic table.
Elements with similar properties recur at regular intervals and fall in certain groups or families.
The elements in each group were similar to each other in many properties.

Merits of Mendeleev’s periodic table

MendeleevÊs periodic table was one of the greatest achievements in Chemistry with some of its
important contributions as follows :

PERIODIC TABLE & PROPERTIES


4 QUIZRR
Ć As the arrangements are classified into groups, it was very useful in studying and
remembering the properties of a large number of elements in a systematic way.
Ć This classification helped in correcting the atomic masses of some of the elements like gold,
beryllium and platinum based on their positions in the periodic table.
Ć Mendeleev could predict the properties of some undiscovered elements like scandium,
galliumand germanium. By this intuition, he had left gaps for the undiscovered elements
while arranging elements in his periodic table.

Demertis of Mendeleev’s periodic table

In spite of the above advantages, MendeleevÊs periodic table suffered defects as follows :
Ć The position of hydrogen was not correctly defined. It was placed in group I although it
resembles both the group I elements - the alkali metals and the group VII elements-the
halogens, in their properties.
Ć In some cases Mendeleev placed elements according to their similarities in properties and not
in increasing order of their atomic masses. Thus, the position of these elements was not
justified e.g. cobalt (atomic mass 58.9) was placed before nickel (atomic mass 58.6).
Ć Isotopes were not given separate places in the periodic table although MendeleevÊs classification
is based on the atomic masses.
Ć Some similar elements were grouped separately while some dissimilar elements were grouped
together. For example copper and mercury are similar in their properties but were placed
separately. Copper was placed in group I although it did not resemble the elements of this
group.
Ć Mendeleev could not explain the cause of periodicity in the elements.
Ć the position for lanthanides and actinides were not included in this table.

Long form of periodic table

With developments on the structure of atoms, it was discovered that the atomic number (Z) is the
important characteristic of the atom and not the atomic mass. This led to the development of the
modern periodic law by Moseley in 1942. The modern periodic law states that „the physical and
chemical properties of the elements are periodic function of their atomic numbers.‰
Thus, when the elements were arranged in the order of their increasing atomic numbers, the
elements of similar properties recur at regular intervals.
The long form of the periodic table or the modern periodic table is based on the general plan of
the table as proposed by Mendeleev. This table is the most widely used periodic table of today.

Structural Features of the Modern Periodic Table

This table consists of horizontal rows called as ÂperiodsÊ and vertical columns called as ÂgroupsÊ.

PERIODIC TABLE & PROPERTIES


QUIZRR 5

PERIODIC TABLE & PROPERTIES


6 QUIZRR
P eriods
There are seven periods in the periodic table and each period starts with a different principal
quantum number.
The first period corresponding to ÂnÊ = 1 consists of only two elements hydrogen (1s1) and helium
(1s2). This is because the first energy shell has only orbital (1s), which can accommodate only two
electrons.
In the second period corresponding to ÂnÊ = 2, there are four orbitals (one Â2sÊ and three Â2pÊ)
having a capacity of eight electrons and so contains eight elements. This period starts with lithium
(Z = 3) with electron entering the Â2sÊ orbital and ends with neon (Z = 10) where the second shell
is complete (2s22p6).
In the third period corresponding to ÂnÊ = 3, there are nine orbitals (one Â2sÊ three Â2pÊ and five
Â3dÊ). As Â3dÊ orbitals are higher in energy, they are filled after the Â4sÊ orbitals. This period
involves filling of only four orbitals (Â3sÊ and Â3pÊ) containing eight elements. It starts with sodium
(Z = 11) with electron entering the Â3sÊ orbital and ends with argon (Z = 18) where the third shell
is partially complete (3s2 3p6).
The fourth period corresponding to ÂnÊ = 4, consists of filling of one Â4sÊ and three Â4pÊ orbitals. The
Â4dÊ and Â4fÊ orbital are higher in energy than the Â5sÊ orbitals and are filled later. The five Â3dÊ
orbitals have energies in between Â4sÊ and Â4pÊ orbitals and so are filled accordingly. Totally nine
orbitals are filled and therefore, there are eighteen elements in this period starting from potassium
with electron entering the Â4sÊ orbital (Z = 19) to krypton (Z = 36) where the third shell gets
completed (4s2 3d10 4p6).
In the fifth period there are 18 elements like the fourth period. It begins with rubidium (Z = 37)
with the filling of Â5sÊ orbital and ends with xenon (Z = 54) with the filling of Â5pÊ orbital.
The sixth period contains 32 elements (Z = 55 to 86) and the successive electrons enter into Â6sÊ,
Â4fÊ, Â5dÊ and Â6pÊ orbitals in that order. It starts with caesium and ends with radon.
The seventh period, though expected to have 32 elements is incomplete and contains only 19
elements at present.

Number of elements in each period


Period (n) Orbital Number of electron/
filled up elements in the period
First (1) 1s 2 = 2
Second (2) 2s 2p 2 + 6 = 8
Third (3) 3s 3p 2 + 6 = 9
Fourth (4) 4s 3d 4p 2 + 10 + 16 = 18
Fifth (5) 5s 4d 5p 2 + 10 + 16 = 18
Sixth (6) 6s 4f 5d 6p 2 + 14 + 10 + 6= 32
Seventh (7) 7s 5f 6d 7p 2 + 14 + 10 + 6= 32

PERIODIC TABLE & PROPERTIES


QUIZRR 7

There is a periodicitiy occurring at regular intervals of 2, 8, 8, 18 and 32 and so the numbers 2,


8, 18 and 32 are called magic numbers. The first three periods are called short periods while the
other three periods are called long periods.

G roups

The vertical column in the periodic table is called as group. There are 18 groups in the long form
of the periodic table and they are numbered from 1 to 18 in the IUPAC system. In the old system
of naming they are numbered as I to VIII with A and B groups. This convention is followed in
many places.

Merits of the long form of the periodic table

Ć This classification is based on the most fundamental property of the elements - the atomic
number, so it is more accurate.
Ć With the atomic number as the basis of this classification, the position of isotopes in one
place is justified.
Ć The systematic grouping of elements into the four blocks of ÂsÊ, ÂpÊ, ÂdÊ and ÂfÊ has made the
study of elements simpler.
Ć The electronic configuration determines the properties of the elements. The position of elements
governed by this feature is useful in studying the properties of elements.
Ć The position of the element, which were misfit on the basis of atomic mass is now justified
on the basis of atomic number.
Ć The lanthanides and actinides have been placed separately due to their properties different
from other groups.
Ć The whole table is easy to remember and reproduce in terms of electronic configuration and
properties of the elements.

Demerits of the long form of the periodic table

Although the long form of the period table has been able to help in systematic studying the
elements to a great extent, it has some minor defects.
Ć Hydrogen resembles both the alkali metals and halogens. But it has been placed with the
alkalis.
Ć The lanthanides and actinides have not been placed in the main body of the table.

Analysis of Table in form of Blocks

With a better understanding of the part that the electron plays great role in the properties of
elements, a corresponding understanding of the periodic system came about.
On the basis of electronic configuration the elements may be divided into four groups.

PERIODIC TABLE & PROPERTIES


8 QUIZRR
S-block elements

Ć These are present in the left part of the periodic table.


Ć These are IA and II A i.e. 1 and 2 group elements.
Ć These are metals.
Ć In these elements last electron fills in the s-orbital.
Ć Electron configuration of valence shell is ns1-2 (n = 1 to 7)

P-block elements

Ć These are present in the right part of the table.


Ć These constitute the groups III A to VII A and zero groups i.e. groups 13 to 18 of periodic
table.
Ć Most of these elements are metalloids and non-metals but some of them are metals also.
Ć The last electron fills in p-orbital of valence shell.
Ć The electronic configuration of valence shell is ns2 np1ă6 (n = 2 to 7)
Ć ns2 np6 is stable noble gas configuration.

s-block elements

These are present in the middle part of the periodic table (between s and p block elements).
Ć These constitute III B to VII B, VIII, I B and II B i.e., 3 to 12 groups of the periodic table.
Ć All are metals
Ć The last electrons fill in (n ă 1) d orbital
Ć The outermost electronic configuration is i(nă1) d 1ă10 ns1ă2 (n = 4 to 7)
Ć There are three series of d-block elements as under :
3d series ă Sc (21) to Zn (30)
4d series ă Y (39) to Cd (48)
5d series ă La (57), Hf (72) to Hg (80)

f-block elements

Ć These are placed separately below the main periodic table.


Ć These are mainly related to III B i.e, groups of the periodic table.
Ć There are two series of f-block elements as under :
4f series ă Lanthanides ă 14 elements from Ce (58) to Lu (71)
5f series ă Actinides ă 14 elements from Th (90) to Lw (103)
Ć The last electron fills in (n-2)f orbital
Ć Their outermost electronic configuration is (nă2)f1ă14 (n-1)s2 (nă1) p6 (n-1)d0-1 ns2 (n = 6 and 7)

PERIODIC TABLE & PROPERTIES


QUIZRR 9

Example 1

Predict the period, group number and block of the following elements. A (at. number = 8),
B (at. number = 11), C (at. number = 28)
Solution :
Electronic configurations of different elements are
A  1s2 2s2 2p4
B  1s2 2s2 2p6 3s1
C  1s2 2s2 2p6 3s2 3p6 3d8 4s2
Element A : p ă block element
Group number = 10 + number of electron in the valence shell = 10 + 6 = 16
Period of the element = Principal quantum number of the valence shell = 2nd
Element B : s ă block element
Group number = number of electrons in valence shell = 1
Period number = 3rd
Element C : d ă block element

The Types of Elements

Using electronic configuration as the criterion, we ordinarily recognize some general type of
elements.

Typical Elements

Elements of third period are also called as TYPICAL ELEMENTS. These include Na, Mg, Al, Si,
P, S, Cl. The properties of all the elements belonging to a particular group resembles the properties
of the corresponding typical element of that group. For example, the general properties of Alkali
Metals can be predicted from the behaviour of Na, not Li, the first member of the family.
The typical elements (all having n = 3) can take up 18 electrons. Note that, for these elements
3d sub-shell is available, but it is not filled i.e., these have vacant d sub-shell. This is not the case
with second period elements, hence they have somewhat different properties than the rest of the
group or we can say that it is the typical element, which in true sense represents a group.

Bridge Elements

Elements of second period are also called as BRIDGE ELEMENTS. The properties these elements
resemble with the properties of elements belonging to third period placed diagonally. This is
illustrated as follows :

PERIODIC TABLE & PROPERTIES


10 QUIZRR

2nd Period Li Be B C N O F

3rd Period Na Mg Al Si P S Ci

Noble or Inert Gases

Elements of VIII A group or zero group are called as INERT or NOBLE GASES. They have
completely filled (2 or 8 electrons in outermost shell) outermost shells, called as stable configuration.
Their valency is zero. They are almost inert in their chemical behaviour. They have weak
intermolecular forces in them and hence are gases and exists in mono-atomic states.

Transition Elements

Ć Last two shells of these elements namely outermost and penultimate shells are incomplete.
Ć The last shell contains one or two electrons and the penultimate shell may contain more
than eight up to eighteen electrons.
Ć Their outermost electronic configuration is similar to d-block elements i.e. (n-1)d1-10 ns1-2.
Ć According to latest definition of transition elements those elements which have partly filled
d-orbitals in neutral state or in any stable oxidation state are called transition elements.
According to this definition Zn, Cu and Hg (II B group) and d-block elements but not
transition elements because these elements have d10 configuration in neutral as well as in
stable +2 oxidation state.

Inner Transition Elements

Ć In these elements last three shell i.e., last, penultimate and perpenultimate shells are
incomplete.
Ć These are related to III B i.e., group 3
Ć The last shell contains two electrons. Penultimate shell may contain eight or nine electrons
and pre-penultimate shell contains more than 18 upto 32 electrons.
Ć Their outermost electronic configuration is similar to f-block element i.e., (n-2)f1ă14 (n-1)s2
(n-1)p6 (n-1)d0-1 ns2
Elements of the seventh period after atomic number 93 (i.e. actinides are synthetic elements
and are called transuramium elements.
We can further divide the elements into three classes based on their positions in periodic table.

Metals

This is the largest class of the elements. This includes elements belonging to IA, IIA, IB to VIIIB
(i.e., all transition and inner-transition elements) and some elements of groups IIIA to VA lying
near the bottom of the table. The metals are characterised by their nature of readily giving up
the electron apart from shinning lustre. The oxides of metals are basic in nature.

PERIODIC TABLE & PROPERTIES


QUIZRR 11

N on-metals

These do not give up electron, in fact like to take up the electron to form negative ion. These
include 10 elements lying to the right side of the table. They are C, N, O, F (2nd period), P, S,
Cl (3rd period), Se, Br (4th period) and I (5th period). The oxides of non-metals are acidic in
nature.

Metalloid

You are very easily observed that metallic character has decreased when one moves to the right
of the table across a row. It is observed that some elements lying at the border of metallic and
non-metallic behavior, exhibit both the metallic and non-metallic character, these are called as
metalloids. These include 8 elements namely; B, Si, Ge, As, Sb, Te, Po and At. The oxides of
metalloids are generally amphoteric in nature.
Note : The elements in group VIII A do not behave like metals, nor do they behave like non-
metals. So they are classified separately as Noble Gases. Also the element Hydrogen (H) is
different from any other element and can not be easily classified into a particular group (however
it is placed along with the Alkali Metals, though it does not exhibit metallic character).

Periodic Properties

From the discussion of the periodic table, it is evident that those properties which depend upon
the electronic configuration of an atom will vary periodically with atomic number. On the other
hand, those properties which depend upon the total number of electrons will show no such
variations. Some of the common properties which depend upon electronic configuration are :

Valency

It is defined as combining capacity of an element. It can also be defined in terms of valence


electrons (electrons in the outermost shells). The valency is equal to number of valence electrons
(or equal to 8 minus the number of valence electrons).
For representative elements, in general, valency of elements belonging to a chemical family i.e.,
in a group is constant. It is known from its general electronic configuration. To illustrate, for
group I A (Alkali metals), valency is 1 (ns1), for II A (Alkaline earth metals), it is 2 (ns2) and for
group III A, it is 3 (ns2 np1).
For transition elements, no general trend is observed in the valency of elements. The reason is
that those elements have variable valencies due to availability of vacant d-subshells in them.
Similar is the case with inner-transition elements (f-block elements) i.e., no general trend is
observed due to variable valency.

PERIODIC TABLE & PROPERTIES


12 QUIZRR
Effective Nuclear Charge

Between the outer most valence electrons and the nucleus of an atom, there exists finite number
of shells containing electrons. Due to the presence of these intervening electrons, the valence
electrons are unable to experience the attractive pull of the actual number of protons in the
nucleus. These intervening electrons act as shield between the valence electrons and protons in
the nucleus. Thus, the presence of intervening (shielding) electrons reduces the electrostatic
attraction between the proton in the nucleus and the valence electrons because intervening
electrons repel the valence electrons. The concept of effective nuclear charge allows us to account
for the effects of shielding on periodic properties.
For example, let us consider a helium atom, which has ground state electron configuration as 1s2.
Helium has two protons in the nucleus which gives it a charge of +2 but the full attractive force
of this charge on the two 1s electrons is partially nullified by electronăelectron repulsion.
Consequently, the 1s electrons shield each other from the nucleus.
The effective nuclear charge (Zeff) is the charge felt bythe valence electron. Zeff is
given by Zeff = Z ă 
where Z is the actual nuclear charge (atomic number of the element) and  is the shielding
(screening) constant. The shielding constant is greater than zero but smaller than Z and depends
on the number of intervening electrons and their type of subshell.
To illustrate shielding effect, let us remove both the electrons from a helium atom, one by one.
It would take 2370 kJ of energy to remove the first electron from 1 mole of He atoms and 5250
kJ of energy to remove the remaining electron from 1 mole of He+ ions. The removal of second
electrons takes much more energy than the removal of first electron because in He+ ion, only one
electron is present and there would be no shielding and the electron experiences the full effect
of +2 nuclear charge.
For atoms with three or more electrons, the electrons in a given shell are shielded by electrons
in the same shell and electrons in the inner shells but not by electrons in outer shells. For
example, in lithium atom (electron configuration 1s22s1), the 2s electron is shielded by the two 1s
electrons but 1s electrons are not shielded by 2s electron. In addition, the electrons of the inner
shell shield the electrons of outer shell, more effectively than the electrons of the same inner shell.
There are some simple rules for estimating the degree to which electrons in the various types of
orbitals shield other electrons from the nucleus & hence for estimating the Zeff experienced by
other electrons.
1. After writing the full electron configuration it in the following groupings and order. Group
ns and np orbitals together while all other type of orbitals are grouped separated. For
example,
(1s) (2s, 2p) (3s, 3p) (3d) (4s, 4p) (4d) (4f) (5s, 5p)....
2. Electrons to the right of the group of electrons in question contribute nothing to the shielding
of that group of electrons.

PERIODIC TABLE & PROPERTIES


QUIZRR 13

3. All other electrons in the same group as the electron in question shield that electron to an
extent of 0.35 unit of nuclear charge each.
4. If the electron in question is an s or p electron : (a) All electrons with principal quantum
number one less than the electron in question shield it to an extent of 0.85 unit of nuclear
charge each (b) All electrons with principal quantum number two or more less than the
electron in question shield it completely i.e., to an extent of 1 unit.
5. If the electron in question is a d or f electron : All electrons to the left of the group of the
electron in question shield the ÂdÊ or ÂfÊ electron completely (to an extent of 1 unit). This is
due to poor penetration effect of ÂdÊ or ÂfÊ electron.
6. Sum the shielding constants from steps 2ă5 and subtract them from the actual nuclear
charge (Z) of the atom in question to obtain the Zeff felt by the electron in question.

Atomic Radius

Electron clouds do not have sharp edges but when atoms are packed together in solids, their
centres are found at definite distances from one another.
For atoms, which link together, to form an extensive threeădimensional network, atomic
radius is oneăhalf the distance between the nuclei in two neighbouring atoms. This is
called metallic or crystal radius.

For elements which exist as simple diatomic molecules, the atomic radius is oneăhalf
the distance between the nuclei of the two atoms in a particular molecule. This is
called covalent radius.

In case of mono atomic gases (like noble gases), the atomic radius is oneăhalf of the
distance between the nuclei of two atoms at some distance. This is called van der
Waals radius.

PERIODIC TABLE & PROPERTIES


14 QUIZRR

Thus, rcovalent < rmetallic < rvan der Waals


Let us see the variation of atomic radius in a period. For example, consider the second period
elements from Li to F. On moving from left to right, we can see that the number of electrons in
the inner shell (1s2) remains constant while the nuclear charge increase. The electrons that are
added to counter balance the increasing nuclear charge are ineffective in shielding one another.
Consequently, the effective nuclear charge increases steadily while the principal quantum number
remains constant (n = 2). For example, the outer 2s electron in lithium is shielded from the
nucleus (which has 3 protons) by the two 1s electrons. Each 1s electron shields the 2s electron
by 0.85 units, so net shielding by two 1s electrons is 1.7 (). Thus, the effective nuclear charge
of Li is 1.3. The electron configuration of Be is 1s22s2. One 2s electron shields other 2s electron
by 0.35 unit while each 1s electron shields 2s electron by 0.85. So, the Zeff for Be would be 4ă
[(2 ï 0.85) + 0.35] = 1.95. Similarly, the Zeff for B would be 5ă[(2 ï 0.85)] = 2.6. Thus, it is evident
that the effective nuclear charge increases, so progressively the outermost electrons are held more
strongly by the nucleus and thus the atomic radius decreases from Li to F.
Now, let us observe the trend of atomic radius in a group. For the alkali metals in group IA (Li,
Na, K, Rb, Cs & Fr), the outermost electron resides in the ns orbital. In moving from Li to Na,
the increasing principal quantum number n, which superseeds the effect of increasing Zeff. So,
thus the size of the metal atoms increase from Li to Na and so on. Thus, atomic radius increases
on moving from top to bottom in a group.

I onic Radius

Ionic radius is the radius of a cation or an anion. When a neutral atom is observed to an
ion, these would be change in size, as the Zeff will change but the number of protons in nucleus
remains same.
If the atom forms an anions, its size (or radius) increases, since the nuclear charge remains the
same but the repulsion resulting from the additional electron(s) enlarges the domain of the
electron cloud. On the other hand, if one or more electrons are removed from an atom, it reduces
the electronăelectron repulsion but the nuclear charge remains the same, so the electron cloud
shrinks and the cation is smaller than the atom. When a lithium atom reacts with a fluorine
atom to form a LiF unit, the changes in size are very peculiar. Out of Li and F, Li is bigger
in size. When lithium changes to Li+, its size decreases and when F changes to Fă, its size
increases. In LiF, Li+ is smaller than Fă (Note that Li+ is smaller than F atom and Fă is smaller
than Li atom).

PERIODIC TABLE & PROPERTIES


QUIZRR 15

+ ă
Li F Li F

The variation of ionic radii in a period and a group is same as that of atomic radii. Thus, ionic
radius decreases in a period while it increases in a group.
For ions derived from elements in different groups, a size comparison is meaningful only if the
ions are isoelectronic. If we examine isoelectronic ions, the cations are smaller than anions. Let
us compare the radius of Na+ ion and Fă ion. Both ions have same number of electrons (10), but
Na (Z = 11) has more protons than F (Z = 9). Thus, Zeff of Na+ is more than that of Fă, so Na+
ion is smaller in size than Fă ion. Similarly, for the three isoelectronic ions of third period, Al3+,
Mg2+ and Na+, they all have the same number of electrons (10) but their number of protons are
13, 12 and 11 respectively. Thus, the electron cloud in Al3+ is pulled inward more than that in
Mg2+ and Mg2+ would be smaller than Na+. Thus in general in an isoelectronic cation series, the
radii of tripositive ions are smaller than those of dipositive ions, which in turn are smaller than
unipositive ions. Similarly, in isoelectronic anions series, the radius increases as we go from
uninegative ion to dinegative ion and so on.

Example 2

Compare the size of Cl, Clă and Fe++ ion.


Solution :

Z 17
ratio for Cl =  1.00
e 17

17
Clă =  0.944
18

26
Fe++ =  1.08
24

Clă > Cl > Fe++

Example 3
The radii of Ar is greater than the radii of chlorine.
Solution :
In chlorine, the radii means the atomic or covalent radii which is actually half the intermolecular
distance between 2 atoms whereas in Argon the radii means the Vanderwaals radii as Argon is
not a diatomic molecule. Vanderwaals radii is actually half the distance between adjacent molecule.
So VanderwaalÊs radii being larger than atomic radii, Argon, has got a larger radii than chlorine.
PERIODIC TABLE & PROPERTIES
16 QUIZRR
I onisation Energy

Ć Ionisation energy (IE), sometimes also called ionisation potential (IP), of an element
is defined as the amount of energy required to remove an electron from an isolated gaseous
 IE 
atom of that element resulting in the formation of positive ion. M( g)   M  g   e

Ć (IE) is thus a direct measure of the ease with which an atom can change into cation. The
smaller the ionisation energy, the easier it is for the neutral atom to change into a positive
ion.
Ć (IE)1, (IE)2, (IE)3... are respectively first, second, third,... ionisation energies required to
remove first, second, third, ... electron from the isolated gaseous atoms.

 IE 1
M( g)   M  g   e

 IE 2
M+ ( g)   M2  g   e

 IE 3
M2+ ( g)   M3  g   e

Thus, second ionisation energy, (IE)2, causes ionisation of M+ (g) to form M2+ (g) and third
ionisation energy, (IE)3 causes ionisation of M2+ (g) to form M3+ (g) and so on...
Ć (IE)3 is also called first ionisation energy of M2+ (g) and second ionisation energy of M+ (g).
(IE)2 is also called first ionisation energy of M+ (g).
Ć If an electron has been removed from an atom, it becomes increasingly difficult
to remove the second and subsequent electrons from the resulting positively
charged ions on account of electrostatic attraction. This is due to the fact that
after the removal of an electron, the number of electrons decreases while the
nuclear charge remains the same. Consequently, the remaining electrons are
held more tightly by the nucleus and it becomes difficult to remove second
electron. Similarly it is further difficult to remove electron from M2+ (g). Thus
(IE)1 < (IE)2 < (IE)3 < ...

Factors Influencing Ionisation Energy

(IE) variation in a period and group may or may not be regular and can be influenced by
(1) Size of the Atom
In a small atom, the electrons are tightly held, while in a larger atom, the electrons are less

k Ze2
strongly held, the Coulombic force of attraction F being F 
r2

PERIODIC TABLE & PROPERTIES


QUIZRR 17

where Ze is charge on nucleus, e the electronic charge, r the atomic size and k the constant.
Thus, the ionisation energy decreases as the size of the atom increases.

(2) Nuclear Charge


As given above, the force of attraction between the nucleus and the outermost electron increases
with increase in nuclear charge. Greater the nuclear charge, greater the energy required to pull
the electron from the atom. Here (IE) increases with increase in nuclear charge.

(3) Shielding Effect


The electrons in the inner-shells act as a screen or shield between the nucleus and the electron
in the outermost-shell. This is called shielding or screening effect. The larger the number of
electrons in the inner-shells, greater is the screening effect and smaller the force of attraction and
thus (IE) decreases.

These electrons shield the outer


electrons from the nucleus

This electron does not feel the full effect


of the positive charge of the nucleus

Shielding is most effective whenever there is a full shell (or shells) of electrons between the
outermost electron and the nucleus, as in case of noble gases (group VIII A). Hence, there is a
sharp decrease in (IE) going from noble gas to alkali metals as given below :

He  LI Ne  Na Ar  K

IE 2372 520 2081 513 1521 419


(kJ molă1)

The shielding is not so important across a period as it is down a group. One reason is that an
electron in, say, a px orbital has little shielding effect on an electron in a py or pz orbital (px, py
and pz are mutually at right angles to one another hence as shielding). s-orbitals are spherically
symmetrical, hence filling of 1s or 2s will experience shielding effect.

PERIODIC TABLE & PROPERTIES


18 QUIZRR
(4) Penetration Effect
The ionisation energy also depends on the type of electron which is removed. s, p, d and f-
electrons have orbitals with different shape. An s-electron penetrates nearer to the nucleus, and
is therefore more tightly held than a d-electron, and a d electron is more tightly held than an f-
electron. Other factors being equal, ionisation energies are in the order s > p > d > f. Thus, the
increase in (IE) is not quite smooth on moving from left to right in the periodic table. (IE)1 of
group 13 (III A) elements (where a p-electron is being removed) is actuall less than adjacent
group 2 (II A) (where an s-electron is being removed).

increasing nuclear charge


Increase wins over shielding
in shielding
wins over Min Max
increasing
nuclear IE increases
charge across a period
IE decreases
down a group

Min

(5) Electronic Configuration


If an atom has fully-filled or half-filled orbitals, its (IE) is higher than expected normally from
its position in the periodic table.

He2
highly stable due to
complete valence
1s2 shell hence
maximum (IE) in
Ne10 their respective periods.

1s2 2s2 2p6

N stable half-filled
p-orbitals
P
ns2 np3

Variation of (IE) in a Group


On moving down a group :
(i) nuclear charge increases
(ii) Z* (effective nuclear charge) due to screening is almost constant.

PERIODIC TABLE & PROPERTIES


QUIZRR 19

(iii) number of shells increases, hence atomic size increases.


(iv) there is increase in the number of inner electrons which shield the valence electrons from
the nucleus.
Thus, force of attraction between electrons and nucleus decreases and tendency to remove the
valence electron increases. Hence (IE) decreases on moving down the group. Variations of (IE)1
of alkali metals (group I) and boron family (group 13) are given in Fig. 9.6.
In general the first ionisation energy decreases in a regular way on descending the main groups.
A departure from this trend occurs in group 13 (boron family), where the expected decrease occurs

B
(IE)1 Tl
Al Ga
Li Na In
K
Rb Cs
Z
Variation of (IE)1 of groups 1 ( ) and 13 (o),
(not according to scale)

between B and A1, but the values for the remaining elements Ga, In and TI do not continue the
trend and are irregular. Ga with completely filled 3d-orbitals (no 3d in Al) decreases screening
thus Ga is smaller in size than it would otherwise be

(IE) (IE)1
in kJ molă1
Group 1 Group 13

Li 520 B 801

Na 496 Al 577 decrease

K 419 Ga 579 constant

Rb 403 In 558 decrease

Cs 376 T1 589 increase

decrease

As expected the first ionisation energies decrease down the groups in case of the main group
elements in the periodic table. But in case of transition elements opposite trends are observed.
Thus, (I E)1 of the corresponding elements of 3d and 4d-series are almost similar but these an
smaller than the (IE)1 of the 5d-series elements. Certainly, the higher values of ionisation energies
of the 5d-transition elements are consistent with the relatively small size of then atoms.

PERIODIC TABLE & PROPERTIES


20 QUIZRR
Variation of (IE) in a Period

On moving across a period, the atomic size decreases and nuclear charge increases and therefore
the force of attraction exerted by the nucleus on the electron in outer most shell increases. Hence

3 4 5 6 7 8 9 10
Z
1.3 1.95 2.6 3.25 3.9 4.55 5.2 5.85
Z*
n
rn
F
(IE)
Li Be B C N O F Ne
Na Z-atomic number,
Z*-effective nuclear charge due to screening
K n- orbits, rn-radius

Rb
Cs (IE) ă ionisation energy
Fr increases in the given direction
constant

Variation of (IE) in a group and period

(IE) increases along a period from left to right. Fig. above explains general variation of (IE)
in a group and period. Fig. below shows the pattern of the ionisation energies of the elements H
to Ca.

2500
He
Ne
lonisation energy (kJ/mol)

2000
2p3 F
Ar
1500
2s2 N Cl
H C O P S
1000 Mg
Be B Ca
Li Si
500
2p1 Na Al
K
0 5 10 15 20
Atomic number
Variation of IE with atomic number showing
apparent anomalies of b and O, and Al and S

Ć There are two bumps, or dips, around beryllium/boron and nitrogen/oxygen. The dip from
beryllium to boron occurs because the extra electron of boron enters a 2p-orbital and this
electron will feel the effects of the shielding by the pair of electrons in 2s-orbital. The
shielding has the effect of lowering of (IE). The increase from boron to carbon reflects the
increased nuclear charge of carbon. Also, because the two 2p-electrons of carbon are in
different orbitals, they have little shielding effect on one another. A similar state of affairs
accounts for the rise from carbon to nitrogen.

PERIODIC TABLE & PROPERTIES


QUIZRR 21

Ć The dip from nitrogen to oxygen is a different matter. Compared to nitrogen, the oxygen
atomÊs extra electron must go into a 2p-orbital that already has one electron in it. Putting
two electrons into the same orbital brings about repulsion between them. For this reason it
is easier to remove an electron from 2p-orbital in oxygen containing two electrons than it
is from a similar orbital in nitrogen containing only one electron. This effect overweights the
effect of nuclear charge from nitrogen to oxygen.
Ć From oxygen to neon, the increase in ionisation energy reflects the increase in nuclear
charge. In a general trend there will be peaks for noble gases for the values of (IE).
Ć metals have relatively low IE compared to nonmetals.
Ć The IE, of the metalloids generally fall between those of metals and nonmetals. The difference
in IE suggests why metals always form cations and non-metals from anions.
Ć It is relatively easy to remove an electron from a partially filled valence shell, where Zeff is
lower but it is relatively difficult to remove an electron from an atom or ion that has a filled
valence shell, when Zeff is higher. In other words, there is a large amount of stability
associated with filled s and p-sub-shells (a noble gas electron configuration), which corresponds
to having eight electrons (an octet) in the valence shell of an atom or ion. Thus,
sodium ([Ne]3s1) loses one electron easily
magnesium (Ne] 3s2) loses only two electrons easily,
aluminium [Ne]3s2 3p1 loses only three electrons easily, and so on
8 electron is valence shells

Na(1s22s22p63s2) = Na+ (1s22s22p6) + eă

Example 4

Two atoms have the electronic configurations (EC) 1s2 2s2 2p6 and 1s2, 2s2 2p6 3s1. (IE)1 of
one is 2080 kJ molă1 and that of the other is 497 kJ molă1. Match each (IE)1, with one of the
given (EC). Justify your choice.
Solution :
Element (EC)
A 1s2 2s2 2p6
1s2 2s2 2p6
stable inert gas E.C. and removal of
electron requires higher energy

B 1s–2 2s2 2p6 3s3

1s2 2s2 2p6 3s1

has unpaired electron, removal of


electron requires lower energy

Thus (IE)1 of A is 2080 kJ molă1 and that of B is 496 kJ molă1.


PERIODIC TABLE & PROPERTIES
22 QUIZRR
Example 5

Arrange the elements Se, Cl and S in increasing order of ionisation energy.


Solution :
Ionisation energy generally increases from left to right across a two of the periodic table and
decreases from top to bottom down a group.
(IE) increases
S16 Cl17
(IE)
decrease
Se34

Thus, (IE) of Cl > S


and (IE) of Se < S
Thus, the order is : Se < S < Cl.

Example 6

Which has the larger fifth ionisation energy, Ge or As ?


Solution :

Group
Ge (32) : [Ar] 4s2 3d10 4p2 IVA
2 10 3
As (33) : [Ar] 4s 3d 4p VA

As their positions in the periodic table indicate, the group IVA element Ge has four valence-shell
electrons and thus four relatively low ionisation energies, whereas the group VA element. As has
five valence-shell electrons and five low ionisation energies. Ge thus has larger fifth ionisation
energy than As.

Example 7

Magnesium has the first and second ionisation potential 7.646 and 15.035 eV respectively.
What is the amount of energy required to convert all the magnesium atoms present in 24
mg to magnesium vapours.
Solution :

Mg(g) + IE1  Mg+ (g) + eă 1E1 = 7.646 eV

Mg+ (g) + IE2  Mg++ (g) + eă IE2 = 15.035 eV

PERIODIC TABLE & PROPERTIES


QUIZRR 23

So, for the reaction Mg(g)  Mg++ (g) + 2eă, energy required

= 7.646 eV + 15.035 eV = 22.681 eV


= 22.681 ï 96.49 kJ molă1
= 2188.489 kJ molă1
24 mg of magnesium = 1 ï 10ă3 moles of magnesium
 Total energy required = 2188.489 ï 10ă3 = 2.188 kJ

Electron Affinity

Electron affinity is the energy change that occurs when an electron is accepted by an
isolated atom in the gaseous state to form an anion.

X(g) + eă  Xă (g)

The sign of electron affinity is opposite to that used for ionization energy. A positive electron
affinity signifies that energy is liberated when an electron is added to an atom. To clarify this
apparent paradox, let us consider the process in which a gaseous fluorine atom accepts an
electron.

F(g) + eă  Fă (g);  H = ă 328 kJ/mole

The sign of the enthalpy change indicates that this is an exothermic process. However, the
electron affinity of fluorine is assigned a value of + 328 kJ/mole. Thus we can think of electron
affinity as the energy that must be supplied to remove an electron from a negative ion. For the
removal of an electron from a fluoride ion, we have

Fă (g) ă
 F (g) + e ;  H = + 328 kJ/mole
Do remember two unique features of electron affinity as
(i) The electron affinity of an element is equal to the enthalpy change that accompanies the
ionization process of its anion and
(ii) a large positive electron affinity means that the negative ion is very stable (i.e. atom has
a great tendency to accept an electron), just as a high ionization energy of an atom means
that the atom is very stable.

Factors Affecting the Magnitude of Electron Affinity

(i) Atomic size : in general electron affinity value decreases with the increasing atomic radius.

1
Electron affinity 
Atomic size

PERIODIC TABLE & PROPERTIES


24 QUIZRR
(ii) Effective nuclear charge : Electron affinity value of the element increase as the effective
nuclear charge on the nucleus increases.
Electron affinity  Effective nuclear charge (Zeff)
(iii) Screening or shielding effect : Electron affinity value of the elements decreases with the
increasing shielding or screening effect. The shielding effect between the outer electrons and
the nucleus increases as the number of electrons increases in the inner shells.

1
Electron affinity 
Shielding effect

(iv) Stability of half filled and completely filled orbitals : The stability of half filled and
completely filled degenerate orbitals of a sub shell is comparatively more, so it is difficult to
add electron in such orbitals and lesser energy is released on addition of electrons hence the
electron affinity value will decrease.

Periodicity in Electron Affinity

(i) In general electron affinity value increases in moving from left to right in a period because
effective nuclear change increases.
(ii) In a group moving from top to bottom the electron affinity value of elements decreases
because the atomic size increases.

Variation of (EA) in a Period

Ć On moving across a period, the atomic size decreases and hence the force of attraction
exerted by the nucleus on the electrons increases. Consequently, the atom has a greater
tendency to attract additional electron, i.e., its electron affinity increases.
Ć Also (EA) values of metals are low while those of non-metals are high.
Ć Halogens have high electron affinities. This is due to their strong tendency to gain an
additional electron to change into the stable ns2 np6 configuration.

Variation of (EA) in a Group

On moving down a group, the atomic size increases and, therefore, the effective nuclear attraction
decreases and thus, electron affinity decreases.
Ć The fact that fluorine has a less negative electron affinity than chlorine seems to
be due to the relatively greater effectiveness of 2p-electrons in the small F atom
to repel the additional electron entering the atom than do 3p-electrons in the
larger Cl atom.
Ć The results have been represented in Fig.

PERIODIC TABLE & PROPERTIES


QUIZRR 25

300
Be Mg
200 Ca

Electron affinity (kJ/mol)


100 Ne Ar
He
N
0 B Al
P
Li Na K
ă100 H
CO Si
ă200 S

ă300
F
Cl
ă400
0 5 10 15 20 25
Atomic number
A plot of electron affinity versus atomic number for the first 20 elements. The general
horizontal trend is that electron affinities become more negative (more energy is
released as an extra electron is added) from group IA (1) through group VIIA (17)
for a given period. Exceptions occur at the IIA (2) and VA (15) elements.

Example 8

Why does nitrogen have a less favourable (more positive) electron-affinity than its neighbours
on either side, C and O ?
Solution :
Electron configurations of these elements are :

C6 : 1s2 2s2 2 p1x 2 p1y 2 p0z

N7 : 1s2 2s2 2 p1x 2 p1y 2 p1z

O8 : 1s2 2s2 2 p2x 2 p1y 2 pz0

Carbon has only two electrons in its 2 p-subshell and can readily accept another in its vacant 2pz-
orbital. Nitrogen, however, has a half-filled 2p-subshell and the additional electron must pair up
in a 2p-orbital, when it feels a repulsion from the electron already present.
Thus the electron affinity of nitrogen is less favourable than that of carbon. Oxygen also must
add an electron to an orbital that already has one electron, but the additional stabilising effect
of increased effective nuclear charge (Zeff), across the periodic table counteracts the effect of
electron repulsion, resulting in a more favourable electron-affinity for O than for N.

PERIODIC TABLE & PROPERTIES


26 QUIZRR
Electronegativity (EN)

Electronegativity is a measure of the tendency of an element to attract electrons to


itself.

X(g) + eă  Xă (g)

In a molecule, tendency of the atom to attract bonding pair towards itself is its electronegativity.
B is said to be more electronegative than A if it pulls bonding pair towards itself.

 
(A)  B    A  ·  B 

Variation of (EN) in a Group

Ć On moving down the group


· Z increases but Zeff. almost remains constant.
· number of shells (n) increases
· rn (atomic radius) increases,

 k Ze2 

· force of attraction  F   between added electron and nucleus decreases,
 rn2 

Therefore, (EN) decreases moving down the group.

Element Z Zeff n rn F (EN)


F 9 max max
Cl 17
Br 35
I 53
constant max max

Variation of (EN) in a Period

Ć While moving across a period left to right


· Z, Zeff. increases,
· n number of shells remains constant,
— rn decreases

 Ze2 

· F  k  increases
 rn2 

PERIODIC TABLE & PROPERTIES


QUIZRR 27

Hence, (EN) increases along a period.

Li Be B C N O F Ne
Z, Zeff
constant
n
max (except for Ne)
rn
max
F
max
EN

Ć Flourine is the most electronegative element in the periodic table.


Fig. represents variation of electronegativity of elements with atomic number. In this :
the strongest non-metals, the halogens, appear at the peaks;
the alkali metals mark the troughs. This is largely a result of new shell of electrons
starting with noble gases which have zero values of electronegativities.

(EN)
F
4.0

3.5 O
Cl
3.0 N Br
S
2.5 C Se
H P As
2.0 B Si
Ga Ge
Al Co Cu
1.5 Be Ni Zn
Mg V Mn Fe
Sc Cr
1.0 Li Na
K CaTi
Rb
0.5
He Ne Ar Ar
0
2 4 6 8 10 12 14 16 18 20 22 24 26 28 30 32 34 36 38

Fig. Electronegativities of the elements H(1) to Rb (37).


The (EN) of the noble gases have been set at zero

Ć the group 1 (IA) metals have their nuclei quite strongly shielded. Hence they show little
tendency to gain new electron.
the electronegativities of transition metals like their (IE), do not change greatly from one
another.

Diagonal Relationship and Electronegativity

Ć Moving one place to the right across period, the increased nuclear charge holds the electrons
more tightly to the atom. Moving down one place in a group, the extra shell of electrons

PERIODIC TABLE & PROPERTIES


28 QUIZRR
decreases the attraction of the nucleus for the outer electrons. A diagonal move means that
these effects tend to compensate for one another.

Ć The electronegativity increases as we go from Li to Be but it decreases as we move from Be


to Mg. As we move diagonally, these two effects partly cancel each other and there is no
marked change in electronegativity and thus Li and Mg have close values of electronegativities.
Because of this, Li and Mg show similarity in properties· a diagonal relationship (between
elements lying diagonally in periodic table).

Ć ElementsÊ pairs showing this relationship are :


Li·Mg, Be·Al, B·Si

(EN) increases
Along a diagonal there is both
(EN) decreases

increase and decrease in (EN)


values, each tends to cancel the
effects of other.

Pauling’s Electronegativity

This scale determines electronegativity of the element X w.r.t. hydrogen H

 x  X   0.208 H x

where  (chi) represents electronegativity, Hăx is the extra bond energy in kcal molă1.

and   x  E x  E H E x  x

EHăx, EHăH and Exăx represent bond energy of H ă X, H ă H and X ă X bonds respectively.
Converting the equation to SI units

 x  H  0.1017  H  x

where Hăx is measured in kJ molă1.


Ć H = 2.05 (and not zero) to avoid any negative value of the element.
Some other elements have following values of electronegativity based on PaulingÊs scale :

PERIODIC TABLE & PROPERTIES


QUIZRR 29

H
2.1
Li Be B C N O F
1.0 1.5 2.0 2.5 3.0 3.5 4.0
Na Cl
0.9 3.0
K Br
0.8 2.8
Rb I
0.8 2.5
Cs
0.7

Ć If two atoms have similar electronegativity, the bond between H and X will be predominantly
covalent.
Ć A large difference in electronegativity leads to a bond with a high degree of polar character,
and bond will be predominantly ionic.
Ć Ionic character of a bond varies with the difference in electronegativity as given in Fig. 9.11.
This is based on the ionic character; HI : 4%, HBr : 11%, HCl : 19% and HF : 45% which
are known from dipole moment measurements.

100

50

0
1 1.7 2
Electronegativity difference

Ć 50% ionic character is shown in the cases when electronegativity difference is about 1.7.
Thus,
(x · H) = 1.7 50% ionic
(x · H) > 1.7 predominantly ionic
(x · H) < 1.7 predominantly covalent

PERIODIC TABLE & PROPERTIES


30 QUIZRR
M ulliken-Jaffe Electronegativity

Electronegativity (EN) can be regarded as the average of the ionisation energy (IE) and the
electron affinity (EA) of an atom.

IE + EA
 EN  
2

If both (EA) and (IE) are determined in eV units then PaulingÊs electronegativity (EN)P is related
to MullikenÊs electronegativity (EN)M by the equation :
(EN)P = 0.336 [(EN)M ă 0.615]
Ć If (EA) and (IE) are in eV, then (EN) in eV is given by
0.187 (IE + EA) + 0.17
Ć MullikenÊs values were about 2.8 times larger than the PaulingÊs values.

Allred-Rochow’s Electronegativity

Allred and Rochow defined electronegativity as the force exerted by the nucleus of an atom on
its valence electrons :

0.359Zeffective
 EN A R   0.744
r2

where Zeffective is the effective nuclear charge and r the covalent radius (in Å).

METALLIC NATURE
Metals have the tendency to form cations by loss of electrons and this property makes the
elements as electropositive elements or metals.

M(g)  M+ (g) + eă

Ć The tendency of an element to lose electron is closely connected to the (IE) of the element.
The smaller the (IE) of an element, the greater will be its tendency to lose electrons
and thus greater will be its metallic character.
Ć Tendency to oxidise itself provides reducing property to the elements thus, smaller the (IE),
greater the metallic character, hence greater the reducing nature : (IE) increases moving
along a period left to right and decreases down the group, hence metallic and reducing
nature decrease along the period and increase down the group.
Nuclea r Tendency to Reducing Meta llic
Cha rge Sheilding ma ke +ve ions power na ture

increases increases But the shielding increases increases increases


effect wins so 

PERIODIC TABLE & PROPERTIES


QUIZRR 31

Nuclear charge increases

Shielding increases

But the nuclear charge wins so 


Tendency to form +ve ions decreases L to R

Reducing power decreases L to R

Metallic nature decreases L to R

Notice that the most reactive metals (the top of the activity series) are on the left of the periodic
table, whereas the least reactive metals (the bottom of the activity series) are in the transition
metal group closer to the right side of the table as shown in Fig. below

More active Less active


metals metals

Redox reactions are common for almost every element in the Periodic table except for the noble
gas element of group VIIIA (18). In general, metals act as reducing agents, and reactive non-
metals such as O2 and the halogens act as oxidizing agents as given in Fig. below

Oxidizing
agents;
tend to
gain eă

Reducing agents;
tend to lose eă

Example 9

The formation of F(g)ă from F(g) is exothermic whereas that of O2ă(g) from O2(g) is endothermic.
Why ?
Solution :
When Fă is formed from F an eă is added to an electronegative, neutral atom and energy [E.A.]
is released by this process but even through Oă from O is energy releasing process, formation of
O2ă from Oă is highly energy consuming. So combined effect is O2ă formation from O is endothermic.

PERIODIC TABLE & PROPERTIES


32 QUIZRR
D iagonal Relationships

A diagonal relationship is a similarity in properties between diagonal neighbours in the main


groups of the periodic table. Because these properties lie in a diagonal pattern, it is not surprising
to find that the elements within a diagonal band show similar chemical properties. Diagonal
relationship helps in making predictions about the properties of elements and their compounds.
Diagonal relationship is exhibited by lithium & magnesium and beryllium & aluminium. For
example, lithium and magnesium react directly with nitrogen to form nitrides. Beryllium and
aluminium, both are amphoteric i.e., reacts with acids as well as bases.

Properties of Oxides

Properties of the representative elements across a period can be compared by examining the
properties of their oxides. Let us see the difference in the behaviour of the oxides of third period
elements. Some elements in the third period (P, S and Cl) form several types of oxides but we will
consider those oxides in which the elements have the highest oxidation number. The oxides of the
elements of third period are Na2O, MgO, Al2O3, SiO2, P4O10, SO3 and Cl2O7.
We know that oxygen has a tendency to form the oxide ion. This tendency is greatly favoured
when oxygen combines with metals having low ionization energies (like I and II A group elements
plus aluminium). Thus, Na2O, MgO and Al2O3 are ionic compounds having extensive threeă
dimensional structures in which each cation is surrounded by a specific number of anions and vice
versa. As the ionization energies of the elements increase from left to right, so does the molecular
structure of the oxides that are formed. Silicon is a metalloid whose oxide (SiO2) has a giant
threeădimensional network, although no ions are present. The oxides of phosphorous, sulfur &
chlorine are molecular compounds composed of small discrete units.
Most oxides can be classified as acidic or basic depending on whether they produce acids or bases
when dissolved in water or react as acids or bases in certain processes. Some oxides are amphoteric,
which mean they display both acidic and basic properties. The first two oxides of the third period
(Na2O & MgO) are basic oxides. For example, Na2O reacts with water to form the base (NaOH).

Na2O + H2O  2NaOH

MgO is quite insoluble in H2O but it reacts with acids showing its basic nature.

MgO + 2HCl  MgCl2 + H2O

Al2O3 is even less soluble in H2O than MgO, it also does not react with water but it shows basic
properties by reacting with acids.

Al2O3 + 6HCl  2AlCl3 + 3H2O

It also exhibits acidic properties by reacting with bases.

Al2O3 + 2NaOH + 3H2O  2NaAl(OH)4

PERIODIC TABLE & PROPERTIES


QUIZRR 33

Thus, Al2O3 is classified as an amphoteric oxide because it has properties of both acids and bases.
Other amphoteric oxides are ZnO, BeO, Bi2O3 etc. SiO2 is insoluble and does not react with water.
It has acidic properties as it reacts with very concentrated bases.

SiO2 + 2NaOH  Na2SiO3 + H2O

For this reason, concentrated aqueous, strong bases such as NaOH should not be stored in
glasswares, which are made of SiO2.
The remaining oxides are acidic. They react with water to form phosphoric acid (H3PO4), sulfuric
acid (H2SO4) and perchloric acid (HClO4).

P4O10 + 6H2O  4H3PO4

SO3 + H2O  H2SO4

Cl2O7 + H2O  2HClO4

Thus the oxides of third period elements shows that as the metallic character of the elements
decreases from left to right across the period, their oxides changes from basic to amphoteric to
acidic. Metallic oxides are usually basic and most oxides of non-metals are acidic. The intermediate
properties of the oxides are exhibited by elements which are placed between metals and non-
metals. As the metallic character of the elements increases from top to bottom in a group of
representative elements, the basic character of oxides also increases from top to bottom. Thus,
Na2O is less basic than K2O, K2O is less basic than Rb2O and so on.

Screening (shielding) Effect

In d-block elements (transition elements), while writing electronic configuration of element, it is


seen that the new electrons are being added to the inner shell i.e. penultimate shell (3d after 4s,
4d after 5s & so on). This affects the nuclear attraction of the positive nucleus for the outer
electrons. As the differentiating electrons enters the inner shells, they tend to screen or shield the
outer shell electrons from the nucleus and hence offset the nuclear attractive force. This is famous
as screening or shielding effect. Due to this effect, the atomic size of transition elements (which
should have been decreasing across a period) remains nearly same or constant. The same is the
case with ionisation energy, electron affinity and other periodic properties when one moves across
a period.

Magnetic Properties

In general, the magnetic properties of matter depends upon the atoms contained in them. The
magnetic behavior of an atom is the direct consequence of its electronic configuration. Any electron
in an atom has two types of motions. One is its motion about the nucleus and other its spin about
its own axis. A single electron spinning about its own axis generates a magnetic field. For two
electrons in an orbital, the spins are opposite and hence the fields cancel each other.

PERIODIC TABLE & PROPERTIES


34 QUIZRR
 Consequently an atom which has a single electron in its orbital generates a magnetic field
while an orbital containing two electrons falls to do so. This type of magnetism is called as
paramagnetism. Any atom or ion or molecule that contains one or more unpaired electrons
will be paramagnetic. The substances that contain such type of atoms are weakly attracted
by the magnets.
Another kind of magnetism is diamagnetism. A diamagnetic substance when placed in an
external magnetic field, is weakly repelled in it. This arises due to orbital motion of electrons
about nucleus. It is a weak effect and is swamped out if the atom is paramagnetic.
 The strongest kind of magnetism is ferromagnetism. When a paramagnetic substance is
placed in a magnetic field, the field will align the unpaired spins & magnetise the substance.
If the substance keeps its magnetism after the field is removed, it is called as ferromagnetism.
It is much stronger than paramagnetism. The most important example of such types is iron
& others are cobalt & nickel.

Hydration And Hydration Energy

Hydra tion energy is the entha lpy cha nge tha t a ccompa nies when one mole of the ga seous
ion is dissolved in wa ter. For example, reaction of the hydration of Li ion is shown as :

Li+ (g) + nH2O  [Li(H2O)n]+(aq); H = ă 806 kJ molă1

Extent of hydration (means the number of water molecules surrounding a given ion) depends on
the size and charge of the ion. If the size of the ion is small and its charge is greater, then there
would be greater attraction for water molecules, hence greater the extent of hydration and greater
the hydration energy. The total hydration energy of a salt (M+ Xă) is related as

 1   1 
   
Total hydration energy  r  r  
 M   X 

The first term in this expression defines the hydration energy due to cation and second term
defines the hydration energy due to anion.
Let us take LI+, Na+, K+, Rb+ and Cs+ ions for comparison. The charge on all the ions is same
but the size increases from Li+ to Cs+. Thus the positive charge density increases from Li+ to Cs+
and hence Li+ ion would attract H2O molecules more strongly than Na+ and so on. Therefore,
hydration energy increases from Li+ to Cs+ ion while the size of the hydrated ion decreases from
Li+ ion to Cs+ ion and thus the ionic mobility decreases from Li+ to Cs+ ion.
In a period, on moving from left to right the hydration energy increases. You can very well see
this trend in the group of ions Na+, Mg2+ and Al3+.

PERIODIC TABLE & PROPERTIES


QUIZRR 35

H ydrides

Hydrogen combines with a number of other elements (meta ls a nd non-meta ls) to form
compounds ca lled hydrides. The hydrides can be either ionic or covalent. The ionic hydrides
are formed by the elements of I A & II A group while other elements mostly form covalent
hydrides.
Covalent nature of hydrides increases across a period and decreases down the group while ionic
nature of hydrides decreases across a period and increases down the group.
Ionic hydrides are better reducing agent than covalent hydrides and reducing nature of hydrides
decreases across a period and increases down the group.

Example 10

Arrange the following compounds in the order of their decreasing stability if the electro
negative values of elements are as follows :
H = 2.1 F = 4, Cl = 3.0, Br = 2.8
I = 2.3, N = 3.0
HF, NCl3, HBr, HI, HCl
Solution :
Electronegativity difference for
HF = 4.0 ă 2.1 = 1.9
NCl3 = 3.0 ă 3.0 = 0.0
HBr = 2.8 ă 2.1 = 0.7
HI = 2.3 ă 2.1 = 0.2
HCl = 3.0 ă 2.1 = 0.9
So their order of their decreasing stability
HF > HCl > HBr > HI > NCl3
1.9 0.9 0.7 0.4 0.0

Example 11

Why are inert gases mono-atomic ?


Solution :
Inert gas have completely octet (ns2, np6) and so do not form bonds.

PERIODIC TABLE & PROPERTIES


36 QUIZRR
Some Other periodic Properties

(i) Atomic Volume : Atomic volume increases in a group from top to bottom. The increase is
due to the increases in the number of energy shells.
In period from left to right, atomic volume varies cyclically, i.e., it decreases at first for some
elements, becomes minimum in the middle and then increases.
(ii) Density : The density of the elements in solid state varies periodically with their atomic
numbers. At first, the density increases gradually in a period and becomes maximum
somewhere for the central members and then start decreasing afterwards gradually.

Example 12

The sum of first and second ionization energies and those of third and fourth ionization
energies in (kJ molă1) of nickel and platinum are
(IE)1 + (IE)2 (IE)3 + (IE)4
Ni 2.49 8.80
Pt 2.66 6.70
Based on this information; write
(i) the most common oxidation states of Ni and Pt
(ii) Name of metal (Ni or Pt) which can more easily form compounds in its +4 oxidation
state.
Solution :
(i) Ni = +2, Pt = +4 since (IE)1 + (IE)2 of Ni is less than its (IE)3 + (IE)4 and reverse is the case
in Pt.
(ii) Platinum forms more stable complexes in +4 state due to its higher stability than +2 state.

Example 13

In alkali metal group which is the strongest reducing agent in aqueous solution and why ?
Solution :
Li is the strongest reducing agent. Since I.P. decreases down the group we would expect that Li
will have the lowest reducing power in that group. But since itÊs hydration energy is very high
and which in fact decreases down the group, Li will have highest reducing power.

PERIODIC TABLE & PROPERTIES


QUIZRR 37

Example 14

The first ionization energy of carbon atom is greater than that of boron atom, whereas
reverse is true for the second ionization energy. Explain.
Solution :
The electronic configurations of carbon and boron are as follows :

C : 1s2, 2s2 2 p1x 2 p1y

1
B : 1s2, 2s2 2 px

Due to higher nuclear charge in carbon, the force of attraction towards valency electron is more
in carbon atom and hence the first ionization energy is greater than boron atom. After loss of one
electron, the mono-valent cations have the configurations as follows :
B+ : 1s2, 2s2
1
C+ : 1s2, 2s2 2 px

The B+ configuration is stable one and hence the removal of electron is difficult in comparison to
C+. Hence, second ionization potential of boron is higher than carbon.

Example 15

Calculate the electronegativity of fluorine from the following data :


EHăH = 104.2 kcal molă1, EFăF = 36.6 kcal molă1
EHăF = 134.6 kcal molă1, XH = 2.1
Solution :
Let the electronegativity of fluorine beXF.
Applying PaulingÊs equation
XF ă XH = 0.208[EHăF ă (EFăF ï EHăH)ó] ó
In this equation, dissociation energies are taken in kcal molă1.
XF ă 2.1 = 0.208 [134.6 ă (104.2 ï 36.6)ó] ó
XF = 3.87

PERIODIC TABLE & PROPERTIES


QUIZRR 1

ATOMIC STRUCTURE

1 . A T OM I C ST RU CT U RE
An atom is the smallest building block of matter. Atoms are made of neutrons, protons and
electrons. The nucleus of an atom is extremely small in comparison to the atom.
Scientists use the Periodic Table in order to find out important information about various elements.
Created by Dmitri Mendeleev (1834-1907), the periodic table orders all known elements in accordance
to their similarities. When Mendeleev began grouping elements, he noticed the Law of Chemical
Periodicity. This law states, „the properties of the elements are periodic functions of atomic number.‰
The periodic table is a chart that categorizes elements by „groups‰ and „periods.‰ All elements are
ordered by their atomic number. The a tomic number is the number of protons per a tom. In a
neutral atom, the number of electrons equals the number of protons. The periodic table represents
neutral atoms. The atomic number is typically located above the element symbol. Beneath the element
symbol is the atomic mass. Atomic mass is measured in Atomic Mass Units where 1 amu = (1/12) mass
of carbon measured in grams. The a tomic ma ss number is equa l to the number of protons plus
neutrons, which provides the average weight of all isotopes of any given element. This number is
typically found beneath the element symbol. Atoms with the same a tomic number, but different
ma ss numbers a re ca lled isotopes. Below is a diagram of a typical cells on the periodic table.

Atomic Number
A

Element Symbol

Atomic Mass Z

1.1 Da l t o n ’s A t o m i c T h e o r y
John Dalton ERS (6 September 1766 ă 27 July 1844) was an English chemist meterologist and
physicist. He is best known for his pioneering work in the development of modern atomic theory, and
his research into colour blindness (sometimes referred to as Daltonism, in his honour).
Five main points of DaltonÊs Atomic Theory
Ć Elements are made of tiny particles called atoms.
Ć All atoms of a given element are identical.
Ć The atoms of a given element are different from those of any other element, the atoms of
different elements can be distinguished from one another by their respective relative weights.
Ć Atoms of one element can combine with atoms of other elements to form chemical compounds;
a given compound always has the same relative numbers of types of atoms.

ATOMIC STRUCTURE
2 QUIZRR

Ć Atoms cannot be created, divided into smaller particles, nor destroyed in the chemical process;
a chemical reaction simply changes the way atoms are grouped together.

1 .1 .1 Dr a w b a c k s o f Da l t o n ’s a t o m i c t h e o r y
Ć Ability of an atom was proved wrong, for, an atom can be further subdivided into protons,
neutrons and electrons. However an atom is the smallest particle, which takes part in chemical
ability.
Ć According to Dalton, the atoms of same element are similar in all respects. This is wrong
because atoms of some elements vary in their mass and density. Such atoms of the same
element having are called isotopes. For example, chlorine has two isotopes having mass numbers
35 and 37 a.m.u.
Ć Dalton also said atoms of different elements are different in all respects. This has been proved
wrong in certain cases like Argon and Calcium atoms, which have the same atomic mass of 40.
Such elements that have the same atomic mass are called isobar.
Ć According to Dalton, atoms of different elements combine in simple whole number ratio to form
compounds. This is not seen in complex organic compounds like sugar C12H22O13.
Ć The theory completely fails to explain the existence of allotropes. The difference in properties
of charcoal, graphite, diamond went unexplained in spite of being made up of same kind of
atoms.

1 .1 .2 M e ri t s o f Da l t o n ’s a t o m i c t h e o r y
Despite the uncertainty at the heart of DaltonÊs atomic theory, the principles of the theory survived.
To be sure, the conviction that atoms cannot be subdivided, created, or destroyed into smaller particles when
they are combined, separated or rearranged in chemical reactions is inconsistent with the existence of
nuclear fusion and nuclear fission, but such processes are nuclear reactions and not chemical reactions. In
addition, the idea that all atoms of a given element are identical in their physical and chemical properties
is not precisely true, as we know that different isotopes of an element have slightly varying weights.
Ć It has enabled us to explain the laws of chemical combination.
Ć Dalton was the first person to recognize a workable distinction between the ultimate particle
of an element (atom) and that of a compound (molecule).

1 .2 T h o m s o n ’s Ca t h o d e Ra y Ex p e r im e n t
Even without consciously realizing it, most of us are already aware of what a cathode ray tube is.
Looking at any glowing neon sign or any Âold fashionedÊ television set, and you are looking at the modern
descendents of the cathode ray tube.
Physicists in the 19th century found out that if they constructed a glass tube with wires inserted
in both ends, and pumped out as much of the air as they could, an electric charge passed across the tube
from the wires would create a fluorescent glow. This cathode ray also became known as an Âelectron gunÊ.
Later and improved cathode ray experiments found that certain types of glass produced a fluorescent
glow at the positive end of the tube. William Crookes discovered that a tube coated in a fluorescing
material at the positive end, would produce a focused ÂdotÊ when rays from the electron gun hit it.
With more experimentation, researchers found that the Âcathode raysÊ emitted from the cathode
could not move around solid objects and so traveled in straight lines, a property of waves. However,
ATOMIC STRUCTURE
QUIZRR 3

other researchers, notably Crookes, argued that the focused nature of the beam meant that they had
to be particles.
Physicists knew that the ray carried a negative charge but were not sure whether the charge could
be separated from the ray. They debated whether the rays were waves or particles, as they seemed to
exhibit some of the properties of both. In response, JJ Thomson constructed some elegant experiments
to find a definitive and comprehensive answer about the nature of cathode rays.

1 .2 .1 T h o m s o n ’s Fi r st Ca t h o d e Ra y Ex p e r im e n t
Ć Thomson had an inkling that the ÂraysÊ emitted from the electron gun were inseparable from
the latent charge, and decided to try and prove this by using a magnetic field.
Ć His first experiment was to build a cathode ray tube with a metal cylinder on the end. This
cylinder had two slits in it, leading to electrometers, which could measure small electric charges.
Ć He found that by applying a magnetic field across the tube, there was no activity recorded by
the electrometers and so the charge had been bent away by the magnet. This proved that the
negative charge and the ray were inseparable and interwined.

1 .2 .2 T h o m s o n ’s Ca t h o d e Ra y Se c o n d Ex p e r im e n t
Ć Like all great scientists, he did not stop there, and developed the second stage of the experiment,
to prove that the rays carried a negative charge. To prove this hypothesis, he attempted to
deflect them with an electric field.
Ć Earlier experiments has failed to back this up, but Thomson thought that the vacuum in the
tube was not good enough, and found ways to improve greatly the quality.
Ć For this, he constructed a slightly different cathode ray tube, with a fluorescent coating at one
end and a near perfect vacuum. Halfway down the tube were two electric plates, producing a
positive anode and a negative cathode, which he hoped would deflect the rays.
Ć As he expected, the rays were deflected by the electric charge, proving beyond doubt that the
rays were made up of charged particles carrying a negative charge. This result was a major
discovery in itself, but Thomson resolved to understand more about the nature of these particles.

1 .2 .3 T h o m s o n ’s T h i rd Ex p e r im e n t
Ć The third experiment was a brilliant piece of scientific deduction and shows how a series of
experiments can gradually uncover truths. Many great scientific discoveries involve performing
a series of interconnected experiments, gradually accumulating data and proving a hypothesis.
Ć He decided to try to work out the nature of the particles. They were too small to have their
mass or charge calculated directly, but he attempted to deduce this from how much the particles
were bent by electrical currents, of varying strengths.
Ć He found out that the mass to charge ratio was so high that the particles either carried a huge
charge, or were a thousand time smaller than a hydrogen ion. He decided upon the later and
came up with the idea that the cathode rays were made of particles that emanated from within
the atoms themselves, a very bold and innovative idea.
Ć Julius Plucker and his student Johann Hittorf were experimenting with a Geissler tube. Plucker
and Hittorf placed two electrodes at each end of the tube.

ATOMIC STRUCTURE
4 QUIZRR

Ć The positive plate was called the anode


Ć The negative plate was called the cathode
Ć Pumped all the air from the tube, creating a vacuum
Ć Connected wires from the electrodes to a battery

ANODE CATHODE
(+) (ă)

AMMETER

Plucker and Hittorf discovered that when electricity flowed through the Geissler
tube, a mysterious greenish glow was produced around the tube.
Sir William Crookes gained more knowledge about the mysterious green glow (ă)
when he created a bent Geissler tube in 1875. He noticed that the glow was the (+)
most intense opposite to the negative electrode, also called the cathode. Crookes
Bent Geisster Tube
reasoned that rays traveled from the cathode and then hit the end of the tube.
Because of this, Crookes named these rays cathode rays.
In later experiments Crookes placed barriers in the path of cathode rays :

(+)
Crookes Tube with Maltese Cross

Because a shadow was produced behind the barrier, Crookes reasoned that cathode rays.
Ć Acted like light
Ć Seemed to travel in straight lines
Next, Crookes was able to change the path of the rays with a magnet. This meant
Ć The cathode rays were NOT light (light is not affected my magnets or magnetic fields)
Ć Cathode rays had some sort of charge.

A f t e r f u r t h e r e x p e r im e n t s, Cr o o k e s p r o v e d :
(1) All cathode rays have identical properties-the material the cathode is made of does not matter.
(2) Cathode rays normally travel in straight lines perpendicular to the cathode.
(3) Magnetic fields change the path of the cathode rays. (Crookes suspected that any charged
object, not just magnets, could change the path of cathode rays.)
(4) Rays caused reactions similar to those caused by light.
(5) Scientists could not decide if cathode rays were electromagnetic waves or negatively charged
particles.

ATOMIC STRUCTURE
QUIZRR 5

1 .3 .1 Plu m -Pu d d i n g M o d e l
The Plum pudding model of the atom by JJ Thomson, who discovered
the electron in 1897, was proposed in 1904 before the discovery of the
atomic nucleus. In this model, the atom is compared of electrons (which
Thomson still called „corpuscles,‰ though G.J. Stoney had proposed that
atoms of electricity be called electrons in 1894), surrounded by a soup of
positive charge to balance the electronÊs negative charge, like negatively-
charged „plums‰ surrounded by positively-charged „pudding‰. The electrons
(as we know then today) were thought to be positioned throughout the
atom, but with many structures possible for positioning multiple electrons.
A schematic representation of the plum pudding model of the atom. In ThomsonÊs mathematical
model the „corpuscles‰ (or modern electrons) were arranged non-randomly, in rotating rings.

1 .3 .2 Ru t h e r f o r d ’s M o d e l
In the year 1898 Thomson created the idea of atom as the positive charged ball in which there are
negative charges placed-the „plum cake‰ model. So summing up the whole atom were to be neutral.
In the years 1909-1911 Ernest Rutherford and his students-Hans Geiger (1882-1945) and Ernest
Marsden conducted some experiment to search the problem of alpha particles scattering by the thin
gold-leaf. Rutherford knew that the particles contain the 2e charge. The experiment caused the creation
of the new model of atom-the “planetary” model. Rutherford suggested to hit the gold-leaf with fast alpha
particles from the source 214Po. (The source R was in the lead lining F). The particles felt from the

The incident
alpha particles

The structure of the gold foil acording to


the Rutherford's atomic model.

The alpha particles propagated on the atomic nucleuses of the gold foil

ATOMIC STRUCTURE
6 QUIZRR

source on the gold-leaf E and were observed by the microscope M. The whole experiment was in the
metal lining A and was covered with the glass plate P. The instrument was attached to the footing B.
The gold leaf was about 5*10ă7 meter thick. The scientist knew that reckoning the scattering angle could
say much about the structure of atoms of the gold-leaf.
Rutherford made a theoretical analysis of angles of scattering in accordance with ThomsonÊs
theory of atom in accordance with his own theory. He assumed that atom consisted of positive charged
nucleus and negative charged electrons circling around the nucleus. Then his theoretic calculations he
compared with the experiment result. Alpha particles going through atom created in accordance with the
„plum cake‰ model wouldnÊt be strong abberated because the electric field in that atom wouldnÊt be
strong. In the model created by Rutherford the field is much stronger near to the nucleus, so some of alpha
particles are much more abberated. The other going in the far distance to the nucleus are almost at all
abberated. The probability that any alpha particle will hit the nucleus is small but there is such a chance.
P

R
E
F
M

Fig. 1. Marsden-Geiger experiment

The alpha particle

The alpha particle


ă ă
ă

ă
ă ă
ă
ă
ă

Thomson's model of atom


Rutherford's model of atom

The models of the ThomsonÊs atom and RurtherfordÊs atom; and the expected aberration of alpha particle in both cases

ATOMIC STRUCTURE
QUIZRR 7

The experiment showed that there are some not much abberated alpha particles but also some
abberated of a very big angle (135-150 degree). That occurrence couldnÊt be explained by some small,
added aberrations. Experiment data proved the „planetary‰ model of atom.
Between an alpha particle and an atomic nucleus there subsist an interaction - the repulsing -
according to Coulomb force :
1 2.Z e2
F . (1)
4.  . 0 r2
where (2*e)-alpha particle charged, (Ze)-atomic nucleus charged, 0 -permittivity of free space,
r-distance between the nucleus and the particle.

1 .3 .2 .1 Co n c l u s i o n s
Summarising the results of his work Rutherford came to the opinion that atom consisted of a
nucleus (of size 10ă15 to 10ă14 meter). The nucleus contains the whole positive charge and almost the
whole atomÊs mass. Around the nucleus, on the area of the size of the order of 10ă10 meter, light
electrons are circling. Electrons have to circle around the nucleus on orbits, not to fell down on the
nucleus. The orbits depend on electrons energy. In atoms of the same element, electrons circle on the
same characteristic, for that element, orbits i.e. the optical spectrum of atoms of the same element is
the same.

The proton (the nucleus)

The electron
v

The Rutherford's atomic model. The electron circulating on the orbit


around the nucleus with the velocity v is attracted by it with the force F
1 .3 .2 .2 Dr a w b a c k s
The model created by Rutherford had still some serious discordance. According to the classic
science, electron moving around the nucleus should emit an electromagnetic wave. That kind of emission
is connected with the escape of some energy from the electron-ion circuit. Electron should then move
not by the circle but helical and finally collide with the nucleus. But atom is stable. Other discordance
regarded the radiation-it were to be constant (because the time of electronÊs cycle in accordance with
the lost of energy should change constantly) and spectral lines shouldnÊt occur.

v
The nucleus
The electron

In the planetary model of atom, the electron should


emit energy and spirally fall on the nucleus.
ATOMIC STRUCTURE
8 QUIZRR

1 .3 .3 B o h r ’s M o d e l o f a n A t o m
RutherfordÊs model had a major drawback. It could not explain why ultimately electrons did not
fall into the nucleus by taking a spiral path. This was in concurrence with the electromagnetic theory
that states Âif a charged particle undergoes accelerated motion then it must radiate energy
(lose) continuouslyÊ.
In order to explain the stability of an atom Neils Bohr in 1913 gave a new arrangement of electrons
in the atom. According to Neils Bohr, the electrons could revolve around the nucleus in only
Âcertain orbitsÊ (energy levels), each orbit having a different radius. When an electron is revolving
in a particular orbit or particular energy level around the nucleus, the electron does not radiate energy
(lose energy) even though it has accelerated motion around the nucleus.
Arrangement of energy levels around the nucleus
4 N Shell
3 M Shell
2 L Shell
1 K Shell
+ Positively charged
Nucleus
K
L
M
N
Arrangement of energy levels around the nucleus

1 .3 .3 .1 Co n c e p t i n v o l v e d
An atom is made up of three particles, electrons, protons and neutrons. Electrons have a negative
charge and protons have a positive charge whereas neutrons have no charge. They are neutral. Due to
the presence of equal number of negative electrons and positive protons the atom as a whole is electrically
neutral.
Ć The protons and neutrons are located in a small nucleus at the centre of the atom. Due to the
presence of protons the nucleus is positively charged.
Ć The electrons revolve around the nucleus in fixed circular paths called energy levels or shells.
The Âenergy levelsÊ or ÂshellsÊ or ÂorbitsÊ are represented in two ways : either by the numbers
1, 2, 3, 4, 5 and 6 or by letters K, L, M, N, O and P. The energy levels are counted from centre
outwards.
Ć Each energy level is associated with a fixed amount of energy. The shell nearest to the nucleus
having minimum energy and the shell farthest from the nucleus having maximum energy.
Ć There is no change in the energy of electrons s long as they keep revolving in the same energy
level. But when an electron jumps from a lower level to a higher one, some energy is absorbed
while some energy is emitted. When an electron jumps from a higher energy level to a lower
one, the amount of energy absorbed or emitted is given by the difference of energies associated
with the two levels. Thus if an electron jumps from orbit 1 (energy E1) to orbit 2 (energy E2)
the change in energy is given by E2 ă E1. The energy change is accompanied by absorption of
radiation energy of E = E2 ă E1 = h  where h is a constant called ÂPlanckÊs constantÊ and 

ATOMIC STRUCTURE
QUIZRR 9

is the frequency of radiation absorbed or emitted. The value of h is 6.626 ï 10ă34 J-s. The
absorption and emission of light due to electron jumps are measured by use of spectrometers.
This model of the atom was able to explain the stability of the atom. It also explained the phenomenon
of atomic spectra and ionization of gases.
Combining of energy of the classical electron orbit with the quantization of angular momentum, the
Bohr approach yields expressions for the electrons orbit radii and energies.

mv2  mv2 n2 h2 Ze2


  
2 2mr 2 8 2 mr 2 80 r
This is for hydrogenic atoms; the
kinetic expressed Use quantization Set equal to use of the atomic number Z is
energy of in terms of of angular total energy appropriate only if there is only
electron angular momentum of classical
momentum orbit
one electron.

Substitution for r gives the Bohr energies and radii :

Z2 me4  13.6Z2 n2 h2  0 n2 a0
E  eV & rn  
8n2 h202 n 2
zxme2 z

a0 = 0.0529 nm = Bohr radius

1 .3 .3 .3 A n g u l a r M o m e n t u m Qu a n t iza t io n
In the Bohr model, the wavelength associated with the electron is given by the DeBroglie relationship.

h

mv
and the standing wave condition that circumference = whole number of wavelengths. In the hydrogenic
case, the number n is the principal quantum number.
2r = nn
These can be combined to get an expression for the angular momentum of the electron in orbit. (Note
that this assumes a circular orbit, a generally unwarranted assumption.)

hr hr nh
L  mvr   
  2r  2 (used in Bohr orbit)
 n 

Thus L is not only conserved, but constrained to discrete values by the quantum number n. This
quantization of angular momentum is a crucial result and can be used in determining the Bohr orbit
radii and Bohr energies.
The velocity of an electron in the nth state Vn is :

2KZe2 z
Vn   2.165  106 m/s
nh n

ATOMIC STRUCTURE
10 QUIZRR

1 .3 .3 .3 En e r g y Co n s i d e r a t i o n s
Classical Electron Orbit
NewtonÊs Second Law Kinetic Energy T
e
ă
mv2 Ze2 mv2 Ze2
= r  So T 
F 4  0 r 2 2 80 r
centripetal
+ Ze Potential Energy Total Energy

 Ze2  Ze2
U TU
4 0 r 80 r

The orbit energy is negative This is the energy of a single electron


because this is a bound state. in orbit around a bare nucleus.

Note : The total energy of the electron is negative because of attraction of electron by the nucleus.
In the Bohr theory, this classical result was combined with the quantization of angular momentum
to get an expression for quantized energy levels.
Substituting the value of r and x, we get the following reaction.

 22K 2 e4 mZ2 Z2
En    2.178  1018 J/atom
n2 h2 n2

Z2
Also En  13.6
n 2 
eV 1eV =1.6 10 19 J 
Note : The value of En in terms of eV is more important.

1 .3 .3 .4 Ele c t ro n T ra n s i t i o n s
The Bohr model for an electron transition in hydrogen between quantized energy levels with
different quantum numbers n yields a photon by emission with quantum energy.
E2 Ephoton = hv = E2 ă E1
n2 A downward transition involves emission of a
photon of energy
n1 E1

Given the expression for the energies of the hydrogen electron states :
1
22 me2 k21  
2 1 1 
hv        13.6Z    eV
h2 2
 n1 n22  2
 n1 n22 
This is often expressed in terms of the inverse wavelength or „wave number‰ as follows :

1  1 1  22 me4 k2
= R H  2  2  where R H  is called the Rydberg constant.
  n1 n 2  C h3
RH = 1.0973731 ï 107 mă1.
ATOMIC STRUCTURE
QUIZRR 11

Ex a m p l e 1
Determine the frequency of revolution of the electron in 2nd BohrÊs orbit in hydrogen atom.
Solution :
1
The frequency of revolution of electron is given by : frequency =
time period

total distance covered in 1 revolution 2r


Time period = 
velocity v
v
Hence frequency =
2r
First we will have to calculate velocity (v) and radius (r) of electron in second BohrÊs orbit by using
the standard relations for v and r. (Z = 1 for H-atom)

n2 h2
rn   (0.53  1010 n2 ) m
2 2
4  me K
Now, n = 2
r2 = 0.53 ï 10ă10 (2)2m = 2.12 ï 10ă10 m

2Ke2 2.165  106


vn = = m/s
nh n

2.165  106
v2 = = 1.082 ï 106
n
v2 1.082  106
Hence frequency = 
2r2 2()(2.12  10 10 )
 = 8.13 ï 1014 Hz.

Ex a m p l e 2
The Velocity of electron in a certain BohrÊs orbit of H atom bears the ratio 1:275 to the
Velocity of Light :
(a) What is the quantum number (n) of the orbit ?
(b) Calculate the wave number of radiations emitted when electron jumps from (n + 1)
state to ground state.
Solution :

1
Velocity of electron =  velocity of light
275

1
=  3 ï 1010 = 1.09 ï 108 cm secă1
275
2e2
Since, un =
nh
ATOMIC STRUCTURE
12 QUIZRR

2  3.14  (4.803  10 10 )2


 1.09 ï 108 =
6.625  10 27  n

n = 20.06 ï 10ă1  2
Also when electron jumps from (n + 1) i.e. 3, to ground state

v 1 1 1 
=  RH   
(wave number)  12
32 

1 1 
= 109678   
1 9 

= 9.75 ï 104 cmă1

1 .3 .3 .5 En e r g y L e v e l s o f H y d ro g e n A t o m
The spectrum of H-atom studied by Lyman, Balmer, Paschen, Brackett and Pfund can now be
explained on the basis of BohrÊs Model.
It is now clear that when an electron jumps from a higher energy state to a lower energy state,
the radiation is emitted in form of photons. The radiation emitted in such a transition corresponds to
the spectral line in the atomic spectra of H-atom.

Spectral Lines & Energy Levels of H-atom

n =6
n =5
p fund n =4
Brackett n =3
Paschen n =2
Balmer

Lyman
n=1

L y m a n Se r ie s
When an electron jumps from any of the higher state to the ground state or Ist state (n = 1), the
series of spectral lines emitted lies in ultra-violet region and are called as Lyman Series. The
wavelength (or wave number) of any line of the series can be given by using the relation :

 1 1 
v  RZ2  2  2  n2 = 2, 3, 4, 5, ...
1 n 
 2 

Note : For H-atom, Z = 1, He+ ion, Z = 2 & Li2+, Z = 3

ATOMIC STRUCTURE
QUIZRR 13

B a l m e r Se r ie s
When an electron jumps from any of the higher states to the state with n = 2 (IInd state), the series
of spectral lines emitted lies in visible region and are called as Balmer Series. The wave number
of any spectral line can be given by using the relation :

 1 1 
v  RZ2  2  2  n2 = 3, 4, 5, ...
2 n 
 2 

Pa s c h e n Se r ie s
When an electron jumps from any of the higher states to the state with n = 3 (IIIrd state), the
series of spectral lines emitted lies in near infra-red region and are called as Paschen Series. The
wave number of any spectral line can be given by using the relation :

 1 1 
v  RZ2  2  2  n2 = 4, 5, 6 ...
3 n 
 2 

B r a c k e t t Se r ie s
When an electron jumps from any of the higher states to the state with n = 4 (IVth state), the
series of spectral lines emitted lies in near in far infra-red region and are called as Brackett
Series. The wave number of any spectral line can be given by using the relation :

 1 1 
v  RZ2  2  2  n2 = 5, 6, 7 ...
4 n2 

Pf u n d Se r ie s
When an elelctron jumps from any of the higher states to the state with n = 5 (Vth state), the series
of spectral lines emitted lies in far infra-red region and are called as P fund Series. The wave
number of any spectral line can be given by using the relation :

 1 1 
v  RZ2  2  2  n2 = 6, 7 ...
5 n 
 2 

Ex a m p l e 3
Estimate the wavelength of second line in Balmer series.
Solution :
The transition responsible for second Balmer line is
4  2, so n1 = 2, n2 = 4

 1 1 

E = 13.6 Z2  2   2.55 eV
 n1  n2  2

ATOMIC STRUCTURE
14 QUIZRR

hc 6.63  10 34  3  108


Now  
E 2.55  1.6  1019
  = 4.862 ï 10ă7 m = 4862 Å
Important Note : In any series,
first line = transition from (n1 + 1)  n1; also called as -line & so on.
For example : In Lyman series -line = 2  1; -line = 3  1; -line = 4  1

Ex a m p l e 4
Calculate the shortest and longest warelength in H speetrum of lyman series RH = 109678 cmă1
Solution :
For lyman series n1 = 1
For shortest of lyman series; energy difference in two levels showing transition should be maximum,
i.e., nz = 

1  
= RH  1  1 

  2 
1
= 109678

  = 911.7 ï 10ă8 cm = 911.7AĈ
For longest of lyman series; energy difference in two levels showing transition should be minimum,
i.e., n2 = 2

1  
= RH  1  1   109678  3
 2 2 4
1 2 
  = 1215.67 ï 10ă8 cm = 1215.67 AĈ

1 .3 .3 .6 H y d ro g e n En e r g y L e v e l s
The basic hydrogen energy level structure is in agreement with the Bohr model. Common pictures
are those of a shell structure with each main shell associated with a value of the principal quantum
number n.
Electron energy Electron
Orbit Radius
ă13.6eV Wavelength Ionization
E=
n2 r = n 2 a0 = 2na0 n=5 energy
n=4
n=3 ă1.5 eV
n=2
n=2 ă3.4 eV
n=3 n=4 n=5
n=1

a0 = 0.0529nm = Bohr radius


n=1 ă13.6 eV

ATOMIC STRUCTURE
QUIZRR 15

This Bohr model picture of the orbits has some usefulness for visualization so long as it is realized
that the „orbits‰ and the „orbit radius‰ just represent the most probable values of a considerable range
of values. If the radial probabilities for the states are used to make sure you understand the distributions
of the probability, then the Bohr picture can be superimposed on that as a kind of conceptual skeleton.
H y d ro g e n En e r g y L e v e l Plo t
The basic structure of the hydrogen energy levels can be calculated from the Schrodinger equation.
The energy levels agree with the earlier Bohr model, and agree with experiment within a small fraction
of an electron volt. The significance of the zero in
energy is that the electron is -.278 n=7
free from the hydrogen nucleus. -.378 n=6
Electron is free -.544 n=5
0 n=4 5 6 7 -.850 n=4
n=3 -1.511 n=3
-2.0
n=2 -3.4 n=2
-4.0
Binding energy (eV)

-6.0 Scaled plot of hydrogen levels electron volts.

-8.0 ă13.6eV The levels get closer together


E=
n2 as they approach the lonization
-10.0 energy.

-12.0 Electron is bound in atom


n=1 Ground state of the hydrogen electron
-14.0 -13.6 n=1

HYDROGEN SPECTRUM

Lyman Series Balmer Series


(Ultraviolet) (Visible)
410.2nm
violet

n=2
434.1 nm violet
n=4 n=5
n=1
n=3
656.3 nm
red
Pachen Series
(infrared)
486.1 nm
bluegreen

ATOMIC STRUCTURE
16 QUIZRR

This spectrum was produced by exciting a glass tube of hydrogen gas with about 5000 volts from
a transformer. It was viewed through a diffraction grating with 600 lines/mm.

Qu a n t ize d En e r g y St a t e s
The electrons in free atoms can be found in only certain discrete energy states. These sharp energy
states are associated with the orbits or shells of electrons in an atom, e.g., a hydrogen atom. One of the
implications of these quantized energy states is that only certain photon energies are allowed when
electrons jump down from higher levels to lower levels, producing the hydrogen spectrum. The Bohr
model successfully predicted the energies for the hydrogen atom, but had significant failures that were
corrected by solving the Schrodinger equation for the hydrogen atom.

n=4 Energy n=4


n=3
Levels
n=2 of n=3
Hydrogen
n=2
Violet

Red Bluegreen
n=1
Visible light transitions
of the hydrogen atom
shell or
"orbits"
of electron

1 .3 .3 .7 Fa i lu r e s o f t h e B o h r M o d e l
While the Bohr model was a major step toward understanding the quantum theory of the atom,
it is not in fact a correct description of the nature of electron orbits. Some of the shortcomings of the
model are :
1. It fails to provide any understanding of why certain spectral lines are brighter than others.
There is no mechanism for the calculation of transition probabilities.
2. The Bohr model treats the electron as if it were a miniature planet, with definite radius and
momentum. This is an direct violation of the uncertainty principle which dictates that position
and momentum cannot be simultaneously determined.
The Bohr model gives us a basic conceptual model of electrons orbits and energies. The precise
details of spectra and charge distribution must be left to quantum mechanical calculations, as with the
Schrodinger equation.

ATOMIC STRUCTURE
QUIZRR 17

Ex a m p l e 5
Calculate the energy emitted when electrons of 1.0 gm atom of Hydrogen undergo transition
giving the spectral lines of lowest energy in visible region of its atomic spectra. Given that,
RH = 1.1 ï 107 mă1, c = 3 ï 108 m/sec, h = 6.625 ï 10ă34 J sec.
Solution :
Visible region of H-spectrum correspond to Balmer series n1 = 2, n2 = 3 (for minimum energy transition),

1 1 
1 /   RH  2  2 
 n1 n2 

1 1 1
  RH  2  2 
 2 3 

7
1  1 1  1.1 10  5
  1.1 107    
 4 9  49

 = 6.55 ï 10ă7

hc 6.625  1034  3  108


E 
 6.55 107

= 3.03 ï 10ă19 joule / molecule


 Energy released by 1 gm atom of H (1 mol)
= En = 3.03 ï 10ă19 ï 6.023 ï 1023 = 18.25 ï 104 J = 182.5 kJ

Ex a m p l e 6
A doubly ionized Lithium atom is hydrogen like with atomic number 3.
(i) Find the wavelength of radiation required to excite the electron in Li++ from the first
to the third Bohr Orbit. (Ionization energy of the hydrogen atom equal 13.6 eV).
(ii) How many spectral lines are observed in the emission spectrum of the above excited
system?
Solution :

 13.6 Z2
(i) En  , Z = 3, n2 = 3, n1 = 1
n2

1 1
Excitation energy, E = E3 ă E1 = ă 13.6 ï 32  2  2 
3 1 
= + 13.6 ï 9 [1 ă 1/9]
= 13.6 ï 9 ï 8/9 = 108.8 eV

ATOMIC STRUCTURE
18 QUIZRR

hc 6.63  10 34  3  108


Wavelength,  = E 
13.6  8  1.6  10 19
= 114.26 ï 10ă10 m = 114.3 Å
(ii) From the excited state (E3), coming back to ground state, there can be 3 possible radiations.

Ex a m p l e 7
Wavelength of high energy transition of H-atom is 91.2 nm. Calculate the corresponding
wavelength of He+ ion.
Solution :
For H atom

1  1 1 
 RH  2  2  ... (i)
n n n2 
 1

1  1 1 
For He+ ion   R H Z2  2  2  ... (ii)
n n2 
He   1
By eq. (i) and (ii)

 He  1
 2
H Z

1 1
He+ = H ï 2 = 91.2 ï
Z 22
= 22.8 nm

Ex a m p l e 8
 21.7  1012
The electron energy in hydrogen atom is given by E  ergs. Calculate the energy
n2
required to remove an electron completely from n = 2 orbit. What is the longest wavelength
(in cm) of light that can be used to cause this transition?
 21.7  1012
Solution : En  ergs
n2
 21.7  1012
E2 
22
Removing the electron completely from the atom implies that the electron has been exited to the
level .

 21.7  1012
 E  0
2
21.7  1012
E = E  E2  ergs
4
ATOMIC STRUCTURE
QUIZRR 19

21.7  1012
E =  107 Joule = 5.425 ï 10ă19 J
4

hc
E =

hc 6.626  1034  3  108


  = E  19
 3.66414  107
5.425  10
  = 3664.14 Å

Ex a m p l e 9

There is a hydrogen atom in the ground state. It is exited to a higher energy level n. When
the electron comes back to the ground state, it emits radiation. What is the maximum number
of unique wavelengths it can emit?

Solution :

n=1

Let us imagine the electron to be in the energy level n. When it comes to the ground state it can come
in many no. of ways. Either it will directly come to n = 1, in which case it will emit only one wavelength
(corresponding to n to 1) or it can fall to n = 2 and then from n = 2 to n = 1, in which case it will emit
two unique wavelengths (one corresponding to n to 2 and the other corresponding to n = 2 to n = 1) and
so on. We have to find how the electron must come back by emitting maximum number of unique
wavelengths. It is very clear that every jump of the electron would radiate a unique wavelength as

1 1 1 
 R 2  2
  n1 n2 

 cannot be same for different sets of n1 and n2.

The only way maximum wavelengths can be emitted is when the electron comes back to ground
state by maximum jumps. This means the electron must jumps stepwise i.e., n to n ă 1 to n ă 2 to n
ă 3 and so on. This means the total jumps would be n ă 1 and so would be the number of unique
wavelength.

 nă1

ATOMIC STRUCTURE
20 QUIZRR

Ex a m p l e 1 0
Calculate the wavelength and wave number of the spectral line when an electron in H-atom
falls from higher energy state n = 3 to a state n = 2. Also determine the energy of a photon
that can remove the electron from state n = 2. Compare it with the energy of photon required
to remove the electron from ground state.
Solution :
Now there are two ways of solving this problem. One by using the relation for the energy emitted
(E) by an electron and then calculating the wavelength and wave number. Other by directly using the
expression for wave number and then calculating wavelength. In the second method, the value of
Rydberg constant (R) is required. The value of R should be given to you otherwise its value cannot be
used and the problem should be done by the former method i.e. by finding E.

 1 1 
E  2.17  1018 Z 2  2  J
 n1  n 2 
 2 

 1 1 
E  2.17  1018  2  2  = 3.04  10 19 J
2 3 

Now E = h = hc / 


hc

 
6.63  1034 3  108 

E 3.04  1019
 = 6.59 ï 10ă7 m = 6590 Å

1 1
 v  m1 = 1.517 ï 106 mă1
 659  107

Alternative Method :
To be used only when the value of R is given.

Using the relation for wave number v : 


 1 1 
v  RZ2  
  n 2  n 2 
 1 2 

 1 1 
= 1.097 ï 107 ï (1)2  2  2  (R = 1.097 ï 107 mă1)
2 3 

= 1.52 ï 106 mă1


Hence  = 1/v = 6.56 ï 10ă7 m/s = 6560 Å

ATOMIC STRUCTURE
QUIZRR 21

To remove electron from n = 2, the responsible transition will be n = 2  n = .

 1 1  19
E = 2.17 ï 10ă18  2  2   5.42  10 J
2  

To remove electron from ground state (n = 1), the transition is n = 1  n = .

 1 1  18
E = 2.17 ï 10ă18  2  2   2.17  10 J
1  

Ex a m p l e 1 1
A hydrogen atom in the ground state is hit by a particle exciting the electron to 4s orbital.
The electron then drops to 2s orbital. What is the frequency of radiation emitted and absorbed
in the process?
Solution :

Energy is absorbed when electron moves from ground state (n = 1) to 4s orbital (n = 4).

 1 1 
  eV
E = 13.6 Z  2 
2
(E : energy absorbed)
 n1  n2 
2 

Putting n1 = 1 & n2 = 4 we get,

E = 12.75 eV = 12.75 (1.6 ï 10ă19) J = 2.04 ï 10ă18 J

E 2.04  10 18
frequency () = h   3.07  1015 Hz
6.63  1034

When electron jump from n = 4 to n = 2 (2s), energy is emitted and is given by the same relation :

Put n1 = 2, n2 = 4 in the expression of E.

E = 2.55 eV = 4.08 ï 10ă19 J

  = E/h = 6.15 ï 1015 Hz

ATOMIC STRUCTURE
22 QUIZRR

Ex a m p l e 1 2
Calculate the wavelength of light radiation that would be emitted, when an electron in the
fourth BohrÊs orbit of He+ ion falls to the second BohrÊs orbit. To what transition does this-
light radiation correspond in the H-atom ?
Solution :
Let us calculate E first
 1 1 
E = 21.7 ï 10ă19 Z2  2  2 
 n1 n2 
substituting n1 = 2 and n2 = 4, Z = 2 we get;
E = 1.632 ï 10ă18 J
hc
Now   1.218  10 7 m  1218 Å
E
The value of  = 1218 Å implies that, in H-atom this transition would lie in Lyman Series.
Hence our aim is now to find the transition : n2  1

 1 1 
 E = 21.7 ï 10ă19 ï 12  2  2 
 n1 n2 

 1 
 1.632 ï 10ă18 = 21.7 ï 10ă19  1  2 
 n2 

 n2 = 2
Hence the corresponding transition in H-atom is 2  1.

Ex a m p l e 1 3
1.8 g hydrogen atoms are excited to radiations. The study of spectra indicates that 27% of the
atoms are in IIIrd energy level and 15% of atoms in IInd energy level and the rest in ground
state. IP of H is 13.6 eV. Calculate
(a) No. of atoms present in III and II energy level.
(b) Total energy evolved when all the atoms return to ground state.
Solution : 1 g H contains = N atoms
 1.8 H contains = N ï 1.8 atoms
= 6.023 ï 1023 ï 1.8
= 10.84 ï 1023 atoms

10.84  1023  27
(a)  No. of atoms in III shell =
100

= 292.68 ï 1021 atoms

ATOMIC STRUCTURE
QUIZRR 23

10.84  1023  15
 No. of atoms in II shell =
100

= 162.6 ï 1021 atoms

10.84  1023  58
and No. of atoms in I shell =
100

= 628.72 ï 1021 atoms

(b) When all the atoms return to I shell, then

E'   E3  E1   292.68  1021

 13.6 
=   13.6   1.602  1019  292.68  1021
 9 

= 5.668 ï 105 Joule

E´´ = (E2 ă E1) ï 162.6 ï 1021

 13.6 
=   13.6   1.602  1019  162.6  1021
 4 

= 2.657 ï 105 Joule


 E = E´ + E´´ = 5.668 ï 105 + 2.657 ï 105 Joule
= 832.50 kJ

Ex a m p l e 1 4
Find out the energy of H atom in first excitation state. The value of permittivity factor 40
= 1.11264 ï 10ă10 C2 Nă1 mă2.

Solution :

In M.K.S. system

22 Z2 me4
En   n= 2
 4 0 2 n2 h2

 
4
2   3.14   1  9.108  1031  1.602  1019
2 2

=
1.11264  10   
2 2
10
  2   6.625  1034
2

= 5.443 ï 10ă19 Joule

ATOMIC STRUCTURE
24 QUIZRR

1 .4 DU A L N A T U RE OF M A T T ER
Wave-particle duality is the concept that all matter and energy exhibits both wave-like and particle-
like properties. A central concept of quantum mechanics, duality addresses the inadquacy of classical
concepts like „particle‰ and „wave‰ in fully describing the behaviour of small-scale objects. Various
interpretations of quantum mechanics attempt to explain this ostensible paradox. Wave-particle duality
should be distinguished from wave-particle complementarity, the latter implying that matter can
demonstrate both particle and wave characteristics, but not both at the same time (that is, not within
one and the same experimental arrangement).

1 .4 .1 d e B r o g l i e Wa v e l e n g t h
The energy of an object (E) in motion is equal to its mass (m) times the square of its speed (s).
E = mc2
The energy of a wave (E) is proportional to its frequency (v) which is proportional to its speed (s)
and inversely proportional to its wavelength () where h is PlanckÊs constant.

hc
E = hv =

If an object acts as both a particle in motion and a wave, then the two equations can be combined :

hc
mc2 

Cancelling like terms and rearranging the equation to solve for the wavelength results in the
following:

h

mc
By definition, momentum (p) of an object is equal to the product of its mass (m) times its speed (c),
so p can be substituted for mc to get the de Broglie equation.
Ć de Broglie equation ă The wavelength () of an object in motion is inversely proportional to
its momentum (p) where h is PlanckÊs constant (6.626 ï 10ă34 J-S).
h

p
With this equation, if the mass of an object is too large (as it is with most objects), the wavelength
would be negligible. Very small particles such as electrons, however, are small enough to exhibit
the properties of both waves and particles.

1 .5 H EI SEN B ERG’S U N CERT A I N T Y PRI N CI PL E


In this work on formulating quantum mechanics, Werner Heisenberg postulated his uncertainty
principle, which states :

h
xp 
2

ATOMIC STRUCTURE
QUIZRR 25

where
 here indicates standard deviation, a measure of spread or uncertainty;
x and p are a particleÊs position and linear momentum respectively.
h is the reduced PlanckÊs constant (PlanckÊs constant divided by 2).
Heisenberg originally explained this as a consequence of the process of measuring : Measuring
position accurately would disturb momentum and vice-versa, offering an example (the „gamma-ray
microscope‰) that depended crucially on the de-Broglie hypothesis. It is now thought, however, that this
only partly explains the phenomenon, but that the uncertainty also exists in the particle itself, even
before the measurement is made.

Ex a m p l e 1 5
An electron is accelerated through a potential difference of V volts. Find the debroglie
wavelength associated with the e.
Solution :
When the electron is accelerated through a potential difference of V volts, it acquires a kinetic
energy given by E = eV, where e is the charge on the electron. Also, if m be its mass and v be the velocity
then,

1 2E
E mv2  v
2 m
h
deBroglie wavelength  
mv
h
 
2Em

Note : The above result can be used directly, whenever required.


In the given case E = eV
h



2 eV m 

Ex a m p l e 1 6
Calculate the uncertainty in position assuming uncertainty in momentum within 0.1% for :
(a) a tennis ball weighing 0.2 kg and moving with a velocity of 10 m/s
(b) a electron moving in an atom with a velocity of 2 ï 106 m/s
Solution : Using Uncertainty Principle,

h
x p 
4

(a) p = mv = 0.2 ï 10 = 2.0 Kgm/s


p = 0.1% of p = 2 ï 10ă3

ATOMIC STRUCTURE
26 QUIZRR

h 6.63  10 34
 x  
4 p 4  3.14  2  103

= 2.135 ï 10ă32 m
(b) For an electron, p =mv
= 9.1 ï 10ă31 ï 2 ï 106
= 1.82 ï 10ă24 Kgm/s
p = 0.1% of p = 1.82 ï 10ă27 Kgm/s
h 6.63  10  34
x  
4 p 2  3.14  1.82  10 21
 x = 2.89 ï 10ă8 m

Ex a m p l e 1 7
Show that the circumferenceof an orbit of Bohr hydrogen atom is an integral multiple the
de Broglie wavelength associates with the electron revolving round the nucleus.
Solution :
According to BohrÊs postulates,

nh
mvr 
2

nh
 the circumference, 2r =
mv

h
From de BroglieÊs equation 
mv

 2r = n

Ex a m p l e 1 8
(a) If a 1 g body is travelling along the x-axis with an uncertainity in veocityof 1 cm/s, what
is theoretical uncertainty in its position ? (b) If an electron is traveling with uncertainity
in velocity of 1 m/s, what is the theoretical uncertainity in its position ?
Solution :
(a) According the uncertainity
h
x  m v 
4

x 
4 mv

ATOMIC STRUCTURE
QUIZRR 27

6.626  1027 ergs sec


=
4  3.14 1g 1cms1
= 5.275 ï 10ă28 cm = 5.275 ï 10ă30 m

6.626 10 34 Js


(b) x 
4  3.14  9.110 31 kg 1 ms1

= 5.797 ï 10ă5 m

1 .6 PH OT OEL ECT RIC EFFECT


1 .6 .1 I n t r o d u c t i o n
The photoelectric effect is a phenomenon in which electrons are emitted from matter after the absorption
of energy from electromagnetic radiation such as X-rays or visible light. The emitted electrons can be
referred to as photoelectrons in this context. The effect is also termed the Hertz Effect, due to its
discovery by Heinrich Rudolf Hertz, although the term has generally fallen out of use.
Photoelectric effect takes place with photons with energies of about a few electronvolts. If the
photon has sufficiently high energy, Compton scattering (ă keV) or pair production (ă MeV) may take
place.
Study of the photoelectric effect led to important steps in understanding the quantum nature of
light and electrons and influenced the formation of the concept of wave-particle duality.

Photoelectric effect

- -

- - - - - -
-
- - - -

The term may also refer to the photoconductive effect (also known as photoconductivity or
photoresistivity), the photovoltaic effect, or the photoelectrochemical effect.

1 .6 .2 Co n c e p t
When a metallic surface is exposed to electromagnetic radiation above a certain threshold frequency
(typically visible light), the light is absorbed and electrons are emitted. In 1902, Philipp Eduard Anton
von Lenard observed that the energy of individual emitted electrons increased with the frequency, or
color of the light. This was at odds with James Clerkk MaxwellÊs wave theory of light, which predicted
that the electron energy would be proportional to the intensity of the radiation. In 1905, Einstein solved

ATOMIC STRUCTURE
28 QUIZRR

this paradox by describing light as composed of discrete quanta, now called photons, rather than
continuous waves. Based upon Max PlanckÊs theory of black-body radiation, Einstein theorized that the
energy in each quantum of light was equal to the frequency multiplied by a constant, later called
PlanckÊs constant. A photon above a threshold frequency has the required energy to eject a single
electron, creating the observed effect. This discovery led to the quantum revolution in physics and
earned Einstein the Nobel Prize in 1921.
The photons of a light beam have a characteristic energy determined by the frequency of the light.
In the photoemission process, if an electron within some material absorbs the energy of one photon and
thus has more energy than the work function (the electron binding energy) of the material, it is ejected.
If the photon energy is too low, the electron is unable to escape the material. Increasing the intensity
of the light beam increases the number of photons in the light beam, and thus increases the number
of electrons emitted, but does not increase the energy that each electron possesses. Thus the energy of
the emitted electrons does not depend on the intensity of the incoming light, but only on the energy of
the individual photons.
Electrons can absorb energy from photons when irradiated, but they follow an „all or nothing‰
principle. All of the energy from one photon must be absorbed and used to liberate one electron from
atomic binding, or the energy is re-emitted. If the photon energy is absorbed, some of the energy
liberates the electron from the atom, and the rest contributes to the electronÊs kinetic energy as a free
particle.

1 .6 .3 Ex p e r im e n t a l r e s u l t s
1. For a given metal and frequency of incident radiation, the rate at which photoelectrons are
ejected is directly proportional to the intensity of the incident light.
2. For a given metal, there exists a certain minimum frequency of incident radiation below which
no photoelectrons can be emitted. This frequency is called the threshold frequency.
3. Above the threshold frequency, the maximum kinetic energy of the emitted photoelectron is
independent of the intensity of the incident light but depends on the frequency of the incident
light.
4. The time lag between the incidence of radiation and the emission of photoelectron is very small,
less than 10ă9 second.
In effect quantitatively using EinsteinÊs method, the following equivalent equations are used :
Energy of photons = Energy needed to remove an electron + Kinetic energy of the emitted electron
Algebraically :
hf =  + Ekmax
where
Ć h is PlanckÊs constant
Ć f is the frequency of the incident photon

ATOMIC STRUCTURE
QUIZRR 29

Ć = hf0 is the work function (sometimes denoted W instead), the minimum energy required to
remove a delocalised electron from the surface of any given metal.

1 2
E k max  mvm is the maximum kinetic energy of ejected electrons.
2

Ć f0 is the threshold frequency for the photoelectric effect to occur.


Ć m is the rest mass of the ejected electron, and
Ć vm is the speed of the ejected electron.
Since an emitted electron cannot have negative kinetic energy, the equation implies that if the
photonÊs energy (hf) is less than the work function (), no electron will be emitted.

Ex a m p l e 1 9
Find the ratio of frequencies of violet light ( = 4.10 ï 10ă5 cm) to that of red light
( = 6.56 ï 10ă5 cm). Also determine the ratio of energies carried by them.
Solution :
Using c = v ;
c : speed of light; v = frequency &  : wavelength

v1 2
 1 : red & 2 : violet
v2 1

v1 6.56  105
   1.6
v2 4.10  10 5

Now the energy associated with electromagnetic radiation is given by E = hv.

E1 v1 2
    1.6
E2 v2 1

Hence the ratio of energies is same as that of frequencies.

Ex a m p l e 2 0
A bulb exits light of  = 4500 AĈ. The bulb is rated as 150 watt and 8% of the energy is emitted
as light. How many photons are emitted by the bulb per second ?
Solution :
hc
Energy of one photon =

6.625  1034  3  108


=
4500  1010
= 4.42 ï 10ă19 J

ATOMIC STRUCTURE
30 QUIZRR

8
Energy emitted by bulb = 150 ï J
100

= 12 J
ă19
 n ï 4.42 ï 10 = 12 (n = no. of photons)

12
n =
4.42  10 19

= 2.72 ï 1019

Ex a m p l e 2 1
An electronic beam can under go diffraction by crystals. Through what potential should a
beams of electrons be accelerated so that its wave length becomes equal to 1.54 AĈ
Solution :
We know that

1
mv2 = eV
2

h h
and  = or v =
mv m

2 h2
or v =
m2  2

1 h2
or m = eV
2 m2  2

1 h2
or V = m
2 m2  2  e

1 h2
= 
2 m2 e

1 (6.62  10 34 )2
Substituting the values, we get V = 2 
9.108  1031  (1.54  10 10 )2  1.602  10 19

= 63.3 volt.

ATOMIC STRUCTURE
QUIZRR 31

Ex a m p l e 2 2
A photon of light with  = 470 nm falls on a metal surface. As a result, photoelectrons are
ejected with a velocity of 6.4 ï 104 m/s. Find :
(a) the kinetic energy of emitted photoelectrons;
(b) the work function (in eV) of the metal surface.
Solution :
(a) Kinetic energy of electron = ó mv2
KE = ó (9.1 ï 10ă31)(6.4 ï 104)2 = 7.45 ï 10ă22 J
(b) From EinsteinÊs photoelectric equation, we have,
Kinetic energy = incident energy ă work function
KE = Eincident ă  (= work function)
 = Eincident ă KE

hc 6.63  10 34  3  108


  KE =  7.45  1022
 9
470  10

= 4.224 ï 10ă19 J
  = 4.224 ï 10ă19 J
= 2.64 eV
Note : More tough problems on photoeletric effect will be dealt with in Modern Physics.

Ex a m p l e 2 3
The dissociation energy of H2 is 430.53 KJ molă1. If H2 is exposed to radiation energy of wave
length 253.7 nm. What % of radiant energy will be converted into kinetic energy ?
Solution :

430.53  103
Energy required to break HăH bond = J/molecule
6.023  1023
= 7.15 ï 10ă19 J
hc
Energy of photon used for this purpose =

6.625  1034  3.0  108


=
253.7  109
= 7.83 ï 10ă19 J
 Energy left after dissociation of bond
= (7.83 ă 7.15) ï 10ă19
or Energy converted into K.E. = 0.68 ï 10ă19 J

0.68  1019
 % of energy used in kinetic energy =  100
7.83  1019
= 8.68%
ATOMIC STRUCTURE
32 QUIZRR

Ex a m p l e 2 4
Find out the number of waves made by a Bohr electron in one complete revolution in its 3rd
orbit.
Solution :
For an electron to be in a particular energy level of radius r, with de Broglie wavelength  :
2r = n
(where n = number of waves in one revolution)

h 2r mv
  n
mv h
for third orbit;
r = 0.53 ï 10ă10(3)2 m; v = 2.165 ï 106 (1/3)
substituting the value of r, v, m and h, we get,
n= 3

Ex a m p l e 2 5
An electron in the first excited state of H-atom absorbs a photon and is further excited. The
debroglie wavelength of the electronic this state is found to 13.4 A0. Find the wavelength of
photon absorbed by the electron in A0. Also find the longest wavelength emitted when this
electron de-excites back to ground state.
Solution :
First excited state refers to the energy level n = 2
The electron from n = 2 absorbs a photon and is further excited to an higher energy level (let us
say n)
The electron in this energy level (n) has de Broglie wavelength () = 13.4 Å.

h Z
 & vn = 2.165 ï 106
mv n

h 6.63  10 34 1
1  2.165  106 

m
 2.165  106 
n

13.4  10    9.1  10 
10 31 n

 n = 4
First find the wavelength of the photon responsible for the excitation from n = 2 to n = 4
Using the relation :

hc  1 1 
E =  2.17  1018 Z2  2  2 
  n2 
 n1
    = 4861.33 Å
The longest wavelength emitted when this electron (from n = 4) falls back to the ground state will
corresponds to the minimum energy transition.

ATOMIC STRUCTURE
QUIZRR 33

The transition corresponding to minimum energy will be 4  3


Note : The transition corresponding to maximum energy will be 4  1
Using the same relation :

hc  1 1 
E   2.17  1018 Z2  2  2 
 n n2 
 1
    = 18750.85 Å
Note : If the value of R is given then use :

1  1 1 
v  R Z2  2  2  R = 1.097 ï 107 mă1.
 n n2 
 1

Ex a m p l e 2 6
The IP, of H is 13.6 eV. It is exposed to electromagnetic waves of 1028AĈ and gives out induced
radiations. Find the wave length of these induced radiations.
Solution :
E1 of H atom = ă13.6 eV

6.625  1034  3  108


Energy given to H atom =
1028  10 10
= 1.933 ï 10ă18 J
= 12.07 eV
 Energy of H atom after exitation = ă13.6 + 12.07
= ă 1.53 eV

E1
En =
n2

13.6
n2 =  9
1.53
n =3
rd
Thus, electron in H atom is excited to 3 shell

hc
 induced 1 =
(E3  E1 )

E1 = ă 13.6 eV
E3 = ă1.53 eV

6.625  1034  3  108


1 =
(1.53  13.6)  1.602  1019
= 1028 ï 10ă10m.
 = 1028 AĈ

ATOMIC STRUCTURE
34 QUIZRR

1 .7 .1 Qu a n t u m N u m b e r s
To understand the concept of Quantum Numbers, we must know the meaning of some terms
clearly so as to avoid any confusion.
Energy Level :
The non-radiating energy paths around the nucleus are called as Energy Levels or Shells. These
are specified by numbers having values 1, 2, 3, 4,... or K, L, M, N, ... corresponding to 1, 2, 3, 4, ... in
order of increasing energies. The energy of a particular energy level is fixed.
Sub-Energy Level :
The phenomenon of splitting of spectral lines in electric and magnetic fields reveals that there
must be extra energy levels with-in a definite energy level. These were called as Sub-Energy Levels or
Sub-Shells. There are four types of sub-shells namely; s, p, d, f.
First energy level (K or 1) has on sub-shell designed as 1s, the second energy level (L or 2) has
two sub-shells as 2s & 2p, the third energy level (M or 3) has three sub-shells as 3s, 3p & 3d, and the
fourth energy level (N or 4) has four sub-shells s 4s, 4p, 4d & 4f. The energy sub-shells increases roughly
in the order : s < p < d < f.
Orbital :
Each sub-energy level (sub-shell) is composed of one or more orbitals. These orbitals belonging to
a particular sub-shell have equal energies and are called as degenerate orbitals. Sub-shell s has one
orbital, p has three orbitals, d has five orbitals and f has seven orbitals.
To describe or to characterise the electrons around, the nucleus in an atom, a set of four numbers
is used, called as Quantum Numbers. These are specified such that the states available to the electrons
should follow the laws of quantum mechanics or wave mechanics.

Principal Quantum Number : (n)

This number represents the main energy levels (principal energy levels) designated as n = 1, 2, 3,
... or the corresponding shells are named as K, L, M, N, ... respectively. It gives an idea of position and
energy of an electron. The energy level n = 1 corresponds to minimum energy and subsequently n = 2,
3, 4, ... are arranged in order of increasing energy.
Ć Higher is the value of n, greater is its distance from the nucleus, greater is its size and also
greater is its energy.
Ć It also gives the total electrons that may be accommodated in each shell, the capacity of each
shell is given by the formula 2n2, where n : principal quantum number.

Azimuthal Quantum Number : (l)

This number determines the energy associated with the angular momentum of the electron about
the nucleus. It is also called as the angular momentum quantum number. It accounts for the appearance
of groups of closely packed spectral lines in electric field (Zeeman Lines).
Ć It can assume all integral values from 0 to n-1. The possible values of l are :
1, 2, 3, ...., n - 1.

ATOMIC STRUCTURE
QUIZRR 35

Ć Each value of l describes a particular sub-shell with in the main energy level and determines
the shape of the electron cloud.
Ć When n = 1, l = 0, i.e., its energy level contains one sub-shell which is called as a sub-shell.
So for l = 0, the corresponding sub-shell is s sub-shell. Similarly when l = 1, 2, 3, the sub-shells
are called as p, d, f sub-shells respectively.
Ć As you know for n = 1, l = 0, there is only one sub-shell. It is represented by 1s. Now for n =
2, l can take two values (the total number of values taken by l is equal to the value of n in a
particular energy level). The possible values of l are 0, 1. The two sub-shells representing the
IInd energy level are 2s, 2p. In the same manner, for n = 3, three sub-shells are designated as
3s, 3p, 3d corresponding to l = 0, 1, 2 and for n = 4, four sub-shells are designated as 4s, 4p,
4d 4f corresponding to l = 0, 1, 2, 3.

h
Ć The orbital Angular momentum of electron = l  l  1
2

Note that its value does not depend upon value of n.

Magnetic Quantum Number : (m)

An electron with angular quantum number can be thought as an electric current circulating in a
loop. A magnetic field due to this current is observed. This induced magnetism is determined by the
magnetic quantum number. Under the influence of magnetic field, the electrons in a given sub-energy
level prefer to orient themselves in certain specific regions in space around the nucleus. The number
of possible orientations for a sub-energy level is determined by possible values of m (this corresponds
to the number of orbitals in a given sub-energy level).

• m can have any integral values between -l to + l including 0, i.e., m = - l, -l + 1, ......, 0, 1, 2,


3, ......, l - 1, + l. We can say that a total of (2l + 1) values of m are there for a given value
of l.

Ć In s sub-shell, there is only one orbital [l = 0,  m = (2l + 1) = 1].

Ć I p sub-shell, there are three orbitals corresponding to three values of m : - 1, 0, + 1.


[l = 1  m = (2l + 1) = 3]. These three orbitals are represented as px, py, pz along X, Y, Z axes
perpendicular to each other.

Ć In d sub-shell, there are five orbitals corresponding to - 2, - 1, 0, + 1, + 2. [l = 2  m =


(2l + 1) = 5]. These five orbitals are represented as d xy, d yz, d zx, d x2-y2, d z2.

Ć In f sub-shell there are seven orbitals corresponding to - 3, - 2, - 1, 0, + 1, + 2, +3.


[l = 3  m = (2l + 1) = 7].

Spin Quantum Number : (s)

When an electron rotates around a nucleus it also spins about its axis. If the spin is clockwise, its
spin quantum number is (+ ó ) and is represented as  or 1. If the spin is anti-clockwise, its value is

ATOMIC STRUCTURE
36 QUIZRR

ă ó and is represented as  or 1. If the value of s is (+ ó ) then by convention, we take that electron


as the first electron in that orbital and if the value of s is (ă ó ) it is taken as second electron.

1 .7 .2 Sh a p e s o f A t o m i c Orb i t a l s
(i) s-orbital : An electron is considered to be smeared out in the form of a z
cloud. The shape of the cloud is the shape of the orbital. The cloud is not
uniform but denser in the region where the probability of finding the
electron in maximum. + x
The orbital with the lowest energy is the 1s orbital. It is a sphere with
its centre at the nucleus of the atom. The s-orbital is said to spherically
symmetrical about the nucleus, so that the electronic charge is not y
concentrated in any particular direction. 2s orbital is also spherically Shape of s-orbital
symmetrical about the nucleus, but it is larger than (i.e., away from) the
1s orbital.
(ii) p-orbitals : There are three p-orbitals :
z
px, py and pz. They are dumb-bell shaped, z z
the two levels being separated by a nodal +
+
plane, i.e., a plane where there is no likely ă + x x x
ă y
hood of finding the electron. The p-orbitals y y py
ă
px
have a marked directional character, pz
depending on whether px, py and pz orbital Shapes of p-orbital
is being considered. The p-orbitals consist
of two lobes with the atomic nucleus lying between them. The axis of each p-orbital is
perpendicular to the other two. The px, py and pz orbitals are equivalent except for their
directional property. They have the same energy. Orbitals having the same energy are said
to be degenerate.
z z z
(iii) d-orbitals : There are five
d-orbitals. The shapes of four
d-orbitals resemble four leaf
cloves. The fifth d-orbital loops
x x x
differently. The shapes of these
orbitals are given below. y y
3dxy y
3dxz
3dyx

z
z

x x
y y
3dx2 ă y2
3dz2

ATOMIC STRUCTURE
QUIZRR 37

Qu a n t u m M e c h a n i c a l M o d e l
In the quantum mechanical model of an atom, there is only a „chance‰ or probability of finding an
electron in space. Where this probability is maximum (over 95%), that region of space is called Âan
orbitalÊ.
The concept of orbital comes from the Schrodinger wave equation.

d2 d 2 d 2 82 m
   E  V    0
dx2 dy2 dz2 h2

The symbols have the following meanings :


m  Mass of an electron
h  PlanckÊs constant
E  Total energy of electron
V  Potential energy of electron
 Wave function
Electrons have associated wave functions denoted by 
 itself has no physical significance. Its square, however, gives the probability of finding an
electron at a given point. More often, we are interested in finding the probability of finding an electron
in a region. For this, consider a sphere of radius r and thickness dr.
The volume of the shell of thickness dr is 4r2dr.
r
This volume multiplied by the probability (2) gives the probability of finding an
electron in the shell of thickness dr.
dr
Thus, p (r) = 4r2dr |2|
Since this function depends only on the value of radial distance from the centre, it is called radial
probability function.
Radial node or Spherical nodes : Now, in any atom, the quantum number n gives the distance
between the nucleus & the outermost shell. So, it is a measure of the radial distance of an electron. It
so happens that for certain orbitals, the probability of finding an electron drops to zero at certain
distances from the nucleus. Such points are called radial nodes. (A node is a region where the probability
of finding an electron is zero).
Following graphs will further illustrate this point
The number of radical nodes increased with the increase in distance between nucleus and the
outermost shell.

s
P(r) P(r) P(r)
= 4r2| 2|dr = 4r2| 2|dr = 4r2dr| 2|
2s wide 3s
wide

r wide r
r
ATOMIC STRUCTURE
38 QUIZRR

Angular or Planar node : This type of node corresponds to the quantum number l. In s orbitals,
due to their spherical symmetry, there are no nodal plane. Consider px orbital.
In the y-z plane, the probability of finding an electron is O. So, y-z is a nodal plane.
Similarly, for d-orbitals there are 2 nodal planes. y

Formulae :
No of radial nodes = n ă l ă 1 x
No. of angular nodes = l
Total no. of nodes = n ă 1

1 .8 EL ECT RON I C CON FIGU RA T I ON OF EL EM EN T S :


Quantum numbers can now characterise the electrons in an atom. To describe the arrangements
and distribution of electrons for different elements, following rules and selective principles are used. The
distribution of electrons in an atom is known as the electronic configuration of that element.

Aufba u Principle

An atom in its lowest state of energy is said to be in ground state. The ground state is the most
stable state in an atom. According to Aufbau principle :
„electrons are added progressively to the various orbitals in their order of increasing energy starting
with the orbital of lowest energy.”
The order of increasing energy may be summed up as follows :
1s, 2s, 2p, 3s, 3p, 4s, 3d, 4p, 5s, 4d, 5p, 6s, 4f, 5d, 6p, 7s, 5f, 6d,.....
As a working rule, a new electron enters an empty orbital for which the value of (n + l) is minimum. If the
value of (n + l) is same for two or more orbitals, the new electron enters an orbital having lower value of n.

Pauli Exclusion Principle

According to this principle :


„no two electrons in an atom can have the same set of all the quantum numbers or one can say that
no two electrons can have the same quantised states.”
Consider an electronic arrangement in Ist energy level (n = 1). For n = 1, l = 0, & m = 0. Now s
can have two values corresponding to each value of m i.e. s = + ó, ă ó. Hence, the possible designation
of an electron in a state with n = 1 is 1, 0, 0, + ó and 1, 0, 0, ă ó (n, l, m, s) i.e., two quantised states.
This implies that an orbital can accommodate (for n = 1, m = 0,  one orbital) maximum of two electrons
having opposite spins.
The maximum number of electrons in the different subshells are :
s sub-shell = 2, p sub-shell = 6, d sub-shell = 10 and f sub-shell = 14.

ATOMIC STRUCTURE
QUIZRR 39

Hund’s Rule of Ma ximum Multiplicity

According to this rule :


„electrons never pair until no available empty degenerate orbitals are left to them.”
This means an electron always occupies a vacant orbital in the same sub-shell (degenerate orbital)
and pairing starts only when all of the degenerate orbitals are filled up. This means that the pairing
starts with 2nd electron in s sub-shell, 4th electron in p sub-shell, 6th electron in d sub-shell and 8th
electron in f sub-shell.
By doing this, the electrons stay as far away from each other as possible. This is highly reasonable
if we consider the electron-electron repulsion. Hence electrons obey HundÊs rule as it results in lower
energy state and hence more stability.

Extra Sta bility of Half a nd Fully Filled Orbita ls

A particularly stable system is obtained when a set of equivalent orbitals (degenerate orbitals) is
either fully filled or half filled, i.e., each containing one or a pair of electrons. This effect is more
dominant in d and f sub-shells.
This means three or six electrons in p sub-shell, five or ten electrons in d sub-shell, and seven or
fourteen electrons in f sub-shell forms a stable arrangement. Note this effect when filling of electrons
takes place in d sub-shells (for atomic numbers Z = 24, 25 and 29, 30).

In the following table you should a na lyse how to employ the a bove rules to write electronic
configura tion of various elements.

Electronic configuration of an element is represented by the notation n lx :


x : number of electrons present in an orbital
l : denotes the sub-shell
n : principal quantum number

ATOMIC STRUCTURE
40 QUIZRR

Ground-State Electronic Configuration of the Elements


Atomic
Number Symbol 1s 2s 2p 3s 3p 3d 4s 4p 4d 4f
1 H
2 He 1
2
3 Li 2 1
4 Be 2 2
5 B 2 2 1
6 C 2 2 2
7 N 2 2 3
8 O 2 2 4
9 F 2 2 5
10 Ne 2 2 6
11 Na 2 2 6 1
12 Mg 2 2 6 2
13 Al 2 2 6 2 1
14 Si 2 2 6 2 2
15 P 2 2 6 2 3
16 S 2 2 6 2 4
17 Cl 2 2 6 2 5
18 Ar 2 2 6 2 6
19 K 2 2 6 2 6 1
20 Ca 2 2 6 2 6 2
21 Sc 2 2 6 2 6 1 2
22 Ti 2 2 6 2 6 2 2
23 V 2 2 6 2 6 3 2
24 Cr 2 2 6 2 6 5 1
25 Mn 2 2 6 2 6 5 2
26 Fe 2 2 6 2 6 6 2
27 Co 2 2 6 2 6 7 2
28 Ni 2 2 6 2 6 8 2
29 Cu 2 2 6 2 6 10 1
30 Zn 2 2 6 2 6 10 2
31 Ga 2 2 6 2 6 10 2 1
32 Ge 2 2 6 2 6 10 2 2
33 As 2 2 6 2 6 10 2 3
34 Se 2 2 6 2 6 10 2 4
35 Br 2 2 6 2 6 10 2 5
36 Kr 2 2 6 2 6 10 2 6
Note : In atomic numbers, 24 and 29, observe the rule of extra-stability of half filled and fully filled orbitals.

ATOMIC STRUCTURE
QUIZRR 41

Ex a m p l e 2 7
(a) Find the orbital angular momentum of an electron in the following orbital
(i) 3p (ii) 3d (iii) 3s
(b) Arrange the electrons represented by the following sets of quantum numbers in
decreasing order of energy
(i) n = 4, l = 0, m = 0, s = ó (ii) n = 3, l = 2, m = 0, s = ó
(iii) n = 3, l = 1, m = 1, s = ă ó (iv) n = 3, l = 0, m = 0, s = ă ó
Solution :
h
(a)   l  l  1  l  l  1 h
2

(i) For 3p, l = 1,  l  1 1  1  h  2 h

h
(ii) For 3d, l = 2,  l  2  2  1  6h
2
(iii) For 3s, l = 0, øl = 0
(b) Higher the value of (n + l) higher will be the energy of orbital. If the two values are same then
one having higher value of n the energy will be higher
For (i) (n + l) = 4 4s orbital
For (ii), (n + l) = 5, 3d orbital
For (iii) (n + l) = 4 3p orbital
For (iv), (n + l) = 3 3s orbital
Decreasing order of energy : (ii) > (i) > (iii) > (iv)

Ex a m p l e 2 8
Consider the following electronic configuration of the element
(i) 1s22s1 (ii) 1s23s1
(a) Name the element corresponding to (i)
(b) Does (ii) correspond to the same or different element
(c) How can (ii) be obtained from (i)
(d) Is it easier to remove one electron from (ii) or (i) ? Explain.
Solution :
(a) The element corresponding to (i) is lithium (Li).
(b) This electronic configuration represents the same element in the excited state.
(c) By supplying energy to the element when the electron jumps from the lower energy 2s orbital
to the higher energy 3s orbital.
(d) It is easier to remove an electron from (ii) than from (i) since in the former case the electron
is present in a 3s orbital and hence is less strongly attached by the nucleus than a 2s
orbital.

ATOMIC STRUCTURE
42 QUIZRR

Ex a m p l e 2 9
A neutral atom of an element has 2K, 8 L, 5M electrons. Find out the following :
(a) Atomic Number
(b) Total number selectrons
(c) Total number of p electrons
(d) Number of protons
(e) Valency
Solution : Electronic configuration of tievtral atom :

1s2 2s2 2P 6 3s2 3P3


, ,
K L M
(a) Atomic number = Total number of electrons in neutral atom = 15
(b) Total number of s electrons = 6
(c) Total number of p electrons = 9
(d) Number of protons = 15
(e) Valency = 3

Ex a m p l e 3 0
An electron in a Bohr orbit of hydrogen atom in quantum level n2 has an angular momentum
of 4.276 ï 10ă34 kgm2 secă1. If this electron drops from this level to the next lower level, find
the wavelength of this spectral line. (given RH = 109679 cmă1)
Solution : According to BohrÊs theory

nh
mvr 
2
mvr = 4.2176 ï 10ă34 kg m2să1

n

2  3.14  4.2176  1034 kg m2 s1 
6.626  10 34
Js 
= 3.99
  4
when an electron falls from n = 4 to n = 3 in a hydrogen atom, the wavelength emitted is
calculated as,

1 1 1 
 RH  2  2 
  n1 n2 

1 1 
= 109679  2  2 
3 4 

ATOMIC STRUCTURE
QUIZRR 43

 = 1.8756 ï 10ă4 cm
= 1.8756 ï 10ă6 m
= 18756 ï 10ă10 m
= 18756 Å

Ex a m p l e 3 1
Calculate the energy emitted when electrons of 1.0 g atom of hydrogen undergo transition
giving the spectral lines of lowest energy in the visible region of its atomic spectra.
RH = 1.1 ï 107 mă1, c = 3 ï 108 m secă1 and h = 6.62 ï 10ă34 J sec
Solution :
For visible line spectrum, i.e., Balmer series n1 = 2. Also for minimum energy transition n2 = 3.

1 1 1 
  R H  2  2  for H-atom
  n1 n2 

1 1 1
  RH  2  2 
 2 3 

7 1 1 7 5 1
= 1.1  10     1.1  10  m
4 9 36

  = 6.55 ï 10ă7 m

hc 6.62  1034  3.0  108


E 
 6.55  10 7

= 3.037 ï 10ă19 Joule


If N electrons show this transition in 1 g atom of H then
Energy released = E ï N
= 3.03 ï 10ă19 ï 6.023 ï 1023
= 18.29 ï 104 J
= 182.9 kJ

Ex a m p l e 3 2
Consider the hydrogen atom to be a proton embedded in a cavity of radius (Bohr radius)
whose charge is neutralized by the addition of an electron to the cavity in a vacuum, infinity
slowly. Estimate the average total energy of an electron of an electron in its ground state as
the work done in the above neutralization process. Also, if the magnitude of the average
kinetic energy is half of the magnitude of the average potential energy, find the average
potential energy.

ATOMIC STRUCTURE
44 QUIZRR

Solution :
The work done in the neutralization process is the energy due to the electrostatic force of attraction
between the electron and the proton.

kZe  ( e)
 Potential energy =
a0

 kZe2 ke2
= 
a0 a0

where e is the charge on the electron/proton, a0 is the Bohr radius, k is the coulombic law constant
and Z is the atomic number = 1 ( one proton)
Kinetic energy of the electron moving in the orbit = ó mv2 (m is the mass of electron and v is the
velocity).
 Total energy of the electron

 ke2 1
E  mv2 ... (i)
a0 2

For the electron to stable in the orbit, the electrostatic force of attraction should be equal to the
centripetal force.

ke2 mv2
 
a 20 a0

ke2
 mv2
a0

Substituting in (i)

 ke2 1 ke2  ke2


E  
a0 2 a0 2 a0

Ex a m p l e 3 3
Positronium consists of an electron and a positron (same mass, opposite change) orbiting
around their common centre of mass. The spectrum is therefore expected to be hydrogen
like, the difference arising from the mass differences. Calculate the wave number of the first
three lines of Balmer series of positronium.
Solution : The mass of the nucleus and the electron are comparable and in such systems the mass of
the nucleus is also to be taken into account. In the energy equation, instead of the mass of the electron,
the reduced mass is to be taken.
By definition, reduced mass ø is
1 1 1
 
 m M

ATOMIC STRUCTURE
QUIZRR 45

m is the mass of the electron and M is the mass of the nucleus

mM

m+M

Since the mass of positron = mass of electron,

mm m
 
2m 2
Therefore with equation for Rydberg constant, for positronium is

22e4 k2 22  m / 2  e4 k2
R pos  
h3 c h3 c

RH
=
2

109737
=  54868.5 cm 1
2
First line of Balmer series :

 1 1 
v1  54868.5  2  2   7620.6 cm 1
2 3 
Second line :

 1 1 
v2  54868.5  2  2   10288 cm 1
2 4 

Third line :

 1 1 
v3  54868.5    11522 cm 1
2 2 
2 5 

Ex a m p l e 3 4
The de Broglie wavelength of electron of He+ ion is 3.329 Å. If the photon emitted upon de-
exiciation of this He+ ion is made to hit H atom in its ground so as to liberate electron from
it, what will be the de BroglieÊs wavelength of photoelectron?

h
Solution : 
2 mE

 6.62510 
2
34
h2
 E 
2m 2 2  9.11 10   3.329 10 
2
31 10

= 21.73 ï 10ă19 J = 13.58 eV

ATOMIC STRUCTURE
46 QUIZRR

this is the K.E. of electron


Therefore, total energy = ă 13.58 eV

Z2
En   13.6  eV
n2
For H+ ion, Z = 2 and En = ă13.58 eV

 13.6  4
So, n2 = n= 2
 13.58

Thus, He+ ion is in the 1st excited state.

 1 1 
Energy of photon emitted = 13.6 Z2  2  2  eV
 n1 n2 

 1
= 1.36 ï 4  1    40.8 eV
 4

Energy of photon = I.P. of H + K.E. of photoelectron


Thus, K.E. of photoelectron = 40.8 ă 13.6 = 27.2 eV

h 6.625  1034
 
2mE = 2.351 ï 10ă10 m = 2.351 Å
2  9.111031  27.2 1.602 1019

Ex a m p l e 3 5
The wavelength of the photoelectric threshold of a metal is 230 nm. Determine the K.E. of
photoelectron ejected from the surface by UV radiation emitted from the second longest
wavelength transition (downward) of electron in Lyman series of the atomic spectrum of
hydrogen. (R = 1.9677 ï 107 mă1)
Solution :
The minimum energy (E) required to eject an electron per atom from metal surface is given by
PlanckÊs quantum theory as adopted by Einstein to explain photoelectric effect.

hc 6.625  1034  3  108


E0  hv0  
0 230  109

= 8.63 ï 10ă19
For spectral line in Lyman series of H atom

1  1 
 R 1  2 
  n 

ATOMIC STRUCTURE
QUIZRR 47

For the 2nd longest wavelength transition, n = 3, so

1 8 8
 R   1.09677  107 m1
 9 9

9
or 
8 1.09677 107

= 1.025 ï 10ă7 m = 102.5 nm


Energy of photon corresponding to this wavelength is as given below

c 6.625 1034  3 108


Eh 
 1.025 10 7

= 1.937 ï 10ă18 J
 K.E. of photoelectron = E ă E0
= 19.37 ï 10ă19 ă 8.63 ï 10ă19 = 1.074 ï 10ă18 J

Ex a m p l e 3 6
The second ionization potential of Be is 17.98 eV. If the electron in Be3+ is assumed to move
in a spherical orbit with a central field of effective nuclear charge (Zeff) consisting of the
nucleus and other electrons, by how many units of charge is the nucleus shielded by other
electrons? (The energy of electron in first Bohr orbit of H is ă 13.6 eV). If the extent of
shielding by the K electrons of Li atom is the same as you have calculated above, find the
ionization potential of Li.
Solution :

13.6Z2eff
I.E.   17.98
n2
 Zeff = 2.3
Shielding extent = Z ă Zeff = 4 ă 2.3 = 1.7
Zeff for lithium = 3 ă 1.7 = 1.3
 I.E. of lithium = 13.6 ï (1.3)2 = 22.984 eV

ATOMIC STRUCTURE
STOICHIOMETRY
QUIZRR 3

2 STOICHIOMETRY

2 .1 ST OI CH I OM ET RY :
Derived from the Greek word „stoichio‰ and „metry‰, Stoichiometry literally translates to
Measurement of elements. It deals with determination of quantities of products or reactants of a chemical
reaction.
Stochiometry can be divided into two sub-categories :
(a) Gravimetric analysis
(b) Volumetric analysis.
There are two methods to solve problems in stochiometry, one using the mole concept while the
other using equivalent concept. Both methods are equally important and would be discussed in the text
below.
Let a balanced equation
mA + nB  pC + qD
In this reaction
A and B are the reactants
C and D are the products
m, n p and q are stoichiometric coefficient.
Mole concept states that
m moles of A = n moles of B = p moles of C = q moles of D.

2 .2 CH EM I CA L EQU A T I ON S A N D ST OI CH I OM ET RY
Let the balanced chemical equation we have is
MnO2 + 4HCl  MnCl2 + 2H2O + Cl2
The quantitative information drawn from this balanced chemical equation is
(a) The molar ratio in which two reactants (MnO2 and HCl) reacting is 1 : 4.
(b) The molar ratio between two products can be known i.e. moles of H2O produced would be
double the moles of MnCl2 produced.
(c) The initial moles of MnO2 and HCl (to be taken in vessel) for the reaction to occur may not
necessarily be 1 and 4 respectively or also should not be in the molar ratio of 1 : 4.
(d) We can start reaction with MnO2 and HCl taken in any molar ratio, but the moles of two
reacting will always be in the molar ratio of 1 : 4.
(e) The balanced chemical equation should follow the law of conservation of mass.

STOICHIOMETRY
4 QUIZRR

Let us consider the same chemical system with initial composition (in terms of mole) as

n0MnO2 , n0MnCl2 , n0H2O and n0Cl2 . The n0HCl is four times of n0MnO2 . When the reaction occurs, these mole
numbers change as the reaction progresses. The mole numbers of the various species do not change
independently but the change are related by the stoichiometric coefficients in the chemical equation. Let
after time ÂtÊ from the commencement of reaction, the moles of MnO2 reacting be x, then the moles of
HCl reacting in the same time interval be 4x since MnO2 and HCl react in the molar ratio of 1 : 4.
Thus, after time t, the composition of the system would be

nMnO = n0MnO2 ă x
2

nHCl = n0HCl ă 4x

nMnCl = n0MnCl2 + x
2

nH = n0MnO2 + 2x
2O

nCl = n0Cl2 + x
2

The algebric signs, + and ă indicates that the reactants are consumed and products are produced.
After long time interval from the commencement of reaction i.e. after  time, the composition of
the system would be
nMnO = 0, nHCl = 0
2

n0HCl
nMnCl = n0MnCl2 + n0MnO2 = n0MnCl2 +
2 4

n 0HCl
nH O
= n0H2O + 2n0MnO2 = n0H2O +
2 2

n 0HCl
nCl = n0Cl2 + n0MnO2 = n0Cl2 +
2 4

Ex a m p l e 1
Calculate the weight of time (CaO) that can be prepared by heating 200 kg of limestone
(CaCO3) which is 95% pure.
Solution :
CaCO3  CaO + CO2

95
Actual amont of CaCO3 =  200000g  190000 g.
100

Since Ca atoms are conserved,


 moles of Ca in CaCO3= moles of Ca in CaO

STOICHIOMETRY
QUIZRR 5

1 ï moles of CaCO3 = 1 ï moles of CaO

190000 Wt. of CaO



100 56

Wt. of CaO = 106400 = 106.4 kg

2 .3 CON CEPT OF L IM IT I N G REA GEN T


In the single-reactant reactions, the calculations are carried out with only that amount of the
reactant which has converted to the product.
In the reactions where more than one reactant are involved, one has first to identify the limiting
reactant, i.e., the reactant which is completely consumed. All calculations are to be carried out
with the amount of the limiting reactant ony. Now the question is how to know the limiting
reactant? The procedure is simple as illustrated below :

Initially ... 5 mole 12 mole 0 mole


A + 2B  4C
If A is the limiting reactant : Moles of C produced = 20
If B is the limiting reactant : Moles of C produced = 24

The reactant producing the least number of moles of the product is the limiting reactant
and hence A is the limiting reactant. Thus;
Initially ... 5 mole 12 mole 0 mole
A + 2B  4C

Finally ... 0 mole 2 mole 20 mole

The limiting reactant can also be ascertained by the knowing the initial number of equivalents (or
milli-equivalents) of each reactant. The reactant with the least number of equivalents (or milli-
equivalents) is the limiting reactant. The equivalent method to identify the limiting reactant can
be applied even if the chemical equation is not balanced.
Suppose you are given 5 moles of MnO2 and 18 moles of HCl and you have to calculate the
maximum amount of MnCl2 being produced.
The balanced equation is :
MnO2 + 4HCl  MnCl2 + 2H2O + Cl2
initial moles 5 18
Now since 1 mole of MnO2 reacts completely with 4 mole of HCl to produce 1 mole of MnCl2.
Therefore, 5 mole of MnCl2 will react with 20 mole of HCl. But since moles of HCl is only 18. So,
HCl will be the reagent, which will be consumed first, and hence HCl is the limiting reagent. If
MnO2 have been the limiting reagent 5 mole of it would have given 5 moles of MnCl2 but it is not so.
As one can see from the balanced chemical equation that 1 mole of MnCl2 is produced from 4 mole

STOICHIOMETRY
6 QUIZRR

of HCl. So 18 moles of HCl will produce 4.5 moles of MnCl2. So, one can define limiting reagent.
„The reagent producing the least number of moles of products is the limiting reagent.‰

Y I EL D I N A REA CT I ON WI T H L I M I T I N G REA GENT


For reactions where one of the reagent is limiting, the product yield would not be 100% but would
depend on amount of limiting reagent.

actual yield
Percentage yield = Theoretical maximum yield  100

actual yield
 % yield of the product = Theoretical maximum yield  100

Ex a m p l e 2
The reaction, 2C + O2  2CO is carried out by taking 24 g of carbon and 96 g O2, find out :
(a) Which reactant is left in excess ?
(b) How much of it is left ?
(c) How many mole of CO are formed ?
(d) How many gm of other reactant should be taken so that nothing is left at the end of
reaction ?
Solution :
2C + O2  2CO

24 96
Mole before reaction
12 32

=2 =3 0
Mole after reaction 0 2 2
(a) O2 is left in excess.
(b) 2 mole of O2 or 64 gm O2 is left
(c) 2 moe of CO or 56 gm CO is formed
(d) To use O2 completely total 6 mole of carbon or 72 gm carbon is needed.

2 .4 GRA V I MET RI C A N A L Y SIS


Gra vimetric a nalysis is a n a nalytica l tec hnique ba sed on the mea surement of ma ss of
solid substances a nd/or volume of ga seous spec ies.
The gravimetric analysis is broadly classified into three heads.
(a) Mass-mass relationship
(b) Mass-volume relationship and
(c) Volume-volume relationship

STOICHIOMETRY
QUIZRR 7

2 .4 .1 M a s s –Ma s s Re l a t io n s h i p
This relates the mass of a species (reactant or product) with the mass of another species (reactant
or product) involved in a chemical reaction.
Let us consider a chemical reaction

CaCO3 (s)   CaO(s)  CO2 (g)
Let the mass of CaCO3 taken be x g and we want to calculate the mass of CaO obtained by heating
x
x g CaCO3. Then the moles of CaCO3 taken would be (where M1 represents the molar mass of
M1
CaCO3). According to the balanced reaction, the molar ratio of CaCO3 and CaO is 1 : 1. So same number
 x 
of moles   of CaO would be formed. Now for converting the moles of CaO into mass of CaO
 M1 
obtained, we need to multiply the moles of CaO with the molar mass of CaO. Let the molar mass of CaO
 x 
be M2, so the mass of CaO obtained by heating x g of CaCO3 would be   M2  g .
 M1 

Ex a m p l e 3
What will be the mass of CaO produced by 10 gm of CaCO3. The reaction is


CaCO3  s   CaO  s   CO2 (g)

Solution :


CaCO3  s   CaO(s)  CO2 (g)

at t = 0 10 gm 0 0
at t =  0 ? ?
(completely
consumed)

wt. in gm 10
No. of moles of CaCO3 = 
Mol. wt 100

= 0.1 mole
from mole concept
1 mole of CaCO3 = 1 mole of CaO
 moles of CaO produced will be equal to moles of CaCO3
moles of CaO(s) = 0.1 mole
wt. of CaO = 0.1 ï mol. wt. of CaO
= 0.1 ï 56
= 5.6 gm

STOICHIOMETRY
8 QUIZRR

2 .4 .2 M a s s -V o l u m e Re l a t io n s h i p
This establishes the relationship between the mass of a species (reactant or product) and the
volume of a gaseous species (reactant or product) involved in a chemical reaction. Let us take x g of
CaCO3 in a vessel of capacity V L and the vessel is heated so that CaCO3 decomposes as

CaCO3 (s)   CaO(s)  CO2 (g)
We want to find out the volume of CO2 evolved at STP by heating x g of CaCO3. Then

x
Moles of CaCO3 = M (since molar ratio of CaCO3 and CO2 is 1 : 1)
1

 x 
 Volume of CO2 evolved at STP =   22.4  L
 M1 
But, if the volume of CO2 evolved is to be calculated at pressure P and temperature T K. Then,
x
moles of CO2 evolved =
M1
x RT
Volume of CO2 evolved at pressure P and temperature T =  (Using PV = nRT)
M1 P

Ex a m p l e 4
What volume of oxygen (NTP) will be required to react with 1 gm of Ca ?
Solution :

1
Ca(s)  O2  g   CaO(s)
2

t= 0 1 gm ?
Applying mole concept

1
1 mole of Ca = mole of O2
2

Since volume of 1 mole of O2 (g) at NTP = 22.4 lt.

1
 volume of mole of O2 (g) at NTP = 11.2 lt.
2

There can be written as


1 mole of Ca or 40 gm of Ca react with 11.2 lit. of O2 at NTP

11.2
 1 gm Ca react with =  0.28 lt.
40

STOICHIOMETRY
QUIZRR 9

2 .4 .3 V o l u m e -V o l u m e Re l a t io n s h i p
This relationship deals with the volume of a gaseous species (reactant or product) with the volume
of another gaseous species (reactant or product) involved in a chemical reaction.
Let us consider the reaction, N2(g) + 3H2(g)  2NH3(g). We are given x L of N2 at pressure P atm
and temperature T K and we want to know the volume of H2 required to react with it at the same
pressure and temperature.

Px
Moles of N2 =
RT

3P x
Moles of H2 required = (since molar ratio of N2 & H2 is 1 : 3)
RT

3  P  x  RT
 Volume of H2 required at same pressure & temperature =  3 xL
RT  P

This result could also have been obtained by knowing that for a gaseous relation, at the same
pressure and temperature, the moles of gas is directly proportional to volume of the gas (V  n since
P & T are constant) or molar ratio and volume ratio are same.
Thus, when x L of N2 is taken at Pressure P atm temperature T K,then at the same pressure and
temperature, the volume of H2 required would be 3 x L (since the volume ratio of N2 & H2 would be
same as molar ratio i.e. 1 : 3). But if the volume of H2 required is to be calculated at another pressure
P atm and temperature TÊK, then

3P x
Moles of H2 required =
RT

 Volume of H2 required at pressure PÊ atm & temperature TÊK

3  P  x  RT '  3 x  PT ' 
=  L
RT  P'  P' T 

Ex a m p l e 5
8 gm of methane is burnt with 4.48 L of O2 at STP. Find out the volume of CO2 gas produced
at STP and also the weight of CO2 gas.
Solution :
The balanced chemical equation is
CH4 + 2O2  CO2 + 2H2O
1 mol 2 mol 1 mol
Ideal 16 gm 2 ï 22.4 L 22.4 L
44 gm

STOICHIOMETRY
10 QUIZRR

8
No. of moles of CH4 =  0.5 mol
16

4.48 L
No. of moles of O2 = 22.4 L = 0.2 mol

Now since 1 mole of CH4 required 2 mole (i.e., 44.82) of O2 for complete combustion. But the given
moles of O2 is only 0.2 mol. So, O2 is the limiting reagent.

Again, since 2 moles of O2 reacts with 1 mole of CH4 to give 22.4 L of CO2 at STP.

So 0.2 mole of O2 will react with 0.1 mol of CH4 to give 2.24 L of CO2.

Wt. of CO2 produced = 0.1 mol ï 44

= 4.4 gm of CO2

Ex a m p l e 6

NaCl of 95% purity is used to prepare salt cake (Na2SO4) by the reaction,

2NaCl + H2SO4  Na2SO4 + 2HCl

If the product (Na2SO4) is only 85% pure, what weight of NaCl is used up in producing 1 kg
of the impure salt cake?

Solution :

Let the weight of NaCl used up in producing 1 kg of impure product be x g.

x  95
Mass of pure NaCl =
100

x  95
Moles of pure NaCl = 100  58.5

x  95 1
Moles of pure Na2SO4 = 
100  58.5 2

x  95 1 100  85
Mass of pure Na2SO4 =   142 
100  58.5 2 100

 x = 73.72 g

STOICHIOMETRY
QUIZRR 11

Ex a m p l e 7
Calculate the mass of 90% pure MnO2 to produce 35.5 gm of Cl2 according to following
reaction.
MnO2 + 4HCl  MnCl2 + Cl2 + 2H2O
Solution :
MnO2 + 4HCl    MnCl2 + Cl2 + 2H2O
87 gm 71 gm
 71 gm Cl2 is produced by 87 gm of MnO2

87  35.5
 35.5 g Cl2 is produced by =  43.5 g MnO
71 2

 90 gm pure MnO2 is present in 100 gm sample

100  43.5
 43.5 gm pure MnO2 =  48.3 gm
90

Ex a m p l e 8
KClO3 decomposes on heating in two possible ways as

KClO3  KCl  3 / 2 O 2


4KClO3  KCl  3KCl O 4

when 15 g of KClO3 were heated in an experiment, analysis of the product showed that 5.21
g of KCl and 4.59 g of KClO4 were formed. What was the weight of KClO3 remaining
undecomposed ? What fraction of the KClO3 decomposed yielded KClO4 ?
Solution :
In order to calculate the weight of KClO3 remaining undecomposed, we need to find out the total
weight of KClO3 that decomposes. The weight of KClO3 that decomposes in second reaction would
be known using the KClO4 data. Using KClO4 data, we would also calculate the weight of KCl
obtained in second reaction. Since, total weight of KCl obtained is known to us, so the weight of
KCl obtained in first reaction would also be known, using this we will find out the weight of KClO3
decomposed in first reaction.

4.59
Moles of KClO4 obtained =
138.5

4.59 1
Moles of KCl obtained in IInd reaction =   0.011 (since molar ratio of KClO and KCl
138.5 3 4

in IInd reaction is 3 : 1)

STOICHIOMETRY
12 QUIZRR

4.59 4
Moles of KClO3 that decomposes in IInd reaction =  (since molar ratio of KClO4 and
138.5 3
KClO3 in IInd reaction is 3 : 4)

4.59 4
 Mass of KClO3 that decomposes in IInd reaction =  ï 122.5 = 5.41 g
138.5 3

 5.21 
Moles of KCl obtained in Ist reaction =   0.011   0.07  0.011  0.059
 74.5 
Moles of KClO3 that decomposes in Ist reaction = 0.059
(since molar ratio of KCl and KClO3 in Ist reaction is 1 : 1)
Mass of KClO3 that decomposes in Ist reaction = 0.059 ï 122.5 = 7.23 g

Ex a m p l e 9
In a process for producing acetic acid, oxygen gas is bubbled into acetaldehyde, CH3CHO,
under pressure at 60ĈC in presence of suitable catalyst.
2CH3CHO (l) + O2 (g)  2CH3COOH (l)
In a laboratory test of this reaction, 20.0 g CH3CHO and 10 g O2 were put into a reaction
vessel.
(a) How many grams of acetic acid can be produced by these amounts of reactants ?
(b) How many gram of the excess reactant remain after the reaction is complete ?
(c) If actual yield is 23.8 g, calculate percentage yield.
Solution :

20 g
(a) 20 g CH3CHO =  0.455 mol
44 g mol 1

10 g
10 g O2 =  0.313 mol
32 g mol 1

CH3COOH obtained from 0.455 mol. of CH3CHO = 0.455 mol


CH3COOH obtained from 0.313 mol of O2 = 0.313 ï 2 = 0.626 mol
Thus CH3CHO is the limiting reactant.
Hence CH3COOH formed = 0.455 mol
= 0.455 ï 60 = 27.3 g
(b) O2 is in excess

0.455
0.455 mol of CH3CHO will react with mol of O2
2

= 0.2275 mol of O2

STOICHIOMETRY
QUIZRR 13

O2 left unreacted = 0.313 ă 0.2275


= 0.0855 mol
= 2.736 g O2

actual yield
(c) Percentage yield = thheoretical yield ï 100

23.8
=  100 = 87.2%
27.3

Ex a m p l e 1 0
Titanium, which is used to make airplane engines and frames, can be obtained from titanium
tetrachloride, which in turn is obtained from titanium oxide by the following process :
3TiO2 (s) + 4C (s) + 6Cl2 (g) · 3TiCl4 (g) + 2CO2 (g) + 2CO (g)
A vessel contains 4.15 g TiO2, 5.67 g C and 6.78 g Cl2 suppose the reaction goes to
completion as written, how many grams of TiCl4 can be produced ? (Ti = 48)
Solution :
3TiO2 (s) + 4C (s) + 6Cl2 (g)  3TiCl4 (g) + 2CO2 (g) + 2CO (g)
Firt we determine the quantity of TiCl4 from each reactant.
4.15 g TiO2 = 0.052 mol of TiO2 = 0.052 mol of TiCl4
5.67 g C = 0.4725 mol of C = 0.3543 mol of TiCl4
6.78 g Cl2 = 0.0955 mol of Cl2 = 0.0477 mol of TiCl4
Thus Cl2 is limiting reactant.
TiCl4 obtained = 0.0477 mol = 9.072 g.

Ex a m p l e 1 1
Calculate the weight of FeO produced from 2 g VO and 5.75 g of Fe2O3. Also report the
limiting reagent.
VO + Fe2O3  FeO + V2O5
Solution Balancing the given equation
2VO + 3Fe2O3  6FeO + V2O5
2 5.75
Mole before reaction
67 160
= 0.0298 0.0359 0 0

 0.0359  2  1
Mole after reaction  0.0298  0 0.0359 ï 2 0.0359 
 3  3
 Mole of FeO formed = 0.0359 ï 2
 Weight of FeO formed = 0.0359 ï 2 ï 72
= 5.17 g

STOICHIOMETRY
14 QUIZRR

Ex a m p l e 1 2
A mixture of FeO and Fe3O4 when heated in air to a constant weight, gains 5% in its weight.
Find the composition of the initial mixture.

Solution :
When FeO and Fe3O4 are heated, both change to Fe2O3. Let the weights of FeO and Fe3O4 be x g and
y g respectively.
 total weight of reactant = (x + y) g
Since weight increases 5% on heating,
 when FeO and Fe3O4 change completely to Fe2O3,

105
the wt. of Fe2O4 =   x  y  1.05  x  y g
100

Now,
FeO + Fe3O3  Fe2O3
xg yg 1.05 (x + y) g,
Now for Fe atom,
moles of Fe in FeO + moles of Fe in Fe3O4 = moles of Fe in Fe2O3
1 ï moles of FeO + 3 ï moles of Fe3O4 = 2 ï moles of Fe2O3

 In this example, since we do not know 


 the balanced equation, but all the   FeO  72 
 
 products and reactant contain Fe,  x 3y 2  1.05  x  y  
Fe3 O4  232 
 thus we can directly balance Fe atom   
 and solve it 72 232 160  Fe O  160 
  2 3 

Dividing by y, we get

1 x 3 2  1.05 x 2  1.05
  =  
72 y 232 160 y 160

x 1 2.1  2.1 3
    
y  72 160  160 232

81
  100  20.02%
% of FeO =
 81  319 

319
and % of Fe3O4 =  100 = 79.98%.
 81  319 

STOICHIOMETRY
QUIZRR 15

Ex a m p l e 1 3
Crude CaC2, is made in an electric furnace by the following reaction :
CaO + 3C  CaC2 + CO 
The product contains 85% CaC2 and 15% unreacted CaO.
(a) How much CaO is to be added to the furnace charge for each 1000 Kg of pure CaC2
produced ?
(b) How much CaO is to be added to the furnace charge for each 1000 Kg of crude
product ?
Solution :
(a) for pure CaC2 (1000 Kg)
1 mole CaC2  1 mol of CaO
 1000/64 mole  1000/64 mole of CaO
(1000/64) ï 56 kg of CaO
Also, the product (final) contains 15% unreacted CaO.
for 85 kg of CaO (pure) produced, 15 kg is CaO. (unreacted)

 15 
1000 kg of CaO    1000  kg of unreacted CaO
 85 

 1000   15 
total amount of CaO to be used =   56     1000  kg
 64   85 

= 875 + 176.47 = 1051.47 kg


(b) for crude CaC2 (1000 kg)
In 1000 kg crude, there will be 150 kg CaO & 850 pure CaC2.
so amount of CaO to be used

850
= 150   56
64

= 150 + 743.75 = 893.75 kg.

2 .5 V OL U MET RI C A N A L Y SI S
The volumetric analysis is an analytical method of estimating the concentration of a substance in
a solution by adding exactly same number of equivalent of another substance present in a solution of
known concentration.
Titration : The process of determination of concentration of a solution of unknown concentration
with the help of a solution of known concentration i.e., standard solution, is known as Titration.
Before approaching towards the depth of voumetric analysis one must know the following terms
very clearly.

STOICHIOMETRY
16 QUIZRR

Molarity (M) : The no. of moles of solute present in one litre of solution is called the molarity (M).

No. of moles of solute


Molarity 
Volume of solution in litre

Wt. of solute/Molecular wt. of solute


or Molarity 
Volume of solution in litre

So, no. of moles of solute = Volume of solution (in litre) ï Molarity of solution
Molality (m) : No. of moles of solute present in one kilogram of solvent is know as molality.

No. of moles of solute



Molality (m) = Wt. of solvent in kg

Normality (N) : No. of equivalents of solute present in one litre of the solution is known as
Normality (N).

No. of equivalents of solute


Normality (N) =
Volume of solution in litre

Wt. of solute/equivalent weight of solute


=
Volume of solution in litre

No. of equivalent of solute = Volume of solution (in litre) ï normality of solution


No. of milli equivalents of solute = Normality of solution ï Volume of solution in millilitres
Equivalent Weight : Number of parts by mass of an element which reacts or displaces from a
compound 1.008 parts by mass of hydrogen, 8 parts by mass of oxygen and 35.5 parts by mass of
chlorine, is known as the equivalent weight of that element e.g.
(1) 2Mg + O2  2MgO
48g 32g
12g 8g
 32g of O2 reacts with 48g of Mg

48  8
 8g of O2 =  12g
32
 Equivalent weight of Mg = 12
Similarly, Zn + H2SO4  ZnSO4 + H2
65.5g 2 ï 1.00g
 2 ï 1.008 g of hydrogen is displaced by 65.5 g

65.5
 1.008 g of H =  32.75g
2  1.008

STOICHIOMETRY
QUIZRR 17

65.5
 Equivalent weight of Zn =  32.75
2

3
(2) Al  Cl2  AlCl3
2

3
32g ï 71 g  111.5 g chlorine reacts with 27g of Al
2

27  35.5
 35.5 g chlorine reacts with  9.0 g of Al
111.5

27
 Equivalent weight of aluminium =  9.0
3
As we can see from the above examples that equivalent weight is the ratio of atomic weight and
a factor (say n-factor) which is in above three cases is their respective valencies. So,

atomic weight
equivalent weight =
n  factor

In a similar way, the equivalent weight of acid/base is the ratio of molecular weight and the
basicity/acidity and for oxidizing agents and reducing agent it is the ratio of molecular weight and
the number of moles of electrons gained or lost. So in case of acid/base the n-factor is basicity/acidity
(i.e. number of dissociable H+ ions/number of dissociable OHă ion and in case of oxidizing agent/
reducing agent, n-factor is number of moles of electrons gained/lost per mole of oxidizing agent/
reducing agent. Therefore, in general, we can write.

Atomic or molecular weight


Equivalent weight  E  
n  factor

Wt W W
No. of equivalents of solute   
Eq. wt. E M/n

No. of equivalents of solute = No. of moles of solute ï n-factor

And also Normality = n-factor ï molarity of solution

2 .6 n –FA CT OR CA L CU L A T I ON
2 .6 .1 I n N o n -Re d o x Re a c t io n

1. Ac ids
For acids, n-factor is defined as the number of H+ ions replaced by 1 mole of acid in a reaction.
Note : That the n-factor for acid is not equal to its basicity; i.e. the number of moles of replaceable
+
H atoms present in one mole of acid.

STOICHIOMETRY
18 QUIZRR

For example, năfactor of HCl = 1,


năfactor of HNO3 = 1,
năfactor of H2SO4 = 1 or 2, depending upon extent of reaction it undergoes.
H2SO4 + NaOH  NaHSO4 + H2O
Although one mole of H2SO4 has 2 replaceable H atoms but in this reaction H2SO4 has given only
one H+ ion, so its năfactor would be 1.
H2SO4 + 2NaOH  Na2SO4 + 2H2O
The năfactor of H2SO4 in this reaction would be 2.
Similarly, năfactor of H2SO3 = 1 or 2
năfactor of H2CO3 = 1 or 2
năfactor of H3PO4 = 1 or 2 or 3
năfactor of H3PO3 = 1 or 2 because one of the H is not replaceable in H3PO3.

O
||
P
This can be seen using its structure H  O | O  H . The H atoms which are linked to oxygen are
H
replaceable while the H atom linked directly to central atom (P) is non replaceable.
năfactor of H3BO3 = 1
In H3BO3, although all three H are linked to oxygen, yet all 3 H are not replaceable. Here boron
atom is electron deficient, so its acts as a Lewis acid. When H3BO3 oxygen atom of H2O through its lone
pair attack the boron atom, as follows
OH OH H OH
| .. | | .. |
 HO  B  OH  H3 O
H2O
..

B< + OH2  HO  B  O  H 


..
..

 |
HO OH OH
The net reaction is H3BO3 + 2H2O  [B(OH)4]ă + H3O+
Thus, one mole of H3BO3 in solution gives only one mole of H+, so its năfactor is 1.

2. Bases
For bases, năfactor is defined as the number of OHă ions replaced by 1 mole of base in a reaction.
Note that năfactor is not equal to its acidity i.e. the number of moles of replaceable OHă ions present
in 1 mole of base.
For example, năfactor of NaOH = 1
năfactor of Ca(OH)2 = 1 or 2
năfactor of Al(OH)3 = 1 or 2 or 3
năfactor of NH4(OH) = 1

STOICHIOMETRY
QUIZRR 19

2 .6 .2 I n Re d o x Re a c t io n s
1 . Sa l t s i n w h i c h n o a t o m u n d e r g o e s c h a n g e i n o x i d a t i o n s t a t e i n r e a c t io n s
The năfactor for such salts is defined as the total moles of cationic/anionic charge replaced in 1
mole of the salt. For the reaction,
Na3PO4 + BaCl2  NaCl + Ba3(PO4)2
To get one mole of Ba3(PO4)2, two moles of Na3PO4 are required, which means six moles of Na+
are completely replaced by 3 moles of Ba2+ ions. So, six moles of cationic charge is replaced by 2 moles
of Na3PO4, thus each mole of Na3PO4 replaces 3 moles of cationic charge. Hence, năfactor of Na3PO4
in this reaction is 3.

2 . Sa l t s w h ic h re a c t in a m an n e r t h a t o n l y on e a t o m u n d e r g o e s c h a n g e in o x i da t i o n
st at e and goes in only one produc t
The năfactor of such salts is defined as the number of moles of electrons exchanged (lost or gained)
by one mole of the salt.
Let us have a salt AaBb in which oxidation state of A is + x. It changes to a compound, which has
atom D in it. The oxidation state of A in AcD be + y.
+x y
A a Bb  Ac D

The năfactor of AaBb is calculated as


n = |ax ă ay|
To calculate the năfactor of such salts, we apply following method :
1. Take one mole of rectant whose oxidation state is changing.
2. Multiply this with oxidation state of element in reactant to give us total oxidation state of
element in reactant.
3. Calculate total oxidation state of the same element in the product for same number of moles
of atom in reactant.
Another similar case would be that the salt has only one atom undergoing change in oxidation state
but going into two different products with the same oxidation state. Try it yourself.
For example, let us calculate the năfactor KMnO4 for the given chemical change.

H
KMnO4  Mn 2

In this reaction, oxidation state of Mn changes from + 7 to + 2. Thus; KMnO4 is acting as oxidising
agent, since it is reduced.
 năfactor of KMnO4 = |1 ï (+ 7) ă 1 ï (+ 2) = 5
Similarly

(a) H 2O
KMnO4   Mn 4

năfactor of KMnO4 = |1 ï (+ 7) ă 1 ï (+ 4) = 3

STOICHIOMETRY
20 QUIZRR


(b) OH
KMnO4   Mn 6

năfactor of KMnO4 = |1 ï (+ 7) ă 1 ï (+ 6)| = 1


It can be seen that in all the above chemical changes, KMnO4 is acting as oxidising agent, yet its
năfactor is not same in all reactions. Thus, the năfactor of a compound is not fixed, it depends on
the type and the extent of reaction it undergoes.

Ex a m p l e 1 4
Calculate the năfactor of reactants in the given chemical changes ?


(a) H
K 2 Cr2 O7  Cr 3

(b) C2 O 24  CO 2

S 2 O 23  
 SO 24
alkaline
(c)

(d) I   ICl

Solution :

+6 
(a) H
K 2 Cr 2 O7  Cr3 

The oxidation state of Cr changes from + 6 to + 3.


 n = |2 ï (+ 6) ă 2 ï (+ 3)| = 6

+3 +4
(b) C2 O24  CO2

Carbon get oxidized from + 3 to + 4


 n = |2 ï (+ 3) ă 2 ï (+ 4)| = 2

+2 +6
(c) S 2 O23  
alkaline
 S O24 

The oxidation state of sulphur changes from + 2 to + 6


 n = |2 ï (+ 2) ă 2 ï (+ 6)| = 8

+1
(d) I   ICl

Iă get oxidized to I+.


 n = |1 ï (ă 1) ï (+ 1)| = 2

STOICHIOMETRY
QUIZRR 21

3 . Sa l t s w h i c h re a c t in a m a n n e r t h a t o n l y o n e a t o m u n d e r g o e s c h a n g e i n o x i da t i o n
s t a t e b u t g o e s i n t w o p r o d u c t s w it h d i f f e r e n t o x i d a t i o n s t a t e (d i f f e r e n t t h a n i n
t he reac t ant ) as a result of eit her ox idat ion or reduc t ion.
Let the chemical change be
x +y +z
A a B b  A c D + A e F

The number of moles of electrons lost or gained by one mole of AaBb would depend on the fact that
how much of A underwent change to oxidation state + y and how much of A underwent change to
oxidation state + z. năfactor calculation in such cases can be done as follows.

Let us take a chemical change 2Mn 7  Mn 4  Mn 2 out of the two moles of Mn+7, one mole
Mn+7 changes to Mn+4 by gaining 3 mole of electrons and the other mole of Mn+7 changes to Mn+2
by gaining 5 mole of electrons, so in all 8 mole of electrons are gained by 2 mole of In+7. So each
mole of Mn+7 has gained 8/2 = 4 mole of electrons. Thus, 4 would be the năfactor on Mn+7 in this
reaction.
If the reaction would have been

3Mn 7  2Mn 2  Mn 4

Out of 3 moles of Mn+7, two moles of Mn+7 changes to Mn+2 by gaining 10 mole of electrons and
one mole of Mn+7 changes to Mn+4 by gained 3 mole of electrons. Thus each mole of Mn+7 have
gained 13/3 mole of electrons. Therefore, the năfactor of Mn+7 in this reaction would be 13/3.
ÂÂNote that năfactor can be a fraction because it is not the number of electrons exchanged but it
is the number of moles of electrons exchanged which can be a fraction.ÊÊ

4 . Sa l t s t h a t r e a c t i n a m a n n e r t h a t t w o t yp e o f a t o m s i n t h e s a l t u n d e r g o c h a n g e
i n o x i d a t i o n s t a t e (b o t h t h e a t o m s a r e e i t h e r g e t t i n g o x i d i s e d o r r e d u c e d ).
Let the change be represented as
x y
A a Bb  A c D  E f B z

In such cases, the năfactor of the compound would be the sum of individual năfactors of A and B.
năfactor of A = |ax ă ay|
năfactor of B = |ax ă bz| because the total oxidation state of ÂbÊ BÊs in the reactant is ă ax (as the
total oxidation state of ÂaÊ AÊs in the reactant is + ax and the total oxidation state of y BÊs in the
product is bz.
 năfactor of AaBb = |ax ă ay|+ |ax ă bz|
In general, the năfactor of the salt will be the total number of mole of electrons lost or gained by
one mole of the salt.
+4
For example, we have a reaction, Cu1 S2  Cu 2  S O
2 2

in which Cu+ and S2ă both are getting oxidised to Cu2+ and S+4 respectively.
 năfactor of Cu2S = 2 ï (+1) ă 2 ï (+2) + 1 ï (ă2) ă1 ï (+4) = 8

STOICHIOMETRY
22 QUIZRR

5 . Sa l t s t h a t r e a c t i n a m a n n e r t h a t t w o a t o m s i n t h e s a l t u n d e r g o e s c h a n g e i n
o x i d a t i o n s t a t e (o n e a t o m i s g e t t i n g o x i d i s e d a n d t h e o t h e r i s g e t t i n g r e d u c e d ).
The năfactor of such a salt can be calculated either by taking the total number of moles of electrons
lost or total number of mole of electrons gained by one mole of the salt.

+5 2 1 0
K ClO 3  KC l  O2

In this reaction, O2ă is getting oxidised to O2 and Cl+5 is getting reduced to Clă1. In each case, 6
mole of electrons are exchanged whether we consider oxidation or reduction.
năfactor of KClO3 considering oxidation = |3(ă 2) ă 3 (0)| = 6
or năfactor of KClO3 considering reduction = |1 ï (+ 5) ă 1 ï (ă 1) = 6

6 . Sa l t s o r c o m p o u n d s w h i c h u n d e r g o e s d i s p r o p o r t i o n a t io n r e a c t io n
Disproportionation reactions are the reactions in which oxidising and reducing agents
are same or the same element from the same compound is getting oxidised as well as
reduced. năfactor of a disproportionation reaction can only be calculated using a balanced chemical
reaction. We will categorize disproportionation reactions into two types.
(a) Disproportionation reactions in which moles of reducing and oxidizing agents are the same.
In such compounds we either calculate the năfactor for reducing agent or oxidizing agent since
calculating both would be simply a waste of time.
For example, 2H2O2  2H2O + O2
Out of the 2 mole of H2O2 used in reaction, one mole of H2O2 gets oxidised to O2 (oxidation state
of O changes from ă 1 to 0) while the other mole of H2O2 gets reduced to H2O (oxidation state of
O changes from ă 1 to ă 2). When 1 mole of H2O2 gets oxidised to O2, the half-reaction would be

O22  O02  2e  and when 1 mole of H2O2 gets reduced to H2O, the half-reaction would be

O22  2e   2O2

Thus, it is evident that one mole of H2O2 (which is either getting oxidised or reduced) will lose or
gain 2 mole of electrons. Therefore năfactor of H2O2 as oxidizing as well as reducing agent in this
reaction is 2. Thus

H2O2 + H2O2  2H2O + O2

Reducing agent Oxidizing agent


(n = 2) (n = 2) (n = 1) (n = 2)

(b) Disproportionation reactions in which moles of compound getting oxidised and reduced are not
same i.e. moles of oxidising agent agent and reducing agent are not same.

STOICHIOMETRY
QUIZRR 23

For example,
6Br2 + 12 OHă  10Bră + 2BrO3ă + 6H2O
In this reaction, the mole of electrons lost by the oxidation of some of the moles of Br2 are same
as the number of mole of electrons gained by the reduction of rest of the moles of Br2. Of the 6
moles of Br2 used, one mole is getting oxidized, loosing 10 electrons (as reducing agent) and 5 moles
of Br2 are getting reduced and accepts 10 moles of electron (as oxidizing agent).
Br2     · 2Br+5 + 10eă
5Br2 + 10eă  · 10Bră
Br2 + 5Br2 · 10Bră + 2Br+5

Reducing agent Oxidizing agent


(n = 10) (n = 2) (n = 1) (n = 5)

Thus, năfactor of Br2 acting as oxidizing agent is 2 and that Br2 acting as reducing agent has nă
factor 10.
Or when the reaction is written as
6Br2   10Bră + 2Br+5
where, Br2 is not distinguished as how much of it functions as oxidizing agent and how much as
reducing agent, then for calculating năfactor of compound in such reactions, first find the total
number of mole of electrons exchanged (lost or gained) using the balanced equation and divide it
with the number of mole of Br2 involved in the reaction to get the number of mole of electrons
exchanged by one mole of Br2.
In the overall reaction, the number of mole of electrons exchanged (lost or gained) is 10 and the
moles of Br2 used in the reaction are 6. Thus, each mole of Br2 has exchanged 10/6 or 5/3 mole of
electrons. Therefore, the năfactor of Br2 when the reaction is written without segregating oxidising
and reducing agent is 5/3.
6Br2  10Bră + 2Br+5
(n = 5/3) (n = 1) (n = 5)

2 .1 0 L A W OF EQU I V A LA N CE A N D DI LU T I ON EFFECT
L a w o f Eq u i v a l e n c e
It is not practically possible to remember all the chemical reactions in balanced forms though that
is how we require them to be for solving problems.
Hence we use the law of equivalence. This law helps us determine the molar ratio of reactants and
products. It can be stated as :
According to the law of equivalence, whenever two substances react, the equivalents of
one will be equal to the equivalents of other and the equivalents of any product will also
be equal to that of the reactant.

STOICHIOMETRY
24 QUIZRR

Law of equivalence
For a reaction
aA + bB  mM + nN
eq. of A = eq. of B = eq. of M = eq. of N

wt. of A wt. of B wt. of M wt. of N


or   
EA EB EM EN

Let us suppose we have a reaction, A + B  C + D. In this reaction, the number of moles of


electrons lost by 1 mole of A are x and the number of mole of electrons gained by 1 mole of B are
y. Since, the number of mole of electrons lost and gained are not same, the molar ratio in which
A & B react cannot be 1 : 1. Thus, if we take y moles of A, then the total moles of electrons lost
by y moles of A would be (x ï y). Similarly, if x moles of B are taken, then the total mole of
electrons gained by x moles of B would be (y ï x). Thus, the number of electrons lost by A and
number of electrons gained by B becomes equal. For reactant A, its năfactor is x and the number
of moles used are y. So,
The equivalents of A reacting = moles of A reacting x năfactor of A = y ï x
Similarly, for reactant B, its năfactor is y and the number of moles used are x. So,
The equivalents of B reacting = moles of B reacting ï năfactor of B
=xï y
Thus, the equivalents of A reacting would be equal to the equivalents of B reacting. Thus, the
balancing coefficients of the reactant would be as
yA + xB  C + D
(năfactor = x) (năfactor = y)
The năfactor of A & B are in the ratio of x : y, and their molar ratio is y : x. Thus, molar ratio
is inverse of the năfactor ratio.
In general, whenever two substances react with their năfactors in the ratio of a : b, then
their molar ratio in a balanced chemical reaction would be b : a.
To get the equivalents of a substance, its năfactor is to be known. Let the weight of the substance
used in the reaction be w g.
w w w
Then, equivalents of substance reacted would be or M /n  M  n (where E and M1 are the
E 1 1
equivalent mass and molar mass of the substance) Thus, in order to calculate the equivalents of
a substance, knowledge of năfactor is a must.
When a solution is diluted, the moles and equivalents of solute do not change but molarity and
normality change while on taking out a small volume of solution from a larger volume, the molarity
and normality of solution do not change but molar and equivalents change proportionately. This
concept is useful in solving dilution problems.

STOICHIOMETRY
QUIZRR 25

2 .1 1 So m e I m p o r t a n t Fo r m u l a e

gB
1. moles = MVlt  (M = Molarity of solution
MB

V = Volume of solution_

millimoles = MVcc  g B  100 gB/MB = Gram equivalent of substance


MB

gB
2. gm eq. = NVlt  N = Normality of solution
EB

V = volume

gB gB
meq = NVcc   1000 = Gram equivalents
EB EB

103  d 103  d
3. Molarity (M) = Normality (N) =
M0 E

d : density of solution in g/cc; x : %age strength of solution (by weight)


M0 = Molar mass of substance
4. N=xM
x : acidity for base or basicity for acid or electron transfer/mole for O.A and R.A
N = Normality
M = Molarity

1000  xB
5. m = M x
A A

m = molarity
xB = mole fraction of B
xA = mole fraction of A
6. Strength of a solution is generally expressed as grams of solute in one litre of solution.

gB
Strength =
litres of solution

Strength = NE = M M0

STOICHIOMETRY
26 QUIZRR

Ex a m p l e 1 5

N
How many milli equivalent of H2SO4 in 10 ml. of H 2SO 4 .
10

Solution :
no. of equivalent = NV (in lt)
no. of m.eq. = NV ï (100)

=
1
10
 
 10  10 3  1000

m.eq. = 1

Ex a m p l e 1 6
What volume of 0.1 M KMnO4 in acidic medium will be required to react completely with 100
ml; 0.1 M NaHC2O4.
Solution :
In redox reaction, n-factor of NaHC2O4 is 2.8 and that of KMnO4 is 5.
Therefore,
From law of equivalence,
(N1 V1) KMnO4 = (N2 V2)NaHC2O4
0.1 ï 5 ï V1 = 0.1 ï 2 ï 100  V1 = 40 ml

Ex a m p l e 1 7
What volume of 0.1 NaOH will be required in previous problem.
Solution :
NaOH + NaHC2O4 Acid base reaction
Here, n-factor of NaHC2O4 = 1
(N1 V2) NaHC2O4 = (N2 V2) NaOH [from law of equivalence]
 100 ï 0.1 = .1 ï V
V2 = 100 ml
N1 = Normality of NaHC2O4
N2 = Normality of NaOH
V1 = Volume of NaHC2O4
V2 = Volume of NaOH

STOICHIOMETRY
QUIZRR 27

Ex a m p l e 1 8
What is the normality and nature of a mixture obtained by mixing 0.62 g of Na2C2O3. H2O
to 100 mL of 0.1 N H2SO4 ?
Solution :

0.62 w 
Meq. of Na2CO3.H2O =  100   1000  Meq. 
62  E 

Meq. of H2SO4 = 100 ï 0.1


Na2CO3 + H2SO4  Na2SO4 + H2O + CO2 
Meq. added 10 10 0 0 0
Meq. left 0 0 10 10 10

10
 N Na2SO4   0.1
100

Solution becomes neutral since both acid and base are used up and Na2SO4 does not show hydrolysis.

Ex a m p l e 1 9
A sample of an alloy weighing 0.50 g and containing 90% Ag was dissolved in conc. HNO3. Ag
was analysed by Volhard method in which 25 mL of KCNS were required for complete
neutralization. Determine normality of KCNS.
Solution :

90  0.5
Wt. of Ag =  0.45
100

0.45
 Meq. of Ag =  1000  4.17
108

HNO3 KCNS
NowAg   AgNO3   AgCNS

 Equal Meq. reacts and therefore

Meq. of KCNS = Meq. of AgNO3

= Meq. of HNO3 = Meq. of Ag

N ï 25 = 4.17

N = 0.167

STOICHIOMETRY
28 QUIZRR

Ex a m p l e 2 0
HNO3 used as a reagent has specific gravity of 1.42 g mLă1 and contains 70% by strength
HNO3. Calculate,
(a) Normality of acid
(b) Volume of acid that contains 63 g pure acid
(c) Volume of water required to make 1N solution from 2 mL conc. HNO3.
Solution : (a) Strength of HNO3 = 70%
i.e. Volume of solution = 100 mL
and Volume of HNO3 = 70 mL
 Wt of HNO3 in solution = 70 ï 1.42 g

70  1.42  Eq. 
 Eq. of HNO3 in solution =  N = 
63  100 /1000  V in litre 

= 15.78
(b)  70 ï 1.42 g of pure acid is present in 100 mL

100  63
 63 g of pure acid present in = 70  1.42

= 63.38 mL
(c) Meq. of conc. HNO3 = Meq. of dil. HNO3
 2 ï 15.78 = V ï 1
V = 31.56 mL
 Volume of water added = 31.56 ă 2 = 29.56 mL

2 .1 2 V o l u m e t ri c A n a l y s is
Now, we have developed enough platforms to understand the law of equivalents and volumetric
before analysis.
The volumetric analysis is an analytical method of estimating the concentration of a
substance in a solution by adding exactly same number of equivalents of another
substance present in a solution of known concentration.
This is the basic principle of titration. Volumetric analysis is also known as titrimetric analysis.
The substance whose solution is employed to estimate the concentration of unknown solution is
called titrant and the substance whose concentration is to be estimated is called titrate.
The volumetric analysis is divided into following types :
(a) Simple titrations
(b) Back titrations
(c) Double titrations

STOICHIOMETRY
QUIZRR 29

SI MPL E T I T RA T I ON
A known volume of the solution of unknown concentration is taken in a flask and required reagents
are added to it. The solution of known concentration is added from the burette in the solution of
unknown concentration till the latter reacts completely. This process is called titration. At the end
point (equivalence point) the equivalents or milliequivalents of the two reacting substances are
equal.
Volume of solution (A) = VA litres
Normality of solution (A) = NA
Equivalents of substance (A) = NA VA
Similarly, equivalents of substance (B) = NB VB
At the equivalence point (end point) the equivalents (not the moles) of the two substances equal.
NA VA (litre) = NB ï VB (litre)

Ex a m p l e 2 1
40 ml N/10 HCl and 60 ml N/20 KOH are mixed together. Calculate the normality of the acid
or base left. What is the normality of the salt formed in the solution?
Solution :

1  40
Milli equivalents of HCl = N ï V (ml) = 4
10

1  60
Milli equivalents of KOH = N ï V (ml) = 3
20

One milli equivalent of an acid neutralizes one milli equivalent of a base


Milli equivalent of HCl left = 4 ă 3 = 1
Total volume of the solution = 40 + 60 = 100 ml
Milli equivalents of HCl = N ï V (ml)
1 = N ï 100
Normality (N) of HCl left in solution = 0.01
Salt formed = Milli equivalent of acid or base neutralized
Milli equivalents of the salt formed = N ï V (ml)
3 = N ï 100
Normality (N) of salt formed = 0.03

STOICHIOMETRY
30 QUIZRR

DOU B L E T I T RA T I ON
If an aqueous solution contains a mixture of any two of the three NaOH, NaHCO3 and Na2CO3 and
it has to be titrated against an acid HCl or H2SO4, it will require two indicators to determine the
strength of the bases present. The two indicators used are phenophthalein and methyl orange.

M et ho d
A given volume of the aqueous solution of the bases is taken and phenophthalein indicator is added
to it. This solution is titrated with an acid of known normality to the end point the volume of the
acid used to noted. This end point is called first end point. Now in the same solution methyl orange
is added and gain titrated with an acid of known strength to the end point. It is called second end
point. The volume of acid, used in the second end point is also noted. The normality of the bases
present is then calculated.
Principle : In the presence of phenophthalein indicator
(i) NaOH is completely neutralized by the acid
(ii) Only half of the milliequivalents of Na2CO3 present are titrated as the phenophthalein will
show the colour change when only NaHCO3 (weak base) and neutral species are left in the
reaction mixture. The following reactions take place,

NaOH + HCl  NaCl + H2O

Na2CO3 + HCl  NaHCO3 + NaCl


weak base

Since phenolphthalein is a weak organic acid, and it changes its colour in weakly basic medium
(NaHCO3), so as soon as the Na2CO3 is converted to NaHCO3, phenophthalein shows the colour
change indicating the completion of the reaction.
In the presence of methyl orange, all the basic structures, left in the mixture will be neutralized
by acid and methyl orange will show the colour change when the medium is weakly acidic
(H2O + CO2 i.e. H2CO3).
1. Titration of the solution containing both NaOH and Na2CO3 : A given volume of the
aqueous solution is titrated with an acid of known normality using phenophthalein indicator.
Suppose ÂaÊ milli equivalents of acid are used in the first end point then,
milli equivalents of NaOH + ó milli equivaent of Na2CO3
= milli equivalent of acid = a .... (1)
Now in the same already titrated solution methyl orange indicator is added and again
titrated to the end point. Suppose ÂbÊ milli equivalents of the acid are used at the second end
point.
ó milli equivalents of Na2CO3 = milli equivalents of acid = b ...... (2)

STOICHIOMETRY
QUIZRR 31

From equation (1) and (2)


Milli equivalents of acid used by Na2CO3 = 2b
= milli equivalents of Na2CO3
Milli equivalents of acid used by NaOH = a ă b
= milli equivalent of NaOH
Knowing the milli equivalents of Na2CO3 or NaOH and the volume of the solution titrated,
their normality can be calculated.

Ex a m p l e 2 2
NaOH and Na2CO3 are dissolved in 200 ml aqueous solution. In the presence of phenophthalein
indicator, 17.5 ml of 0.1 N HCl are used to titrate this solution. Now methyl orange is added
in the same solution titrated and it requires 2.5 ml of the same HCl. Calculate the normality
of NaOH and Na2CO3 and their mass present in the solution.
Solution :
Milli equivalents (a) of HCl used in the presence of phenolphathalein indicator
= N ï V (ml)
= 0.1 ï 17.5 = 1.75
1.75 (a) = milli eq. of NaOH + ó milli eq. of Na2CO3 .... (1)
Milli eq. (b) of HCl used in the presence of methyl orange indicator
= N ï V (ml)
= 0.1 ï 2.5 = 0.25
0.25 (b) = ó milli equivalents of Na2CO3 ... (2)
For Na2CO3 solution : From equation (2)
Milli eq. of acid used by Na2CO3 = 2b
= 2 ï 0.25 = 0.5
Volume of Na2CO3 solution = 200 ml
Suppose, Normality of Na2CO3 = N
Milli equivalents of Na2CO3 = N ï V (ml) = 200 N
Putting equivalents of acid and Na2CO3 equal
200N = 0.5

1
or (Normality of Na2CO3 solution) N =
400

Mass of Na2CO3 = N ï E ï V (litre)

STOICHIOMETRY
32 QUIZRR

1
(E for Na2CO3 = 53) =  53  0.2
400

= 0.0265 gram
For NaOH solution : From equation (1) and (2)
Milli eq. acid used by NaOH = a ă b = 1.75 ă 0.25
= 1.50
Volume of NaOH solution = 200 ml
Suppose, Normality of NaOH solution = N
Milli eq. of NaOH = N ï V (ml) = 200 N
Putting the milli eq. of NaOH and acid used equal
200 N = 1.5

1.5
(Normality of NaOH solution) N =
200

Mass of NaOH = N ï E ï V (litres)

1.5
=  40  0.2
200

(E for NaOH = 40) = 0.06 g

2. Titration of the solution containing both Na2CO3 and NaHCO3 : Given volume of the solution
is titrated by an acid using phenolphthalein indicator. Suppose ÂaÊ milli equivalents of acid are used
in the first end point. Then ó milli equivalent of Na2CO3 = milli equivalents of acid ... (1)
Now in the same already titrated solution methyl orange indicator is added and again titrated to
the end point : Suppose ÂbÊ milli equivalents of the acid are used at the second end point. Then

ó milli equivalents of Na2CO3 + milli equivalents of NaHCO3


= milli equivalents of acid = b ... (2)
From equation (1) and (2)
Milli equivalents of acid used by Na2CO3 = 2a
= milli equivalents of Na2CO3
Milli equivalents of acid used by NaHCO3 = b ăa
= milli equivalent of NaHCO3
Knowing the milli equivalents of the base and volume of the solution titrated, the normality
(strength) of the bases can be calculated.

STOICHIOMETRY
QUIZRR 33

Ex a m p l e 2 3
Both Na2 CO 3 and NaHCO 3 are present in an aqueous solution. In the presence of
phenolphthalein indicator 10 ml of this solution requires 2.5 ml of 0.1 M H2SO4 for titration.
After this methyl ornge is added in the same solution and titration requires 5 ml M H2SO4.
Calculate the concentration of Na2CO3 and NaHCO3 in g/litre.
Solution :
Milli eq. (a) of H2SO4 used in the presence of phenolphthalein indicator
= N ï V (ml) = 0.1 ï 2 ï 2.5 = 0.5

1
a = 0.5 = milli equivalents of Na2CO3 ... (1)
2

Milli eq. of NaHCO3 + ó milli eq. of Na2CO3 = (b) ... (2)


For Na2CO3 solution : From equation (1)
Milli eq. of acid used by Na2CO3 = 2 ï 0.5 = 1
Suppose, Normality of Na2CO3 solution = N
Volume of Na2CO3 solution taken = 10 ml
Putting the milli eq. of H2SO4 and Na2CO3 equal
1 = 10 N
or (Normality of Na2CO3) N = 0.1
Strength (S) in g/litre = N ï E
= 0.1 ï 53 (E for Na2CO3 = 53)
= 5.3 g/litre
For NaHCO3 solution : From equation (1) and (2) milli eq. of acid used by
NaHCO3 = b ă a = 1.0 ă 0.5 = 0.5
Suppose, Normality of NaHCO3 solution = N
Volume of NaHCO3 solution taken = 10 ml
Milli equivalents of NaHCO3 taken = 10 N
Putting the milli eq. of H2SO4 and NaHCO3 equal,
0.5 = 10 N
or (Normality of NaHCO3 solution) N = 0.05
Strength (S) in g/litre =N ï E
(E for NaHCO3 = 84) = 0.05 ï 84 = 4.2 g/litre

STOICHIOMETRY
34 QUIZRR

B A CK T I T RA T I ON
Let us consider that we have an impure solid substance ÂZÊ weighing ÂwÊ g and we are required to
calculate the percentage purity of ÂZÊ in the sample. We are also provided with two solutions ÂXÊ and ÂYÊ,
where the concentration of ÂYÊ is known (N1) and that of ÂXÊ is unknown. For the back titration to work,
following conditions are to be satisfied.
(a) Compounds ÂXÊ, ÂYÊ and ÂZÊ should be such that ÂXÊ and ÊYÊ reacts with each other.
(b) ÂXÊ and pure ÂZÊ also reacts with each other but impurity present in ÂZÊ does not react with ÂZÊ.
Z + X (excess)  Product 1
Remaining (X) + Y  Product 2
Note : Product 1 should not react with Y
Milli equivalent of Y = N2V2
Where N2 and V2 (ml) is the normality and volume of Y
Initial milli equivalent of X = N1V1
Where N1 and V1 (ml) is the normality and volume of X
Remaining milli equivalents of X after reacting with Y = N1V1 ă N2V2
Remaining milli equivalents of X = milli equivalents of Z
a ï 100
N1V1 ă N2V2 = Equivalent weight

Where ÂaÊ is the weight of pure Z which is reacted.

Molecular weight  N1 V1  N2 V2 
a=
n  factor

 N1 V1  N1 V2  Molar Mass of Z
 Percentage purity of ÂZÊ =   100
n  factor W

Ex a m p l e 2 4
20 g of sample of Ba(OH)2 is dissolved in 10 ml of 0.5 N HCl solution. The excess of HCl was
titrated with 0.2 N NaOH. The volume of NaOH used was 10 cc. Calculate the percentage of
Ba(OH)2 in the sample.
Solution :
Milli eq. of HCl initially= 10 ï 0.5 = 5
Milli eq. of NaOH consumed = Milli eq. of HCl in excess
= 10 ï 0.2 = 2
 Milli eq. of HCl consumed = Milli eq. of Ba(OH)2
= 5ă2= 3
 eq. of Ba(OH)2 = 3/1000 = 3 ï 10ă3
Mass of Ba(OH)2 = 3 ï 10ă3 ï (171/2) = 0.2565 g
% of Ba(OH)2 = (0.2565/20) ï 100 = 1.28%

STOICHIOMETRY
QUIZRR 35

(i) IODIMETRY
This titration involves free iodine. Such direct estimation of iodine is called iodimetry. This
involves the titration of iodine solution with known sodium thiosulphate solution, whose normality
is N. Let the volume of sodium thiosulphate used by V litre.

I2  2Na 2S2O3  2NaI  Na 2 S4 O6


( n  2) ( n  1)

Equivalents of I2 = Equivalents of Na2S2O3 used = N ï V

NV
Moles of I2 =
2

N  V 
Mass of free I2 in the solution =   254  g
 2 

(ii) IODOMETRY
This is an indirect method of estimation of iodine. An oxidising agent is made to react with excess
of solid KI. The oxidising agent oxidises Iă to I2. This liberated iodine is then made to react with
Na2S2O3 solution of normality N. Let the volume of thiosulphate solution required be V litre.
2Na 2S2O3
Oxidising Agent (A)  KI  I2   2NaI  Na 2S 4 O6

reduced form of oxidizing agent


Equivalents of ÂAÊ = equivalents of I2 = Equivalents of Na2S2O3 used = N ï V
Equivalents of I2 liberated from KI = N ï V
Equivalents of ÂAÊ = N ï V
Let the năfactor of ÂAÊ in its reaction with KI be x, then

NV
Mass of ÂAÊ consumed =  M A (where M is the molar mass of A)
x A

Ex a m p l e 2 5
A 1.1 g sample of copper are is dissolved and Cu2+ (aq.) is treated with KI. I2 liberated
required 12.12 mL of 0.1 M Na2S2O2 solution for nitration. What is % of Cu in the ore ?
2Cu2+ + 4Iă  Cu2I2 + I2
I2 + 2S2O32ă  S4O62ă + 2Iă
2S2O32ă  I2  2Cu2+
Thus S2O32ă  I  Cu2+
hence equivalent weight of S2O32ă = mol. wt.
and that of Cu2+ = mol. wt.

STOICHIOMETRY
36 QUIZRR

Equialent of Na2S2O3 = equivalent of I2


= equivalent of Cu2+
NEV
w (pure (Cu)) =
1000
0.1  63.5  12.12
=
1000
= 0.077 g
This is present in 1.1 g copper ore.
 % of copper = 7.0%

Ex a m p l e 2 6
A 0.589 g sample of pyrolusite ore (impure MnO2) is treated with 1.65 g hydrate oxalic acid
(H2C2O4.2H2O) in an acidic medium. Following this reaction, the excess oxalic acid required
30.6 mL of 0.1 M KMnO4. What is % of MnO2 in pyrolusite ?
Solution : Oxalic acid will react with pure MnO2
unreacted oxalic acid  30.06 mL of 0.1 M KMnO4

1.65
oxalic acid taken = 1.65 g =  0.013 mol
126
MnO2 + C2O42ă + 4H+  2CO2 + Mn2+ + 2H2O ...(i)
2MnO4ă + 5C2O42ă + 16H+  10CO2 + 2Mn2+ + 8H2O ...(ii)
unreacted
30.06 mL of 0.1 M KMnO4 = 30.06 mL of 0.5 N KMnO4.

NEV
unreacted oxalic acid =
1000

0.5  63  30.06
=
1000
0.95
= 0.95 g =
126
= 0.0075
used oxalic acid in reaction (i) = 0.013 ă 0.0075
= 0.00558 mol
= 5.58 ï 10ă3 mol
hence mol of MnO2 (pure) = 5.58 ï 10ă3 mol
= 0.485 g

0.485
% of pure MnO2 =  100 = 82.42%
0.589

STOICHIOMETRY
QUIZRR 37

Ex a m p l e 2 7
A 0.608 g sample of fertiliser contained nitrogen as ammonium sulphate, (NH4)2SO4. It was
analysed for nitrogen by heating with sodium hydroxide.
(NH4)2SO4(s) + 3NaOH(aq)  Na2SO4(aq) + 2H2O(l) + 2NH3(g)
The ammonia was collected in 36.3 mL of 0.213 M HCl.
NH3 (g) + HCl (aq)  NH4Cl(aq)
This solution was titrated for excess HCl with 44.3 mL of 0.128 M NaOH. What is the percentage
of nitrogen in the fertiliser ?
Solution :
Equivalent of acid for NH3  equivalent of NH3 from fertiliser
first we determine unreacted HCl (V mL)
V mL of 0.213 M HCl  44.3 mL of 0.128 M NaOH

44.3  0.128
V  26.62 mL
0.213

used HCl = 46.30 ă 26.62


= 19.68 mL

NEV
amount of nitrogen in fertiliser =
1000

0.213  14  19.68
equivalent to 19.68 mL of 0.213 M HCl = g
1000
= 0.0587 g
This amount is present in 0.608 g sample of fertilisers

0.0587
% of nitrogen =  100
0.608

= 9.65%

Ex a m p l e 2 8
What mass of Na2SO3 must have been present in a sample that required 26.50 mL of 0.0510
M KMnO4 for its oxidation to Na2SO4 in acidic solution ?
Solution :
Redox reaction is

5SO32  2MnO4  6H   2Mn 2  5S42   3H2 O

+ 5eă
ă 2eă

STOICHIOMETRY
38 QUIZRR

M
equivalent weight of MnO4 
5

M
equivalent weight of SO23  
2

26.50 mL of 0.510 M MnO4  26.50 mL of 0.2550 N MnO4

 26.50 mL of 0.255 N SO23

NEV
 weight of Na2SO3 =
1000

0.2550  63  26.50
=
100

= 0.426 g

STOICHIOMETRY

You might also like